Manual de Trabajo del Curso ENARM CMN Siglo XXI_booksmedicos.org

561 Pages • 459,037 Words • PDF • 24.3 MB
Uploaded at 2021-09-24 14:00

This document was submitted by our user and they confirm that they have the consent to share it. Assuming that you are writer or own the copyright of this document, report to us by using this DMCA report button.


MANUAL DE TRABAJO DEL CURSO ENARM CMN SIGLO XXI

Pharmed Solutions Institute Dr. Angel Mauricio Soriano Perez

MANUAL DE TRABAJO DEL CURSO ENARM CMN SIGLO XXI INTRODUCCION

Este manual de trabajo del curso para aprobar el ENARM es el resultado de más de 13 años de estudio de los contenidos del ENARM desde el año 2000 a la fecha, la modalidad, contenido y características pedagógicas han cambiado de forma importante desde entonces hasta ahora, con la versión moderna del ENARM podemos mencionar que es la mejor. Los cursos de preparación para el ENARM que se imparte en diversas instituciones públicas y privadas han tenido que adaptarse a estos cambios, sin embargo ni todos los cursos ni los diferentes libros dedicados a la preparación de los médicos aspirantes ha tomado con toda seriedad y responsabilidad modificando y adaptándose a los cambios cruciales para así ofrecer una verdadera alternativa al médico general que desea prepararse para aprobar el ENARM. Nuestro curso, los medios que son empleados y la metodología han sido ampliamente analizados para ser un curso integral que ofrezca un medio apropiado para incrementar las probabilidades de ser un seleccionado nacional. Agradecemos a todos los médicos especialista que participaron en la realización del presente manual de trabajo así como a los participantes de los diversos cursos.

Dirección General. Dr. Ángel Mauricio Soriano Pérez

Ponente Titular Medicina Interna. Dra. Luz Elena Castro Vargas . Ponente Titular Pediatría. Dra. Gabriela Arenas Ornelas. Ponente Titular Gineco-Obstetricia. Dra. Rosario León Gayosso. Ponente Titular Urgencias. Dr. Juan Manuel Lara Hernández. Ponente Titular Cirugia. Dr. Ängel Santino Figueroa. Ponente Titular Inglés. Dr. Juan Carlos Díaz Torre.

“No hay hombre más digno de estimación que el médico que, habiendo estudiado la naturaleza desde su juventud, conoce las propiedades del cuerpo humano, las enfermedades que le ataca, los remedios que pueden beneficiarle y que ejerce su arte”. Voltaire

CURSO ENARM CMN SIGLO XXI TEL: 36246001

Pharmed Solutions Institute

PÁGINA 2

MANUAL DE TRABAJO DEL CURSO ENARM CMN SIGLO XXI

INSTRUCCIONES

Este manual de trabajo esta formada por las areas de medicina interna, urgencias, ginecología y obstetricia, pediatría y cirugía asi como una sección de compresión de ingles. El manual de trabajo presenta un resumen minimo por cada patología previo a los casos clínico, las patologías fueron seleccionadas mediante un análisis de salida de los sustentantes del ENARM cada año. Comprende 450 casos clínicas distribuidas por especialidad proporcionalmente y semejante al ENARM, 405 en español y 45 casos clínicos en ingles. Cada caso clínico tiene de 1, 2 ó 3 preguntas con 4 posibles respuestas, todas las preguntas se realizaron buscando la mejor respuesta donde comprenden las siguientes areas de exploración: diagnostico, tratamiento, patogenia, complicaciones, salud publica, ciencias básicas y rehabilitación. El orden de los temas se realizará como esta en el índice, posterior a la sesión clínica, se resolvera los casos clínicos del tema, en conjunto al profesor. Posteriormente resolverá en el auditorio o en su domicilio, las 6 opciones más de exploración denominado “explicaciones y preguntas complementarias”. 1.- Sesión clínica. 2.- Casos clínicos. 3.- Explicaciones y preguntas complementarias. 4.- Videosesiones para despejar dudas. 5.- Simuladores mensuales. 6.- Elaboración de casos clínicos por los participantes.

CURSO ENARM CMN SIGLO XXI TEL: 36246001

Pharmed Solutions Institute

PÁGINA 3

MANUAL DE TRABAJO DEL CURSO ENARM CMN SIGLO XXI MEDICINA INTERNA 1) 2) 3) 4) 5) 6) 7) 8) 9) 10) 11) 12) 13) 14) 15) 16) 17) 18) 19) 20) 21) 22) 23) 24) 25) 26) 27) 28) 29)

HIPERTENSION ARTERIAL PRIMARIA Y SECUNDARIA INSUFICIENCIA CARDIACA AGUDA Y CRONICA CARDIOPATIAS CONGENITICAS Y VALVULOPATIAS MIOCARDITIS, ENDOCARDITIS Y PERICARDITIS ASMA, BRONQUITIS, NEUMONIAS, NEUMOPATIAS. HIPERTENSION PULMONAR Y COR PULMONAR, EPOC CARCER PULMONAR Y BRONCOPULMONAR. TRASTORNOS HIPOTALAMICOS E HIPOFISIARIOS. TRASTORNOS TIROIDEOS Y PARATIROIDEOS. TRASTORNOS SUPRARRENALES Y GONADALES. SOBREPESO, OBESIDAD Y DISLIPIDEMIAS. DIABETES MELLITUS Y COMPLICACIONES. PANCREATITIS AGUDA Y CRONICA. ESTEATOSIS, HIGADO GRASO, CIRROCIS HEPATICA Y HEPATOCARCINOMA LEUCEMIAS, LINFOMAS, LINFOMA NO HODGKIN, LINFOMA HODGKIN FIEBRE REUMATICA, ARTRITIS REUMATOIDE, OSTEARTROSIS, OSTEOCONDRITIS Y GOTA. LUPUS ERITEMATOSO SISTEMICO, VASCULITIS, ARTERITIS DE TAKAYASU, SX DE SJOGREN TUBERCULOSIS, PALUDISMO Y DENGUE, ENFERMEDAD DE CHAGAS, LEPRA, TRIPANOSOMIASIS, LEISHMANIASIS, ESQUISTOSOMIASIS, FILARIASIS, ONCOCERCOSIS. RICKETTSIAS, LEGIONELLA, PSEUDOMONA AERUGINOSA, LISTERIA MONOCYTOGENES, CRIPTOCOCOCIS LEPTOSPIROSIS, ACTINOMICOSIS, NORCARDIA. CMV Y VEB. STAFILOCOCCUS AUREUS, EPIDERMIDIS, ESTREPTOCOCCUS PNEUMONIAE Y HAEMOPHILUS. BORTEDELLA PERTUSIS, MORAXELLA CATARRHALIS, BRUCELLA, FRANCISELLA TULARENSIS, RABIA VIH Y SIDA, ETS, NEISSERIA GONORROEAE, SIFILIS, HERPES, CHLAMYDIA. CANCER E INFECCIONES DEL SISTEMA NERVIOSO CENTRAL. CEFALEA, EPILEPSIAS Y ENCEFALOPATIAS DEMENCIA VASCULAR, ALZHEIMER Y PICK, HUNTINGTON Y PARKINSON ESCLEROSIS MULTIPLE, ESCLEROSIS LATERAL AMNIOTROFICA Y MIASTENIA GRAVIS. DEPRESION, ANSIEDAD, ESQUIZOFRENIA Y TRASTORNOS DE LA ALIMENTACION.

URGENCIAS 1) MUERTE SUBITA, RCP. 2) ESTADO CHOQUE 3) URGENCIAS, EMERGENCIA HIPERTENSIVA. HIPERTENSION MALIGNA. 4) INSUFICIENCIA CARDIACA AGUDA Y CRONICA AGUDIZADA. 5) TAPONAMIENTO CARDIACO, DISECCION DE LA AORTA, ANEURISMA AORTICO. 6) ANGINA ESTABLE, ANGINA INESTABLE, ANGINA DE PRNIZTMETAL. 7) SICA I, SICA II 8) TRASTORNOS DEL RITMO (ARRITMIAS CARDIACAS) 9) FALLA ORGANICA MULTIPLE, COAGULACION INTRAVASCULAR DISEMINADA. 10) ANAFILAXIA Y ALERGIAS. 11) EPOC AGUDIZADO, TROMBOSIS VENOSA PROFUNDA, TROMBOEMBOLIA PULMONAR. 12) SINDROME DE INSUFICIENCIA RESPIRATORIA AGUDA, EDEMA AGUDO PULMONAR. 13) NEUMOTORAX, NEUMOMEDIASTINO, HEMOTORAX, CONTUSION CARDIACA. 14) TRAUMA TORACICO CERRADO Y ABIERTO. 15) TRAUMA ABDOMINAL ABIERTO Y CERRADO. 16) INSUFICIENCIA RENAL AGUDA, GLOMERULOPATIAS AGUDAS. 17) SINDROME NEFRITICO Y NEFROTICO. 18) TRASTORNOS ACIDO-BASE AGUDOS. 19) TRASTORNOS ELECTROLITICOS AGUDOS. 20) HIPOGLUCEMIA, HIPERGLUCEMIA, ESTADO HIPEROSMOLAR, CETOACIDOSIS DIABETICA. 21) TRAUMATISMO CRANEOENCEFLICO, ISQUEMIA CEREBRAL TRANSITORIA, ACCIDENTE VASCULAR CEREBRAL. 22) CRISIS CONVULSIVAS, ESTATUS EPILEPTICO, COMA Y MUERTE CEREBRAL. 23) TRASTORNO BIPOLAR, DELIRIUM, PSICOSIS, SUICIDIO. GINECOLOGIA Y OBSTETRICIA 1) PLANIFICACION FAMILIAR, ANTICONCEPCION, INFERTILIDAD, ESTERILIDAD. 2) AMENORREA PRIMARIA Y SECUNDARIA, ENDOMETRIOSIS. 3) CERVICOVAGINITIS, ENFERMEDAD PELVICA INFLAMATORIA, ENFERMEDADES DE TRANSMISION SEXUAL. 4) INFECCION POR VIRUS DEL PAPILOMA HUMANO. DISPLASIAS, CANCER CERVICOUTERINO Y OVARICO. 5) SANGRADO UTERINO ANORMAL Y DISFUNCIONAL. SINDROME DE OVARIO POLIQUISTICO. 6) MASTOPATIA FIBROQUISTICA, CANCER DE MAMA, MENOPAUSIA Y CLIMATERIO

CURSO ENARM CMN SIGLO XXI TEL: 36246001

Pharmed Solutions Institute

PÁGINA 4

MANUAL DE TRABAJO DEL CURSO ENARM CMN SIGLO XXI OBSTETRICIA 1) CONTROL PRENATAL Y DEL EMBARAZO. 2) EMBARAZO ECTOPICO. AMENAZA DE ABORTO, ABORTO, INCOMPETENCIA CERVICAL. 3) HIPEREMESIS GRAVIDICA, ENFERMEDAD TROFOBLASTICA. 4) HIPERTENSION ARTERIAL Y ENFERMEDAD HIPERTENSIVA DEL EMBARAZO. 5) PREECLAMPSIA Y ECLAMPSIA. 6) DIABETES MELLITUS, INTOLERANCIA A LA GLUCOSA, DIABETES GESTACIONAL. 7) POLIHIDRAMNIOS, OLIGOHIDRAMNIOS, PLACENTA PREVIA, DESPRENDIMIENTO DE PLACENTA. 8) ANEMIAS, IVU, PARTO PRETERMINO, PARTO NORMAL Y CESAREA. 9) COMPLICACIONES DEL PUERPERIO, HEMORRAGIA OBSTETRICA, PLACENTA ACCRETA. 10) SINDROME DE TENSION PREMENSTRUAL, PSICOSIS POSPARTO Y DEPRESION POSTPARTO. PEDIATRIA 1) SX DE ADAPTACION PULMONAR, SX DE DIFICULTAD RESPIRATORIA, TAQUIPNEA TRANSITORIA DEL RN 2) ASPIRACION DE MECONIO, ENFERMEDAD POR MEMBRANA HIALINA, NEUMONIA NEONATAL. 3) SEPSIS NEONATAL, ENTEROCOLITIS NECOTRIZANTE, MENINGITIS NEONATAL. 4) HIPOGLUCEMIA, ICTERICIA, KERNICTERUS. 5) ATRESIA ESOFAGICA, FISTULA TRAQUEOESOFAGICA, HERNIA HIATAL, ESTENOSIS PILORICA. 6) CARDIOPATIAS CONGENITAS Y ADQUIRIDAS 7) KWASHIORKOR, MARASMO, AVITAMINOSIS, 8) OBESIDAD, VACUNACION, CONTROL DEL NIÑO SANO. 9) DERMATITIS, HIPERQUERATOSIS HIPEDERMOLITICA. 10) IMPETIGO ERISIPELA, PIODERMA, TB CUTANEA, MAL DEL PINTO. 11) HERPES SIMPLE, ZOSTER, VERRUGAS VIRALES, MOLUSCO CONTAGIOSO 12) FOLICULITIS, PEDICULOSIS, ESCABIASIS. 13) TIÑAS, PITIRIASIS VERSICOLOR, CANDIDIASIS, MICETOMA. 14) URTICARIA, PRURIGOS, ERITEMA PALMAR, ERITEMA NODOSO. 15) DERMATITIS DE CONTACTO, DERMATITIS DEL PAÑAL. 16) PSORIASIS, DERMATITIS SEBORREICA, ACNE 17) PURPURA DE SCHÖNLEIN-HENOCH, ANEMIAS, 18) LEUCEMIAS, RETINOBLASTOMA, TUMORES DEL SNC. 19) PAROTIDITIS, HEPATITIS, INFLUENZA, MENINGITIS 20) OTITIS, SINUSITIS, FARINGITIS, AMIGDALITIS 21) EPIGLOTITIS, LARINGOTRAQUEITIS, BRONQUITIS. 22) BRONQUIOLITIS, BRONCONEUMONIA, NEUMONIAS 23) ALERGIAS, ASMA, FIBROSIS QUISTICA. 24) URETRITIS, CISTITIS, PIELONEFRITIS. 25) GLOMERULONEFRITIS, SINDROME NEFRITICO, SINDROME NEFROTICO. 26) GASTROENTERITIS, PARASITOSIS. 27) DESHIDRATACION, LIQUIDOS Y ELECTROLITOS 28) QUEMADURAS, ACCIDENTES 29) PICADURAS, MORDEDURAS 30) INTOXICACIONES, ENVENENAMIENTOS 31) TRAUMATISMO CRANEOENCEFALICO, CEFALEA Y EPILEPSIAS 32) ENURESIS, ENCOPRESIS, TDAH, TRASTORNOS DEL DESARROLLO. CIRUGIA 1) ESOFAGITIS, ACALASIA, REFLUJO ESOFAGICO Y ERGE. 2) DISPLASIAS DE ESOFAGO, ESOFAGO DE BARRETT, CANCER DE ESOFAGO. 3) VARICES ESOFAGICAS, RUPTURA ESOFAGICA Y HERNIA HIATAL. 4) GASTRITIS, ULCERA GASTRICA Y ULCERA DUODENAL, CANCER GASTRICO. 5) COLANGITIS, COLECISTITIS, COLEDOCOLITIASIS 6) CANCER DE PANCREAS, COLANGIOCARCINOMA, 7) ESTEATOSIS Y CIRROCIS HEPATICA, CARCINOMA HEPATOCELULAR. 8) PANCREATITIS AGUDA Y CRONICA. 9) ISQUEMIA MESENTERICA, INFARTO MESENTERICO, DIVERTICULITIS. 10) APENDICITIS, POLIPOS, COLITIS, CROHN, CUCI. 11) HEMORROIDES, FISTULA ANAL, CANCER DE COLON 12) HERNIAS DIAFRAGMATICAS, HIATAL, DE PARED E INGUINAL, 13) ORQUIEPIDIDIMITIS, TORSION TESTICULAR, CANCER TESTICULAR. 14) HIPERTROFIA PROSTATICA BENIGNA, CANCER DE PROSTATA 15) TCE, TRAUMA FACIAL Y TRAUMA RAQUIMEDULAR. 16) TRAUMA TORACICO, TRAUMA ABDOMINAL Y PELVICO 17) PIE DIABETICO Y COMPLICACIONES QUIRURGICAS DE LA DM.

CURSO ENARM CMN SIGLO XXI TEL: 36246001

Pharmed Solutions Institute

PÁGINA 5

MANUAL DE TRABAJO DEL CURSO ENARM CMN SIGLO XXI 18) RETINOPATIA DIABETICA E HIPERTENSIVA. 19) URGENCIAS OFTALMOLOGICAS, PERFORACION, GLAUCOMA. (CIRUGIA) 20) TRAUMA OTICO RUPTURA, PERFORACION, TRAUMA FACIAL.

CURSO ENARM CMN SIGLO XXI TEL: 36246001

Pharmed Solutions Institute

PÁGINA 6

MANUAL DE TRABAJO DEL CURSO ENARM CMN SIGLO XXI MEDICINA INTERNA 1) 2) 3) 4) 5) 6) 7) 8) 9) 10) 11) 12) 13) 14) 15) 16) 17) 18) 19) 20) 21) 22) 23) 24) 25) 26) 27) 28) 29)

HIPERTENSION ARTERIAL PRIMARIA Y SECUNDARIA INSUFICIENCIA CARDIACA AGUDA Y CRONICA CARDIOPATIAS CONGENITICAS Y VALVULOPATIAS MIOCARDITIS, ENDOCARDITIS Y PERICARDITIS ASMA, BRONQUITIS, NEUMONIAS, NEUMOPATIAS. HIPERTENSION PULMONAR Y COR PULMONAR, EPOC. CARCER PULMONAR Y BRONCOPULMONAR. TRASTORNOS HIPOTALAMICOS E HIPOFISIARIOS. TRASTORNOS TIROIDEOS Y PARATIROIDEOS. TRASTORNOS SUPRARRENALES Y GONADALES. SOBREPESO, OBESIDAD Y DISLIPIDEMIAS. DIABETES MELLITUS Y COMPLICACIONES. PANCREATITIS AGUDA Y CRONICA. ESTEATOSIS, HIGADO GRASO, CIRROCIS HEPATICA Y HEPATOCARCINOMA LEUCEMIAS, LINFOMAS, LINFOMA NO HODGKIN, LINFOMA HODGKIN FIEBRE REUMATICA, ARTRITIS REUMATOIDE, OSTEARTROSIS, OSTEOCONDRITIS Y GOTA. LUPUS ERITEMATOSO SISTEMICO, VASCULITIS, ARTERITIS DE TAKAYASU, SX DE SJOGREN TUBERCULOSIS, PALUDISMO Y DENGUE, ENFERMEDAD DE CHAGAS, LEPRA, TRIPANOSOMIASIS, LEISHMANIASIS, ESQUISTOSOMIASIS, FILARIASIS, ONCOCERCOSIS. RICKETTSIAS, LEGIONELLA, PSEUDOMONA AERUGINOSA, LISTERIA MONOCYTOGENES, CRIPTOCOCOCIS, LEPTOSPIROSIS, ACTINOMICOCIS, NORCARDIA. CMV Y VEB. STAFILOCOCCUS AUREUS, EPIDERMIDIS Y ESTREPTOCOCCUS PNEUMONIAE. HAEMOPHILUS. BORTEDELLA PERTUSIS, MORAXELLA CATARRHALIS, BRUCELLA, FRANCISELLA TULARENSIS, RABIA, VIH Y SIDA, ETS, NEISERIA GONORRAE, SIFILIS, HERPES, CHLAMYDIA. CANCER E INFECCIONES DEL SISTEMA NERVIOSO CENTRAL. CEFALEA, EPILEPSIAS Y ENCEFALOPATIAS DEMENCIA VASCULAR, ALZHEIMER Y PICK, HUNTINGTON Y PARKINSON ESCLEROSIS MULTIPLE, ESCLEROSIS LATERAL AMNIOTROFICA Y MIASTENIA GRAVIS. DEPRESION, ANSIEDAD, ESQUIZOFRENIA Y TRASTORNOS DE LA ALIMENTACION.

CURSO ENARM CMN SIGLO XXI TEL: 36246001

Pharmed Solutions Institute

PÁGINA 7

MANUAL DE TRABAJO DEL CURSO ENARM CMN SIGLO XXI HIPERTENSIÓN ARTERIAL PRIMARIA Y SECUNDARIA (TEMA) DEFINICION: Es el incremento sostenido de la tensión arterial diastólica > 90 mmHg, La OMS la define como un trastorno cuya patogenia se desconoce en muchos casos, que en última instancia causa aumento de la presión diastólica y sistólica, así como alteraciones del lecho vascular y alteraciones funcionales de los tejidos afectados. EPIDEMIOLOGIA: Prevalencia mundial: 1 billón de individuos, 7.1 millones de muertes al año, OMS: 62 % de la enfermedad cerebrovascular y 49 % de la enfermedad isquémica cardiaca. Prevalencia incrementa conforme avanza la edad, 50% de personas entre 60 y 69 años y 3/4 de los mayores de 70 años están afectados. La prevalencia de hipertensión sistólica se incrementa con la edad. A partir de los 50 años forma más común. PAD factor de riesgo CV más potente hasta los 50 años. A partir de entonces, la PAS >140mmHg es de importancia mayor como factor de riesgo de (ECV). En los ensayos clínicos, la terapia antihipertensiva se ha asociado con reducciones en incidencias de ictus de un 35-40 %, de infarto de miocardio de un 20-25 %, y de insuficiencia cardiaca en más de un 50 %. Por cada incremento de 20 mmHg en PAS ó 10 en PAD, hay el doble de mortalidad por enfermedad isquémica e ictus. Estudio Framingham: PA de 130 a 139/85 a 89 mmHg: 2 veces de incremento RR de enfermedad cardiovascular comparados con los niveles por debajo de 120/80 mmHg. El riesgo de ECV comienza a partir de 115/75 mmHg se dobla con cada incremento de 20/10 mmHg. ETIOLOGIA: HTA primaria es más frecuente, la secundaria causada por: estenosis de arteria renal, insuficiencia renal, enfermedad de las glándulas suprarrenales, feocromocitomas, hiperplasia cortical, síndrome de cushing, hiperaldosteronismo, otras causas son la preeclampsia, coartación de la aorta, enfermeades cerabrales. Factores de Riesgo Mayores, Hipertensión, Edad (>55 en varones, 65 en mujeres), Diabetes mellitus, LDL elevado, Colest total alto ó bajo HDL, TFG < 60 ml/min, Historia familiar de ECV prematura (H 30 Kg/m2) Inactividad Física, Tabaco. FISIOPATOGENIA: Patologia multifactorial producto de la interacción de factores prevenibles y no prevenibles, además de hiperactividad del sistema nervioso simpatico, alteración de catecolaminas y quimio y barorreceptores. Alteraciones del sistema renina-angiotensina-aldosterona, altaracion de resistencias vascular, del volumen intravascular, constricion arteriolar, e hipopotasemia y defecto natriuretico por capacidad disminuida de reducir la carga de sodio e hídrica. Aumento de sodio y calcio intracelular, alteración del transporte Na/K. CUADRO CLINICO: Buscar intensionadamente, medida apropiada de PA, Verificación en el brazo contralateral, Examen del fondo de ojo, IMC, Auscultación carotídea, abdominal y soplos femorales, Palpación de la glándula tiroidea, Examen cardiaco y pulmonar completo, Exploración abdominal: riñones, detección de masas y pulsaciones aórticas anormales, palpación de pulsos y edemas en extremidades inferiores, Valoración neurológica, buscar síntomas de órgano blanco, cefalea, acufenos, fosfenos, apresion torácica, palpitaciones, fatiga, adinamia, cansancio, edema, alteraciones visuales, alteraciones sensitivas. DIAGNOSTICO: Se recomienda realizar toma de TA a pacientes mayores de 35 años con o sin síntomas sugestivos de HTA. La clasificación de la presión arterial según la OMS: Normal menor de 140/90 mmHg. Limítrofe: 140-160 y 90-95 respectivamente, HTA (leve) 140-180 y 90-105; HTA moderada y severa > 180 y > 105 mmHg, o la JNC7: optima 45%, PREGUNTA Cuáles son las manifestaciones clínicas más especificas para establecer el diagnostico de la enfermedad? RESPUESTA a.- Ortopnea, disnea y tos noctura. b.- Edema, fatiga y congestion torácica. c.- Electrocardiograma y radiografia de torax. d.- Ingurgitacion yugular, hepatomegalia y fracción de eyección. PREGUNTA Que de Framingham para el diágnosticos de IC tiene este paciente? RESPUESTA a.- 4 mayores y 2 menores. b.- 4 mayores y 3 menores. c.- 3 mayores y 2 menores. d.- 3 mayores y 3 menores. PREGUNTA Cuál es el factor de riesgo que puede conducir a un estado descompensado en este caso? RESPUESTA a.- Incumplimiento del tratamiento. b.- SAOS. c.- Arritmia, anemia ó hemorragia. d.- Antiarritmico de clasa Ia y Ic. PREGUNTA Cuáles son las manifestaciones clínicas mas útiles para realizar un diagnostico diferencial de la IC sistólica vs IC diastólica? RESPUESTA a.- No son distinguibles. b.- Disnea y edema. c.- Taquicardia y taquipnea. d.- Congestión hepática. PREGUNTA Cuáles son los auxiliares diagnosticos mas útiles que no han sido realizados en el paciente para identificar las complicaciones de la enfermedad?

CURSO ENARM CMN SIGLO XXI TEL: 36246001

Pharmed Solutions Institute

PÁGINA 14

MANUAL DE TRABAJO DEL CURSO ENARM CMN SIGLO XXI RESPUESTA a.- Puebas de función hepática. b.- Electrolitos sericos. c.- Pruebas de estrés. d.- Ecocardiograma. PREGUNTA Cuál es la medida farmacológica que intervendrá predominantemente sobre la respuesta neurohumoral en la retención de sodio y liquidos? RESPUESTA a.- Sistema renina-angiotensina-aldosterona. b.- Sistema arginina-vasopresina. c.- Liberación de péptido natriuretico auricular. d.- Inhibe el sistema de transporte Na + Cl -. PREGUNTA Cual de los siguientes factores no es determinante del consumo de oxigeno miocardico? RESPUESTA a.- Estrés parietal. b.- Contractilidad. c.- Frecuencia cardiaca. d.- Fracción de eyección. PREGUNTA Antes de definir el tratamiento farmacológico, cual de las causas reversibles menos probable se encuentra en el caso? RESPUESTA a.- Depresión miocárdica inducida por alcohol. b.- Antiinflamatorio no esteroideo. c.- Estado de poscarga elevado. d.- Uso de bloqueador de canales de calcio. PREGUNTA Cual de las siguientes medidas no es la mas recomendable para el actual caso clínico? RESPUESTA a.- Dieta con 5 a 6 g. de sal al día. b.- Dieta sin sodio absoluta. d.- Programa de gradual de ejercicio. c.- Dieta normokalemica. PREGUNTA Cosiderando el estado actual de velocidad de filtrado glomerular del paciente, cual de los siguientes diureticos proporcionan un equilibrio adecuado de natriuresis y caliuresis? RESPUESTA a.- Espironolactona. b.- Hidroclorotiazida. c.- Furosemida. d.- Clortalidona. PREGUNTA Considerando el esquema farmacológico actual del paciente, que medida farmacológica es la mas adecuada para promover un menor índice de morbilidad y mortalidad? RESPUESTA a.- Incrementar la dosis de enalapril. b.- Añadir losartan. c.- Iniciar esquema con digoxina. d.- Iniciar sildenafil. PREGUNTA Cuál de los cambios neurohumorales que se presentan en la insuficiencia cardiaca es menos probable en este caso?

CURSO ENARM CMN SIGLO XXI TEL: 36246001

Pharmed Solutions Institute

PÁGINA 15

MANUAL DE TRABAJO DEL CURSO ENARM CMN SIGLO XXI RESPUESTA a.- Aumento de la contractibilidad miocárdica. b.- Aumento de la frecuencia cardiaca. c.- Aumento del tono venoso. d.- Disminución del volumen sanguíneo central efectivo. PREGUNTA Cuál de los cambios funcionales que se presentan en el paciente con insuficiencia cardiaca es menos probable en este caso? RESPUESTA a.- Volumen sistólico reducido. b.- Poscarga excesiva. c.- Depresión de la contractibilidad. d.- Disminución del volumen telediastolico. CASO CLINICO Se trata de paciente masculino de 63 años de edad la cual acude a consulta de revisión, en primer nivel de atencion, cuenta con antecedentes de dislipidemia, hipertensión arterial y diabetes mellitus, actualmente tratada con atorvastatina, hidroclorotiazida, losartan y glibenclamida, a dosis minima toxica, hace 2 años presento IAM, el cual se trato oportunamente, acude debido a la presencia de edema de miembros inferiores, fatiga, disnea de medianos esfuerzos, ortopnea y tos, a la exploración física se aprecia reflujo hepatoyugular, soplo carotideo, estertores finos bibasales, FC 91 lpm, FR 26, TA 150/80 mmHg. PREGUNTA Se solicita estudios de laboratorio y gabinete, los cuales están pendientes de resultado, considerando los criterios de framingham para el diagnostico de la insuficiencia cardiaca, cuantos mayores y menores respectivamente presenta? RESPUESTA a.- 2 mayores y 3 menores. b.- 2 mayores y 2 menores. c.- 3 mayores y 3 menores. d.- 3 mayores y 2 menores. PREGUNTA Cual es la clase funcional y el ICT que en este momento presenta el paciente? RESPUESTA a.- Clase funcional I e ICT 0.62. b.- Clase funcional III e ICT 0.59. c.- Clase funcional IV e ICT 0.68. d.- Clase funcional II e ICT 0.70 CASO CLINICO Masculino de 43 años de edad el cual reingresa a urgencias debido a dificultad respiratoria, disnea de minimos esfuerzos, tos incluyendo en la noche, además de dolor toraccico cuando se presenta la disnea, este dolor disminuye muy lentamente con el reposo, el paciente cuenta con el antecedente de IAM egresando 15 dias antes de este evento. Se observa reflujo hepatoyugular con hepatomagalia, se ausculta tercer ruido, Se observo es siguiente ECG. Sus contantes vitales son: IMC 31, FC 119 lpm, FR 31, TA 140/70 mmHg, PREGUNTA Cuales son los criterios de framingham presenta el paciente en este momento? RESPUESTA a.- 3 menores y 4 mayores. b.- 4 menores y 4 mayores. c.- 3 menores y 3 mayores. d.- 4 menores y 2 mayores. PREGUNTA Cual es condición que se presenta tomando la clasificación de killip y Kimball. RESPUESTA a.- Clase funcional I. b.- Clase funcional II. c.- Clase funcional III. d.- Clase funcional IV.

CURSO ENARM CMN SIGLO XXI TEL: 36246001

Pharmed Solutions Institute

PÁGINA 16

MANUAL DE TRABAJO DEL CURSO ENARM CMN SIGLO XXI PREGUNTA Considerando el estado actual del paciente cual es el pronostico? a.- Tasa de mortalidad del 6%. b.- Tasa de mortalidad del 17%. c.- Tasa de mortalidad del 38%. d.- Tasa de mortalidad del 81%. CASO CLINICO Ingresa paciente de 39 años de edad el cual ingresa a urgencias 4 horas antes, por presencia súbita de dolor retrosternal acompañado de presión toracicca “sensación de mucho peso sobre su pecho” que se presento en la madrugada, nausea, dificultad para respirar, refiere que la noche anterior comio abundante y consumio alcohol hasta la embriaguez, al ingreso se observa confuso, diaforético con palidez y cianosis distal, ingurgitación yugular, se asculta estertores cripitantes bibasales, tercer ruido, TA 90/60 mmHg, FC 132 lpm, FR 36 rpm, Temp. 36.3 grados. Gasto urinario de < 20 ml/h, se observaron los siguientes estudios. PREGUNTA Cual es la clase funcional de Kllip que presenta el caso? RESPUESTA a.- Clase funcional Killip I. b.- Clase funcional Killip II. c.- Clase funcional Killip III. d.- Clase funcional Killip IV. PREGUNTA Cual es el pronostico para la vida en este caso considerando la clase funcional? a.- Tasa de mortalidad del 6%. b.- Tasa de mortalidad del 17%. c.- Tasa de mortalidad del 38%. d.- Tasa de mortalidad del 81% PREGUNTA Se calculo los siguientes resultados (LC) de 2 L/m2/min. Considerando los elementos clínicos y el (LC), en la curva de starling cual es el diagnostico? RESPUESTA a.- Insuficiencia cardiaca. b.- Edema agudo pulmonar. c.- Choque cardiogenico. d.- Choque distributivo. PREGUNTA Se realizó ecocardiograma transesofagico de urgencias, donde se reporta FE < 30%, cual es % de mortalidad a 6 meses del paciente? RESPUESTA a.- del 15 al 20 % b.- del 20 al 25 % c.- del 25 al 30 % d.- 30 al 35 % CASO CLINICO Se trata de masculino de 62 años de edad diabético e hipertensión controlado con glibenclamida y enalapril a dosis minimas toxicas respectivamente, fue fumador hasta los 45 años. Hace 5 años presento infarto al miocardio tratado con trombolitico con buena respuesta, acude a consulta debido cansacio, fatiga y dolor toracicco opresivo que sede con el reposo, ocasionalmente el dolor toraccico es ardoroso o urente, además de palpitaciones ocacionales, disnea de medianos esfuerzos, ortopnea así como fatiga, a la EF presenta ingurgitación yugular, frote carotideo, area cardiopulmonar con estertores crepitantes finos, palidez de tegumento, cianosis distal, miembros con edema de miembros inferiores, TA 140/70 mmHg, FC 93 lpm, FR 26 rpm, IMC 29.5. PREGUNTA Se realizarón los siguientes estudios de rutina hemoglobina glucosilada 8, creatinina 0.5, acido urico 7.0 mg/dl, colesterol HDL 27 mg/dl, triglicéridos 210 mg/dl, considerando el estado actual de caso clínico, cual es la conducta a seguir? RESPUESTA a.- Solicitar BNP ó NT-proBNF. b.- Solicitar ecocardiograma transesofagico. c.- Solicitar ECG en reposo y holter.

CURSO ENARM CMN SIGLO XXI TEL: 36246001

Pharmed Solutions Institute

PÁGINA 17

MANUAL DE TRABAJO DEL CURSO ENARM CMN SIGLO XXI d.- Solicitar prueba de estrés. PREGUNTA Cual es la conducta terapéutica a seguir para disminuir el riesgo cardiometabolico? RESPUESTA a.- Iniciar hidroclorotiazida b.- Iniciar verapamilo. c.- Iniciar betabloqueador. d.- Iniciar atorvastatina PREGUNTA Se realizo ecocardiograma que reporta una FE > 45 %, el electrocardiograma reporta extrasístoles ventriculares, cual es la conducta farmacológica mas adecuada para disminuir el remodelado ventricular y mejorar la actividad eléctrica cardiaca? RESPUESTA a.- Iniciar betabloqueador. b.- Iniciar verapamilo. c.- Iniciar losartan. d.- Iniciar diltiazem. PREGUNTA El paciente se mantiene en buenas condiciones sin embargo 12 meses después acude a cita de control, se realizo electrocardiograma de control, donde se apresia bloqueo de primer grado, el paciente agrega que la monitorización ambulatoria de la presión fue adecuada, sin embargo la opresión toracicca se ha presentado de forma expontanea y con mayor duración, asi como la disnea se ha presentado mas frecuente, cual es el fármaco mas adecuado? RESPUESTA a.- Isosorbide 10 mg. b.- Verapamilo 80 mg. c.- Diltiazem 60 mg. d.- Nitroglicerina 18 mg. CASO CLINICO Se trata de paciente femenino de 67 años de edad, que acude a consulta por cansancio, fatiga, dificultad para respirar, dolor toraccico que disminuye con reposo, disnea de moderados esfuerzos, edema de miembros inferiores hasta 2/3 de la pierna, a la exploración física se observo palidez de piel y mucosas, llenado capilar lento, estertores crepitantes leves, presencia de galope y trill. La paciente es hipertensa desde hace 20 años con moderado apego a tratamiento con captopril 50 mg/dia, hidroclorotiazida 25 mg/12 hrs. Ha presentado 2 crisis hipertensivas, asi como un evento de insuficiencia mesentérica, sus laboratorios EGO proteínuria, glucosuria, uratos, densidad urinaria disminuida, colesterol 289 mg/dl, triglicéridos 720 mg/dl, creatinina 6.2 mg/dl, BUM 29, kalemia de 5.3, los signos vitales fueron TA 160/105 mmHg, FC 96, FR 28, IMC 32. PREGUNTA Se realizo un ecocardiograma donde se observa una FE de < 40 %, durante su ingreso la paciente presento volumen urinario de 20 ml/hr. Considerando las características actuales cual es la clase funcional que presenta el caso? RESPUESTA a.- Clase funcional I. b.- Clase funcional II. c.- Clase funcional III. d.- Clase funcional IV. PREGUNTA Considerando las características actuales cual es el K/DOQI que presenta el paciente, cual es el método mas adecuado para determinar el estadio de la IRC? RESPUESTA a.- Depuracion de creatinina de 24 hrs. b.- Biopsia renal. c.- Ultrasonografia renal. d.- Urografia escretora. PREGUNTA Cual es la conducta farmacológica mas adecuada considerando el K/DOQI y la clase funcional? RESPUESTA

CURSO ENARM CMN SIGLO XXI TEL: 36246001

Pharmed Solutions Institute

PÁGINA 18

MANUAL DE TRABAJO DEL CURSO ENARM CMN SIGLO XXI a.- Verapamilo. b.- Diltiazem. c.- Telmisartan. d.- Isosorbide. PREGUNTA Cual es la conducta inmediata a seguir para mejorar la precarga disminuyendo los niveles de potasio? RESPUESTA a.- Furosemide. b.- Dialisis peritoneal. c.- Hemodialisis. d.- Soluciones metabolizadas. PREGUNTA Cual de las siguientes patologias es la principal causa de IRC? RESPUESTA a.- Diabetes mellitus. b.- Hipertensión arterial. c.- Glomerulonefritis. d.- Enfermedad renal poliquística. PREGUNTA Cuales factores de riesgo aceleran el deterioro del funcionamiento renal en el caso actual? RESPUESTA a.- Dislipidemia. b.- Edad avanzada. c.- Diabetes mellitus. d.- Anemia. CASO CLINICO Se trata de masculino de 59 años de edad, ingresa a urgencias debido a dificultad para respirar, cansancio, fatiga, tos productiva desde hace 15 dias intensificandoce la noche previa al ingreso actual, cuenta con antecedentes de EPOC diagnosticado hace 10 años, posterior a tabaquismo (una cajetilla diaria), con tratamiento de bromuro de iprapropio, salbutamol y betametasona, además fue diagnosticado como hipertenso hace 2 años, inicio con cambios del estilo de vida y dieta con restricción de sodio, hace un año se considero incluir captopril que fue cambiado 3 meses después por incremento de tos, desde entoces toma losartan, hidralazina. A la EF se observa facies pletórica con hiperemia conjuntival, mucosas orales cianóticas moderadamente, se ausculta un soplo carotideo derecho, ingurgitación yugular grado II, estertores subcrepitantes bibasales, con resonancia tipanica a la percusión, los ruidos cardiacos levemente disminuidos, rítmico, abdomen con hepatomegalia 5 cm debajo del borde costal. PREGUNTA Considerando el estado actual del paciente cual es la conducta mas apropiada a seguir, para establecer la función cardiopulmonar? RESPUESTA a.- Espirometria. b.- Ecocardiograma. c.- Valor de natriuretico. d.- Radiografia de torax. PREGUNTA Luego de realizar las pruebas anteriores se observo una FE menor a 40 %, la radiografia de torax mostro un indice cardiotoraccico patológico, exudados algodonosos bilaterales, trama brocovascular incrementada, gases con PCO 49, PO2 85, potasio 6.3, Factor natriuretico disminuido, a la expxloracion física se auscultan estertores crepitantes, sibilancias moderadas, a la percusión hiperresonancia, se alcanza apresiar galope, cual es la conducta mas apropiada a seguir para disminuir la precarga, considerando todos los factores? RESPUESTA a.- Retirar diurético ahorrador de potasio por diurético de asa. b.- Iniciar con calcioantagonista dihidropiridinico de acción rápida. c.- Incrementar la capacitancia vascular periférica con nitritos. d.- Restriccion hídrica, dieta libre de sodio, diurético de asa.

CURSO ENARM CMN SIGLO XXI TEL: 36246001

Pharmed Solutions Institute

PÁGINA 19

MANUAL DE TRABAJO DEL CURSO ENARM CMN SIGLO XXI CASO CLINICO Paciente varón, de 41 años de edad, diagnosticado previamente de SAF y con antecedentes personales de infarto lacunar protuberancial izquierdo hacía 4 años, hipertensión arterial e hipercolesterolemia. Acudió a urgencias por dolor retrosternal al esfuerzo rápidamente progresivo, con algún episodio en reposo, durante las dos semanas previas. El ECG demostraba amputación de los vectores iniciales en las precordiales derechas y supradesnivel del segmento ST, de hasta 2 mm, en las derivaciones V1 a V4. En la auscultación cardíaca se detectaba un soplo holosistólico rudo de intensidad II-III/VI en el foco aórtico y diastólico II/IV en el borde esternal izquierdo bajo. CPK máxima de 71 U/l, troponina T de 0,17 ng/ml, 58.000 plaquetas, TTPA 44 s y en el estudio inmunológico, un anticoagulante lúpico positivo débil y unos anticuerpos anticardiolipina (AAC) IgG moderadamente elevados (68 unidades fosfolipídicas GPL). En la radiografía de tórax se observaba cardiomegalia global con campos pulmonares limpios. El ecocardiograma transtorácico objetivó una válvula aórtica engrosada, con apertura estenótica y gradiente sistólico máximo instantáneo estimado de 59 mmHg y regurgitación. PREGUNTA Cual es la conducta a seguir mas adecuada para identificar el compromiso fisiológico coronario? RESPUESTA a.- Ecocardiograma de estrés con dobutamina. b.- Prueba de esfuerzo convencional c.- Coronariografia percutenea. d.- IRM-f con FDG cardiaca. PREGUNTA El paciente presento durante las siguientes semanas se present ortopnea, disnea de medianos esfuerzo, disminución de la uresis, cabe destacar de hipokalemia, la gasometría reporto pH 7.21, PCO 41, Na 121, Cl 115, HCO3 42, se diagnostica acidosis respiratoria secundaria, considerando la patologia de base, cual es la causa menos probable, RESPUESTA a.- Depresion respiratoria central secundaria. b.- Sindrome de Pickwickian. c.- Edema pulmonar crónico. d.- Compensación respiratoria por alcalosis metabolica. PREGUNTA 24 horas después de estancia intrahospitalaria, el paciente presenta esputo salmonelado, disminución del gasto urinario < 25 ml/hora, confusión con agitación psicomotriz, diaforético, cianosis central, ingurgitación yugular III, estertores crepitantes, TA 70/50 mmHg, FC 139 lpm, FR 50 rpm. Cuál es la conducta a seguir para mantener la fracción de eyección adecuada sin incrementar la precarga? RESPUESTA a.- Dobutamina 20 µg/kg/min. b.- Dopamina > 20 µg/kg/min. c.- Epinefrina > 10 µg/min. d.- Digoxina 10 ug/Kg/min. PREGUNTA Considerando la clasificación funcional clásica de la insuficiencia cardíaca (NYHA), cuál es la clase del paciente previa a la desición anterior? RESPUESTA a.- Grado I b.- Grado IIA c.- Grado IIIA d.- Grado IVA PREGUNTA Considerando los lineamientos del American College of Cardiology y el American Heart Association, en que estadio se encuentra? RESPUESTA a.- Estadio A b.- Estadio B c.- Estadio C d.- Estadio D CASO CLINICO Se ingresa a urgencias a paciente femenino de 48 años de edad, originaria de Veracruz, empleada de agrícola, casada desde los 17 años, gesta 5, para 3, aborto 1, cesarea 1, histerectomía a los 38 por miomatosis uterina, no fuma ni toma bebidas alcoholicas, aparentemente sana, no toma medicamentos, anteriormente presento caries dentales pero fueron tratadas hace 7 meses sin embargo

CURSO ENARM CMN SIGLO XXI TEL: 36246001

Pharmed Solutions Institute

PÁGINA 20

MANUAL DE TRABAJO DEL CURSO ENARM CMN SIGLO XXI hasta hace 3 meses que comenzó padecimiento actual caracterizado por fatiga, disnea de medianos esfuerzos, sensación de ahogamiento de predominio nocturno, es ingresada debido a que presenta tos con esputo fino color rosa, disnea de leves esfuerzo, a la exploración física se observa paciente con estado grave, edematizada con cianosis central y periférica, ingurgitación yugular, soplo mediodiastolico III/IV el cual predomina en posición erecta, edema de miembros pélvicos con lesiones isquémicas en dedos 1 y 2 de pie derecho, tratado y contaminado con remedios locales, no se apercibe olor ni pus, los signos vitales son los siguientes TA 125/85 mmHg, FC 102, FR 26, Temperatura 37.5 C. La paciente recibe diagnostico de insuficiencia cardiaca congestiva. PREGUNTA Cuál es la causa más probable para la insuficiencia cardiaca congestiva de este caso. RESPUESTA a.- Endocarditis bacteriana con insuficiencia valvular. b.- Estenosis de la valvula mitral. c.- Mixioma auricular. d.- Melanoma metastasico. CASO CLINICO Se estabiliza a la paciente y se realizan estudios de gabinete donde se identifica una masa tumoral intracavitaria de 6 centímetros, dentro de los estudios se realizaron TAC, IRM y cateterismo, 4 horas después la paciente presenta dificultad respiratoria intensa, presencia de tos abundante con esputo asalmonelado, por lo que se intuba para soporte ventilatorio, se observa cianosis periférica con pérdida de pulsos distales de miembros pélvicos, ausencia de ruidos abdominales seguido de parada cardiaca en dos ocasiones, declarando su muerte 2 horas más tarde. PREGUNTA Cuál es la fue la intervención más probable que culmino con la embolia masiva en esta paciente. RESPUESTA a.- No se anticoagulo profilácticamente. b.- El cateterismo cardiaco innecesario. c.- El desprendimiento del mixioma. d.- El paciente ya tenía signos de embolia. CASO CLINICO Hombre de 20 años de edad sin antecedentes mórbidos conocidos. Presentó cuadro de 3 días de evolución caracterizado por odinofagia, tos seca, dolor precordial y dorsal asociado a disnea de reposo. Consultó en centro de atención donde se le solicita radiografía de tórax, destacando gran cardiomegalia y signos de congestión pulmonar. Se contaba con examen físico y radiografía de tórax de 3 años atrás sin ninguna alteración. PREGUNTA Fue derivado a centro de atención terciaria al cuarto día de evolución del cuadro. Hemodinamia siempre estable. Se realizo electrocardiograma (ECG). Cual es la conducta farmacológica inicial mas apropiada para el caso? RESPUESTA a.- Amiodarona de 100 a 400 mg cada 24 horas. b.- Diltiazem de 90 mg a 120 mg cada 12 horas. c.- Verapamilo de 240 a 360 mg cada 24 horas. d.- Esmolol de 50 a 100 µg/kg/min. PREGUNTA Se realizo un electrocardiograma previo a la decisión farmacológica, considerando el ECG el cual se observa anormal, cual de las siguientes alteraciones del ritmo es más probable encontrar considerando la evolucion? RESPUESTA a.- Taquicardia ventricular intermitente. b.- Taquicardia auricular intermitente rapida c.- Taquicardia sinusal. d.- Taquicardia auricular. PREGUNTA Ingresó hipotenso (84/56 mmHg), taquicárdico (116 latidos por minuto, irregular), afebril, frecuencia respiratoria de 18 por minuto y saturando 92% con oxígeno ambiental, bien perfundido. La faringe se encontraba algo enrojecida, yugulares ingurgitadas. Sin congestión pulmonar y examen cardíaco demostró tercer ruido y ritmo de galope. Extremidades sin edema. De los exámenes de laboratorio destacaba discreta leucocitosis, PCR baja, enzimas cardíacas y troponinas normales, cual es la conducta mas adecuada a seguir?

CURSO ENARM CMN SIGLO XXI TEL: 36246001

Pharmed Solutions Institute

PÁGINA 21

MANUAL DE TRABAJO DEL CURSO ENARM CMN SIGLO XXI RESPUESTA a.- Prueba de esfuerzo con dobutamina. b.- Ecocardiograma. c.- Biopsia endocardica. d.- Holter ambulatorio. PREGUNTA Entre otros estudios se realizó ecocardiograma, considerando el cuadro clínico actual cual de las siguientes aseveraciones es menos probable esperar? RESPUESTA a.- Ventrículo izquierdo (VI) severamente dilatado. b.- Disfunción sistólica global severa. FE > 40 %. c.- Hipokinesia difusa, disfunción diastólica tipo III de IV. d.- Disfunción sistólica ventricular derecha e (HTP). CASO CLINICO Una paciente de 41 años se encontraba en espera para ablación de vía accesoria (VA) en el contexto de un síndrome de WolffParkinson-White (WPW). La paciente, estando todavía asintomática, había sido diagnosticada de WPW 9 años antes en una revisión rutinaria. Tres años antes del actual ingreso empezó a referir episodios cortos (< 5 min) de palpitaciones de inicio y fin súbitos, que nunca fueron registradas por electrocardiograma (ECG). En los últimos 9 meses empezó a sufrir clínica compatible con insuficiencia cardiaca en forma de disnea de esfuerzo clase funcional II de la New York Heart Association. La exploración física era normal, pero en la ergometría solo alcanzó 6 MET. PREGUNTA Cuales son los datos que se registraron en la imagen 1A antes de la ablación? RESPUESTA a.- Ritmo sinusal con BRD. b.- Ritmo sinusal con preexitacion con BRI. c.- Ritmo sinusal con preexcitación por una VA anteroseptal derecha con (BRI). d.- Ritmo sinusal con preexitacion por una VA anterioseptal derecha con BRD. PREGUNTA Considerando la evolución clínica y el ECG 1B, el paciente paso de una FE del 48 % al 63 %, cual es la tasa de mortalidad en este momento para el pacientes? RESPUESTA a.- Tasa de mortalidad del 6%. b.- Tasa de mortalidad del 17%. c.- Tasa de mortalidad del 38%. d.- Tasa de mortalidad del 81%. CARDIOPATIAS CONGENITICAS (ADULTO) DEFINICION: La incidencia de las formas moderadas a severas de las cardiopatías congénitas es de 6 por 1000 nacidos vivos, los pacientes sin manejo farmacológico y/o quirúrgico temprano no llegan a la vida adulta, ya que con el tratamiento adecuado el pronostico es de 85% para llegar a la vida adulta, la mayor parte de los adultos con cardiopatías congénitas no requieren tratamiento llegando a la vida adulta. Los defectos mas frecuentes son 1.- defecto septal ventricular estrecho, 2.- defecto artrial septal secundario, 3.- estenosis pulmonar leve a moderada, 4.- valva aortica bicuspidea, 5.- prolapso de valvula mitral. DIAGNOSTICO: Los adultos con cardiopatías congénitas presentan tres formas básicas: con una historia de cirugía previa paliativa o reparativa durante la infancia, 2.con conocimiento de la cardiopatía congénita sin intervención, 3.- presentación en la vida adulta sin antecedentes de la infancia del padecimiento. La excepción se presenta en los pacientes con persistencia de conducto arterioso leve. Cianosis en el adulto debe buscar la causa: Eritrocitosis, deficiencia de hierro, sangrado, o cardiovascular. Cuando se presentan las siguientes alteraciones: Alteracion de la función ventricular (es la mas frecuente que presente síntomas), arritmias (las atriales es la mas frecuente, en las ventriculares son secundarias a fibrosis, dilatación ventricular, y son causas de muerte súbita). Enfermedad por conducción, (intriseco o posoperatorio de enfermedad de noso SA con disfunción) se puede observar fracción baja de eyección, disincronia, puede causar bloqueos. DEFECTO ATRIAL SEPTAL: 75 % es defecto secundario a ostium encontrado en el adulto, se observa murmullo y alteraciones inespecíficas de RX y ECG, presencia de segundo ruido, sobrecarga ventricular derecha crónica, hipertensión pulmonar, arritmias atriales, raramente embolizacion paradoxica). Tratamiento: Especifico por alteración. PREVENCION: Endocarditis (es mas frecuente cuando hay antecedentes de cardiopatía congénita), Embarazo (cuando hay antecedentes de lesiones estenoticas severas pulmonar, hipertensión pulmonar, shunts derecha a izquierda, regurgitación, vigilancia estrecha por riesgo de endocarditis durante el embarazo y sobrecarga a las 20 a 22 SDG indicado ecocardiografia), Anticoncepcion (Los Estrogenos incrementan mas los riesgos para tromboembolico, usar contracepción con progestágenos); Ejercicio (identificar taquicardia inducida, evaluar la competencia cronotropica, se contraindica el ejercicio intenso isométrico). DEFECTO SEPTAL VENTRAL: Existen 4 tipos, la perimembranosa es la mas común, la muscular es común en la infancia con cierre expontaneo, puede presentar murmullo holosistolico inversamente relacionado al tamaño del defecto, pueden desarrollar el complejo Eisenmenger al no tratarse. TRATAMIENTO: al desarrollar hipertensión pulmonar puede estar indicado

CURSO ENARM CMN SIGLO XXI TEL: 36246001

Pharmed Solutions Institute

PÁGINA 22

MANUAL DE TRABAJO DEL CURSO ENARM CMN SIGLO XXI sindanafil o análogos de prostanglandinas. DEFECTO SEPTAL ATRIOVENTRICULAR: Se presenta lesiones en los septos atriales y ventriculares clasificados en parcial o complejo, se acompaña comúnmente de subestenosis aortica, puede coexistir regurgitación mitral. El ECG puede mostrar desviación a la izquierda del eje, y bloqueo incompleto, lesión frecuente en síndrome de down, requiere reparación antes de los 6 meses de edad para prevenir la hipertensión pulmonar y consecuentemente el complejo Eisenmenger. TETRALOGIA DE FALLOT: Es la mas común de las cardiopatías congénitas cianogenas representa del 7 al 10 %, difícilmente paciente sin antecedentes solo en forma minimas, pueden presentar alteraciones de las intervenciones realizadas durante la vida adulta. Las arritmias son las manifestaciones mas frecuentes en la vida adulta, junto a la disfunción ventricular derecha, Tratamiento con marcapaso y defibrilador interno. ESTENOSIS PULMONAR AISLADA: Presentan estenosis infundivular subpulmonar pudiendo sobrevivir a la vida adulta, presente en sx de noonan, Tratamiento con valvuloplastia con balón. OBSTRUCCION DEL TRACTO DE SALIDA DEL VENTRICULO IZQUIERDO: Obstruccion valvular bicuspidea, subvalvular o supravalvular, puede presentarse afecciones en multiples niveles. COARTACION DE LA AORTA: Presente en el adulto con menor posibilidad de reparación, se presenta predominantemente distal al inicio de la subclavia izquierda, es silenciosa hasta que se presenta un evento súbito como hipertensivo. TRANSPOSICION DE GRANDES ARTERIAS: La sobrevida a la vida adulta es casi imposible, se realiza un swich de vasos en la infancia, 50 % de los casos desarrollan trasntornos sinusales, arritmias, regurgitación valvular, falla ventricular derecha. CASO CLINICO Posteriormente a la corrección de la tetralogía de Fallot realizada con 19 años de edad, con ampliación del tracto de salida del ventrículo derecho, valvotomía pulmonar y cierre de la comunicación interventricular, deliberadamente quedaron como defectos residuales, la estenosis pulmonar al nivel valvular por un pequeño anillo pulmonar y la comunicación interatrial. Pero incluso así, permaneció con una buena evolución por 28 años cuando surgió la primera complicación por embolia cerebral paradójica con un discreto accidente cerebrovascular, sin consecuencias sensoriales o motoras. Además, hace un año se suscitó una complicación arrítmica por fibrilación atrial con frecuencia cardíaca elevada, que fue revertida con amiodarona y choque eléctrico. Recientemente, una nueva fibrilación atrial, pero ahora con una frecuencia ventricular baja (entre 50 a 60 lpm), y insaturación arterial (70%), posteriormente a la deambulación en pocos minutos, motivó la investigación diagnóstica y una conducta más precisa. Al examen físico, el paciente se presentó en un buen estado general, eupneico, enrojecido, con pulsos normales, pesando 114 kg, altura de 170 cm, PA de 110/80 mmHg y FC de 60 lpm en ritmo de fibrilación atrial. La aorta se palpaba discretamente en la fúrcula. En el precordio, no había impulsos o deformidades y el ictus cordis no se palpaba. Los ruidos eran hipofonéticos y se auscultaba un soplo sistólico, +/++ de intensidad, de eyección, rudo, en el 1º, 2º y 3º espacios intercostales izquierdos en el borde esternal y en la fúrcula. El hígado no se palpaba. PREGUNTA Cual de las siguientes condiciones es la mas probable causa del estado actual del paciente? RESPUESTA a.- Estenosis pulmonar valvular moderada e hipertrofia del ventrículo derecho b.- Shunt de sangre de la derecha hacia la izquierda por comunicación interatrial c.- Defectos residuales posteriores a la corrección de la tetralogía de Fallot d.- Insaturación arterial grave a pequeños esfuerzos. CASO CLINICO Mujer de 69 años, intervenida a los 48 años por CIA tipo ostium primum con disnea de medianos esfuerzos. Se cerró quirúrgicamente el defecto con parche de teflón y se corrigió, mediante sutura, la hendidura de la valva anterior mitral. Precisó implante de marcapasos VVI por bloqueo auriculoventricular completo. Permaneció asintomática hasta 22 años después, cuando presentó dolor torácico inespecífico y una prueba de esfuerzo interrumpida por la disnea. Ecocardiográficamente presentaba un ventrículo izquierdo hipertrófico con función conservada e hipertensión pulmonar severa. La válvula mitral estaba fibrosada, con insuficiencia moderada e insuficiencia aórtica ligera. Presentaba un gradiente sistólico fijo en el tracto de salida del ventrículo izquierdo (TSVI) de 84 mmHg, ocasionado por una membrana, que parecía partir de la valva anterior mitral. En el cateterismo cardíaco se diagnosticó una estenosis severa del tronco de la coronaria izquierda. PREGUNTA La ESA tras cirugía correctora de CIA de tipo ostium primum es infrecuente, considerando las datos previos referidos cual de las siguientes afirmaciones es menos probable? RESPUESTA a.- Afecta al 2-15% de los casos b.- Suele diagnosticarse 6-8 años tras corrección del defecto en la infancia. c.- Falta de seguimiento postoperatorio en adultos por evolución más lenta. d.- Su incidencia es de 60-70% y permanece silente durante pocos años. CASO CLINICO Una mujer de 43 años de edad. Su historial médico incluye la reparación de una transición atrioventricular (AV), comunicación interventricular (D-AV) a la edad de 14 años, que consiste en un ostium primum AVSD y musculoso comunicación interventricular restrictiva (VSD), asociado con un velo anterior mitral hendida. Después de su cirugía, ella permaneció asintomática hasta que tenía 34 años, cuando experimentó la fatiga progresiva. Una ecocardiografía transtorácica mostró shunt residual, tricúspide moderada e insuficiencia aórtica, insuficiencia mitral moderada y estenosis subaórtica con un tracto de salida del ventrículo izquierdo fijo (TSVI)

CURSO ENARM CMN SIGLO XXI TEL: 36246001

Pharmed Solutions Institute

PÁGINA 23

MANUAL DE TRABAJO DEL CURSO ENARM CMN SIGLO XXI gradiente pico de 52 mm Hg. La estenosis subaórtica se agravó en los últimos años, alcanzando un máximo gradiente sistólico de 100 mm Hg. Permaneció activa, pero experimentó episodios diaforéticas más frecuentes con actividad leve. PREGUNTA ¿Cual de las siguientes alteraciones favorece más la condición actual de ESA? RESPUESTA a.- Crestas subaórtica fibromuscular. b.- Anormales válvula AV izquierda. c.- Cuerdas tendinosas de los archivos adjuntos. d.- Inserción anómala de los músculos papilares del ventrículo izquierdo y generalizada hipoplasia del LVOT. CASO CLINICO Masculino de 20 años con soplo cardíaco, cianosis y cansancio observados desde el nacimiento, con nítida acentuación en los últimos años. Sin control médico, hematocrito de 83% y hemoglobina de 28 g. Hemaféresis repetidas disminuyeron el hematocrito para 73%. Ninguna medicación específica había sido administrada desde el nacimiento. Examen físico: Regular estado general, eupneico, cianosis acentuada, pulsos normales. Peso: 42,6 Kg; Altura: 160 cm; PA: 105/65 mmHg; FC: 82 lpm; FR: 20 rpm. Saturación O2: 75%. La aorta era discretamente palpada en la fúrcula. En el precordio había pulsos sistólicos discretos, frémito sistólico nítido en todo el borde esternal izquierdo. Ictus cordis no era palpado. Los ruidos cardíacos eran muy hiperfonéticos y se auscultaba soplo sistólico acentuado en todo el borde esternal izquierdo, con nítida irradiación hacia el área aórtica. El hígado no era palpado y los pulmones eran limpios. PREGUNTA Considerando el cuadro clínico, antecedetnes y frecuencia de las cardiopatías congenitcas, cual es la condición mas probable del caso? RESPUESTA a.- Estenosis pulmonar. b.- Doble vía de salida de ventrículo derecho. c.- Doble vía de entrada de ventrículo único. d.- Estenosis pumonar aislada. CASO CLINICO Varón de 21 años con atresia pulmonar y comunicación interventricular intervenido de anastomosis aortopulmonar de WaterstonCooley (WC) en el primer mes de vida. A los 2 años se le realizó fístula de Blalock-Taussig (BT) izquierdo modificado y a los 8, ampliación de arterias pulmonares con parche de Dacron, implantándose además un conducto no valvulado entre el ventrículo derecho y las arterias pulmonares. A los 12 años se cerró la comunicación interventricular mediante un parche en teja y se reconstruyó el conducto con técnica de Danielson. Dos años más tarde, el cateterismo cardíaco demostró estenosis severa entre el tronco pulmonar y ambas ramas, con presiones sistémicas en el ventrículo derecho. Se realizó angioplastia con balón de ambas ramas, que fue ineficaz. Nueve años después el paciente se encontraba en grado funcional III de la NYHA y la ecocardiografía puso de manifiesto presión suprasistémica en el ventrículo derecho, comunicación interventricular residual pequeña con cortocircuito bidireccional e insuficiencia tricuspídea grado III/IV. PREGUNTA Cual de las siguientes causas es menos frecuente en la estenosis e hipoplasias localizadas de las arterias pulmonares secundarias a cirugía paliativa? RESPUESTA a.- Constriccion progresiva. b.- Ineficacia de angioplastia con balón. c.- Distorsión de tejido fibroso en su interior. d.- Crecimiento de tejido fibroso en su interior. CASO CLINICO Mujer de 28 años con atresia pulmonar y comunicación interventricular intervenida quirúrgicamente a los 4 años de edad, practicándosele fístula de BT izquierdo clásica. A los 6 años se le realizó anastomosis de WC por estenosis de la fístula. A los 14 años presentaba cianosis progresiva severa y disnea de esfuerzo. El cateterismo cardíaco demostró estenosis severa de la fístula de BT e hipoplasia de las arterias pulmonares, descartándose para cirugía correctora. A los 26 años, la paciente presentaba cianosis severa, acropaquias y disnea de mínimos esfuerzos, decidiéndose la realización de una nueva fístula paliativa, que no pudo completarse por la aparición de sangrado muy abundante durante la disección quirúrgica. Mediante angiografía se demostró obstrucción completa de la fístula de BT en su tercio distal y estenosis severa de la anastomosis de WC. PREGUNTA El tratamiento quirúrgico de esta complicación es técnicamente difícil y la dilatación con balón habitualmente ineficaz por que razo? RESPUESTA a.- Retroceso elástico de la arteria. b.- Fibrosis persistente suprapulmonar.

CURSO ENARM CMN SIGLO XXI TEL: 36246001

Pharmed Solutions Institute

PÁGINA 24

MANUAL DE TRABAJO DEL CURSO ENARM CMN SIGLO XXI c.- Falta de angenten inmunomoduladores. d.- Persistencia de las resistencias intracavitarias. CASO CLINICO Varón de 19 años diagnosticado de tetralogía de Fallot severa a los 3 años de edad, practicándosele fístula de BT izquierdo seis meses después. A los 5 años se rechazó la cirugía correctora por presentar hipoplasia severa de tronco y arterias pulmonares. Desde entonces presentaba cianosis progresiva severa, disnea de esfuerzo, acuclillamiento y síncopes. A los 9 años se realizó ampliación con parche del tracto de salida del ventrículo derecho y del origen de ambas ramas pulmonares, así como cierre de la comunicación interventricular con parche fenestrado, quedando con presión sistémica en el ventrículo derecho, gradiente transpulmonar severo y cortocircuito bidireccional ventricular. PREGUNTA Cual de las siguientes aseveraciones es correcta, al manejo de stens en esta patologia?. RESPUESTA a.- La dilatación mediante stent no es un medoto seguro ni eficaz. b.- La combinación de stent y valvuloplastia con balón tiene mayor eficacia y eficiencia. c.- La dilatación mediante stent es un método seguro y eficaz de tratamiento. d.- En las estenosis de las arterias pulmonares en adultos con cardiopatías congénitas operadas en la infancia no es útil los Stens. VALVULOPATIAS (TEMA) CIENCIAS BASICAS: Estenosis valvular o dificultad para que la válvula pueda abrirse. Insuficiencia valvular o imposibilidad de que la válvula al cerrarse ocluya completamente el orificio valvular, reflujo anormal de sangre. Focos de auscultación: foco mitral o apexiano (5to. espacio intercostal izq. con línea medio clavicular); foco aórtico (2do. espacio intercostal con paraesternal der.); foco tricuspídeo (porción inferior del esternón); foco pulmonar (2do. espacio intercostal paraesternal der.). Pueden generarse a consecuencia de fiebre reumática (infección de origen estreptocócico más mecanismo inmunológico afectando endocardio valvular), endocarditis bacteriana (da lugar a formación de vegetaciones en las válvulas mutilándolas), malformaciones congénitas, degeneración mixomatosa (prolapso de los velos valvulares en insuficiencia), disfunción y/o rotura de las cuerdas tendinosas (IAM). ESTENOSIS MITRAL: Valvulopatía más frecuente. Su principal causa es la fiebre reumática. Más a mujeres. PATOGENIA: Disminución del área del orificio valvular mitral (normal 5 cm), con lo cual hay dificultad para el relleno del VI, durante la diástole, esto va generando hipertrofia de la pared auricular, porque se requiere mayor presión para mantener el flujo hacia el VI, que puede desencadenar una FA, puede haber estasis de sangre en la aurícula, generar un trombo y este embolismo sistémico. Área inferior a 2cm, genera síntomas al ejercicio, área 1.5 leve; 1.0-1.5 moderada; < 1.0 severa)y el gradiente transvalvular. TRATAMIENTO: De elección es quirúrgico (prótesis mecánica). INSUFICIENCIA AÓRTICA: Causas fiebre reumática, sífilis y endocarditis bacteriana. PATOGENIA: Los velos valvulares aórticos, no tienen un cierre

CURSO ENARM CMN SIGLO XXI TEL: 36246001

Pharmed Solutions Institute

PÁGINA 25

MANUAL DE TRABAJO DEL CURSO ENARM CMN SIGLO XXI hermético, lo que genera reflujo diastólico esto aumenta la precarga y genera hipertrofia del VI, que puede llevar a IC, por otro lado el aumento en el consumo de O2 y la disminución de la presión diastólica, lleva a isquemia, frecuente en el Sx. de Marfan, disección aórtica entre otras. DIAGNOSTICO: Cuadro clínico: manifestaciones de IC o isquemia hasta 4 o 5 década de la vida, disnea en reposo, ortopnea, disnea paroxística nocturna, angina, palpitaciones, pulso capilar (signo de Quincke)c se presiona sobre la uña se observa, como el borde de la zona pálida avanza y retrocede, durante la sístole y diástole. Danza carotidea (signo de Corrigan), latido carotideo visible y muy amplio, ampliación de la TA diferencial por elevación de la TA sistólica y disminución de la diastólica, que incluso puede ser 0. Auscultación: 1er y 2do ruidos apagados, soplo pandiastólico decreciente (por la progresiva caída de la presión en la raíz de la aorta durante la regurgitación), de tono alto, suave y aspirativo, irradiándose a lo largo del borde esternal. ECG: Signos de crecimiento del VI, sobrecarga diastólica (ondas T picudas y supradesnivel del segmento ST. Rx. cardiomegalia con signos de crecimiento del VI y dilatación de la raíz aortica, ecodopler permite medir grado de ingurgitación. TRATAMIENTO: Enfocado a corregir síntomas de IC, oxigeno, diuréticos, inotrópicos, IECAS. Qx definitivo, se recomienda en pacientes con diámetros ventriculares >50mm. ESTENOSIS TRICÚSPIDEA: Condiciona sobrecarga de presión en sentido retrógrado sobre la aurícula y congestión venosa con clínica de IC der. Auscultación en foco tricúspide soplo diastólico, con refuerzo presistólico y chasquido de apertura de la tricúspide. INSUFICIENCIA TRICÚSPIDEA: Clínica de IC der. por regurgitación sistólica, hacia la aurícula. Auscultación: soplo sistólico, similar al de la insuficiencia mitral, pero se ausculta mejor en el foco tricúspide., se incrementa durante la inspiración, por aumentar esta condición el retorno venoso. ESTENOSIS PULMONAR: IC der., sobrecarga de presión del ventrículo der.. Auscultación: Foco pulmonar muestra un clic de eyección y un soplo sistólico de morfología romboidal. INSUFICIENCIA PULMONAR: IC der., regurgitación y sobrecarga del VD. Auscultación: soplo diastólico decreciente en foco pulmonar, equiparable en su génesis al de la insuficiencia aortica. CLAVES: La profilaxis con antibióticos es recomendada, si hay procedimientos invasivos, por alto riesgo de endocarditis infecciosa. CASO CLINICO Paciente de 66 años de edad, con obesidad, hipertensión arterial y dislipemia con diagnóstico de estenosis aórtica grave y cardiopatía coronaria al realizarse ecocardiografías transtorácica y transesofágica; se concluyó estenosis aórtica grave (válvula aórtica con gradientes máximo de 68, medio de 46 y área de 0,95 cm2) con función sistólica conservada e hipertrofia concéntrica del ventrículo izquierdo (septum interventricular de 15 mm y pared posterior de 14 mm). El cateterismo cardiaco demostró enfermedad coronaria (lesión oclusiva del 80% en el tercio proximal de la descendente anterior y del 75% en el tercio medio de la circunfleja). PREGUNTA Considerando los factores del paciente cual es la causa mas probable? RESPUESTA a.- Calcificacion valvular. b.- Degenerativa. c.- Cardiopatia reumática. d.- Fibrosis autoinmune. PREGUNTA Cual es el pronóstico del paciente conociendo la mortalidad anual de esta patologia? RESPUESTA a.- 3 %. b.- 6 %. c.- 9 %. d.- 12 %. PREGUNTA Cual de las siguientes etiologias es menos frecuente en este caso? RESPUESTA a.- Sindrome de marfan. b.- Hipercolesterolemia. c.- Cardiopatia reumática. d.- Lupus eritematoso. CASO CLINICO Un varón de 87 años con estenosis aórtica sintomática fue remitido para implante de válvula aórtica percutánea. El ecocardiograma preoperatorio mostraba estenosis valvular aórtica severa (área calculada, 0,6 cm2), fracción de eyección ventricular izquierda normal y anatomía cardiaca adecuada para el implante percutáneo. La coronariografía previa no mostró lesiones significativas. El paciente fue rechazado para cirugía por su elevada comorbilidad (EuroSCORE, 23%). PREGUNTA Considerando la clasificación del padecimiento este paciente esta considerado como? RESPUESTA a.- Leve. b.- Moderada.

CURSO ENARM CMN SIGLO XXI TEL: 36246001

Pharmed Solutions Institute

PÁGINA 26

MANUAL DE TRABAJO DEL CURSO ENARM CMN SIGLO XXI c.- Grave. d.- Critica. PREGUNTA Cual de los siguientes síntomas no esta relacionado con el incremento de la presión venocapilar pulmonar? RESPUESTA a.- Disnea en reposo. b.- Ortopnea c.- Disnea paroxística nocturna. d.- Opresion toracicca. CASO CLINICO Paciente de femenino de 65 años de edad, se detectó hipertensión arterial (160/100 mmHg), fibrilación auricular de inicio indeterminado. Se inició tratamiento con atenolol 50 mg/día, enalapril 20 mg/día. Desde el último año disnea progresiva con esfuerzos menores de los habituales y palpitaciones frecuentes espontáneos. En la auscultación cardíaca se apreció arritmia completa por fibrilación auricular, un soplo holosistólico 4/6 de alta frecuencia en el foco mitral irradiado a la axila y otro sistólico, de menor intensidad, en el borde esternal izquierdo bajo, que aumentaba con la inspiración. En la radiografía de tórax se apreció una cardiomegalia global grado III/IV con campos pulmonares normales. El ECG demostró una fibrilación auricular controlada, con signos de crecimiento ventricular izquierdo y alteraciones secundarias de la repolarización. PREGUNTA Cuál es el manejo más adecuado para el manejo de la insuficiencia cardiaca. RESPUESTA a.- Amiodarona. b.- Hidroclorotiazida. c.- Digoxina. d.- Nifedipina. PREGUNTA Cuál es el manejo más adecuado para el manejo de los tratornos del ritmo. RESPUESTA a.- Amiodarona. b.- Hidroclorotiazida. c.- Digoxina. d.- Nifedipina. PREGUNTA Cual es la etiología mas probable considerando los factores de riesgo que presenta la paciente? RESPUESTA a.- Fiebre reumática. b.- Sindrome de lutenbacher. c.- Enfermedad de fabry. d.- Enfermedad de whipple. PREGUNTA Considerando la condición de la paciente cual de las siguientes causas es la más frecuente de muerte? RESPUESTA a.- Embolia sistémica. b.- Insuficiencia cardiaca. c.- Embolia pulmonar. d.- Infección CASO CLINICO Mujer de 83 años, con prótesis mitral mecánica. Fibrilación auricular crónica. Ecocardiograma transesofágico: área valvular de 0,5cm2, hipertensión pulmonar de 75mmHg, fracción de eyección del 65%, anillo aórtico de 18 mm y prótesis mitral normofuncionante. La paciente evolucionó sin complicaciones. En la revisión de 1 y 3 meses se encuentra en clase funcional I. PREGUNTA Cual es el agente etiológico mas frecuente de esta patología. RESPUESTA a.- Estreptococcos

CURSO ENARM CMN SIGLO XXI TEL: 36246001

Pharmed Solutions Institute

PÁGINA 27

MANUAL DE TRABAJO DEL CURSO ENARM CMN SIGLO XXI b.- Estafilococcos. c.- Treponema. d.- Cocobacilos CASO CLINICO Mujer de 74 años de edad hipertensa, dislipémica, diabética tipo 2, con nefropatía diabética e insuficiencia renal. Fue sometida a una comisurotomía mitral, sustitución valvular mitral con prótesis. Sufre fibrilación auricular crónica. Deterioro de clase funcional, tras descartar disfunción de la prótesis mitral, se la remitió para valoración de TAVI. Rechazada para cirugía (EuroSCORE, 25%). Coronariografía: sin estenosis angiográficas. En el ecocardiograma transesofágico se observó lo siguiente: presión pulmonar sistólica de 80mmHg, anillo aórtico de 22mm y área valvular de 0,6cm2. Distancia mitroaórtica, 7 mm. PREGUNTA Cual es el factor de mayor influencia en el pronóstico a mediano plazo. RESPUESTA a.- Diabetes mellitus. b.- Dislipidemia. c.- Hipertensión d.- Cardiopatia coronaria. CASO CLINICO Paciente masculino, 20 años, previamente sano, presenta episodio de disnea durante el sueño, se quejaba de cansancio a los esfuerzos medios con evolución de alrededor de 15 días, siendo caracterizado como clase funcional II de la NYHA (New York Heart Association). Al examen físico, presentaba presión arterial 110/70 mmHg, ritmo cardíaco regular en dos tiempos, soplo sistólico en foco mitral. El ECG evidenciaba bloqueo de rama derecha. PREGUNTA Cual es la terapeutica mas apropiada para el trastorno del ritmo. RESPUESTA a.- Marcapasos. b.- Amiodarona. c.- Amlodipido. d.- Digoxina. CASO CLINICO Posteriormente a la corrección de la tetralogía de Fallot realizada a los 19 años de edad de la paciente, con ampliación del tracto de salida del ventrículo derecho, valvotomía pulmonar y cierre de la comunicación interventricular, quedaron como defectos residuales, estenosis pulmonar al nivel valvular por un pequeño anillo pulmonar y comunicación interatrial. Permaneció con una buena evolución por 28 años cuando presenta embolia cerebral por fibrilación atrial. PREGUNTA Considerando el cuadro clínico cual de los siguientes fármacos es el mas apropiado para mantener controlada la agregación plaquetaria. RESPUESTA a.- Acetilsalicilico. b.- Clopidrogel. c.- Acecumarol. d.- Enoxoparina. CASO CLINICO Mujer de 59 años con historia de implantación de marcapasos definitivo endocavitario por bloqueo auriculoventricular completo. Presentó trombosis de vena cava, que originó un síndrome de vena cava superior y requirió de la colocación de un stent autoexpandible. Presentó estenosis de la vena subclavia izquierda y tromboembolia pulmonar. Es egresado luego de 30 dias de estancia hospitalaria. PREGUNTA Cual de los siguientes factores etiológicos es mas frecuente en esta patología. RESPUESTA a.- Insuficiencia venosa profunda. b.- Arterio esclerosis de grandes vasos. c.- Insuficiencia tricuspidea. d.- Anticoagulacion insuficiente.

CURSO ENARM CMN SIGLO XXI TEL: 36246001

Pharmed Solutions Institute

PÁGINA 28

MANUAL DE TRABAJO DEL CURSO ENARM CMN SIGLO XXI PERICARDITIS DEFINICION: El pericardio consiste en una capa visceral y una lamina parietal que entre si forma un espacio potencial, la cavidad pericardica contiene 50 ml de plasma infiltrado, sus fnciones limitan la distención cardiaca, facilita el acople e interacion cardica, mantienen el volmen de presión cardiaca, la ergometría del ventrículo izquierdo, además de que lubrica, minimiza la friccion, regula la inercia, las fuerzas hidrostática y gravitacionales del coraccion y es una barrera mecánica contra la infecccion, presenta acciones inmunológicas, vasomotoras, fibrinoliticas, modula la estructura del miocito, su función, asi como su expresión génica, es un vehiculo para fármacos y terapia génica. No es escencial para la vida, sin embargo puede comprometerlo al presentar una patologia. PERICARDITIS AGUDA: Es un estado agudo firbinoso o sudativo por pericarditis, es un Sx caracterizado por dolor en pecho, friccion pericardica, cambios ECG, manifestando grado, severidad y localización, puede modificar el ST o alteraciones de la repolarizacion como en el IAM, aun cuando estos son difusos, el RX puede ser normal o relevar la silueta cardiaca por exudado aumentado, sugiriendo miocarditis o taponade por ejemplo, los marcadores imflamatorios puede incrementarse, en caso de isoenzimas cardiacas es mas frecuente en epicarditis, en epicarditis idiopáticas pueden elevarse semejando IAM, la hospitalización de estos casos son para diferenciar los casos, responden frecuentemente a AINES, añadiendo profilaxis con H2 bloqueadores, la adicion de colchicina es útil en casos de recurrencias, el dolor permanece 1 a 2 dias, el frote y el ECG giualmente, en caso viral se prolonga a mas de 4 dias o en caso de drogadicción el dolor es mas severo, requiririendo corticoides sistémicos. DERRAME PERICARDICO: Es el acumulo de transudado, exudado o hemosudado en el saco pericaidico frecuente en las complicaciones de enfermedades del pericardio, en pericarditis aguda, el hidropericardio se presenta en pericarditis crónica frecuente en retenion de agua y sodio, o procesos inflamatorios crónicos, la presencias hemática o sero hemática es mas frecuente en infecciones o inflamaciones, en pericarditis quilosa  lesiones u obstruccion del conducto torácico, por coresterol, hipotiroidismo, artritis reumática, o tuberculososis. El ECG es el procedimiento de elección, la TAC y el IRM son utiles para identificar la inoculación típica o atípica, la etiología del derrame pericardico deberá ser identificado por citología, análisis inmunológico o por biopsia guiada. La pericardiocentesis es diagnostica y terapéutica, ya que evita la falla del corazón hemodinamicamente hablando, por pulso paradójico, disfunción atrial y ventricular diastólica, generando pulso paradoxico, disminusion de la presión arterial, hipotensión arterial, la fusión cardiaca decrese hasta verse negativa a negativa, hasta llegar al prolapso y falla cardiaca por agotacion de los mecanismos compensatorios. TAPONADE CARDIACO: Definido como agudo o crónico, se observa como una alteración dinámica continua, con presiones de 10 a 20 mmHg como máximas, en los casos leves es asintomática, en la moderada el disconfort y la disnea predominan, las ondas yugulares predominan, se presenta bajo voltaje en el ECG, durante la inspiración disminuye, se debe realizar pericardocentesis cuando los síntomas los amerite, a los 50 ml ya iniciaran, sin embargo se recomienda cutltivo, citología. PERICARDITIS CONSTRICTIVA: Es una condición calcificante que limita la función diastólica de los ventrículos, idiopático principalmente, trauma cardiaco, quirúrgico o tuberculoso otras infecciones, neoplasias, radioterapia, falla renal y enfermedades del tejido conectivo como menor causa. La condición crónica congestivo a semeja enfermedad miocárdica y enfermedad hepática crónica. El paciente refiere fatiga, disnea, aumento de peso, disconfort abdominal y nausea. Puede llegar a ascitis, hepatoesplenomegalia, edema confundiendoce con cirrosis, signo de Kussmaul, Knock, tercer ruido, pulso paradójico. QRS de bajo voltaje, cambios en P y T, fluter atrial es común, la TAC e IRM define entre pericarditis restrictiva y cardiomiopatía contrictiva, pueden necesitar pericardiectomia con una mortalidad del 30 al 40 %, CASO CLINICO Varón de 48 años, fumador. Había participado recientemente en la recogida de restos de aves enfermas sacrificadas en granjas. Consultó en urgencias por dolor retroesternal que aumentaba con la inspiración profunda y la tos empeoraba con el decúbito. En la exploración física destacaba el hallazgo de febrícula (37,5 °C) y roce pericárdico. En el electrocardiograma y en la radiografía de tórax no se observaron en aquel momento hallazgos patológicos. Con el diagnóstico de pericarditis aguda el paciente fue remitido a su domicilio bajo tratamiento con ácido acetilsalicílico a dosis antiinflamatorias. Tras una mejoría inicial, consultó diez días más tarde por fiebre de 39 °C, tos con escasa expectoración mucosa y dolor torácico de características pericardíticas, de tres días de evolución. PREGUNTA Cual de las siguientes manifestaciones es menos probable encontrar? RESPUESTA a.- Roce pericárdico. b.- Alteración difusa de la repolarización con aplanamiento de las ondas T. c.- Radiografía de tórax con cardiomegalia e infiltrado pulmonar en la língula. d.- Determinación seriada de creatincinasa y de su fracción MB elevadas. CASO CLINICO Mujer de 60 años con DM diagnosticada 3 años, HTA de 10 años de evolución. Un año antes tenía una creatinina de 0,8 mg/dl; FGE (MDRD) >60 ml/min/1,73 m; sistemático y sedimento de orina sin alteraciones. Refiere en las 6 semanas antes del ingreso astenia progresiva, con posterior aparición de fiebre y tos seca. No presentaba disnea, dolor torácico ni alteración en la diuresis. En la exploración tenía una presión arterial (PA) de 190/80 mmHg, estaba eupneica, con presión venosa yugular elevada, auscultación cardíaca rítmica con roce pericárdico, normoventilación en ambos hemotórax y ausencia de edemas. En la analítica, presentaba una hemoglobina de 7,7 mg/dl, urea de 217 mg/dl, creatinina 4,5 mg/dl, potasio 3,6 mEq/l, saturación basal de O2 del 98%; en orina, tenía proteinuria de +++ y 60 hematíes/campo. En el ECG no había alteraciones. En la radiografía de tórax se observó una gran cardiomegalia, se solicitó una ecocardiografía de urgencia, en la que se evidenció un derrame pericárdico moderado-grave. PREGUNTA Cual es la conducta mas apropiada a seguir?

CURSO ENARM CMN SIGLO XXI TEL: 36246001

Pharmed Solutions Institute

PÁGINA 29

MANUAL DE TRABAJO DEL CURSO ENARM CMN SIGLO XXI RESPUESTA a.- Pericardiocentesis. b.- Pericardiectomia. c.- Diuretico, esteroide. d.- Esteroide, ciclofosfamida. CASO CLINICO Mujer de 69 años de edad que sufre un trauma torácico cerrado por compresión antero posterior al quedar atrapada por las puertas de un autobús y 30 días después comienza a presentar falta de aire a los esfuerzos, aumentando progresivamente hasta desencadenarse a los pequeños esfuerzos. Ingresa en el hospital con diagnóstico de cardiopatía isquémica. Se realiza ecocardiograma y se comprueba gran colección líquida pericárdica. PREGUNTA Cual de las siguientes manifestaciones es mas frecuente encontrar para establecer el diagnostico actual? RESPUESTA a.- Triada de Beck. b.- Signo de Kussmaul. c.- Disminución del voltaje de QRS. d.- Elevación de CPK-MB y Troponinas. PREGUNTA Considerando la fisiopatogenia del presente caso, que tipo de choque es mas frecuente que presente? RESPUESTA a.- Choque Distributivo. b.- Choque Obstructivo. c.- Choque Cardiogenito. d.- Choque hipovolemico. CASO CLINICO Varón de 55 años con recambio valvular mitral (1998) y síndrome pospericardiotomía, que cursó con fiebre y cansancio a mínimos esfuerzos. Posteriormente quedó asintomático. En abril de 2004 presentó derrame pericárdico severo y taponamiento cardiaco. Requirió pericardiocentesis. Tras ésta, quedó inicialmente asintomático. Reingresó en julio de 2004 y refierió que tras el alta clínica se sintió bien, pero que en pocas semanas comenzó a sentir cansancio a grandes y luego a moderados esfuerzos. En el momento de su reingreso la presión arterial (PA) era 110/70 mmHg, sin pulso paradójico, pero presentaba ingurgitación yugular. La auscultación cardiaca era rítmica, con 70 lat/min y ruidos protésicos normales. Presentaba una discreta hepatomegalia y edemas con fóvea en los miembros inferiores. PREGUNTA Cual de las siguientes afirmaciones no es correcta para el PEC? RESPUESTA a.- Es una infrecuente forma de síndrome pericárdico. b.- Donde la constricción cardiaca ocurre en presencia de derrame pericárdico significativo. c.- Los pacientes presentan taponamiento cardiaco, sin sintomáticos pospericardiocentesis. d.- Ésta se resuelve sólo tras pericardiectomía. CASO CLINICO Mujer de 32 años con disnea a los pequeños esfuerzos, edema de miembros inferiores y astenia hace tres meses, se le prescribió digoxina y furosemida, con una ligera mejora. Rechazaba comorbilidades cardiovasculares, pero con una historia de artralgia inespecífica en la adolescencia y tres abortos entre 20 y 30 años de edad. El paciente estaba demacrado con auscultación pulmonar normal y las extremidades bien perfundidas. En la evaluación cardíaca, se hallaba con FC = 98 lpm, PA = 100 / 70 mmHg con un ritmo cardíaco regular y un ruido diastólico mitral de +++/4+, definido como un ruido protodiastólico ("knock" pericárdico). PREGUNTA Cuál es la conducta a seguir para establecer el diagnostico. RESPUESTA a.- ECG. b.- TAC c.- Rx de torax. d.- IRM. PREGUNTA Cual de las siguientes manifestaciones no es propio de la pericarditis?

CURSO ENARM CMN SIGLO XXI TEL: 36246001

Pharmed Solutions Institute

PÁGINA 30

MANUAL DE TRABAJO DEL CURSO ENARM CMN SIGLO XXI RESPUESTA a.- Localización retrosternal súbito. b.- Naturaleza pleurítica. c.- Se exacerba con la inspiración. d.- Disminuye al inclinarse. PREGUNTA Cuales no son cambios que se presentan en ECG del paciente? RESPUESTA a.- Elevación del segmento ST de forma cóncava. b.- Hay desarrollo de ondas Q. c.- El voltaje de la onda R se mantiene. d.- Depresión del segmento PR. PREGUNTA Considerando los antecedentes del caso clínico cual es la etiología mas frecuente? RESPUESTA a.- Idiopatico. b.- Traumatico. c.- Bacteriano. d.- Viral. PREGUNTA Cual de no es una secuela de la patologia del paciente? RESPUESTA a.- Taponade cardiaco. b.- Pericarditis recurrente. c.- Prericarditis restricitva. d.- Pericarditis constrictiva. CARDIOMIOPATIA Y MIOCARDITIS (TEMA) Es un espectro de patologías con diversos mecanismo patogénicos, con un final semejante a un síndrome congestivo de falla cardiaca, el daño o padecimiento del miocardio puede ser primario o secundario, las causas de miocardiopatias mas frecuentes son: miocarditis infecciosa viral (coxsaxkievirus, echovirus, HIC, Epstein barr, influenza, CMV. Bacteriano (Corynebacterium diphtheriae, streptococcus pyogenes, staphylococcus aureus, haemophilus pneumoniae, salmonella spp, neisseria gonorrhoeae, leptospirosis, lyme, syphilis, brucelosis; Fungico (Candida spp, arpergillus spp, histoplasmosis, blastomicosis, cryptoccosis, coccidioidomycosis; Parasitario (Toxoplasmosis, schistosomiasis, trchinosis). Dilatada (desconocida); Infiltrativa (amiloidosis, sarcoidosis, hemocromatosis). MIOCARDITIS: Las causas mas frecuentes son procesos infecciosos virales del 1 al 9 % son coxsackie virus B, las manifestaciones son variadas, puede pasar asintomáticas hasta un profundo choque cardiogenico, el antecedente de afeccion viral de 7 a 10 dias y ataque al estado generalizado el 60 % de los casos, 35% presentan dolor torácico, otra presentación son los bloqueos cardiacos completos o taquicardia ventricular. Las manifestaciones clínicas incluyen fiebre, quicardia, signos de insuficiencia cardiaca, disminución de ruidos cardiacos, galope, murmullo por regurgitación mitral, frote pericardico, laboratorios son leucocitosis, eosinofilia, incremento de IgM, IgG, CPK en aproximadamente 10 %, el ECG taquicardia, alteración de ST y cambios de la onda T, prolongación QT, el ECOCG puede manifestar alteraciones de la función ventricular, se clasifica en fulminante, agudo, crónico activo, crónico persistente. Sobrevida a 5 años de 50 %. El tratamiento es de soporte, diuréticos, ECA´s, bloqueadores antagonistas de aldosterona, la digoxina puede incrementar la expresión inflamatoria, AINE´s no son utiles, son utiles; esteroides con azatioprine o esteroide con ciclosporina. ENFERMEDAD DE CHAGAS La tripanosomiasis o enfermedad de chagas es mas común en america central y del sur, es causada por tripanosoma cruzi, la lesión cardiaca es mediada por inumunocomplejos. Se disemina via hematogena a varios órganos y sistemas con una intensa reacción inflamatoria, fiebre, sudoración, mialgias y miocarditis. El 5 % son fatales, del 20 al 30 % permanecen asintomáticos en fase latente, los casos crónicos presentan fibrosis de miofibrillas causando cardiomegalia, falla cardiaca, bloqueos y arritmias. El tratamiento es sintomático, marcapasos y agente antiparasitarios- ENFERMEDAD DE LYME: Es causada por la infección con la espiroqueta borrelia burgdorferi introducido por picadura, los síntomas iniciales del padecimiento es bloqueo cardiaco completo, puede verse disfunción ventricular izquierda, la miopsia muestra datos de miocarditis activa. MIOCARDITIS REUMATICA: Se puede observar durante la fiebre reumática aguda seguido de faringitis streptococcica del grupo A, El diagnostico clínico se hace con los criterios de JONES. Mayores; carditis, poliartritis, corea, eritema marginatm, nódulos subcutáneos, y evidencia de infección previa. Los criterios Menores son fiebre artralgias, fiebre reumática, elevada sedimentación eritrocitaria, proteína C-reactiva y prolongación del intervalo PR. El diagnostico es con dos mayores y un mayor y dos menores, del 5 al 10 % desarrollan insuficiencia cardiaca. Se presenta carditis, lesion valvular aortica predominantemente. ECG prolongación de PR y cambios inespecíficos del ST-T. Tx aspirina, corticoides y 1.2 millones U de penicilina G benzatina. MIOCARDITIS NO INFECCIOSAS: La miocarditis hipersensitivas y son causadas por reacción a medicamentos, caracterizada por eosinofilia, infiltración al miocardio de eosinofilos, células gigantes multinucleadas, y leucocitos, los fármacos mas frecuentes son: metildopa, penicilina, tetraciclina y antituberculosos, pueden causar emadamente raras que generan miocariditis, progresiva sin respuesta a tratamiento, mas frecuente en adulto jove, asociado a enfermedades autoinmunes 20 %, se

CURSO ENARM CMN SIGLO XXI TEL: 36246001

Pharmed Solutions Institute

PÁGINA 31

MANUAL DE TRABAJO DEL CURSO ENARM CMN SIGLO XXI observa histiocitos y linfocitos y eosinofilia, es fatal solo ttx transplante. Cardiomiopatía periparto, 1 en 3,000 partos, multiples factores, obesidad, gestacion multiple, preeclapmsia, hipertensión crónica, px con 30 años aproximadamente. La falla cardiaca es variable inicia en el tercer trimestre, ECG con hipertrofia ventricular izquierda, pronostico reservado. Miocardiopatias por enfermedades neuromusculares, distrofias neuromusculares hereditarias, asociadas a cardiomiopatías Becker, Duchenne, Steinert, distrofia miotonica, Friedreich, Barth. CARDIOMIOPATIAS POR ENFERMEDADES ENDOCRINAS: El exceso o disminución de hormonas tiroideas, en la tirotoxicosis se compromete eventualmente la función ventricular izquierda con dilatación pudiendo llegar a falla cardiaca, El feocromocitoma es otra causa con hipertensión, sudoración, palpitaciones e hipotensión ortostatica, puede presentarse miocarditis inducida por catecolaminas, puede llegar a fallar cardiaca o arritmias ventriculares malignas. La acromegalia presenta cardiomiopatía en el 10 al 20 % de los casos por el exceso de hormona de crecimiento que genera hipertrofia de miocitos que se fibrosan, con disfunción diastólica y sistólica finalmente, también degeneración del nodo AV con bloqueo completo. MIOCARDIOPATIAS POR TOXINAS: El abuso crónico de ALCOHOL es un riesgo mayor para desarrollar cardiomiopatía congestiva, 45 % de todas las dilatadas, puede generar insuficiencia cardica, hipertensión y arritmias, el daño es directo, acetilaldehido y metabolitos, asi como deficiencia nutrimentales, estimulación simpatica y cofactores toxicos. La edad promedio de 30 a 55 años, con historia de 10 años de consumo intenso, la fibrilación atrial es la forma mas frecuente de preentacion luego la muerte súbita. COCAINA produce isquemia miocárdica, infarto, espasmo coronario, arritmias cardiacas, muerte súbita, miocarditis y cardiomiopatía dilatada. Mal pronostico, tratamiento sintomático los betabloqueadores puede disminuir espasmos. QUIMIOTERAPICOS: doxirrubicina, ciclofosfamida, generan citotoxicidad, la radiación de mediastino, el trastuzumab es un anticuerpo monoclonal se puede presentar miotoxicidad. La CLOZAPINA antipsicotico de uso crónico. Fenotiacinas, cloroquina, litio, cobalto, hidrocarbonos, interferon alfa IL-2. CARDIOMIOPATIAS ASOCIADAS CON DEFICIENCIAS NUTRIMENTALES: Deficiencia de TIAMINA, resulta en beri beri con síntomas caracteristicos de falla cardiaca, pronostico fatal sin tratamiento, Deficiencia de VITAMINA D, la deficiencia o el exceso aumenta el riesgo, falta de absorsion de SELENIO, la disminución de L-CARNITINA. CARDIOMIOPATIAS TAKO-TSUBO: La miocardiopatía de Takotsubo es un síndrome recientemente descrito atribuido al exceso de catecolaminas, probablemente relacionado con una hiperactividad simpática inducida por un factor estresante de cualquier tipo. Son numerosas las enfermedades neurológicas que se han descrito relacionadas con este síndrome: hemorragia subaracnoidea, ictus, esclerosis múltiple, síndrome de Guillain-Barré, crisis miasténicas y crisis epilépticas. Del mismo modo, son numerosos los casos de síndrome de Takotsubo asociados a enfermedades psiquiátricas, dependencia a opioides, alcoholismo, trastorno maníaco-depresivo, trastorno depresivo, esquizofrenia. También se han relacionado diversos factores emocionales (malas noticias, fiestas sorpresa, discusiones, divorcio, muertes inesperadas...) y físicos (ejercicio, neumotórax, hipoglucemia, ataque de asma, cirugías...). El síndrome de Takotsubo, descrito por primera vez en Japón en 1991, se caracteriza por acinesia o discinesia de la porción apical y media del ventrículo izquierdo que no corresponde con un único territorio vascular en ausencia de enfermedad coronaria, demostrada mediante cateterismo cardíaco. El electrocardiograma puede mostrar elevación del segmento ST o inversión de la onda T. Como criterios de exclusión, entre otros, se encuentran antecedentes recientes de hemorragia intracraneal, feocromocitoma, miocarditis, miocardiopatía hipertrófica, hipertiroidismo, pancreatitis y envenenamiento. Como desencadenantes se han relacionado factores de estrés tanto de tipo emocional como físico. CARDIOMIOPATIA DILATADA: Es una causa frecuente de insuficiencia cardíaca y es el diagnóstico más frecuente en pacientes sometidos a trasplante cardíaco. Desde el punto de vista clínico, la MD se caracteriza por dilatación y disfunción contráctil del ventrículo izquierdo o de ambos ventrículos. La dilatación ventricular es generalmente severa y se acompaña siempre de hipertrofia. La MD puede ser idiopática, genética/familiar, viral y/o inmune, alcohólica/tóxica, o asociada a otras cardiopatías en las cuales el grado de disfunción miocárdica no se explicaría por una determinada sobrecarga hemodinámica o severidad de daño isquémico. Probablemente, el síndrome clínico de la MD representa un final común al que se llega a través de múltiples mecanismos citotóxicos, metabólicos, inmunológicos, infecciosos y familiares. Anatomía patológica: Los estudios post mortem demuestran habitualmente dilatación de las cuatro cámaras cardíacas, en especial los ventrículos, que se acompaña a veces de aumento de grosor de la pared. Las válvulas cardíacas son intrínsecamente normales y es frecuente la presencia de trombos intracavitarios. Es típica la presencia de miocitos hipertróficos y muertos, sustituidos por fibrosis con variable afectación del sistema de conducción. Los componentes de la matriz extracelular están aumentados de forma no selectiva. Aunque la ausencia de células inflamatorias se utiliza como criterio para diferenciar la MD de la miocarditis, en algunos casos de MD se detectan células T inflamatorias y células endoteliales activadas, sugiriendo la posible contribución de un proceso inflamatorio crónico en la patogenia de esta enfermedad. La etiología y los mecanismos patogénicos son desconocidos en alrededor de la mitad de los casos de MD. Para explicar el daño miocárdico crónico y progresivo se han propuesto tres principales etiopatogenias: a) infección viral crónica del miocardio que produce daño celular; b) alteración de los mecanismos inmunes que conduce probablemente a una enfermedad autoinmune, y c) factores genéticos que serían directa o indirectamente responsables de la enfermedad. DIAGNÓSTICO: El estudio del paciente con MD debe enfocarse no sólo al establecimiento del diagnóstico sindrómico, sino hacia la identificación, por los métodos de diagnóstico habituales, de posibles causas tratables o reversibles de la enfermedad. La historia clínica debe incluir preguntas relativas al posible consumo de alcohol y cocaína, medicamentos, hábitos nutricionales, estancias en zonas endémicas para infecciones, relación con animales, embarazos recientes, transfusiones sanguíneas, historia familiar de MD, somnolencia diurna y exposición profesional a tóxicos. Asimismo, la idea de que la MD idiopática es con frecuencia un problema genético hereditario debe ser tenida en cuenta en la práctica clínica, estudiando sistemáticamente a los familiares de primer grado del paciente. En la mayoría de los pacientes la MD se manifiesta clínicamente entre los 20 y 60 años de edad, aunque la enfermedad puede afectar también a niños y ancianos. Los síntomas más frecuentes son los de insuficiencia cardíaca (disnea de esfuerzo progresiva, ortopnea, disnea paroxística nocturna y edemas periféricos). Otras formas de presentación son la detección accidental de cardiomegalia asintomática y los síntomas relacionados con arritmias, alteraciones de conducción, complicaciones tromboembólicas o muerte súbita. La exploración física suele revelar diferentes grados de cardiomegalia y signos de insuficiencia cardíaca. La presencia de un galope presistólico (cuarto ruido) puede preceder a la aparición de insuficiencia cardíaca. El ritmo de galope ventricular (tercer ruido) es la regla en los casos con descompensación de la IC. Es frecuente la presencia de soplos sistólicos de insuficiencia mitral o, menos frecuentemente, tricuspídea. Las dos causas más frecuentes de muerte en pacientes con MD e insuficiencia cardíaca son la muerte súbita y el fallo de bomba progresivo. MIOCARDIOPATÍA HIPERTRÓFICA: La miocardiopatía hipertrófica (MH) es una enfermedad con una importante heterogeneidad en cuanto a su base genética, manifestaciones clínicas y pronóstico 93-96. Se caracteriza, fundamentalmente, por la presencia de una hipertrofia ventricular de causa desconocida (generalmente de predominio septal) y por una excelente función sistólica 93-96. Los gradientes intraventriculares

CURSO ENARM CMN SIGLO XXI TEL: 36246001

Pharmed Solutions Institute

PÁGINA 32

MANUAL DE TRABAJO DEL CURSO ENARM CMN SIGLO XXI dinámicos constituyen un rasgo primordial de esta entidad, pero sabemos que sólo se detectan en una parte de los pacientes y buena parte de las manifestaciones son consecuencia de las alteraciones diastólicas. El diagnóstico clínico de los pacientes con MH sigue basándose en la demostración de una hipertrofia ventricular en ausencia de factores cardíacos o sistémicos que la justifiquen. Los síntomas más frecuentes -disnea, angina, palpitaciones-, se manifiestan en más de la mitad de los pacientes, pero son comunes en otras patologías cardiovasculares. Sin embargo, algunos datos clínicos como, por ejemplo, disnea en presencia de una buena función sistólica o angina con coronarias angiográficamente normales pueden orientar hacia el diagnóstico. Mucho más específica, sin embargo, sería la aparición de síncope o presíncope en individuos jóvenes previamente asintomáticos. Un episodio de muerte súbita recuperada en un niño, joven, o adulto, sin duda obliga a descartar esta patología. Tras la anamnesis, la exploración física dirigida puede ser diagnóstica cuando se detecta una semiología característica de gradiente intraventricular dinámico, pero en caso contrario, es poco reveladora. Con frecuencia, un electrocardiograma manifiestamente patológico (ondas Q, hipertrofia ventricular, patrones de preexcitación o severas alteraciones de la repolarización), muchas veces en un individuo asintomático, es la primera clave diagnóstica. CASO CLINICO Masculino de 38 años de edad, sin antecedentes personales ni familiares de interés, sin hábitos tóxicos, ni tratamiento habitual. Refería un cuadro catarral de 7 días de evolución tratado con amoxicilina-clavulánico. Un día antes del ingreso presentó disnea progresiva hasta la ortopnea, presentaba una tensión arterial indetectable y una expectoración espumosa, abundante y hemoptóica, junto a signos de insuficiencia respiratoria severa (taquipnea, cianosis, crepitantes pulmonares generalizados y desaturación del 70%). El ECG mostraba una taquicardia sinusal a 180 lpm, con QRS ancho, disociación auriculoventricular y morfología de bloqueo de rama izquierda, criterios de taquicardia ventricular (TV) y, la radiografía torácica, unos infiltrados algodonosos bilaterales. Analíticamente, salvo poliglobulia, acidosis metabólica e intensa hipoxemia, no presentaba otros hallazgos de interés incluyendo los enzimas cardiacos. PREGUNTA Cual es la forma mas frecuente que debute esta patologia MCG? RESPUESTA a.- Taquicardia ventricular. b.- Insuficiencia cardiaca. c.- Infarto al miocardio. d.- Bloqueo auriculoventricular completo el 5%. CASO CLINICO Paciente mujer de 18 años de edad, natural y procedente de Lima. Alergia a AINES. Acudió por presentar en forma brusca, malestar general, nauseas, vómitos y deposiciones líquidas sin moco, ni sangre (4 horas de inicio). En el examen de ingreso se constató que sus funciones vitales estaban dentro de límites normales. Se le diagnosticó gastroenterocolitis aguda y deshidratación leve-moderada; en ese sentido, y de acuerdo con los diagnósticos planteados, en el manejo inicial se administró hidratación endovenosa con cloruro de sodio al 9%, pargeverina clorhidrato 10mg y dimenhidrinato 50mg. Una hora después de la admisión, la paciente desarrolló un cuadro de hipotensión (presión arterial: 70/50 mmHg) y palidez marcada, recibió tratamiento de soporte, pero ante el deterioro clínico paulatino la paciente fue conducida a la unidad de cuidados intensivos. PPREGUNTA Cual es la imprension diagnostica inicial del caso? RESPUESTA a.- Estado de choque séptico. b.- Estado de choque anafiláctico. c.- Estado de choque cardiogenico. d.- Estado de choque hipovolemico. CASO CLINICO Un varón de 42 años fue hospitalizado por fiebre y síntomas de dolor torácico. Antecedente de (VIH), diagnosticado 15 años antes del ingreso. El recuento de células T CD4+ al ingreso era de 874 cél./μl y la carga viral era indetectable (< 50 copias/ml). En tratamiento con tenofovir + didanosina + atazanavir + ritonavir, con una carga viral indetectable en los últimos 6 años. El cuadro se inició con fiebre, diarrea, mal estado general y mialgia de 1 semana de duración. En los 2 días previos al ingreso sufrió un dolor torácico punzante intenso y progresivo, en reposo, con irradiación a la espalda, que se agravaba con el decúbito y la inspiración profunda. La intensidad del dolor torácico disminuyó tras la administración de analgésicos intravenosos, pero no con nitroglicerina sublingual. En la exploración física, las constantes vitales eran normales, excepto la temperatura corporal, que era de 38 °C. No presentaba signos de congestión sistémica o pulmonar ni había signos de bajo gasto ni dificultad respiratoria. Los ruidos cardiacos eran normales. Los datos de laboratorio revelaron leucocitosis con linfocitosis, proteína C reactiva en 19 mg/ml y un pico de troponina I cardiaca que alcanzó 30 ng/ ml. La radiografía de tórax fue anodina. El ECG mostró un ritmo sinusal normal, con una elevación del segmento ST de 1 mm en las derivaciones DI, aVL y V4V6. PREGUNTA Cual es el agente mas frecuente de esta patologia? RESPUESTA a.- Parvovirus B19.

CURSO ENARM CMN SIGLO XXI TEL: 36246001

Pharmed Solutions Institute

PÁGINA 33

MANUAL DE TRABAJO DEL CURSO ENARM CMN SIGLO XXI b.- H1N1. c.- VIH. d.- Cocxackie B6. CASO CLINICO Mujer de 43 años con historia de epilepsia focal del lóbulo temporal izquierdo criptogénica farmacorresistente, trastorno límite de la personalidad, etilismo crónico, infección por el virus de la hepatitis C. Fue ingresada en Urgencias por presentar una crisis focal motora secundariamente generalizada. Posteriormente, durante las primeras 24 horas de observación, sufrió cuatro crisis tonicoclónicas generalizadas más, la frecuencia cardíaca durante la crisis descendió hasta 35 latidos por minuto. Mostró también disnea y se objetivó en la auscultación cardiopulmonar crepitantes bibasales, bradicardia extrema. PREGUNTA Considerando las manifestaciones clínicas cual de las siguientes condiciones es la mas probable que presenta el paciente? RESPUESTA a.- Choque Distributivo. b.- Choque Cardiogenico. c.- Choque Hipovolemico. d.- Choque Oculto. CASO CLINICO Mujer de 50 años, procedente de zona rural. Presentó un cuadro clínico de 15 días de evolución consistente en fiebre, deposiciones diarréicas y, posteriormente, disnea, el cual la obligó a consultar en varias ocasiones. La sintomatología, especialmente la disnea, empeoró, por lo cual consultó nuevamente, encontrándose en el examen físico signos claros de insuficiencia cardiaca. En la radiografía de tórax se 7evidenció cardiomegalia y derrame pleural, y el ecocardiograma reportó taponamiento cardiaco. PREGUNTA Cual es su conducta a seguir mas adecuada? RESPUESTA a.- Pericardiocentesis. b.- Pericardiotomia. c.- Diuretico, corticosteroide, b-bloqueador. d.- Antiparasitario, pericardiocentesis. CASO CLINICO Hombre de 26 años, procedente de zona rural. Consultó por cuadro de cinco días de fiebre, escalofríos, dolores musculares, cefalea, artralgias, astenia y adinamia. Se realizó estudio para hemoparásitos, encontrándose ocasionales tripomastigotes. El ecocardiograma mostró leve derrame pericárdico y leve dilatación ventricular izquierda. Además, se encontró franca leucopenia (3.200 leucocitos/µl) y plaquetopenia (69.000 plaquetas/µl). PREGUNTA Cual es la conducta farmacológica especifica para el caso?. RESPUESTA a.- Metronidazol. b.- Albendazol. c.- Benzonidazol. d.- Prazicuantel. CASO CLINICO Varón de 36 años, labrador, sin antecedentes de interés salvo picaduras frecuentes de garrapatas. Ingresa en nuestro hospital por episodio sincopal y bloqueo AV avanzado. En las horas siguientes al ingreso se continúa apreciando un bloqueo AV completo, con pausas de hasta 6 segundos de duración, implantándose un marcapasos provisional. Un mes previo al ingreso refiere un eritema en región glútea (cuyas características no se pudieron precisar), fatiga y mareos. Dos semanas más tarde, comienza con artralgias en codos y rodillas, astenia y disnea. La exploración física fue anodina salvo por febrícula vespertina. La radiografía de tórax y la analítica básica fueron normales. PREGUNTA Considerando el agente causal mas probable del caso por sus antecedentes, cual es la pauta antibiótica mas adecuada? RESPUESTA a.- Ceftriaxona. b.- Doxiciclina. c.- Ciprofloxacina. d.- Gentamicina.

CURSO ENARM CMN SIGLO XXI TEL: 36246001

Pharmed Solutions Institute

PÁGINA 34

MANUAL DE TRABAJO DEL CURSO ENARM CMN SIGLO XXI CASO CLINICO Una mujer de 41 años con antecedente de HTA ligera no tratada acude a urgencias de su hospital presentando dolor torácico de características anginosas y elevación del segmento ST en cara lateral. Desde 3 años antes sufría un trastorno de ansiedad que había requerido ingreso en una ocasión. En las últimas semanas la paciente sufría un gran estrés laboral. Presión arterial de 110/70 mmHg; tras nitroglicerina sublingual, la clínica cedió y se normalizó el ECG. A las 12 h vuelve a tener dolor con elevación del segmento ST, cateterismo cardiaco normal, pero la ventriculografía muestra una disfunción medioventricular severa, con hipercontractilidad de los segmentos basales y apicales. Se inició tratamiento con BB e IECA, manteniendo la anticoagulación y la aspirina. La paciente permaneció asintomática, con creatincinasa normal y troponina I de 1,4. A los 5 días del ingreso se realizó una RM cardiaca que todavía mostró una hipocinesia medioventricular, sin realce tardío tras la administración de gadolinio. Al ingreso en nuestro hospital se habían solicitado catecolaminas en sangre y en orina para una mejor caracterización del cuadro. Éstas mostraron una elevación supranormal (noradrenalina y adrenalina en sangre > 5.000 y 190 pg/ml; noradrenalina y metanefrina en orina de 24 h, 582 y 5.386 μ g/24 h). PREGUNTA Cual es la conducta mas apropiada a seguir? RESPUESTA a.- Resonancia magnetica abdominal. b.- Tomografia axial computada. c.- USG abdominal. d.- Tomografia helicoidal. CASO CLINICO Mujer de 30 años sin antecedentes de cardiopatía que ingresa en nuestro centro debido a un cuadro de insuficiencia cardiaca congestiva un mes y medio después de un parto sin complicaciones. Se realizó un ecocardiograma en el que se objetivó dilatación moderada del ventrículo izquierdo (diámetro telediastólico, 64 mm; diámetro telesistólico, 46 mm) junto con hipocinesia generalizada y fracción de eyección del ventrículo izquierdo del 32%; la aurícula izquierda estaba ligeramente dilatada y las cavidades derechas, en el límite superior de la normalidad; se apreciaron regurgitaciones mitral y tricuspídea moderadas-severas sobre válvulas estructuralmente normales, con estimación de presión arterial pulmonar sistólica de 46 mmHg. Tras la estabilización, se realizó una cardiorresonancia magnética, en la que se confirmó que el ventrículo izquierdo estaba moderadamente dilatado, con fracción de eyección del 37%, así como hipocinesia generalizada del ventrículo derecho con función sistólica general severamente deprimida (fracción de eyección, 25%); en el estudio de retención miocárdica tardía de contraste con gadolinio, se observó un depósito mesocárdico lineal a nivel septal extenso. En una coronariografía no se objetivaron lesiones coronarias. Con el diagnóstico de miocardiopatía periparto. PREGUNTA Cuales son los siguiente criterios no es útil para establecer el diagnostico de cardiomiopatía periparto? RESPUESTA a.- Desarrollo de insuficiencia cardiaca en el último mes de embarazo. b.- Desarrollo en los 2 meses siguientes al parto. c.- Ausencia de causa identificable para el desarrollo de insuficiencia cardiaca. D.- Ausencia de enfermedad cardiaca primaria en el último mes de embarazo. ENDOCARDITIS La endocarditis infecciosa es una enfermedad causada por un agente microbiano que afecta la capa endotelial de estructuras intracardiacas que invaliablemente es fatal sin tratamiento. La infección más frecuente reside en una o mas válvulas que envuelve el endocardio mural, miocardio y pericardio. Los dispositivos intracardiacos o endovascular es una fuente de infección. La mortalidad es del 25 % dentro de los 6 meses. EPIDEMIOLOGIA: La causa inicial de EI era una complicación de fiebre reumática o de origen dental, actualmente la degeneración de válvulas cardiacas o dispositivos se han incrementado. Los pacientes con DM, VIH, IRC esta en mayor riesgo de EI con riesgo nosocomial. PATOGENESIS: La EI es una bacteremia persistente y continua endocardica o endovascular. El agente patógeno lesiona estructuras cardiacas y presenta capacidad de adhesión, y evitar la actividad inmunológica del huésped, generandoce vegetaciones en válvulas. PATOGENIA: 80 % son estreptococos y estafilococos, en el mundo el Staphylococcus aureus es el más común. ENDOCARDITIS DE VALVULAS NATIVAS: El estreptococo viridans o el estreptococo alfa-hemolitico es el agente mas común adquirido en la comunidad. Es un agente nativo en la orofaringe que fácilmente entra a la circulación via daño dental o gingival. El estreptococo viridan (S. bovis, S. mutans, S. mitor) nativos en sistema gastrointestinal o debido a patologia del sistema GI. El grupo D de estrptococos en particular el enterococos spp. Causan hasta el 18 % de los casos de EI. E. fecalis se presenta hasta el 80 %, su ingreso a circulación es por manipulación con sonda Foley, colonoscopia, etc. El estreptococo del grupo A, es una causa rara el S. pneumonie causa 10 % de EI es agudo, fulminante, asociado a daño severo valvular con extensión perivalvular, complicaciones embolicas, pericarditis, meningitis con mortalidad del 25 al 50 %. El grupo B (S. agalactiae) se presenta en neonatos, parturientas, en pie diabético, carcinoma, falla hepática, alcoholismo, uso de drogas inyectables. Estafilococo aureus causa del 80 a 90 % de IE, es la causa aguda mas frecuente, la nasofaringe es el sitio mas común de conolizacion, los factores de riesgo son diálisis, diabetes, quemaduras, VIH, usuarios

CURSO ENARM CMN SIGLO XXI TEL: 36246001

Pharmed Solutions Institute

PÁGINA 35

MANUAL DE TRABAJO DEL CURSO ENARM CMN SIGLO XXI de drogas, en condiciones dermatológicas crónicas. La IE fulminante es causada por El S. aureus con falla carrdiaca, afeccion perivalvular, trastornos de conducción, infección metastasica con mortalidad del 25 al 30 %, ocacionalmente es debido a estafilococo coagulaza positivo nativo de la piel. El S. epidermidis es una causa importante en dispositivos ol prótesis valvulares. La IE debido a bacilos gran negativos, es infrecuente, en patologia hepática o prótesis cardiaca, es difícil cultivarlo, Coxiella burneti agente causante de la fiebre Q, la Bartonella es otra causa. La IE la causa fungica presenta alta mortalidad 20 % en pacientes inmunocomprometidos, con dispositivos, candida y aspergelius son los mas comunes, generan vegetaciones tratamiento con anfotericina. Puede presentarse por causas autoinmune. En caso de IE se presenta entre los 2 meses y el primer año del tratamiento quirúrgico, las causas nosocomiales son estafilococo coagulasa negativo (s. epidemidis). CASO CLINICO Mujer de 46 años, afectada de nefropatía lúpica IV, en programa de diaslisis peritoneal. Desde entonces mantiene marcadores positivos. Tuvo un brote cutáneo-articular, por lo que recibía con micofenolato sódico a dosis de 180 mg y prednisona a dosis de 5 mg diarios. Ingresa por disnea y malestar general progresivo, de 15 días de evolución. Dolor torácico en el hemitórax izquierdo que aumenta con la inspiración profunda y que mejora relativamente en anteversión. No refiere síndrome febril, ni otra clínica acompañante. EF soplo diastólico en el foco aórtico, irradiado a las carótidas, con roce pericárdico importante, sin signos de fallo cardíaco. El resto de la exploración física fue anodina. En la analítica destacan: leucocitos * 21,3 K/µl (4,4-11,3), cayados 3%, neutrófilos * 92,0% (50-70), linfocitos * 3,0% (25-40), proteína C reactiva (PCR) * 17,73 mg/dl (0,1-0,5), procalcitonina * 4,84 ng/ml (65 años, alcoholicos, EPOC); Haemophilus influenzae 8-10% (EPOC, fumadores); Staphylococcus aureus 3-5% (abseso pulmonar), otros Klebsiella, Escherichia, Moraxella catharralis, Mycoplasma pneumoniae, legionella pneumophila, pesudomona auregunosa, anaerobios (aspiración, obstrucción de vía aérea). SALUD PUBLICA: Es una causa de frecuente de morbi-mortalidad en la población general. Incidencia de 2-10 casos/1000 habitantes/año. 20-35% requieren ingreso hospitalario. Mortalidad del 5% de hospitalizados y de un 35% ingresados a la UCI. La mortalidad oscila entre un máximo del 61% para las NAC debidas a Pseudomonas y un 35% para las producidas por enterobacterias, Staphylococcus aureus y las de etiología mixta. Las tasas de mortalidad por NAC la sitúan en el quinto lugar como causa más frecuente de muerte en los países industrializados, tras las enfermedades cardiovasculares, neoplásicas y cerebrovasculares y la enfermedad pulmonar obstructiva crónica (EPOC). 60-70% de todos los casos de NAC son originados por Streptococcus pneumoniae. En los últimos

CURSO ENARM CMN SIGLO XXI TEL: 36246001

Pharmed Solutions Institute

PÁGINA 40

MANUAL DE TRABAJO DEL CURSO ENARM CMN SIGLO XXI años se ha informado la aparición de Staphylococcus aureus resistente a la meticilina como causa de infecciones extrahospitalarias graves. PATOGENIA: Vías de entrada: Aspiración de organismos que colonizan la orofaringe (mecanismo más común, sobre todo en ancianos, alcoholicos, residentes de silos, DM; mas propensos a infecciones por gram -, provenientes del estómago, comida contaminada), inhalación de aerosoles infecciosos, diseminación hematógena (por el embolismo séptico a partir de focos de infección distantes, adictos a drogas parenterales, endocarditis bacteriana, colonización de catéteres intravenosos, S. aureus. patógeno más frecuente), inoculación directa (es muy rara, el microorganismo traspasa las barreras). Hay factores que inhiben la actividad mucocoiliar como: El humo del tabaco, el aire frío, fármacos (anestésicos), óxido de azufre, óxido de nitrógeno, fibrosis quística. DIAGNOSTICO: Cuadro clínico: Tos con o sin esputo, escalofríos, fatiga, disnea y dolor torácico pleurítico, hemoptisis, mialgias, en caso de legionella; puede existir sintomatología gastrointestinal. Exploración física: taquipnea, matidez a la percusión estertores y/o crepitancias, frémitos y egofonía. Rx de tórax; neumonía típica (causada usualmente por Streptococcus pneumoniae, mas en niños y ancianos): consolidación lobar y en neumonía atípica (influenzae, mycoplasma, legionella, no detectables en tinción de gram -, de ahí su nombre, más en jóvenes) infiltrados difusos: de no observar nada repetir Rx a las 24-48 hrs. Los estudios especiales dirigidos a un agente como hemocultivo, , cultivos de esputo, tinción gram, Ag urinario para legionellla y Streptococcus solo cuando: neumonía extensa que ingresa a UCI, falta de respuesta al tx., infiltrados cavitarios, leucopenia, alcoholismo severo. ESCALAS DE GRAVEDAD: Útiles para no sobreestimar la intensidad de los cuadros neumónicos y menor tasa de hospitalización. Pneumonia Severity Index Características del paciente Características demográficas Sexo masculino Sexo femenino Asilo Comorbilidades Enf. Neoplásica Enf. Hepática Enf. Cardiaca Enf. cerebrovascular Insuficiencia renal Exploración física Alteración del estado mental FR > 30rpm P.sistólica 125 lpm Estudios de laboratorio y Rx pH arterial 10.7 mmol/l Na < 130 mEq/l Glucosa >250 mg/dl Hto. 30rpm o necesidad de apoyo ventilatorio Indice de PaO2/FiO2 20 mg/dl) Cualquier Leucopenia 65 años, 6-49 años con factores de riesgo.; revacunación anual. CASO CLINICO Hombre de 68 años de edad, con historia de consumo de licor cada fin de semana hasta la embriaguez, cuatro días de tos con expectoración purulenta, disnea de moderados esfuerzos, fiebre de 38,5°C, dolor tipo pleurítico en el hemitórax derecho y automedicación con una dosis de dexametasona de 4 mg intramuscular dos días antes de la admisión. Ingresó febril, taquicárdico, taquipneico, hipo-xémico y en la radiografía de tórax presentaba signos de derrame pleural y consolidación en la base derecha. La evaluación de laboratorio muestra leucocitosis de 14.500 por mm3, neutrofilia de 96%, hiperglucemia de 638 mg/dl, hiperazoemia (nitrógeno ureico en sangre de 27,45 mg/dl). El paciente fue hospitalizado con diagnóstico de neumonía grave extrahospitalaria IIIA, según las guías nacionales de manejo de esta enfermedad. PREGUNTA ¿Cuál es el agente etiológico más probable en este paciente? RESPUESTA a.- Mycoplasma pneumoniae b.- Legionella pneumoniae c.- Pseudomona

CURSO ENARM CMN SIGLO XXI TEL: 36246001

Pharmed Solutions Institute

PÁGINA 41

MANUAL DE TRABAJO DEL CURSO ENARM CMN SIGLO XXI d.- Strepococcus pneumoniae PREGUNTA ¿Qué criterios de neumonía grave tiene este paciente, para ser tratado intrahospitalariamente? RESPUESTA a.- leucocitosis, derrame pleural, tx con dexametasona b.- Hiperazoemia, hiperglucemia, taquipnea c.- Hiperazoemia, leucocitosis, consolidación en base derecha d.- Hiperglucemia, leucocitosis, tx. con dexametasona CASO CLINICO Hombre de 54 años de edad, con historia de consumo de licor cada fin de semana hasta la embriaguez, cuatro días de tos con expectoración purulenta, disnea de moderados esfuerzos, fiebre de 38,5°C, dolor tipo pleurítico en el hemitórax derecho y automedicación con una dosis de dexametasona de 4 mg intramuscular dos días antes de la admisión. Ingresó febril, taquicárdico, taquipneico, hipoxémico y en la radiografía de tórax presentaba signos de derrame pleural y consolidación en la base derecha. Leucocitosis de 14.500 por mm3, neutrofilia de 96%, hiperglucemia de 638 mg/dl, hiperazoemia (nitrógeno ureico en sangre de 27,45 mg/dl). El paciente progresó rápidamente a falla respiratoria hipoxémica, con choque séptico y, posteriormente, disfunción orgánica múltiple. PREGUNTA Considerando el cuadro clínico, a que grupo pertenece para considerar tratamiento antibiótico? RESPUESTA a.- Grupo I b.- Grupo II. c.- Grupo III d.- Grupo IV. PREGUNTA Considerando el Pneumonia Severity Index, que puntaje presenta el caso? RESPUESTA a.- 130 PREGUNTA Considerando la escala de riesgo y mortalidad de pneumonia severity index que riesgo de motralidad presenta el caso? RESPUESTA a.- 0.6 b.- 2.8 c.- 8.2 d.- 29.2 NEUMONIA NOSOCOMIAL: CIENCIAS BASICAS: Se desarrolla después de una estancia intrahospitalaria de 48hrs, algunos requerirán intubarse y pueden desarrollar neumonía asociada al ventilador. SALUD PUBLICA: La neumonía es la 2da. Infección nosocomial más frecuente, con una mortalidad asociada de 27-50%. Mas en niños, adultos >65años o con comorbilidades graves asociadas, o sometidas a cirugía toracoabdominal. Debidas generalmente a la aspiración de secreciones orofaríngeas o de tracto respiratorio superior, contaminación de equipo de terapia respiratoria. Las infecciones generalmente son polimicrobianas, con alto contenido de bacilos gram- ; pseudomona aeruginosa (más frecuente) escherichia coli, klebsiella pneumoniae y Acinetobacter (55-85%), originan las neumonías nosocomiales, los gram + , son responsables del 20-30%; rara vez por hongos y virus (inmunodeprimidos). Recientemente ha emergido infecciones por Staphylococcus aureus, particularmente resistente a meticilina, más frecuente en DM, y pacientes de UCI. DIAGNÓSTICO: La sospecha dx. surge cuando el paciente presenta infiltrados radiográficos nuevos y/o progresivos, en conjunto con dos o más hallazgos clínicos sugerentes de infección (fiebre de recién aparición >38°C, leucocitosis o leucopenia, esputo purulento). Rx de tórax; desde infiltrados, consolidación, derrames, cavitaciones. Para dx. etológico: cultivos, hemocultivos, identificación de legionella requiere inmunofluorescencia. Si hay derrame pleural de gran tamaño y/o datos de sepsis, realizar toracocentesis dx. para descartar empiema o derrame paraneumonico. TRATAMIENTO: Inicio empírico; 1. Esquema de espectro limitado (menos de 5 días hospitalizado y sin factores de riesgo para bacterias multiresistente) Ceftriaxona, levofloxacino, ampi/sulbactam, ertapenem. 2. Esquema de espectro amplio (>5 dias hospitalizado, factores de riesgo para multiresistentes). Esquema 1: a.- cefalosporina o carbapenem antiseudomonica (cefepime, ceftazidima) (imipenem, meropenem) o un b lactamico + un inhibidor de b- lactamasas (piperacilina/tazobactam). b.fluroquinolona antiseudomonica (cipro o levofloxacino). Esquema 2: a.- aminoglucocido. b.- linezolid o vancomicina. La mejoría clínica se observa dentro de 48-72 hrs, posteriores al inicio del antibiótico, por esta razón la terapia antibiotica, no debera de ser modificada

CURSO ENARM CMN SIGLO XXI TEL: 36246001

Pharmed Solutions Institute

PÁGINA 42

MANUAL DE TRABAJO DEL CURSO ENARM CMN SIGLO XXI hasta después de 72 hrs. L a duración deberá individualizarse, para la neumonía por bacterias gram - usualmente es de 14-21 días. PREVENCION: Control estricto de infecciones, desinfección de manos con soluciones etílicas, vigilancia microbiológica, monitoreo y retiro temprano de dispositivos invasivos y evitar prácticas erróneas en prescripción de antibióticos, métodos de ventilación no invasivos. CASOS CLINICOS Varón de 34 años, trabajador en la hostelería, fumador y bebedor importante (con dependencia alcohólica). Fue traído a urgencias por un traumatismo craneoencefálico con hematoma subdural. El paciente ingresó en la unidad de cuidados intensivos (UCI) intubado y conectado a ventilación mecánica, y recibió tratamiento antibiótico empírico con amoxicilina-ácido clavulánico. La cifra de leucocitos en sangre era de 12,05 × 103/l. En el primer día del ingreso se realizó un broncoaspirado (BAS). En el cultivo creció una flora mixta respiratoria. A los 8 días del ingreso presentó fiebre y secreciones purulentas espesas, y en la radiografía de tórax se objetivó una neumonía en el lóbulo inferior izquierdo complicada con atelectasias bibasales. La concentración de leucocitos en sangre era en ese momento de 4,10 × 103/l. PREGUNTA Cual es el agente etiológico mas probable para este caso? RESPUESTA a.- Pseudomona aeruginosa. b.- Escherichia coli. c.- klebsiella pneumoniae. d.- Acinetobacter. PREGUNTA Cual es el tratamiento de primera intensión en este caso? RESPUESTA a.- Ceftriaxona, levofloxacino. b.- Cefepime, ceftazidima. c.- Imipenem, meropenem. d.- Piperacilina/tazobactam NEUMOCONIOSIS CIENCIAS BASICAS: Definición: Enfermedad producida por acumulación de polvos minerales secundaria a la inhalación crónica de los mismos. Los materiales mas implicados son: asbesto, sílice, carbón, berilio, hierro, el daño tisular que provocan depende de; tamaño, tiempo e intensidad de exposición, estado inmunológico, relación inhalación/ aclaramiento ciliar pulmonar. ASBESTOSIS: Hay 3 tipos de asbesto: amosita, asbesto azul (cocidolite) y asbesto blanco (crisolito, el 90%del consumo de asbesto es bajo esta forma, es mas soluble y tiende a fragmentarse). Las formas en las que se inhalan las fibras de asbesto es en anfibole y serpentinas. PATOGENIA: Exposición directa, cuando se trabaja con materiales o en lugares donde hay asbestos. Exposición indirecta, cuando se vive o trabajo cercano a un lugar de concentración de asbesto. Lo podemos encontrar en materiales para pulir piedra preciosa, afiladores de piedra, resistencias de algunos electrodomésticos y algunos plásticos. La enfermedad se puede presentar como: placas pleurales benignas 40%, enf., maligna 20% (mesotelioma maligno), asbestosis como tl 40%. Una vez inhalada las partículas, llegan a los espacios alveolares y son fagocitados por lo macrófagos, si son menor a 3 µm, si son mayores son fagocitadas incompletamente ( cuerpos de asbesto)e, ya fagocitadas son trasportadas a los ganglios y después a las pleuras, donde se depositan para formar las placas pleurales, los cuerpos de asbesto se acumulan de manera progresiva en espacios aéreos e intersticio, las células afectadas liberan citocinas, fibronectina y colágena, causando así migración de otras células de defensa y favoreciendo proliferación de fibroblastos, provocando zonas de regeneración parenquimatosa, provocando la asbestosis, es decir la fibrosis pulmonar afecta mas zonas inferiores y regiones subpleurales. DIAGNÓSTICO: Relación clara entre exposición de asbesto y fibrosis pulmonar. El consumo de tabaco aumenta el riesgo de asbestosis. Cuadro clínico: lo primero es disnea, la cual se va haciendo progresiva, después tos seca pertinaz, pueden auscultarse estertores crepitantes basales, a veces hipocratismo. Rx.: opacidades irregulares que al principio de la enfermedad son de localización periférica y basal, hiperclaridad alrededor de silueta cardiaca. TAC: se puede ver fibrosis, como vidrio despulido, hasta lesiones fibroticas con engrosamiento de los septos interlobares, engrosamiento de pleura viscerales. El patrón funcional observado en pacientes con asbestosis es restrictivo. Se puede realizar lavado bronquioalveolar. SILICOSIS: Provocada por inhalación de cristales de sílice, en la mayoría en sus formas de cuarzo, las actividades de riesgo son la construcción, la fundición, demolición o reparación de estructuras de concreto, taladrar piedras. PATOGENIA: Hay silicosis aguda, donde se observa proteinosis y la crónica la mas frecuente, donde se observan nódulos con centro fibrotico, rodeado de parénquima con cúmulos de sílice, que ha sido fagocitado por macrófagos. En la forma complicada llamada fibrosis masiva progresiva, se observan nódulos confluentes, que se localizan principalmente en lóbulos superiores, que se relacionan con una pobre calidad de vida. La silicosis se relaciona con una mayor susceptibilidad a Tb, artritis reumatoide, esclerodermia, lupus y enf., renal progresiva. DIAGNÓSTICO: Cuadro clínico: síntoma principal disnea, la cual es progresiva, raros otros síntomas. Rx: pequeñas opacidades redondeadas, con distribución casi siempre bilateral, de localización en lóbulos superiores, puede haber crecimiento de adenopatías hiliares, las cuales pueden calcificarse en una forma característica conocida como "cascara de huevo". Funcionalmente la silicosis se comporta con limitación al flujo aéreo y con reducción de la capacidad de difusión. ANTRACOSIS: Provocada por inhalación crónica de carbón masiva, la forma mas común es el antracite. Existen 2 formas principales: la mácula o nódulo antracotico y fibrosis pulmonar masiva. PATOGENIA: En la antracosis hay mayor susceptibilidad a presentar artritis reumatoide, y cuando se presentan ambas condiciones, se conoce como Sx., de Caplan, en el cual debe haber un nódulo reumatoideo, que en algunas condiciones puede cavitarse debido a la necrosis central o bien calcificarse. DIAGNÓSTICO: La macula o mancha de

CURSO ENARM CMN SIGLO XXI TEL: 36246001

Pharmed Solutions Institute

PÁGINA 43

MANUAL DE TRABAJO DEL CURSO ENARM CMN SIGLO XXI carbón suele ser una lesión de 5cm que puede observarse principalmente en regiones apicales, en ocasiones rodeadas de parénquima que muestra enfisema centroacinar. Existen tantas neumoconiosis como diversidad de polvos minerales y son una causa importante de morbimortalidad a nivel mundial. CASOS CLINICOS 70 años, minero de minas carbón jubilado, fumador de 30 cigarrillos/día hasta hace 10 años, bebedor de ¼ l de vino al día y 2 cervezas. HTA en tratamiento con diuréticos y dieta hiposalina. Hipercolesterolemia en tratamiento con dieta. Desde hace 25 años presenta tos y expectoración crónica de predominio matutino. Desde hace 15 años se asocia disnea de esfuerzo progresiva con aumento de la disnea en relación con las infecciones respiratorias. Presenta con frecuencia expectoración de color negro. Paciente que acude al servicio de urgencias por un cuadro de aumento de su tos habitual, aumento de su expectoración habitual, siendo actualmente mucopurulenta y aumento de la disnea que se hizo de reposo. Desde hace una semana nota que se le hinchan los tobillos. Paciente consciente, orientado. Cianótico. Taquipneico (30 respiraciones por minuto). Hepatomegalia blanda y no dolorosa de 2 cm. Ingurgitación yugular +++. Edemas maleolares ++++. Tonos cardiacos rítmicos a 130 por minuto. Roncus y sibilantes diseminados por ambos campos pulmonares.TA: 160/100. Hb 16, Hto 49, leucocitos 13500 con aumento de neutrófilos en la formula leucocitaria. P02 52 PC02 40, PH 7.35. RX de torax: Signos de atrapamientoretroesternal y retrocardiaco, engrosamientos peribronquiales. 3 imágenes de aumento de densidad nodulares de 3-7 cmde diámetro en Lóbulo superior derecho y 2 en Lóbulo superior izquierdo. PREGUNTA Se realizaron pruebas funcionales respiratorias con los siguientes resultados FVC 60%, FEV1 30%, cual es la FEV1-FVC? RESPUESTA a.- 40 b.- 50 c.- 60 d.- 30 HIPERTENSION PULMONAR PRIMARIA (HAP) CIENCIAS BASICAS: Definición: Entidad que afecta a la circulación CAUSAS DE HIPERTENSION PULMONAR pulmonar a nivel de las pequeñas arterias y arteriolar, caracterizada por Hipertensión arterial pulmonar: Hipertensión pulmonar primaria (idiopatica o familiar). Enfermedades vasculares de la colágena (esclerodermia, artritis, una proliferación fibromuscular y remodelación vascular endotelial, que lupus). Cortocircuitos congénitos sistémico-pulmonares (CIV, CIA, PCA). da lugar a estenosis de la luz vascular. Hemodinámicamente se define Hipertensión portal. Infección por VIH. Fármacos anorexigenicos como una presión arterial pulmonar media > 25mmHg en reposo o de 30 (fenfluramina, fentermina). Otros: Enf. De depósito de glucógeno, de Gaucher, hemoglobinopatías, mmHg durante el ejercicio, con la adición de una presión capilar Hipertensión venosa pulmonar: EPOC. Enfermedad pulmonar intersticial pulmonar < 15mmHg, con RVP elevada. SALUD PUBLICA: Incidencia difusa. Apnea del sueño. Enfermedad con hipotensión alveolar crónica anual es de 1-2 casos por 1000 000, con edad media de diagnóstico de Tromboembolia pulmonar: Embolismo pulmonar agudo. Embolismo pulmonar crónico. 36 años (4ta-5ta décadas de la vida). La HAP primaria (idiopática) 3040%; predomina en la mujer. Los casos de HAP familiar representan el 10%. La HAP asociada a fenómeno de Eisenmenger (cortocircuito derizq), representa 30-35%. La hipertensión es la causa más común de cor pulmonale. PATOGENIA: Se han demostrado 3 factores trascendentales que incrementan las resistencias vasculares pulmonares: a) vasoconstricción sostenida, b) proliferación vascular pulmonar, c) trombosis in situ, condicionando así una arteriolopatía obstructiva, dificultando flujo sanguíneo, lo que eleva la presión en las arterias pulmonares, generando demasiada tensión en el VD, hipertrofiándolo y llevándolo a insuficiencia. La disfunción endotelial juega un rol pivote y trascendental, sobre todo por el imbalance entre las sustancias vasodilatadoras (óxido nítrico, prostaciclina, PG- 12, péptido intestinal vasoactivo) y vasoconstrictoras (Tx A-2, endotelina-1, serotonina, factor de crecimiento derivado de cel. endoteliales). En HAP familiar, se encuentran alteraciones en los receptores que codifican para los factores transformadores de crecimiento tipo beta ((BMPR-II), que regulan la angiogénesis y apoptosis celular. DIAGNÓSTICO: Cuadro clínico: El síntoma más frecuente y más temprano es la disnea progresiva 60%, acompañada de fatiga, presíncope o síncope, edema de miembros inferiores, cianosis %20, angina, así como dolor precordial secundario a isquemia del VD. Puede haber ingurgitación yugular, por contractilidad aumentada de la de la AD, por falla ventricular derecha crónica. Auscultación: segundo ruido, en su componente pulmonar desdoblado o acentuado, datos de regurgitación tricúspidea, así como la presencia de galope sobre el precordio correspondiente al VD. El diagnostico de certezas solo puede establecerse, según la definición mediante el cateterismo derecho. ECG, tele de tórax, ecocardiografía transtoracica (valorar afección valvular o miocárdica como pb. causa, así como cortocircuitos izq-der). Pruebas de función respiratoria para determinar neumopatías. El gamagrama pulmonar ventilatorio-perfusorio es estudio clave para detectar HAP tromboembolica crónica, confirmación dx. con angiografía pulmonar. TRATAMIENTO: Si está limitada la actividad física usar diuréticos, para edema periférico, O2 suplementario si esta reducida la PO2 y anticoagulación crónica con warfarina (objetivo INR= 2.0-3.0). Como los vasoconstrictores juegan un papel importante, el tx. con vasodilatadores se basa en tales procesos, pero no se puede predecir quien responderá sin resistencia a los vasodilatadores orales como a los antagonistas de canales de calcio, ya que estos mejoran la sobrevida. Mediante una prueba se puede predecir que pacientes tendrán respuesta sostenida a dichos agentes, se define una prueba positiva, cuando al administrar vasodilatadores endovenosos de acción corta como adenosina o epoprostenol, existe una disminución de 10mmHg o mayor en la presión arterial pulmonar media o como un descenso en más de 25% de las resistencias vasculares pulmonares sin presentar variación en la presión arterial sistémica, solo responden 7-10% a dicho reto, habrá que monitorizar hipotensión o empeoramiento de la insf. Cardiaca der. Otros fármacos son los prostanoides, son sustancias tanto vasodilatadoras como antiproliferativas, usados por varias vías, epoprostenol endovenoso, iloprost inhalado, treprostinil subcutáneo, el mecanismo de acción radica en estimular la producción de AMP cíclico como segundo mensajero induciendo vasodilatación. En pacientes con hipertensión arterial pulmonar existe un déficit relativo de óxido nítrico, por lo que los inhibidores de fosfodiesterasas tipo 5, al prolongar la vida media de los segundos mensajeros GMPc, prolongan el efecto vasodilatador y antiproliferativo del óxido nítrico, como es el caso del

CURSO ENARM CMN SIGLO XXI TEL: 36246001

Pharmed Solutions Institute

PÁGINA 44

MANUAL DE TRABAJO DEL CURSO ENARM CMN SIGLO XXI sildenafil, el cual es de 1ra línea en el tx. por vía oral de la hipertensión arterial pulmonar. Si persiste la Insf. Cardiaca derecha, debe considerarse el trasplante pulmonar bilateral. PRONÓSTICO: Deletéreo y muy pobre. Sobrevida de 2.5 años a partir del dx. sin tratamiento. Supervivencia a 5 años de 75-85% en pacientes con tx. Adecuado y adyuvante con combinación o sinergismo de fármacos con diferentes mecanismos de acción. Las variables más importantes para establecer pronóstico son presión media de la arteria pulmonar, presión de AD y GC. CASO CLINICO Paciente varón de 58 años de edad, ex fumador (con una dosis acumulada de 20 paquetes-año) y ex enolismo moderado, y con antecedentes personales de hipertensión arterial sistémica, linfangiectasias duodenales, epistaxis de repetición y cirrosis hepática enólica ChildA (MELD13) con un único episodio de hemorragia digestiva alta por sangrado de varices esofágicas. Tiene 2 hijas, una de las cuales refería episodios de epistaxis de repetición. Ingresó por clínica de infección respiratoria e insuficiencia respiratoria aguda con una saturación de O2 de 77% (FIO20,21). En la radiografía de tórax se observó un infiltrado en la base derecha que se interpretó como una probable neumonía adquirida en la comunidad y se inició tratamiento antibiótico con fluoroquinolonas (levofloxacino). PREGUNTA Ante la persistencia de hipoxemia severa refractaria cual es la conducta mas adecuada? RESPUESTA a.- Cambiar esquema antibiótico. b.- Realizar TAC. c.- Realizar ecocardiograma doppler. d.- Realizar gamagrafia. CASO CLINICO Mujer de 40 años, con xeroftalmia de 5 años de evolución a la que se habían añadido en los últimos 6 meses xerostomía y artralgias, sin otras manifestaciones extraglandulares. Las pruebas complementarias nos permitieron hacer el diagnóstico de SS: ANA positivo, antiSSa positivo, anti-SSb positivo, test de Shirmer anormal, datos de inflamación crónica en biopsia de glándula salival menor y gammagrafía de glándulas salivales compatible. Se realizó ecocardiograma de rutina donde apareció por primera vez datos de HTP ligera (32 mm Hg) que se confirmó con cateterismo derecho. La paciente presentó disnea de esfuerzo. En ecocardiografía se estimó una HTP de 46 mm Hg. Mediante TC torácica se descartó la existencia de fibrosis pulmonar y eventos tromboembólicos. Actualmente persisten artralgias que ceden con analgésicos habituales y su disnea está estable. PREGUNTA Cual de las siguientes afirmaciones es la incorrecta, respecto a la HTP del caso? RESPUESTA a.- En asociación con enfermedades del tejido conectivo CREST. b.- En asociación con LES c.- Enfermedad mixta del tejido conectivo. d.- La asociación con SS es común. CASO CLINICO Un hombre de 33 años fue admitido, con disnea reciente y progresiva, edema y pérdida de peso de aproximadamente 10 kg. Dos hermanas a los 11 y 14 años de edad. Al EF, parecía "débil" (subnutrido) (peso 60 Kg; altura 1,82 m), presentaba hepatomegalia leve y edema de miembros inferiores. Un sonido alto de cierre de la válvula pulmonar era oído en el precordio. El examen de los pulmones reveló ronquidos leves y estertores finos en ambos lados, que desaparecieron completamente después de la administración de diuréticos. Los datos de la cateterización cardíaca derecha fueron compatibles con hipertensión arterial pulmonar (HAP). Óxido nítrico inhalado y sildenafila fueron eficaces en la reducción de la resistencia vascular pulmonar. Una acentuada elevación en la presión de enmentoencuñamiento pulmonar fue registrada durante la inhalación del óxido nítrico. PREGUNTA Cual es la conducta adecuada para establecer un diagnostico definitivo? RESPUESTA a.- Gamagrama pulmonar. b.- Ecodoppler pulmonar. c.- Biopsia pulmonar abierta. d.- Tomografia axial computada. CASO CLINICO Mujer de 20 años, de profesión camarera, que consultó por cuadro de disnea, edema en el miembro superior derecho y síncope. Tenía antecedentes de ser fumadora activa de 15 cigarrillos/día y padecer de rinoconjuntivitis y asma bronquial. La paciente comenzó un mes previo con disnea a moderados esfuerzos. Cuando consultó en urgencias, presentaba disnea de reposo, debilidad, astenia y refería haber tenido un síncope. A la exploración presentaba PA de 120/80mmHg, FC de 110 lat/min y SatO2 del 95% sin O2 suplementario. Se apreciaba aumento del perímetro del miembro superior derecho, doloroso a la palpación y sin signos de empastamiento. La analítica

CURSO ENARM CMN SIGLO XXI TEL: 36246001

Pharmed Solutions Institute

PÁGINA 45

MANUAL DE TRABAJO DEL CURSO ENARM CMN SIGLO XXI presentaba perfiles bioquímico, hepático, renal, hemograma y coagulación dentro de la normalidad, excepto los dímeros D (2.500 ng/ml). PREGUNTA Cual es la conducta a seguir mas adecuada, para confirmar el diagnostico para el caso actual? RESPUESTA a.- Realizar rx de torax. b.- Realizar TAC de torax. c.- Realizar Angio-TAC de torax. d.- Realizar IRM de torax. COR PULMONALE Enfermedad cardíaca pulmonar, y se define como una dilatación y/o hipertrofia del ventrículo derecho a consecuencia de problemas en la vasculatura o el parénquima pulmonar y puede conducir a insuficiencia cardíaca derecha. El término se usa para describir cambios en la estructura y función del ventrículo derecho como resultado de un desorden respiratorio que produzca hipertensión pulmonar. La hipertrofia del ventrículo derecho es el cambio principal en el cor pulmonale crónico y dilatación del ventrículo en los casos agudos, ambos consecuencia de un incremento de presión en el ventrículo derecho del corazón. Sin tratamiento, el cor pulmonale puede causar insuficiencia cardíaca derecha y muerte. ETIOLOGÍA: Para que una alteración del corazón derecho sea clasificado como cor pulmonale, el origen de la anomalía debe encontrarse bien en el sistema de circulación pulmonar o bien en el parénquima pulmonar, produciéndose en cualquier caso una hipertensión arterial pulmonar (HTAP). Las causas principales de HTAP son: enfermedades que producen la oclusión de la red vascular pulmonar: tromboembolismo pulmonar recurrente, hipertensión pulmonar primaria, enfemedad veno-oclusiva, enfermedad del colágeno vascular o enfermedades pulmonares inducidas por drogas; enfermedades que producen vasoconstricción pulmonar hipóxica crónica, como ocurre en la bronquitis crónica, la enfermedad pulmonar obstructiva crónica (EPOC), la fibrosis quística, la hipoventilación crónica (que tiene lugar en la obesidad, las enfermedades neuromusculares como la distrofia muscular de Duchenne o la disfunción de la pared torácica), o en las personas que viven en altitud. Enfermedades que producen alteraciones del parénquima pulmonar, como ocurre en la bronquitis crónica, la EPOC, la bronquiectasia, la fibrosis quística, las neumoconiosis, la sarcoidosis o la fibrosis pulmonar idiopática. El compromiso del ventrículo derecho causado por un defecto sistémico, diferente al sistema respiratorio no es considerado cor pulmonale, debe ser de origen pulmonar subyacente. La prevalencia del cor pulmonale es difícil de definir, porque no todos los casos de enfermedad pulmonar desarrollan cor pulmonale, la capacidad de diagnosticar la HTAP mediante el examen físico de rutina es baja y los tests de laboratorio son relativamente poco sensibles. Sin embargo, avances recientes en imaginería con ecografía Doppler y biomarcadores hace más fácil el diagnóstico del cor pulmonale. Según la agudeza y la severidad del estímulo que genera la disfunción cardíaca, se puede distinguir: Cor pulmonale agudo: ocurre tras un estímulo repentino y severo, que produce dilatación y fallo del ventrículo derecho, pero no hipertrofia; puede ser producido por: Embolismo pulmonar masivo; Maligno: carcinomatosis miliar, linfangitis carcinomatosa. Empeoramiento del cor pulmonale crónico. Cor pulmonale crónico: se produce a causa de un aumento progresivo y lento de la HTAP, que conduce a la hipertrofia y dilatación del ventrículo derecho; puede generarse a causa de: Enfermedad obstructiva crónica (EPOC), como bronquitis crónica y enfisema, en la que la alteración de la estructura alveolar y la consecuente insuficiencia respiratoria representa el inicio del cor pulmonale. Pérdida de tejido pulmonar por razones traumáticas o quirúrgicas. Pneumocistitis terminal. Escoliosis, especialmente con desviación severa de la columna lo cual interfiere con el desarrollo normal de los pulmones. Dicha restricción respiratoria causa hipoxemia, hipercapnia con subsecuente vasoconstricción pulmonar e hipertrofia del ventrículo derecho. Sarcoidosis por fibrosis del parenquima pulmonar en una pequeña porción de los pacientes (apróx. 5%). FISIOPATOLOGÍA: Como se indica en el apartado "Etiología", existen muchas patologías diferentes que pueden producir cor pulmonale, pero todas ellas tienen en común la generación de hipertensión arterial pulmonar (HTAP), suficiente para generar la dilatación del ventrículo derecho, con o sin hipertrofia asociada. Anatómicamente, el ventrículo derecho tiene una pared delgada, con una compliancia elevada, por lo que está mejor preparado para adaptarse a variaciones de volumen que a variaciones de presión. Por ello, cuando se produce un aumento de la presión pulmonar (HTAP) y un aumento de la resistencia vascular pulmonar, el ventrículo derecho no puede desarrollar la fuerza suficiente como para superar dicho aumento, por lo cual se dilata (en los casos agudos) o se hipertrofia (en los casos crónicos), pudiendo llegar a fallar. La severidad de la HTAP y el fallo del ventrículo derecho están influidos por múltiples factores que pueden producirse de manera intermitente, como hipoxia secundaria a alteraciones en el intercambio gaseoso, hipercapnia y acidosis, además de cambios en la sobrecarga de volumen del ventrículo derecho que se producen con el ejercicio, con el aumento del ritmo cardíaco, la policitemia o la retención de sales debido a una disminución del gasto cardíaco. MANIFESTACIONES CLÍNICAS: Síntomas: Generalmente están asociados al problema pulmonar de fondo. El síntoma más común es disnea (dificultad para respirar), debido a que hay un aumento del trabajo necesario para respirar, a consecuencia de los cambios en la elasticidad del pulmón (como ocurre en la fibrosis) o a modificaciones en la mecánica respiratoria (como sucede en la EPOC), y ambos pueden verse agravados por un incremento de la hipoxemia. La hipoxemia pulmonar puede generarse por una disminución de la permeabilidad de los capilares, por desequilibrios ventilación-perfusión o por la presencia de un shunt cardíaco o pulmonar. La presencia de ortopnea o disnea paroxística nocturna son raramente síntomas aislados de fallo cardíaco derecho. Pero si se presentan, indican que hay un aumento del trabajo respiratorio en el decubito. También puede presentarse tos o síncope en pacientes con cor pulmonale con HTAP severa, debido a la incapacidad del corazón derecho de bombear sangre a través de los pulmones hacia el corazón izquierdo. También puede presentarse dolor abdominal y ascitis, así como edema de las extremidades inferiores. Signos: Muchos de los signos que se encuentran en los pacientes con cor pulmonale también se encuentran en pacientes con una fracción de eyección reducida: taquipnea, presión venosa yugular elevada, hepatomegalia y edema en las extremidades inferiores. Además, el ventrículo derecho puede palparse a lo largo del borde izquierdo del esternón o en el epigastrio. En el cor pulmonale, la aparición de cianosis es un evento tardío, secundario a una reducción del gasto cardíaco con vasoconstricción sistémica y desequilibrio ventilación-perfusión en el pulmón. EDEMA DE PULMÓN AGUDO: La causa más común del edema pulmonar agudo es el aumento de la presión capilar (desequilibrio de las fuerzas de Starling) en la microvasculatura pulmonar, causando fuga plasmática hacia el espacio

CURSO ENARM CMN SIGLO XXI TEL: 36246001

Pharmed Solutions Institute

PÁGINA 46

MANUAL DE TRABAJO DEL CURSO ENARM CMN SIGLO XXI intersticial. Cualquier evento que curse con hipertensión venosa pulmonar ocasionará una congestión venosa y un aumentado volumen de sobrecarga al ventrículo derecho. La incapacidad del ventrículo derecho de expandir para adaptarse a ese incrementado volumen aumenta aún más el volumen venoso, y por ende retrógrada, la presión capilar. La fuga de plasma por una mayor presión hidrostática en el capilar pulmonar que en el espacio intersticial acumula líquido en el parénquima pulmonar. La suma de estas anormalidades en el pulmón y su circulación vascular es la complicación más importante producida por el edema pulmonar en la instalación del cor pulmonale. COMPLICACIONES: El cor pulmonale retrasa el flujo de la sangre desde la circulación venosa hacia la arterial. En la hiperemia, la sangre se acumula en el sistema venoso, incluyendo la vena hepática. La congestión prolongada de sangre en la región centro-lobulillar del hígado conlleva a hipoxia y cambios grasos de los hepatocitos periféricos, produciendo agrandamiento voluminoso del hígado, llegando a tener hasta el doble del volumen de sangre. El aspecto cianótico combinado con islotes grasos amarillentos le da la apariencia al hígado de nuez moscada. Primero el hígado se congestiona, aumenta de tamaño, se vuelve cianótico y los sinusoides, en especial en la zona centrolobulillar, se repletan de sangre. Comienza también una infiltración grasosa que termina por darle el tinte amarillento final. De persistir la deficiencia de la circulación pulmonar, se instala en las trabéculas del hígado una atrofia que suele ser cianótica. Con el tiempo se agrega una acumulación fibrótica (cicatrización) que tiende a reducir al hígado de tamaño y a endurecerlo. Un proceso patológico similar ocurre en los riñones y el bazo: congestión sanguínea, atrofia cianótica y endurecimiento. En los pulmones, la hiperemia produce ingurgitación capilar, edema en el intersticio pulmonar y, como consecuencia, fibrosis entre los alvéolos y la matriz capilar, traduciéndose en trastornos de la relación perfusión y ventilación pulmonar. El pulmón adopta una consistencia dura y (macroscópicamente) de color "rojo morena". El cor pulmonale puede también llevar a insuficiencia cardíaca congestiva (ICC), empeorando la respiración por razón del edema pulmonar, hinchazón de las piernas debido a edema en la periferia y hepatomegalia congestiva y dolorosa. La ICC es un indicador negativo en el pronóstico del cor pulmonale. DIAGNÓSTICO: La causa principal de fallo cardíaco derecho no es la disfunción de la vasculatura o el parénquima pulmonar, sino el fallo cardíaco izquierdo. Por ello es importante evaluar la posible disfunción del ventrículo izquierdo en un paciente con síntomas de fallo cardiaco derecho. Varios exámenes son de utilidad para el diagnóstico del cor pulmonale: Electrocardiograma (ECG): los pacientes con HTAP severa muestran una onda P pulmonar, una desviación del eje derecho e hipertrofia del ventrículo derecho; Ecocardiograma: útil para determinar el grosor del ventrículo derecho y las dimensiones de las cámaras cardíacas, así como la anatomía de las válvulas; Ecografía Doppler, para verificar la presión arterial pulmonar; Radiografía de tórax, que puede mostrar un agrandamiento de la arteria pulmonar principal, los vasos del hilio pulmonar y la arteria pulmonar descendente derecha; TAC del tórax, útil en el diagnóstico de tromboembolia pulmonar aguda; además, es el método más fiable para diagnosticar enfisema y enfermedad intersticial pulmonar; Pruebas de función pulmonar; Gases sanguíneos; Cateterización de Swan-Ganz: el cateterismo del ventrículo derecho permite confirmar el diagnóstico de HTAP y excluir la presión elevada del ventrículo izquierdo como causa del fallo del ventrículo derecho. TRATAMIENTO: La eliminación de la causa es la intervención más importante. En el embolismo pulmonar, se apela a trombolisis (disolución enzimática del coágulo sanguíneo), en particular si hay trastornos del ventrículo derecho. En la EPOC, la terapia con oxígeno a larga duración puede mejorar el cor pulmonale. Los principios generales de tratamiento del cor pulmonale incluyen la reducción del trabajo respiratorio, mediante ventilación mecánica no invasiva, broncodilatadores y esteroides, además de tratar las infecciones subyacentes. Una oxigenación adecuada (saturación de oxígeno 90-92%) también permite reducir la resistencia vascular pulmonar y reducir la demanda sobre el ventrículo derecho. Los pacientes anémicos necesitan una transfusión y si el hematocrito excede 65% debe efectuarse una flebotomía para reducir la HTAP. El edema requiere diuréticos (reduciendo el sobreuso del corazón), a veces nitratos para mejorar el flujo sanguíneo, inhibidores de la fosfodiesterasa como sildenafil y tadalafil, y ocasionalmente fármacos ionotropos para mejorar la contractilidad del corazón. Otros medicamentos ayudan a mejorar la hipertensión pulmonar, tales como la prostaciclina o sus análogos. Asimismo se han prescrito anticoagulantes y bloqueadores de los canales de calcio para las etapas iniciales del trastorno. CASO CLINICO Mujer de 48 años, exfumadora de 10 paquetes/año, diagnosticada de síndrome de CREST e hipertensión pulmonar secundaria en seguimiento en consulta de Neumología desde hace seis años. En la actualidad, se encuentra en grado funcional III de la NYHA y está en tratamiento con acenocumarol, sildenafilo, treprostinil subcutáneo y oxigenoterapia domiciliaria. Desde hace dos meses, presenta disnea de mínimos esfuerzos (grado funcional IV de la NYHA) y edemas en miembros inferiores, por lo que se añadió furosemida y espironolactona al esquema terapéutico. Hace una semana, comenzó con disnea de reposo y aumento de los edemas, por lo que se decidió ingreso hospitalario. A la exploración estaba afebril, taquicárdica y taquipneica. A la auscultación cardiaca presentaba un soplo sistólico en foco pulmonar con refuerzo del segundo tono y en la auscultación pulmonar, crepitantes finos bibasales. En la exploración del abdomen se palpaba ascitis. En miembros inferiores presentaba edemas con fóvea hasta rodillas. Se realizó analítica sin alteraciones significativas y radiografía de tórax con aumento del índice cardiotorácico e infiltrado intersticial bilateral. En el ecocardiograma se apreciaba dilatación severa de cavidades derechas con regurgitación tricuspídea severa y PAPs de 59mmHg. PREGUNTA Cual de las siguientes aseveraciones no es correcta en el manejo de esta patologia? RESPUESTA a.- El tratamiento de la hipertensión pulmonar se basa en tres pilares fundamentales: medidas generales, anticoagulación y tratamiento vasodilatador. b.- Entre las medidas generales están evitar el ejercicio físico que provoque síntomas, las grandes altitudes, las situaciones de estrés agudo y el embarazo. c.- La anticoagulación hay que iniciarla siempre que no exista contraindicación por el aumento del riesgo de trombosis derivado del estado de hipercoagulabilidad de la hipertensión pulmonar. d.- En todos los casos deberá realizar vacunación antigripal, antineumococica y mantener profilaxias antibiótica.

CURSO ENARM CMN SIGLO XXI TEL: 36246001

Pharmed Solutions Institute

PÁGINA 47

MANUAL DE TRABAJO DEL CURSO ENARM CMN SIGLO XXI CASO CLINICO Un hombre, albañil de 50 años de edad y agricultor fue admitido por falta de aire y tos, de 4 años de duración, y edema bilateral inferior durante 2 meses. En el EF dificultad respiratoria, una frecuencia respiratoria de 50 rpm, pulso de 120 latidos por minuto, cianosis, hipocratismo digital, pedal edema con fóvea bilateral, a la izquierda paraesternal tirón, p2 voz alta, grado 3 pansistólico murmullo, máxima abajo a la izquierda borde esternal, aumento de la presión venosa yugular, hepatomegalia, ascitis, así como las conclusiones del pecho de parches sonidos respiratorios bronquiales y crepitantes. PREGUNTA Cual de las siguientes acciones no es adecuada para el caso? RESPUESTA a.- Oxígeno intermitente 4 l / min. b.- Furosemida 80 mg cada 12 horas y espironolactona 50 mg. c.- Ceftriaxona intravenosa y enoxaparina 40 mg subcutánea. d.- Prednisolona 80 mg al día. ENFERMEDAD PULMONAR OBSTRUCTIVA CRONICA (EPOC) CIENCIAS BASICAS: Definición: Enfermedad inflamatoria, prevenible y tratable con efectos extrapulmonares significativos que pueden contribuir a la gravedad de los individuos. Su componente pulmonar se caracteriza por limitación al flujo aéreo, que no es totalmente reversible y es usualmente progresiva. El tabaquismo es el mayor factor de riesgo ambiental para EPOC, otros serian ciertas exposiciones ocupacionales (minería de carbón, oro, fábricas de algodón, etc), en algunas ciudades la combustión de biomasa con mala ventilación usada para cocinar, también incrementan el riesgo de EPOC. PATOGENIA: El agente irritante (humo de cigarro, biomasa), desencadena inflamación de las vías aéreas y del parénquima pulmonar, activando macrófagos alveolares, PMN, linfocitos T, con la consiguiente liberación de mediadores de inflamación (leucotrienos, TNF a, IL-8) y estos estimulan la liberación de proteasas, destruyendo bronquios y parénquima = disminución de la elasticidad, atrapamiento aéreo, y obstrucción al flujo aéreo. La bronquitis crónica se caracteriza por hiperplasia de las glándulas submucosas de los bronquios con calibre mayor a 2mm con infiltrado inflamatorio. El enfisema pulmonar se clasifica en centroacinar cuando se afectan las partes centrales del acino (bronquiolo respiratorio) y están preservados los alveolos distales y se observa en los ápices; el panacinar los acinos están afectados uniformemente desde el bronquiolo respiratorio hasta el alvéolo y es más frecuente en las bases. DIAGNOSTICO: Síntomas cardinales: disnea (progresiva, persistente, y empeora con el ejercicio), tos crónica (>8sem.), intermitente y presenté durante todo el día, expectoración crónica, sibilancias, sensación de opresión torácica, pérdida de peso, ansiedad. Estos síntomas serán más perceptibles en los períodos de agudización, que pueden deberse a infecciones respiratorias virales o bacterianas e incrementar la severidad del EPOC. Existe una escasa respuesta a esteroides. Para dx. es indispensable la espirometría, que demuestre obstrucción, que no es del todo reversible, posterior a aplicar broncodilatadores. Se concluye obstrucción tras relación FEV 1/CVF menor a 70%, el FEV1 sirve para clasificar enf. de acuerdo a intensidad. La paO2 se mantiene cerca de lo normal hasta que el FEV1 cae 80% II

MODERADO

FEV1 50-80%

III

SEVERO

FEV1 30-50%

IV

MUY SEVERO

FEV1 20 paquetes/ año; se considera con incremento en factor de riesgo para cáncer pulmonar. Otros son asbesto, arsenico, niquel. SALUD PUBLICA: Primer lugar de mortalidad en todos los tipos de cáncer en el mundo occidental. En México la tasa de mortalidad para cáncer pulmonar es de 125.2 por cada 100,000 en hombres y 48.8 por cada 100,000 en mujeres. Mas comun en hombres de 50-70 años. PATOGENIA: El humo de tabaco contiene alrededor de 4800 compuestos; 60 son cancerígenos, ya que reaccionan con el DNA humano, formando complejos con las bases nitrogenadas, así en la replicación de DNA, se introducen errores en la copia, dando lugar a mutaciones. El mas estudiado es el benzopireno, que hace una tramsversion de guanina por tiamina y las nitrosaminas que median reacciones de alquilación de DNA. Se han detectado mutaciones en el gen p53 (cromosoma 17), el cual es un gen supresor de tumores, que al no funcionar adecuadamente permite la sobrevida de elementos geneticamente dañados, que conducen a la transformación tumoral. También hay alteraciónes en el gen ras y myc. CLASIFICACIÓN: Se dividen en benignas (2-5%) y malignas (95%). Dentro de los benignos, el hamartoma es el mas frecuente, en general asintomático, los sintomas dependerán de localización, intratraqueales; tos, estridor, disnea incluso hemoptisis. Endobronquiales; obstrucción bronquial, neumonía, neumonitis, hiperinsuflacion. Intraparenquimatosos; gnral. asintomaticos sin importar tamaño. El hamartoma es una masa formafa de cartilago, tej. conectivo, grasa, musculo liso y epitelio respiratorio, en Rx. presentan calcificaciones en palomita de maiz, caracteristico. Malignas: Se dividen en 2: Cáncer broncogenico de celulas pequeñas (20%; subtipos: carcinoma de cel. pequeñas, mixto de cel. pequeñas y grandes, combinado de cel. pequeñas) y de celulas no pequeñas (80%; subtipos: adenocarcinoma 60%, escamoso o epidermoide 30%, carcinoma indiferenciado de cel. grandes 400mg/dl, calcificación de tejidos blandos, evidencia radiográfica de osteítis fibroquistica, ulcera péptica o pancreatitis recidivante. HIPOPARATIROIDISMO: El hipoparatiroidismo es un trastorno causado por la hipofunción de las glándulas paratiroides, caracterizada por una muy baja concentración de hormona paratiroidea (PTH), de calcio y un aumento en la concentración de fósforo sanguíneo. La PTH es la hormona encargada de mantener los niveles normales de calcio en el suero sanguíneo, favoreciendo su absorción en el tracto digestivo por medio de la activación de la vitamina D3 en el riñón con su subsiguiente migración hacia en epitelio intestinal y la resorción ósea aumentando indirectamente la actividad de los osteoclastos en el hueso, aparte del incremento de la captación por parte de los túbulos distales de las nefronas que reabsorben este mineral. CAUSAS: Esta patología puede ser producida por una variada gama de factores, entre los que encontramos: La extirpación quirúrgica accidental o intencional durante la tiroidectomía, siendo esta la causa más frecuente. Dentro de las causas no quirúrgicas los procesos autoinmunes son los más importantes, encontrando dentro de ellos el síndrome poliglandular autoinmune La Ausencia o disfunción congénita de la glándula paratiroides la cual se ha relacionado con microdelecciones dentro del cromosoma 22q11 (síndrome de DiGeorge). Las deficiencias de magnesio. La Hemocromatosis, causante de un cuadro de disfunción de los órganos endocrinos debido al acumulo de hierro en el interior de las células. Idiopáticos. PATOLOGÍA: Siendo el calcio uno de los iones más importantes en el mantenimiento de la homeostasia normal del cuerpo, además de estar con implicado en la contracción del músculo esquelético estriado, liso y cardiaco, cualquier variación en sus concentración desencadena un alto número de desordenes funcionales todo esto producido en el caso del hipoparatiroidismo por la disminución del potencial de acción, lo que lleva al aumento de la excitabilidad celular, favoreciendo en el caso del músculo la aparición de tetania y el SNC de convulsiones. Además de esto el calcio está relacionado con los procesos de secreción glandular favoreciendo la unión de las vesículas de secreción a la pared celular promoviendo así su exocitosis, la disminución de este mineral afecta significativamente este suceso lo que se ve dilucidado en la reducción de la sudoración que presentan los paciente que tienen este proceso patológico. CLÍNICA: La aparición de síntomas varían entre una persona y la otra, en especial en los niños, sin embargo, los más frecuentes pueden incluir: Disnea, Dolor cólico agudo, Hormigueos, Contracciones de distintos grupos musculares. Son muy característicos las localizadas en algunos músculos del brazo y mano. Esta posición incluso puede ser provocada estimulando los músculos del brazo mediante una compresión con el manguito de un esfigmomanómetro, lo que se conoce como el signo de Trousseau. También aparecen frecuentemente cataratas, trastornos psíquicos, y piel seca y otros trastornos tróficos de la piel. Los trastornos mentales más frecuentes de la hipocalcemia aguda causada por el hipoparatiroidismo es el delirium y la psicosis. En la hipocalcemia crónica son más frecuentes los síntomas de ansiedad, irritabilidad, labilidad emocional, depresión, psicosis, alteración de la memoria y de la concentración, deterioro cognitivo y a veces retraso mental en función de la edad de comienzo. El compromiso intelectual aparece en el 50% de los casos de hipoparatiroidismo. Variantes clínicas: Pseudohipoparatiroidismo: En este cuadro, además de los datos descritos, se añaden trastornos constitucionales como talla corta, cara ancha, y metacarpianos y metatarsianos cortos. Pseudopseudohipoparatiroidismo: En el que sólo aparecen los trastornos constitucionales descritos, sin alteración del calcio. Tratamiento: El tratamiento consistiría en la administración de la hormona paratiroidea (PTH) pero principalmente de la administración de calcio y vitamina D en caso de que se trate de la verdadera forma del hipoparatiroidismo la cual presenta bajos niveles de calcio sérico. CASO CLINICO Mujer de 77 años que consulta por disnea progresiva de medianos esfuerzos en el último año, ortopnea e hinchazón de los miembros inferiores. Entre sus antecedentes no había factores de riesgo cardiovascular, no tenía hábitos tóxicos ni recibía ninguna medicación. El examen físico reveló una presión arterial de 160/100 mmHg, leve cianosis perioral e ingurgitación yugular hasta ángulo mandibular. No tenía bocio ni adenopatías periféricas. Los tonos cardíacos eran rítmicos, sin soplos, y en la auscultación pulmonar había crepitantes bibasales. Tenía leves edemas maleolares bilaterales. La exploración neurológica era normal, con negatividad de los signos de Trousseau y Chvostek. En el hemograma destacaba macrocitosis y en la bioquímica, un calcio corregido con la albúmina de 4,2 mg/dl, fósforo 8 mg/dl, magnesio 1,6 mg/dl y la función renal normal. La CPK fue de 739 con CPK-Mb normal, GOT 305 y LDH de 1.038. PREGUNTA Cual de las siguientes manifestaciones esperaría mas frecuente encontrar en el caso? RESPUESTA a.- QT estrecho. b.- QT alargado. c.- T picudas. d.- T aplanada. CASO CLINICO Varón de 21 años, con cefalea y vómitos de 24h de evolución. En la EF sólo se encontró un discreto edema y eritema amigdalar. Leucocitosis con desviación izquierda (14.700leucocitos/μl con 85% neutrófilos). A las 24–48h del ingreso, el cuadro clínico empeoró

CURSO ENARM CMN SIGLO XXI TEL: 36246001

Pharmed Solutions Institute

PÁGINA 60

MANUAL DE TRABAJO DEL CURSO ENARM CMN SIGLO XXI con la aparición de obnubilación y somnolencia. En la nueva analítica, destacaba una calcemia de 15,8mg/dl. Se administró fluidoterapia, corticoides y aporte adicional de potasio y magnesio, evolucionando favorablemente, con recuperación del estado de consciencia. El electrocardiograma se normalizó y la sintomatología digestiva desapareció en 24h. PTH 237pg/ml (9–65), calciuria 290mg/24h (5–35), magnesio sérico 1,63mg/dl (1,69–2,29), creatinina 0,46mg/dl (0,53–0,79). PREGUNTA Cual es el diagnostico mas probable? RESPUESTA a.- Adenoma de paratiroides. b.- Hiperplasia de paratiroides. c.- Carcinoma de paratiroides. d.- Sindrome paraneoplasico. PREGUNTA En caso de la presencia de síndrome paraneoplasico, cual es la fuente mas frecuente? RESPUESTA a.- Riñon b.- Pulmon c.- Cabeza d.- Cuello CASO CLINICO Mujer de 38 años con IRC secundaria a nefropatía tubulointersiticial crónica. A su ingreso destacaban niveles séricos de calcio de 12 mg/dL, Fósforo 6,6 mg/dL, FA 2.490 UI/L, PTHi 1.450 pg/ml. Destacaba la presencia de una tumoración cervical de 3 x 2 x 1 cm situada en la zona inferior del lóbulo derecho de la glándula tiroides con 4 ganglios linfáticos palpables. Radiológicamente se observaba resorción subperióstica a nivel de las falanges proximales de ambas manos y una estructura granular a nivel del cráneo. Se ingresó por un cuadro de astenia, debilidad muscular y dolores óseos generalizados. PREGUNTA Cual es la conducta a seguir mas adecuada para el caso clínico? RESPUESTA a.- Hidroxido de aluminio. b.- Quelantes cálcicos. c.- Vitamina D. D. Paratiroidectomía. TRASTORNOS DE GLANDULAS SUPRARRENALES CIENCIAS BASICAS: Las suprarrenales están compuestas de medula y corteza está dividida en glomerular (produce aldosterona-regula Na y vol. Extracelular por medio de la retención de H2O, Na y la excreción renal de K), fascicular (produce cortisol-aumenta niveles de glucosa, metabolismo de proteínas y lípidos, efecto inmunosupresor, antiinflamatorio, actúa en SNC) y reticular (produce cortisol y esteroides androgénicos: dihidroepiandrostendiona y androstendiona). En médula las células cromafines secretan adrenalina y adrenalina, cuando la medula produce catecolaminas en exceso conduce a feocromocitoma. HIPERFUNCION DE GLÁNDULAS SUPRARRENALES: SINDROME DE CUSHING: La causa más común es la iatrogénica, debido a la administración de glucocorticoides. El Cushing endógeno resulta del exceso de producción de cortisol (y otras hormonas esteroideas). La mayor causa de hiperplasia suprarrenal bilateral secundaria es la hipersecreción de Hormona adrenocorticotropica (ACTH) por la pituitaria (Enf. de Cushing) o de fuentes ectópicas tales como carcinoma de cel. Pequeñas de pulmón, carcinoma medular de tiroides o tumores de timo, páncreas, ovario. Los adenomas y carcinomas de glándulas suprarrenales representan aproximadamente el 25% de los casos de síndrome de Cushing. CUADRO CLÍNICO: Más común; obesidad central, HTA, osteoporosis, psicosis, acné, amenorrea y DM, pero inespecíficas, más específicos incluyen hematomas, estrías púrpura, miopatía proximal, la deposición de grasa en la cara y las zonas interescapular (facies de luna y joroba de búfalo), y virilización, piel delgada y frágil. La hipopotasemia y alcalosis metabólica son prominentes, sobre todo con la producción ectópica de ACTH. DIAGNÓSTICO: Se requiere de demostración del incremento de cortisol y supresión anormal de cortisol en respuesta a dexametasona. Para el cribado, medir cortisol libre urinario en 24 hrs. TRATAMIENTO: Para adenoma o carcinoma suprarrenal requiere escisión quirúrgica, dar glucocorticoides antes y después de la operación, para regular el estrés. Metástasis y carcinomas irresecables se tratan con mitotano con incrementos graduales de 6g por día dividido c/8-6h. En ocasiones la citoreducción de carcinoma de pulmón, o resección de otros tumores, podemos remitir el Sx. de Cushing ectópico. Si las fuentes de ACTH no pueden ser resecadas, hacer adrenalectomía bilateral total administrar ketoconazol (600-1200mg/dl), metirapona (2-3g/d), mitotano (2-3mg/d), podrían aliviar las manifestaciones por exceso de cortisol. HIPERALDOSTERONISMO: Es causada por hipersecreción de aldosterona adrenal. PRIMARIO hipersecreción autónoma de aldosterona (suprarrenal) con supresión de los niveles de renina, puede ser debido a un adenoma suprarrenal 35% (productor de aldosterona=Sx. de Conn) o hiperplasia suprarrenal bilateral 60%, patogenia: aldosterona produce retención renal de sodio y pérdida de potasio. Esto se traduce en la expansión de contenido de sodio del cuerpo, lo que lleva a la supresión de la síntesis de renina renal. La acción directa de la aldosterona en la nefrona distal provoca la retención de sodio y pérdida de hidrógeno y los iones de potasio, lo que resulta en una alcalosis hipocalemia, la aldosterona tiene efectos fisiopatológicos en un rango de otros tejidos, causando fibrosis cardiaca, disfunción vascular endotelial y nefroesclerosis.

CURSO ENARM CMN SIGLO XXI TEL: 36246001

Pharmed Solutions Institute

PÁGINA 61

MANUAL DE TRABAJO DEL CURSO ENARM CMN SIGLO XXI El SECUNDARIO ocurre secundario a elevación de los niveles de renina circulante, esto es típico en falla cardiaca, cirrosis, síndrome nefrótico, pero podría ser debido a estenosis de la arteria renal o tratamiento diurético, muy raro reninoma. CUADRO CLINICO: Primario; tienen dolores de cabeza e hipertensión diastólica. El edema es característicamente ausente, a menos que la insuficiencia cardíaca congestiva o la enfermedad renal está presente. Las pérdidas de potasio urinario, puede causar debilidad muscular y fatiga, aunque los niveles de potasio pueden ser normales en aldosteronismo primario suave. También se puede producir alcalosis metabólica e hipernatremia. DIAGNOSTICO: Se sugiere por la hipertensión y es asociado con hipocalemia persistente, en un paciente sin edema, que no este recibiendo diuréticos ahorradores de K. TRATAMIENTO: La cirugía puede ser curativa en adenoma adrenal, pero no efectiva en hiperplasia suprarrenal, que se maneja con restricción de Na y espironolactona. El secundario; es tratado con restricción de sal y corrección de la causa subyacente. HIPOFUNCIÓN DE GLÁNDULAS SUPRARRENALES: La insuficiencia suprarrenal primaria es debida a falla de la glándula suprarrenal, mientras la insuficiencia suprarrenal secundaria es debida a falla de la producción o liberación de ACTH. ENFERMEDAD DE ADDISON: Ocurre cuando más >90% del tejido suprarrenal está destruido quirúrgicamente, por enf. granulomatosa (Tb, histoplasmosis, coccidioidomicosis, criptococosis), vía autoinmune, metástasis bilateral, hemorragia bilateral, CMV,HIV, Amiloidosis, sarcoidosis. CUADRO CLINICO: Fatiga, debilidad, anorexia, nausea y vómito, pérdida de peso, dolor abdominal, pigmentación cutánea y mucosa, deseo de sal, hipotensión y ocasionalmente hipoglicemia, si hay depleción del fluido extracelular se acentúa la hipotensión. DIAGNOSTICO: La mejor prueba de detección es la respuesta del cortisol 60 min después de 250 µg de ACTH IV o IM. Los niveles de cortisol deben exceder de 18 mg / dl 30 a 60 minutos después de ACTH. Si la respuesta es anormal, la deficiencia primaria y secundaria se puede distinguir mediante la medición de aldosterona en sangre. TRATAMIENTO: Hidrocortisona 20-30mg/d, dividido 2/3 en la mañana y 1/3 en la tarde es el pilar de la sustitución de glucocorticoides. La sustitución de mineralocorticoides es necesaria en la insuficiencia suprarrenal primaria. Durante la crisis suprarrenal se usan dosis altas de hidrocortisona (10mg/h continuas IV o 100mg bolo IV tres veces al día). HIPOALDOSTERONISMO: Deficiencia aislada de aldosterona. Con producción normal de cortisol, se produce con hiporeninismo, como un defecto biosintético hereditario, después de la extirpación de los adenomas secretores de aldosterona, y durante el tratamiento prolongado con heparina. Hipoaldosteronismo hiporreninémico es más frecuente en adultos con insuficiencia renal leve y la diabetes mellitus en relación con la hiperpotasemia desproporcionada. Fludrocortisona oral, restablece el equilibrio electrolítico si la ingesta de sal es adecuada. En IRC y falla cardiaca está aprobado el uso de furosemide. MASAS SUPRARRENALES INCIDENTALES (INCIDENTALOMA): Las masas suprarrenales son común encontrarlas en TAC o RMI. La mayoría (7080%), son no funcionantes y la probabilidad de un carcinoma suprarrenal es baja (102 cm y mujeres con >88 cm, tendrán un mayor riesgo de desarrollar diabetes, dislipidemia, hipertensión y enf. cardiovasculares, enf. de vesicula biliar; estos pacientes deben ser colocados en una categorías de riesgo superior, que aquella que les corresponde por IMC. La incidencia de cáncer endometrial, de mama, próstata, colorectal, esteatohepatitis, osteoartritis y gota en hombres y mujeres esta incrementado por la obesidad. PATOGENIA: Puede ser resultado de aumento del aporte calórico o disminución gasto energético o la combinación de ambos. La susceptibilidad a la obesidad es de naturaleza poligénica, 30-50% de la variabilidad de reservas de grasa puede ser determinada genéticamente, este es el principal factor. Las causas secundarias de obesidad incluyen: enf. Hipotalámica, hipotiroidismo, Sx. de Cushing e hipogonadismo. El aumento de peso también es inducido por fármacos, común en aquellos que usan antidiabéticos, glucocorticoides, agentes psicotrópicos, estabilizadores del humor (litio), antidepresivos, antiepilépticos. Existen factores que al combinarse con el sobrepeso y la obesidad, colocan al paciente en un altísimo riesgo de morir prematuramente, entre ellos: infartos, angina de pecho, cirugía de arterias coronarias, ateroesclerosis, DM tipo II (esta coloca al paciente en riesgo absoluto muy alto). Tres o mas de los siguientes factores de riesgo definitivamente añaden un riesgo absoluto: HTA, tabaquismo, elevación del colesterol LDL, glucosa en ayuno alterada, antecedente de enf. cardiovascular prematura y edad (hombres >45 y mujeres >55). TRATAMIENTO: En >25kg/m2 se les sugieren cambios en estilo de vida (dieta, comidas pequeñas, frecuentes, desayuno, ejercicio. La pérdida de peso se recomienda en pacientes con IMC >30, en quienes tienen IMC 25-29.9 con factores de riesgo o aquellos que se encuentren en la categoría "alto riesgo" con base en la circunferencia abdominal. El primer objetivo es una reducción de 10% de la masa corporal en un periodo de 6 meses (a una velocidad de 0.5-1kg por sem), mediante una reducción calórica de 500-1000kcal/día, posteriormente enfocarse en mantenimiento de peso con la combinación de dieta, actividad física y cambios en la conducta. Esta fase de mantenimiento se define como una ganancia de peso no mayor a 3kg durante 2años y una reducción sostenida de la circunferencia abdominal de por lo menos 4 cm. Se usan cambios en el estilo de vida y adicionar farmacoterapia: sibutramina (inhibidor de la recaptura de norepinefrina y serotonina), produce pérdida de peso de 5-9% en 12 meses, aunque produce alteraciones en la frecuencia cardiaca y presión arterial, COFEPRIS pidió su retiro e 2010; orlistat (inhibidor de la lipasa intestinal= inhibe la absorción de grasas), produce pérdida de peso de 9-10%, en 12 meses con cambios en el estilo de vida; metformina tiende a la disminución del peso corporal. Si logramos la primer meta, nos enfocamos en la tercera fase que es pérdida de peso adicional. En pacientes que no es posible perder peso, la meta es prevenir mayor ganancia de peso. El tratamiento quirúrgico: Estará indicado según la NOM exclusivamente en los individuos adultos con obesidad severa e índice de masa corporal >40, o >35 asociado a comorbilidad importante y cuyo origen en ambos casos no sea puramente de tipo endócrino. Deberá existir el antecedente de tratamiento médico integral reciente, por más de 18 meses sin éxito; salvo ocasiones cuyo riesgo de muerte, justifique el no haber tenido tratamiento previo. Deberá ser resultado de la decisión de un equipo de salud multidisciplinario. Las cirugías para bajar de peso son A) Restrictivas (limita la cantidad de comida que el estómago puede contener y frenar el vaciamiento gástrico), pérdida significativa y sostenida por mas de 5 años; aquí encontramos, la colocación laparoscópica de la banda gástrica ajustable, gastroplastia de banda vertical, , gastrectomía vertical en banda (qx. De emergencia). B) Cirugías que limitan la ingesta de alimentos y alteran la digestión. La mas famosa es el puente gástrico tipo Y de Roux. PRONÓSTICO: Personas con obesidad mórbida viven entre 8-10 años menos que quienes tienen un peso normal. Por cada 15kg arriba del peso ideal se incrementa el riesgo de muerte temprana en 30%. Incrementan riesgo de morbilidad respecto a HTA, dislipidemia, DM tipo II, EVC, apnea del sueño, enf. arterial coronaría. 2

CASO CLINICO Varón obeso de 59 años de edad con hemiplejía izquierda. A los 54 años de edad presento disnea intensa e hipertensión arterial descontrolada. Recibió el diagnóstico de hipertensión arterial a los 44 años de edad. El examen físico reveló peso de 163,8kg, altura de 1,74 m, índice de masa corpórea 54,1 kg/m2, pulso de 84 lpm, presión arterial de 200/110 mmHg. El shock de punta del corazón fue palpado en el 6º espacio intercostal, hacia fuera de la línea hemiclavicular izquierda, y la ausculta no reveló ruidos accesorios. Había soplo sistólico en área mitral y borde esternal izquierdo. El abdomen estaba voluminoso sin visceromegalias. Había edema discreto de miembros inferiores y los pulsos en estos miembros estaban disminuidos. PREGUNTA Cual es la conducta a seguir mas apropiadas. RESPUESTA a.- Cirugia bariatrica. b.- Colocación de banda gástrica.

CURSO ENARM CMN SIGLO XXI TEL: 36246001

Pharmed Solutions Institute

PÁGINA 65

MANUAL DE TRABAJO DEL CURSO ENARM CMN SIGLO XXI c.- Orlistat mas sibutramina. d.- Dieta estricta, orlistad y metformida. CASO CLINICO Mujer de 39 años con hipertensión arterial, en tratamiento con candesartán, y obesidad grado III que comenzó a tratar con sibutramina 12 días antes. Desde el inicio de la toma de medicación refería cifras más elevadas de presión arterial y palpitaciones. El día del ingreso presentó en reposo dolor intenso retroesternal irradiado a la extremidad superior izquierda y sudoración de unos 15 min de duración, que cedió tras nitroglicerina sublingual, y llegó a urgencias asintomática. Los máximos séricos de creatincinasa y troponina T fueron 388 UI/l (normal hasta 140) y 0,23 ng/ml (normal hasta 0,035), respectivamente, con curva enzimática típica de infarto agudo de miocardio. El electrocardiograma realizado sin dolor torácico fue normal durante todo el ingreso. El ecocardiograma no mostró alteraciones de la contractilidad. La coronariografía mostró coronarias normales PREGUNTA Se programará para cirugía bariatrica más liposucción, considerando la comorbilidad, cual es la complicación aguda mas frecuente. RESPUESTA a.- Embolia grasa. b.- Embolia pulmonar. c.- Insuficiencia cardiaca. d.- Sindrome de absorción. CASO CLINICO Se trata de una mujer de 50 años de edad con obesidad mórbida (peso 105kg, talla 155cm e IMC de 43) y con antecedentes de trombosis venosa profunda de repetición. Ingresó en Dermatología por lesiones cutáneas ampollosas generalizadas con biopsia cutánea compatible con eritema polimorfo que se atribuyó al tratamiento anticoagulante. Tras valoración por el servicio de alergología, se aconsejó su retirada y su no reintroducción. Se indicó tratamiento con heparina de bajo peso molecular, desarrollando importantes hematomas dolorosos en el sitio de la administración, motivo por el que se suspendió. PREGUNTA Cual es la medida terapéutica que presenta menos complicaciones por las características del caso?. RESPUESTA a.- Cirugia bariatrica. b.- Colocación de banda gástrica. c.- Dieta y ejercicio. d.- Orlistat. CASO CLINICO Paciente varón de 60 años, con síndrome de Alport, microhematuria y proteinuria de 4gr/día, creatinina de 2 mg/dl, urea 123mg/dl, perdida moderada de la audición, hipertensión arterial tratada con enalapril 20 mg, obesidad con IMC=36,26 (P 120Kg, h 1,81mts), glucemias en ayunas alteradas (menor a 126mg/dl)Hb glicosilada normal, dislipidemia e hiperuricemia en tratamiento. Se indica dieta bajas en proteínas, hipocalórica y se agrega al tratamiento losartan en dosis de 75 mg/ día. Se realiza cirugía de by pass gástrico presentado en su evolución disminución de 35 kg en total llegando a IMC de 25,6. Control nefrológico: creatinina 1,56mg/dl, proteinuria 0,3 g/dia, Urea 65mg/dl, normotension con enalapril 5 mg/d, se mantiene dosis mínima de hipolipemiantes, normouricemia y normoglucemias. PREGUNTA Cual es la complicación más importante que presenta a largo plazo el paciente?. RESPUESTA a.- Insuficiencia renal. b.- Hipotiriodismo. c.- Sindrome de mala absorción d.- Anemia perniciosa.

CURSO ENARM CMN SIGLO XXI TEL: 36246001

Pharmed Solutions Institute

PÁGINA 66

MANUAL DE TRABAJO DEL CURSO ENARM CMN SIGLO XXI DISLIPIDEMIAS CIENCIAS BASICAS: Definición: son un grupo de trastornos caracterizados por la presencia de concentraciones anómalas de lípidos (LDL, HDL, triglicéridos) en sangre. Son importantes por su contribución en la génesis de ateroesclerosis, por lo que se relacionan directamente con la enf. cardiovascular. SALUD PUBLICA: Prevalencia en adultos mexicanos es de 30% para hipercolesterolemia, 40% hipertrigliceridemia y 50% para hipoalfalipoproteinemia, siendo la combinación de esta última mas hipertrigliceridemia, lo más frecuente en el paciente diabético. CLASIFICACION: Primarias: Secundarias a causa de obesidad, diabetes mellitus, consumo de alcohol, anticonceptivos orales, glucocorticoides, falla renal, daño hepático e hipotiroidismo, o empeorar las subyacentes. CARACTERISTICAS DE LAS PRINCIPALES DISLIPIDEMIAS PRIMARIAS HIPERCOLESTEROLEMIA AISLADA Hipercolesterolemia LDL colesterol elevados; colesterol familiar total de 275-500mg/dl Autosómica dominante Puede ser a consecuencia de mutación para el receptor LDL Xantomas tendinosos en edad adulta y xantelasmas, enfermedad vascular Complicaciones: cardiopatía isquémica Hipercolesterolemia poligénica

LDL elevada, colesterol total 300mg/dl), xantomas eruptivos (borde eritematoso y centro blanquecino, confluentes; TG >1000mg/dl), el xantoma estría palmaris o depósito de lípidos en pliegues de las manos (disbetalipoproteinemia), el xantelasma es una pápula o placa amarillenta en párpados de forma bilateral. Escrutinios a partir de 20 años y repetirse cada 5 años, dx., se basa en lípidos séricos, con ayuno de 8-10hrs. NIVELES NORMALES DE LIPIDOS LDL 190 muy alto

Colesterol total 240 alto

HDL 60 alto

bajo 40-59

TG 500 muy alto

TRATAMIENTO: Determinar número de factores de riesgo cardiovascular (hombre >45, mujer >55 años, historia en familiares de primer grado de cardiopatía isquémica a edad temprana hombre 126mg/dl (normal 60-100mg/dl). 3. Glucemia plasmatica casual >200 mg/dl, en un paiente con sintomas clasicos de hiperglucemia (normal 100-200 mg/dl). 4. Glucemia >200mg/dl a las 2 hrs después de una carga oral de 75mg de glucosa (PTOG). Prediabetes: Persona que tiene antecedente padre, madre o ambos con estado metabólico entre lo normal y la diabetes glucosa anormal en ayuno 100-125 mg/dl, PTOG; >140 y 60 años o con diabetes gestacional previa. Pacientes sin sobrepeso se puede iniciar con sulfonilureas, no control. 2. metformina + sulfonilurea (1ra eleccion). Metformina + tiazolidendionas (2 elección) no control. Metformina + sulfonilurea + insulina o metformina + tiazolidendionas + sulfonilureas. (tiazolidendionas contraindicadas en pacientes con insuficiencia cardiaca o juntas con insulina). sintomas marcados o glucemias o hb glicosilafas muy elevafas iniciar con insulina con o sin agentes adicionales. PREVENCION: En diabeticos control glicemico con hb glicosilada 2 por años, si hay descontrol hacer cada 3 meses, es lo que tiene de memoria, valores 25), con uno o mas factores de riesgo, hacer prueba cada 3 meses si salen normales. Asintomático con glicosa en ayuno >100 y 126 hacer PTGO. Prediabetes; seguimiento anual. Paciente con cifras normales sin factores de riesgo cada 3 años. Deteccion de albuminuria una vez por año y al momento del dx., toda cifra superiora 300mg/dl = albuminuria clinica o macroalbuminuria. VAloracion por oftalmologia una vez por año. PRONÓSTICO: Las variables que nos pueden hacer predecir el peor pronostico estan relacionadas con el grado de sedentarismo, coexistencia de IC, descenso del aclaramiento de creatinina (nefropatia principal), y de hemoglobina. REHABILITACION: MIcro y macroalbuminuria, iniciar trtatamiento con IECAS o ARA II, para nefroproteccion ( sin olvidar medir niveles de K+). Reducir ingesta de proteinas de 0.8-1 g/kg/día. CLAVES: Biguanidas (Metformina) 1ra. Eleccion en obesos, complicacion mas grave acidosis láctica. Sulfonilireas (glibenclamida) se eliminan por riñón si hay IRC, se acumula; con una depuración renal 44%, hiperglucemia, hipocalcemia (25%), hiperbilirrubinemia, hipertrigliceridemia (15-20%) AST y DHL elevadas, nivel de proteína C >150mg/dl, habla de necrosis y enfermedad grave. Estudios de imagen, asa centinela es un hallazgo radiológico sugestivo de pancreatitis aguda (no especifico), ultrasonido; para visualizar páncreas, gas intestinal, litos, pesudoquistes, edema o crecimiento de páncreas. TAC gold standard para dx., podemos ver zonas de necrosis, la cual tiene una elevada tasa de infección hasta de 50% y mortalidad asociada hasta de 30%, podemos utilizar criterios de Balthazar. Determinación de gravedad: Con criterios de Ranson. TRATAMIENTO: La piedra angular es el ayuno (sonda nasoenteral o parenteral), iniciar VO de 3-5 días de iniciado el tx., soluciones IV abundantes (mas cristaloides), analgésicos, O2, corrección de alteraciones hidroelectrolíticas. CEPRE en pacientes con pancreatitis secundaria a litos biliares y colangitis. Antibióticos se recomienda en casos con evidencia de necrosis pancreática mayor de 30%, y alta sospecha de infección. El fármaco recomendado es imipenem 500mg tres veces al día por 7 días. Cirugía indicada en casos de necrosis infectada. COMPLICACIONES: necrosis pancreática (40-60%), pseudoquistes (15%), abscesos pancreáticos, ascitis pancreática, insuficiencia renal y/o respiratoria, sangrado gastrointestinal, choque séptico. PANCREATITIS CRÓNICA: Daño permanente en la estructura, como consecuencia de un proceso inflamatoria crónica, superpuesto sobre un páncreas previamente lesionado, que lleva a fibrosis y perdida de la función. Se puede dividir en calcificante obstructiva (más común, relacionada con alcohol, hereditaria, hiperlipidemia) y obstructiva (tumores, estenosis o el páncreas divisum). PATOGENIA: Lo mas común el consumo crónico de alcohol, en niños la principal causa fibrosis quística. Menos relacionada con enf. autoinmunitaria, hipertrigliceridemia, pancreatitis tropical, obstrucción del conducto e idiopáticas. DIAGNÓSTICO: Cuadro clínico: Dolor síntoma cardinal. Triada presencia de calcificaciones pancreáticas, diabetes mellitus y esteatorrea. Mala digestión y dolor abdominal muy variable, algo característico es la relación que guarda con los alimentos, por lo que el paciente disminuye los mismos y pierde peso. Para el desarrollo de esteatorrea es necesaria la pérdida de 90% o más de tejido exocrino, lo cual disminuye la producción de amilasa, lipasa y proteasas con la consecuente mala digestión de los nutrientes. No existe un incremento en los niveles de enzimas pancreáticas, las bilirrubinas en suero y la fosfatasa alcalina pueden estar elevadas. TAC, calcificaciones pancreáticas en 30-60%. CEPRE, provee información acerca de los conductos pancreáticos. COMPLICACIONES: Síntomas por Malabsorción de vitamina B12 en 40%, de alcohólicos y fibrosis quística, tolerancia a la glucosa alterado, retinopatía no diabética debido a deficiencia de vitamina A y zinc, necrosis grasa subcutánea y dolor óseo. Se incrementa el riego de cáncer de páncreas. TRATAMIENTO: Los ataques intermitentes se tratan como la pancreatitis aguda. Alcohol, comidas grandes y grasosas deben ser evitadas, narcóticos en dolor severo, mantener una adecuada hidratación y deberían de ser hospitalizados, si hay síntomas leves de manera ambulatoria. Cirugía podría controlar el dolor si hay una estenosis ductal. Pancreatectomia subtotal podría controlar el dolor pero el costo es insuficiencia exocrina y diabetes.

CURSO ENARM CMN SIGLO XXI TEL: 36246001

Pharmed Solutions Institute

PÁGINA 73

MANUAL DE TRABAJO DEL CURSO ENARM CMN SIGLO XXI CASO CLINICO Mujer de 34 años sin antecedentes; ingresó en el hospital por pancreatitis aguda biliar leve. La evolución inicial fue favorable, con comienzo de tolerancia oral al tercer día. Al quinto día presentó dolor abdominal espigástrico y en el hipocondrio izquierdo de inicio brusco, asociado a palidez, sudoración, taquicardia a 120 lpm y PA de 105/52 mmHg. A la exploración existían signos de irritación peritoneal. Analíticamente destacaba hemoglobina (Hb) 8,4g/dl, 4 puntos menos que al ingreso. En la TC abdominal se objetivó la existencia edema pancreático y un gran hematoma subcapsular esplénico de 16 x 13cm, con líquido libre peritoneal denso de alta cuantía. No se consiguió la estabilización hemodinámica con reposición de volemia y se decidió realizar una laparotomía exploradora urgente. Tras acceder a la cavidad abdominal a través de laparotomía media, se puso de manifiesto hemoperitoneo masivo (2.500ml), un gran hematoma periesplénico y un bazo decapsulado en su cara anterior. Se realizó una esplenectomía y una colecistectomía. PREGUNTA Cual de los siguientes factores predictivos tiene mayor relevancia para el pronostico inmediato. RESPUESTA a.- Hemoglobina. b.- Leucocitos. c.- Glucemia. d.- TAC. CASO CLINICO Paciente de 74 años con antecedentes de hipertensión arterial, diabetes mellitus no insulinodependiente y enfermedad de Parkinson. Acudió a las urgencias hospitalarias por dolor abdominal epigástrico de 3 días de evolución, mareos y síncope. A la exploración presentaba una PA de 80/40 mmHg y una frecuencia cardíaca de 102 lpm. El abdomen presentaba dolor meso-epigástrico y sensación de masa. Analítica: amilasemia de 854 U/l, 13.500 leucocitos/μl y Hb de 8g/dl. En la TC se confirmó pancreatitis aguda en la cabeza pancreática, con desestructuración del parénquima y signos sugestivos de necrosis a ese nivel, complicada con un hematoma retroperitoneal de 20 x 7 cm, con signos de sangrado arterial activo que situaba en una rama cercana al tronco celiaco. PREGUNTA Cual es la complicación crónica mas frecuente. RESPUESTA a.- Insuficiencia pancreática. b.- Anemia crónica. c.- Diabetes mellitus. d.- Pancreatitis crónica. CASO CLINICO Mujer de 78 años de edad, diabética e hipertensa con dolor abdominal de 24 horas de evolución. Con inestabilidad hemodinámica, insuficiencia respiratoria grave e irritación peritoneal y equimosis periumblical. Se realiza TAC abdominal compatible con pancreatitis enfisematosa, preservándose solo la cola pancreática y observando en su lugar aire extraluminal que rodea los vasos regionales, así como de forma difusa intraabdominal. Múltiples infartos hepáticos. Se realiza cirugía urgente objetivándose líquido libre peritoneal serohemorrágico con crepitación de tejidos blandos y esteatonecrosis difusa. Se reinterviene donde se observa páncreas desestructurado con aspecto necrohemorrágico. La paciente presenta mala evolución y fallece a las 48 horas del ingreso. PREGUNTA Cual es la complicación aguda mas frecuentes con descenlace fatal. RESPUESTA a.- Sepsis abdominal. b.- Falla organica multiple. c.- Coagulacion intravascular diseminada. d.- Tromboembolia pulmonar. CASO CLINICO Masculino de 89 años de edad, refiere dolor abdominal con náuseas y vómitos de 2 días de evolución. En la exploración destaca un abdomen doloroso de forma generalizada con predominio en hemiabdomen derecho. En la analítica se detectan 20.500 leucocitos (78% neutrófilos, 17 bandas), hematocrito 42%, actividad de protrombina 80%, fibrinógeno 600 mg/dl, bilirrubina 3,3 mg/dl, amilasa 229 U/l, lipasa 310 U/l, GPT 141 mg/dl; sedimento de orina y radiografía de tórax sin alteraciones significativas. Se mantiene en observación progresando el cuadro clínico requiriendo cirugía y estancia en terapia intensiva, se resuelve el cuadro agudo y es egresado un mes después de su ingreso. PREGUNTA Cual es la sintomatología crónica más probable del caso. RESPUESTA a.- Esteatorrea.

CURSO ENARM CMN SIGLO XXI TEL: 36246001

Pharmed Solutions Institute

PÁGINA 74

MANUAL DE TRABAJO DEL CURSO ENARM CMN SIGLO XXI b.- Mala absorción. c.- Anemia perniciosa. d.- Hiperglucemia. ESTEATOSIS HEPATICA (EH): CIENCIAS BASICAS: Definición: es la acumulación de lípidos histológicamente visible (grasa en hígado), en el citoplasma de los hepatocitos. Es la alteración más frecuente a nivel hepático. Definición de ETIOPATOGENIA esteatosis no alcohólica: entidad anatomoclínica caracterizada por esteatosis ESTRÉS OXIDATIVO hepática, diferentes grados de inflamación y fibrosis, hallazgos semejantes a PEROXIDACION LIPIDICA hepatopatía alcohólica, ausencia de ingesta significativa de alcohol. Se puede clasificar de acuerdo a su etiología en esteatosis alcohólica y esteatosis no ESTIMULOS TNF-alfa CELULAS ANTIGENOS alcohólica (Etiologia: síndrome metabólico, obesidad, diabetes tipo II e QUIMIOTACTICOS IL-6, IL-8 ESTRELLADAS PROTEICOS hiperlipidemia, rápida pérdida de peso en los obesos, nutrición parenteral INFLAMACION FIBROSIS H. MALLORY total, síndrome de intestino corto, gastroplastia, hipolipoproteinemias, tirosinemi, enf. de Wilson, tratamiento con drogas como amiodarona, estrógenos, tetraciclinas, coticoides, tamoxifeno, nifedipino, lipodistrofia, hígado graso del embarazo). SALUD PUBLICA: La esteatosis hepática no alcohólica es un padecimiento que afecta al 20-30% de la población general en varios países; la prevalencia aumenta en los sujetos con obesidad mórbida 75-92%, mientras que en la población pediátrica es de aproximadamente 13-14%. La forma progresiva de la esteatosis hepática no alcohólica, la esteatohepatitis, se estima que aproximadamente en 3-5% de los casos puede progresar a cirrosis. En México la prevalencia de esteatosis hepática no se conoce con precisión; sin embargo, si podemos extrapolar datos de obesidad obtenidos de la encuesta nacional de salud y nutrición (ENSANUT) de 2006; en ella se informa que el 30% de la población reúne criterios de obesidad y de ellos, más de las dos terceras partes pueden tener esteatosis en el hígado; por otra parte, la prevalencia promedio de DMT 2 es de 7%, y el 92% de estos pacientes tienen síndrome metabólico y resistencia a la insulina. CLASIFICACION: Según el porcentaje de hepatocitos afectados, se clasifica en: 1) Leve; menos de 25% de los hepatocitos afectados, 2) Moderada; de 25-50%, 3) Severa; más de 50%. PATOGENIA: La EH, resulta de un desequilibrio entre la síntesis hepatocitica de triglicéridos a partir de ácidos grasos y de la secreción desde el hepatocito bajo forma de lipoproteínas. La retención de triglicéridos en los hepatocitos es requisito indispensable para el desarrollo de esteatosis hepática. El metabolismo de los triglicéridos en el tejido adiposo da lugar a la liberación de ácidos grasos libres a la circulación y éstos son captados por los hepatocitos. El acumulo de lípidos en el hígado resulta de una pérdida del balance entre la captación, la síntesis, la salida y la oxidación de los ácidos grasos libres. Estudios recientes demuestran que el flujo de ácidos grasos libres provenientes del tejido adiposo y que llegan al hígado representa la mayor fuente de la grasa intrahepática, 62% – 82% de los triacilgliceroles del hígado y que la contribución de lipogénesis “de novo” que en sujetos normales es menor del 5%, en pacientes con esteatohepatitis aumenta hasta ser del 26%. La insulina, inhibe el metabolismo de los triglicéridos en el tejido adiposo, aumenta la síntesis intrahepática de ácidos grasos libres y de triglicéridos e inhibe la beta oxidación de los ácidos grasos libres en los hepatocitos. Los estados de hiperinsulinismo como la DMT2 y la obesidad disminuyen la sensibilidad tisular a la insulina, esta resistencia a la acción de la insulina es el factor más reproducible en el desarrollo de esteatosis hepática. Muchos casos son atribuidos al alcohol, su presencia se asocia con un espectro de enfermedades tales como: ingesta de drogas, tóxicos, obesidad, dislipidemias, diabetes, caquexia y nutrición parenteral. En la esteatosis hepática al igual que en otros padecimientos metabólicos se altera la liberación de adipocinas. Estas proteínas sintetizadas específicamente en el adipocito, comparten funciones que regulan energía y procesos de inmunidad. Cuando se altera el lenguaje y comunicación entre el hígado y el tejido adiposo da como resultado una serie de anormalidades metabólicas e inflamatorias. ESTEATOHEPATITIS: La esteatohepatitis es una forma más agresiva de esteatosis hepática en la cuál la infiltración grasa del hígado en un porcentaje variable de pacientes (que puede llegar hasta el 20–30%), se acompaña de intensa actividad necro-inflamatoria y puede progresar a fibrosis y cirrosis hepática y eventualmente a insuficiencia hepática y hepatocarcinoma. De los factores que participan en la progresión de hígado graso a esteatohepatitis se reconoce que tanto el síndrome metabólico (Resistencia insulínica / hiperinsulinismo / DM tipo II,Obesidad, Dislipemia (TG >180 o HDL-col 5mmHg, lo cual favorece el desarrollo de circulación colateral y derivación del flujo portal hacia la circulación sistémica, es causada por aumento en la resistencia intrahepatica e incremento en el flujo sanguíneo esplácnico secundario a vasodilatación, sus 3 principales complicaciones son: varices gastroesofágicas con hemorragia, ascitis e hiperesplenismo. La vena porta está formada por vena gástrica isq., esplénica y mesentérica superior. 2.Varices gastroesofágicas con hemorragia: Son vasos colaterales generalmente con un gradiente de presión >12mmHg, para la prevención primaria del sangrados se utilizan bloqueadores beta no selectivos (propanolol). 3.-Ascitis: acumulación de líquido en cavidad peritoneal, se considera la complicación más frecuente de la cirrosis, se asocia a una mortalidad de 50% a los 2 años, dx., por exploración física y USG abdominal, se debe realizar paracentesis para determinar la albumina, proteínas, glucosa, LDH, conteo celular, tinción de Gramm y cultivo. El tx., consiste en restricción de la ingesta de Na, diuréticos (espironolactona, furosemida). 4.-Peritonitis bacteriana espontanea (PBE): ocurre en 10-20% de pacientes con cirrosis y ascitis, factores de riesgo, proteínas totales menores a 1g/dl, antecedente de PBE y sangrado gastrointestinal, se manifiesta con fiebre dolor abdominal y encefalopatía, dx., con conteo celular de liq. de ascitis debe tener >250 neutrófilos/mm3 en el cultivo lo mas frecuente es E. coli, Klensiella (70%), Enterococos y Streptococcus (30%). 5.- Encefalopatía hepática. 6.- Otros com: síndrome hepatorrenal tipo I,II , síndrome hepatopulmonar, hipertensión portopulmonar, malnutrición, coagulopatía, fibrinólisis, trombocitopenia, osteopenia, osteoporosis,, anemia, hemolisis TRATAMIENTO: La cirrosis hepática compensada no requiere de tratamiento, especifico, solo dieta restringida en proteínas (1-1.2 g/Kg), evitar consumo de alcohol y endoscopia al momento del dx., y periódica, cuando no hay varices esofágicas o son muy pequeñas, realizar cada 2 años; con varices grado II y III iniciar bloqueadores β, y cuando hay sangrado endoscopia cada 6-12 meses. El único tratamiento que modifica claramente el pronostico es el trasplante hepático. Trasplante de hígado: Indicaciones: Niños; atresia biliar, hepatitis neonatal, fibrosis hepática congénita, enfermedad de Angille´s, enfermedad de Byler´s, desordenes inherentes del metabolismo, enfermedad de Wilson´s, enfermedades de depósito lisosomal, Crigler-Najar tipo I, hipercolesterolemia familiar. Adultos; cirrosis biliar primaria y secundaria, colangitis esclerosante primaria, hepatitis autoinmune, enfermedad de Caroli´s, cirrosis

CURSO ENARM CMN SIGLO XXI TEL: 36246001

Pharmed Solutions Institute

PÁGINA 76

MANUAL DE TRABAJO DEL CURSO ENARM CMN SIGLO XXI criptogenica, trombosis venosa hepática, hepatitis crónica con cirrosis, hepatitis fulminante, cirrosis alcohólica, malignidad hepatocelular primaria, esteatohepatitis no alcohólica Contraindicaciones: Infección extrahepatobiliar no controlada, sepsis activa sin tratar, anomalías congénitas incorregibles que limitan la vida, abuso de sustancias o de alcohol, enfermedad cardiopulmonar avanzada, malignidad extrahepatobiliar, colangiocarcinoma, SIDA. CIRROSIS ALCOHOLICA: Se debe documentar abuso de alcohol en la historia clínica, asintomática, clínica >10 años de abuso de alchol. En este tipo de cirrosis la atrofia testicular debida a alteraciones hormonales o efecto toxico del alcohol. Pueden presentar anemia hemolítica por efecto de la hipercolesterolemia, en la membrana eritrocitica, la cual genera acantocitos. Es característica la elevación de AST sobre ALT, lo produce relación AST/ALT >2. La biopsia hepática reporta necrosis, cuerpos de Mllory e infiltración por neutrófilos. ESTEATOHEPATITIS NO ALCOHOLICA: Es la causa mas frecuente de cirrosis criptogénica. Enfermedad metabólica adquirida que se origina por el depósito de triglicéridos en los hepatocitos, asociada a inflamación y fibrosis, ciertos factores predisponen como, sexo femenino, obesidad, DM y dislipidemia. Los pacientes suelen presentar dolor en hipocondrio derecho y hepatomegalia, Laboratorio: hipertransaminemia con predominio de ALT, diagnostico mediante biopsia hepática. CIRROSIS BILIAR PRIMARIA (CBP): Enfermedad progresiva, con inflamación y destrucción de conductos biliares intrahepáticos, lo cual produce colestasis crónica y cirrosis; predomina en mujeres (95%), entre 30-65 años de edad, probablemente debida a un trastorno autoinmune además se asocia con frecuencia a enfermedades autoinmunes. Asintomáticos, manifestación inicial con frecuencia es prurito, predominio nocturno y se asocia a piel seca, se puede ver ictericia, hiperpigmentación, xantelasma, y xantomas, hepatoesplenomegalia, malabsorción intestinal. Laboratorio: anemia normocitica normocromica, eosinofilia, elevación de FA, GGT, AST, ALT, bilirrubinas normales al principio, después elevadas, hipergamaglobulinemia, anticuerpos antimitocondriales (AMA), presentes hasta en 95%, tienen sensibilidad de 95% y especificidad de 98%. Anticuerpos antinucleares (ANA) en 70% de los casos. Tratamiento alivi0 del prurito (colestiramina, colestipol, fenobarbital), correcion de las consecuencias de malabsorción intestinal. El tx., especifico de la CBP se basa en administración de esteroides y ac. ursodexosicolico (13-15mg/Kg/24h), y en el trasplante hepático. COLANGITIS ESCLEROSANTE PRIMARIA (CEP): Inflamación y fibrosis de los conductos biliares intra y extrahepáticos, los cuales se estenosan y obliteran, ocasionando cirrosis hepática. La CEP suele asociarse a colitis ulcerativa crónica inespecífica (40-80%), LES y arttris reumatoide. Predomina en hombres. Presentan astenia progresiva, prurito e ictericia. El hallazgo característico en la biopsia es la fibrosis concéntrica periductal (en cascara de cebolla). No hay tratamiento específico, se debe considerar el trasplante hepático, ya que la supevivencia después del dx., es de 12 años. CASO CLINICO Hombre de 65 años, con antecedentes de cirrosis hepática por hepatopatía crónica VHC (Child-Pugh 5). Habia sufrido un episodio de hemorragia digestiva alta por sangrado de varices esofágicas, resuelto con ligadura. Mientras pasea y de forma aguda presenta dolor intenso a nivel hipogástrico. A la llegada del equipo de asistencia médica extrahospitalario el paciente ya está inconsciente (GCS 7), con signos de mala perfusión periférica, intensa palidez y situación de shock. No hay evidencia de hemorragia digestiva alta. Se procede a intubación y reposición de volemia. PREGUNTA Cual de los siguiente sitios de sangrado en la cirrosis hepática es la menos frecuente y muy grave? RESPUESTA a.- Hemo-retroperitoneo. b.- Hemo-peritoneo. c.- Varices esofagogastricas. d.- Varices hemorroidales. PREGUNTA Considerando la respuesta anterior cuales signos y síntomas son indicativos del sitio de sangrado? RESPUESTA a.- Dolor abdominal intenso de inicio brusco y distención súbita. b.- Distension abdominal súbita y signos de choque. c.- Manifestaciones de hipovolemia y dolor abdominal. d.- Signos de choque y dolor abdominal intenso. CASO CLINICO Mujer de 70 años con cirrosis hepática por el virus de la hepatitis C, en estadio B de Child, con trombosis portal, episodios de hemorragia digestiva alta, descompensación hidrópica y un último ingreso por ascitis e hidrotórax secundario, que se resolvió con tratamiento diurético. Acudió a urgencias por un cuadro de ascitis, edemas y aumento de la disnea de 15 días de evolución. En la auscultación pulmonar tenía disminución del murmullo vesicular en los dos tercios inferiores del hemitórax izquierdo. En el hemograma destacaba únicamente la trombopenia (76.000 células/µl) y en la bioquímica, la elevación de la creatinina (1,9 mg/dl), bilirrubina (3,1 U/l), gammaglutamiltranspeptidasa (71 U/l) y fosfatasa alcalina (268 U/l), así como el descenso de la albúmina (2,3 U/l). En la radiografía de tórax se objetivaba un derrame pleural que ocupaba los dos tercios inferiores del hemitórax izquierdo y sin focos de compensacion. PREGUNTA Considerando el cuadro clínico, cual de las siguientes complicaciones es la mas probable que presenta este caso?

CURSO ENARM CMN SIGLO XXI TEL: 36246001

Pharmed Solutions Institute

PÁGINA 77

MANUAL DE TRABAJO DEL CURSO ENARM CMN SIGLO XXI RESPUESTA a.- Peritonitis. b.- Neumonia. c.- Sindrome hepato-renal. d.- Derrame pericardico. CASO CLINICO Paciente de sexo femenino de 61 años de edad, con antecedentes de diabetes mellitus tipo 2, hipertensión arterial y cirrosis hepática Child B de etiología no precisada, Consultó por cuadro de una semana de evolución caracterizado por fiebre de predominio nocturno, cuantificada hasta en 39°C y precedida de un día de diarrea no disentérica y autolimitada. Se evaluó en Servicio de Urgencia, se realizaron exámenes dentro de los cuales destacan: hemograma sin leucocitosis ni desviación a izquierda, pruebas de coagulación, electrolitos plasmáticos y función renal normales. A los tres días presenta espectoracion la cual se cultiva con resultado de L. monocytogenes. PREGUNTA Cual es el tratamiento de primera elección ya que el paciente es alérgico a la penicilinas, además de la patologia de base? RESPUESTA a.- Cotrimoxazol. b.- Eritromicina. c.- Cloranfenicol. d.- Tetraciclinas. CASO CLINICO Paciente de 38 años ex-adicto a drogas por vía parenteral sufre de forma espontánea sin traumatismo previo dolor abdominal de localización dorsal con irradiación a ambos hipocondrios, hipotensión arterial y anemización importante (5g/dl de hemoglobina), se realiza TC abdominal ante la sospecha de rotura de anerurisma aórtico, y se evidencia la existencia de rotura de tumoración hepática (hepatocarcinoma) en segmento VIII sobre un hígado cirrótico y hemoperitoneo masivo. El paciente es intervenido quirúrgicamente de forma inmediata realizándose alcoholización de la lesión hepática y sutura hemostática. PREGUNTA Cuál es el agente etiológico más probable. RESPUESTA a.- Virus tipo A b.- Virus tipo B c.- Virus tipo C d.- Virus tipo D CASO CLINICO Hombre de 65 años con DHC Child A y HCC único de 2 cm en lóbulo hepático derecho. Sin ascitis al momento del diagnóstico pero con signos de hipertensión portal (HTP) en la resonancia magnética (RM) y adecuada permeabilidad de la vena porta. Fue definido etapa 1 según la clasificación de Okuda. PREGUNTA Cuál es la conducta a seguir mas adecuada. RESPUESTA a.- Radioterapia. b.- Quimioterapia. c.- Quimioembolizacion. d.- Transplante hepático. PREGUNTA Respecto a las MET´s en hígado, cual de las siguientes observaciones son menos probable? RESPUESTA a.- MT´s hepática desde Ca. Mamario. b.- MT´s Hepática desde CA. de Vesícula. c.- Lesión solitaria de MT. d.- Gran metástasis hiperecoicas creando efecto de masa. CASO CLINICO Masculino de 74 años de edad, refiere cambios evacuatorios hace 2 meses, liquidas, amarillentas, cuatro por días, sin moco ni sangre, a la semana se anexa al cuadro fiebre recurrente nocturna, no cuantificada precedida de escalofríos la cual atenúa con medio físicos. A los 15 días se suma alcuadro dolor abdominal de fuerte intensidad tipo cólico, Antecedente de esplenectomía comoresolución a anemia

CURSO ENARM CMN SIGLO XXI TEL: 36246001

Pharmed Solutions Institute

PÁGINA 78

MANUAL DE TRABAJO DEL CURSO ENARM CMN SIGLO XXI hemolítica a los 46 años. Padre fallece a los 73 años por CA. Gástrico. Hermanos: 12 Vivos, 3 fallecen por CA. Pulmón, CA. Óseo. y CA. Gástrico. Hijo fallecido por CA. Testicular. Alcohol desde los 22 años hasta los 46 años los fines de semana tipo cerveza llegando a la embriaguez. Tabáquico: Desde los 14 años hasta los 46 años 10 paquetes por año. Refiere disminución de 20 Kg aproximadamente en 2 meses. Cuello: Adenopatía cervical de 2 cm aproximadamente. Pulmón: Crepitantes bilaterales difusos. Abdomen: Hepatometria 12/17/18 cm, bordes lisos irregulares, Hemograma: WBC 9.7, HGB 10.3, PLT 386. NEUT: 73%, LINF: 21,7%. BILIRRUBINA TOTAL 3,50 mg/dl. BILIRRUBINA DIRECTA: 2,12 mg/dl. FOSFATASA ALCALINA 200 UL. UROANALISIS: Bacterias moderadas, leucocitos 4-8 x cpo. Trazas proteínas, Pig. Biliares +++, Bilirrubina +++. TGO: 198,3 U/L. TGP: 30,7 U/L. ALBUMINA 2,5; PROTEINA C REACTIVA 36. PREGUNTA Considerando el cuadro, la evolución y antecedentes, cual es la condición secundaria que presenta el paciente al padecimiento de base? RESPUESTA a.- Cirrosis b.- Hepatitis Viral c.- Adenoma hepático d.- Hiperplasia adenomatosa PREGUNTA Considerando la presencia de los adenomas hepáticos, cual de las siguientes afirmaciones cual es menos frecuente? RESPUESTA a.- Un adenoma hepatocelular es un tumor frecuente y nocanceroso del hígado. b.- Los adenomas hepatocelulares inciden principalmenteen mujeres en edad fértil. c.- En general, no presenta síntomas, de modo que muchos de los casos no se llegan a detectar. d.- En casos moderados, una denoma puede hacerse canceroso. PREGUNTA Relacionado al carcinoma hepatocelular, cual de las siguientes afirmaciones es mas frecuente? RESPUESTA a.- Carcinoma hepatocelular en paciente con cirrosis. b.- Carcinoma hepatocelular multifocal en paciente sin cirrosis. c.- Un paciente con Sindrome de Bud-Chiari cronico con una lesion nodular sospechosa. d.- Mismo paciente con el uso de contraste, ayuda a delimitar mejor la lesion. CASO CLINICO Mujer de 26 años de edad. Acudió por un cuadro de una semana de evolución de dolor en hipocondrio derecho que se acompaña de astenia, náuseas, coluria, acolia, y febrícula. En la exploración física sólo destacaba la presencia de ictericia cutaneomucosa y un reborde hepático doloroso a la palpación a 4 cm de la arcada costal. PREGUNTA Cuál es factor etiológico más probable. RESPUESTA a.- Viral. b.- Alcohol. c.- Oncologico. d.- Autoinmune. CASO CLINICO Mujer de 77 años diabética, hipertensa y con hepatopatía crónica por el virus de la hepatitis C que consultó en Urgencias por cefalea de dos días de evolución con posterior aparición de confusión, desorientación, agitacion y fiebre de 39,2°C. Tos productiva, con campos pulmonares con ruidos crepitantes, FR 12, FC 92, TA 140/100 mmHg. PREGUNTA Cuál es la conducta a seguir mas adecuada. RESPUESTA a.- Evaluar enzimas hepáticas. b.- Evaluar funcionamiento renal. c.- Realizar medidas para encefalopatía. d.- Iniciar antibióticos de amplio espectro.

CURSO ENARM CMN SIGLO XXI TEL: 36246001

Pharmed Solutions Institute

PÁGINA 79

MANUAL DE TRABAJO DEL CURSO ENARM CMN SIGLO XXI CASO CLINICO Paciente de 40 años de edad, sexo masculino, con antecedente de depresión en tratamiento con psicoterapia. Consultó con cuadro de un mes de compromiso del estado general. Una semana antes de consultar presentó dolor en hipocondrio derecho, intermitente, que no cedía con la ingesta de analgésicos. Tres días antes de consultar presentó fiebre no cuantificada. FC de 110 lx', FR de 29 rpm, temp de 38,5°C, murmullo disminuido en la base pulmonar derecha y dolor a la palpación en hipocondrio derecho. Leucocitos 12.000 células/mm3, con neutrófilos de 78% y baciliformes de 1%; hemoglobina 12,6 g/dl; velocidad de eritrosedimentación 85 mm/h; proteína C reactiva (PCR) 239 mg/dl; bilirrubina total 0,34 mg/ di; bilirrubina directa 0,11 mg/dl; fosfatasa alcalina 151 U/L; tiempo de protrombina 71,9%; amilasa 44 U/L; lipasa 116 U/L. Se realizó ecotomografía abdominal que mostró una lesión de márgenes irregulares predominantemente hipoecogénica, con zonas hiperecoicas en su interior, de 14 cm x 7 cm, ubicada en el lóbulo hepático derecho, antígeno carcinoembrionario (ACE), 0,6 ng/ml; alfa feto proteína (AFP), 0,87 ng/ml, CA-19,9, 0,15 U/ml; anticuerpos IgM para hepatitis A, negativos; antíge-no de superficie de virus de hepatitis B, negativo; anticuerpos anti-hepatitis C, negativos; anticuerpos IgG e IgE para hidatidosis, negativos; anticuerpos IgG para amebiasis, negativos. PREGUNTA Considerando el cuadro clínico y los estudios de laboratorio y gabinete, cual es su impresión diagnostica inicial? RESPUESTA a.- Absceso Hepatico. b.- Quiste Hepatico. c.- Sarcoma Hepatico. d.- Hepatocarcinoma. PREGUNTA El paciente continuo con fiebre motivo por lo que se instalo tratamiento antibiótico, cual es la conducta diagnostica mas adecuada en este momento? RESPUESTA a.- Realizar Biopsia Percutanea. b.- Realizar Laparatomia Exploratoria. c.- Realizar Gamagrafia Hepatica. d.- Realizar Arteriografia Hepatica. PREGUNTA Considerando la respuesta previa se determino un HCTS, cual de las siguientes aseveraciones es mas probable para su desarrollo? RESPUESTA a.- La patogénesis de la transformación sarcomatoide del carcinoma hepático no ha sido clarificada. Sin embargo, la evidencia sugiere que las células fusadas del hepatocarcinoma representan una diferenciación sarcomatosa de las células epiteliales más que una combinación de sarcoma y hepatocarcinoma. b.- El mecanismo de degeneración, necrosis y regeneración de las células de carcinoma debidas a fármacos antineoplásicos o quimioembolización transarterial, han sido postulados como posibles inductores en otros casos. c.- Los trabajos previos muestran combinaciones con componentes condrosarcomatosos, rabdo-miosarcomatosos y con presencia de células gigantes tipo osteoclasto. d.-Los hepatocarcinomas en general, se desarrollan en hígados previamente dañados con cirrosis post virales y de tipo nutricioalcohólico. En el caso de los hepatocarcinomas con componente sarcomatoide. HEPATITIS: CIENCIAS BASICAS: Definición: Infección sistémica que afecta principalmente el hígado (inflamación). Causada por los virus hepatotroficos (A,B,C,D,E) que es la causa más común y por otros virus (VEB, CMV, coxackievirus, etc), alcohol, drogas, hipotensión e isquemia y enfermedades de la vía biliar. Hepatitis aguda: Enfermedad inflamatoria de hígado, de menos de 6 meses de evolución, por lo general produce anorexia, ictericia, coluria, y alteraciones en pruebas de función hepática. Hepatitis crónica: inflamación persistente de hígado, de más de 6 meses de evolución, puede desarrollarse de manera favorable o condicionar cirrosis hepática. Hepatitis fulminante: necrosis masiva del hígado, se manifiesta por encefalopatía hepática aguda, coagulopatía, insuficiencia renal y coma, aparece en una enfermedad de hígado con menos de 8 semanas de evolución. SALUD PUBLICA: 85% de los pacientes con hepatitis A, tienen recuperación clínica y bioquímica aprox. a los 3 meses. Cerca de 5% de la población mundial está infectada con VHB, en EU, es responsable de 5-10% de los casos de hepatopatía crónica y cirrosis. El riesgo de transmisión por punción con aguja es VHB se transmite en 30% de las exposiciones, VHC en 3% y VIH en 0.3%. La prevalencia mundial es de 0.5-2%, en México es de 1.4% La evolución natural de la infección por VHC es la hepatitis crónica activa (hasta 74%). El riesgo de presentar carcinoma hepatocelular en pacientes con cirrosis por VHC es de 1-4% por año. DIAGNOSTICO: Cuadro clínico: Manifestaciones comunes: Malestar, nausea, vómito, diarrea, fatiga, febrícula seguida de orina oscura, ictericia y hepatomegalia dolorosa puede ser subclínica y detectarse por niveles elevados de AST y ALT. Manifestaciones poco comunes: vasculitis, artritis, neuritis óptica, anemia aplasica, aplasia de serie roja, y mielitis transversa. HEPATITIS VIRALES AGUDAS Y CRONICAS: HEPATITIS A: El VHA, es picornavirus RNA no cubierto. Principal vía de transmisión fecal-oral ( alimentos contaminados, guarderías), población de riesgo son niños y adolescente, la infección no suele ser grave (autolimitada), seguido del proceso se producen anticuerpos contra el virus de la hepatitis A, lo cual confiere inmunidad. Incubación promedio de 30 días, la hepatitis se produce por daño secundario de la respuesta del sistema inmune del huésped mediada por daño citotóxico por linfocitos T CD8 y células NK. Puede presentarse desde asintomática hasta insuficiencia hepática aguda. DIAGNOSTICO: IgM anti-VHA

CURSO ENARM CMN SIGLO XXI TEL: 36246001

Pharmed Solutions Institute

PÁGINA 80

MANUAL DE TRABAJO DEL CURSO ENARM CMN SIGLO XXI en una muestra de suero de convalecientes aguda o temprana, en la exposición pasada se caracteriza por anticuerpos anti-IgG positivos con anti-IgM negativos (las cuales se negativizan a los 6 meses), elevación de ALT, AST, bilirrubinas y FA. TRATAMIENTO: Medidas de soporte, cuando hay complicaciones como falla hepática fulminante, valorar terapia intensiva y trasplante hepático. PREVENCION: Inmunización pasiva posexposición, inmunoglobulina humana 0.2 ml/Kg IM, protección por 6 meses, indicado en pacientes que planeen viajar a zona endémica, o contacto íntimo con alguien infectado con VHA. Inmunización activa: Vacuna contra VHA (1ml IM y refuerzo a los 6 y 12, 0.5ml para niños) indicado en pacientes con enf. hepática crónica, varones homosexuales, drogadictos, personal de salud que trabaje con VHA. HEPATITIS B: El VHB, es hepadnavirus DNA, está compuesto por la polimerasa con actividad de transcriptasa reversa, una proteincinasa rodeada por el antígeno del núcleo (HBcAg) y una envoltura que contienen el antígeno de superficie glucoproteico (HBsAg). El HVB puede causar acción citopatica directa y a través de respuesta inmune. Factores de riesgo son; contacto sexual, uso de drogas intravenosas (20%), transfusiones, contaminación con suero o leche materna, principal vía perinatal. Periodo de incubación de 40-150 días, puede presentarse desde asintomática hasta hepatitis fulminante (10 copias/ml, aumento de aminotransferasas persistentes, biopsia hepática que muestre inflamación. DIAGNOSTICO: HBsAg en suero infección aguda o crónica (si persiste por mas de 6 meses). IgM anti-HBc (indica infección aguda o reciente). La prueba más sensible para la detección de HVB DNA en suero (actividad viral replicativa en suero). Paciente con AntiHBs es una persona sana vacunada. Las transaminasas pueden aumentar hasta 1000 o 2000 y si permanecen por más de 6 meses se vuelve crónica. TRATAMIENTO: En aguda es de sostén, en la crónica, suprimir la replicación del virus con el fin de evitar progresión a cirrosis y cáncer (realizar tamizaje de alfafetoproteina y ultrasonido hepático cada 6 meses). El interferón alfa-2b, 5-10 millones de U, diarias, 3 veces a la semana SC, por 16 sem. Actualmente se usa alfa-2b pegilado, se aplica una vez por semana a una dosis de 180mg SC por 48 sem. La lamivudina (3TC), inhibe la transcriptasa reversa, disminuyendo la carga viral, otros adefovir y entecavir. PREVENCION: Vacunación administrar HBsAg, para estimular la producción de antiHBs, en individuos no infectados, efectividad >95%; vacunación universal en todos los recién nacidos, así como en trabajadores de la salud, paciente s en hemodiálisis, familiares y parejas sexuales de pacientes con HBsAg. Vacunación via IM a los 0, 1, 6 meses. HEPATITIS C: El VHC, es RNA monocatenario. Periodo de incubación de 2 meses. Factores de riesgo, uso de drogas IV, transfusión sanguínea antes de 1990, la transmisión vertical es infrecuente, está mas relacionada con una coinfección con VIH-1, en la madre. La transmisión sexual tiende a ser menos frecuente que otros virus (VIH), por la menor carga viral en líquidos y tejidos genitales. Las células blanco del VHC son los hepatocitos y lis linfocitos B. La sintomatología aparece aprox. De 6-8 sem de la exposición, puede pasar asintomático, ictérica, malestar general, nausea. La infección crónica se caracteriza por presentar periodos prolongados sin manifestaciones, la aguda progresara a crónica, con diversos grados de hepatitis, fibrosis y cirrosis; entre los factores que promueven la progresión clínica se observan el consumo de alcohol, coinfección con VIH-1 o VHB, género masculino, y edad mayor al momento de infección. Otras manifestaciones están asociadas a enf. autoinmuines (tiroideas, DM). DIAGNOSTICO: La prueba de rutina es la serológica mediante ensayo inmunoenzimático, que detecta anticuerpos a partir de 4-10 sem posteriores a la infección. El inmunoblot se usa como prueba confirmatoria. Los estudios moleculares se basa en detección del RNA viral mediante PCR, pueden ser cuantitativos (limite menor de detección 100 copias/ml) o cualitativos, la carga viral nos ayuda con la evolución y respuesta a tx. Los anticuerpos anti-VHC se pueden detectar en >97% de las personas a los 6 meses posteriores a la exposición. La biopsia es el estándar de oro para determinar la actividad de VHC, es el único predictor pronóstico de la progresión. TRATAMIENTO: Indicado en: carga viral positiva, biopsia hepática con fibrosis > grado 1 o enfermedad hepática compensada (CHILD A o B). Contraindicaciones: cirrosis descompensada, embarazo, enf. psiquiátrica, consumo activo de drogas, enf. pulmonar o cardiaca grave, DM descontrolada, enf. autoinmunes. Fármacos: interferón alfa en monoterapia o combinado con ribavirina; la combinación confiere 55% de respuesta viral sostenida. HEPATITIS D: El VHD, es RNA monocatenario, requiere la presencia del VHB para su supervivencia y replicación, la forma más común de contagio es por compartir agujas en personas que usan drogas IV, también transmisión sexual y perinatal. La coinfección aguda es la infección por exposición simultánea a VHB y VHD. La superinfección es la exposición del VHD, en un individuo ya infectado con VHB. El diagnóstico con IgM e IgG antiVHD o por la detección de RNA de VHD en suero. HEPATITIS AIUTOINMUNE: Sospechar ante una hepatitis que presente aumento de globulinas plasmáticas y autoanticuerpos específicos; es una enfermedad necroinflamatoria crónica del hígado de causa desconocida, parece haber predisposición genética (HLA-B8, DR3 y DR5). La hepatitis autoinmune se divide en 3 de acuerdo con los anticuerpos en suero: Tipo I: Clásica o lupoide, más común en mujeres jóvenes, 70% menores de 40 años, los anticuerpos que se distinguen son los antinucleares (ANA), antimusculo liso (AML), antiactina y antineutrofilos (pANCA) e hipergammaglobulinemia. Tipo II: Más común en Europa, en especial en niños, aquí encontramos annticuerpos anticromosomas de hígado y riñón tipo 1 (anti-LKM1), asociada a enf. inmunológicas, tiende a progresar rápido a cirrosis. Tipo III: 90% mujeres entre 20-40 años, se observa anticuerpo antiSLA/LP, amplia relación con tiroiditis autoinmune. La hepatitis autoinmune es de progresión lenta, como hepatitis aguda además, múltiples telangiectasias, estrías, cutáneas, acné, hirsutismo, hepatomegalia, artralgias y amenorrea. Para tx se usan glucocorticoides de síntesis (prednisona, metilprednisolona), también aziatropina en biterapia con cortoicoesteroides (éxito de 80-90%). HEPATITIS ALCOHOLICA: Se presenta con niveles de aminotransferasas 2.1 y GGT elevada, y deficiencia concomitante de vitamina B12, los gravemente afectados presentan al inicio fiebre, hepatomegalia, leucocitosis, ictericia y coagulopatías, asi como manifestaciones de hipertensión portal, es común que desarrollen cirrosis. El pronóstico se ve en particular afectado por la presencia de elevadas concentraciones de bilirrubina (>12mg/dl), ascitis e insuficiencia renal. Tx.: indicado en pacientes con encefalopatía, administración de prednisona, pentoxifilina por un mes disminuye mortalidad e incidencia de sx. hepatorrenal. TOXICIDAD POR PARACETAMOL: Se presenta con dosis mayores de 10 gr, puede presentarse en pacientes con desnutrición o alcoholismo con 2-6 gr, parece proveer un reservorio de grupos sulfidrilo y esto obliga a los metabolitos tóxicos o estimulación de la síntesis de glutatión hepático si la falla hepática se presenta en las primeras 4 hrs, el tx.: consiste en la administración de carbón activado o colestiramina y lavado gástrico. La N-acetilcisteina debe ser iniciado dentro de las primeras 8 hrs de la ingestión, pero puede ser efectivo incluso en las 24-36hrs después de la sobredosis, se prefiere vía oral con una dosis carga de 140mg/Kg y después 70mg/Kg cada 4 hrs por 17 dosis. Otros medicamentos que producen hepatitis son: amiodarona. Azoles, isoniacida, metildopa, fenitoina, rifampicina, sulfas. Hay que observar al paciente de 72-96hrs para evaluar daño hepático.

CURSO ENARM CMN SIGLO XXI TEL: 36246001

Pharmed Solutions Institute

PÁGINA 81

MANUAL DE TRABAJO DEL CURSO ENARM CMN SIGLO XXI CASO CLINICO Varón de 33 años no fumador con el único antecedente personal de hipercolesterolemia a tratamiento con dieta. Realiza ejercicio físico de forma regular y su trabajo es sedentario. Refiere malestar general acompañado de cefalea, astenia y fiebre vespertina de hasta 39,2 ºC, acolia y coluria. En la EF no se observa ictericia, se auscultan niveles hidroaereos aumentados, Refiere haber comido ostras vivas una semana antes, las PFH se encuentran en parámetros elevados, mas un patrón colestasico, se realiza serología resultado positivo a virus a hepatitis A (IgM positiva e IgG negativa). PREGUNTA Cual es la complicación mas frecuente observar en esta patologia? RESPUESTA a.- Hepatitis recidivante. b.- Hepatitis colestásica. c.- Hepatitis aguda fulminante. d.- Síndrome post-hepático. PREGUNTA Considerando la fisiopatogenia del caso, en cuanto tiempo se puede eliminar el virus por las heces? RESPUESTA a.- Una semana antes de la clínica. b.- Durante la presentación de la clínica. c.- Una semana después de la clínica. d.- dos semanas después de la clínica. PREGUNTA Habitualmente la acolia y la coluria se presenta cuantos días antes de la ictericia? RESPUESTA a.- 1 a 5 dias. b.- 6 a 10 dias. c.- 11 a 15 dias. d.- 16 a 20 dias. CASO CLINICO Hombre de 27 años sin antecedentes mórbidos, consultó por compromiso del estado general y fiebre de 5 días, con posterior aparición de ictericia. Se hospitalizó con bilirrubina total de 13 mg/dl, aminotransferasa oxalacética de 1.977 mU/ml y pirúvica de 5.975 mU/ml, fosfatasas alcalinas de 157 U/L, y tiempo de protrombina 15%. Del panel viral resultó positivo un HBsAg. Evolucionó con compromiso de conciencia hasta Glasgow 8, con tomografía computada (TC) de cerebro normal. Fue recibido en la Unidad de Cuidados Intensivos con Glasgow 5 sospechándose broncoaspiración. Se conectó a ventilación mecánica, se iniciaron antibióticos y N-acetil cisteína. La TC de tórax reveló neumonía, y la de cerebro edema cerebral leve. La ecotomografía abdominal muestró hígado normal y ascitis leve. Dentro del estudio etiológico presenta serología para virus hepatitis C y A y estudio de autoinmunidad (anticuerpos y recuento de inmunoglobulinas) negativos. Se repitió el estudio del VHB destacando aparición de anticuerpos contra el HBsAg (antiHBs) y negativización del HBsAg, con anticore positivos, IgM y total. Se realizó ELISA para VIH, que resultó positivo, y recuento de CD4 de 634 células/ul. La carga viral (CV) del VHB fue de 140 copias/ml, y la del VIH 16.900 copias RNA/ml. 12 dias después fallece el paciente. PREGUNTA Cual de las siguientes observaciones sobre el presente caso clínico explica mas adecuadamente la complicacion del paciente? RESPUESTA a.- Falla Hepatica Fulminante por manejo inoportunamente diagnosticada y tratada. d.- La falla hepatica por VHB con HIV ocurre en aproximadamente 20 %. c.- En la falla hepática, mientras mayor es la respuesta del hospedero mayor es su probabilidad. d.- En estados de inmunodepresión como es el tener VIH avanzado, el desarrollo de Falla Hepatica sea infrecuente. PREGUNTA Considerando la evolución del caso, cual es el periodo necesario para considerar falla hepática fulminante. RESPUESTA a.- Dentro de las 72 horas. b.- Dentro de la primera semana. c.- Dentro de las primeras 8 semanas. d.- Dentro de las primeras 12 semanas. PREGUNTA De las siguientes observaciones relacionadas a esta patologia cual es la menos probable?

CURSO ENARM CMN SIGLO XXI TEL: 36246001

Pharmed Solutions Institute

PÁGINA 82

MANUAL DE TRABAJO DEL CURSO ENARM CMN SIGLO XXI RESPUESTA a.- Mexico se encuentra dentro de los países de baja seroprevalencia. b.- Se estima que hay almenos de 110,000 portadores crónicos. c.- El VIH tiene una mayor virulencia que el VHB. d.- La infección crónica por el VHB después de una exposición aguda es del 5 %. PREGUNTA La infección crónica por el VHB, tiene cuatro fases, de acuerdo a la presencia o ausencia del HBeAg, carga viral, nivel de ALT y hallazgos histológicos, cual de las siguiente no es correcta? RESPUESTA a.- Inmunotolerancia. b.- Portador activo. c.- Inmunoeliminacion. d.- Hepatitis crónica HBeAg negativa. CASO CLINICO Masculino de 70 años con hepatitis crónica por el VHC genotipo 1 de cronología incierta, infección pasada por el VHB y bebedor de unos 60 gramos de alcohol al dia. Se observo elevación de GGT y FA, en la ecografía/TAC abdominal convencional en fase venosa se puso de manifiesto una tumoración de 7 cm de características solidas, en lóbulo hepático izquierdo, sobre el hígado de morfología normal, la biopsia revelo que se trataba de un CHC, sin presencia clínica ni paraclinica de cirrosis previa y el valor de la alfafetoproteina era de 6.9, se realizo tratamiento QX. PREGUNTA Considerando el caso clínico cual es la tasa de sobrevida al año del presente caso? RESPUESTA a.- del 15 %. b.- del 45 %. c.- del 75 % d.- del 95%. PREGUNTA Relacionado a la patologia descrita en el caso previo, cual de las siguientes aseveraciones no es cierta? RESPUESTA a.- La hepatitis C es una hepatopatía crónica que cursa silenciosa y tiene una prevalencia de 1 a 1.9 en México. b.- La muerte por infección crónica de VHC están esencialmente relacionadas con la descompensación hepática, enfermedad hepática terminal y el carcinoma hepatocelular. c.- Las pruebas serológicas actuales permiten distinguir entre infección aguda, crónica o resuelta, por lo cual se puede llevar un buen monitoreo. d.- La infección por el VHC se ha situado como la principal causa de hepatopatía crónica, cerca del 27 % de los casos de cirrosis y el 25 % de los casos de hepatocarcinomas. PREGUNTA Cual de las siguientes aseveraciones no es correcta en el diagnostico de infección por VHC? RESPUESTA a.- Durante la fase aguda de la infección la dereccion de acido nucleico puede generar inadecua interpretación de resultados. b.- Con una prueba anti-VHC positiva se debe realizar prueba PCR-RNA-VHC. c.- Se debe solicitar prueba de RNA del VHC en los pacientes que se esta considerando la administración de tratamiento antiviral. d.- Se debe solicitar una prueba anti-VHC en pacientes con VIH para determinar comorbilidad. CASO CLINICO Mujer de 62 años que refiere, desde hace 13 años, múltiples episodios de visión borrosa en ambos ojos con recuperación posterior completa de la agudeza visual tratada con esteoides, TA 100/70mmHg, Glucosa 84 mg/dl, con antecedente de una hermana con el mismo cuadro, presión intraocular normal, inicia cuadro actual con dispepsia, vomito, prurito, hiporexia, astenia, GOT 110, GPT 90, GGT 110, hipergammaglobulinemia (IgG 1620), ANA 1/320, AMA negativos, Antigeno de superficie VHB, Anticuerpo anti HBs, Anticuerpo anti-HBc y Anticuerpo anti-VHC negativos, USG hepático normal. PREGUNTA Considerando las manifestaciones clínicas asi como los estudios de laboratorio y gabinete, cual es la conducta mas adecuada? RESPUESTA a.- Repetir estudios serológicos más sensibles y específicos. b.- Realizar biopsia hepática percutánea.

CURSO ENARM CMN SIGLO XXI TEL: 36246001

Pharmed Solutions Institute

PÁGINA 83

MANUAL DE TRABAJO DEL CURSO ENARM CMN SIGLO XXI c.- Mantener en observacion y repertir estudios en 6 meses. d.- Manejo conservador y corticoides. PREGUNTA Considerando la respuesta anterior, los antecedentes familiares y personales cual es el diagnostico mas probable? RESPUESTA a.- Hepatitis B. b.- Hepatitis A. c.- Hepatitis autoinmune. d.- Hepatitis Ideopatica. CASO CLINICO Masculino de 64 años con historia de multiples ingresos por alteraciones del estado de conciencia debido a insuficiencia hepática y atrofia cortical, acude 14 dias después su ultimo internamiento por alteración del estado de conciencia, por fiebre, desorientación y dolor abdominal, urea 103, creatinina 2,6. LDH 699, CPK 864, GOT 57, GPT 46. La gasometría revela una acidosis respiratoria. En la radiografia se observa torax enfisematoso e importante cardiomegalia sin signos neumónicos ni deramme pleural. PREGUNTA Se diagnostico como neumonía, considerando el caso, cual de los siguientes agentes es mas frecuente? RESPUESTA a.- Streptococcus neumonae. b.- Stafilococcus aureus. c.- Listeria monocitogenes. d.- Campylobacter jejuni. CASO CLINICO Masculino de 54 años con DM, artrosis lumbar y hepatopatía alcoholica, acude por dolor abdominal generalizado, fiebre, perdida de 20 kg en 6 meses. Leucocitosis a expensas de neutrofilos, acompañado de trobocitosis, glucosa 263, urea 185, creatinina 1,2. LDH 204 y PCR 358. La radiografia de torax no muestra imágenes de condensación. PREGUNTA Se diagnostico como neumonía, considerando el caso, cual de los siguientes agentes es menos frecuente? RESPUESTA a.- Streptococcus neumonae. b.- Stafilococcus aureus. c.- Listeria monocitogenes. d.- Campylobacter fetus. CASO CLINICO Masculino de 18 años de edad que consulta de rutina, los laboratorios de rutina reportaron elevación de GOT de 56 y el GPT de 78 las cuales fueron incrementando lentamente desde hace 16 meses. GGT estaban discretamente aumentada y el resto de parámetros hepáticos eran normales. No había serología positiva a virus de la hepatitis B y C. Los autoanticuerpos para descartar hepatopatía autoinmune eran negativos. No hay antecedentes de de hemotransfusion ni quirúrgicos. Hace ejercicio y como único antecedente refiere tomar finasterida 1 mg diario para evitar la alopecia desde hace 2 años. PREGUNTA Cual es la conducta mas apropiada a seguir? RESPUESTA a.- Retirar finasterida. b.- Realizar USG hepático. c.- Realizar biopsia hepática. d.- Repetir serología. CASO CLINICO Varón de 60 años con antecedentes de hipertensión arterial, adenoma de próstata e hiperuricemia ocasional. Había sido diagnosticado de miocardiopatía dilatada idiopática hacía 5 años y seguía tratamiento con digoxina, ibopamina, quinapril, espironolactona, hidralacina, dicumarina, furosemida y nitratos transdérmicos, con lo que se mantenía en CF-II. Presentaba además fibrilación auricular paroxística por lo que esporádicamente había recibido AMD por vía oral (200 mg/día). No tenía antecedentes de enolismo. Nunca se detectaron alteraciones de la función hepática. Ingresó por un cuadro de taquicardia ventricular sostenida a 190 por minuto con morfología de bloqueo de rama derecha. La arritmia cursó con regular tolerancia clínica y sin constatarse en ningún momento signos de compromiso hemodinámico (hipotensión, oliguria, etc.). Se había restablecido el ritmo sinusal por medio de un bolo de 100 mg de lidocaína y se había instaurado tratamiento con AMD intravenosa a dosis de 1.200 mg/día durante los 7 días previos. A su llegada a la

CURSO ENARM CMN SIGLO XXI TEL: 36246001

Pharmed Solutions Institute

PÁGINA 84

MANUAL DE TRABAJO DEL CURSO ENARM CMN SIGLO XXI unidad coronaria el paciente presentaba estabilidad hemodinámica con presión arterial de 120/80 mmHg y frecuencia cardíaca irregular a 90 por minuto con pulsos normales. Se apreciaban a la auscultación cardíaca un tercer sonidos y un débil soplo holosistólico en foco mitral. No había edemas, ingurgitación yugular o alteraciones en los ruidos respiratorios. Destacaba especialmente una ictericia franca en piel y conjuntivas, y una evidente depresión del nivel de conciencia con flapping-tremor. No se apreciaron hepatomegalia, ascitis ni ningún estigma de hepatopatía crónica. En el electrocardiograma se observaba fibrilación auricular a 90 por minuto con bloqueo de rama izquierda. La radiografía de tórax presentaba cardiomegalia y signos de hipertensión venocapilar pulmonar. En la analítica al ingreso se observaba como único dato destacable una alanina-aminotransferasa (ALT) de 52,4 µkat/l (normal: 0,46-0,77). PREGUNTA Cual es la conducta mas apropiada a seguir? RESPUESTA a.- Retirar amiodarona. b.- Realizar USG hepático. c.- Realizar biopsia hepática. d.- Repetir serología LINFOMAS CIENCIAS BASICAS: Todas las neoplasias linfoides son malignas y derivan de una célula que ha sufrido mutación maligna son monoclonales, también alteran la inmunidad. 80-85%, tiene su origen en células B, los derivados de NK o histiociticas son raros. Se clasifican en Linfoma de Hodgking (EH) y linfomas no Hodgkin (LNH). SALUD PUBLICA: LH: Constituye el 10% de todos los linfomas. Más frecuente de 25 años y >60 años. LNH constituyen el 3% de todas las neoplasias y son más frecuentes que Los LH (60% de los linfomas en adultos, edad promedio 64 años) El subtipo histológico más común es el difuso B de cel. Grande (LDGBC)s con incidencia en individuos infectados por el VIH de más de 100 veces la incidencia en la población en geral (25%), seguido del linfoma folicular. LINFOMA DE HODGKIN (LH): Origen en cel. B activados del centro germinal. El de celularidad mixta es el subtipo histológico más frecuente en LH extranodales, mientras que el subtipo esclerosis nodular es predominante en los de afectación nodal. PATOGENIA: Se sospecha del VEB como agente de transformación maligna, pacientes con mononucleosis infecciosa, tienen mayor riesgo, se reconoce el genoma de VEB en 70%, también común en casos de inmunodeficiencia. Características; extensión ordenada, localizado, rara vez afecta ganglios mesentéricos, rara extensión extraganglionar, muy común dolor en ganglios linfáticos, después del consumo de alcohol (38°C, de duración mayor a 2 semanas; sudoración nocturna profusa. Prurito en 10-15% de los casos, no es síntoma B. Fiebre Pel- Ebstein; fiebre cíclica que dura de 3-5 días, seguida de un periodo equivalente sin fiebre. Biopsia excisional ganglionar cel de Reed Sternberg 2 o más núcleos con nucléolo prominente. Inmunohistoquimica: positivos para CD15 y 30, nos ayuda a diferenciar entre LNH. PCR en ganglio detecta VEB en 60-80%. Tac de tórax, abdomen y pelvis, PET. ESTADIFICACION: La clasificación mas usada es la Ann Arbor- Costwold. ESTADIO I

Afecta una sola región de ganglios linfáticos Afecta a un solo órgano extralinfático ESTADIO II Afecta 2 o mas áreas ganglionares en el mismo lado del diafragma Afecta a un solo órgano extralinfático y sus ganglios regionales al mismo lado del diafragma ESTADIO III Afecta áreas ganglionares a ambos lados del diafragma Puede acompañarse de afección esplénica Órganos extralinfático localizados ESTADIO IV Afecta de forma difusa a uno o más órganos extralinfático (hígado, pulmón, medula ósea) con o sin involucro ganglionar Agregar A cuando no se presentan síntomas y B si la persona refiere síntomas B

I IE II IIE

IIIS IIIE

TRATAMIENTO: Radioterapia en estadios tempranos sin factores desfavorables. Quimioterapia; tratamiento de elección para casos localizados (IA y IIA), 2 ciclos de quimioterapia con ABVD (adriamicina, vinplastina, bleomicina, y decarbazina)+ radioterapia en caso de urgencias avanzadas ( sx. de vena cava superior, obstrucción de vía aérea). Estadios III y IV quimio combinada. PRONOSTICO: Factores de mal pronóstico: Hb 15,000, linfocitosis 53%. Tasa de curación 65%, para pacientes con trastornos avanzados LINFOMA NO HODGKING (LNH): Tumores sólidos malignos de los tej. Linfoides, suelen provenir de cel. B (80%) Diseminación menos previsible, generalmente diseminada al realizar el dx., suele afectar ganglios mesentéricos, frecuentemente afecta tej. extraganglionar. Entre los factores de riesgo están ; inmunodefoiciencias (LNH sistémico puede afectar el miocardio, particularmente en pacientes inmunocomprometidos), tratamientos con radioterapia o quimioterapia, enfermedades autoinmunes y enfermedades previas. Los linfomas no hodgkiniano son la causa más frecuente de linfoma de cabeza y cuello. Se clasifican en bajo grado (transformación en agresivos, no curables, edad avanzada, sobrevida larga a 8 años) y agresivos: potencialmente curables, sobrevida corta si no remiten; cuadro clínico, síntomas B crecientes. Dentro de los de bajo grado; linfoma folicular, sus células neoplásicas se parecen a cel. B normales, adenopatías indoloras generalizadas, incurables. LNH de bajo grado Linfoma de la zona marginal (MALToma= tumores de los tej. linfoides asociados a mucosas) se origina, en bazo, ganglios y tej. extraganglionares, afectados por procesos inflamatorios autoinmunitarios. LNH agresivo en general, debilidad, fatiga, pérdida de peso, 50% pérdida de peso, hepato y esplenomegalia, hiperviscosidad (que genera mareos, sordera, cefalea, formación de complejos y crioglobulinemia) linfoma difuso de cel. B grandes; afecta una sola región ganglionar o extraganglionar, primera manifestación tubo digestivo, piel, hueso cerebro. LNH agresivo Linfoma de Burkit; tumor de cel. B relativamente maduras, translocación del gen c- myc en cromosoma 8, en tej. extraganglionar, el LB africano

CURSO ENARM CMN SIGLO XXI TEL: 36246001

Pharmed Solutions Institute

PÁGINA 85

MANUAL DE TRABAJO DEL CURSO ENARM CMN SIGLO XXI endémico; masa en mandíbula, vísceras abdominales (riñones, ovarios), agresivo pero responde a quimioterapia fácil. DIAGNOSTICO: Sospecha clínica, biopsia de nódulo o órgano involucrado. Estos se diagnostican por resultados de laboratorios anormales; citopenias, hipercalciemia, hiperuricemia, proteinemia o LDH elevada) TRATAMIENTO: LNH; linfoma folicular R-CHOP (rituximab, ciclofosfamida, hidrixidaunomicina, vincristina y prednisona). LNH agresivos; R-CHOP más de 50% de curación. En todos los linfomas en casos refractarios trasplante de medula ósea. PRONÓSTICO:. LNH agresivos; tasa de remisión de 60-80% , para estadios II al IV. PREVENCION: Hombre almacenar espermas, mujeres no embarazarse en 2 años posteriores. CASO CLINICO Una paciente de 27 años, del sexo femenino, se presentó en la Sala de Emergencias con palpitaciones de inicio súbito y disconfort torácico. La presión arterial (PA) era de 68 x 40 mmHg y la frecuencia cardíaca (FC) era de 184 lpm. Al examen físico, la paciente presentaba aumento de volumen de la mama izquierda y masa abdominal difusa. El electrocardiograma (ECG) de 12 derivaciones realizado en la admisión reveló taquicardia de complejo ensanchado con estándar de bloqueo de ramo izquierdo y concordancia negativa en los electrodos precordiales. Con base en la presentación inicial, el diagnóstico de taquicardia ventricular monomórfica fue establecido y la paciente fue sometida a cardioversión eléctrica para el ritmo sinusal normal. El ECG fue repetido y mostró ritmo sinusal normal, sin alteraciones del segmento ST (Figura 1B). La paciente recibió una infusión intravenosa de amiodarona y entonces fue admitida en la Unidad de Terapia Intensiva (UTI). La investigación fue negativa para isquemia miocárdica o embolia pulmonar. Una ecocardiografía transtorácica (ETT) reveló funciones ventriculares izquierda y derecha normales, derrame pericárdico posterior difuso y engrosamiento del miocardio basal-septal, que no estaba presente en examen anterior realizado un año antes para investigación de disnea durante actividad física. PREGUNTA Cuál de las siguientes condiciones patológicas previas es más probable para el desarrollo del LNH (LDGCB)? RESPUESTA a.- Carcinoma metastasico. b.- Carcinoma pulmonar. c.- Antecedente de VIH. d.- Lupus eritematoso sistémico discoide. PREGUNTA ¿En pacientes con LNH, cual es la presentación mas común de compromiso cardíaco? RESPUESTA a.-Dolor torácico B.-Insuficiencia cardíaca congestiva C.-Derrame pericárdico D.-Bloqueo atrioventricular PREGUNTA ¿Cuál es el manejo inicial de la taquicardia ventricular en este caso? RESPUESTA a.- Cardioversión eléctrica b.- Verapamilo c.- Amiodarona d.- Desfibrilación CASO CLINICO Varón de 39 años con antecedentes de amigdalectomía en infancia, sin hábitos tóxicos referidos y abuela materna con leucemia linfática crónica. El paciente es visto en consulta por insuficiencia respiratoria nasal y ronquido asociado a fatiga diurna, más acentuada en el último año, sin otra clínica asociada. Es trabajador a turnos y conductor profesional por lo que se solicita interconsulta en la Unidad de Trastornos Respiratorios del Sueño, con diagnóstico de trastorno de sueño leve. En la exploración se visualiza, por rinoscopia anterior, una desviación septal derecha con hipertrofia turbinal compensatoria. En nasofibroscopia, se identifica una masa en cavum, compatible con hipertrofia adenoidea. La rinomanometría demuestra mayor resistencia aérea en fosa nasal derecha, sin gran mejoría tras vasoconstrictor, siendo el resto de la exploración ORL normal. Dado el agravamiento de la clínica en el último año y el tamaño de la masa en cavum, se realiza biopsia de la misma, informada como hiperplasia folicular linfoide, y RM, objetivándose una masa de 2,3cm de diámetro compatible con hipertrofia de tejido adenoideo. Teniendo en cuenta la clínica referida y la exploración realizada, el paciente es intervenido de septoplastia con turbinectomía y adenoidectomía. El informe AP es de linfoma de Hodgkin (LH) clásico rico en linfocitos, sin detectar presencia de virus Epstein Barr (VEB). PREGUNTA ¿Cuál es la manifestación clásica de Linfoma Hodgking? RESPÚESTA a.- Paciente joven, masculino, linfadenopatías periféricas. b.- Adulto mayor, masculino, afección extraganglionar

CURSO ENARM CMN SIGLO XXI TEL: 36246001

Pharmed Solutions Institute

PÁGINA 86

MANUAL DE TRABAJO DEL CURSO ENARM CMN SIGLO XXI c.- Paciente joven, femenino, linfadenopatías periféricas d.- Paciente joven, masculino, afección extraganglionar PREGUNTA ¿Qué utilidad tiene hacer estudio inmunohistoquimico a este paciente? RESPUESTA a.- Decidir tratamiento b.- Diferenciar LNH bajo grado de un LNH agresivo c.- Establecer un diagnóstico diferencial con el linfoma no hodgkiniano d.- Saber si es un tumor en cabeza y cuello PREGUNTA El paciente descrito presenta un LH, sin síntomas B, localizado en adenoides, sin afectación de otras regiones ni adenopatías asociadas. ¿ En qué estadio de la clasificación de Ann Arbor- Costwold se encuentra? a.- Estadio IV b.- Estadio III E-A c.- Estadio IE-A d.- Estadio IE-B CASO CLINICO Una paciente de 27 años, del sexo femenino, se presentó en la Sala de Emergencias con palpitaciones de inicio súbito y disconfort torácico. La presión arterial (PA) era de 68 / 40 mmHg y la frecuencia cardíaca (FC) era de 184 lpm. Al examen físico, la paciente presentaba aumento de volumen de la mama izquierda y masa abdominal difusa. El ECG reveló taquicardia de complejo ensanchado con estándar de bloqueo de ramo izquierdo, fue sometida a cardioversión eléctrica para el ritmo sinusal normal. El ECG fue repetido y mostró ritmo sinusal normal, sin alteraciones del segmento ST. La paciente recibió amiodarona. Una ecocardiografía transtorácica reveló derrame pericárdico posterior difuso y engrosamiento del miocardio basal-septal. VHI positiva y SIDA con CD4 de 39 células/mm3. El diagnóstico de linfoma difuso de grandes células B (LDGCB), un linfoma no Hodgkin fue el diagnostico definitivo. PREGUNTA Cuál es el medio diagnostico mas adecuado. RESPUESTA a.- Biopsia. b.- IRM-f c.- Frotis periférico. d.- BH. CASO CLINICO Se trata de masculino de 21 años de edad el cual acude a consulta externa por cuadro faríngeo, se observa odinofagia, rinorrea hialina con descarga retrofaringea, amígdalas hiperemicas e hipertróficas, adenopatía cervical y axilar, al interrogatorio que presenta frecuentemente cuadros infecciosos, pero en la actualidad refiere que desde hace 5 meses ha presentado fiebre intermitente, ataque al estado generalizado, fatiga, adinamia, hiporexia, con pérdida de peso del 10 % aproximadamente, además de prurito generalizado. PREGUNTA Cuál es la condición más probable en este caso. RESPUESTA a.- Considerando la edad es más frecuente la mononucleosis infecciosa. b.- Lo más probable es Leucemia Linfoblastica Aguda. c.- Lo más frecuente es Leucemia Linfocitica Aguda. d.- Considerando el cuadro Linfoma de Hodgkin. CASO CLINICO Varón de 69 años de edad con antecedentes personales de LMA M1 (mieloblástica sin diferenciación), en remisión completa (RC) desde hace 11 meses tras tratamiento quimioterápico según protocolo PETHEMA-LAM 99. Ingresa por cuadro respiratorio catarral, con importante congestión nasal, sin respuesta a antibiótico. Presenta, a su vez, pápulas eritematosas en espalda, adenopatías supraclaviculares y submaxilares. Presenta ademas ojo rojo de dos días de evolución, sin otra sintomatología ocular. En la exploración presentaba una agudeza visual con corrección de 20/20 en OD y 18/20 en OI. Los reflejos, motilidad y tonometría oculares se encontraban dentro de la normalidad. En la biomicroscopía de polo anterior se observó en OD una lesión carnosa sobreelevada de color salmón en el tarso superior e inferior. En el OI se encontró una lesión similar en conjuntiva tarsal inferior. En el fondo de ojo no se apreciaron alteraciones. PREGUNTA Cual es la conducta mas apropiada a seguir?

CURSO ENARM CMN SIGLO XXI TEL: 36246001

Pharmed Solutions Institute

PÁGINA 87

MANUAL DE TRABAJO DEL CURSO ENARM CMN SIGLO XXI RESPUESTA a.- Biopsia de lesión. b.- Busquedad de marcadores. c.- Administracion de esteroides tópicos. d.- Punsion medular. PREGUNTA Sobre la patologia anterior descrita, cual de las siguientes afirmaciones es menos frecuente? RESPUESTA a.- La afectación ocular por la leucemia es bien conocida, b.- Los más frecuente es la hemorragia retiniana. c.- Es debida a la leucocitosis. d.- La retina es, a su vez, la estructura ocular más frecuentemente afectada de forma directa por células leucémicas, hasta en un 30%. CASO CLINICO Paciente de sexo femenino de 32 años de edad que consulta por presentar equimosis, hematomas generalizados, gingivorragia e hipertrofia gingival, astenia, adinamia y síndrome febril. Tras un laboratorio, se asume a la paciente como Leucemia Aguda. Se realiza punción-aspiración de médula ósea, cuyo análisis citogenético es compatible con una Leucemia Mieloide Aguda de subtipo M3. Se inicia el tratamiento quimioterápico con Mitoxantrona y Tretinoina, farmacoprofilaxis con Dexametasona y Omeprazol. A los 20 días de iniciado el dicho tratamiento, la paciente desarrolla Síndrome de Diferenciación. PREGUNTA Cual de los siguientes mecanismos fisiopatológicos no intervienen en el síndrome de diferenciación presente en el caso? RESPUESTA a.- Respuesta inflamatoria sistémica. b.- Daño endotelial con síndrome de fuga capilar. c.- Obstrucción de la microcirculación que produce infiltración tisular. d.- Disminución de la presión oncotica sobre la hidrostática. CASO CLINICO Varón de 14 años, acude por esplenomegalia. Historia pre-hospitalaria de palidez de piel y mucosas, de moderada intensidad, de un mes de evolución, fiebre de 38ºC sin escalofríos ni sudoración, dolor en el pecho y vómito de contenido alimentario con estrías de sangre, 24hs antes del ingreso. Al examen físico presenta petequias dispersas en ambas piernas y pápulas en miembro inferior derecho. El laboratorio informa Leucocitos: 25.200/uL con 68% de blastos, Hemoglobina: 6,1g/dL, Hematocrito: 17%, Plaquetas: 31.000/uL. PREGUNTA Cual es la conducta diagnostica mas adecuada? RESPUESTA a.- Aspirado de medula osea. b.- Gamagrama oseo. c.- IRM. d.- Marcadore tumorales. FIEBRE REUMATICA (FRA): CIENCIAS BASICAS: Enfermedad inflamatoria ocurre como secuela alejada de una infección por estreptococo beta hemolítico grupo A, habitualmente faringoamigdalitis o escarlatina. Su nombre proviene del daño articular, pero los daños más importantes se producen en corazón. SALUD PÚBLICA: La FRA aparece de 0.5-5% de pacientes que hacen faringoamigdalitis estreptocócica. Aparece especialmente entre 5-15 años. Mortalidad T CD8 y monocitos afectación musculo-esquelética variable. Exploración física: desviación Manifestaciones generales por citocinas secretadas cubital, nódulos de Bouchar (interfalangicas proximales), sinovitis, por los macrófagos dedos en cuello de cisne o botonero, hallux valgus (miembros inferiores), hipotrofia en músculos interóseos en manos (por vasculitis), Formación de tejido de disminución de fuerza prensil, derrame bilateral de rodillas de granulación por activación de fibroblastos (pannus) con Destrucción articular y predominio izq. (complicación frecuente de AR, el quiste de Barker en hiperplasia de células móviles ósea por citocinas hueco poplíteo asociado a ruptura e invasión muscular). formadas en el pannus. Manifestaciones extraarticulares: piel; nódulos reumatoides (25-50%). Hematológicos; anemia normocitica normocrómica (35-30%), trombocitosis (>50,000), trombocitopenia, linfadenopatía. Hepático; transaminemia. Pulmón (30%), dolor pleurítico (25%), nódulos pulmonares, arteritis pulmonar, enfermedad pulmonar intersticial. Ojos; queratoconjuntivitis Sicca (35%), ojo seco, epiescleritis, uveítis, queratitis ulcerativa. Corazón; pericarditis, enf. Ateroesclerótica avanzada. Neurológico; neuropatía periférica, síndrome del túnel del carpo. Renal; nefropatía Laboratorio: Factor reumatoide (FR=presente hasta 65%): Auto-anticuerpos dirigidos contra la porción de Fc de la IgG tiene sensibilidad de 70%, especificidad 80%; también presente en otros procesos infecciosos crónicos e inmunitarios (Tb, endocarditis, neoplasias, lupus esclerodermia). Nos indica artritis seronegativa o seropositiva esta última se relaciona con forma más agresiva de la enfermedad. Autoanticuerpos anti-péptido cíclico citrulinado (Anti-Pcc =presente hasta 95%), sensibilidad de 70% y especificidad 90-95%, al igual que FR se relaciona con formas más agresivas de la enfermedad. VSG PCR elevadas. Anti-DNA negativo (especifico para (LES). Radiológico (PA de mano): Inflamación de articulaciones metacarpofalángicas, desviación cubital, pulgar en Z, dedos en cuello de cisne, erosiones óseas y osteopenia yuxtaarticular, luxaciones metacarpofalángicas proximales. CRITERIOS DIAGNOSTICOS: 1. Rigidez matutina al menos de 1 hora de la mejoría máxima. 2. Artritis de 3 o más articulaciones. 3. Artritis de las manos. 4. Artritis simétrica. 5. Nódulos reumatoideos (sobre prominencias óseas, superficies extensoras o regiones yuxtaarticulares). 6. Factor reumatoide positivo. 7. Cambios radiológicos. El diagnostico se hace con 4 de 7 criterios. TRATAMIENTO: AR activa leve: hidroxicloroquina (HCQ) o sulfazalazina (SSZ). AR moderada a severa: Considerar dosis bajas de prednisona 5-10mgs e iniciar metotrexate (MTX) 10mg, se puede incrementar hasta 25mg/sem. Si hay una respuesta inadecuada se puede agregar otro FARME (SSZ, HCQ, d-penicilamina)los cuales previenen el daño erosivo , terapia bilógica (abatacep, etarnecep), anti TNF. Inmunosupresores (aziatropina, leflunomide, ciclosporina). Cirugía puede ser considerada en una deformidad funcional severa. La prednisona al igual que los AINEs se puede dar como medidas de control, pero no modifican el curso de la enfermedad. El metotrexate es la piedra angular del tratamiento de la AR (por ser un inductor de la remisión). SINDROMES ESPECIFICOS EN PACIENTES CON AR: Síndrome de Sjogren secundario= AR + Sicca (xerostomía-xeroftalmia). Síndrome de Felty= AR + Esplenomegalia + leucopenia (neutropenia). Síndrome de Caplan= AR + Nódulos reumatoideos pulmonares + Neumoconiosis. Amiloidosis en AR= AR de larga evolución y grave + proteinuria o insuficiencia renal. CASO CLINICO Se trata de un varón de 63 años con antecedentes de AR de más de 15 años de evolución bajo tratamiento. Entre otros antecedentes padece hipertensión arterial, dislipidemia y cardiopatía isquémica. Ingresa por síndrome nefrótico observándose a la exploración física: 100,7 kg de peso, regular estado general, eupneico en reposo, saturación del 96%; hábito cushingoide; tensión arterial de 130/70mmHg y temperatura de 37°C; auscultación cardiopulmonar normal; abdomen globuloso, blando y depresible; edemas con fóvea hasta la raíz de los muslos, abdomen y manos; artritis bilateral de las articulaciones metacarpofalángicas y limitación a la flexo-extensión en ambos carpos; deformidad de ambos tobillos y pies, más el derecho. Entre las exploraciones complementarias destaca anemia normocítica normocrómica; velocidad de sedimentación globular (VSG) de 130 mm/1.ª hora; hiperfibrinogenemia reactiva; urea de 112 mg/dl; creatinina de 3,8 mg/dl; aclaramiento de creatinina de 45,4 ml/min; ácido úrico de 8,2 mg/dl, proteínas totales de 4,5 g/dl; proteína C reactiva de 10,9 mg/dl; proteinograma con aumento de fracciones alfa 1, alfa 2 y betaglobulinas y disminución de las gammaglobulinas; albúmina de 1,5 g/dl y en el sistemático de orina se detecta proteinuria, microhematuria y abundantes cilindros granulosos-hialinos y más de 11 g de proteínas en orina de 24 h. PREGUNTA Considerando el manejo habitual del padecimiento de base AR, cual de las siguientes observaciones es mas probable que se encuentre relacionado con el síndrome nefrotico? RESPUESTA a.- Uso crónico de AINES. b.- Uso de inmunomoduladores. c.- Presencia de depósitos inmunologicos. d.- Presencia de amiloidosis secundaria a AR.

CURSO ENARM CMN SIGLO XXI TEL: 36246001

Pharmed Solutions Institute

PÁGINA 91

MANUAL DE TRABAJO DEL CURSO ENARM CMN SIGLO XXI PREGUNTA Considerando la pregunta anterior, cual de los siguientes estudios es de mayor utilidad en este caso? RESPUESTA a.- Tomografia computada abdominal. b.- Biopsia renal. c.- Radiografia de torax. d.- Ecocardiografia. PREGUNTA Cual de las siguientes observaciones referentes al diagnostico de AR es de mayor utilidad? RESPUESTA a.- Los anticuerpos anti-CCP tiene un cociente de probabilidad para el diagnostico de AR superior al de Factor Reumatoide y es pronostico de la enfermedad. b.- La evaluación radiográfica por método de Sharp/van der Heijde permite evaluar daño estructural para (erosion y pinzamiento articular). c.- Inflamación articular con anticuerpos anti-CCP y factor reumatoide elevadoes indicativo de enfermedad agresiva con mal pronostico. d.- Se deberá realizar IRM precoz para detectar erosiones, sinovitis o derrame articular. CASO CLINICO Mujer de 68 años de edad con antecedentes de AR de 20 años de evolución, en tratamiento con AINE, glucocorticoides a dosis bajas (5 mg al día de prednisona) y metotrexate; prótesis bilateral de caderas y rodilla derecha; hipertensión arterial, síndrome mielodisplásico en tratamiento con eritropoyetina e insuficiencia renal crónica leve. Ingresa por cuadro de diarreas de 8 semanas de evolución, pérdida de peso de 20 kg y edema en los miembros inferiores. En la exploración física se aprecia: 45 kg de peso, regular estado general, tensión arterial de 110/60 mmHg y una temperatura de 36°C; auscultación cardiopulmonar normal; abdomen blando y depresible; edemas con fóvea hasta raíz de muslos y en manos. Artritis de metacarpofalángicas bilaterales y limitación a la flexo-extensión en ambos carpos. En la analítica destaca anemia normocítica normocrómica, VSG de 110 mm/1.ª h, urea de 46 mg/dl, creatinina de 1,3 mg/dl, sistemático de orina normal, con proteínas en orina de 24 h negativa, proteinograma con aumento de fracciones alfa 1, alfa 2, beta y gammaglobulinas y albúmina disminuida. PREGUNTA Considerando el cuadro clínico, cual de los siguientes estudios es el mas adecuado para una aproximación dignostica? RESPUESTA a.- Tomografia abdominal. b.- Endoscopia abdominal. c.- Panendoscopia. d.- Biopsia por colonoscopia. PREGUNTA Considerando el manejo habitual del padecimiento de base AR, cual de las siguientes observaciones es mas probable que se encuentre relacionado con el síndrome diarreico? RESPUESTA a.- Uso crónico de AINES. b.- Uso de inmunomoduladores. c.- Presencia de depósitos inmunologicos. d.- Amiloidosis secundaria a AR. CASO CLINICO Paciente varón de 60 años con artritis reumatoide de 6 años de evolución en tratamiento con sulfasalazina 1.500mg/día y deflazacort 6mg/día, con buen control clínico y analítico de su enfermedad. Se trata de un paciente fumador activo de 20 cigarrillos/día y con cardiopatía isquémica crónica. En una revisión habitual refiere la aparición de una tumoración dolorosa en el hueco poplíteo derecho, cuyo dolor se incrementa con la bipedestación prolongada. En la exploración física se objetivó una tumoración en el hueco poplíteo. PREGUNTA Considerando las complicaciones mas frecuentes en la AR cual de las siguientes es la mas probable? RESPUESTA a.- Quiste de Baker. b.- Aneurisma popitleo. c.- Lipoma popitleo. d.- Varice popitlea.

CURSO ENARM CMN SIGLO XXI TEL: 36246001

Pharmed Solutions Institute

PÁGINA 92

MANUAL DE TRABAJO DEL CURSO ENARM CMN SIGLO XXI PREGUNTA Cual de las siguientes aseveraciones relacionadas al diagnostico de AR no es adecuado? RESPUESTA a.- BH, QS, Transaminasas, VSG, EGO, actividad, funcionalidad articular y daño estructural. b.- Compromiso en 3 articulaciones metacarpofalangicas o metatarsofalangicas y rigidez matinal de más de 30 minutos. c.- La elevación de la VSG y la PCR tiene estrecha relación con la actividad inflamatoria de la AR. d.- La presencia de Factor Reumatoide confiere el diagnostico de artritis reumatoide. PREGUNTA Relacionado a al tratamiento cual de las conducta siguientes es la mas apropiada en el presente caso? RESPUESTA a.- Los AINES no debe reemplazar a los FARME. b.- Considerando el uso de corticoides debe usarse calcio y vitamida D. c.- El MTX es el fármaco de elección para AR en riesgo de enfermedad persistente. d.- En caso de falla con MTX se recomienda LEF ó SSZ ó anti-TNF. OSTEOARTRITIS (OA): CIENCIAS BASICAS: Enfermedad degenerativa crónica y forma más común de enfermedad de las articulaciones, caracterizada por falla común y progresiva de todas las estructuras de la articulación, cartílago, revestimiento, ligamentos y hueso. Acompañadas por el aumento de espesor y esclerosis de la placa ósea subcondral. SALUD PUBLICA: Mas de 70% de personas >70 años, tienen evidencia radiográfica de OA. Es considerada la causa de dolor e incapacidad más frecuente en ancianos. La incidencia de osteoartrosis de rodilla se ha calculado en 240 casos por 100, 000 habitantes y es una causa de morbilidad y alto costo para la comunidad.. Su prevalencia es igual para ambos géneros entre los 45 y 55 años, pero después de los 55 años es más frecuente en mujeres. Enfermedad articular mas frecuente en mujeres. PATOGENIA: Deterioro de la articulación debido con frecuencia a tensión mecánica, alteraciones biomecánicas y celulares que producen un desequilibrio entre la degradación y la síntesis de los condrocitos, la matriz extracelular y del hueso subcondral, lo que hace que el hueso que está bajo el cartílago falle, tiende afectar las articulaciones que se usan con frecuencia (manos, columna, cadera, rodillas). Los 2 principales componentes del cartílago son: colágeno tipo 2 (provee fuerza tensil) y un proteoglucano. El OA es caracterizado por depleción gradual del proteoglucano, desplegando de la matriz el colágeno y perdiendo colágeno tipo 2, lo que nos lleva a incremento de la vulnerabilidad. CLASIFICACION DE LA OSTEOARTRITIS Dentro de la fisiopatología ocurren cambios morfológicos: Primaria: localizada; Cadera Rodilla fragmentación del cartílago, formación de fisuras y ulceración de la Apófisis espinosas y cuerpos vertebrales superficie articular, eburneación, la posterior proliferación celular en (cervical,lumbar) acúmulos. Cambios bioquímicos: mayor cantidad de agua, sintesisi de Mano (interfalangicas distales) Pie (primer dedo) fibras de colágeno pequeñas y frágiles, menor concentración de Generalizada: Manos proteoglucanos y sulfato de queratán con aumento de la Manos y rodillas concentración de sulfato de condritin que desencadena la producción Secundaria: Displasica: condrodisplasias, displasias epifisiarias, Desplazamiento congénito articular de proteoglucanos de menor tamaño y de menor capacidad para Postraumática: aguda, repetitiva, posoperatoria agregarse al ácido hialurónico, que predispone a fragilidad. Cambios Insuficiencia estructural: osteonecrosis, osteocondritis biomecánicos: perdida de la rigidez y elasticidad del cartílago, Factores Posinflamatoria: artrits reumatoide y séptica Endocrina y metabolica: acromegalia, hemocromatosis de riesgo: edad avanzada, tener familiares con OA, obesidad, lesiones Tejido conectivo: Hipermovilidad (Ehlers- Danlos) en articulación o uso repetitivo, defectos congénitos, densidad ósea, Formas generalizadas: Hiperparatiroidismo, acromegalia ocupación, enfermedades concomitantes. DIAGNOSTICO: Clínico; Dolor (empeora con la actividad, mejora con el reposo), rigidez matutina breve (45°, irregularidad de platillos vertebrales, estrechamiento del espacio intervertebral. CASO CLINICO Femenino de 69 años de edad con cuadro clínico de 10 meses de evolución de inicio súbito; caracterizado por dolor tipo punzada en rodilla izquierda que aparece luego de mantenerla en reposo prolongado, sin predominio de horario, asociado a crepitación, rigidez e inflamación de la rodilla; intensidad 5/10 en la escala visual análoga, la cual cede a medicación con acetaminofén. Fue valorada por medicina general quien indico 4 ampollas de betametasona; sulfato de glucosamina, diclofenaco, colestiramina; con mejoría de síntomas. La radiografía ántero-posterior de rodilla izquierda muestra una lesión radio lúcida subcondral, rodeada por un halo de esclerosis. Al examen físico, presenta crepitación bilateral de rodilla de predominio izquierdo con inflamación moderada de rodilla izquierda. Sinovitis moderada en rodilla izquierda. PREGUNTA Considerando el cuadro clinico y las imágenes cual es su impresión dianostica? RESPUESTA a.- Osteocondritis Aseptica. b.- Osteocondritis Disecante de Rodilla. c.- Osteonecrosis. d.- Osteocondritis osificante. PREGUNTA Considerando la respuesta previa, cual la localizacion mas frecuente? RESPUESTA a.- Lateral del cóndilo femoral medial. b.- Rotula. c.- Cóndilo femoral externo. d.- Meniscos discoideos.

CURSO ENARM CMN SIGLO XXI TEL: 36246001

Pharmed Solutions Institute

PÁGINA 95

MANUAL DE TRABAJO DEL CURSO ENARM CMN SIGLO XXI GOTA: CIENCIAS BASICAS: La gota se caracteriza por hiperuricemia y la presencia de episodios inflamatorios habitualmente monoarticulares y recurrentes con lapsos asintomáticos, que son intensos y autolimitados. La inflamación está directamente relacionada con la presencia de cristales de urato monosódico (UMS) intraarticular de forma constante. Hiperuricemia: Nivel plasmático de acido úrico >7.0mg/dl en hombres o más de 6.0mg/dl en mujeres, surge de causas en la disminución de su eliminación, de aumento de la producción o de ambos. El acido úrico es el producto final de la degradación de los nucleótidos de purina, su producción está relacionada con la concentración intracelular de 5-fosforibosil-1-pirofosfato (PRPP), es el mayor determinante de la velocidad de biosíntesis del acido úrico. El acido úrico es excretado principalmente por el riñón a través de mecanismos de filtración glomerular, secreción tubular y reabsorción. SALUD PÚBLICA: Gota causa común de artritis en hombres mayores de 40 años y mujeres posmenopáusicas. La prevalencia de hiperuricemia en población adulta es de alrededor de 15%. CAUSAS DE HIPERURICEMIA PATOGENIA: La enfermedad comienza con la formación y la Por disminución en la Hiperuricemia primaria (idiopática) subsecuente acumulación de acido úrico secundario a un eliminación Hiperuricemia secundaria: aumento en su producción y/o disminución en su eliminación Función renal disminuida Inhibición de la secreción de uratos (cetoacidosis, acidosis (fase asintomática). Posteriormente factores láctica). desencadenantes como alcohol, diuréticos, obesidad, Por aumento en su Hiperuricemia primaria: trauma, pH acido, ambientales y otros ocasionan producción Hiperactividad de la fosforibosil fosfato sintetasa Deficiencia de hipoxantina-guanina fosforibosiltransferasa acumulación de cristales de urato monosódico en las Hiperuricemia secundaria: articulaciones (artritis), riñones (nefropatía) y vías urinarias. Ingesta excesiva de purinas El depósito de cristales produce liberación de citocinas y Recambio de nucleótidos elevado: psoriasis Incremento de la degradación de ATP (ejercicio) activación, así como reclutamiento de células inflamatorias Mecanismo Deficiencia de glucosa-6-fosfatasa (PMN) y no inflamatorias (endotelio y sinoviocitos) y combinado Hipoperfusión tisular quimiotaxis. No se conoce a detalle el proceso completo de Alcoholismo la formación de cristales pero algunos de los factores son: el acido úrico tiene menor solubilidad a menor temperatura, por lo cual pueden formarse cristales con mayor facilidad en lugares distales como dedos de los pies y de las orejas. La solubilidad de los UMS es menor cuando el pH es ácido, microtraumas repetidos, alteración de la matriz extracelular. DIAGNSOTICO: Clínico; Artritis aguda, la más frecuente y temprana manifestación de gota, inicialmente afecta una articulación (suele localizarse en la primera metatarsofalangica = podagra), pero puede ser poliarticular en episodios tardíos. La gota aguda inicia por la noche con dolor severo, hinchazón, calor. El ataque generalmente desaparece espontáneamente en 3-10 días. Aunque algunos pacientes suelen tener solo un ataque, otros episodios recurrentes, con intervalos de variación sin síntomas. La gota aguda puede ser precipitada por exceso de purinas en la dieta, trauma cirugía, ingestión excesiva de alcohol, terapia hipouricemica y enfermedades serias como infarto al miocardio. Artritis crónica; una proporción de pacientes con gota pueden tener una sinovitis unilateral crónica, esta suele ser rara. Puede también presentarse con tofos (agregados de UMS, rodeados por células de reacción inflamatoria, nódulos, subcutáneos, de aspecto amarillento y duros) periarticulares. Tofos extraarticulares: Algunos están en la cápsula del olecranon, hélix y antihélix del pabellón auricular, superficie cubital o antebrazo, tendón de Aquiles, generalmente se desarrollan entre 5-10 años después del inicio de ataques de gota. Tenosinovitis. Nefropatía por uratos: por depósito de UMS en el intersticio y pirámides, puede causar IRC. Nefropatía aguda por acido úrico: causa reversible de falla renal aguda debida a precipitación de los uratos en los túbulos, los pacientes que reciben tratamiento citotóxico para neoplasia están en riesgo. Nefrolitiasis por acido úrico: responsable de 10% de las litiasis renales. CRITERIOS PARA LA CLASIFICACION de artritis gotosa aguda: A) La presencia de cristales de UMS característicos en el liquido sinovial (gold estándar para dx) o bien B) Un tofo cuyo contenido de cristales de UMS este comprobado por medios químicos o microscopia de luz polarizada o bien C) La presencia de 6 de los siguientes 12 fenómenos: 1. Mas de un ataque de artritis aguda. 2. Desarrollo máximo de brote inflamatorio en el curso del día. 3. Ataque monoarticular. 4. Franco enrojecimiento del area articular. 5. Localización primera articulación metatarsofalángica. 6. Ataque unilateral que afecte la primera articulación metatarsofalngica. 7. Ataque unilateral en una articulación del tarso. 8. Tofo: sospecha o confirmación. 9. Hiperuricemia. 10. Infamación asimétrica en estudio radiográfico. 11. Quistes subcutáneos sin erosiones óseas. 12. Cultivo negativo del liquido articular obtenido durante un ataque agudo. Los ataques de gota agudos se relacionan con cambios en el nivel plasmático de AU (tanto aumento como disminución). La identificación de UMS en liquido sinovial es patognomónico, se ven al microscopio como agujas y birrefringentes. Radiológico: En ataque agudos pueden mostrar tumefacción de tejidos blandos. Los tofos son radiotransparentes, aunque en ocasiones pueden tener calcificaciones en su interior, los cambios característicos son edema difuso, prominencias nodulares, erosiones intraarticulares y extraarticulares con imagen de “techo colgante” y calcificaciones. TRATAMIENTO: Dentro de la medidas generales dieta baja en alimentos ricos en purinas como: carnes, vísceras, piel, alimentos ahumados, rostizados, los dos factores más importantes que favorecen la hiperuricemia son: ingesta de alcohol y ayuno prolongado. Artritis gotosa aguda: Analgesia, AINES (indometacina, diclofenaco, naproxen) primera elección. Colchicina efectiva dentro de las primeras 24hrs del ataque, contraindicada cuando hay insuficiencia renal o citopenias. Dosis de 1-6 mg/día. Glucocorticoides intraarticulares; se debe descartar primero artritis séptica. Glucocorticoides sistémicos; uso breve, considerado en pacientes con ataque de gota poliarticular. Agentes hipouricemicos: Indicados para iniciar a bajar el acido úrico, incluyendo recurrencias de artritis gotosa aguda, artritis poliarticular, gota tofosa, litiasis renal. No deben iniciarse en el ataque agudo, pueden precipitar una llamarada y se debe considerar su uso concomitante con colchicina. Alopurinol: disminuye la síntesis de acido úrico por inhibición de la xantino-oxidasa, reducir dosis en insuficiencia renal. Dosis de 100-300mgs, pueden darse hasta 800mgs. Uricosúricos (probenecid, sulfinpirazona): Incrementan la excreción de acido úrico, por inhibición de la reabsorción tubular (inefectivo en insuficiencia renal), no debería usarse en >60 años de edad. Iniciarse una vez que haya pasado el ataque agudo. CASO CLINICO Varón de 71 años con antecedentes personales de hipertensión arterial sistémica en tratamiento con enalapril, 20 mg diarios, e hiperuricemia, con episodios de monoartritis en el pie izquierdo, Ingresa por dolor y tumefacción en el segundo dedo de la mano

CURSO ENARM CMN SIGLO XXI TEL: 36246001

Pharmed Solutions Institute

PÁGINA 96

MANUAL DE TRABAJO DEL CURSO ENARM CMN SIGLO XXI derecha y en la cuarta articulación interfalángica proximal de la mano izquierda, sin fiebre ni otros datos clínicos. La exploración física mostró nódulos de Heberden en ambas manos, con signos de artritis en la segunda interfalángica distal de la mano derecha, con gran tumefacción distal y aumento de partes blandas, con un tofo acompañante, así como en la cuarta interfalángica proximal de la mano izquierda. En las pruebas de laboratorio destacó una uricemia de 7,20 mg/dl, con hemograma, perfil bioquímico y reactantes de fase aguda dentro de la normalidad (VSG: 14 mm/h, PCR: 0,49 mg/dl). En las radiografías de las manos se apreciaron cambios degenerativos en las articulaciones metacarpofalángicas, interfalángicas y trapeciometacarpianas. La interfalángica distal del segundo dedo presentó aumento de partes blandas, con disminución del espacio articular y erosiones en sacabocados. La ecocardiografía y los hemocultivos seriados fueron negativos. PREGUNTA Cual de los siguientes resultados del aspirado de liquido sinovial en este caso es mas frecuente encontrar? RESPUESTA a.- Cristales de urato monosódico. b.- Cristales de pirofosfato cálcico. c.- Ambos tipos de cristales. d.- Cristales de pirofosfato. LUPUS ERITEMATOSO SISTEMICO (LES) CIENCIAS BASICAS: Prototipo de enf. autoinmune, multiorganica, multisistémica, se asocia con la producción de autoanticuerpos contra componentes del núcleo, mediada por inmunocomplejos (hipersensibilidad III). SALUD PUBLICA: Más en mujeres de 20-40años. Relación H:M es de 9:1. Lupus eritematoso sistémico 70%, lupus cutáneo l5%, Sx de sobreexposicion/Enf. mixta de tejido conectivo 10%, lupus eritematoso inducido por drogas 5%. PATOGENIA: Enfermedad de etiología desconocida. Participan para el desarrollo factores genéticos como: HLA-DR2, DR3, DR4 (75%), deficiencia de complemento C1, C2, C4 (a mayor deficiencia mayor riesgo para nefritis lupica), mayor incidencia en familiares de 1er grado. Factores ambientales: Rayos UVB (apoptosis de queratinocitos), medicamentos Clasificación de nefritis lupica según la OMS (hidralacina, procainamida, fenitoina, isoniazida), infecciones (VEB), TIPO I Nefropatía lupica mesangial mínima mimetismo molecular. Factores hormonales: estrógenos son permisivos TIPO II Nefropatía lupica mesangial de autoinmunidad, puerperio y anticonceptivos exacerban él LES. Factores TIPO III Nefropatía lupica focal inmunológicos: falla en el control del sist. Inmunológico, producción TIPO IV Nefropatía lupica difusa TIPO V Nefropatía lupica membranosa excesiva de autoanticuerpos. Se producen autoanticuerpos, se forman TIPO VI Nefropatía lupica esclerosante (terminal) complejos inmunes, disminuye la apoptosis de cel. T autoreactivas, hay actividad aumentada de linfocitos T cooperadores (CD4) y disminución de linfocitos T supresores (CD8); todo esto junto con los factores nos lleva a LES. DIAGNÓSTICO: Cuadro clínico: Hay períodos de remisión y exacerbación, síntomas inespecíficos en 95%; fiebre, astenia, adinamia. Manifestaciones musculo-esqueléticas son las más frecuentes (90-95%); artralgias (70-100%), artritis no erosiva (60%), pulgar en z, mialgias, hiperlaxitud, osteonecrosis avascular (dolor en cadera y/o hombro). Mucocutaneas frecuentes (80-90%); lo mas común presencia de lupus discoide (15-30%: cara, hombro, cuello, retroauricular), rash en alas de mariposa (50%), eritema malar tiene la característica de inestabilizarse e incrementarse con la exposición al sol, fotosensibilidad (70%), ulceras orales (40%), vasculitis cutánea y livedo reticularis. Hematológicas frecuentes (80-90%); anemia (70%), leuco, linfo y trombocitopenia, linfadenopatía y esplenomegalia. Neurológicas (50-60%); neuralgia, meningitis aséptica, disfunción cognitiva (50%), sx. neural orgánico (35%), convulsiones y psicosis (20%), neuropatía periférica, EVC, cefalea. Cardiopulmonares; pleuritis (50%), derrame pleural (30%), neumonitis lupica ( en 10%; de las complicaciones más graves, mortalidad 60-70%), pericarditis (30%), endocarditis de Libman-Sack (insf. aórtica y mitral: es aséptica). Renales frecuentes (50%); principal factor que condiciona la sobrevida del paciente (Cr >1.4, hipotensión y sx. nefrótico), glomerulonefritis membranosa y mesangial, nefritis lupica factor de mal pronóstico y es la principal causa de muerte en lupus hasta 50%. Laboratorio: Perfil metabólico (tienen aterogenesis acelerada y lípidos alterados), EGO (proteinuria), reactantes de fase aguda (PCR y VSG), anticuerpos antinucleares (ANA; considerados gold standar para LEG, especificidad media), C3 y C4, anti-DNA ds más específico para lupus ( especificidad excelente), anti- histona ( relacionado con lupus inducido por drogas), anti sm ( especificidad excelente), anti-Ro (SSA, se relaciona con lupus neonatal, pedir en embarazadas), anti-La (SSB, si sale positivo protege contra daño renal). Anti-Ro y anti-La; son específicos pero son más sensibles para Sx. de Sjogren. CRITERIOS DIAGNOSTICOS: Cutáneos 1. Rash malar: eritema malar fijo, plano o palpable 2. Rash discoide: parches eritematosos palpables, con ampliación queratosica y taponamiento folicular, pueden existir cicatrices atroficas. 3. Fotosensibilidad: reacción inusual a la luz solar, observado por medico 4. Ulceras orales: o nasofaringeas, usualmente indoloras, observadas por medico Sistémicos 1. Artritis: no erosiva que involucre 2 o más articulaciones periféricas, caracterizadas por hipersensibilidad, inflamación o derrame. 2. Serositis: pleuritis o pericarditis 3. Trastorno renal: proteinuria persistente (>0.5 g/d o más de +++), o cilindros celulares de cualquier tipo. 4. Trastorno neurológico: convulsiones o psicosis en ausencia de otras causas. Laboratorio: 1. Trastorno hematológico: Anemia hemolítica o leucopenia (>4000 en 2 ocasiones), linfopenia (>1500 en 2 ocasiones) o trombocitopenia (>100,000 en ausencia de drogas causantes) 2. Trastorno inmunológico: Anti-DNA o anti- sm, o anticuerpos antifosfolipido 3. Anticuerpos antinucleares: en ausencia de drogas conocidas, que puedan ser asociadas a lupus inducido por drogas. SE DICE QUE UNA PERSONA TIENE LEG, SI PRESENTA 4 DE LOS 11 CRITERIOS DE FORMA SIMULTÁNEA O SERIADA EN EL CURSO DE LA ENFERMEDAD, DURANTE O EN CUALQUIER PERIODO DE LA MISMA.

CURSO ENARM CMN SIGLO XXI TEL: 36246001

Pharmed Solutions Institute

PÁGINA 97

MANUAL DE TRABAJO DEL CURSO ENARM CMN SIGLO XXI FARMACOS QUE INDUCEN LUPUS: Constituye el 10%; Hidralacina, procainamida, isoniacida, clorpromacina, d-penicilamina, quinidina, interferon alfa, fenitoina, anti-TNF,predominan manifestaciones constitucionales y pleuropericarditis, no afección renal, ni neurológica, 90% remite a los 3 meses de suspender el fármaco. Asociado a HLA-DR4, anti- DNA negativos, ANA positivos, anti-histona positivo e hipocomplementemia. LUPUS NEONATAL: Se produce en menos de 5% de los RN de madres portadoras de autoanticuerpos anti-Ro y anti-La, hasta en 25% de madre sanas con anti- Ro circulante, lesiones cutáneas en ateas fotoexpuestas a partir de los 2 meses de vida, bloqueo A-V completo (complicación más grave del lupus neonatal) y alteraciones hematológicas. TRATAMIENTO: Basado en el tipo y la severidad. AINES (Ibuprofeno 400-800mg c/6-8hrs)Corticoesteroides (prednisona 1mg/Kg, en caso de daño orgánico agudo; metilprednisolona 1g/d x3dias ), hidroxicloroquina (antimalaricos) y aspirina (estos autorizados por la FDA). No exponerse al sol, dietas sin alfalfa. Manifestaciones leves: AINES, esteroides a dosis bajas (>20mg/kg/día), hidroxicloroquina (400mg/día. efecto secundario toxicidad retiniana). Lesiones cutáneas, fotoprotección (FPS>100), corticoides tópicos. Manifestaciones graves (neurolupus, glomerulonefritis, neumonitis, trombocitopenia, anemia hemolítica) pulsos de metilprednisolona, prednisona, inmunosupresores como aziatropina, metotrexate (son ahorradores de corticoides). 1ra elección en nefritis lupica ciclofosfamida (10_15mg/kg/día). Aziatropina; nefritis lupica y daño hematológico. Metotrexatearticulares (piedra angular en artritis reumatoide), manifestaciones articulares 7.515mg/sem. Si no responden a la terapia inicial lo más usado en terapia biológica es rituximab (anti-CD-20).). Anticoagulación puede estar indicada en complicaciones tromboticas. CASO CLINICO Mujer de 17 años que padecía de LES. En el momento del diagnóstico debutó con una nefritis lúpica manifestada por un síndrome nefrótico, con proteínas en orina de 24 horas de 3,3 g, colesterolemia de 280 mg% y albuminemia de 1,8 g/dL, por lo cual recibió tratamiento con metil-prednisolona succinato en bolos durante tres días (tres dosis), y se mantuvo con prednisona 1 mg/kg de peso/día, observándose mejoría del cuadro. Igualmente la paciente presentó una cefalea intensa que calmaba parcialmente con analgésicos y sin signos de irritación meníngea, que fue interpretada como un síntoma asociado al lupus. Continuó el tratamiento con prednisona hasta 15 días después de su egreso, pero disminuyó a una dosis de 35 mg/día. Un mes después, la paciente acudió nuevamente para control. Los exámenes demostraron deterioro de la función renal, asociado a un síndrome nefrótico impuro, con hipertensión arterial (160/110). Se aumentó la dosis de prednisona a 1 mg/kg de peso/día y se agregó captopril 50 mg cada 8 horas. La paciente refería persistencia de la cefalea frontoparietal, opresiva, de fuerte intensidad, que ya no mejoraba con analgésicos y que era exacerbada por los movimientos de la cabeza, sin otros signos y síntomas asociados. A los pocos días del ingreso presentó fiebre de 40 °C, náuseas, vómitos y fotofobia. PREGUNTA Cual es la conducta a seguir mas adecuada? RESPUESTA a.- Agregar diurético, neuroproteccion y antibiótico. b.- Realizar punsion lumbar para citoquimico, citológico y cultivo. c.- Realizar IRM de cráneo. d.- Aumentar corticoide y FARME. PREGUNTA Entre 30 y 50% de la morbilidad y mortalidad de los pacientes con lupus eritematoso sistémico (LES) son atribuidas a las infecciones, cual de las siguientes es la menos frecuente? RESPUESTA a.- SNC. b.- Piel. c.- Pulmones. d.- Sistema genitourinario. CASO CLINICO Mujer de 67 años de edad diagnosticada de hipertensión arterial. Ingresó por sensación nauseosa, infección urinaria y anemia. Análisis: Htº 27,8%; Hb 9,5 g/dl. VSG 63 mm. Urea 182 mg/dl; creatinina 4,7 mg/dl; calcio 9 mg/dl; fósforo 4,2 mg/dl, proteínas totales 9 g/dl. Aclaramiento de creatinina (fórmula de Cockroft-Gault): 17,73 ml/min. Inmunoproteínas y complemento normal. Cadenas Kappa-s 774 mg/dl, cadenas Lambda 392 mg/dl. En el proteinograma se observó un pico de amplia base en región Gamma, con aumento de IgG (193%) y cadenas ligeras Kappa (191%) y Lambda (180%). Índice K/L= 1,97. Cadenas ligeras en orina: cadenas Kappa 13,7 mg/dl (0-0,7); cadenas Lambda 6,880 (0-0,39). Se realizó aspirado y biopsia de médula ósea, que resultó normal. THS: 4,85 µUI/ml, T4 libre 1,03 ng/fl; Ac anti-TPO 22,5 UI/ml; Ac anti-Tiroglobulina 115,3 UI/ml. PTH: 110 pg/ml. Orina (tira reactiva): proteínas 25 mg/l; sedimento: abundantes leucocitos. Marcadores tumorales: normales. Serología viral: negativa. Autoanticuerpos: ANA +, anti-ADN positivos. PREGUNTA Considerando las características del caso clínico, cual es el abordaje terapéutico mas adecuado? RESPUESTA a.- Metotrexate. b.- Ciclosporina. c.- Prednisona. d.- Aziatropina.

CURSO ENARM CMN SIGLO XXI TEL: 36246001

Pharmed Solutions Institute

PÁGINA 98

MANUAL DE TRABAJO DEL CURSO ENARM CMN SIGLO XXI PREGUNTA El lupus eritematoso sistémico (LES) es una enfermedad inflamatoria con afectación sistémica, cual es el porcentaje de pacientes que presentan signos de nefropatía?. RESPUESTA a.- Al menos 60%. b.- Al menos 50 %. c.- Al menos 40 %. d.- Al menos 30% CASO CLINICO Mujer de 47 años de edad con Nefropatía lúpica. AHP a los 17 años presenta Púrpura Trombopénica que lleva a una esplenectomía, a los 19 años de edad ingresa tras presentar un brote de artralgias, fiebre, edemas y lesiones eritematosas en cara, siendo diagnosticado de LES. En ese momento destaca la presencia de síndrome nefrótico con alteraciones en el sedimento, una creatinina sérica de 1.9 mg/dl y en el estudio cardiológico se objetiva prolapso de la válvula mitral. Se practica biopsia renal. El cilindro contiene 12 glomérulos, grandes, lobulados, con proliferación mesangial difusa de distribución irregular, núcleos en carirorrexis, trombos intracapilares ocasionales y paredes capilares engrosadas en “asa de alambre”. Las pequeñas arterias, no tienen alteraciones. La técnica de inmunofluorescencia directa demuestra la presencia de depósitos subendoteliales y mesangiales de IgG, C3, C1q, IgM e IgA. Diagnóstico: Nefritis lúpica Clase IV. Se decide tratamiento con corticoides y azatriopina, normalizándose la función renal y negativizándose la actividad inmunológica, persistiendo proteinuria de 2 gr. /24 h. A los 21 años se detectan cifras elevadas de tensión arterial. En su primer embarazo, presenta aborto espontáneo tras 2 meses de amenorrea. A los 22 años se retira la inmunosupresión en el curso de su segundo embarazo que llega a término, presentando hipertensión arterial en el octavo mes. Inmediatamente tras el parto en mayo de presenta empeoramiento de su situación con hipertensión arterial y proteinuria de 3 gr. /24 h, reiniciándose el tratamiento con esteroides y azatriopina, quedando la paciente asintomática, sin proteinuria, con función renal normal, estudio inmunológico negativo pero persistiendo la hipertensión arterial. A los 25 años de edad comienza con crisis epilépticas detectándose en TAC cerebral lesiones isquémicas múltiples y un mes después, segundo brote lúpico con fiebre, afectación articular, cutánea, reaparición de proteinuria con función renal normal y actividad inmunológica. A los 36 años de edad, es intervenida de aneurisma fusiforme de la arteria carótida derecha. Un año después presenta un accidente cerebro vascular con hemiplejia izquierda y recuperación total, se observaron dilataciones en carótida y aorta abdominal en estudios posteriores. PREGUNTA Considerando la comorbilidad del LES cual de las siguientes entidades clínicas es mas probable considerando la evolución del caso? RESPUESTA a.- Sindrome infeccioso. b.- Arteriosclerosis. c.- Sindrome antifosfolipidico. d.- Necrosis avascular ósea. CASO CLINICO Un varón de 70 años, con antecedentes de diabetes mellitus, ingresó por febre de 38,5ºC de una semana de evolución. A la exploración destacaba presión arterial 106/75 mmHg, frecuencia cardíaca de 95 lpm y edema importante hasta ambas rodillas. En el tobillo derecho y en el talón izquierdo presentaba úlceras cutáneas con un exudado purulento del cual se tomaron muestras. En la bioquímica, al ingreso, destacaba leucocitosis 19.800 cél/mm3, AST 68 U/L, creatinina 260 μmol/L, urea 16,4 mmol/L; el resto fue normal. El ECG era anodino. La radiografía de tórax evidenció una cardiomegalia moderada. El Eco-Doppler cardíaco mostró un derrame pericárdico difuso de aproximadamente 1 cm, con imágenes compatibles con depósito de fibrina y signos sugestivos de compromiso hemodinámico. En las 12 horas siguientes presentó taquicardia y oliguria progresivas, con hipotensión arterial mal tolerada. PREGUNTA Cuál es la conducta a seguir más adecuada. RESPUESTA a.- Pericardiocentesis. b.- Pericardientomia urgente. c.- Manejo conservador. d.- Corrección del estado hemodinamico. CASO CLINICO Paciente con 50 años de edad, con LES hacía cuatro años. Durante su evolución presentó como intercurrencia un síndrome antifosfolipídico y nefritis lúpica que requirió tratamiento con anticoagulantes orales, azatioprina 100 mg/día, hidroxicloroquina 200 mg/día y prednisona 20 mg/día en dosis decreciente. Es ingresado a urgencias por familiares debido a que presenta desorientación, dificultad para movilizar el brazo y pierna del mismo lado y cegura súbita. PREGUNTA Cuál es la complicación mas probable en este caso?

CURSO ENARM CMN SIGLO XXI TEL: 36246001

Pharmed Solutions Institute

PÁGINA 99

MANUAL DE TRABAJO DEL CURSO ENARM CMN SIGLO XXI RESPUESTA a.- Vasculitis cerebral. b.- Isquemia cerebral transitoria. c.- Embolia cerebral. d.- Neurolupus. CASO CLINICO Se trata de paciente femenino de 32 años de edad la cual se encuentra diagnosticada con lupus eritematoso sistémico con 4 años de evolución con tratamiento irregular, ingresa a admisión continua debido a que desde hace 48 horas inicia súbitamente con estado confusional agudo previamente con letargia, a la exploración física se observa ictérica, su estado no permite responder preguntas, sus signos vitales se encuentra taquicardia, hipotensión, palidez y disnea, no se auscultan alteraciones cardiorespiratorios, sin embargo en abdomen se detecta esplegnomegalia, se realizan exámenes de rutina con hemoglobina de 6 g/dl, leucos de 6,200 y plaquetas de 300,000, bilirrubina 4,5 g/dl con reticulocitos de 19 % así como microesferocitosis, resto sin datos por agregar: PREGUNTA Cual es la complicación actual que presenta esta paciente que es potencialmente mortal aguda? RESPUESTA a.- Sindrome hemolítico-uremico. b.- Purpura trombocitopenica trombotica. c.- Anemia hemolítica autoinmunitaria. d.- Anemia Hemolitica microangiopatica. VASCULITIS: CIENCIAS BASICAS: Grupo heterogéneo de enfermedades raras, que pueden ocurrir de forma independiente o como complicaciones de una enfermedad previamente establecida. Involucran la vasculatura de cualquier órgano, la presentación depende del calibre del vaso afectado. CLASIFICACION: ARTERITIS DE CELULAS GIGANTES (ACR): La más común de las vasculitis VASOS DOMINANTES VASCULITIS sistémicas. También llamada enf. de Horton, afecta a las ramas craneales Grandes vasos Arteritis de células gigantes (ACR) del arco aórtico, especialmente a la arteria temporal. Más común en Arteritis de Takayasu Medianos vasos Poliarteritis nodosa (PAN) mayores de 50 años. Más frecuente en mujeres en relación 2:1. CLINICO: Enf. De Kawasaki (AT) Fiebre de bajo grado, fatiga, pérdida de peso, mialgias, anorexia, cefalea Pequeños vasos y Granulomatosis de Weneger (GW) (60%). Afección de la arteria oftálmica (20%), neuritis óptica, diplopía, arterias medianas Sx. de Churg-Strauss (SCS) ASOCIADAS A ANCAS Poliangitis microscópica (PA) amaurosis fugaz y ceguera (dar tx. médico precoz urgente; esteroides en Pequeños vasos Purpura de Henoch-Scholein (PHS) dosis altas). Afección de la A. facial: claudicación mandibular. CRITERIOS (leucocitoclástica) Arteritis leucocitoclástica cutánea DIAGNOSTICOS: 1. Edad >50 años 2. Cefalea de reciente inicio 3. Hipersensibilidad de la arteria temporal (piel cabelluda) o disminución de la pulsación 4. VSG >50mm/h 5. Biopsia granulomas y vasculitis. Con 3 de 5 criterios se hace el diagnostico. DIAGNOSTICO: VSG (98%), PCR elevadas, anemia y alteración de pruebas de la función hepática (70%), biopsia de la arteria afectada (afección en parches, no es continua); infiltrado mononuclear, con presencia de granulomas y células gigantes. TRATAMIENTO: Prednisona 1mg/kg/día, durante las primeras semanas con descenso gradual. ARTERITIS DE TAKAYASU (AT): Llamada enf. “sin pulso” o síndrome del arco aórtico. Más común en mujeres en edad reproductiva. Proceso inflamatorio crónico granulomatoso, que afecta la aorta y sus ramas, provocando generalmente síntomas isquémicos. CLINICA: Se divide en fases: fase I; periodo inflamatorio con fiebre, artralgias y pérdida de peso, trill aórtico o subclavio. Fase II; hipersensibilidad y dolor en vasos sanguíneos, disminución asimétrica en pulsos en extremidades, claudicación, hipertensión renovascular, sincope neurogénico. Fase III; periodo fibrotico. La artería más afectada hasta 93% es la subclavia. CRITERIOS DIAGNOSTICOS: 1. Edad 100mmHg en los brazos 5. “Trill” sobre arterias subclavia o aorta. 6. Arteriografía anormal. Con 3 de 6 criterios se hace el diagnostico. CLASIFICACION: de acuerdo al sitio afectado: I: Ramas del arco aórtico. IIa Aorta ascendente, arco aórtico y sus ramas. IIb Aorta descendente, arco aórtico y sus ramas , A. torácica descendente. III. A. torácica descendente, A abdominal y/o arterias renales. IV. A. abdominal y/o A. renales. V. Combinación de los tipos IIb y IV. DIAGNOSTICO: VSG Y PCR elevadas. Arteriografía=Gold estándar; oclusión, estenosis, irregularidad y aneurisma. TRATAMIENTO: Esteroides, MTX, Aziatropina y antiplaquetarios, en enfermedad refractaria micofenolato y ciclofosfamida. PANARTERITIS NODOSA CLASICA (PAN): Inflamación necrotizante de arteria de pequeño y mediano calibre (respeta capilares y vénulas). Disminución de la luz vascular. Se asocia a VHB 30% y a VHC 5% CLINICA: Insuficiencia renal, HTA (70%), purpura papable (50%), mononeuritis múltiple (50%), dolor abdominal (40%), ICC, IAM, pericarditis (30%), raras afecciones pulmonares. CRITERIOS DIAGNOSTICOS: 1. Pérdida de peso >4Kg 2. Livedo reticularis 3. Dolor e hipersensibilidad testicular (orquitis) 4. Mialgias, debilidad, dolor de extremidades (50-60%). 5. Mononeuropatía o polineuropatía 6. Presión diastólica >90mmHg 7. BUN elevado (>40mg/dl) o Cr mayor de 1.5 mg/dl 8. Virus de hepatitis B 9. Arteriografía anormal (aneurisma, oclusión, estenosis de arterias viscerales) 10. Biopsia; vasculitis de mediano y pequeños vasos. Con 3 de 10 criterios se hace el diagnostico. DIAGNOSTICO: Angiografía, biopsia; infiltrado de PMN y necrosis fibrinoide (característico ausencia de granulomas y eosinofilos).TRATAMIENTO: Glucocorticoides, ciclofosfamida, antiviral si está asociada a VHB O VHC. GRANULOMATOSIS DE WENEGER (GW): Enfermedad inflamatoria granulomatosa necrotizante con vasculitis sistémica. Predomina en adultos jóvenes. Predomina en capilares y vénulas. CLINICO: Vías aéreas superiores (95%); Sinusitis, otitis media y destrucción de tabique nasal. Pulmón (90%); Infiltrados pulmonares cavitados bilaterales y no migratorios. Nódulos hemorrágicos y hemoptisis. Glomerulonefritis (75%); Focal y segmentaria rápidamente progresiva. Ocular (50%); epiescleritis, uveítis. Neuropatías periféricas o craneales. CRITERIOS DIAGNOSTICOS: 1. Inflamación oral o nasal; ulceras orales, descarga nasal purulenta o hemorrágica 2. Radiografía de tórax con presencia de nódulos, infiltrados fijos o

CURSO ENARM CMN SIGLO XXI TEL: 36246001

Pharmed Solutions Institute

PÁGINA 100

MANUAL DE TRABAJO DEL CURSO ENARM CMN SIGLO XXI cavitaciones 3. Hematuria microscópica o restos de eritrocitos 4. Biopsia con inflamación granulomatosa. Con 2 de 4 criterios se hace el diagnostico. DIAGNOSTICO: c-ANCA (Anticuerpos antiproteinasa-3) positivos 90%. TAC o Rx de tórax; nódulos, infiltrados, cavitaciones. Elevación de BUN, Cr, proteinuria, sedimento urinario activo. Biopsia; inflamación granulomatosa necrotizante de arteriolas, capilares y vénulas. TRATAMIENTO: Ciclofosfamida + prednisona, MTX o aziatropina. TMP/SFX puede prevenir las recaídas de infecciones de vías aéreas superiores. POLIANGITIS MICROSCOPICA (PA): Vasculitis necrotizante de pequeños vasos, puede haber o no granulomas. CLINICO: Síntomas constitucionales; pérdida de peso, fiebre (50%), fatiga, mialgias. Renal; hematuria, glomerulonefritis rápidamente progresiva (90%). Pulmonar; tos y/o hemoptisis, hemorragia pulmonar (30%). Neurológico; mononeuritis multiplex (40%). DIAGNOSTICO: p-ANCA (anti-mieloperoxidasa) positivos 70%. Biopsia; inflación pauci-inmune necrotizante de arteriolas capilares y vénulas, con o sin granulomas o infiltrados eosinofilicos. TRATAMIENTO: Ciclofosfamida + aziatropina, MTX o aziatropina (ahorradores de esteroides). SINDROME DE CHURG-STRAUSS (SCS): Inflamación granulomatosa eosinofilica que involucra pulmón. A cualquier edad más entre 30-40 años. CLINICO: Asma de difícil control (95%), enfermedad eosinofilica infiltrativa, vasculitis de pequeños vasos con granulomas. Neuropatía; mononeuritis multiplex (60-70%), parestesias en guante o calcetín, arteritis coronaria y miocarditis. Glomerulonefritis. CRITERIOS DIAGNOSTICO: 1. Asma 2. Eosinofilia (>10%) 3. Mono o polineuropatía 4. Infiltrados pulmonares migratorios transitorios 5. Anormalidades paranasales 6. Eosinofilia extravascular en la biopsia. Con 4 de 6 criterios se hace el diagnostico. DIAGNOSTICO: Dato cardinal para diagnostico: eosinofilia periférica 80%. ANCA (c-ANCA o p-ANCA) positivo en 50%. Biopsia: microgranulomas, necrosis fibrinoide y trombosis de péquelas arterias y venas con infiltrados eosinofilicos. Rx. de tórax: puede haber infiltrados pulmonares. TRATAMIENTO: Ciclofosfamida + prednisona, MTX o aziatropina. PURPURA DE HENOCH-SCHONLEIN (PHS): La más común de las vasculitis (leucocitoclástica) en niños (3 episodios anuales), de base necrótica amarillenta, se curan en 1 o 2 sem sin dejar cicatriz. Ulceras genitales (80%), si dejan cicatriz. Foliculitis (80%), eritema nodoso (50%), erupciones acneiformes. Ocular; manifestación más grave, uveítis posterior que puede condicionar pérdida de visión (20%, tx. Agresivo con ciclosporina o anti-TNF). Puede haber meningitis aséptica, meningoencefalitis. DIAGNOSTICO: Fenómeno de patergia (patognomónico), se inocula sol. Inyectable lesión de >2cm es positiva. Biopsias de ulceras. Fondoscopia. CRITERIOS DIAGNOSTICOS: 1. Ulceras aftosas orales recurrentes 2. Ulceras genitales recurrentes 3. Lesiones oftálmicas: uveítis (con hipopión), escleritis, vasculitis retiniana, neuritis óptica. 4. Lesiones cutáneas; pápulas foliculitis, eritema nodoso 5. Patergia positiva. El diagnostico se hace con el N.1 y 2 más del resto. TRATAMIENTO: Aziatropina, para ulceras (colchicina, dapsona, talidomida), corticoides, para uveítis la ciclosporina. ENFERMEDAD DE KAWASAKI: Sindrome linfomucocutáneo. Más común en menores de 5 años (80%), ligero predominio en sexo masculino. Etiología desconocida. CLINICA: Cuadro exantemático febril, que no responde a antibióticos y congestión conjuntival o no. Alteraciones labiales; eritema, fisuras, sangrado con formación de costras y lengua en frambuesa. Adenopatías cervicales dolorosas y exantema en tronco y extremidades. Formación de aneurismas (25%), vasculitis coronarias (forma más mortal), carditis (80%). DIAGNOSTICO: Clínico, factores reactantes de fase aguda, angiografía coronaria, no se asocia a ningún autoanticuerpo. CRITERIOS DIAGNOSTICOS: Fiebre persistente por al menos 5 dias, mas 2 de los siguientes: 1. Cambios en extremidades inferiores 2. Exantema polimorfo 3. Inyección conjuntival bilateral 4. Cambios en labios y cavidad oral 5. Linfadenopatía cervical. TRATAMIENTO: Gammaglobulina IV (2g/Kg dosis única o 400 mg/kg, durante 4 dias. ASA de 100-1500mg/día. Controversial uso de esteroides CASO CLINICO Mujer de 60 años, con antecedentes de hipertensión arterial, que bruscamente presentó un cuadro de diarrea matutina, acompañada durante 30 minutos de cefalea frontal invalidante, que cedió con analgesia convencional y repitió varias mañanas sucesivas. Quince días después, durante tres horas y coincidiendo con la cefalea, presentó una hipoestesia derecha global, acompañada de parestesias y debilidad en la extremidad inferior derecha. TAC craneal y resonancia magnética nuclear (RMN) vertebromedular que no mostraron enfermedad, ecocardiográfico fue normal. A las 72 horas, presentó otro episodio de debilidad, también de inicio brusco, con una duración de 10 horas y que afectó al hemicuerpo izquierdo. Una nueva TAC craneal tampoco mostró enfermedad. El estudio del líquido cefalorraquídeo incluyendo bioquímica, inmunoglobulinas, anticuerpos antineuronales y bandas oligoclonales resultó normal, salvo una discreta linfocitosis, así como negativos fueron los cultivos, las serologías y diversas PCR como virus herpes, BK y JC. Se descartó la enfermedad de Whipple. PREGUNTA Cuál es el estudio más adecuado para establecer el diagnostico de vasculitis? RESPUESTA a.- Angio-IRM.

CURSO ENARM CMN SIGLO XXI TEL: 36246001

Pharmed Solutions Institute

PÁGINA 101

MANUAL DE TRABAJO DEL CURSO ENARM CMN SIGLO XXI b.- Angiografia. c.- Biopsia cerebral. d.- ANA y ANCA. CASO CLINICO Paciente de sexo femenino, de 21 años de edad, diagnosticada de AT. Consultó por dolor articular y claudicación del miembro superior izquierdo de 4 meses de evolución. Al examen físico presentaba ausencia de pulso radial izquierdo y signos de insuficiencia aórtica. Se realizó una angiografía digital (AD) que mostró hallazgos típicos de AT. Por falta de respuesta al tratamiento con prednisona, se indicó una angioplastia de arteria carótida primitiva izquierda y tratamiento con ácido acetilsalicílico (AAS) y metotrexato, logrando mejoría clínica. Pero iniciando con diarrea y dolor abdominal. Se solicitó angiografía por resonancia magnética abdominal que no evidenció alteraciones; la inmunoglobulina A (IgA) anti-endomisio resultó positiva y biopsia de duodeno. PREGUNTA Que resultados histopatológicos espera encontrar. RESPUESTA a.- Atrofia difusa de vellosidades. b.- Infiltrado inflamatorio en la lamina propia. c.- Datos de isquemia generalizado. d.- Infiltrado inflamatorio crónico. CASO CLINICO Paciente femenina de 22 años de edad con historia de 2 meses de evolución con sinusitis, otitis media bilateral complicada con mastoiditis, conjuntivitis bilateral, artralgia en hombros, codos, rodillas y tobillos, tos con expectoración sanguinolenta, debilidad generalizada y cansancio fácil, asociado a fiebre no cuantificada sin predominio de horario y a pérdida de peso aproximadamente de 10% de su peso habitual. Hace más o menos tres semanas presenta cuadro de odinofagia, lumbalgia, disuria, coluria y refiere un episodio de epistaxis aislada. SV: Temperatura: 38,2 °C, FC: 110 latidos/minuto, FR: 22 ciclos/minuto. Tensión Arterial: 120/90 mmHg. Radiografía de tórax: se identifica infiltrado de tipo intersticial de predominio derecho parahiliar, en menor grado, este tipo de infiltrado se visualiza hacia la base pulmonar izquierda. No hay signos de lesiones nodulares ni cavitaciones. TAC: infiltrados nodulares en ambos campos pulmonares, cavitación en lóbulo superior derecho, tenue patrón de vidrio esmerilado posterobasal derecho y crecimientos ganglionares precarinales, parahiliares y subcarinales. PREGUNTA Considerando el diagnostico del caso, cual de las siguientes aseveraciones sobre su tratamiento no es adecuada? RESPUESTA a.- Pulsos de ciclofosfamida 1 g intravenoso (I.V.) mensuales por 6 meses y luego pulsos de ciclofosfamida 1 g I.V. cada 3 meses por un año. b.- Pulsos de Metilprednisolona 1 g I.V. cada día por 7 días, luego se administra 1 mg/kg/día por 6 semanas y posterior reducción gradual. c.- La combinación de ciclofosfamida y metilprednisolona se utiliza para inducir la remisión del cuadro. d.- Leuprolide 11,5 mg I.M. cada mensual, para evitar amenorrea como efecto secundario de la ciclofosfamida. PREGUNTA Según los criterios diagnósticos de GW según el American College of Rheumatology (ACR), son: Sedimento urinario anormal (cilindros hemáticos o más de 5 eritrocitos por campo de alto poder). Radiografía de tórax con hallazgos anormales (nódulos, cavidades o infiltrados). Ulceras orales o secreción nasal; y biopsia con inflamación granulomatosa. Cuantos criterios se requieren para confirmar el diagnostico? RESPUESTA a.- 1. b.- 2. c.- 3. d.- 4 CASO CLINICO Paciente femenina, de 44 años de edad, con antecedentes patológicos personales de asma bronquial de inicio tardío (35 años), quien llevaba tratamiento esteroideo regular en los períodos de crisis, y que comenzó 4 años después con entumecimiento y debilidad en ambos miembros inferiores, por lo que se interpretó como un síndrome polineuropático. A los 3 meses comenzó con ligera mejoría del cuadro, fue ingresada en varias ocasiones con la misma impresión diagnóstica pero agrego gran debilidad en ambos miembros inferiores. Al interrogatorio se constató: asma bronquial de inicio tardío precedida de rinitis, fenómeno de Raynaud, diarreas frecuentes, con antecedentes de gastritis. Al examen físico: cuadriparesia con predominio en miembros inferiores, con cambios tróficos dístales, dedos en martillo con aumento del arco plantar, disminución del tono muscular, así como abolición de los reflejos osteotendinosos. Eosinofilia marcada (valores de 4). El estudio de rayos X de tórax mostró infiltrado inflamatorio. El estudio de conducción nerviosa (ECN) evidenció la presencia de un síndrome polineuropático. Se comenzó tratamiento con corticosteroides, metilprednisolona y se continuó con prednisona y vitaminoterapia, con lo cual la paciente mostró una mejoría significativa. El ciclo de

CURSO ENARM CMN SIGLO XXI TEL: 36246001

Pharmed Solutions Institute

PÁGINA 102

MANUAL DE TRABAJO DEL CURSO ENARM CMN SIGLO XXI tratamiento se repitió cada 2 meses. Al cabo de 6 meses la paciente se encuentra deambulando sin apoyo y con discreta afección del ECN. PREGUNTA Se diagnostico SCS, considerando la patologia cual de las siguientes manifestaciones no es frecuente observar? RESPUESTA a.- Vasculitis de los pequeños vasos. b.- Granulomas vasculares y/o extravasculares. c.- Eosinofilia periférica que ocurre en los pacientes asmáticos sin historia de rinitis alérgica o sinusitis. CASO CLINICO Paciente femenina, de 45 años de edad, con antecedentes de asma bronquial que comenzó a los 27 años. Por un cuadro clínico, 3 meses antes, caracterizado por lesiones en piel, diagnosticado una vasculitis. Comenzó con tratamiento esteroideo con lo que mejoró su cuadro de forma relativa. Posteriormente la paciente comenzó a presentar entumecimiento en el miembro superior derecho y posteriormente en la extremidad inferior contralateral, con ligera impotencia funcional. Después padeció de diarreas, edemas en la cara y en miembros inferiores y cefalea de característica vascular. Se encontró hipoestesia distal, superficial y profunda, cambios tróficos, con disminución de reflejos osteotendinosos, además se encontró el fenómeno de Raynaud, hipereosinofilia y aumento del valor de creatinina (210 ml/L). El ecocardiograma mostró evidencias de pericarditis de pequeña cuantía y el rayos X de tórax evidenció infiltrado inflamatorio. Se encontró evidencia de mononeuropatía multiplex. La TAC de cráneo evidenció la presencia de 3 pequeñas lesiones hipodensas con características vasculares. Se comenzó tratamiento con methilprednisolona, no teniendo una respuesta adecuada, solo con leve mejoría sintomática. La paciente se mantiene actualmente bajo tratamiento inmunosupresor, azatioprina 100/diarios. PREGUNTA Los criterios para la clasificación del SCS 6 criterios: el asma, la eosinofilia mayor que 10 % en recuento de leucocitos diferenciales, la mononeuropatía o polineuropatía, infiltrados pulmonares, anormalidad del seno paranasal y la biopsia de vasos sanguíneos con eosinofilia intra/extravascular. Cuantos criterios son necesarios para establecer el diagnostico? RESPUESTA a.- 2. b.- 3. c.- 4. d.- 5. CASO CLINICO Paciente varón de 62 años que acude a consulta externa para una exploración rutinaria. En la exploración oftalmológica la agudeza visual (AV) fue de 1 en ambos ojos (AO). Con pupilas isocóricas y normorreactivas, sin defectos aferentes, tensión por aplanación de 16 mmHg en AO. En el estudio biomicroscópico anterior destacaba una esclerosis cristaliniana moderada. En la retina del ojo izquierdo (OI) había dos manchas algodonosas asintomáticas, una de 250 micras y otra puntiforme a lo largo de la arcada temporal inferior. Un estudio físico completo no detectó patología. Los estudios de laboratorio fueron normales salvo una proteína C reactiva de 4,4 mg/dl, una velocidad de sedimentación glomerular (VSG) de 56 mm/1.ª hora. PREGUNTA Cual es la conducta a seguir mas adecuada para establecer el diagnostico? RESPUESTA a.- Biopsia de la arteria temporal. b.- Biopsia de la arteria carótida. c.- Biopsia de la occipital. d.- Biopsia de la oftálmica. CASO CLINICO Mujer de 83 años que acude a consulta por disminución bilateral de la agudeza visual. Entre sus antecedentes destacaba cirugía bilateral de cataratas hace 5 años. La AV era de 0,4 en el ojo derecho (OD) y 0,3 en OI. La tensión por aplanación fue de 18 mmHg en OD y de 16 mmHg en OI, con pupilas isocóricas y normorreactivas. En la exploración fundoscópica destacaba una esclerosis coroidea y una mancha algodonosa de 20 micras de diámetro en el OD. La exploración física demostró una dificultad importantísima en la movilidad con claudicación de las extremidades inferiores, astenia, un leve soplo carotídeo, cefalea en casco durante 8 meses y unas arterias temporales endurecidas. Durante el estudio (en 4 días) la paciente desarrolló una neuritis óptica anterior en el OD y con velocidad de sedimentación glomerular de 94 mm/1.ª hora, proteína C de 5,85 mg/dl, 400 Mil/mm3 plaquetas, anemia hipoproliferativa leve y elevación de la fosfatasa alcalina (134 UI/L). PREGUNTA Considerando las sintomatologia oftalmológica cuales la causa mas frecuente? RESPUESTA

CURSO ENARM CMN SIGLO XXI TEL: 36246001

Pharmed Solutions Institute

PÁGINA 103

MANUAL DE TRABAJO DEL CURSO ENARM CMN SIGLO XXI a.- Diabetes mellitus. b.- Hipertensión arterial. c.- Colagenopatías. d.- Patología tumoral. CASO CLINICO Mujer de 27 años que consultó por un cuadro de un mes de evolución de dolor en brazo izquierdo acompañado de parestesias y paresias homoloterales que aumentaban con la actividad y cedían con el reposo. Antecedentes: 12 años antes: Fiebre Reumática tratada con penicilina benzatínica durante 7 años, 4 años antes: Embarazo ectópico en trompa de Falopio con ooforectomía izquierda 3 años antes: Episodio de dolor lumbar de comienzo súbito durante una actividad física tratado con corticoides que abandonó por cuenta propia 2 años antes: Diagnóstico de anemia, tratada con hierro intramuscular durante un año 1 año antes: Disminución de la agudeza visual y fotofobia durante la actividad física que persiste hasta la actualidad. Episodio de taquicardia y disnea de comienzo súbito. Se diagnosticó taquicardia sinusal y comenzó tratamiento con atenolol 8 meses antes: Cefalea de inicio súbito, intensa, holocraneana acompañada de hipertensión arterial en tratamiento con enalapril. Examen Físico: Signos vitales: PA 140/60 mmHg en brazo derecho, inaudible en brazo izquierdo; FC 96 lpm; FR 18 cpm; Tº 36.1º C. Cabeza y Cuello: pulsos carotídeos asimétricos con disminución franca del lado izquierdo, frémito carotídeo derecho, soplo carotídeo bilateral. Cardiovascular: soplo sistólico 5/6 en focos aórtico y pulmonar que irradia a cuello y aumenta con la inspiración. Pulsos radial, femoral, tibial posterior y pedio asimétricos, con disminución en hemicuerpo izquierdo. Abdomen: hígado palpable a 1cm del reborde costal, Miembros: disminución de la fuerza en hemicuerpo izquierdo. Tono, trofismo, temperatura conservada. Hematocrito 36%, Hemoglobina 12 g/dL, Glóbulos blancos 9300/mm3, Plaquetas 284000/mm3, Glicemia 112 mg/dL, Urea 37 mg/dL, Creatinina 0.6 mg/dL, Na 134 mEq/L, K 3.3 mEq/L, plasma no ictérico, ASAT 17 UI/L, ALAT 37 UI/L, Proteínas totales 8.8 g/dL, Albúmina 4.7 g/dL, Tiempo de protrombina 13.3 segundos, KPTT 33 segundos, Velocidad de eritrosedimentación 70 mm/1º hora. PREGUNTA Ante la presencia de un cuadro clínico con manifestaciones sistémicas, anemia, velocidad de eritrosedimentación (VES) elevada y alteración de la aorta y sus ramas, debe pensarse en aortitis, de las siguientes patologías cual es la menos frecuente? RESPUESTA a.- Espondolitis anquilosante. b.- Arteritis de células gigantes. c.- Enfermedad de Behcét. d.- Sindrome de Marfan. PREGUNTA Considerando su diagnostico, cual es la indicación mas frecuente para revascularizar? RESPUESTA a.- Hipertensión arterial renovascular. b.- Claudicación severa de los miembros. c.- Isquemia mesentérica d.- Infarto agudo de miocardio CASO CLINICO Un varón de 22 años, consumidor ocasional de cocaína y fumador de 20 cigarrillos/día, fue remitido a nuestro centro por dolor torácico. La exploración física y la analítica de control fueron normales. El electrocardiograma mostró bloqueo de rama derecha con ondas q en V1, V2 y V3. La radiografía de tórax mostró una imagen calcificada, redondeada superpuesta a la silueta cardiaca. En el ecocardiograma se objetivó un ventrículo izquierdo dilatado y adelgazado, con hipocinesia general más marcada en el septo y la cara anterior, con disfunción sistólica severa (fracción de eyección del 30%). Se realizó una coronariografía que mostró oclusión completa de la arteria descendente anterior distal a un aneurisma gigante calcificado y oclusión completa de la coronaria derecha. PREGUNTA Cual es el porcentaje de afeccion a las arterias coronarias en esta patologia? RESPUESTA a.- 25 %. b.- 35 %. c.- 45 % d.- 55 %. PREGUNTA El diagnóstico clásico de la EK se basa en la presencia de los siguientes síntomas, cual no es frecuente? RESPUESTA a.- Fiebre. b.- Alteraciones cutáneas en las extremidades. c.- Exantema polimórfico.

CURSO ENARM CMN SIGLO XXI TEL: 36246001

Pharmed Solutions Institute

PÁGINA 104

MANUAL DE TRABAJO DEL CURSO ENARM CMN SIGLO XXI d.- Conjuntivitis bilateral exudativa. CASO CLINICO Femenino de 14 años de edad, acude a consulta por presentar sangramiento en las encías desde hace varios meses, refiere la madre que el estomatólogo le indicó el cepillado correcto y buchadas de manzanilla 3 veces al día. Examen físico: Se observa presencia de caries, encías edematosas, al sondeo bolsas de 4mm en el sector anteroinferior (brecha edente de 3.1-3.2), perdidos por movilidad, no restituidos por prótesis, sarro supragingival y manchas de origen medicamentoso. Se aprecia pigmentación melánica en la encía adherida, hay xerostomia, lengua lisa y brillante e higiene deficiente. PREGUNTA Cual es la conducta a seguir en relación al caso? RESPUESTA a.- Yodopovidona oral. b.- Pilocarpina oral. c.- Clorhexidina oral. d.- Prednisona oral. CASO CLINICO Mujer hipertensa de 74 años. Unas semanas antes comienza con astenia, fatigabilidad y disnea progresiva, dolor torácico de características inespecíficas, hipotensión sintomática y síncope; no refería fiebre. En el momento del ingreso presentaba una presión arterial sistólica (PAS) de 90 mmHg, un electrocardiograma con taquicardia sinusal, bloqueo de rama derecha más hemibloqueo anterior izquierdo, con PR normal y cardiomegalia-congestión en la radiografía de tórax. Analítica básica: creatinina 1,4 mg/dl, Na 121 mEq/l, K 5,6 mEq/l, aclaramiento de creatinina 95 ml/min, microalbuminuria 9,4 mg/dl, troponina I 2,4 ng/ml, cLDL 78 mg/dl, GOT-GTP 239 U/l, GGT 136 U/l, FA 154 U/l, LDH 904 U/l. Leucocitos 5,9 3 103/ μ l; hemoglobina 11 g/dl; plaquetas 233 3 103/ μ l; actividad de protrombina 87%, VSG 133 mm en la primera hora y PCR 13,2 mg/dl. Hemocultivos y serologías varias negativas. La ecocardiografía en el momento del ingreso muestra un ventrículo izquierdo no dilatado, sin alteraciones segmentarias de la contractilidad y disfunción sistólica severa, biventricular, con fracción de eyección del ventrículo izquierdo (FEVI) del 15%, sin otros hallazgos. PREGUNTA Considerando los hallazgos clínicos mas relevantes cual es el fármaco de elección. RESPUESTA a.- Corticosteroides. b.- Azatioprina c.- Metotrexato d.- Ciclofosfamida. TUBERCULOSIS: CIENCIAS BASICAS: Es una enfermedad infecciosa que suele afectar a los pulmones y es causada por el complejo Mycobacterium tuberculosis (M. tuberculosis, M. bovis, M. africanum). La infección por M. tuberculosis suele ser asintomática en personas sanas, dado que su sistema inmunitario actúa formando una barrera alrededor de la bacteria. SALUD PÚBLICA: Considerada una emergencia mundial por la OMS, es prioridad casos complicados en los extremos de la edad, multifármacorresistencia y vínculo con el sida y la diabetes, e inmunocomprometidos (neoplasias, quimioterapias, trasplantes). OMS informa que 1/3 de la población mundial está infectada por Mycobacterium tuberculosis. Cada año: >10 millones de casos nuevos y 3.5 millones de defunciones por TB. En África y este de Europa: incidencia es de >300 casos/100,000 hab. por año. La TB que responde al tratamiento estándar tiene una tasa de curación >95%, en multiresistencia es de 50-80%. Menos de 30% de los inmunocompetentes logran la curación y >del 50% muere dentro de los primeros 5 años. En multiresistencia la tasa de mortalidad es de 90%. PATOGENIA: La transmisión de bacilos tuberculosos se produce básicamente por vía aérea (también ingestión de leche de vaca infectada, contacto con personas enfermeas baciliferas o animales bovinos enfermos). Las personas infectantes eliminan bacilos a partir de aerosoles (tos, expectoración) y la infecciosidad depende del número de bacilos eliminados y la susceptibilidad del huésped. Las partículas aerosolizadas que contienen bacilos, son suficientemente pequeñas para eludir la 1ª barrera defensiva (aparato muco-ciliar), para alcanzar los alveolos pulmonares, donde comienza la multiplicación de los bacilos, son fagocitados por lo macrófagos alveolares, se liberan multiples citocinas proinflamatorias, TNF (ocasiona fiebre y caquexia), IL-1 y IL-6, además liberan proteasas, urocinasa, implicadas en la destrucción tisular. Secundariamente los bacilos son transportados por los propios macrófagos a los ganglios regionales donde se produce la respuesta inmunitaria mediada fundamentalmente por los linfocitos T (inmunidad celular). Período de incubación que oscila entre 6 a 8 semanas. Puede ocurrir que antes del desarrollo de la respuesta inmunitaria celular se produzca una diseminación vía linfo-hematógena que dé lugar a siembra de bacilos en diversos tejidos: zonas apicales de pulmón, vértebras, epífisis de huesos largos etc. que condicionen la evolución ulterior a enfermedad progresiva tras períodos largos de latencia. En la mayoría de los casos de infección tuberculosa, hay una destrucción rápida de bacilos y no se produce enfermedad, el único indicio residual es la positividad de la PPD. El granuloma se forma por la interacción del M. tuberculosis, con la respuesta inmune del huésped y liberación tisular de proteasas; al inicio se hallan neutrofilos, mas tarde necrosis caseosa (típica de granulomas producidos por micobacterias) y finalmente calcificaciones. El bacilo no siempre es eliminado y permanece inactivo, causando una infección latente. INFECCION PRIMARIA: Afecta pulmón, se adquiere al inhalar bacilo, se desarrolla después de una exposición inicial. Se necesitan de 50-200 microorganismos para producir la infección. De las personas expuestas al bacilo 30% se infecta y 5% desarrolla enfermedad. Los bacilos inhalados causan alveolitis, adenopatías y linfagitis, lo que se conoce como complejo primario de Ghon. Clínica; hemoptisis, dolor pleurítico, disnea, fiebre, diaforesis nocturna y pérdida de peso.

CURSO ENARM CMN SIGLO XXI TEL: 36246001

Pharmed Solutions Institute

PÁGINA 105

MANUAL DE TRABAJO DEL CURSO ENARM CMN SIGLO XXI Complicación: TB endobronquial. REACTIVACION: Es progresiva se desarrolla de un foco previo de TB, el cual puede producir TB pulmonar o extrapulmonar. Más común en inmunocomprometidos (IRC, DM, esteroides, desnutrición, deficiencia de vitamina D. Clínica; tos, pérdida de peso, fatiga, fiebre y diaforesis nocturna. Complicaciones agudas; hemoptisis y neumotórax. Cavitaciones en (20-45%), manifestación radiológica más común consolidación Los aspergillomas se desarrollan dentro de la cavitación (20%). Hasta en 5% de pacientes reactivación la manifestación principal es el tuberculoma (centro: material caseoso y periferia histiocitos epiteliales y cel. Gigantes multinucleadas). DEFINICIONES (Estándares para la atención de la TB en México): Caso de tuberculosis: Persona en quien se establece el diagnostico de tuberculosis pulmonar o extrapulmonar y se clasifica por bacteriología o estudio histopatológico en caso confirmado o no confirmado. Caso de tuberculosis confirmado: Toda persona con cuadro clínico compatible con tuberculosis pulmonar o extrapulmonar que cumpla además cualesquiera de los siguientes criterios: 1. Aislamiento de Mycobacterium tuberculosis por cultivo. 2. Resultado positivo en la baciloscopia. 3. Deteccion de genes de micobacterias por métodos de biología molecular (PCR o ampliación de RNA). Caso de tuberculosis no confirmado: Toda persona con cuadro clínico compatible con tuberculosis pulmonar o extrapulmonar sin confirmación por baciloscopía, cultivo o estudios de biología molecular pero presenta uno o mas criterios como: 1. Cuantificación de adenosin desaminasa (ADA): los niveles de ADA parab TB peritoneal y pleural, están >70U/ml; para SNC >7U/l. 2. Antecedente epidemiológico de convivencia con un caso bacilifero. 3. Reactor a la PPD, con o sin antecedente de BCG. 4. Respuesta favorable al tratamiento antituberculoso. 5. Estudio histopatológico compatible con TB. Y alguno de los siguientes estudios de gabinete: A. TB pulmonar; las radiografías de tórax PA y lateral pueden mostrar imagen de síndrome del lóbulo medio, derrame pleural, ensanchamiento mediastinal o patrón miliar. B. TB ganglionar; ultrasonido con imágenes de material calcificado y liquido. C. TB del SNC; la TAC de cráneo puede mostrar datos de aracnoiditis e hidrocefalia; la radiografía de cráneo, datos de hipertensión endocraneana. D. TB osea o enfermedad de Pott: las rx., de columna vertebral AP y lateral muestran destrucción de las vertebras dorsolumbares y rotoxifoescoliosis. E. TB geniturinaria: urografía excretora muestra imágenes tortuosas debido a rigidez de uréteres. F. TB abdominal; USG o TAC muestran ascitis e imágenes compatibles con tabicaciones. La laparoscopia muestra lesiones granulomatosas ascitis y fibrina. DIAGNOSTICO: Búsqueda entre contactos de un caso de TB y en grupos o poblaciones de alto riesgo. La baciloscopía se debe realizar en A) En cualquier muestra clínica excepto orina. B) En todos los casos probables, entre los contactos de un caso, en grupos o poblaciones de alto riesgo, pacientes dados de alta que acudan con tos productiva, en 3 muestras sucesivas de esputo. C) Independientemente de la edad, en en quienes clínica y radiológicamente, se sospecha TB, si la primera serie de 3 hubiera sido negativa. D) En el control del tratamiento antituberculoso, con una muestra cada mes, la ultima al terminar el tratamiento E) En caso de sospecha de TB extrapulmonar, F) Sospecha de TB en niños. G) Pacientes en tx., estrictamente supervisado, en quienes al cuarto mes, persiste baciloscopia positiva. H) Para confirmar fracaso de tratamiento. Prueba de la tuberculina (PPD), con derivado de proteína puirificado, tiene un papellimitado en el diagnostico de TB activa. Reactor al PPD, persona que a las 72hrs presenta induración intradérmica de 10mm o mas en el sitio de la aplicación. En menores de 5 años con o sin BCG, recién nacidos, niños desnutridos y personas inmunodeprimidas, la induración de 5mm o mas. Clínico; TB pulmonar activa son tos, a veces con esputo que puede ser sanguinolento, dolor torácico, debilidad, pérdida de peso, fiebre y sudoración nocturna. RADIOLOGICO: Linfadenopatía; alteración más común niños (9095%), adultos (40%), más frecuentes unilaterales, paratraqueal derecho e hiliar y su frecuencia disminuye con edad. La combinación de ganglios hiliares calcificados y focos de Ghon, se conoce como Complejo de Ranke (sugiere TB previa, histoplasmosis). En infección primaria: se puede observar focos de consolidación generalmente en lóbulo medio o inferior, derrame pleural (60%), más unilateral, se presenta 3-7 meses después de la exposición inicial. En reactivación: afección del segmento apical y posterior de lóbulos superiores (85%), la cavitación secundaria a necrosis caseosa, se observa como nivel hidroaereo; se pueden ver atelectasias lobares. TAC: Los ganglios muestran hipodensidad central (necrosis caseosa) y reforzamiento periférica, que representa el anillo vascular de tejido granulomatosos inflamatorio que indica enfermedad activa. En pacientes de alto riesgo, se pueden formar cavitaciones, diseminación hematogena y tuberculosis miliar. TUBERCULOSIS MILIAR: Se da por una infección diseminada por vía hematogena. Afecta de 1-7%, frecuente en ancianos, lactantes e inmunocomprometidos. Suele manifestase con fiebre, con frecuencia existen granulomas pulmonares, afeccion del SNC, en raros casos sx. de dificultad respiratoria aguda con mortalidad de 90%. En casos crónicos caquexia, lesiones dérmicas maculares o papulares y tuberculos coroides (2-3mm), estos últimos patognomónicos de tuberculosis miliar. RX: Múltiples imágenes nodulares 100 000 parásitos/microl de sangre con un >5-10% de eritrocitos parasitados, hemoglobina debajo de 7g/dl, ictericia. COMPLICACIONES: Malaria cerebral, insuficiencia renal, fiebre biliosa hemoglobinurica, anemia severa, edema pulmonar, daño hepático, hemorragia, coagulopatia. TRATAMIENTO: Actualmente se dispone de la combinación de artesunato (4mg/kg) y mefloquina en una sola tableta para falciparum, no complicado. Para P. vivax, el tratamiento de elección para cura radical es primaquina (0.25-0.5 mg/kg), es esquizonticida mas cloroquina (25mg/kg), son tabletas de 150mgs, por 3 días. La OMS considera que el fármaco de elección para P. vivax es cloroquina, las combinaciones de artesunato, se recomiendan en casos de P. vivax resistente a cloroquina. La quinina, es un ezquizonticida hemático muy eficaz, gametocida eficaz contra P. vivax y P. malarie, inactiva frente a P. falciparum. Artemisina por via parenteral acción rápida

CURSO ENARM CMN SIGLO XXI TEL: 36246001

Pharmed Solutions Institute

PÁGINA 108

MANUAL DE TRABAJO DEL CURSO ENARM CMN SIGLO XXI empleado para P. falciparum resistente a cloroquina, sus derivados son artesunato y artemer ambos en presentación via oral y parenteral. La ONU recomienda no emplear artemisina en monoterpia, porque acelera la resistencia, debe tratar e agregarse siempre algún otro antipalúdico. Si no se trata en las primeras 24 hrs el paludismo por P. falciparum, puede agravarse llevando a la muerte. En caso de P. vivax y ovale, pueden producirse recidivas clínicas semanas o meses después de la infección inicial, aunque el paciente haya abandonado la zona palúdica. Estos nuevos episodios se deben a formas hepáticas "durmientes" del parasito (inexistentes en falciparum, malariae) y para lograra curación completa es obligatorio tratamiento dirigido a esas formas hepáticas. PREVENCIÓN: La lucha antivectorial es el medio principal de reducir la transmisión del paludismo. Algunas medidas son: mosquitos tratados con insecticidas (peritroides), fumigación de interiores con insecticidas, vigilancia continua y eliminación del mismo. CASO CLINICO Masculino de 35 años de edad residente del estado de mexico, inicia padecimiento 9 dias posteriores a su regreso de la ribera del rio Niger en Malí donde permaneció 30 dias, cursando con un cuadro clínico caracterizado por fiebre de 40 C, de predominio vespertino y nocturno terciada que se controlaba con paracetamol 500 mg cada 8 hrs. Al cuadro clínico se agrego estreñimiento, disfagia, tos no productiva, cefalea 8/10 holocraneana y perdida ponderal de 4 kg en una semana. EF. Temp.39.2 C FC 127, FR 24, TA 122/72 mmHg, somnoliento, orientado en tres esferas, mucosas secas, ruidos cardiacos aumentados en frecuencia sin agregados, campos pulmonares bien ventilados, abdomen distendido, doloroso a la palpación media y profunda en ambos hipocondrios, timpánico a la percusión. PREGUNTA Cual es la conducta a seguir en el caso? RESPUESTA a.- Artesunato/mefloquina. b.- Cloroquina. c.- Cloroquina/mefloquina. d.- Mefloquina. CASO CLINICO Ingresa paciente masculino de 48 años de edad, ingeniero que acaba de regresar de convención en medio oriente, presenta fiebre, diaforesis, ataque al estado generalizado, fatiga y adinamia, así como obnubilación, a la exploración se observa hepato y esplegnomegalia leve a moderada, es ingresado para realizar estudios encontrando paludismo por plasmodium falciparum, parasitemia de 7 %, hematocrito de 21 %, bilirrubina de 7.8 mg/100 ml, creatinina 2.6 mg/100 ml. Se ingresa posteriormente a cuidados intensivos, se realiza asistencia ventilatoria y se indica neuroproteccion debido a la gravedad del cuadro. PREGUNTA Cual es el mejor esquema de tratamiento para este caso: RESPUESTA a.- Quinina intravenosa. b.- Quinidina Intravenosa. c.- Artesunato Intravenoso. d.- Cloroquina. DENGUE: CIENCIAS BASICAS: Enfermedad viral de carácter endemico-epidemico. Producido por un arbovirus (flaviviridae). Transmitido por Aedes aegypti (vector), compuesto de ARN. El cual tiene 4serotipos (DE 1,2,3,4), el 2 es el serio tipo más peligroso. Cada serio tipo proporciona una inmunidad específica para toda la vida, así como inmunidad cruzada a corto plazo. Dos clases de dengue el clásico y el hemorrágico. SALUD PÚBLICA: México ocupa el segundo lugar en casos y defunciones de América, después de Brasil, mayormente entre 10-19 Años, ligeramente más en sexo femenino.OMS; Entre 50-100 millones por año en el mundo, 500mil hospitalizados. 50mil muertes en más de 100 países y 2 mil millones de personas en riesgo. Mayor incidencia en épocas de lluvia. En México hay condiciones que propician la presencia de mosquitos, temperatura lluvias constantes, condiciones inadecuadas de saneamiento e higiene. PATOGENIA: EL virus ingresa al organismo por la picadura del artrópodo se replica en los macrófagos y monocitos, lo que produce supresión de la medula ósea. El periodo de incubación es de 5-7 días. Se desarrolla una respuesta inmune que acaba en el periodo de viremía e induce producción duradera. Existen 3 etapas: 1. Febril; se da entre el 3-6to días, es variable, presencia de virus en sangre (viremía). En esta etapa no es posible reconocer si el paciente va a evolucionar a la curación espontánea o si es el comienzo de un dengue grave. Al disminuir la fiebre, el dolor abdominal se hace intenso y mantenido, se puede observar derrame plural o ascitis. 2. Crítico; momento de mayor frecuencia de instalación de choque. Hay extravasación de plasma (vasodilatación), su manifestación más grave el choque, grandes hemorragias digestivas, alteraciones hepáticas y quizá de otros órganos, ascitis o derrame pleural derecho o bilateral, aumento de hematocrito (la máxima elevación de este coincide con el choque) y disminución de plaquetas. 3. Recuperación: evidente mejoría del paciente a veces hay sobrecarga de liquido, así como coinfeccion bacteriana. Esta etapa es la de mayor riesgo de aparición de complicaciones, hay que vigilar y controlar al paciente 48hrs posteriores al cese de la fiebre, los signos de alarma son: dolor abdominal intenso y continuo, vómitos frecuentes, somnolencia o irritabilidad, derrame seroso (en peritoneo, pleura o pericardio), sangrado de mucosa, hepatomegalia (>2cm), aumento rápido de hematocrito y disminución rápida de plaquetas. DIAGNOSTICO: Dengue clásico: Inicio repentino, fiebre (de menos de 7 días) bradicardia, pulso lento, ex antena máculas punto forme, rinitis, tos, ardor faríngeo, adenopatías, fatiga, cambios en el sentido del gusto, anorexia, cefaleas, mialgias, artralgias, dolor óseo y retroorbitario, congestión conjuntival, edema palpebral, petequias, equimosis, miocarditis. Dengue hemorrágico: Mas grave, incremento en permeabilidad vascular. El choque depende de múltiples factores como la presencia de anticuerpos no neutralizantes. Dengue grave;

CURSO ENARM CMN SIGLO XXI TEL: 36246001

Pharmed Solutions Institute

PÁGINA 109

MANUAL DE TRABAJO DEL CURSO ENARM CMN SIGLO XXI shock hipovolemico por fuga de plasma, diestres respiratorio por acumulación de líquidos, sangrado grave, daño orgánico importante. BH; leucopenia y trombocitopenia, a veces transaminasas elevadas. ELISA, para determinación de anticuerpo IgM e IgG antidengue, se puede tomar prueba pareada una en fase aguda y otra en convalecencia. PCR o ELISA se usan para detección de antígenos virales. Anticuerpos IgM específicos para virus de dengue: Si es negativo se descarta el diagnostico de dengue; si es positivo es un caso probable de dengue, se requiere una segunda muestra para confirmar mediante prueba de neutralización; negativo=se descarta caso de dengue, constante=caso de dengue anterior, negativo=se confirma dengue. CASO PROBABLE DE FIEBRE HEMORRÁGICA POR DENGUE: Toda persona que además de un cuadro de probable fiebre por dengue desarrolle fiebre persistente y uno o más de los siguientes: A) Datos de fuga de plasma (ascitis, derrame pleural, edema, hipoalbuminemia). B) Datos de fragilidad capilar (petequias, equimosis, hematomas). C) Hemorragias (gingivorragia, hematemesis, metrorragia). D) Tombocitopenia 25 años. PATOGENIA: El modo de transmisión e invasión sigue siendo controvertido. Es de notar que pacientes con lepralepromatosa la mayor diseminación del bacilo al ambiente se realiza a través de la descarga nasal, el estornudo y la tos; por ello se plantea que la principal ruta de infección es la via aérea (microgotas), la diseminación a través de la piel es menos importante. Incubación de 2-4 años o más. Para adquirir la infección se requiere la convivencia estrecha y prolongada con un enfermo bacilifero y con inmunidad celular disminuida identificada. DIAGNOSTICO: Clínica; En piel maculas, pápulas, nódulos, placas o infiltraciones, afectando especialmente macrófagos. La hipopigmentación o eritema de la piel con déficit sensorial (hiperestesia, hipoestesia o anestesia), es uno de los signos clínicos más importantes en el diagnostico, se observan en la palma de la mano o planta del pie, y los origina una infección de la bacteria en las fibras nerviosas (engrosamiento de troncos nerviosos periféricos), alteraciones motoras. El daño al sistema nervioso es una característica, a pesar de no tener habilidad locomotora, puede moverse en el endotelio a través del tejido conectivo y alcanzar las células de Schwan. Las pruebas inmunológicas son muy limitadas, no existe un examen confirmatorio, baciloscopia positiva con resultado completo, detección de anticuerpos anti-PGL-I, donde se presentan grandes cantidades de IgM. ELISA, pero tienen sensibilida y especificidad baja. El diagnostico histopatológico es obligatorio para el pronóstico y de esta manera favorecer el tratamiento, se pueden obsrevar infiltrados en la dermis, hipodermis y órganos internos tales como células de Virchow, que son macrófagos con muchos bacilos y gotas de lípidos en su citoplasma, con apariencia espumosa. En secreciones nasales y cutáneas el hallazgo del bacilo aislado en forma de globia puede ser detectado cpon coloración de Zieehl Neelsen. CLASIFICACION: Para fines de control sanitario los casos se clasifican en: Multibacilares (MB); los lepromatosos, dimorfos y con infiltración difusa . Paucibacilares (PB); los tuberculoides e indeterminados. Clasificación clínica: Características Tipo de lesión Numero lesiones

de

Distribución de las lesiones Definición de las lesiones Sensibilidad Bacilos en las lesiones en la piel

5 LEPROMATOSA (progresa a necrosis y ulceración) Maculas, pápulas, nódulos, infiltración difusa Numerosas

3 BODERLINE LEPROMATOSA Maculas, pápulas, nódulos, infiltración Muchas

4 BODERLINE BODERLINE (dimorfa) Placas y lesiones en forma de cúpula y sacabocados Muchas

Simétricas (afecta más labios, encías, dorso de lengua, paladar duro) Imprecisas, difícil de definir la enfermedad y la afección de la piel

Con tendencia simetría

Evidentemente Asimétricas

No afectada Muchas globias

Disminuida Muchos

a

la

Imprecisas delimitadas por los bordes externos

Imprecisas delimitadas por los bordes externos mal definido Disminuida Muchos

2 BODERLINE TUBERCULOIDE Placas de infiltrados

1 TUBERCULOIDE

Única, usualmente con lesiones satélites o >5 lesiones No difusas y asimétricas

10 % de linfocitos atípicos. El aumento de los niveles séricos de transaminasas y fosfatasa alcalina así como anormalidades inmunológicas (por ejemplo, la presencia de crioglobulinas o aglutininas frías) pueden ser evidentes. La recuperación es completa, pero astenia postviral puede persistir durante meses. La excreción de CMV en la orina, las secreciones genitales, y/o la saliva pueden continuar durante meses o años. CMV es el patógeno viral más común e importante que complica el trasplante de órganos, la infección por CMV es un factor de riesgo de pérdida del injerto y muerte. El riesgo de infección es mayor 1-4 meses después del trasplante, pero la retinitis puede ocurrir más tarde. El órgano trasplantado es un riesgo especial, por ejemplo, neumonitis por CMV suele seguir un trasplante de pulmón. Alrededor del 15-20% de los receptores de trasplante de médula ósea va a desarrollar neumonía por CMV, 5-13 semanas después del trasplante, con una tasa de letalidad del 84-88 %. El riesgo de enfermedad grave se puede reducir por la profilaxis o terapia antiviral preventiva. El CMV es un patógeno importante en pacientes con infección por el VIH cuyo recuento de células CD4 + han caído por debajo de 50 a 100 / l. En esta configuración, el CMV causa retinitis, colitis y enfermedad diseminada. Pacientes inmunodeprimidos con infección por CMV desarrollan fiebre, malestar general, anorexia, fatiga, sudores nocturnos y artralgias o mialgias. Taquipnea, hipoxia, y tos no productiva. La participación del tracto gastrointestinal puede ser localizada o extensa, con úlceras que pueden sangrar o perforar. La hepatitis es común. SNC, con mayor frecuencia afectan a pacientes infectados por el VIH, retinitis grave, encefalitis, polirradiculomielopatía. Retinitis por CMV puede dar lugar a ceguera. Las lesiones comienzan como pequeñas áreas blancas de necrosis retiniana granular, con el desarrollo posterior de las hemorragias y edema retiniano. Infección grave se asocia con viremía persistente y compromiso multiorgánico. Amplia necrosis suprarrenal se ve a menudo en la autopsia. El cultivo viral (diagnostico de certeza) y los anticuerpos monoclonales se utilizan para detectar el antígeno temprano inmediato de CMV. La detección de antígenos de CMV en los leucocitos de sangre periférica o de ADN de CMV en sangre. Los anticuerpos no pueden ser detectados por un máximo de 4 semanas de la infección primaria, y los títulos pueden permanecer elevados durante años. IgM puede ser útil en el diagnóstico de infección aguda. Cuando sea posible, los donantes seronegativos deben ser utilizados para los receptores de trasplante seronegativos. TRATAMIENTO: Ganciclovir (o valganciclovir, el profármaco oral de ganciclovir ) se usa como tratamiento y como profiláctico, produce tasas de respuesta del 70-90 % entre los infectados por el VIH, pacientes con retinitis por CMV o colitis. La terapia de inducción con ganciclovir (5-12mg/kg/día IV 2-7 semanas) o valganciclovir (900 mg/24hrs VO) se da durante 14-21 días. En los receptores de trasplante, se ha demostrado reducción de la presencia de CMV, cuando se administra tratamiento profiláctico a base de vanganciclovir cuando el paciente es susceptible. Actualmente se puede utilizar el fármaco durante los primeros 100 días postrasplante. La dosis de inducción es de 900mg c/12 hrs y después 900mg c/24 hrs. Otros fármacos activos contra CMV son: El foscarnet es activa contra la infección por CMV, pero se reserva para casos de insuficiencia a ganciclovir o intolerancia a causa de sus efectos tóxicos, que incluyen la disfunción renal, hipomagnesemia , hipopotasemia , hipocalcemia , y parestesia. Este medicamento debe ser dado a través de una bomba de infusión, y su administración debe ser monitoreado de cerca. Un régimen de inducción de 60 mg / kg cada 8 horas o 90 mg / kg cada 12 horas durante 2 semanas es seguido por los regímenes de mantenimiento de 90-120 mg / kg al día. Cidofovir (en casos de retinitis) tiene una intracelular larga vida media. Los regímenes de inducción de 5 mg / kg por semana durante 2 semanas son seguidos por los regímenes de mantenimiento de 3-5 mg / kg cada 2 semanas. Cidofovir produce nefrotoxicidad grave por lesión de las células del túbulo proximal. El uso de la hidratación salina y probenecid reduce este efecto adverso. CASO CLINICO Hombre de 53 años, sometido a trasplante renal hace un mes, que acude a urgencias por cuadro febril con temperatura de 38,5 ºC de 24 horas de evolución con escalofríos, dolor abdominal leve difuso, astenia, anorexia y disminución subjetiva del volumen de diuresis. El cuadro progresó con un aumento del dolor abdominal, aparecieron nuevos picos febriles junto con cuadro diarreico, con melena, asociado a deterioro neurológico, hepatoesplenomegalia, así como alteración función hepática, anemia y trombocitopenia. Analítica: GOT/GPT 135/156 U/l; LDH 558 U/l; sodio 130 meq/l, fibrinógeno 133 mg/dl, hemoglobina 9,2 g/dl, y hematocrito del 26,8% y plaquetas de 48.000 µl con leucocitos normales (5.500 µl con fórmula normal), aumento muy importante de triglicéridos (738 mg/dl), deterioro progresivo de función renal (creatinina entorno a 4-5 mg/dl). Se solicitaron nuevas pruebas en las que destacaron: antígeno precoz CMV positivo con PCR de CMV mayor a 100.000 copias/ml. PREGUNTA Considerando la complicación, cual es la medida profiláctica mas apropiada? RESPUESTA a.- Ganciclovir 200 mg por 100 días. b.- Aciclovir 200 mg por 75 dias. c.- Cidofovir 500 por 100 dias. d.- Oseltanmivir 100 mg por 74 dias. CASO CLINICO Femenino de 42 años. Manifiesta 8 meses con herpes recurrente en boca. Ingresa por padecimiento de 2 meses con tos seca en accesos y disnea rápidamente progresiva. Además dolor torácico bilateral, fiebre hasta 39°C y pérdida de peso de 14kg. La recurrencia de la lesión herpética le ocasionaba disfagia y odinofagia. Al examen físico presentaba placas blanquecinas en orofaringe; la exploración del tórax, con disminución del ruido respiratorio y estertores finos. Al aire ambiente la saturación de oxígeno era del 86%. El reporte de la gasometría arterial con oxígeno suplementario al 70% fue: pH 7,30, pCO2 40,5mmHg, pO2 132mmHg, HCO3 19,5mmol/l, exceso de base −5,8mmol/l, saturación de oxígeno al 97,9%. Índice de oxigenación (IO) de 188. Laboratorio: linfopenia de 600células/mm3, Hb

CURSO ENARM CMN SIGLO XXI TEL: 36246001

Pharmed Solutions Institute

PÁGINA 118

MANUAL DE TRABAJO DEL CURSO ENARM CMN SIGLO XXI 11,8gr/dl, deshidrogenasa láctica de 971UI/l y albúmina 3,3gr/dl. La Rx de tórax presentaba opacidades bilaterales en parche con vidrio deslustrado y neumomediastino, por lo cual, en el diagnóstico diferencial se incluyó inmunosupresión asociada a VIH y neumonía por P. jirovecii (PJP). El análisis para VIH por ELISA fue POSITIVO, se confirmó por Western Blot. Se le realizó broncoscopia con biopsia transbronquial y lavado broncoalveolar (LBA). El estudio histopatológico se reporta en la figura 1. Recibió tratamiento con Trimetoprim/Sulfametoxasol y Prednisona en dosis de reducción por 21 días. Se somete a LBA cuyo estudio de patología confirma la presencia de hemorragia alveolar reciente y activa (fig. 2B). Además, por rt-PCR se documenta infección por CMV PREGUNTA Considerando la complicación, cual es la medida terapeutica mas apropiada? RESPUESTA a.- Ganciclovir. b.- Aciclovir. c.- Cidofovir. d.- Gamaglobulina. MONONUCLEOSIS INFECCIOSA (VIRUS DE EPSTEIN-BARR): CIENCIAS BÁSICAS: Enfermedad causada por el virus de Epstein-Barr (VEB), pertenece a la familia de los gammaherpesvirus (DNA bicatenario), también llamada enfermedad del beso. El VEB se descubrió al observar en el microscopio biopsias de linfoma de Burkitt, un hallazgo fortuito lo asocio a mononucleosis infecciosa. Se trasmite por secreciones de oro faringe y por contacto sexual. SALUD PUBLICA: Distribución mundial, con mayor frecuencia en mujeres adolescentes y adultos jóvenes. La incidencia a nivel mundial se aproxima al 100%. La enfermedad recurrente es una fuente de contagio y puede causar diseminación asintomática. PATOGENIA: El VEB infecta las células epiteliales de la cavidad oral donde se multiplica en los linfocitos, causa lisis celular, pasa a la saliva y se disemina en el tejido linfático, es mitogeno para las células B y permanece en ellas en forma latente. La inmunidad celular permanente participa en el control de la infección. DIAGNOSTICO: Clínica; en niños generalmente es subclínica, a veces, faringitis, otitis, adenopatías cervicales. Periodo de incubación de 10-14 dias, en adultos jóvenes fiebre, adenopatías en 2 cadenas ganglionares, exantema maculopapular, faringitis, fatiga, anorexia, petequias en paladar y frecuentemente hepatoesplenomegalia. Serología, buscar aglutininas heterofilicas (titulo >40) o identificación de cuerpos de inclusión intracelular. En infección aguda se observa IgM e IgG contra el antígeno de la casi de viral (anti-VCA), además del anticuerpo contra el antígeno temprano (anti-EA). COMPLICACIONES: Puede originar trastornos neurológicos como: meningoencefalitis, parálisis de Bell, síndrome de Guillan-Barre, mielitis transversa, neuritis periférica, anemia hemolítica, CID. Se le a asociado carcinoma nasofaríngeo, linfoma de Burkitt y leucoplasia oral vellosa. TRATAMIENTO: No especifico, solo sintomático, reposo para evitar rotura espléndida. Corticoides, si hay obstrucción de vía aérea. CASO CLINICO Varón de 19 años de edad, estudiante, tres semanas antes del ingreso había comenzado con fiebre, adenopatías laterocervicales bilaterales, rash cutáneo y dolor faríngeo. Había sido seguido de forma ambulatoria, pero se decidió su ingreso ante la intensa afectación del estado general. Presentaba leucocitosis (24.500cel/μl) con predominio de linfocitos (71%) que en el frotis tenían aspecto atípico en un alto porcentaje, trombopenia ligera (106.000plaquetas/μl), alargamiento discreto de los tiempos de coagulación (INR 1,7) y datos de colostasis (bilirrubina directa 3,1mg/dl). Tenía un título alto (>1/256) de IgM frente al antígeno de la cápside del VEB. El estudio serológico de CMV, VHA, VHB y VIH era negativo. Una TAC de tórax y abdomen puso de manifiesto la presencia de esplenomegalia, hepatomegalia, ascitis y derrame pleural bilateral. Se instauró tratamiento con metilprednisolona (1mg/kg/día) que se mantuvo durante 30 días. Tras su inicio remitió la fiebre y se produjo una lenta, pero progresiva mejoría en un periodo de 35 días. El paciente fue dado de alta a los 10 días, pero en el seguimiento posterior refería debilidad y dificultad para la concentración durante tres meses más. Los parámetros hepáticos se normalizaron a los 6 meses del inicio de los síntomas. PREGUNTA La manifestación más frecuente de este padecimiento es una mononucleosis severa o fatal, cual de los siguientes cuadros mas frecuente observar como diagnostico diferencial? RESPUESTA a.- Síndrome hemofagocítico asociado a virus. b.- Trastornos linfoproliferativos. c.- Disgammaglobulinemias (hipogammaglobulinemia). d.- Anemia aplástica. RABIA: CIENCIAS BÁSICAS: Zoonosis causada por Rabdovirus, se trasmite al hombre principalmente por la saliva de animales infectados (y en periodo de transmisión) a partir de una mordedura, rasguño o solución de continuidad en la piel o mucosas, se presenta como una encéfalomielitis de curso agudo. PATOGENIA: La velocidad con se manifiesta la rabia, depende de la cepa del virus, de la concentración de receptores para el virus en las células nerviosas del músculo esquelético, de la magnitud del inoculo, de la internación en el sitio de entrada y de la proximidad de la lesión al SNC. Cuando la vía de entrada es transcutanea o epidérmica, el virus se queda un tiempo en el sitio de inoculación. En ese lapso puede ocurrir una primera replicación en las células nerviosas de las placas musculares más cercanas a la herida y con ello aumenta la carga viral, después el agente infeccioso se desplaza de 8-mm por día, avanzando por continuidad al infectar nuevas neuronas del SNP. Por endocitosis o fusión de membranas. El camino que sigue el virus de SNP al SNC se denomina diseminación centrípeta. Una vez en el SNC la infección es irreversible, conduciendo invariablemente a la muerte del individuo. La multiplicación del virus en el encéfalo inicia en el sistema límbico y luego al resto del cerebro, produciendo edema,

CURSO ENARM CMN SIGLO XXI TEL: 36246001

Pharmed Solutions Institute

PÁGINA 119

MANUAL DE TRABAJO DEL CURSO ENARM CMN SIGLO XXI congestión vascular, infiltración directa de linfocitos e hiperemia de las leptomeninges, después de haber completado su invasión al cerebro comienza la diseminación centrifuga, en la cual el virus regresa a los órganos con alta intervención nerviosa, cuando llega a glándulas salivales, se difunde por el nervio trigémino, lo que inicia la eliminación del virus a través de la saliva. En perros la eliminación de virus por saliva es de 3-10 días antes de que se manifiesten los primeros signos clínicos, esto es importante ya que el animal esta asintomático, pero ya es infectivo. CURSO CLÍNICO: Periodo de incubación: Va desde la exposición hasta inicio de signos clínicos, depende de la cantidad de virus inoculados y sitio anatómico (cabeza 30-48 días, mano 40-59 días, pierna 38-72 días). Las primeras manifestaciones se presentan de 1-3 meses después de la agresión. Prodromo y primeros síntomas: Dura de 1-20 días, síntomas neurológicos como ansiedad y agitación que son precedidos por malestar general, parestesias en el sitio de la lesión, fiebre, anorexia, vomito, dolor de cabeza, letargo y fiebre. Periodo neurológico agudo: Signos de daño a SNC, se puede presentar como rabia paralítica o furiosa. La paralítica es más común en ganado bovino y roedores, la furiosa en mamíferos carnívoros que son reservorios de la enfermedad. Los signos y síntomas para ambas son: fiebre, rigidez de la nuca, parestesias, fascículacion muscular, convulsiones generalizadas y focalizadas, hiperventilación e hipersalivacion, la presencia de hidrofobia que es patognomónico de rabia, dura de 1-5 min y se espacia por periodos de lucidez, es producto de un reflejo exagerado para proteger el tracto respiratorio, más que como espasmo de faringe y laringe. En esta fase evoluciona hasta la afección del SNC, que incluye hiperactividad, desorientación, delirio, alucinaciones, convulsiones, rigidez de nuca y progresan hasta que se presenta parálisis irregular de músculos respiratorios lo que conduce a estado de coma. Durante el periodo de transición entre la fase neurológica y el coma se ostentan lapsos de lucidez y locura, respiración rápida e irregular, ya posteriormente parálisis generalizada particularmente en la zonas inervadas por los nervios craneales y en el músculo cardiaco, se paralizan los músculos respiratorios y sobreviene la muerte. DIAGNOSTICO: Se pueden hacer muestras postmortem (biopsia 10-20g de cerebro, 1-3 g de hipocampo o 5-10g de medula espinal) en animales y seres humanos y antemortem (biopsia de cuero cabelludo, impronta de cornea, saliva suero y liquido cefalorraquiedeo), en seres humanos, tomar muestras de 7-14 días de iniciados los signos neurológicos. La técnica de rutina para diagnostico de rabia es inmunofluorescencia directa, otras técnicas auxiliares son determinación en LCR, ELISA. TRATAMIENTO: Observar al perro o gato 10 días posteriores a la agresión. La atención inmediata de la lesión es; lavar con agua y jabón de forma abundante, desinfectar con agua oxigenada o tintura de yodo, si requiere sutura aplicar primero inmunoglobulina antirrábica humana y se aproximan los bordes, valorar aplicación de antibióticos y toxoide tetánico en heridas contaminadas o punzantes, secar con gasa y cubrir. Si animal agresor no se localiza iniciar tratamiento antirrábico: A) Exposición leve; una dosis de vacuna en los días 0, 3, 7, 14, 28, en la región deltoidea por vía IM. B) Exposición grave; aplicar inmunoglobulina antirrábica humana lo más cerca al día de la agresión (20 UI/kg), la mitad alrededor de la herida y el resto IM. También debemos aplicar la vacuna antirrábica humana, si el animal sigue sano durante 5 dias posteriores se suspende la vacuna, de lo cobtrario prodeguir. PREVENCIÓN: El control en las poblaciones y la variación parenteral y oral de reservorios silvestres y domésticos es ahora el mejor tipo de prevención. CASO CLINICO Niña de 12 años ingresa al Hospital General O´Horán de la Ciudad de Mérida, referida de la localidad de Tekax, Yucatán, por una historia de dolor en miembro torácico derecho por aparente traumatismo que no había cedido al tratamiento médico, sin más antecedentes de importancia. La afección continuó con limitación funcional de la muñeca y disminución de la fuerza de dicha extremidad superior, la cual fue progresiva, hasta acompañarse de dolor torácico con limitación de la respiración, fiebre, odinofagia, presentando aerofobia e hidrofobia y cambios en la conducta que se hicieron mas evidentes, por lo que se sospechó de una encefalitis por virus rábico. La niña continúo con deterioro general que hizo necesaria la asistencia ventilatoria, con evolución tórpida y datos clínicos de muerte cerebral declarada 12 dias después. PREGUNTA Cual es la conducta a seguir? RESPUESTA a.- Profilaxia con inmunoglobulina a contactos. b.- Vacunacion antirrábica canina en cerco sanitario. c.- Cerco epidemologico con vacunación antirrábica a contactos. d- Busqueda de perros sospechos para rabia. PREGUNTA Cual de los siguiente vectores no puede transmitir el virus? RESPUESTA a.- Canes. b.- Felinos. c.- Murcielagos. d.- Roedores. CASO CLINICO Se trata de masculino de 25 años de edad dedicado al campo, refiere que al estar trabajando fue mordido por perro desconocido, el cual no pudo capturar, se conoce de casos de rabia en la zona en animales no domesticados, acude a consulta una hora después del hecho. PREGUNTA Cuál es la conducta más apropiada a seguir en este caso.

CURSO ENARM CMN SIGLO XXI TEL: 36246001

Pharmed Solutions Institute

PÁGINA 120

MANUAL DE TRABAJO DEL CURSO ENARM CMN SIGLO XXI RESPUESTA a.- Lavar la herida y vigilancia. b.- Lavar la herida y administración de toxoide. c.- Lavar la herida, administración de toxoide y aplicar globulina inmunitaria humana. d.- Lavar la herida, administrar toxoide, aplicar globulina inmunitaria humana y vacuna de células diploides humanas. ENFERMEDAD DEL LEGIONARIO (LEGIONELLA PNEUMOPHILA): CIENCIAS BÁSICAS: Llamada así porque afecto a los asistentes de una reunión de la Legión Americana, celebrada en un hotel de Filadelfia (1976). Legionella bacteria gran negativa, de forma bacilar crece en aguas (entre 20- 50) naturales, lagos, ríos, arroyos, lodos. Supervivencia en aire es corta SALUD PÚBLICA: Legionella es causa frecuente de neumonía comunitaria y nosocomial, con una mortalidad que oscila entre el 3-50%. CLASIFICACIÓN: Fiebre de pontiac; El 95%, la forma más leve y la más frecuente se parece a una "gripe", no produce neumonía es autolimitada, con buena evolución. Enfermedad del Legionario; De 1-5%, es la forma más grave, provoca neumonía y afecta al estado general. La mayoría de casos se dan en forma esporádica. PATOGENIA: Factores de riesgo: uso previo de corticoides, e inmunosupresores, neoplasias, trasplante, diálisis, sexo masculino, mayores de 50 años, tabaquismo, EPOC, vía de contagio parece ser a través de aerosoles de gotas de agua contaminada que son aspirados o inhalados. Estas gotas preceden de reservorios de agua infectados por legionella (aparatos de refrigeración, condensados de agua, duchas, sistemas de canalización y distribución de agua que sirve de reservorio en hospitales, edificios). Sospechar en un contexto epidemiológico adecuados datos clínicos, un habito tabáquico importante, la ausencia de supera a antibióticos betalactamicos y la observación de escasos microorganismos en un esputo llenos de neutrofilos son característicos. Solicitar antígeno de legionella en orina. DIAGNOSTICO: Clínico: Inicio brusco con fiebre alta de predominio matutino, con afección general, tos no productiva, esputos, escalofríos, cefalea, presencia de manifestaciones extrapulmonares (diarreas o confusión mental), hiponatremía y elevación de la creatincinasa. La neumonía por legionella se incluye dentro de las neumonías de presentación graves. TRATAMIENTO: De elección clásico es la eritromicina (efectos secundarios; flebitis, alteraciones gastrointestinales, ototoxicidad), por ello se puede utilizar claritromicina (500mg c/12hrs 10-14 días). También útil azitromicina (500mg/día). La experiencia con fluoroquinolonas como levofloxacino mas en caos graves o neumonía intrahospitalaria (500mg/día en los primeros días c/12 hrs e intravenoso) y ofloxacino son positivas. CASO CLINICO Varón de 46 años, fumador de 40 paquetes al año y cirugía de colesteatoma con parálisis facial periférica derecha residual. Una semana antes de la clínica refiere estancia en un hotel del Levante. Acude al médico de guardia del centro de salud por fiebre de más de 39º C, sin otra sintomatología acompañante y con exploración física normal. Se le pauta tratamiento sintomático con paracetamol. Al día siguiente acude a su médico de cabecera por persistencia de la fiebre, no existiendo variaciones en la exploración. Se decide continuar con el tratamiento prescrito el día anterior y observación domiciliaria. A los cinco días tras persistir la fiebre, y aparecer disnea y tos con expectoración hemoptoica, el paciente es remitido al Servicio de Urgencias hospitalario, donde se le practican diversas pruebas complementarias: hemograma (16.430 leucocitos con 93% neutrófilos), bioquímica (Na 134, PCR 340), GAB (pH 7.46, PO2 60, PCO2 30), Rx de tórax (infiltrado alveolar bilateral con derrame pleural derecho). PREGUNTA Considerando el agente infeccioso presuntivo, cual es la conducta diagnostica mas apropiada? RESPUESTA a.- Realizar Hemocultivo. b.- Baciloscopia de Esputo. c.- Antigeno en Orina. d.- Inmunoglobulinas. CASO CLINICO Se trata de paciente masculino de 57 años de edad el cual acude al servicio de urgencias debido a tos intensa no productiva y fiebre, el paciente cuenta con los siguientes antecedentes, originario de guanajuato, empleado de la construcción, alcoholismo y tabaquismo positivo desde hace 40 años, actualmente bajo tratamiento por EPOC y cirrosis hepática, se realizan estudio de laboratorio y gabinete de rutina donde se observa un reporte con disminución de la respuesta inmunológica esperada, la radiografia de torax presenta un patrón característico, sin embargo el cuadro respiratorio bajo se ha intensificado por lo que se traslada a terapia intensiva. PREGUNTA Cual es tratamiento antibiótico más apropiado para este caso. RESPUESTA a.- Azitromicina. b.- Levofloxacina. c.- Trimetoprim/sulfametoxazol. d.- Penicilina. RICKETTSIASIS (R. RICKETTSIA; R. CORONII): CIENCIAS BÁSICAS: Son bacterias estrictamente intracelulares gramm negativas, que se transmiten a través de vectores artrópodos (garrapatas, pulgas y piojos) a diferentes mamíferos que actúan como reservorios. SALUD PÚBLICA: Sus características epidemiológicas, su alta infectividad y elevada mortalidad de algunas, pueden hacer que se conviertan en armas biológicas ideales. CLASIFICACIÓN: Existen 2 tipos de vasculitis que en función de ellas podemos diferenciar en 2 grupos: grupo de las fiebres manchadas; afectan

CURSO ENARM CMN SIGLO XXI TEL: 36246001

Pharmed Solutions Institute

PÁGINA 121

MANUAL DE TRABAJO DEL CURSO ENARM CMN SIGLO XXI endotelio, capa intima y media, entre ellas Fiebre manchada de Montañas Rocosas (en América, transmitida por garrapatas causada por R. rickettsia), Fiebre Botónosa mediterránea (causada por R. coronii, transmitida por garrapata de perro). El grupo de las fiebres tíficas; afectan endotelio y capa intima entre ellas Tifus o exantema epidémico (causado por R. prowazekii, transmitido por piojos)y tifus murino (causado por R. typhi, transmitido por pulgas). PATOGENIA: La rickettsia penetra en el organismo, tras la picadura, o atraves de una solución de continuidad contaminada con heces de los artrópodos (ingresan a través del piquete a la hora des rascado), o bien a través de la conjuntiva o vías respiratorias, rápidamente alcanza torrente sanguíneo, allí se une a la célula endotelial y se producen cambios en la conformación de la membrana que facilitan la fagocitosis de la bacteria, esta rompe la membrana fagosomica y escapa libre a citosol aquí se produce la multiplicación (intracelular)a través de un mecanismo de expulsión activo, se produce inoculación en células contiguas al área infectada, también hay diseminación linfática. Y este ciclo, ocasiona la aparición de múltiples focos de vasculitis y un estado procoagulante endovascular erupción, hemorragias, trombosis y a veces gangrena). La destrucción celular y liberación de rickettsias explican la fiebre, las lesiones capilares son la base del colapso vascular y alteración del estado de conciencia. El aumento de permeabilidad capilar, produce extravasacion de líquidos, y proteínas, dando edema, y agravamiento del estado de choque. DIAGNOSTICO: Clínico; fase de pródromo; malestar general, cefalea y febrícula. Fase de estado; fiebre elevada sostenida de 2-3 semanas, cefalea intensa, vértigos ataque al estado general con gran postración, piel seca, estado de conciencia alterado con alternancia de estupor y delirio, tos seca, nausea, vomito. Fase de exantema; máculas eritematosas en axilas y flancos que se extienden al tronco y finalmente a extremidades. En la eruptiva, es común que exista oliguría, taquicardia desproporcionada a la temperatura, polipnea. Complicaciones; gangrena por obstrucción. Serologico: IgM e IgG aparecen después de 1 semana a 10 días, y duran de 3-4 meses. Existencia de antígenos con prueba de Weil-Félix. En México prueba de Ruiz-Castañeda. TRATAMIENTO: primera elección tetera inclinas, clorhidrato de tetraciclina 40mg/kg dividida c/6 o 8 o 12 hrs VO o parenteral, doxiciclina 2-4mg/kg/día (dosis máxima 200mg/ dia en dos dosis), mantenerse 3 días después de remisión térmica. Alternativa cloranfenicol, el tratamiento debe continuar hasta 24 hrs después de haber desaparecido la fiebre. El estado de choque, hiponatremía, edema y coagulopatia de consumo; infusiones de albúmina, heparina (la sangre total y plasma contraindicados por CID). Los medicamentos solo detienen el desarrollo, de las rickettsias sin destruirlas, no son raras las recaídas. CASO CLINICO Paciente masculino de 8 años de edad, que inició 5 días previos con presencia de fiebre intensa de hasta 40ºC de difícil control, dolor faríngeo y tos productiva, astenia y adinamia, llevado con médico 4 días previos quien recomienda trimetoprim/sulfametoxazol a dosis de 7mg/kg/día (8 dosis), ambroxol y clorfenamina con paracetamol para tratamiento decuadro respiratorio. El paciente no muestra mejoría clínicamente, e incluso aparecen múltiples zonas de erupción en lapiel con eritema y petequias, pruriginosas, principalmente encara y que se han ido extendiendo a tronco y extremidades.Con patrón tipo centrípeto. Al no encontrar mejoría es traído a revisión médica a la consulta externa de con diagnóstico probable de enfermedad de Kawasaki. El cuadro clínico inició posterior a una visita de establo y en contacto con ganado bovino y ovino, se desconoce sí tenían vectores ya que el paciente luce descuidado pues no vive con los padres. Tiene hábitos de higiene regulares. Baño cada tercer día pero en vacaciones hasta cada semana. No muestra cartilla, desconoce cuáles vacunas le faltan. Tuvo varicela a los 4 años. El paciente se observó hipotrófico, consciente y cooperador, con buena coloración de piel y lapresencia de una erupción maculo papular eritematosa, descamativa, petequial o micromacular en parpados, así como otras regiones de cara, pecho, dorso, abdomen y extremidades, respetando palmas y plantas de pies. Kerning y Brudsinskinegativos y sin otros datos de meningismo; cuello simétricocon un ganglio palpable en región media, móvil no doloroso menor a 1cm. Hemorragia en flama conjuntival bilateral, sin secreción conjuntival ni prurito, mucosa nasal conrinorrea hialina, mucosa oral bien hidratada y faringe hiperémica, sin exudados. Tórax sin dificultad respiratoria, con estertores gruesos bilaterales transmitidos, sin problemas a la ventilación. Precordio rítmico sin soplos y pulsos periféricos normales. Abdomen blando, levemente distendido, con peristalsis normal, sin hepatomegalia ni esplenomegalia. PREGUNTA Cual es el agente etiológico mas probable? RESPUESTA a.- Rickettsia prowazekii b.- Rickettsia typhi c.- Rickettsia rickettsii d.- Rickettsia coronii CASO CLINICO Se trata de masculino de 19 años de edad ingresado al servicio de urgencias presentando fiebre, cefalea intensa, dolos abdominal difuso pero ligero, pero con mialgias intensas, al examen físico se observa con 38,5 c, FC 112, TA 120/70 su estado general es adecuado en crecimiento y desarrollo con diaforesis, se mantiene alerta y orientado, se realizan estudios de laboratorio y gabinete de rutina donde se observa únicamente plaquetas con 84,000, se realiza puncion lumbar donde se observa auscencia de eritrocitos, proteínas y glucosa normales, solo se observa 5 monocitos. Como antecedente de importancia refiere regresar de un campamento y realizar escalado de montañas, actualmente continua en el servicio de medicina interna. PREGUNTA Considerando los hallazgos asi como los antecedentes usted integra un diagnostico, y debe iniciar tratamiento, cual es el más adecuado: RESPUESTA a.- Rifampicina. b.- Doxiciclina.

CURSO ENARM CMN SIGLO XXI TEL: 36246001

Pharmed Solutions Institute

PÁGINA 122

MANUAL DE TRABAJO DEL CURSO ENARM CMN SIGLO XXI c.- Vancomicina. d.- Ceftriaxona. LISTERIOSIS (LISTERIA MONOCYTOGENES): CIENCIAS BÁSICAS: Bacteria Gramm positiva anaerobia, facultativa intracelular, puede causar infecciones invasoras muy graves. Principales reservorios son el suelo, forrajes el agua, los silos y tracto gastrointestinal en aves, peces y mamíferos incluyendo el hombre. Suele considerarsele un patógeno oportunista. SALUD PUBLICA: Distribución universal, relativamente resistente a la refrigeración, la sequedad y el calor extremo. Afecta a inmunodeprimidos, pacientes con enfermedades crónicas debilitantes mayores de 60 años, cirróticos, embarazadas en (en la Ciudad de México 1 por 4000) y neonatos (1 por 1500). Continúa causando una elevada morbilidad y una preocupante mortalidad. PATOGENIA: la infección se adquiere mediante el consumo de Alimentos contaminados, carnes, pescado y vegetales, crudos y lácteos no pasteurizados y productos refrigerados, los recién nacidos la adquieren a través de la placenta o del canal de parto infectado. Puede sobrevivir en el entorno gástrico, colonizar el intestino y cruzar la barrera hematoencefalica y maternofetal, con sus proteínas de superficie internalinas, se favorece su invasión celular por un mecanismo tipo zipper, en que la bacteria se hunde progresivamente en la superficie celular, después estas se lizan y escapan, la proteína polímerizadora de activa, le permite azucares en el citosol celular durante su replicación intracelular. DIAGNOSTICO: Clínica; La más habitual es la listeriosis gastrointestinal; importante puede causar brotes de toxiinfeccion alimentaria, puede ser asintomática o sufren deposiciones acuosas, nauseas, vómitos, cefalea, artromialgias y fiebre, síntomas que se limitan a 2 días, salvo que estén inmunocomprometidos. Listeriosis invasora; enfermedad gestacional y neonatal, frecuentemente fiebre sin foco aparente, la gravedad de Listeriosis materna radica en el tercer trimestre, ya que puede haber aborto, muerte fetal intrautero, prematuro dad, sepsis, muerte neonatal. Es fundamental hacer hemocultivos a toda mujer embarazada con fiebre, aunque semiología parezca banal. Los neonatos pueden sufrir sepsis, microabscesos y granulomas diseminados (granulomatosis infantoseptica), infección respiratoria o meningitis, que conlleva una mortalidad y morbilidad importantes como la hidrocefalia persistente en el caso de meningitis. Infección del SNC; pueden tener meningitis, meningoencefalitis o cerebritis. Endocarditis es una rara y grave complicación de la bacteriemia (forma clínica mas frecuente en inmunodeprimidos, sin foco identificable, ocurre con fiebre, deteriroro rapido y a menudo fulminante) por listeria. El diagnostico definitivo es aislamiento en sangre, LCR, liquido articular, placenta. TRATAMIENTO: La mayoría de los antibióticos incluyendo penicilinas son bacteriostáticos contra la listeria, los aminoglucosidos, glucopeptidos y cotrimoxaxol son bactericidas. Neonatos y formas graves; cotrimoxaxol 20mg/kg/día, en 4 dosis o gentamicina 5-7mg/kg/día en dosis única por 3 días. Infección del SNC ampicilina 400mg/kg/ día en 6 dosis, asociada a gentamicina (si no hay daño renal) o ampicilina y cotrimoxaxol (mas eficacia, menos nefrotoxicidad). En embarazadas ampicilina mejor opción. CASO CLINICO Femenino de 33 años con fiebre de una semana de evoluacion, de tipo intermitente, mialgias, artralgia y escalofríos, acude a consulta a primer nivel donde no se integra proceso infeccioso, la paciente se encuentra embarazada con 21 SDG por FUM, a la exploración se observa con mucosas orales deshidratadas, durante su ingreso presenta nausea, cefalea y diarrea, al tercer dia se observa con somnolencia con inversión del ciclo vigilia sueño, posteriormente refiere sensación de salida de liquido transvaginal, al tacto no se observa elementos ovulo placentarios. PREGUNTA Cual es la conducta terapéutica mas adecuada sobre el agente casusal del caso? RESPUESTA a.- Ampicilina. b.- Gentamicina. c.- Cotrimoxaxol. d.- Ceftriaxona. PSEUDOMONA AERUGINOSA (PAE): CIENCIAS BÁSICAS: Es una bacteria bacilo Gramm negativo, aerobio estricto, dispersa en el ambiente, emergente relevancia como patógeno oportunista causante de infecciones en pacientes hospitalizados (equipos de ventilación mecánica, soluciones de limpieza, instrumental, medicamentos), críticos y/o inmunodeprimidos (quemados, ventilados, fibrosis quística, EPOC, pos quirúrgicos).m La PAE causa infecciones en la mayoría de las partes del cuerpo, se adapta rápidamente al tracto respiratorio y es el lugar más frecuente de infección por PAE (principal causante de la neumonía asociada a la ventilación mecánica NAR). SALUD PUBLICA: Diseminada prácticamente en toda la geografía mundial, es cosmopolita. Tasa de mortalidad atribuible a pseudomoma es de 34%. PATOGENIA: Factores de riesgo; inmunodeprimidos. La fuente de origen puede ser hematogena o como puerta de entrada en un sitio de punción o a través de vías centrales o catéteres de diálisis que pueda tener el paciente al parecer la lesión inicial provocada por la P. aeruginosa al epitelio respiratorio y otras mucosas esta mediada por pili o fimbrias y por un exapolisacorido mucoide conocido como alginato. Existen receptores de estas adhesinas en las células epiteliales. El microorganismo produce diversas enzimas extra celulares como la proteasa alcalina, elastasa, fosfolipasa, citotoxina y exoenzimas A y S, la alteración de los tejidos del huésped por estos productos bacterianos, crea las condiciones necesarias para la proliferación e invasión bacteriana y la consiguiente destrucción del tejido. Infecciones óseas (fracturas, cirugías traumatológicas) y articulares (osteomielitis vertebral), por bacteriemia, inoculación directa dentro del hueso y por diseminación contigua desde otro sitio de infección. Infección en SNC (meningitis, infecciones subdurales o extradurales) secundarias a cirugía y a trauma de cráneo o bacteriemias. Infección urinaria secundaria a cuerpo extraño (calculo, atenta, sonda) o a estenosis. Una situación importante es la infección en tejidos quemados. DIAGNOSTICO: El cuadro clínico es el de un paciente séptico, el único punto diferente con otros tipos de sepsis por gramm negativos es la presencia de lesiones cutáneas llamadas ectima gangrenoso, que se ve mas frecuente en pacientes neutropenicos. Lesión maculopapular, pequeña, roja, dolorosa, mal circunscrita, que comienza de color rosa, se oscurece hasta volverse púrpura y finalmente negra y necrótica. Debe sospecharse neumonía nosocomial en un paciente con

CURSO ENARM CMN SIGLO XXI TEL: 36246001

Pharmed Solutions Institute

PÁGINA 123

MANUAL DE TRABAJO DEL CURSO ENARM CMN SIGLO XXI uno infiltrado radiológico nuevo o progresivo asociado a datos clínicos sugestivos de infección; fiebre, esputo purulento o leucocitosis, tras diagnostico sindromatico, intentantar confirmación microbiología. . La característica más importante en laboratorio es la producción de pigmento llamado piocianina, que le da un color azulado. Obtención de material respiratorio (BAL, aspirado traqueal, minibar) TRATAMIENTO: Las pseudomonas son productoras de enzimas MBL (metalo-b-lactamasas) capaces de resistir prácticamente a todo tipo de antibiótico, por ello su actividad es devastadora. Basándose en esquema para neumonías nosocomiales A) inicio reciente y sin factores de riesgo, cualquier gravedad: ceftriaxona, fluroquinolonas, ampicilina- sulbactam o ertapenem B) inicio tardío o factores de riesgo, cualquier gravedad: beta-lactamico antipseudomonico (carbenicilina, ticarcilina, mezlocilina, piperacilina) mas fluoroquinilona o aminoglucosidos mas linezonid o vancomicina. Antibióticos mas utilizados céfepime (1-2 g cada 8 hrs IV), ceftazidime ( 2g c/8hrs IV), imipenem (500mgs c/6 o 1g c/8hrs IV), meropenem (1g c/8hrs), gentamicina (7mg/kg/día IV), amikacina (20 mg/kg/día IV), levofloxacino (750mg/día IV), colistin 100mg c/8hrs, piperacilina/tazobactam (4.5g c/6hrs IV). No hay un esquema especifico lo que realmente cambia la mortalidad es el tratamiento empírico inicial adecuado. CASO CLINICO Un varón de 27 años, diabético, fue internado por fiebre, compromiso de conciencia y rigidez de nuca aparecidos tras tres días de cursar con una infección respiratoria aguda. Se le diagnosticó una meningoencefalitis aséptica (LCR de aspecto claro, con presión de 12 cm H2O; leucocitos: 200/mm3; tinción de Gram directa y cultivo bacteriano: negativos) y recibió ceftriaxona más vancomicina y corticoterapia (dexametasona, 10 mg iv cada 6-horas durante 4 días). Inicialmente ingresó en una UCI y estuvo en ventilación mecánica, siendo desconectado a los 4 días. Desarrolló fiebre y tos productiva tras una semana de evolución. Una Rx de tórax detectó un infiltrado pulmonar bilateral. Tres días más tarde, una segunda Rx reveló la presencia de múltiples abscesos pulmonares con niveles hidro-aéreos. PREGUNTA Cual es la conducta diagnostica mas adecuada? RESPUESTA a.- TAC de torax. b.- Toracostomia. c.- Cultivo de esputo. d.- Hemocultivo. PREGUNTA Se obtiene cultivo positivo para P. aeruginosa, cual es la conducta terapéutica asociada mas apropia? RESPUESTA a.- Linezolid. b.- Meropenem. c.- Amikacina. d.- Ciprofloxacina. LEPTOSPIROSIS (LEPTOSPIRA INTERROGANS): CIENCIAS BÁSICAS: Se considera la zoonosis más frecuente, la leptospira interrogans, es la única espiroqueta que infecta al hombre, gramm negativo, aerobio y móvil. Frecuente en campesinos, granjeros, taladores, cazadores, trabajadores de cultivo de arroz y militares, actualmente también en niños (natación cabotaje, ciclismo, caza deportiva, animales domésticos infectados). PATOGENIA: Las leptospiras en agua o tierra contaminada por orina de animales infectados, penetran a la piel a través de cortaduras o abrasiones, membranas, mucosas y conjuntivas, se disemina a través de la sangre a todos los órganos y sistemas incluyendo el SNC. Los fenómenos hemorrágicos son secundarios a vasculitis severa con daño endotelial. Los riñones presentan nefritis intersticiales, necrosis tubular y permeabilidad capilar anormal que acompaña a hipovolemia y puede llegar a IRC. La ictericia es secundaria al necrosis centrolobulillar y proliferación de células de Kupffer, con disfunción hepatocelular. Miocarditis y uveítis ocurren por invasión tisular. DIAGNOSTICO: Clínica; leptospirosis anicterica (85-90%), incubación de 7-12 días, inicia fiebre, cefalea, mialgias calosfríos severos y mal estado general, linfadenopatias hepatoesplenomegalia. Durante este periodo pueden aislaras las leptospiras en sangre o LCR. Entran en perdidos asintomático de 1-3 días e inician ahora la llamada fase inmune o desarrollo de anticuerpos IgM específicos, puede reaparecer la fiebre, cefalea y vómitos y se puede desarrollar meningitis aséptica, existe aumento de células en el liquido cefalorraquideo, primero predominando los PMN y después los mononucleraes, las proteínas están elevadas, glucosa normal. Puede o no haber irritación meninges, esta fase dura de 3-40 días, y ya no se pueden aislar las leptospiras de sangre y/LCR, pero si en orina. Leptospirosis ictericia (sx. de Weill); forma grave y de mayor mortalidad, los pacientes pueden desarrollar ictericia pero no daño renal, lo usual es que haya ictericia y elevación de urea y creatinina, así como elevación de transaminasas, proteinuria, hematuria, cilindruria, puede haber anemia, trombocitopenia, leucocitosis, hemorragia, Asi como neumonías hemorrágica. Laboratorio observación de leptospiras es el estándar de oro para el diagnostico, la búsqueda en orina es la más confiable. Las pruebas en suero para detección de anticuerpos de la fase aguda (1-2 semanas) y su elevación en 4 tantos durante la fase convaleciente (2semanas después), incluyen la prueba de aglutinación al microscopio, confirman diagnostico serológico. Una sola determinación de títulos de 1:800 o mayores sugieren fuertemente la enfermedad. Analizar LCR en pacientes con meningitis. La radiografía de tórax puede mostrar cardiomegalia, edema agudo pulmonar asociado a miocarditis o infiltrados en parches asociados a hemorragia alveolar por capilaritis. TRATAMIENTO: La penicilina G sódica (20-24 millones U/día c/4-6hrs), doxiciclina (100mg c/12hrs IV) o eritromicina (500mg c/6hrs IV)¡ Son eficaces para detener la infección y el daño orgánico. La doxiciclina se usa para prevención en dosis semanal de 200mg.

CURSO ENARM CMN SIGLO XXI TEL: 36246001

Pharmed Solutions Institute

PÁGINA 124

MANUAL DE TRABAJO DEL CURSO ENARM CMN SIGLO XXI CASO CLINICO Femenino de 29 años la cual labora como asistente de veterinario, refiere que hace 10 dias inicio con fiebre, escalofríos, cefalea, nauseas, dolor muscular, se automedica con fármacos sintomáticos, mejorando su estado general sin embargo regresan los síntomas y agregandoce ictericia, a la exploración clínica se presenta 38.9 C temperatura, FC 1010, TA 140/90 mmHg, la saturación por oximetro 92 %, a la percusión se presenta dolor hepático, se persibe leve crecimiento, sin esplenomegalia, los estudios de laboratorio BUN de 64, creatinina 3.6, bilirrubina de 64, AST 86, ALT 103, fosfatasa alcalina de 390, leucocitos 11 000, 13% de bandas y 80 % polimorfonucleares, hematocrito 33% y plaquetas de 145, puncion lumbar revela pleocitosis, TAC de torax muestra inflitrados en flama difusos compatibles con hemorragia pulmonar. PREGUNTA Cual es diagnostico más probable? RESPUESTA a.- Neumonitis intersticial aguda. b.- Leucemia mieloide aguda. c.- Fiebre por Streptobacillus moniliformis. d.- Infeccion por Leptospira interrogans. ACTINOMICOSIS (ACTINOMYCES ISRAELII): CIENCIAS BASICAS: Es causada por bacterias anaeróbicas o microaerófilos, gramm positivos principalmente del género Actinomyces (por ejemplo, A. israelii), que está caracterizada por formación de absceso, fibrosis tisular, las regiones más afectadas son la cervicofacial, torácica y pélvico-abdominal. El germen tiene una baja virulencia y produce enfermedad solo cuando la barrera mucosa normal se altera, produciendo formación de abscesos múltiples, fistulización o una masa, requiriendo la presencia de otras bacterias, que destruyan la vascularidad tisular convirtiendo el anaeróbica (generalmente infecciones polimicrobianas). Actinomicosis se asocia con mala higiene dental, absceso dentario, uso de dispositivos anticonceptivos intrauterinos (DIU), y la inmunosupresión. SALUD PÚBLICA: Su incidencia está disminuyendo, probablemente como resultado de una mejor higiene dental y un inicio más temprano del tratamiento antibiótico. PATOGENIA: Los agentes de actinomicosis son miembros de la flora oral normal. La enfermedad se presenta sólo después de la interrupción de la barrera de la mucosa. Infección local se extiende en forma contigua en una manera lenta y progresiva, haciendo caso omiso de los planos del tejido. En crecimiento in vivo produce grumos llamados granos o gránulos de azufre. Las paredes fibróticas de la masa a menudo se describen como "madera”. DIAGNOSTICO: La enfermedad oral - cervicofacial: La infección comienza como una inflamación de los tejidos blandos, abscesos, o masa, a menudo en el ángulo de la mandíbula con extensión contigua al cerebro, espina dorsal cervical, o en el tórax. El dolor, la fiebre, y leucocitosis son variables. Enfermedad torácica: El parénquima pulmonar y / o en el espacio pleural suele participar. Produce dolor en el pecho, fiebre y pérdida de peso. Radiografía de tórax muestra una lesión de masa o la neumonía. Enfermedad cavitaria o adenopatía hiliar pueden ocurrir, y > 50 % de los puntos tienen engrosamiento pleural, derrame o empiema. Las lesiones cruzan fisuras o pleura y pueden implicar el mediastino, el hueso contiguo, o la pared torácica. Enfermedad abdominal: El diagnóstico es difícil y no se puede hacer hasta meses después del evento inicial (por ejemplo, diverticulitis, cirugía intestinal). La enfermedad generalmente se presenta como un absceso, masa o lesión fijado al tejido subyacente y, a menudo se confunde con el cáncer. Fístulas en la pared abdominal, en la región perianal, u otros órganos pueden desarrollar e imitar la enfermedad inflamatoria intestinal. Participación del tracto urogenital puede presentarse como pielonefritis o absceso perirrenal. Actinomicosis pélvica se asocia a menudo con DIU. La presentación es indolente y puede seguir a la extracción del dispositivo se presenta fiebre, pérdida de peso, dolor abdominal y sangrado vaginal anormal. Endometritis progresa a masas pélvicas o abscesos tuboovárico. Actinomicosis pueden afectar el tejido músculo-esquelético, tejido blando, o el SNC y puede difundir vía hematógena, más comúnmente a los pulmones y el hígado. Aspiraciones, biopsias, o la escisión quirúrgica puede ser necesaria para obtener el material para el diagnóstico. La identificación microscópica de los gránulos de azufre en pus o tejidos establece el diagnóstico. Necrosis central de las lesiones con los neutrófilos y los gránulos de azufre es prácticamente de diagnóstico de la enfermedad. Por lo general requieren de 5-7 días, pero puede tardar 2-4 semanas para convertirse en positiva, incluso una sola dosis de antibiótico puede afectar el rendimiento de los cultivos. TRATAMIENTO: Requiere un tratamiento prolongado. Tratamiento IV durante 2-6 semanas (por lo general con la penicilinas- amoxicilina, penicilina G, ampicilina) seguido de tratamiento oral durante 6-12 meses (por ejemplo, con penicilina o ampicilina) se sugiere para la infección grave y enfermedad voluminosa. En particular la enfermedad de la región oral y cervicofacial, se puede curar con un tratamiento más corto. En alérgicos eritromicina o azitromicina. Si el tratamiento se prolonga más allá del punto de la resolución de la enfermedad medible (cuantificada mediante TAC o RM), se minimiza la recaída. Agentes alternativos adecuados incluyen las tetraciclinas (por ejemplo, minociclina, 200 mg/día administrado por vía IV o PO cada 12 horas) o clindamicina (2,7 g /d administrado por vía IV cada 8 horas). CASO CLINICO Mujer de 53 años, con antecedentes de amigdalectomía en la infancia y sin manipulaciones dentales recientes. Consultó por sensación de cuerpo extraño faríngeo derecho de 1 año de evolución, con expulsión por boca de material blanquecino al menos en tres ocasiones. El otorrinolaringólogo extrajo un cúmulo de detritus del lecho amigdalar derecho, y apareció en la zona adyacente una pared de aspecto inflamatorio. El análisis anatomopatológico de la muestra informó de masa (1,5×1,5×1cm). Durante los 6 meses posteriores de seguimiento presentó tres episodios de expulsión con la tos, de masas de aproximadamente 1cm de iguales características. A la exploración física destacaba la visualización repetida de un cúmulo de detritus blanco en una cavidad en el polo superior del lecho amigdalar derecho, cavidad atribuible a una cicatrización anómala de la amigdalectomía previa. La higiene bucal era buena. La analítica básica y la radiografía de tórax eran normales. Serología para el virus de la inmunodeficiencia humana, negativa. La tomografía computarizada cervical identificó una prominencia inespecífica a nivel amigdalar derecho, sin objetivar trayectos en partes blandas ni adenopatías

CURSO ENARM CMN SIGLO XXI TEL: 36246001

Pharmed Solutions Institute

PÁGINA 125

MANUAL DE TRABAJO DEL CURSO ENARM CMN SIGLO XXI PREGUNTA Se realiza cultivo y es positivo para actinomyces israelii, cual es la conducta mas adecuada para minimizar recaidas? RESPUESTA a.- Amoxicilina. b.- Clindamicina. c.- Penicilina. d.- Tetraciclina. PREGUNTA Cual de los siguientes factores de riesgo es mas frecuente para la presencia actinomicosis craneofacial? a.- Otitis media. b.- Sinusitis cronica. c.- Absceso dentario. d.- Amigdalas hipertróficas. PREGUNTA En el caso de la actinomicosis abdominopelviana, cual de los siguientes factores es el mas frecuente? RESPUESTA a.- Cuerpo extraño. b.- Aborto séptico. c.- EPI. d.- LUI. NOCARDIASIS (N. ASTERIODES, N. BRASILIENSIS): CIENCIAS BASICAS: Nocardias son actinomicetos aeróbicos saprófitos comunes en el suelo. Varias especies están asociadas con la enfermedad humana. N. asteroides es la especie más comúnmente asociada con la enfermedad invasiva. N. brasiliensis se asocia más con lesiones cutáneas localizadas. SALUD PÚBLICA: En los Estados Unidos, 1.100 casos de infección por nocardia se producen cada año, de los cuales 85 % son pulmonares o sistémicos. El riesgo de enfermedad es mayor de lo habitual entre las personas con deficiencia de inmunidad mediada por células, por ejemplo, que asocia con el linfoma, el trasplante, la terapia con glucocorticoides, o SIDA. Las tasas de mortalidad son altas entre los pacientes con nocardiosis del cerebro. PATOGENIA: La neumonía y la enfermedad diseminada siguen a la inhalación de micelios. Nocardiosis causa abscesos con infiltración de neutrófilos y necrosis. DIAGNOSTICO: Clínico; Enfermedad pulmonar suele ser subaguda, presentando en días o semanas. Enfermedad extrapulmonar se documenta en > 50 % de los casos, y un poco de afectación pulmonar es evidente en el 80 % de los pacientes con enfermedad extrapulmonar. Una tos productiva importante de pequeñas cantidades de esputo espeso purulenta, fiebre, anorexia, pérdida de peso y malestar general son comunes, disnea, hemoptisis y dolor pleurítico son menos comunes. La radiografía de tórax; suele mostrar infiltrados nodulares únicos o múltiples de diferentes tamaños que tienden a cavitación. Empiema se observó en un tercio de los casos. La infección puede diseminarse a los tejidos adyacentes, como el pericardio o el mediastino. Enfermedad extrapulmonar normalmente se manifiesta como abscesos subagudos en el cerebro, la piel, los riñones, los huesos y / o musculares. Algunos abscesos forman fístulas y pequeñas cantidades de descarga de pus, pero no las de los pulmones o el cerebro. Los abscesos cerebrales son generalmente supratentorial, a menudo multiloculada, pueden ser únicos o múltiples y tienden a enterrarse en los ventrículos o extender en el espacio subaracnoideo. Enfermedad después de la inoculación transcutánea Celulitis: La celulitis subaguda puede presentar 1-3 semanas después de una lesión en la piel (a menudo contaminada con tierra). Síndrome linfocutáneo: Una lesión piodérmica se desarrolla en el sitio de inoculación, con ulceración central y secreción purulenta. Esta lesión suele drenada por nódulos SC a lo largo de los vasos linfáticos. Esta forma se asemeja a la esporotricosis. Actinomicetoma: Son formas nodulares de hinchazón en el sitio de traumatismo local, por lo general en los pies o las manos. Forma de fístulas y la descarga de drenaje seroso o purulento que puede contener gránulos que consisten en masas de micelio. Las lesiones, que se extienden poco a poco a lo largo de los planos faciales involucrar a la piel, tejido subcutáneo y hueso, pueden provocar graves deformidades. Infecciones oculares: queratitis por lo general sigue a un trauma ocular. La endoftalmitis puede ocurrir después de la cirugía del ojo o en la enfermedad diseminada. Exámenes en los que se identifica a la bacteria; broncoscopia, cultivo de esputo o pus (en agar enriquecido con extracto de levadura y carbón activado), se pueden ver filamentos gramm positivos, biopsia de piel, cerebro, pulmón (también aspiración pulmonar percutánea, lavado broncoalveolar). TRATAMIENTO: Las sulfonamidas son los fármacos de elección, Trimetropim (10-20mg/kg) y sulfametoxaxol (50-100mg/kg). Para enfermedad grave, los niveles séricos de sulfonamidas deben ser monitoreadas y mantenidas a 100-150 mg /mL. Una vez que la enfermedad está controlada, la dosis de trimetoprim-sulfametoxazol puede ser reducida por 50%. Las pruebas de sensibilidad pueden guiar tratamientos alternativos.En enfermedad del SNC; minociclina (200-400mg). Sindrome de celulitis linfocutanea; linezolid (1200mg). Actinomicetoma amikacina (1015mg/kg), cefotaxima (6g), ceftriaxona (1-2g), imipenem (2g). La terapia para la nocardiosis debe continuar mientras el paciente permanezca inmunodeprimido. Los abscesos cerebrales que son grandes o no responde a los antibióticos deben ser aspiradas. Las recaídas son comunes. El paciente debe ser seguido por al menos 6 meses después de la terapia se haya completado. CASO CLINICO Mujer de 42 años, diestra, portadora de poliglobulia diagnosticada 15 años antes y catalogada como primaria luego de exhaustivos estudios. Tratada con ácido acetil salicílico en las ocasiones que se le realizó flebotomías, tuvo mala tolerancia a las mismas. Un mes previo fue sometida a un procedimiento de extracción e implante dentario, presentando 10 a 15 días después un cuadro de inicio y progresión insidiosos, en el curso de 15 días, y caracterizado por cefalea, vómitos, fiebre, a lo que se agregaron posteriormente

CURSO ENARM CMN SIGLO XXI TEL: 36246001

Pharmed Solutions Institute

PÁGINA 126

MANUAL DE TRABAJO DEL CURSO ENARM CMN SIGLO XXI alteraciones de la agudeza visual y campimétricas y en las últimas cuarenta y ocho horas, depresión de la vigilia. En el examen físico, la paciente se presentaba confusa, por momentos excitada, con guiño a la amenaza abolido a izquierda, paresia de VI par izquierdo, parálisis facial central izquierda, paresia leve de miembros izquierdos, con hiporreflexia y signo de Babinski. En la paraclínica de laboratorio el hematocrito fue de 6,79 %, hemoglobina 20,9 mg/dL, VCM 90,1, HBCM 30,7%, leucocitos 12,800/mm3, PMN 80%, linfocitos 17%, eosinófilos 1%, basófilos 2%, plaquetas 150/mm3, VES 1 mm 1ª hora PREGUNTA Cual es la conducta diagnostica mas adecuada a seguir? RESPUESTA a.- Hemocultivo. b.- IRM cráneo. c.- TAC cráneo. d.- Serologia PREGUNTA La RM con gadolinio evidencia proceso expansivo cortico-subcortical con anillo grueso irregular de realce con centro necrótico. La orientación diagnóstica imagenológica RM orientó a un absceso cerebral, se realizo drenaje y cultivo, que resulto positvo a Norcardia asteroides, cual es la conducta terapéutica mas adecuada? RESPUESTA a.- Linezolid (1200mg). b.- Minociclina (200-400mg). c.- Amikacina (10-15mg/kg) d.- TMP/SMX (3.200/640mg) INFECCIONES ESTREPTOCOCICAS: CIENCIAS BÁSICAS: Son un genero de los más importantes como patógeno para el ser humano, son bacterias esférica, eróticos facultativos, no forman esporas, catalana negativos e inmóviles. El Streptococcus pyogenes, es el agente etiológico mas frecuente de faringoamigdalitis aguda, pudiendo ocasionar secuelas no supurativas, tales como la fiebre reumática y la glomerulonefritis postestreptocócicas. Otro el Streptococcus pneumoniae. De acuerdo con el tipo de hemolisis de eritrocitos que producen, en beta (hemolisis completa), alfa (hemolisis parcial) y gamma (sin hemolisis). La primera la presentan prácticamente todos los S. pyogenes y la segunda los S. pneumoniae y viridans. SALUD PUBLICA: S. pyogenes incidencia mayor en raza blanca, en adolescencia y sexo masculino. La faringoamigdalitis más frecuentes en zonas templadas y frías y en invierno, mientras el impétigo y la piodermitis lo son en los climas tropicales y en meses calurosos. PATOGENIA: La fuente de infección son las secreciones de nariz, garganta y piel de enfermos, la transmisión se realiza persona a persona, mediante secreciones respiratorias al toser o estornudar o por transferencia manual (piel con traumatismo previo). Incubación de 2-4 días, la fase aguda constituye el periodo de mayor contagiosidad. Ingresa a vías respiratorias superiores y se disemina, tiene especial afinidad por el sistema linfático y el lugar de implantación es el tejido linfoide de faringe. Se extiende por los linfáticos o por continuidad a otras áreas cercanas, causando complicaciones tales como adenitis cervical, absceso periamigdalino o retrofaringeo, sinusitis, otitis. Para el desarrollo de escarlatina, la infección faríngea es por una cepa productora de toxina eritrogenica, la cual es antigénica y estimula producción de anticuerpos protectores (por eso solo da un cuadro). ESTREPTOCOCOS BETA HEMOLITICO DEL GRUPO A o Streptococcus pyogenes: Tiene una estructura celular compleja, su proteína M parece ser su principal factor de virulencia, las cepas ricas en ellas son resistentes a la fagocitosis. Elabora productos extracelulares; la toxina eritrogenica es la responsable del exantema en la escarlatina, su producción es inducida por un bacteriofago. La estreptomicina O y la S ambas tóxicas para los eritrocitos y otras células, incluso miocardio. La elevación de niveles serios de anti estreptomicina O. Es un buen indicador de infección estreptocócica reciente. Por evidencia seroepidemiologica, se considera que ciertos serotipos, causantes de faringoamigdalitis, están más relacionados con fiebre reumática, los de piel no causan fiebre reumática pero ambos son nefritogenicos (glomerulonefritis). DIAGNOSTICO: Clínica; faringoamigdalitis, forma tóxica con fiebre alta, nausea y vómitos, comienzo agudo, odinofagia, cefalea, malestar, faringe hiperemica, exudado blanco-amarillento en parches, adenopatías anterior dolorosa. Escarlatina; fiebre elevada, vomito, dolor faríngeo y cefalea, 12-48 hrs después aparece exantema y el enantema (lengua=fresa roja, paladar, amígdalas y faringe) característico. El rash se inicia en el tronco y se generaliza con rapidez, se presenta con una erupción con fondo eritematosa y a la vez puntico ene, con sección táctil de aspereza (papel de lija), respetando la cara. Siempre se presenta la descamación en mayor o menor grado. Impétigo; vesícula superficial con poco eritema e indolora, progresiva pústula con costra gruesa, con secreción meliserica, mas en extremidades y comisuras nasales y bucales. Erisipela; poco frecuente, celulitis caracterizada por una lesión eritematosa, con engrosamiento de la piel, dolorosa y con aumento de temperatura, borde bien definido. Otras infecciones son; piodermitis, ectima, celulitis o fscitis necrozante. El diagnostico de faringoamigdalitis se fundamenta en la triada amígdalas con exudado, odinofagia y adenopatias cervical anterior dolorosa, en un paciente que NO presenta tos, o síntomas nasales. El dx., de certeza aislamiento de S. pyogenes en cultivo de exudado faríngeo, aglutinación en látex y coaglutinacion, detectan antígeno. En la escarlatina el rash, y se puede hacer cultivo y antiestreptolisinas. TRATAMIENTO: Debido a que S. pyogenes sigue siendo sensible a penicilina, esta es el tx., de elección. Niños menores de 4 años; penicilina procainica 400,000UI c/24 hrs 2-3 días + p. Benzatinica 600,000UI, en una sola dosis, después de la procainica o penicilina benzatinica 600,000UI dosis única. Niños mayores de 4 años y adultos; penicilina procainica 800, 000UI c/24hrs 2-3 días + P. Benzatinica 1,200,000UI dosis única después de la procainica o penicilina benzatinica 1,200,000UI dosis única. El medicamento sustituto en alergia a penicilinas es eritromicina oral 30mg/kg/día, en dos o tres dosis diarias durante 10 días (dosis máxima 2 gramos). En caso de miosotis y fascitis necrozante, se recomienda penicilina, clindamicina y amikacina , hasta corroborar agente, si es pyogenes continuar solo penicilina y clindamicina.

CURSO ENARM CMN SIGLO XXI TEL: 36246001

Pharmed Solutions Institute

PÁGINA 127

MANUAL DE TRABAJO DEL CURSO ENARM CMN SIGLO XXI CASO CLINICO Varón con 43 años de edad, diabético sin control, que 24 horas antes presentó una contusión accidental contra el respaldo de la cama en la zona periorbitaria derecha, por lo que acudió a consultorio, donde se le realizó una curación y prescribió tratamiento con antiinflamatorios (diclofenaco sódico). En las horas siguientes cursó con fiebre, compromiso de conciencia y vómitos. Al examen físico se describió un paciente de aspecto tóxico, con los siguientes signos vitales: frecuencia cardíaca: 130/min, frecuencia respiratoria: 49/min, presión arterial: 77/55 mm Hg, T° rectal: 37,8° C, escala de Glasgow neurológico: 14. Destacaba un gran edema periorbitario bilateral con eritema y calor en la zona palpebral y en la mejilla derecha. Se hospitalizó con el diagnóstico de celulitis preseptal, iniciándose tratamiento con dicloxacilina endovenosa. PREGUNTA Cual de los siguientes agentes infecciosos es mas probable aisla en este caso? RESPUESTA a.- Estreptococos del grupo A b.- Estreptococos del grupo B c.- Estreptococos del grupo C d.- Estreptococos del grupo D CASO CLINICO Varón de 62 años, sin antecedentes clínicos de interés, que ingresó por presentar en las 48-72 h previas un cuadro de fiebre elevada, sudoración profusa, escalofríos, mialgias y posteriormente cefalea frontooccipital intensa, postración y hematuria macroscópica. No presentaba síntomas gastrointestinales, infecciosos, traumatismos ni intervenciones quirúrgicas previas. En la exploración física destacaba rigidez de nuca y afectación del estado general. No se encontraron signos de afectación cardiovascular ni digestiva. En los estudios analíticos destacan la presencia de leucocitosis con desviación izquierda, inicio de patrón analítico de coagulación intravascular diseminada (CID) y leucocituria con nitritos positivos en orina. Se realizó punción lumbar, obteniéndose líquido claro a presión normal cuyo examen mostró: leucocitos 1.200 cél./ml, proteínas 8 g/l, glucosa 9 mg/dl, Gram: se identificaron cocos grampositivos en cuyo cultivo se aisló S. bovis biotipo II PREGUNTA Cual de los siguientes grupos agentes infecciosos es mas probable aisla en este caso? RESPUESTA a.- Estreptococos del grupo A b.- Estreptococos del grupo B c.- Estreptococos del grupo C d.- Estreptococos del grupo D CASO CLINICO Mujer de 59 años con cuadro de 72 horas de evolución con astenia, anorexia y deterioro del nivel de conciencia que se acompañaba en las últimas 24 horas de disnea. Se observo con insuficiencia respiratoria parcial y una fibrilación auricular con respuesta ventricular rápida e inestabilidad hemodinámica secundaria motivo por el que se decidió el traslado a la Unidad de Cuidados Intensivos (UCI). Ingresa a UCI obnubilada, hipotensa (tensión arterial: 90/60 mmHg), en fibrilación auricular a 130 lpm, taquipnéica a 30 rpm y con saturación arterial de oxigeno de 90% según pulsioximetría respirando con mascarilla al 50% de fracción inspirada de O2. En la exploración física destacó únicamente la existencia de ingurgitación yugular bilateral e hipoventilación en la base pulmonar izquierda. A nivel analítico llamó la atención la existencia de leucocitosis con 15.300 leucocitos, neutrofilia y una importante desviación izquierda. Electrocardiograma con fibrilación auricular rápida sin alteraciones de la repolarización. Radiografía de tórax con cardiomegalia y borramiento de ambos hemidiafragmas. Se canalizó un catéter venoso central y un catéter PICCO, objetivándose un patrón hemodinámico compatible con sepsis (gasto cardiaco elevado con resistencias vasculares sistémicas disminuidas) y unos valores de presión venosa central elevados. PREGUNTA Cual de los siguientes grupos agentes infecciosos es mas probable aisla en este caso? RESPUESTA a.- Estreptococos del grupo A b.- Estreptococos del grupo B c.- Estreptococos del grupo C d.- Estreptococos no A no B. CASO CLINICO Paciente de 35 años, sexo femenino, no fumadora, fue sometida a tiroidectomía total por neoplasia folicular de tiroides y fue dada de alta en buenas condiciones generales a las 48 h. Consultó 5 dias despues por malestar general, odinofagia, cefalea, sensación febril, escalofríos, dolor cervical, tos productiva y expectoración mucopurulenta, siendo admitida a sala de cuidados generales. En el examen físico destacaba paciente en regulares condiciones generales, lúcida, orientada, FC: 90 lat/min, PA: 106/60 mmHg, FR: 24 resp/min, T: 37,8°C. Dolor a la palpación en la zona de herida operatoria de la región cervical anterior, faringe sana, examen cardiopulmonar y abdominal normales. Se planteó el diagnóstico de infección de herida operatoria y se inició tratamiento con ceftriaxona 1 g/día EV. La

CURSO ENARM CMN SIGLO XXI TEL: 36246001

Pharmed Solutions Institute

PÁGINA 128

MANUAL DE TRABAJO DEL CURSO ENARM CMN SIGLO XXI paciente evolucionó febril y con dificultad respiratoria progresiva, requiriendo aporte de oxígeno al 35% para corregir el trastorno del intercambio gaseoso (Pa02/Fi02: 259). En el hemograma destacaba leucopenia (2.700/mm3) con desviación a izquierda (13% baciliformes), PCR: 5,4 mg/dL, GSA: pH: 7,46, Pa02: 54,4 mmHg, Sa02: 89%, PaC02: 27,5 mmHg, B. act: 19,6 mEq/L. La radiografía de tórax mostró opacidades parenqui-matosas en lóbulo superior derecho, lóbulo medio, língula y lóbulo inferior izquierdo. PREGUNTA Cual de los siguientes grupos agentes infecciosos es mas probable aisla en este caso? RESPUESTA a.- Estreptococos del grupo A b.- Estreptococos del grupo B c.- Estreptococos del grupo C d.- Estreptococos no A no B.

CURSO ENARM CMN SIGLO XXI TEL: 36246001

Pharmed Solutions Institute

PÁGINA 129

MANUAL DE TRABAJO DEL CURSO ENARM CMN SIGLO XXI INFECCIONES POR ESTAFILOCOCOS: CIENCIAS BASICAS: Los estafilococos son cocos gram-positivos que forman racimos de uvas en la tinción de Gramm, son catalasa positivo (a diferencia de los estreptococos), no móviles, aeróbicos y anaerobias facultativas. Staphylococcus aureus, se distingue de otros estafilococos por su producción de coagulasa, es la especie más virulentas, causando la enfermedad a través de ambos mecanismos no mediadas por toxinas y mediada por toxina. Los estafilococos coagulasa negativos (ScoN) son menos virulentas que S. aureus, pero son causas importantes y comunes de las infecciones de dispositivos de prótesis. S. epidermidis con mayor frecuencia causa la enfermedad, este organismo es un componente normal de la piel, orofaríngea, y la flora vaginal. S. saprophyticus es una causa de las infecciones urinarias. Otras dos especies de ScoN, S. lugdunensis y S. schleiferi, son más virulentas y causan infecciones graves tales como válvulas nativas endocarditis y osteomielitis. SALUD PÚBLICA: S. aureus es responsable 25 a 35 % de los casos de endocarditis bacteriana. Las DEFINICIONES CLINICAS PARA SINDROME DE CHOQUE TOXICO tasas de mortalidad oscilan entre 20 a 40% a pesar de la Temperatura > 38.9° disponibilidad de antibióticos eficaces. PATOGENIA: Una vez que se Exantema macular difuso presenta la infección, la invasión local y sistémica ocurre por vía Descamación 1-2 semanas posteriores al inicio de la enfermedad hematógena y liberación de toxinas. Localmente los organismos (+en palmas y plantas) pueden invadir o necrosar el tejido y ocasionar una potente Hipotensión: Presión sistólica 15mmHg de acostado a sentado, sincope herida, inyecciones o a través de catéteres), mediada ortostático o mareo. Afección multisistémica: 3 o más de las siguientes principalmente por PMN (respuesta primaria). La formación de Gastrointestinal; vomito o diarrea al inicio de la enfermedad abscesos (microcápsula polisacárido antifagocítica, facilita la Muscular; mialgia severa o elevación de CPK, 2 veces arriba de lo evasión de las defensas del huésped, útil en la formación de normal absceso) es común, con un centro necrótico que está formado de Mucosas; hiperemia en conjuntiva, orofaringe, vagina pus y una pared de fibrina, que dificulta la penetración de Renal; nitrógeno ureico o creatinina por lo menos dos veces arriba antibióticos, y protege de defensas del huésped, las bacterias se de lo normal o sedimento urinario con piuria (>5 leucos x campo), adhieren formando una biopelícula similar a la formada por los en ausencia de infección de vías urinarias Hepática; bilirrubina total, transaminasas 2 veces por arriba de lo ScoN. La infección puede diseminarse localmente por la formación normal de trayectos sinuosos y abscesos secundarios. La diseminación 3 Hematológica: plaquetas 65 100mg c/24 hrs) y rimantadina (>10 años 100mgs c/12 hrs >65 100mg c/24 hrs) y los inhibidores de la enzima neuraminidasa; oseltamivir (>13 años 75mg c/12 hrs) y zanamivir (>10 años de edad 10mg c/12 hrs), los cuatro han demostrado utilidad clínica reduciendo la duración de los síntomas. Meningitis en adultos por H. influenzae tipo B: ceftriaxona (2 g cada 12 h durante 1-2 semanas). Meningitis por H. influenzae tipo B en niños: ceftriaxona (75-100 mg / kg por día, divididos en dos dosis cada 12 h) y dexametasona (0,6 mg / kg por día en cuatro dosis divididas durante 2 días a la iniciación del tratamiento con antibióticos para prevenir pérdida de la audición). La epiglotitis: ceftriaxona (50 mg / kg al día durante 1-2 semanas). PREVENCION: Vacunación anual de personas en grupos de alto riesgo, utilizando la vacuna inactivada trivalente por su eficacia y baja reactogenicidad, los grupos son los siguientes; vacunación a personal de salud, personal que labora en asilos de ancianos, personas que viven en contacto intradomiciliario con personas con alto riesgo de desarrollar complicaciones, mujeres con embarazo de alto riesgo (2do y 3er trimestre), personas mayores de 65 años, personas entre 50-64 años (enfermedades cronicodegenerativas), pacientes de cualquier edad con padecimientos crónicos (principalmente respiratorios y cardiovasculares), vacunación de niños de 6-59 meses y

CURSO ENARM CMN SIGLO XXI TEL: 36246001

Pharmed Solutions Institute

PÁGINA 134

MANUAL DE TRABAJO DEL CURSO ENARM CMN SIGLO XXI viajeros (que van a los trópicos o al Hemisferio Sur). La vacuna se administra por vía intramuscular, en región deltoidea (niños región anterolateral de muslo). Contraindicada en alergia al huevo o a otros componentes de la vacuna o cuadro febril agudo. CASO CLINICO Femenino de 46 años con antecedentes de asma bronquial tipo III, para lo cual lleva tratamiento con salbutamol (spray). Acudió por presentar fiebre de 39 0C y trastornos de la conciencia. Cinco días antes había presentado una crisis asmática acompañada de un cuadro febril asociado con síntomas respiratorios altos, que coincidió con un cuadro respiratorio alto en su nieto pequeño atendido por ella; por lo anterior fue vista en su área de salud donde le indicaron tratamiento con penicilina. El día antes presentó vómitos, en número de 12, de tipo bilioso y acompañados de náuseas, con mucha tos y abundante secreción nasal verdosa y fiebre. En el momento de ser valorada en el servicio de urgencia llamaba la atención los trastornos de conciencia que presentaba esta paciente, dados por la agitación psicomotora y el estado de estupor. Al examen físico se constató rigidez nucal y signos de irritación meníngea. Se le realizó punción lumbar (PL) de donde se obtuvo un líquido cefalorraquídeo (LCR) de aspecto turbio, así como otros exámenes complementarios, incluidos hemogramas seriados. El examen citoquímico del LCR mostró los valores siguientes: 435 leucocitos/mm3 a predominio de polimorfonucleares (PMN), hematíes 8/mm3, glucosa en 1,0 mmol/L y proteínas totales en 0,95 g /L. En el examen bacteriológico del líquido se informó pleomorfismo bacteriano gramnegativo. En el leucograma se constató una cifra de 16 000 leucocitos con predominio de PMN. El cultivo reporto haemophilus influenzae en el LCR. PREGUNTA Cual es la conducta mas apropiada a seguir con el diagnostico presuntivo del caso? RESPUESTA a.- Rivavirina y amantadina. b.- Ceftriaxona vía endovenosa. c.- Ceftriaxona y amikacina IV. d.- Cloranfenicol y ampicilina. TOS FERINA O COQUELUCHE (BORDETELLA PERTUSSIS): CIENCIAS BÁSICAS: Bordetella pertussis causa tos ferina, una infección aguda del tracto respiratorio, caracterizada por accesos de tos parodia ticos acompañado de estridor al final de la inspiración. B. pertussis es un bacilo aerobio Gramm negativo. SALUD PÚBLICA: La tos ferina es altamente contagiosa. En los hogares, las tasas de ataque son el 80% de los contactos no vacunados y 20% entre los contactos inmunizados. La tos ferina sigue siendo una causa importante de morbilidad y muerte infantil en los países en desarrollo. En los Estados Unidos, la incidencia ha aumentado lentamente desde 1976, particularmente entre los adolescentes y adultos. Tos persistente de duración> 2 semanas en un adulto puede ser debido a B. pertussis en 12-30% de los casos. La morbilidad y la mortalidad severa se limitan a lactantes 2 semanas del inicio de la sintomatología y la muestra para el cultivo se toma de la nasofaringe posterior. PCR a aumentado su sensibilidad, pero no se usa como método exclusivo y no sustituye al cultivo. Detección de anticuerpos fluorescentes (DAF), útil como prueba de detección rápida, pero la sensibilidad es baja. Serología; demostración de una elevación sustancial 4 veces los títulos de anticuerpos contra diferentes antígenos. COMPLICACIONES: La más frecuente y causas de muerte es la neumonía bacteriana secundaria, otras son crisis convulsivas y encefalopatía TRATAMIENTO: Del 8090% de pacientes con tos ferina sin tratamiento depuran espontáneamente a B. pertussis de la nasofaringeo en las 3-4 semanas. Manejo principal ente de soporte. La eritromicina es el antibiótico de elección, preferentemente en forma de estolato a dosis de 40mg/kg/día por 14 dias. Alternativas claritromicina por 7 días igual efectividad y mejor tolerancia. Azitromicina y TMP/ SFX. En menores de un mes se da azitromicina (eritro y claritromicina están contraindicadas). Para la tos se puede manejar benzonatato o salbutamol en nebulizador. PROFILAXIS POSTEXPOSICION: Contactos asintomáticos de un caso índice que se encuentre dentro de los 21 días del inicio de los accesos de tos, deberá recibir profilaxis con antibióticos antes mencionados. PREVENCIÓN: Vacuna combinada de toxoide tetánico y difterico (DPT). En México A partir de 1999, poco después de la introducción de la vacuna triple viral, se introduce una vacuna pentavalente (DTwP-hepatitis B-Hib), en tres dosis a los 2,4 y 6 meses de edad y refuerzos con DPT a los 4 años de edad. A partir de 2007, se efectuó un cambio biológico a una vacuna pentavalente a base de vacuna acentuar de pertussis (DTaP-IPV-Hib).

CURSO ENARM CMN SIGLO XXI TEL: 36246001

Pharmed Solutions Institute

PÁGINA 135

MANUAL DE TRABAJO DEL CURSO ENARM CMN SIGLO XXI CASO CLINICO Una mujer de 64 años ingresada por una disnea creciente y dolor torácico. La paciente tenía antecedentes de carcinoma pulmonar no microcítico y estaba recibiendo tratamiento. Anteriormente había fumado 4-5 cigarrillos al día durante un periodo de hasta 10 años. A la exploración, se trataba de una mujer con sensación de enfermedad y emaciación, que estaba clínicamente anémica y presentaba signos de un derrame pleural izquierdo masivo en la TC torácica. La frecuencia del pulso era de 85/min, la presión arterial de 120/85, y no había signos de insuficiencia cardiaca. La radiografía de tórax confirmó el derrame unilateral izquierdo. La hemoglobina era de 9,1g/100ml, con un frotis de sangre normocrómico y normocítico. La velocidad de sedimentación globular fue de 90mm en la primera hora. Se realizó una aspiración pleural, en la que se obtuvieron 1.500ml de líquido turbio. Dada la persistencia de las colecciones de líquido pleural, 2 días después de la segunda aspiración se colocó un drenaje de tórax. El nivel de proteínas fue de 35g/l y la concentración de glucosa, de 0,3mmol/l. No se observaron células malignas, pero en la PCR se demostró la presencia de B. pertussis. La paciente no presentó fiebre ni manifestaciones clínicas de infección, y el recuento leucocitario en sangre fue normal. Además, no hubo clínica de diarrea y los coprocultivos, urinocultivos y hemocultivos fueron repetidamente negativos. PREGUNTA Cual es la conducta antibiótica mas apropiada para el caso? RESPUESTA a.- Claritromicina. b.- Azitromicina c.- TMP/ SFX d.- Eritromicina. CASO CLINICO Mujer de 45 años, fumadora de 6 cig/día desde los 20 años, por malestar general, astenia, fatigabilidad y tos seca de predominio nocturno de seis semanas de evolución. No refere congestión nasal, rinorrea, descarga nasal posterior, cefalea, pirosis, fiebre, escalofríos, disnea, dolor torácico, expectoración ni hemoptisis. Antecedentes mórbidos: rinitis alérgica estacional en primavera con test cutáneos positivos a pólenes y pastos; refujo gastroesofágico en tratamiento con medidas dietéticas y Omeprazol 20 mg/día. Actividad laboral: enfermera de unidad de hemodiálisis de adultos. En el examen físico, los signos vitales eran normales con una SpO2 de 97% respirando aire ambiente, el examen cardiopulmonar y abdominal eran normales. La paciente fue tratada con medidas dietéticas y posturales antirefujo. Finalmente, la inmunofuorescencia directa de Bordetella pertussis de hisopado nasofaríngeo fue positiva. PREGUNTA Cual es la conducta antibiótica mas apropiada para el caso? RESPUESTA a.- Claritromicina. b.- Azitromicina c.- TMP/ SFX d.- Eritromicina. MORAXELLA CATARRHALIS: CIENCIAS BÁSICAS: Moraxella catarrhalis es un coco gram-negativo, que se asemeja a Neisseria. En el género Moraxella podemos identificar cuatro especies: M. catarrhalis, M. caviae, M. ovis, M. cuniculi. Forma parte de la flora normal de las vías respiratorias superiores. SALUD PUBLICA: M. catarrhalis coloniza hasta el 50% de los niños sanos (muy relacionados con IRA) y hasta el 3-7% de los adultos sanos. Las tasas de infección pico son a finales de invierno / principios de primavera. Generalmente no se le considera un agente primario en las infecciones del tracto respiratorio inferior, esto cambia al referirse a individuos mayores de 50 años, donde es considerado un patógeno primario de vías respiratorias bajas. Se puede decir que es un agente oportunista que se aprovecha de las condiciones predisponentes del huésped para causar enfermedad y formar parte de los patógenos humanos emergentes. DIAGNOSTICO: La otitis media y sinusitis: M. catarrhalis es la tercera causa más frecuente de otitis media en los niños y es importante aislarlo de los casos de sinusitis aguda y crónica. Conjuntivitis en neonatos. Traqueobronquitis purulenta, bronquitis, neumonía: La mayoría son pacientes > 50 años de edad y con EPOC (a menudo con el cáncer de pulmón también). Laringitis, queratinitis, uretritis. Los síntomas son de leves a moderados, la enfermedad invasiva (por ejemplo, enfisema) es poco frecuente. Laboratorio: Se cultivan muestras de esputo, sangre, secreciones bronquiales, aspirado transtraqueal, lavado broncoalveolar y biopsia pulmonar. Un método de diferenciación con la Neisseria, es que la Moraxella Catarrhalis no fermenta los carbohidratos y produce DNasa. Además produce Butirato esterasa, que constituye la base de la prueba fluorométrica rápida para identificarla. En la tinción aparecen bacilos, cocobacilos o cocos pequeños Gramnegativos. TRATAMIENTO: Este microorganismo presenta una alta resistencia a beta-lactámicos como la penicilina, ampicilina y amoxicilina. Se puede usar amoxicilina/ac. clavulánico, cefalosporinas de segunda y tercera generación, TMP-SMZ (mezcla de una parte de trimetoprim y cinco partes de sulfametoxazol). Esta bacteria presenta sensibilidad a la quinolonas, eritromicina, tatraciclina, amikacina, imipenem, meropenem, cloranfenicol. CASO CLINICO Masculino de 15 años, fue ingresado con una historia de 3 días de dolor de cabeza y de 2 días de petequias generalizadas, náuseas y vómitos. Temperatura 36,9 ℃ , FC 93 latidos / min y la TA 125/82 mmHg. Estaba alerta y se quejó de rigidez en el cuello. No hubo ninguna secuela motora o sensorial , y el resto de su examen físico sin complicaciones. Leucocitos periféricos de 30.290 / l (96 %

CURSO ENARM CMN SIGLO XXI TEL: 36246001

Pharmed Solutions Institute

PÁGINA 136

MANUAL DE TRABAJO DEL CURSO ENARM CMN SIGLO XXI neutrófilos), hemoglobina de 15,4 g / dl, y plaquetas de 151.000 / l . CSF tenía un aspecto turbio con un aumento de la concentración de proteína de 575 mg / dl, leucocitos de 35.500 / l ( 90 % de neutrófilos ) y glucosa de 10 mg / dl. PREGUNTA Cual es la conducta antibiótica mas apropiada para el caso? RESPUESTA a.- Ampicilina y acido clauvulanico. b.- Ceftazidima y netilmicina. c.- Quinolonas e imipenem. d.- Eritromicina y meropenem. CASO CLINICO Masculino de 81 años, con antecedentes de cáncer de páncreas y cirrosis hepática por virus de la hepatitis C. Fue ingresado por inversión del ciclo vigila sueño. A su ingreso, tenía ictericia esclerótica. Leucocitos 6950 / l (73% neutrófilos), hemoglobina de 12,8 g / dl, plaquetas de 60.000 / l. LCR estaba claro con proteína de 38 mg / dl, glóbulos blancos de 9/μL, glucosa de 76 mg / dl, nitrógeno ureico / creatinina 41.8/2.2 mg / dl, amoníaco de 287 mg / dl, bilirrubina total de 3,1 mg / dl. PREGUNTA Cual es la conducta antibiótica mas apropiada para el caso? RESPUESTA a.- Ampicilina. b.- Ceftaxima. c.- Imipenem. d.- Eritromicina. BRUCELOSIS (BRUCELLA MELITENSIS): CIENCIAS BÁSICAS: La Brucella es una bacteria gramm negativa, viven en el interior de las células del sistema fagocitico mononuclear (intracelularfacultativo), por ello tiene un curso recidivante, tratamiento difícil y prolongado. Todas las especies de Brucella tienen como reservorio las ubres o útero gestante de las hembras y los genitales del macho. La B. melitensis (caprinos y ovinos), es el agente causal más importante, seguido de B. abortus (bovinos) y B. suis (porcinos) y B. canis (de perros). La brucelosis es una zoonosis sistémica en el humano puede afectar cualquier órgano o sistema, generalmente adopta un curso crónico, ataque al estado general, baja letalidad. SALUD PÚBLICA: OMS, la considera la zoonosis de mayor distribución en el mundo. Cada año ocurren medio millón por lo menos de casos nuevos. En México existe en todo el territorio nacional predominando en un área triangular con la base en la frontera norte y el vértice en el centro. PATOGENIA: La brucelosis se transmite por ingestión, inhalación o exposición percutánea o mucosa, la enfermedad en los seres humanos por lo general se asocia con la exposición a animales infectados o sus productos, ya sea en lugares de trabajo (por ejemplo, el trabajo de los mataderos, la agricultura) o el contexto familiar (por ejemplo, el consumo de alimentos, especialmente productos como leche no BRUCELOSIS TUBERCULOSIS pasteurizada, queso, mantequiilla) contaminados. SITIO Lumbar y otros Dorsolumbar Ya en el organismo son fagocitados por leucocitos, LAS VÉRTEBRAS Múltiple o contiguas Contiguo PMN, macrófagos tisulares, donde pueden DISQUITIS Tarde Temprano sobrevivir y multiplicarse. Una vez rebasada la CUERPO Intacto hasta el final Morfología de perdida temprana barrera linfática, llegan a la circulación sistémica, y COMPRESION DEL Raro Común de esta manera son transportados a los diferentes CANAL EPIFISISTIS Anterosuperior Signo de General: las regiones de disco órganos del sistema fagocitico mononuclear Pom´s superior e inferior , central, (hígado, bazo, medula ósea, ganglios). La subperióstico persistencia de Brucella dentro de las células es OSTEOFITOS Anterolateral (pico de Inusual debido a la inhibición de la fusión fagosomaloro) lisosomal, la degranulacion y la activación del DEFORMIDAD En cuña poco comun Cuña anterior, giba ABSCESOS Pequeños, bien Pérdida Común y discreta, apófisis sistema mieloperoxidasa. La liberación de las PARAVERTEBRALES localizados transversa bacterias de las células necróticas puede sobrepasar RECUPERACIÓN Esclerosis, todo el variable la capacidad fagocitica. Las manifestaciones clínicas cuerpo están determinadas en gran parte por la liberación de una endotoxina y el grado de hipersensibilidad a los antígenos brucelares. DIAGNOSTICO: Clínico: El período de incubación de 1 semana hasta varios meses es seguido por el desarrollo de la fiebre ondulante, sudores, aumento de la apatía, fatiga y anorexia y síntomas inespecíficos como dolor de cabeza, mialgias y escalofríos. La brucelosis menudo se presenta con uno de los tres patrones: una enfermedad febril similar pero menos grave que la fiebre tifoidea, fiebre y monoartritis aguda, típicamente de la cadera o de la rodilla, en un niño pequeño (artritis séptica), o la fiebre de larga duración y baja de la espalda o dolor en la cadera en un hombre mayor (osteomielitis vertebral). Brucelosis puede causar linfadenopatía, hepatoesplenomegalia, epididimoorquitis, compromiso neurológico y el absceso focal. El microorganismo se cultiva con éxito en el 50-70 % de los casos, pero la cultura de identificación suele tardar hasta 6 semanas. El diagnostico de certeza es el aislamiento de brucella, en hemocultivo o cultivo de medula ósea. Fiebre prolongada acompañada de ataque articular nos hace sospecharla. Ensayos de aglutinación para IgM son positivos en la infección temprana. Los títulos individuales de ≥ 1:160 y 1:320-1:640 son diagnósticos en áreas no endémicas y endémicas, respectivamente. La brucelosis se debe distinguir de la tuberculosis, y si esta distinción no es posible, el régimen debe adaptarse para evitar la monoterapia inadvertida para la tuberculosis. Brucelosis tiende a causar menos destrucción del hueso y la articulación de la tuberculosis. TRATAMIENTO: Se recomienda como primera elección estreptomicina a una dosis de 750 mg a 1 g al día (o gentamicina a 5-6 mg / kg al día) durante 14-21

CURSO ENARM CMN SIGLO XXI TEL: 36246001

Pharmed Solutions Institute

PÁGINA 137

MANUAL DE TRABAJO DEL CURSO ENARM CMN SIGLO XXI días más doxiciclina en una dosis de 100 mg C/12 hrs durante 6 semanas. Alternativa: TMP/SFX + Rifampicina por 6 semanas. Enfermedad compleja (por ejemplo, enfermedad neurológica significativa o endocarditis) requiere al menos 3-6 meses de tratamiento con múltiples agentes. Alternativa: rifampicina (600-900 mg / d) más doxiciclina (100 mg bid) durante 6 semanas. El trimetoprim /sulfametoxazol, se puede dar en vez de doxiciclina - por ejemplo, a los niños o las mujeres embarazadas. La recaída se produce en aproximadamente el 30 % de los casos, por lo general debido a la falta de cumplimiento. El paciente debe controlarse por lo menos 2 años. CASO CLINICO Un paciente varón de 20 años de edad, que se acababa de alistar en el ejército, se presentó refiriendo tos, expectoración, dolor punzante en el hemitórax izquierdo, sudoración nocturna, anorexia, fiebre ondulante y disnea de casi 10 días de evolución. También refirió la pérdida de 3kg de peso en el último mes. Presentaba fiebre de 38,4°C y su frecuencia respiratoria era de 22/min. La auscultación torácica reveló estertores inspiratorios basilares bilaterales. Se detectó un recuento de leucocitos de 27.930/mm3 y una velocidad de sedimentación de 68mm/h. El valor de proteína C reactiva era de 141mg/dl. La radiografía de tórax mostró un infiltrado neumónico en la zona media derecha y una opacidad homogénea en la zona inferior izquierda. Se documentó el crecimiento de Brucella spp. en el hemocultivo. PREGUNTA Cual es la conducta antibiótica mas apropiada para el caso? RESPUESTA a.- Rifampicina b.- Estreptomicina c.- Gentamicina d.- Doxiciclina CASO CLINICO Varón de 74 años había trabajado, la mayor parte de su vida, en criaderos de aves de zonas rurales del Área Metropolitana. Su cuadro clínico se había iniciado aproximadamente tres meses antes con compromiso moderado del estado general, fiebre vespertina y pérdida de peso, que en ese momento cuantificaba en diez kilos. Sólo se verificó la existencia de fiebre irregular, alrededor de 38°C, y decaimiento. Se inició estudio del síndrome febril prolongado. El hemograma mostraba anemia moderada (hematocrito 34,1%, hemoglobina 11,3 gr%), leucocitos en rango normal, neutrofilia de 76%) sin desviación a izquierda y VHS de 36 mm/h. Albuminemia (3,3 gr%), la proteína C reactiva moderadamente elevada. PREGUNTA Cual es la conducta antibiótica mas apropiada para el caso? RESPUESTA a.- Rifampicina b.- Estreptomicina c.- Gentamicina d.- Doxiciclina TULAREMIA (FRANCISELLA TULARENSIS): CIENCIAS BASICAS: Las infecciones humanas causadas por Francisella tularensis, que es una bacteria facultativa intracelular, se producen a través de la interacción cuando el insecto muerde, o chupa sangre (especialmente las garrapatas y las moscas tabanid), animales salvajes o domésticas (por ejemplo, conejos salvajes, ardillas), o el medio ambiente. El organismo puede persistir durante meses en el barro, agua, y los cadáveres de animales en descomposición. Más de la mitad de los casos en Estados Unidos se producen en Arkansas, Oklahoma y Missouri. PATOGENIA: La F. tularensis entra en la piel o las membranas mucosas a través de mordeduras o rasguños inaparentes o se adquiere por inhalación o ingestión. Su patogenicicdad se debe a su capacidad para sobrevivir dentro de macrófagos no estimulados. Los macrófagos fagocitan con facilidad a la F. tularensis pero esta resiste la muerte frente a los radicales derivados del oxigeno, una respuesta neutrofilica local contraresta la infección, esta respuesta evoluciona a una ulcera. DIAGNOSTICO: El período de incubación es de 2-10 días. La tularemia a menudo comienza con un comienzo agudo de fiebre, escalofríos, dolor de cabeza y mialgias. Uno de varios síndromes puede desarrollar: Tularemia Ulceroglandular / glandular (75-85 % de los casos). El sello es una induración, eritema, úlceras que no sanan 1-3 semanas duraderos (forma ulceroglandular) que comienza como una pápula pruriginosa, ulcera, bordes drásticamente marcados y un exudado amarillo, y desarrolla una base de negro. Una lesión primaria de piel puede no ser evidente en el 5-10% de los casos (forma glandular). Linfadenopatía está relacionada con la ubicación de la picadura de la garrapata; linfáticos inguinales / femoral están afectadas con mayor frecuencia en los adultos debido a la frecuencia de picaduras en las piernas. Los ganglios linfáticos pueden llegar a ser fluctuante y drene espontáneamente. Tularemia oculoglandular: Infección de la conjuntiva, por lo general por el contacto con los dedos contaminados, resultados en conjuntivitis purulenta con adenopatía regional y dolor debilitante. Doloroso linfadenopatía preauricular es único a la tularemia. Orofaríngea y gastrointestinal: Adquirida por ingestión, la infección se puede presentar con faringitis y adenopatía cervical, ulceraciones intestinales, adenopatías mesentéricas, diarrea, náuseas, vómitos y dolor abdominal. Tularemia pulmonar: La infección se adquiere por inhalación o por vía hematógena. El paciente se presenta con una tos no productiva, disnea, dolor torácico pleurítico, infiltrados irregulares o lobar bilaterales, derrame pleural y empiema ocasionales en la radiografía de tórax. Tularemia tifoidea: Consta de fiebre y signos de sepsis y sin hallazgos focales. Laboratorio: Tinción policromática de muestras clínicas (tinción de poca ayuda). Serología mediante microaglutinación o la prueba de aglutinación en tubo. Un solo título de ≥ 1:160 o un aumento de cuatro veces en el título después de 2-3 semanas se consideran

CURSO ENARM CMN SIGLO XXI TEL: 36246001

Pharmed Solutions Institute

PÁGINA 138

MANUAL DE TRABAJO DEL CURSO ENARM CMN SIGLO XXI positivos. La cultura es difícil y supone un riesgo importante para el personal de laboratorio. Métodos de reacción en cadena de la polimerasa (PCR) se han utilizado para detectar el ADN de F. tularensis en muestras clínicas. TRATAMIENTO: La gentamicina se considera el fármaco de elección para adultos (5 mg / kg al día C/12hrs) y niños (2,5 mg / kg tres veces al día o 5 mg / kg c/12) con tularemia. Estreptomicina (1 g cada 12 horas) también es eficaz, pero no es tobramicina. Desaparición de la fiebre por lo general ocurre dentro de 2 días, pero la curación de las lesiones de la piel y los ganglios linfáticos puede tardar 1-2 semanas. Late supuración de ganglios linfáticos puede ocurrir, con el tejido necrótico estéril. Enfermedad leve a moderada responden rápidamente a tratamiento pueden ser tratados durante 5-7 días, de lo contrario, el tratamiento se administra durante 7-10 días. Las alternativas incluyen tetraciclinas o cloranfenicol (tasas de recaída de hasta el 20 %). Las fluoroquinolonas han demostrado ser prometedores, pero los ensayos clínicos están pendientes. CASO CLINICO Se trata de un paciente varón de 36 años, trabajador en un matadero industrial de aves. Entre sus aficiones destaca la caza de liebres, conejos y jabalíes, además de su hobby como taxidermista (su última pieza disecada fue un zorro). No tiene perros ni gatos y niega haber recibido transfusiones o tener otras enfermedades en curso. El paciente acudió al médico por presentar fiebre de 40 grados, sudoración, quebrantamiento general, mialgias, odinofagia y cefalea. Se interpretó el cuadro clínico como un síndrome gripal y se trató con paracetamol. La fiebre persistió durante los siguientes 3 días, y por ello acudió al hospital. Tras una exploración física y una radiografía de tórax normal, continuó con el mismo diagnóstico y tratamiento. Aproximadamente una semana después del comienzo de los síntomas, presentó una tumoración dolorosa en la axila derecha que aumentó de tamaño progresivamente hasta alcanzar los 5 cm de diámetro. De las pruebas complementarias que allí se le realizan destacamos las siguientes: la radiografía de tórax es normal, la analítica presenta neutrofilia con desviación a la izquierda y la ecografía axilar derecha revela un conglomerado de adenopatías de 5 x 3 cm. PREGUNTA Cual es la conducta antibiótica mas apropiada para el caso? RESPUESTA a.- Rifampicina b.- Estreptomicina c.- Gentamicina d.- Doxiciclina SINDROME DE INMUNODEDICIENCIA ADQUIRIDA (SIDA); VIRUS DE LA INMUNODEFICIENCIA HUMANA (VIH) CIENCIAS BASICAS: SIDA; Alteraciones inmunitarias profundas, infecciones bacterianas recurrentes o por gérmenes oportunistas y a aparición de formas comunes de neoplasias malignas, causada por VIH -1 (América, Europa, Caribe), VIH-2 (oeste de África). El VIH pertenece a la familia de retrovirus (poseen enzima transcriptasa reversa) citopaticos, no transformantes denominados lentivirus. Las regiones codificadores del VIH son los genes: gag=poliproteina precursora, que es escindida para proteínas de la capside, pol=transcripatasa inversa, integrasa y proteasa, env=proteínas de la cubierta. SALUD PUBLICA: Los primeros casos de SIDA, fueron reconocidos en los Angeles en 1981. Según el CENSIDA, los casos acumulaos suman, hasta junio del 2007, 112,830, de los cuales 2,720, ocurren en el grupo de los menores de 15 años. Proporción hombre: mujer 6:1, en el año 2003, para el grupo más afectado (25-39 años). El riesgo de transmisión vertical del VIH-1 de una madre infectada al producto varía entre 11-50%. La transmisión sexual en hombres aun es predominantemente homo-bisexual. PATOGENIA: La infección por VIH, puede adquirirse por transmisión sexual (8090%) exposición parenteral a sangre o derivados (por agujas en usuarios de drogas, o trabajado a de la salud), y de madres infectadas a sus productos, durante el período perinatal. Rutas menos frecuente a través de trasplante de órganos, tejidos y semen contaminado. El riesgo de adquirir infección depende del tipo de exposición (transfusión de un donador seropositivo es de 100%, exposición única a aguja contaminada es de 0.4%). La transmisión del VIH de la madre al niño además de in útero, puede ocurrir intraparto, por exposición del producto a la sangre materna o las secreciones genitales infectadas, esta transmisión se puede evitar mediante estrategias de profilaxis con antiretrovirales durante el embarazo y parto así como mediante la realización de cesárea programa en mujeres infectadas con VIH-1, con carga viral de >1,000 copias, a la semana 38 de gestación antes de que se haya desencadenado el TDP y de que se presente ruptura de membranas. Existe un riego adicional por lactancia materna del 14% en casos de infección establecida y 29% de infección primaria. No hay evidencia que el VIH se pueda transmitir entre los miembros de una familia, a través de contacto estrecho o por compartir utensilios. El VIH se une a la célula blanco (tiene tropismo por los T CD4), que tiene un receptor de alta afinidad de la glicoproteina de la envoltura gp120 del virus, lo que permite su unión, una vez unido se produce la fusión de su envoltura externa del virus a la membrana celular a través del gp41 con lo que el VIH entra a la célula, pierde su envoltura proteica, libera su ARN y las proteínas codificadas por la región Pol, como la transcriptasa inversa, en el citoplasma. Esta enzima dirige la construcción de una cadena de ADN sobre la plantilla de ARN viral, y en segundo paso cataliza la copia de la cadena recién sintetizada para producir un ADN de cadena doble (proviral). Este ADN proviral se integra en el ADN cromosómico de la célula huésped. Después de su integración, el ADN proviral del VIH pasa a un estado productivo, en el cual el ADN proviral se transcribe en ARN viral y ARN mensajero, el cual codifica la si resistí de proteínas virales, necesarias para la replicación viral, mediante el uso de las funciones metabólicas de la célula huésped. Los estadios finales comprenden el proceso de ensamblaje o encapsulación viral y la protrusión en la superficie celular; en esta etapa el virus adquiere una envoltura lipidica, siendo finalmente liberado para infectar a otras células y repetir el ciclo. Durante la fase asintomática de VIH (Clínica latente), los niveles circulantes del virus y las cuentas de CD4 permanecen esencialmente constantes (equilibrio); el grado de infección de las células es muy semejante al grado de muerte de las células infectadas. Con la replicación viral, la célula experimenta en pocos días degeneraciones balóninformes (tumefacción celular por alteración en la permeabilidad de la membrana), lo que conduce finalmente a la muerte celular. Las células infectadas pueden volverse más susceptibles a la súper infección por otros patógenos, lo cual puede llevar más rápidamente a la depleción de células T, que son las células críticas de la activación de la respuesta inmunitaria especifica, tanto celular como humoral. Esta inmunosupresión adquirida, explica la gran

CURSO ENARM CMN SIGLO XXI TEL: 36246001

Pharmed Solutions Institute

PÁGINA 139

MANUAL DE TRABAJO DEL CURSO ENARM CMN SIGLO XXI susceptibilidad a presentar infecciones por gérmenes poco frecuentes, como neoplasias malignas. No solo los linfocitos CD4 se afectan, también monocitos y macrófagos pueden ser infectados por el VIH. La afección de los macrófagos alveolares, explica la susceptibilidad a neumonía por Pneumocystis jiroveci. La alteración funcional de las células fagociticas también compromete a los neutrófilos PMN, lo cual explica la susceptibilidad a gérmenes piogenos. También hay alteración en linfocitos B, las cuales consisten en activación policlonal de estas células, por lo que se presenta hipergammaglobulinemia haciendo susceptible a gérmenes encapsulados como Streptococcus pneumoniae y Haemophilus influenzas. DIAGNOSTICO: Clínico: La enfermedad de SIDA ocurre cuando el huésped infectado, ya con graves alteraciones en sus mecanismos de defensa, no puede controlar microorganismos oportunistas o neoplasias malignas que rara vez causan enfermedad en el individuo inmunocompetente. Las manifestaciones especificas más frecuentes son: infección primaria=síndrome retroviral agudo, dermatológicamente eritema maculopapular y ulceración mucocutanea, además, fiebre, faringitis, linfadenopatía, artralgias, mialgias, anorexia, náusea, pérdida de peso, diarrea intermitente. A nivel neurológico puede haber; Encefalopatía; cuando VIH llega a SNC, en la fase temprana hay disminución de la capacidad de concentración, perdida de la memoria y lentitud mental, hipertrofia, ataxia, temblores, en la fase tardía incrementan todos los síntomas y signos, puede haber convulsiones y psicosis, hay datos de extrema atrofia cerebral y son común es los cambios en la sustancia blanca. La incidencia de gastroenteritis es hasta de 77% cursan con diarrea (crónica o intermitente) en algún momento, la causa más frecuente es Cryptosporidium. La infección por mycobacterium tuberculosis es la mayor complicación ESTADIOS CLÍNICOS DE LA INFECCIÓN POR EL VIH POR LA CDC 1993 oportunista que ESTADIO A ESTADIO B: Denotan progresión de la ESTADIO C: Define un caso de SIDA por medio de las entidades afecta a los adultos enfermedad, pero que por sí solas no clínicas asociadas que se presentan con VIH, con cualquier clasifican al paciente como enfermo de recuento de CD4 y SIDA Infección o Sintomático (sin condición A ni C) Candidiasis de bronquios, tráquea o pulmones puede incrementar el Síndrome Candidiasis orofaríngea o vaginal, por más Candidiasis esofágica riesgo de retroviral de un mes o que responde pobremente al Cáncer cervical invasivo complicaciones y agudo (SRA) tratamiento Coccidiodomicosis diseminada mortalidad asociada a Infección Leucoplasia vellosa de la lengua Criptococosis crónica intestinal (>1mes) VIH. Muchas de las asintomática Displasia cervical severa o carcinoma in CMV en cualquier órgano (excepto hígado, bazo o ganglios) Linfadenopatía situ Encefalopatía asociada a VIH infecciones generalizada Enfermedad pélvica inflamatoria Herpes simplex, causando ulceraciones crónicas por más de un mes oportunistas en el persistente Herpes zoster en más de una dermatoma o Histoplasmosis diseminada adulto son (LGP) por más de 2 episodios Isosporosis crónica por más de un mes secundarias a Purpura trombocitopenica idiopática Sarcoma de Kaposi Neuropatía periférica Linfoma de Burkitt, inmunoblastico o linfoma primario de cerebro reactivación de Listeriosis Mycobacterium tuberculosis pulmonar o extrapulmonar patógenos Angiomatosis bacilar Mycobacterium avium intracellulare oportunistas como: Síndromes constitucionales (diarrea por Otras micobacterias diseminadas Mycobacterium mas de 1 mes, fiebre prolongada de origen Neumonia por P. jiroveci desconocido) Leucoencefalopatia multifocal progresiva avium, candida, Septicemia recurrente por Salmonella criptococcus Toxoplasmosis cerebral (meningitis con CD4 < Síndrome de emaciación con pérdida de peso de >4.5 kg o más de 50 cels/mcl), herpes 10% del peso usual. simple, toxoplasma, histoplasma, CMV (coriorretinitis, perdida de visión, infiltrados amarillentos y/o hemorragias). Toxoplasma (causa mas común de déficit focal del SNC, cefalea confusión, < 50cels/mcl, TAC lesiones multiples en anillos, tx. Primetamina+ sulfadiazina+ leucovorin por 6 sem), candida albicans (candidiasis esofagiga definitoria de SIDA, tx. Fluconazol 14-21 dias) adquiridos previamente a la infección por VIH. De acuerdo al INDRE, las infecciones oportunistas más frecuentes en pacientes con SIDA en México son: 1. Cándidiosis (37.6%) 2. Neumonía por Pneumocystis jiroveci (12.2%). Tuberculosis (11.1%) herpes simple (7.4%), y Herpes zoster (7.2%). Otras manifestaciones características son las neoplasias malignas. El Sarcoma de kaposi (neoplasia mas común en VIH, nódulos, placas, alargamiento de linfonodos), se observa principalmente en adultos, siendo su presentación poco frecuente en niños. Dentro de este grupo se incluyen las neoplasias del SNC, como los linfomás no Hodking del cerebro (200 veces más común en pacientes con VIH, origen de cel. B, síntomas fiebre, diaforesis, perdida de peso). Laboratorio: la demostración de anticuerpos anti-VIH puede realizarse por varios métodos, como ELISA más utilizado, sensibilidad 95%, especificidad 99%, en las primeras 8-12 semanas puede dar negativo , de ahí que si ELISA sale positiva, se recomienda realizar una segunda prueba más especifica (prueba confirmatoria) y Western el cual reacciona con los antígenos virales, esta prueba tiene especificidad del 100%. Se considera como persona infectada por VIH o serio positiva aquella que presente: a) dos resultados positivos de pruebas de Tamizaje positivos (ELISA, aglutinación o pruebas rápidas) y pruebas confirmatorias de Western Blot positiva. b) dos resultados. Pruebas positivas de Tamizaje, con cuadro clínico sugerente de infección y c) alguna prueba suplementaria positiva (cultivo, Ag p24, PCR). EN infecciones oportunistas, Diagnostico de SIDA: infección declaratoria y CD4 6 sem) simétrica, afecta principalmente manos (metacarpofalangicas 90%, carpo 80%), rodillas 60% y pies (metacarpo e interfalangicas), más de 4 articulaciones afectadas. Rigidez matutina >1hr y dolor inflamatorio que mejora con la actividad física. Astenia adinamia, hiporexia, cervicalgia. Desviación cubital de dedos, nódulos de Bouchar (a nivel de interfalangicas proximales), sinovitis, dedos en cuello de cisne o botonero, hallux valgus (en MsPs), pulgar en “Z”, nódulos reumatoideos (subcutáneos en superficies de extensión). También hay manifestaciones extraarticulares principalmente a pulmón generando dolor pleurítico y derrame pleural (25%), hematológicas: anemia normocitica normocromica (30%), queratoconjuntivitis Sicca (25%) ojo seco, Sx. del túnel del carpo por atrapamiento. Factor reumatoide (FR) positivo en 65% de los pacientes, son autoanticuerpos dirigidos contra IgG, no es tan específicos porque puede encontrase en oras enfermedades inflamatorias autoinmunes. La presencia de FR muy elevado, se relaciona con formas mas agresivas de la enfermedad. Anticuerpos anti-peptido cíclico citrulado (Anti-PCC) mas especifico, tiene una especificidad de 90-95%. VSG y PCR; siempre deben de pedirse, ambos se encuentran elevados. Anti-DNA negativo (especifico para LES). Cambios Rx: inflamación de articulaciones metacarpofalangicas, pérdida de espacio de articulaciones metacarpo e interfalangicas y radiocarpiana, erosiones oseas, osteopenia yuxtaarticular, luxación de metacarpofalangicas, nódulos de Bouchar. Criterios Dx: Rigidez matutina al menos 1 hora o mas. Artritis de 3 o mas articulaciones. Artritis de las manos. Artritis simetrica. Nodulos reumatoideos. FR positivo. Cambios radiológicos; Diagnostico de artritis con 4 de 7 criterios. COMPLICACIONES: Derrame lateral de rodillas de predominio izq., puede presentarse quiste de Barker en hueco poplíteo asociado a ruptura e invasión muscular. TRATAMIENTO: Metotrexate piedra angular 10 mgs inicial, se puede aumentar hasta 25 mgs/sem.. Prednisona no es de elección, se da en dosis bajas y no modifica el curso de la enfermedad. Los AINES se dan junto con los FARMES (fármacos modificadores de la enfermedad). En artritis activa leve: Hidroxicloroquina (HCQ) o sulfazalazina (SSZ) ambos tardan 2-3 meses en iniciar su acción. Artritis activa moderada a severa: iniciar con metotrexate, y considerar prednisona. Si hay una respuesta inadecuada agregar otro FARME (MTX + Abatacep ó MTX + leflunomide ó MTX + SSZ + HCQ ó MTX + Rituximab). Los biológicos (anti-TNF) no son actualmente de 1ra. Elección. PRONOSTICO: Empeoramiento progresivo de la salud, ya que daña permanentemente los huesos, cartílagos y articulaciones. Si FR y ANTI-PCC muy elevados indican mal pronóstico y mayor gravedad. PREVENCION: En los 2 primeros años de inicio de la enfermedad daño severo e irreversible. Un tratamiento precoz disminuye y previene el daño. Derivación precoz ante sospecha dx. A reumatología. Toda paciente requiere contar con radiografía de manos y pies en AP y oblicuas. La suspensión del habito tabáquico puede ayudar a prevenir el desarrollo de AR. CLAVES: Nódulos de Heberden en interfalangicas distales nos habla de osteoartritis. Raro debut con poliartritis aguda con fiebre y linfadenopatías, pensar en rotavirus, si hay monoartritis en extremidades inferiores pensar espondiloartropatias seronegativas aun mas si se acompaña de lumbalgía. Síndromes específicos en pacientes con AR: Sx. de Sjogren secundario= AR + Sx. SICCA (xerostomía, xerostalmía). Sx. de Felty= AR+ esplenomegalia+ leucopenia. Sx. de Caplan= AR + nódulos reumatoideos + neumoconiosis. Amiloidosis = AR de larga evolución y grave proteinuria e Insf. Renal

CURSO ENARM CMN SIGLO XXI TEL: 36246001

Pharmed Solutions Institute

PÁGINA 147

MANUAL DE TRABAJO DEL CURSO ENARM CMN SIGLO XXI NEOPLASIA DEL SISTEMA NERVIOSO CENTRAL Epidemiologia: la incidencia de los tumores del SNC oscilan entre el 5.70 y el 9.63 en hombres y entre el 4.71 y el 6.95 en mujeres. Sin embargo, en todas las series los tumores más frecuentes son los gliomas, y dentro de ellos los glioblastomas. En cuanto a la edad de presentación, los tumores del SNC muestran una distribución bimodal, con un pico en la edad pediátrica y un aumento progresivo de la incidencia en los adultos, hasta alcanzar su máximo entre la sexta y la séptima década de la vida. La incidencia combinada de tumores del SNC en EEUU fue de 6,6 casos nuevos por 100.000 habitantes-año, en tanto que la mortalidad se estimó del 4,7 por 100.000 habitantes/año. Clasificación: El grado I incluye tumores de bajo potencial proliferativo y posibilidad de curación con extirpación quirúrgica completa. El grado II incluye tumores con capacidad infiltrativa y capacidad de recurrencia y progresión a mayor grado de malignidad, a pesar de presentar un bajo nivel de actividad proliferativa (por ejemplo, los gliomas de bajo grado pueden progresar a gliomas anaplásicos y/o glioblastoma). El grado III incluye tumores con evidencia histológica de malignidad, como atipia nuclear y alta actividad mitótica. El grado IV incluye tumores con evidencia histológica de malignidad (atipia nuclear, alta actividad mitótica, necrosis, proliferación microvascular) y comportamiento clínico agresivo con rápida progresión, recurrencia a pesar de tratamiento intensivo y muy alta mortalidad. Algunos tumores de grado IV como los meduloblastomas y tumores de células germinales son rápidamente fatales si no se tratan, pero se asocian a altas tasas de respuesta y supervivencia con tratamiento adecuado. Aunque no son requisitos indispensables, la capacidad de infiltración difusa del tejido sano adyacente y la propensión a diseminación leptomeníngea (e incluso extraneural) son características de algunos tumores de grado IV. Diagnóstico: Los tumores del sistema nervioso suelen presentarse con síntomas neurológicos de instauración progresiva, aunque también es posible la presentación aguda (por ejemplo, crisis comiciales o déficit neurológico agudo secundario a hemorragia intratumoral). Así mismo, los síntomas pueden ser generalizados (por aumento inespecífico de la presión intracraneal) o focales (dependientes de la localización tumoral).La cefalea es el síntoma de presentación más frecuente de los tumores cerebrales, aunque como síntoma aislado sólo se da entre un 2 y un 16% de los casos. Las crisis comiciales son el segundo síntoma en frecuencia, pudiendo desarrollarlas hasta un 35% de pacientes. Son especialmente habituales en gliomas de bajo grado. Pueden tener un inicio focal y reflejar la localización del tumor, aunque en más de la mitad de los casos se produce una generalización secundaria y el inicio focal puede ser muy difícil de identificar en base a criterios clínicos. Otros síntomas frecuentes de presentación son la disfunción cognitiva, las nauseas y vómitos, la disfunción endocrina, los síntomas visuales y los síntomas focales dependientes de la localización tumoral. Diagnóstico por imagen: La técnica de neuroimagen de elección para el diagnóstico y seguimiento de los tumores del sistema nervioso es la resonancia magnética (RM). La tomografía computarizada (TC) presenta limitaciones importantes en esta patología, como son su peor resolución anatómica y la presencia de artefactos de fosa posterior. A pesar de ello, la TC constituye una técnica muy útil para la evaluación de complicaciones hemorrágicas agudas, efecto masa y extensión del edema vasogénico asociado al tumor. Es importante recordar que la TC craneal puede aportar falsos negativos en el diagnóstico inicial de un tumor cerebral, en situaciones como tumores pequeños o con poca densidad celular, masa tumoral isodensa, ausencia de cambios patológicos significativos en la barrera hematoencefálica, o estabilización de la barrera hematoencefálica por uso previo de corticoides. Existe un conjunto de características radiológicas que pueden orientar hacia el grado de malignidad en la TC, como son: número, forma y tamaño de las lesiones, localización, márgenes del tumor, presencia de quistes, calcificaciones o hemorragias, presencia de necrosis intratumoral, edema perilesional, desplazamiento de estructuras de la línea media y captación o no de contraste la lesión. En la TC la imagen de un tumor puede ser hipo, iso o hiperdensa. Diagnóstico anatomopatológico y técnicas de biología molecular: Una vez establecida la sospecha clínica y radiológica, el diagnóstico de certeza de una neoplasia cerebral requiere el estudio directo del tejido tumoral obtenido mediante biopsia o resección quirúrgica. El objetivo es confirmar el diagnóstico de un tumor y establecer su naturaleza, también dar información pronóstica e incluso predictiva de respuesta al tratamiento, como en el caso de la codelección 1p 19q en oligodendrogliomas, o la metilación del promotor de la enzima reparadora de ADN O-6-metil-guanina-ADN-metil-transferasa (MGMT) en glioblastomas. Tratamiento: La cirugía suele ser el primer paso en el tratamiento de los tumores primarios del sistema nervioso. En toda cirugía debe plantearse cuál es el beneficio esperable, que deberá compararse con el riesgo existente para poder indicar la intervención. El beneficio esperable puede incluirse en alguno de estos aspectos: supervivencia, calidad de vida, diagnóstico y obtención de material para investigación. En determinados tumores (meningiomas, neurinomas, gliomas grado I de la OMS) la cirugía puede ser curativa por sí misma si se extirpa totalmente la lesión. En gliomas de grados II, III y IV, la evidencia acerca del beneficio de la cirugía sobre la supervivencia es limitada, pero hoy en día se acepta que los pacientes con una extirpación amplia tienen mayor supervivencia. El efecto de la cirugía sobre la calidad de vida puede ser positivo o negativo. El principal perjuicio ocurre si se produce un déficit neurológico; por ello el objetivo debe ser siempre conseguir la máxima extirpación sin provocar déficit. Los índices de morbilidad actuales se han estimado en torno al 13%, mientras que se considera que la mortalidad en craneotomías por tumor debe ser inferior al 2%. Una cirugía sin complicaciones produce un efecto beneficioso en la calidad de vida, al permitir la desaparición o rápida mejoría de los síntomas y la rápida retirada de corticoides. La radioterapia (RT) es un tratamiento útil y eficaz en múltiples neoplasias primarias y secundarias del sistema nervioso central. Se basa en la inducción de

CURSO ENARM CMN SIGLO XXI TEL: 36246001

Pharmed Solutions Institute

PÁGINA 148

MANUAL DE TRABAJO DEL CURSO ENARM CMN SIGLO XXI daño en el ADN y orgánulos de las células malignas, con lo que consigue producir apoptosis y reducción de la masa tumoral. Sin embargo, es necesario reducir al máximo el área de parénquima sano tratado, pues causa también desmielinización, daño neuronal y cambios vasculares en el tejido normal. La dosis habitual de los gliomas de alto grado es 60 Gy administrados en fracciones de 1,8-2 Gy/día aplicados al tumor y un área peritumoral de 2 cm. Los gliomas de bajo grado pueden tratarse con dosis más bajas. Las nuevas técnicas como RT conformada 3D e IMRT consiguen ajustar mejor el área a tratar y la radiocirugía permite tratar de forma muy precisa volúmenes pequeños con mínima afectación del parénquima circundante. Sin embargo, la radiocirugía tiene un papel muy limitado en el tratamiento de los tumores primarios, siendo su principal indicación las metástasis cerebrales. Quimioterapia y Agentes Biológicos: se basa en tres pilares fundamentales, que son la resección quirúrgica, la radioterapia y la quimioterapia. El papel de esta última ha cambiado en los últimos años, al pasar de su uso limitado en determinados tumores (meduloblastomas, tumores germinales) a formar parte la temozolomida del tratamiento estándar de primera línea del glioblastoma, el tumor primario cerebral más frecuente. Adicionalmente, cada vez más estudios muestran que temozolomida es activa en otras neoplasias como los gliomas de bajo grado y los oligodendrogliomas. Además de la temozolomida, otros fármacos quimioterápicos usados en el tratamiento de los tumores cerebrales primarios incluyen, entre otros, las nitrosoureas (BCNU, CCNU), procarbazina, vincristina, derivados del platino, metrotrexate y citarabina. El descubrimiento de las alteraciones existentes en las vías moleculares que regulan la multiplicación celular, el crecimiento celular, la invasividad de tejidos y la angiogénesis está permitiendo diseñar fármacos que bloquean o revierten dichas alteraciones. Este tipo de fármacos, creados específicamente para inhibir vías moleculares concretas, son los llamados “targeted therapies” o “tratamientos anti-diana”. El primero de ellos que ha llegado a la práctica clínica es el anticuerpo monoclonal anti-VEGF bevacizumab (Avastin®) en gliomas malignos recurrentes, pero existen muchos otros que están siendo evaluados en ensayos clínicos con resultados prometedores. Tratamiento sintomático: Las crisis comiciales pueden aparecer hasta en el 40% de los casos en el momento del debut de la enfermedad, y hasta en el 60% a lo largo de su evolución. Todo paciente diagnosticado de un tumor cerebral que ha sufrido alguna crisis comicial debe recibir tratamiento antiepiléptico. Su uso de forma profiláctica es sin embargo controvertido, y la recomendación actual de la American Academy of Neurology (AAN) es no iniciar tratamiento antiepiléptico en pacientes con tumores cerebrales que no hayan presentado nunca crisis. En caso de ser necesario tratamiento antiepiléptico, deben seleccionarse fármacos con bajo potencial de interacciones (no inductores) y con un buen perfil de efectos secundarios. El edema peritumoral contribuye significativamente en el deterioro clínico de los pacientes con tumores primarios y metástasis cerebrales. El tratamiento fundamental del edema peritumoral son los corticoides, siendo el más utilizado en este contexto la dexametasona, por su buena difusión hacia el parénquima cerebral y su mínima actividad mineralocorticoide. Su uso debe estar guiado por la clínica del paciente, manteniendo siempre la menor dosis posible para el control de los síntomas e iniciando su retirada progresiva posteriormente para evitar los efectos secundarios del tratamiento prolongado con corticoides. La trombosis venosa profunda y el tromboembolismo pulmonar ocurren hasta en el 30% de los pacientes fuera del periodo periquirúrgico. Entre los factores que los favorecen se encuentran la disminución de la movilidad de los miembros paréticos, decúbito prolongado, uso de quimioterapia, liberación de factores procoagulantes por parte del tumor y el uso de tratamientos antiangiogénicos. Se recomienda el uso de medias de compresión elástica o de mecanismos de compresión mecánica secuencial a todos los pacientes tras resección quirúrgica. Las heparinas de bajo peso molecular son el tratamiento del evento tromboembólico, al haberse comprobado que el riesgo de complicaciones hemorrágicas es bajo. La fatiga, las dificultades cognitivas y la depresión y ansiedad son también síntomas frecuentes que pueden tener un gran impacto en la calidad de vida del paciente. Su origen es a menudo multifactorial. El tratamiento incluye la corrección de factores precipitantes si es posible y el uso de farmacoterapia específica (psicoestimulantes, antidepresivos, ansiolíticos). Tumores Cerebrales Primarios: Tumores Neuroepiteliales, Tumores Neuroepiteliales: Neuronales y Neurogliales mixtos, Tumores Neuroepiteliales: Tumores no gliales. Tumores Meníngeos: Tumores de Células Meningoteliales, Tumores Meníngeos: Tumores Mesenquimales, Tumores Meníngeos: Lesiones Melanocíticas Primarias, Tumores de Células Germinales, Tumores de la Región Selar, Linfoma Primario del SNC. Tumores de los Nervios Craneales y Espinales, Tumores Medulares Primarios. CASO CLINICO Caso clínico. Paciente de 19 años, masculino, consulta por crisis de cefalea intermitente desde hace 5 meses, que aumentaron en frecuencia e intensidad agregándose vómitos explosivos 3 días previo a la consulta. Al examen neurológico destacó Glasgow 15, dismetría de extremidad superior derecha y leve paresia facial izquierda. Se le realizó una tomografía computada de encéfalo que mostró tumor de tipo astrocitoma e hidrocefalia. Se realizó resección del tumor y la biopsia definitiva confirmó el diagnóstico preoperatorio y determinó bordes libres de tumor. PREGUNTA Cual es la sobrevida a 10 año del caso clínico? RESPUESTA a.- Más del 90% b.- Más del 80 % c.- Más del 70% d.- Más del 60 %. CASO CLINICO Femenino de 60 años de edad quien tiene el antecedente de tiroidectomía total, cinco meses previos a su ingreso por nódulo tiroideo reportado por histología como hiperplasia multinodular multifocal. Su padecimiento tenía un año de evolución con cefalea holocraneana persistente, hemiparesia corporal izquierda, así como parestesias mismo lado; presentaba además, cacosmia y déficit campimétrico visual ipsilateral. La exploración neurológica demostró funciones mentales superiores conservadas. Nervios craneales: olfación conservada, fondo de ojo sin papiledema, hemianopsia homónima izquierda. Extremidades: hemiparesia corporal izquierda 4/5, reflejos exaltados y Babinski ipsilateral, marcha parética, no se encontraron alteraciones sensitivas del orden interoceptivo, propioceptivo o exteroceptivo; sin afección meníngea o cerebelosa.

CURSO ENARM CMN SIGLO XXI TEL: 36246001

Pharmed Solutions Institute

PÁGINA 149

MANUAL DE TRABAJO DEL CURSO ENARM CMN SIGLO XXI PREGUNTA Los glioblastomas multiformes (OMS grado IV) son las neoplasias más frecuentes y malignas del sistema nervioso central, una variante histológica de éstos, denominada, de células gigantes representa aproximadamente? RESPUESTA a.- 1% de todos los tumores cerebrales. b.- 2% de todos los tumores cerebrales. c.- 3% de todos los tumores cerebrales. d.- 4% de todos los tumores cerebrales. CASO CLINICO Un hombre de 56 años de edad que sufre de dolor bilateral de las piernas durante 2 meses, lo que empeora sobre todo por las noches. No había antecedentes de la incontinencia urinaria. En la exploración neurológica fue normal. Los estudios de neuroimagen como la resonancia magnética (MRI) de la columna lumbar revelaron una masa extramedular intratecal de 23x23x13 mm, en L1-2 Nivel de disco intervertebral. La masa era isointensa en imágenes ponderadas en T1 e hiperintensa en las imágenes potenciadas en T2 con componentes quísticos. PREGUNTA Cual es el porcentaje que presentan los oligodendrogliomas de todolo los tumores de la meula espilana de origen primario? RESPUESTA a.- 1% b.- 1.5% c.- 2% d.- 2.5% CASO CLINICO Femenino de 59 años con antecedente de dificultad para la marcha de mas de 5 años de evolución, acompañada de dolor tipo radicular de miembros inferiores, el dolor era intenso tanto en la region sacrolumbar como en sus extremidades inferiores, que se incrementaban con las maniobras de valsava. Desde el inicio de los síntomas requirió de andarea para demabular, limitando la marcha a pocos metros y presentándose un cuadro de urgencia esfentiriana urinaria y fecal desde hacia 15 meses. Referia dos episodios de empeoramiento, los cuales fueron tratados con AINES, corticoides e inmunoglobulina sistémica, con minima mejoría. PREGUNTA Cual es el diagnostico diferencia mas frecuente en este caso? RESPUESTA a.- Esclerosis multiple. b.- Esclerosis lateral amniotrofica. c.- Mielitis transversa. d.- Sindrome raquimedular traumatico. CASO CLINICO Varón de 17 años y 10 meses, que acudió a su médico por cefalea, vómitos y desorientación. Murió 2 años después tras extensión medular y metástasis ósea. TC craneal con contraste i.v. en el que se observa una masa hipercaptante del vermix que invade el 4º ventrículo con pequeños focos quísticos/necróticos. RM axial en secuencia T2 y sagital en T1 con contraste, en el que se observa una masa muy hiperintensa con edema perilesional en T2, que se tiñe de manera parcheada, comprime el 4º ventrículo generando intensa hidrocefalia obstructiva. PREGUNTA Cual de las siguientes aseveraciones respuesto a los meduloblastomas es menos probable?. RESPUESTA a.- La localización del meduloblastoma desmoplásico presenta más frecuentemente una localización fuera de la línea media (50%). b.- La hemorragia es un hallazgo de imagen frecuente en los meduloblastomas, pero se presenta aproximadamente en el 10% c.- Se ha descrito que el único subtipo que presentaba ligeras diferencias en su apariencia con RM era el desmoplásico, que tiende a presentar edema marcado (75%). d.- Se observan signos radiológicos clásico de hidrocefalia obstructiva. CASO CLINICO Niña de 16 años que acude por cefalea, vómitos, diplopia y síndrome cerebeloso. TC con contraste en el que se objetiva una masa hiperdensa de 4 cms en el vermix cerebeloso, con realce heterogéneo y focos quísticos de necrosis. Masa que comprime el tronco, hiperintensa en T2 axial e hipointensa en T1 coronal con contraste, de bordes bien definidos, con realce parcheado, y que genera dilatación ventricular.

CURSO ENARM CMN SIGLO XXI TEL: 36246001

Pharmed Solutions Institute

PÁGINA 150

MANUAL DE TRABAJO DEL CURSO ENARM CMN SIGLO XXI PREGUNTA Cual de las siguientes aseveraciones respuecto a los meduloblastomas es menos probable?. RESPUESTA a.- La apariencia clásica en TC es una masa vermiana hiperdensa, bien delimitada, con edema vasogénico circundante ligero o moderado. b.- Presencia de hidrocefalia y realce homogéneo con contraste. c.- La presencia de calcificaciones y áreas necrótico/quísticas son comunes. d.- La hemorragia es el signo mas frecuente. PREGUNTA Cual de las siguientes aseveraciones respuecto a los meduloblastomas es menos probable?. RESPUESTA a.- La apariencia típica en RM es hipointensa en secuencias potenciadas en T1. b.- La aparencia típica en RM Isointensa en secuencias potenciadas en T2. c.- Respecto a la substancia gris cerebral, observándose mayores grados de heterogeneidad en la RM que en TC. d.- Presentan captación intensa del gadolinio de forma principalmente uniforme. CASO CLINICO Un hombre de 61 años consulta por un cuadro progresivo de 3 semanas de evolución de debilidad de extremidades inferiores e inestabilidad de la marcha. No refiere haber presentado fiebre ni perdida de peso, ni historia previa de incontinencia o disfunción vesical o anal, convulsiones o trauma. Sin embargo, tiene antecedentes de abuso de cocaína e hipertensión, tratada con lisinopril. Al examen físico, presenta signos vitales normales. Cuando se realiza el examen neurológico, se evidencia debilidad de ambas extremidades inferiores, hiperreflexia, y debilidad muscular bilateral (puede hacer movimientos en contra de la fuerza de gravedad y ante una leve resistencia). Presenta marcha atáxica. Se continuó el estudio con una RMN de la médula espinal, que, en fase T1, muestra una lesión a nivel de de T8, extradural y que comprime la médula. La RMN de cerebro muestra, en fase T1, múltiples lesiones homogéneas. Al realizar una excisión de la lesión medular, el paciente no vuelve a presentar debilidad muscular en las extremidades inferiores. PREGUNTA Los meningiomas son los tumores benignos del sistema central más comunes, totalizando entre un 15 a 20% de todos los tumores primarios cerebrales. Cual es la edad mas frecuente de presentación? RESPUESTA a.- 10 a 20 años. b.- 20 a 40 años. c.- 40 a 60 años. d.- 60 a 80 años. CASO CLINICO Se trata de una paciente mujer de 58 años que consultó por una cervicalgia crónica. La paciente presentó desde dos meses antes de la consulta episodios de parestesias de miembros superiores que se exacerbaban con las maniobras de valsalva. El examen físico de la paciente fue normal. Se le realizó resonancia magnética de columna cervical, donde se observó una gran lesión quística que se extendía desde C2 hasta C3. Con el contraste se observó una captación de un nódulo en la superficie medular que estaba en estrecha relación con el quiste intramedular. La paciente fue llevada a cirugía, donde se le realizó un abordaje por vía posterior. Se le realizó laminectomía C2 y C3, y al abrir la dura se encontró un nódulo tumoral vascular en la pared medular. Con técnicas microquirúrgicas se hizo resección de la lesión espinal medular, y al retirar la lesión se produjo drenaje del quiste. En el postoperatorio la paciente presentó mejora de la sintomatología. El estudio de patología reportó un hemangioblastoma espinal. PREGUNTA Los tumores intramedulares de la columna espinal son raros, cuales su frecuencia dentro de los tumores del sistema nervioso central? RESPUESTA a.- 1 a 2 %. b.- 2 a 8 %. c.- 8 a 10 %. d.- 10 a 12 %. PREGUNTA Se pueden encontrar a lo largo de toda la médula espinal, cual de los siguientes es el mas frecuente? RESPUESTA a.- Ependimomas. b.- Astrocitomas c.- Hemangioblastomas

CURSO ENARM CMN SIGLO XXI TEL: 36246001

Pharmed Solutions Institute

PÁGINA 151

MANUAL DE TRABAJO DEL CURSO ENARM CMN SIGLO XXI d.- Menigiomas. CASO CLINICO Varón de 16 años con historia de cefalea, episodios de pérdida transitoria del conocimiento de dos años de evolución asociadas a pérdida bilateral de la visión desde 4 meses. Como antecedente familiar informa hermano mayor fallecido a los 19 años con enfermedad metastásica por Osteosarcoma de fémur. El TAC demuestra tumoración expansiva en ubicación supraselar, que se refuerza con el medio de contraste. Le realizan craneotomía para resección biopsia de la tumoración con remisión de muestra a patología con el diagnóstico de glioma del quiasma. PREGUNTA Cual es la localización más predominante de esta patologia? RESPUESTA a.- Pineal. b.- Supraselar. c.- Tálamo. d.- Tercer ventrículo. CASO CLINICO Varón de 19 años, con historia de 4 años de hipoacusia que evoluciona a pérdida de la audición del oído derecho. Dos años después se acompaña de cefalea y pérdida de la visión iniciada por el ojo izquierdo que progresa a ceguera total desde 4 meses. Como antecedente familiar presenta abuela materna fallecida con enfermedad metastásica por carcinoma de cuello uterino. El TAC demostró presencia de tumoración en región pineal y tercer ventrículo con importante hidrocefalia. Le realizaron craneotomía con colocación de válvula de derivación revertiendo parcialmente la sintomatología. Dos meses después recurren las manifestaciones clínicas, el estudio de RM demuestra tumoración expansiva e irregular que compromete regiones selar y pineal, piso del tercer ventrículo y núcleos básales PREGUNTA Cual es la tasas de curación mas aproximada de este padecimiento? RESPUESTA a.- 60 %. b.- 70 %. c.- 80 % d.- 90 % CASO CLINICO Una mujer de 52 años consulta por pigmentación de las uñas de un año de evolución. No presentaba síntomas sistémicos y era inmunocompetente. Se le había diagnosticado hace 12 años un macro-adenoma pituitario productor de ACTH, que había sido tratado con neurocirugía endoscópica transnasal. Sin embargo, la cirugía no produjo la cura, por lo que se realizó una adrenalectomía bilateral, y la paciente recibía mineralo-corticoides y glucocorticoides. Al exámen físico, algunas de las uñas de los dedos y de los pies presentaban una banda de melanoniquia longitudinal, mientras que otras manifestaban pigmentación gris negruzca difusa. No se observaron cambios de color en piel ni mucosas. L os estudios de laboratorio mostraron que los niveles plasmáticos de ACTH eran altos, de 100 pg/mL (normal 10-50 pg/mL). Las imágenes de resonancia magnética (MRI) confirmaron vestigios de adenoma pituitario. PREGUNTA Cual de las siguientes afirmaciones relacionadas a la melanoquia es menos probable? RESPUESTA a.- Las drogas más comunes que causan melanoniquia son los agentes quimioterápicos. b.- Las enfermedades endócrinas que causan incremento de los niveles circulantes de ACTH. c.- Los melanomas son la principal causa de melanoquia. d.- Frecuentemente los pinealomas no presentan melanoquia. CASO CLINICO Paciente de 35 años que presenta desde hace 10 años amenorrea, esterilidad, y galactorrea. A veces alteraciones visuales. Sensibilidad en las mamas. Disminución del interés sexual, dolor de cabeza, infertilidad. Se le hizo un TAC y se halló un prolactinoma, al tratarla con dopamina, (carbegolina) el microadenoma remitió. PREGUNTA Cual es el porcentaje de cambio de microadenomas a macroadenomas productores de prolactina? RESPUESTA a.- 2 %. b.- 3 %. c.- 4 %. d.- 5 %.

CURSO ENARM CMN SIGLO XXI TEL: 36246001

Pharmed Solutions Institute

PÁGINA 152

MANUAL DE TRABAJO DEL CURSO ENARM CMN SIGLO XXI CASO CLINICO Preescolar masculino de 2 años y 11 meses de edad hospitalizado como desnutrido severo marasmático. Al momento de ingreso la madre refería un padecimiento de dos meses de evolución caracterizado por vómitos post-prandiales persistentes, hiporexia, disminución subjetiva y progresiva de peso además de compromiso del estado general. Es producto de primer embarazo, nacido de término por parto institucional sin complicaciones de una madre de 24 años sin antecedentes patológicos de importancia, con llanto inmediato al nacer y desarrollo psicomotor adecuado hasta el año de edad, a partir del cual se percibió falta de desarrollo del lenguaje receptivo y expresivo además de retraso del desarrollo motor caracterizado por imposibilidad de ponerse de pie y caminar sin apoyo. Al examen físico de ingreso se encontraba en mal estado general y nutricional. El indicador peso para la talla era menor a -3 desvíos estándar (DE), existía emaciación visible y palidez mucocutánea generalizada. Neurológicamente llamaba la atención una respuesta verbal inapropiada, imposibilidad de bipedestación y aparente afectación visual manifestada por imposibilidad del paciente de seguir objetos con la mirada. Su perímetro cefálico fue de 48 cm. (adecuado para la edad) y las pupilas eran isocóricas reactivas al estímulo luminoso. El informe histopatológico confirmó el diagnóstico de craneofaringioma. PREGUNTA Con respecto al diagnostico, cual de las siguientes aseveraciones en menos probable? RESPUESTA a.- Tiene un leve predominio en el sexo femenino. b.- Es un tumor de origen epitelial de la región sellar que se forma a partir de los restos embrionarios de la bolsa de Rathke. c.- El tumor puede limitarse a la silla turca, o bien puede extenderse a través del diafragma sellar y comprimir la vía óptica, la protuberancia o el tercer ventrículo, produciendo hidrocefalia. d.- Los craneofaringiomas representan entre el 5 y el 10% de todos los tumores intracraneales que se presentan durante la niñez. CASO CLINICO Mujer de 51 años que padecía desde hacía cuatro meses episodios de repetición de amaurosis fugax en su ojo izquierdo. Mediante tomografía axial computerizada se diagnosticó una masa orbitaria circunscrita intraconal izquierda sospechosa de hemangioma cavernoso. Se le practicó una orbitotomía lateral izquierda resecándose una masa rojiza encapsulada, sugestiva de hemangioma cavernoso. Microscópicamente estaba compuesta por áreas de células fusiformes dispuestas en empalizada (Antoni tipo A), entremezcladas con áreas quísticas donde las células estaban rodeadas por una matriz mixoide (Antoni tipo B). PREGUNTA Cual es la edad de presentación mas frecuente de este padecimiento. RESPUESTA a.- entre 20 y 30 años. b.- entre 20 y 40 años. c.- entre 20 y 50 años. d.- entre 20 y 60 años. INFECCIONES DEL SISTEMA NERVIOSO CENTRAL. Introducción: constituyen una emergencia médica, pues su alta morbilidad y mortalidad requieren un diagnóstico y tratamiento oportuno. Muchos factores se involucran en la severidad de las infecciones del SNC. Su ubicación anatómica en un espacio óseo sellado que no permite una expansión fácil ante un proceso inflamatorio difuso contribuye a que las altas posibilidades de daño neurológico ocurran por efecto mecánico, como en los síndromes de herniación. Otros factores como la competencia inmunológica de cada individuo, la penetración y concentración de los agentes antimicrobianos en el sistema nervioso, la edad y las dificultades diagnósticas contribuyen en la evolución de los pacientes con infecciones del sistema nervioso. La edad de aparición más frecuente de meningitis desde la infancia hasta la segunda y tercera década. Por otro lado ha aumentado la proporción de infecciones nosocomiales. Evaluación de laboratorio. El líquido cefalorraquideo (LCR) es el pilar fundamental en el diagnóstico de la mayoría de las infecciones del SNC. Las características iniciales del análisis citoquímico pueden orientar a un diagnóstico específico aunque se requiere la confirmación de éste por otros medios. Sin embargo una buena correlación clínica inicial es suficiente para tomar una conducta terapéutica. En general las características típicas de los diferentes tipos de infección meníngea de acuerdo con el citoquímico se muestran en la Tabla. Los cultivos y el Gram del LCR continúan siendo los exámenes de elección en las meningitis bacterianas. La administración de antibióticos intravenosos por dos a tres días antes de la punción lumbar no altera el conteo de células o las concentraciones de proteínas y glucosa en los casos de meningitis bacterianas, pero si reducirán la probabilidad de obtener un cultivo y el Gram del LCR positivos. La administración de antibióticos orales antes de la punción lumbar no altera ninguno de los parámetros en el LCR pero sí puede disminuir el porcentaje de neutrófilos y la probabilidad de resultados positivos en el cultivo o el Gram. El Gram tiene una sensibilidad cercana a 60-90% y una especificidad de 100%. Los cultivos se pueden cultivar en platos de agar con adición de nutrientes o en caldos. Los

CURSO ENARM CMN SIGLO XXI TEL: 36246001

Pharmed Solutions Institute

PÁGINA 153

MANUAL DE TRABAJO DEL CURSO ENARM CMN SIGLO XXI cultivos en caldos son más susceptibles de ser contaminados con Stophylococcus epidermidis mientras que son mas útiles en caso de infecciones de los equipos de derivación ventricular. Los hemocultivos identifican el germen causal en 80% de los casos de neumococo, 90% de los de meningococo y 94% en los casos de Haemophylus influenzal. Los casos de meningitis tuberculosa (TBC) o meningitis micóticas son difíciles de diagnosticar por cultivo o en el extendido. Son positivos entre 52 a 83% de los casos de meningitis TBC y en 75% de los casos de criptococosis meníngea. La sensibilidad y especificidad de los cultivos en estos dos gérmenes se aumenta hasta en 87% con cultivos repetidos y con grandes volúmenes (hasta 25 ml) de LCR. Las pruebas de látex para identificación de antígenos de los gérmenes son rápidas requiriendo entre 10 a 15 minutos para realizarlas y sin un entrenamiento riguroso. Además tienen la ventaja de tener pocos falsos positivos. Por ejemplo, para criptococo puede haber falsos positivos ante la presencia de factor reumatoideo. En el caso de meningitis por Histoplasma capsulatum, las pruebas son muy sensibles pero la especificidad es baja por tener reactividad cruzada con criptococo, candida y Coccidiodes inmitis. La identificación de anticuerpos específicos en el LCR es de gran utilidad. En la neurosífilis un diagnóstico definitivo se obtiene con la demostración de positividad del VDRL en el LCR. Una prueba reactiva en cualquier título es diagnóstica de neurosífilis. Aunque la prueba es muy específica, la sensibilidad sin embargo varía entre 30-70% y en muchos casos el diagnóstico se basa en la presencia de pleocitosis o proteínas elevadas en el LCR en un paciente con VDRL reactiva en sangre. En los pacientes con VIH las dificultades nacen debido a que puede haber pleocitosis con aumento de proteínas en el LCR debido a la neurosífilis o por la misma infección por el VIH. En estos casos tiene mucha utilidad el FTA-ABS (Treponemal antibodyabsortion test) y el test de hemoaglutinacion para Treponema pallidum (MHA-TP) los cuales son muy sensibles para el diagnóstico de neurosífilis. Un resultado no reactivo descarta el diagnóstico de neurosífilis en los pacientes con infección por VIH. La PCR es la prueba más útil para el diagnóstico de meningitis y encefalitis viral. Además tiene la ventaja de cuantificar el ácido nucleico en las muestras de LCR con lo cual se puede determinar la progresión de la enfermedad y la respuesta al tratamiento. La prueba se fundamenta en realizar mediante un sistema térmico cíclico una copia y amplificación de hasta un millón de veces del contenido de ADN del germen presente en el LCR. En los casos de meningitis TBC los resultados de sensibilidad para la PCR en diferentes estudios han sido variables (entre 54100%) y la especificidad entre 89-100%. Sin embargo hay evidencia consistente de que la PCR es más sensible que el examen microscópico y el cultivo para TBC. La adenosin deaminasa (ADA) es una enzima que está asociada con enfermedades que producen una respuesta inmunológica celular; es de mucha ayuda para el diagnóstico de meningitis TBC. Sin embargo, pueden observarse resultados positivos de esta prueba en linfomas con compromiso meníngeo, sarcoidosis, neurobrucelosis y hemorragia subaracnoidea. También pueden haber falsos negativos. Los diferentes estudios muestran una sensibilidad y especificidad para el diagnóstico de 90% con títulos mayores de 10 unidades internacionales por litro (UI/L) siendo los títulos de corte entre 5-10 UI/L. Los resultados de la ADA pueden elevarse en las dos primeras semanas de tratamiento. Meningitis: Se define meningitis como la presencia de inflamación meníngea originada por la reacción inmunológica del huésped ante la presencia de un germen patógeno en el espacio subaracnoideo. La meningitis es el síndrome infeccioso más importante del sistema nervioso central. El compromiso parenquimatoso adyacente a las meninges definirá la presencia de meningoencefalitis (encéfalo), meningoencefalomielitis (encéfalo y médula), meningomielorradiculitis (encéfalo, médula y raíces nerviosas). Los diferentes tipos de meningitis tienen diferente origen y los patógenos responsables en la mayoría de los casos son predecibles, lo cual permite en muchas situaciones clínicas el inicio de una terapia empírica mientras se obtiene la confirmación del germen involucrado. La identificación de gérmenes que pueden tener influencia de tipo endémico es de importancia para las medidas profilácticas y epidemiológicas en determinadas poblaciones (por ej: meningococo). Los diferentes tipos de meningitis se pueden definir de acuerdo con el perfil clínico, hallazgos de LCR y la etiología. Se definen como meningitis aséptica aquellos casos de meningitis en los cuales luego de un estudio completo no se identifica una bacteria y los cultivos y las pruebas inmunológicas deben ser negativos. Es de origen viral por lo general. La meningitis séptica es causada por una bacteria. Tiene con la meningitis aséptica un perfil diferente del LCR; su tratamiento y pronóstico son completamente distintos a la anterior. La meningitis crónica se define arbitrariamente como aquella con una evolución mayor de cuatro semanas luego de haberse documentado clínicamente que el paciente no se encuentra en una fase de recuperación de una meningitis aguda. La meningitis recurrente se refiere a aquellos casos que clínicamente presenta dos o más episodios de meningitis aguda. El paciente debe haber evolucionado a la mejoría total tanto clínica como en el LCR entre cada uno de los episodios agudos. Meningitis aséptica: Por lo general es de curso benigno y en la mayoría de los casos de etiología viral. Puede haber causas no infecciosas de meningitis aséptica. Tiene un curso clínico bifásico. Primero preceden a su aparición los síntomas o signos de una afección viral respiratoria, gastrointestinal o en la piel, presentándose posteriormente la fase meníngea con síntomas más específicos como cefalea, fiebre y signos meníngeos. Son poco frecuentes los síntomas de compromiso encefálico como convulsiones o alteración del estado mental. En algunos países tiene predilección estacional y es más frecuente en niños. Los enterovirus son los principales agentes causales (entre 55-75%) de los casos de meningitis aséptica y hasta en 95% de los casos cuando se identifica el patógeno. El virus de la parotiditis fue considerado en alguna ocasión responsable de muchos casos de meningitis aséptica pero la incidencia ha declinado desde el uso de la vacuna; sin embargo una meningitis oculta se presenta en más de la mitad de los casos de parotiditis y llega a ser sintomática en solo 30%. Mas aún, puede ocurrir la meningitis sin parotiditis. Los herpes virus raramente causan meningitis; sin embargo el virus del herpes simple (VHS) es el responsable de 1 a 3% de todos los casos de meningitis aséptica. De los dos tipos de herpes simple (VHS 1 oral - VHS 2 genital) el VHS 2 produce meningitis en 11% a 33 % de las personas en el momento de la infección genital primaria. El VIH puede causar una meningitis aséptica principalmente en el período de la infección primaria y durante la seroconversión pero incluso puede producirla muy temprano en las fases iniciales de la infección por VIH. Raramente la tuberculosis se puede comportar como una meningitis aséptica autolimitada. En los casos de endocarditis e infecciones parameníngeas puede presentarse un cuadro de meningitis aséptica aunque las causas sean gérmenes no vírales. La PCR es de gran utilidad con una especificidad del 100% y el resultado es disponible en pocas horas. El manejo es prácticamente de soporte y de alivio de los síntomas. Meningitis séptica (meningitis bacteriana) implican una mayor morbi-mortalidad y requieren un rápido diagnóstico y tratamiento para evitar una evolución fatal o secuelas irreversibles, por lo tanto es una emergencia neurológica. La tasa general de mortalidad es de 25% y de morbilidad hasta 60%. Presentación clínica: los síntomas clásicos de presentación incluyen cefalea, fiebre, escalofríos, alteración de la esfera mental y la presencia de meningismo. Estos síntomas pueden variar en los neonatos y en los ancianos. En los neonatos predominan los síntomas inespecíficos como irritabilidad, vómito, letargia, dificultad respiratoria y síntomas gastrointestinales, siendo el meningismo poco frecuente. Los ancianos presentan con mayor frecuencia alteración en el estado mental asociado a fiebre; la presencia de cefalea y meningismo resulta exótica. La meningitis por meningococo tiene frecuentemente una presentación dramática y puede progresar como una enfermedad fulminante muriendo el

CURSO ENARM CMN SIGLO XXI TEL: 36246001

Pharmed Solutions Institute

PÁGINA 154

MANUAL DE TRABAJO DEL CURSO ENARM CMN SIGLO XXI paciente en pocas horas. Muchas veces los pacientes tienen un rash cutáneo petequial en el tronco y las extremidades inferiores. Este puede, por contigüidad de las lesiones, formar áreas extensas de equimosis. La púrpura fulminante es una forma grave de manifestación de la sepsis por meningococo; es causada por coagulación intravascular diseminada y en muchas ocasiones causa necrosis distal en las extremidades. Los marcadores de riesgo incrementado de muerte en los pacientes con meningitis por meningococo son: la presencia de diátesis hemorrágica, signos neurológicos focales y personas mayores de 60 años. En general muchos síndromes neurológicos diferentes al meningismo pueden ser la forma de presentación de las meningitis bacterianas. La presencia de signos neurológicos focales por compromiso cerebrovascular, hipertensión endocraneana, alteración en pares craneales, crisis convulsivas y estado confusional son algunos de ellos. Tratamiento: el tiempo del inicio del tratamiento influye significativamente en el pronóstico. El LCR es el principal soporte diagnóstico en la escogencia de la terapia adecuada, pero en muchas ocasiones la punción lumbar debe diferirse en aquellos casos de pacientes en estupor o coma, con signos neurológicos focales y ante la presencia de crisis convulsivas. En estos casos las neuroimágenes están indicadas inicialmente para descartar complicaciones asociadas con la meningitis o hacer un diagnóstico diferencial. Una escanografia de cráneo (simple) normal permite una mayor seguridad en la realización de la punción lumbar en estos casos y evitar así complicaciones. Cabe anotar que ante la sospecha diagnóstica debe iniciarse una terapia empírica mientras se realiza la punción lumbar. La terapia antibiótica no variará significativamente el LCR en las primeras 48 horas. La terapia empírica se escogerá de acuerdo con los grupos de edad, la sensibilidad antibiótica de los gérmenes involucrados, su sensibilidad antibiótica y el estado inmune de los pacientes. El Gram puede ser una guía inicial de utilidad pero no es tan eficaz en todos los casos; las pruebas inmunológicas son de gran ayuda para un diagnóstico específico y rápido del germen. La terapia empírica más conveniente para iniciar el tratamiento. Luego, al obtener los resultados del cultivo con las pruebas de sensibilidad antibiótica se decidirá el tratamiento definitivo. El Streptococcus pneumoniae (neumococo) hoy por hoy es el principal germen patógeno en la meningitis bacteriana en cualquier grupo de edad. Penicilina ó cefalosporina con vancomicina. El Haemophylus influenzae es un coco bacilo gramnegativo. El único huésped natural de este germen es el humano y la transmisión persona-persona ocurre por la vía respiratoria. El trauma de cráneo reciente, la cirugía neurológica previa, las sinusitis paranasales, las otitis media y las fístulas del LCR son los factores de riesgo más importantes en adultos para padecer una meningitis por H. influenzae. El tratamiento antibiótico es ceftriaxona ó cefotaxima, por lo tanto son los fármacos de elección. La Listeria monocitogenes es un germen beta-hemolítico facultativamente anaerobio y grampositivo. Los grupos de edad más frecuentemente involucrados en meningitis por L. monocitogenes son los neonatos menores de un mes y los adultos mayores de 60 años. En este grupo la listeria es la causa de 20% de las meningitis. Los principales factores de riesgo para meningitis por L. monocitogenes son el embarazo, inmunosupresión y edad avanzada. El tratamiento de elección es la ampicilina o la penicilina por un tiempo de cuatro semanas en los pacientes inmunosuprimidos. En los pacientes alérgicos a las penicilinas el trimetropim-sulfa-metoxazol es la alternativa más recomendada. El estreptococo del Grupo B es la causa más frecuente de sepsis neonatal y es una causa importante de infección bacteriana invasiva en los adultos. Es el germen causal en 70% de los casos de meningitis bacteriana en los neonatos menores de un mes, mientras que en adultos es responsable de menos de 5% de los casos. El estreptococo del grupo B es altamente susceptible a la penicilina y a la ampicilina siendo las drogas de primera elección. Muchos recomiendan el uso concomitante con gentamicina para el tratamiento de la meningitis neonatal basado en evidencia de estudios en animales que demuestran sinergismo en la eficacia antibiótica. La Neisseria meningitidis (meningococo) es una bacteria gramnegativa encapsulada que aparece en pares en el extendido. Generalmente coloniza la nasofaringe de manera asintomática y la transmisión ocurre de persona a persona mediante secreciones respiratorias. Los individuos esplenectomizados y con deficiencias de complemento tienen un alto riesgo de infección por meningococo. Causa 60% de las meningitis bacterianas entre la población de 2 a 18 años de edad y va declinando su frecuencia hasta 5 % en los mayores de 60 años. Es el responsable en 1/3 de los casos de meningitis en menores de 2 años. El fármaco de elección es la penicilina o la ampicilina. La ceftriaxona y el cefotaxima tienen también una excelente respuesta y alcanzan buenas concentraciones en el LCR. En Estados Unidos no se han identificados cepas productoras de beta-lactamasa. Los contactos cercanos en pacientes con meningococemia tienen un riesgo significativamente alto para desarrollar la infección semanas después de la exposición. Meningitis tuberculosa: Es la manifestación más frecuente de la tuberculosis en el sistema nervioso. Puede presentarse en muchos casos en forma aislada sin compromiso extrameníngeo. Las manifestaciones clínicas son diferentes en niños, adultos y en los infectados por VIH. Su epidemiología ha cambiado significativamente aun en países en donde la incidencia había disminuido de manera importante. Este hecho se explica por la epidemia reciente de infección por VIH y por la resistencia que ha desarrollado el bacilo a las terapias convencionales. Manifestaciones clínicas: con fines pronósticos se reconocen tres estadios de la enfermedad. a) Estadio 1: el paciente está consciente y no presenta signos neurológicos focales. b) El estadio dos: el paciente está confuso pero no está en coma, y tiene signos neurológicos focales como hemiparesia o parálisis de pares craneales. c) Estadio tres: el paciente se encuentra en coma o en estupor; tiene compromiso múltiple de pares craneales, hemiplejía o paraplejía. En los niños es más frecuente identificar la fase inicial de infección a nivel respiratorio, mientras que en el adulto en muchos casos no se puede identificar y puede pasar mucho tiempo desde la infección inicial y la aparición del compromiso del sistema nervioso. Es frecuente la presencia de hiponatremia en adultos encontrándose en 45% de los casos y se debe a una secreción inadecuada de hormona antidiurética (SIDHA). En los niños aunque pueden manifestarse también la cefalea, fiebre y meningismo, es común la presencia de hidrocefalia como primera manifestación de la infección por TBC. Diagnóstico: el diagnóstico recae principalmente en el LCR. La pleocitosis a expensas de linfocitos, las proteínas altas y la glucorraquia significativamente baja son los hallazgos más

CURSO ENARM CMN SIGLO XXI TEL: 36246001

Pharmed Solutions Institute

PÁGINA 155

MANUAL DE TRABAJO DEL CURSO ENARM CMN SIGLO XXI frecuentemente encontrados. En algunos casos hay un predominio polimorfonuclear en las fases iniciales de la infección, que también suele observarse cuando se inicia el tratamiento. Este último coincide con el empeoramiento clínico que se observa en algunos pacientes cuando se inicia la terapia y es muy característico de la meningitis TBC. Por lo general hay un viraje posterior al predominio de linfocitos pero en algunas ocasiones puede persistir el predominio de polimorfonucleares denominándose meningitis neutrofílica persistente. Se ha encontrado un mayor contenido de proteínas en el estadio tres de la enfermedad. Como ya se comentó anteriormente el cultivo y los extendidos para BK tienen una mayor probabilidad de positividad cuando se estudian muestras repetidas en altos volúmenes. La ADA y la PCR recientemente han sido de gran ayuda para un diagnóstico más rápido y en los casos de cultivos negativos. Las neuroimágenes brindan apoyo en el diagnóstico. En la RMC se puede observar el compromiso de las meninges basales que realzan al inyectar el gadolinio; los sitios más comprometidos son la fosa interpeduncular, la cisterna ambiens y la región quiasmática. Este realce es más frecuentemente observado en los pacientes VIH positivos. Las neuroimágenes también son de ayuda para el diagnóstico de hidrocefalia, la presencia de vasculitis por la infección y para caracterizar los granulomas. Complicaciones: aún con tratamiento y cuidados adecuados las complicaciones se pueden presentar. Algunas de ellas son de tipo cerebrovascular por compromiso de las arterias intracraneales debido a una panarteritis, producto de la infiltración de las paredes de los vasos por el exudado inflamatorio. Lo anterior produce infartos cerebrales. El territorio carotídeo es el más frecuentemente comprometido. Una complicación metabólica que puede agravar el cuadro clínico es la hiponatremia por SIDHA; ésta debe corregirse rápida y lentamente para evitar el riesgo de mielinolisis póntica. Se han informado casos de siringomielia muchos años después de la infección inicial. Tratamiento: va orientado a eliminar las formas intra y extracelulares del bacilo. Resultan importantes dos factores para evitar la resistencia: el primero, utilizar múltiples fármacos y el segundo la adherencia al tratamiento. Por lo general el inicio del tratamiento es empírico, sin obtener una confirmación por cultivo que puede tardar varias semanas. Lo anterior repercute significativamente en el pronóstico. Se sabe bien que el número de organismos presentes en los casos de meningitis TBC es inferior al de las otras formas de la infección; esta ventaja potencial se ve disminuida por el hecho de la penetración de los fármacos anti-TBC en el SNC. La isoniacida y la pirazinamida son bactericidas y penetran las meninges inflamadas o no inflamadas alcanzando buenas concentraciones bactericidas. Las concentraciones que alcanzan en el LCR la estreptomicina intramuscular, la rifampicina y el etambutol apenas sobrepasan las concentraciones inhibitorias mínimas para la micobacteria; además no penetran las meninges no inflamadas. Mientras que la rifampicina es bactericida, el etambutol y la estreptomicina son tuberculostáticos. Se aceptan dos tipos de régimen: uno corto de seis meses, con cuatro fármacos. En los primeros dos meses: isoniacida 300 mg, rifampicina 600 mg, pirazinamida 1,5 gr al día y estreptomicina 1g IM (500 mg en ancianos o con peso menor de 50 kg). Luego un peróodo de cuatro meses con isoniacida y rifampicina dos veces por semana o diario. En los niños este segundo período debe ser de 10 meses. En los casos de resistencia se debe incluir otro medicamento como etambutol en vez de estreptomicina y seguir luego con dos drogas (isoniacida y rifampicina) por 9 a 18 meses. La respuesta a la terapia debe evaluarse por cultivos y por la evolución clínica de los síntomas. Meningitis por criptococo: En general todas las meningitis por hongos se consideran que ocurren en pacientes inmunosuprimidos. La vía de entrada en la mayoría de los casos es a través de las vías respiratorias superiores, ingresando en forma de levadura en el alvéolo, allí puede producir un proceso inflamatorio que puede ser completamente asintomático o producir síntomas respiratorios leves y el sistema inmune puede terminar allí todo el proceso; si esto no ocurre así, pasan a la circulación sistémica. Un defecto en los sistemas celulares permite que los hongos atraviesen la barrera hematoencefálica. Las infecciones micóticas del sistema nervioso pueden coexistir o no con la infección en otros órganos blanco en el organismo. En el caso del criptococo, el riñón es el órgano mas frecuentemente comprometido. En el SNC el criptococo puede comprometer únicamente las meninges o invadir el parenquima y los vasos sanguíneos produciéndose una forma de arteritis infecciosa. Presentación clínica: la presentación clínica de las meningitis por hongos en general es insidiosa, de instauración subaguda o crónica; se presenta cefalea, fiebre, escalofríos, alteración del estado general y cambios mentales en un paciente por lo general con factores de riesgo para adquirir la infección (SIDA, cáncer, terapia inmunosupresora, trasplante de órganos). Diagnóstico: requiere de sospecha clínica en los pacientes con riesgo y recae principalmente en el aislamiento del criptococo en el LCR. El LCR muestra generalmente un aumento moderado de la presión; es de aspecto claro y hay pleocitosis que puede alcanzar desde 20 hasta 1.000 células por milímetro cúbico. El predominio celular es de linfocitos, la glucosa es frecuentemente baja (alrededor de 30 mg/dl) y las proteínas son elevadas entre 50 a 1.000 mg/dl; la tinta china puede mostrar la presencia del criptococo. La prueba debe hacerse con un control negativo de agua y así estar seguro que la tinta no está infectada por espora. La prueba serológica de látex en el LCR es muy sensible para el diagnóstico; se puede cuantificar y es de gran ayuda en el seguimiento. Además es rápida y de fácil interpretación. Tratamiento: el tratamiento de primera línea para la meningitis por criptococo es la anfotericina B a la dosis de 0,7 mg/kg/día más 5 flucytocina a la dosis de 25 mg/kg cada seis horas por vía oral hasta que los cultivos sean negativos o haya mejoría clínica (entre cuatro a seis semanas). Posteriormente se debe continuar una terapia de mantenimiento con fluconazol por ocho o diez semanas; pero si el paciente es inmunosuprimido (SIDA, trasplantado, cáncer, etc. ) esta terapia de mantenimiento es de por vida. CASO CLINICO Paciente de 30 años, sexo masculino, en tratamiento con prednisona por Lupus Eritematoso Sistémico, que comenzó con un cuadro de aproximadamente dos semanas de evolución caracterizado por compromiso progresivo del estado general, cefalea holocraneana de intensidad progresiva, fiebre y diaforesis vespertina por lo que consultó. Se le realizó punción lumbar, obteniéndose LCR de aspecto opalescente, cuyo estudio citoquímico fue compatible con meningitis bacteriana; el cultivo demostró desarrollo de un bacilo Gram (+) pequeña, cuya identificación microbiológica confirmó Listeria monocytogenes. PREGUNTA Cual es la conducta terapéutica mas apropiada a seguir. RESPUESTA a.- Cefotaxima mas vancomicina. b.- Ampicilina mas gentamicina. c.- Trimetroprima mas sulfametoxazol y ampicilina. d.- Gentamicina mas imipenem.

CURSO ENARM CMN SIGLO XXI TEL: 36246001

Pharmed Solutions Institute

PÁGINA 156

MANUAL DE TRABAJO DEL CURSO ENARM CMN SIGLO XXI CASO CLINICO Paciente de 87 años, sexo masculino, hipertenso, portador de marcapasos definitivo, además de multiinfarto cerebral y bronquitis crónica. Ingresó con historia de tos productiva y fiebre de aproximadamente 1 semana de evolución, agregándose posteriormente desorientación progresiva y franco compromiso de conciencia que motivó hospitalización. Desde su ingreso, muy grave, en sopor, con signos meníngeos esbozados y neumonía confirmada tanto clínica como radiológicamente. Se manejó con antibioticoterapia agresiva y de amplio espectro para neumonía adquirida en la comunidad con sospecha de aspiración; a pesar de lo anterior evoluciona a la falla orgánica múltiple.

PREGUNTA Cual es la conducta terapéutica mas apropiada a seguir. RESPUESTA a.- Cefotaxima mas vancomicina. b.- Ampicilina mas cefotaxima. c.- Ampicilina mas levofloxacina. d.- Gentamicina mas imipenem. CASO CLINICO Un varón de 56 años de edad, con antecedentes de meningitis criptogena y meningitis aséptica que presenta hace 10 dias fiebre, escalofríos, rigidez en el cuello y dolor de cabeza. El EF temperatura 36.9 c, FR de 121 lpm. Parecía incómodo, pero sin rigidez de nuca franca. LCR revelo eritrocitos 34 células/mm3, 154 células/mm3, leucocitos ( 77 % de neutrófilos , 12 % de linfocitos, histiocitos 10 % ), la glucosa de 63 mg / dl , la proteína de 82 mg / dl. TAC de la cabeza revela estable, leve prominencia difusa del sistema ventricular. El paciente es alérgico a la penicilina. PREGUNTA Cual es la conducta terapéutica mas apropiada a seguir. RESPUESTA a.- Cefotaxima mas vancomicina. b.- Meropenen mas aciclovir. c.- Amikacina mas levofloxacina. d.- Tratamiento sintomatico. CASO CLINICO Varón de 51 años, ingresó por cefalea de 4 semanas y somnolencia. La EF con deterioro neurológico con 10 de Glasgow. Se realizó una punción lumbar (PL): 40células/mm3, el 95% mononucleares (MN); glucosa: 4mg/dl, proteínas: 0,68mg/l. En la tinción de tinta china se apreciaron células levaduriformes. El título de antígeno criptocócico en plasma era superior a 1/256 y en líquido cefalorraquídeo (LCR), superior a 1/1.024. No se midió la presión intracraneal. En el cultivo de LCR se aisló C. neoformans, se inició tratamiento con anfotericina B i.v. a dosis de 4mg/kg/día y 5 flucitosina oral a dosis de 100mg/kg/día. El paciente desarrolló sordera bilateral como complicación. A los 12 días reingresó con afectación de VII par craneal izquierdo. PREGUNTA Cual es la conducta terapéutica mas apropiada a seguir. RESPUESTA a.- Anfotericina B iv. b.- Meropenem i.v. a dosis de 1g/8h. c.- Fluconazol 400mg/día/VO. d.- Linezolid i.v. a dosis de 600mg/12h. CASO CLINICO Varón de 47 años de edad ex-adicto a drogas por vía parenteral y con infección por VIH conocida desde hacía 6 años, en tratamiento con antirretrovirales que cumplía de forma irregular. Ingresa por cefalea de 3 semanas de evolución de localización holocraneal que aumentaba tras maniobras de Valsalva. Su intensidad ha seguido un perfil progresivo, interfiriendo el sueño y requiriendo dosis crecientes de analgesia. Ha asociado vómitos en los últimos días. La familia reconoce enlentecimiento mental y episodios confusionales recortados durante los cuales realizaba movimientos repetitivos de las manos y de succión. En los 4 días previos a su asistencia en Urgencias, refería inestabilidad en la marcha, diplopía e hipersomnia diurna. Paciente desorientado parcialmente en el tiempo (desconocía día del mes y de la semana), aunque orientado en espacio y persona. Bradipsíquico. Con rigidez de nuca. Borramiento de bordes papilares con incipientes hemorragias peripapilares y desaparición del pulso venoso. Paresia bilateral del VI par. Deficiente estado nutricional y palidez cutáneomucosa. Hemograma: anemia normocítica y normocrómica (hemoglobina 10.5 g/dl). Leucocitos 3200 con linfopenia (15 %). Serie plaquetaria normal. Velocidad de sedimentación globular 56 mm en la primera hora. Hipernatremia leve (149 mg/dl), hipertransaminasemia (glutámico-oxalacético transaminasa 78 unidades por litro, glutámico-pirúvico transaminasa 120 unidades por litro, gamma-glutamil-transpeptidasa 320 unidades por litro), resto de parámetros normales. Estudio de coagulación normal, salvo hiperfibrinogenemia (765). Hipergammaglobulinemia policlonal e hipoalbuminemia discreta.

CURSO ENARM CMN SIGLO XXI TEL: 36246001

Pharmed Solutions Institute

PÁGINA 157

MANUAL DE TRABAJO DEL CURSO ENARM CMN SIGLO XXI PREGUNTA Cuál de los siguientes diagnósticos le parece el más probable? RESPUESTA a.- Complejo demencia-SIDA (Síndrome de Inmunodeficiencia Adquirida) d.- Enfermedad de Alzheimer en estadio leve c.- Pseudodemencia depresiva d.- Meningitis tuberculosa CASO CLINICO Agricultor, del sexo masculino, de 64 años, admitido en Urgencias con un cuadro de fiebre baja, mialgias, artralgias e hiporexia, hacía ya tres semanas y evolucionando hacía ya tres días con tos, disnea y compromiso del estado general. A su llegada, estaba taquipneico, depletivo, con vómitos, confusión mental y agitación. Presentaba de rigidez cervical terminal. Estertor crepitante en la base del hemitórax derecho. Soplo diastólico en foco mitral +2/+6, sin irradiación y soplo sistólico en foco aórtico +3/+6, con irradiación cervical. Leucograma de 18.700 cels/mm³ y 6% de bastones. La radiografía de tórax mostró opacidad alveolar en la base derecha. El paciente evolucionó con insuficiencia respiratoria aguda, siendo necesario la realización de intubación orotraqueal y ventilación mecánica. Fue realizada tomografía computadorizada (TC) de cráneo que no arrojó alteraciones, y la punción lumbar con líquido cefalorraquídeo presentando 78 cels/mm³, siendo 87% polimorfonucleares, hipoglicorraquia. PREGUNTA Cual es la conducta terapéutica mas apropiada a seguir. RESPUESTA a.- Ceftriaxone mas vancomicina. b.- Meropenen mas aciclovir. c.- Amikacina mas levofloxacina. d.- Bencilpenicilina benzatinica. ENFERMEDADES VASCULARES DEL SISTEMA NERVIOSO: Comprenden un conjunto de trastornos de la vasculatura cerebral que conllevan a una disminución del flujo sanguíneo en el cerebro (flujo sanguíneo cerebral o FSC) con la consecuente afectación, de manera transitoria o permanente, de la función de una región generalizada del cerebro o de una zona más pequeña o focal, sin que exista otra causa aparente que el origen vascular. La enfermedad cerebrovascular trae como consecuencia procesos isquémicos o hemorrágicos, causando o no la subsecuente aparición de sintomatología o secuelas neurológicas. La hipertensión arterial (HTA) es el principal factor de riesgo de la enfermedad cerebrovascular. Epidemiología: Actualmente la cifra de muertes por ECV supera los 5 millones anuales, lo que equivale a 1 de cada 10 muertes. Las enfermedades cerebrovasculares ocupan el tercer lugar como causa de muerte en el mundo occidental, después de la cardiopatía isquémica y el cáncer, y la primera causa de invalidez en personas adultas mayores de 65 años. La enfermedad cerebrovascular fue también la quinta causa principal de pérdida de productividad, medido por los años de vida ajustados por discapacidad. Ello incluye los años de pérdida de productividad por razón de muerte o distintos grados de discapacidad. Las mujeres son más propensas a padecer ECV, sobre todo cuando han perdido el factor protector estrogénico. Factores de Riesgo: El consumo de cigarrillos es el factor de riesgo modificable más poderoso que contribuye a la enfermedad cerebrovascular, independiente de otros factores de riesgo. Otros factores de riesgo demostrados son la hipertensión arterial y la diabetes mellitus. Por su parte, las dislipidemias suelen ser factores de riesgo más importantes en la enfermedad coronaria que en la cerebrovascular. Clasificación: Según el tiempo de evolución de la ECV se agrupan en: Accidente isquémico transitorio (AIT). Es cuando los síntomas de la focalidad neurológica se recupera en menos de 24 horas sin secuelas, de inicio súbito que por lo general dura menos de 15 minutos. Déficit isquémico neurológico reversible (RIND). Es cuando la duración del déficit persiste por más de 24 h, pero los síntomas desaparecen en un plazo de 7 a 21 días, sin secuelas. Accidente cerebrovascular (ACV) establecido: el déficit neurológico de origen vascular persiste y no se modifica por más de tres semanas desde su instauración y deja secuelas. ACV estable: el déficit neurológico persiste sin modificaciones por 24 horas (en los casos de origen carotídeo) o 72 horas (en los casos de origen vertebrobasilares), pudiendo luego evolucionar hacia un RIND o ACV establecido. ACV en evolución o progresivo: la focalidad neurológica aumenta y el cuadro empeora o aparece nueva clínica en 24 a 48 horas. ACV con tendencia a la mejoría o secuelas mínimas: son casos con un curso regresivo de modo que la recuperación al cabo de 3 semanas es mayor al 80%. Lo más frecuente es dividirlos en dos grupos según el mecanismo, así suele ser de tipo isquémico o hemorrágico: Isquémico (85% de los casos). Trombótico (Infarto lacunar: oclusión de vasos cerebrales pequeños ocluyendo la irrigación sanguínea de un volumen pequeño de tejido cerebral. Infarto de un gran vaso sanguíneo. Embólico (Cardioembólico: la embolia proviene del corazón, con frecuencia, de la aurícula cardíaca). Arteria-arteria. Criptogénico: la oclusión de un vaso intracraneal sin causa aparente. Hemorrágico: (Intraparenquimatoso, Subdural, Epidural, Subaracnoideo). También pueden ser clasificados en función del área en el cerebro donde ocurre el mayor daño o por el territorio vascular afectado y el curso clínico del trastorno. Etiología: Trombosis en pacientes con arteroesclerosis, la hemorragia cerebral hipertensiva, el accidente isquémico transitorio, el embolismo y la rotura de aneurismas. Las malformaciones arteriovenosas, la vasculitis y la tromboflebitis también causan con frecuencia ECV. Otras causas menos frecuentes incluyen ciertas alteraciones hematológicas como la policitemia y la púrpura trombocitopénica, los traumatismos de la arteria carótida, los aneurismas disecantes de la aorta, la hipotensión arterial sistémica y la jaqueca con déficit neurológico. Fisiopatología: El flujo sanguíneo cerebral (FSC) es aproximadamente 15-20% del gasto cardíaco total, de 550 - 750 ml de sangre por cada kg de tejido por cada minuto. La disminución o interrupción del flujo sanguíneo cerebral produce en el parénquima cerebral daño celular y funcional que empeora con el transcurrir del tiempo. Penumbra isquémica: En el tejido cerebral donde ocurre la isquemia resultado de la disminución del FSC, se distinguen dos zonas: Un núcleo isquémico de isquemia intensa: A los 10 s de isquemia se aprecia pérdida de la actividad eléctrica neuronal por alteraciones en los potenciales de membrana notable en el EEG. A los 30 s se observa fallo de la bomba sodio-potasio con alteraciones

CURSO ENARM CMN SIGLO XXI TEL: 36246001

Pharmed Solutions Institute

PÁGINA 158

MANUAL DE TRABAJO DEL CURSO ENARM CMN SIGLO XXI en el flujo ionico y desequilibrio osmótico con pérdida de la función neuronal y edema citotóxico. Al cabo de 1 min y por el predominio de la glucólisis anaeróbico, aumenta a niveles letales la concentración de ácido láctico y los mediadores de la cascada isquémica. Después de 5 min se aprecian cambios irreversibles en los orgánulos intracelulares y muerte neuronal. Rodeando a este núcleo isquémico evoluciona la llamada penumbra isquémica donde el efecto de la disminución en el flujo sanguíneo cerebral, el cual ha descendido a niveles críticos alrededor de 15 a 20 mL/100 g/min,7 no ha afectado la viabilidad celular. La extensión del área de penumbra depende del mejor o peor funcionamiento de la circulación colateral. Vulnerabilidad celular: Las neuronas más sensibles a la isquemia son las células de la corteza cerebral, del hipocampo, el cuerpo estriado y las células de Purkinje del cerebelo. De las neuroglías, se afecta primero los oligodendrocitos, los astrocitos (gliosis reactiva) y por último la microglía. Las células del endotelio vascular son las últimas en ser afectadas. También se ha notado una menor concentración de células progenitoras endoteliales en pacientes con enfermedad cerebrovascular. Factores de influencia: Ciertos factores participan en el daño cerebral progresivo, como el calcio, acidosis láctica, radicales libres, glutamato, el factor de adhesión plaquetaria y la descripción genética del individuo. El daño por isquemia cerebral se verá mayor o menor dependiendo también de: El estado del flujo sanguíneo cerebral regional; El tiempo que dura la oclusión vascular; El funcionamiento de la criculación colateral; El grado de vulnerabilidad celular frente a la isquemia; La presencia de sustancias vasoactivas como ácidos grasos y radicales libres en la zona afectada; Hiperglicemia; Hipertermia; Los valores de la tensión arterial; El grado de hipoxia. Cuadro clínico: La presentación clínica de la ECV se da fundamentalmente de dos formas: aguda, manifestada por el accidente cerebrovascular y la hemorragia subaracnoidea; y una forma crcónica manifestada por demencia y epilepsia. Síntomas neurológicos como debilidad, cambios del lenguaje, visión o cambios en la audición, trastornos sensitivos, alteración del nivel de conciencia, ataxia, u otros cambios en la función motora sensorial. La enfermedad mental puede también producir trastornos de la memoria. Diagnóstico: Ante la sospecha de enfermedad cerebrovascular, se necesita identificar la lesión y su ubicación y obtener información sobre el estado estructural del parénquima del cerebro y su condición hemodinámica como consecuencia de la lesión. La evaluación neuropsicológica de sujetos con daño producido por enfermedad cerebrovascular está enfocada en conocer las funciones afectadas y depende del tipo de evento. En el infarto cerebral se estudia por imágenes radiológicas los aspectos topográficos de la lesión, especialmente antes de las 24 horas del inicio del trastorno súbito. Los más utilizados son la tomografía computarizada, la resonancia magnética y el estudio del flujo sanguíneo regional cerebral. Tratamiento: El tratamiento debe ser individualizado, según las condiciones de cada paciente y la etapa de la enfermedad cerebrovascular, sopesando los riesgos frente a los posibles beneficios. En general, hay tres etapas de tratamiento: la prevención del accidente cerebrovascular; la terapia provista inmediatamente después de la persona sufrir un accidente cerebrovascular; y la rehabilitación del paciente después de sufrir el accidente cerebrovascular. Para la prevención de eventos cerebrovasculares, un estudio demostró que el uso de ramipril era efectivo en pacientes de alto riesgo con o sin hipertensión arterial o ECV previo. La vitamina E no parece ser efectivo en reducir el riesgo de accidente cerebrovascular fatal o no fatal. Terapia farmacológica: El tratamiento médico está destinado a reducir los riesgos y/o complicaciones de un accidente cerebrovascular a corto y largo plazo. El uso de antitrombóticos se indica tan pronto como se ha descartado una hemorragia intracraneal. Las enfermedades cerebrovasculares no cardioembólicas, no hemorrágicas son tratadas con agentes antiplaquetarios, en lugar de la anticoagulación oral como tratamiento inicial. La aspirina, administrada entre 50-325 mg diarios, o combinada con dipiridamol de liberación prolongada, y el clopidogrel (75 mg diarios) son las opciones recomendadas de primera línea. La combinación de aspirina y clopidogrel, que bloquea la activación de la plaqueta por el difosfato de adenosina, puede ser superior a la aspirina sola en la reducción de riesgo de eventos cerebrovasculares isquémicos. La aspirina en combinación con el clopidogrel aumenta el riesgo de hemorragia y no se recomienda combinada de forma rutinaria para los pacientes con accidente isquémico transitorio. Para casos con enfermedad cerebrovascular cardioembólico como la fibrilación auricular, prótesis de las válvulas cardíacas o prolapso de la válvula mitral, se indica la anticoagulación a largo plazo, principalmente con heparina no freccionada, obteniendo un INR de 1,5-2,5. La administración de 325 mg diarios de aspirina se recomienda para aquellos que no pueden tomar anticoagulantes orales. En casos de infarto agudo de miocardio con trombo ventricular izquierdo puede combinarse la anticoagulación oral con aspirina hasta 162 mg diarios. En pacientes con miocardiopatía dilatada también se indican anticoagulantes orales o algunos clínicos consideran iniciar la terapia antiplaquetaria. Normalmente no se recomienda añadir agentes antiplaquetarios a la warfarina en casos con enfermedad reumática de la válvula mitral, a menos que el paciente tenga una embolia recurrente a pesar de tener un INR terapéutico. En casos de calcificación del anillo mitral se suele administrar tratamiento antiplaquetario. Los pacientes con regurgitación mitral pueden recibir warfarina o aspirina. Cerca de un 4-28% de los pacientes con hemorragia intracerebral presentan convulsiones, las cuales pueden ser rápidamente controladas con una benzodiazepina, como lorazepam o diazepam, acompañado de fenitoína o fosfenitoína. El uso de una terapia anticonvulsiva de manera profiláctica en todos los casos de hemorragia intracerebral es controvertido, ya que no hay ensayos controlados que han demostrado un beneficio claro. La trombólisis con activador tisular del plasminógeno se ha definido como el tratamiento de primera línea del infarto isquémico agudo, pero debe ser administrada durante las tres horas posteriores al accidente cerebrovascular. Control de la tensión arterial: Aunque no hay estudios controlados que definan los niveles óptimos de presión arterial en pacientes con ECV, el seguimiento de los niveles de presión arterial es importante. Se cree que la presión arterial muy elevada puede conducir a nuevas hemorragias y/o la expansión de un hematoma intracraneal. Por otra parte, la bajada súbita de una presión arterial elevada puede comprometer la perfusión cerebral. Las dos excepciones al manejo conservador de la hipertensión arterial son posterior al uso de activador del plasminógeno tisular y ante un concomitante infarto de miocardio. El nicardipino, labetalol, esmolol, y la hidralazina son agentes que pueden ser utilizados cuando sea necesario el control de la presión

CURSO ENARM CMN SIGLO XXI TEL: 36246001

Pharmed Solutions Institute

PÁGINA 159

MANUAL DE TRABAJO DEL CURSO ENARM CMN SIGLO XXI arterial. No se suele usar el nitroprusiato ya que puede elevar la presión intracraneal. La American Heart Association publica las siguientes pautas para el tratamiento de la hipertensión arterial: Si la presión arterial sistólica es> 200 mmHg o la presión arterial media (PAM) es > 150 mmHg, se considera la reducción agresiva de la presión arterial con la infusión intravenosa continua con valoraciones frecuentes de la presión arterial (cada 5 min). Si la presión arterial sistólica es > 180 mmHg o PAM es > 130 mmHg y hay evidencia o sospecha de hipertensión intracraneal (PIC), entonces se considera la vigilancia de la PIC y se reduce la presión arterial con el uso de medicamentos por vía intravenosa de manera intermitente o continua para mantener la presión de perfusión cerebral > 60-80 mmHg. Si la presión arterial sistólica es > 180 o PAM es > 130 mmHg y no hay pruebas ni sospecha de elevación de la PIC, se considera una modesta reducción de la presión arterial (PAM diana de 110 mmHg o presión arterial deseada de 160/90 mmHg) con controles de la presión arterial cada 15 minutos. Pronóstico: La complicación más catastrófica de los diversos tipos de enfermedad cerebrovascular es la isquemia súbita e irreversible de alguna parte del cerebro, es decir, el accidente cerebrovascular, especialmente frecuente en ancianos. La severidad varía, desde la recuperación total de las funciones cerebrales y aquellas que del cerebro dependen, en menos de 24 horas, hasta la discapacidad severa y la muerte. La mortalidad por ECV isquémico en el primer mes oscila entre el 17 y 34%, mientras la del hemorrágico puede ser dos veces mayor. CASOS CLINICOS Mujer de 73 años sin factores de riesgo vascular que acudió a urgencias por presentar cuadro de inicio súbito de disminución de la movilidad en hemicuerpo izquierdo con caída al suelo. A su llegada a urgencias estaba consciente con desviación conjugada de la mirada a la derecha y hemiplejía completa izquierda, hemihipoestesia y reflejo de Babinsky izquierdo presente. Puntuación en la escala NIHSS de 16. TC craneal sin lesiones evidentes. Tras consentimiento informado escrito se aplicó tratamiento trombolítico a los 150min del inicio de los síntomas. PREGUNTA Considerando la sintomatologia, cual es la arteria mas probablemente involucrada? RESPUESTA a.- Arteria cerebral anterior izquierda. b.- Arteria cerebral media Izquierda. c.- Arteria cerebral anterior derecha. d.- Arteria cerebral media derecha. CASO CLINICO Varón de 75 años, con dislipemia en tratamiento con hipolipemiante como único factor de riesgo cardiovascular. Traído a urgencias por cuadro de hemiparesia izquierda, parálisis facial central izquierda, hipoestesia en hemicuerpo izquierdo, disartria y Babinsky izquierdo. TC cráneo sin alteraciones. NIHSS 19. Tras consentimiento informado escrito se administró tratamiento trombolítico con 150min de evolución desde el inicio de los síntomas. PREGUNTA Considerando la sintomatologia, cual es la arteria mas probablemente involucrada? RESPUESTA a.- Arteria cerebral anterior izquierda. b.- Arteria cerebral media Izquierda. c.- Arteria cerebral anterior derecha. d.- Arteria cerebral media derecha. CASO CLINICO Varón de 54 años fumador y con episodio compatible con accidente isquémico transitorio dos años antes. Trasladado a urgencias por hemiplejía derecha de 5 horas de evolución, objetivándose plejia completa de la extremidad superior y paresia de la extremidad inferior, NIHSS 7. PREGUNTA Cual es la conducta a seguir mas apropiada al caso? RESPUESTA a.- Tratamiento sintomático. b.- Tratamiento trobolitico. c.- Tratamiento antiplaquetario. d.- Tratamiento anticoagulante. CASO CLINICO Mujer de 52 años fumadora, sin otro factor de riesgo que fue trasladada a urgencias por hemiplejía derecha y afasia de instauración brusca. Destacó a su ingreso hemiplejía completa derecha y Babinsky derecho, NIHSS de 18. Se realizó TC craneal sin que se objetivaran alteraciones. Tras realizar consentimiento informado se aplicó trombolisis a los 150min del inicio de los síntomas. A las 24h persistía en la misma situación neurológica, realizando TC de control que mostraba infarto isquémico extenso con edema y efecto masa, por lo que se asoció tratamiento antiedematoso. Pese a ello 48h después presentó rápido deterioro neurológico, con nuevo TC mostrando

CURSO ENARM CMN SIGLO XXI TEL: 36246001

Pharmed Solutions Institute

PÁGINA 160

MANUAL DE TRABAJO DEL CURSO ENARM CMN SIGLO XXI progresión del edema y signos de enclavamiento de tronco, desestimándose cirugía descompresiva. Finalmente la paciente fue incluida en protocolo de donación de órganos. PREGUNTA Considerando la sintomatologia, cual es la arteria mas probablemente involucrada? RESPUESTA a.- Arteria cerebral anterior izquierda. b.- Arteria cerebral media Izquierda. c.- Arteria cerebral anterior derecha. d.- Arteria cerebral media derecha. ESCLEROSIS MÚLTIPLE (EM). CIENCIAS BÁSICAS: La esclerosis múltiple es caracterizada por la inflamación crónica (reacción linfocitaria y mononuclear) y destrucción selectiva de la mielina del SNC, importante con conservación de los axones, el sistema nervioso periférico está a salvo. Hay daño de los oligodendrocitos con proliferación astrocí ca. Los hallazgos anteriores junto con la pérdida de la mielina cons tuyen la placa de desmielinización. Aunque pueden verse en la médula espinal, el tallo cerebral y el nervio óptico, la localización más frecuente es la periventricular (90%). Estas también CRITERIOS DE SCHUMACHER: pueden encontrarse en la sustancia gris, principalmente en Examen neurológico que evidencie compromiso del sistema nervioso localización subpial. Las placas aisladas raramente miden Evidencia clínica de 2 o más lesiones del sistema nervioso más de 1,5 cm. Etiología se piensa que es autoinmune, Compromiso principalmente de la sustancia blanca El compromiso del sistema nervioso debe seguir el siguiente patrón: dos o más con la susceptibilidad determinada por factores genéticos episodios de la enfermedad con una duración no menor de 24 hrs y usualmente y ambientales. SALUD PÚBLICA: La prevalencia de la menos de un mes, o una lenta progresión de la enfermedad por un periodo hasta esclerosis m l ple es muy variada en diferentes partes del de 6 meses. mundo, y es así como en las la tudes norte la enfermedad Inicio de los síntomas en edades comprendidas entre 10-50 años Los signos y síntomas no deben ser mejor explicados por otra enfermedad es más frecuente. Por ejemplo, su prevalencia en algunas Esclerosis múltiple definitiva apoyada en clínica zonas de Escocia es de 309 por cada 100.000 habitantes, Según clínica, la esclerosis múltiple definitiva deberá reunir una de estas mientras que en el Norte de los Estados Unidos la condiciones: prevalencia es de 58 por cada 100.000 habitantes y en 2 episodios de la enfermedad y evidencia de 2 lesiones separadas Japón, África y Sudamérica la prevalencia desciende hasta 2 episodios de la enfermedad, evidencia clínica de una lesión y paraclínica de otra. cuatro personas por cada 100.000 habitantes. La incidencia de la enfermedad es casi 1,8 veces mayor en mujeres que en hombres. Compromete más la raza blanca y la edad de aparición más frecuente está entre los 20 y los 40 años. ASPECTOS CLINICOS: Lo más común son los ataques recurrentes de disfunción neurológica focal, por lo general duran semanas o meses. Los pacientes con la forma de recaída remisión tienen en promedio un episodio de disfunción neurológica por año. Cada recaída es seguida por una secuela o recuperación total de su cuadro clínico, las recaídas están separadas por un periodo asintomático. Cincuenta por ciento de los pacientes con la forma de recaída remisión pueden virar a un deterioro más progresivo que se denomina la forma secundariamente progresiva. Aproximadamente 10 a 15 % de los pacientes tienen una forma primariamente progresiva. En esta forma de la enfermedad hay un deterioro progresivo desde su inicio. Esta es quizás la clasificación clínica más reciente de esclerosis múltiple: Asintomática (hallazgos incidentales en RMN de cerebro) y la sintomática que son: 1. Recaída remisión; se caracteriza por ataques recurrentes de disfunción neurológica general con o sin recuperación entre los ataques, no se observa la progresión de la discapacidad neurológica. Representa el 85 % de los casos de EM de nueva aparición. 2. Secundariamente progresiva; no siempre se presenta inicialmente como recaída remisión sino que evoluciona para ser de forma progresiva. 3. Primariamente progresiva; se caracteriza por una progresión gradual de la discapacidad desde el inicio sin ataques discretos, el 15% de los casos de EM de nueva aparición. 4. progresiva con exacerbación; es una forma rara que comienza con un curso de primaria progresiva, pero se producen recaídas después superpuestas. La EM es una enfermedad crónica, 15 años después del diagnóstico, sólo el 20 % de los pacientes tienen ninguna limitación funcional, de un tercio a la mitad habrán progresado y requerirán ayuda para la deambulación. Los síntomas pueden desarrollarse de minutos a horas en 40% de los enfermos, durante varios días en 30% y lentamente durante semanas a meses, en 20% de los pacientes. DIAGNOSTICO: Clínico; El inicio puede ser brusco o insidioso. Algunos pacientes tienen síntomas que son tan triviales que no pueden buscar atención médica durante meses o años. Algunos pacientes se presentan inicialmente con deterioro neurológico lentamente progresivo. Los síntomas a menudo empeoran transitoriamente con fatiga, el estrés, el ejercicio o el calor. Las manifestaciones de la EM normalmente incluyen debilidad y/o síntomas sensoriales que implican una extremidad, dificultades visuales, alteraciones de la marcha y la coordinación, urgencia o frecuencia urinaria y fatiga anormal. Participación Motor puede presentarse como una pesada rigidez débil o torpe extremidad. Hormigueo localizado, "alfileres y agujas" y sensaciones "muertos" son comunes. La neuritis óptica puede resultar en visión borrosa, especialmente en el campo visual central, a menudo con dolor retroorbital acentuado por el movimiento del ojo. La participación del tronco cerebral puede causar diplopía, nistagmos, vértigo o dolor facial, entumecimiento, debilidad, hemiespasmo o mioquimia (ondulación contracciones musculares). Ataxia, temblor, disartria y siempre son representativas de la enfermedad de las vías cerebelosas. Síntoma de Lhermitte, una sensación momentánea eléctrica similar al shock provocado por la flexión del cuello, indica enfermedad en la médula espinal cervical. CRITERI S DIAGN STIC S: El diagnóstico de esclerosis múltiple es de predominio clínico, y existen algunos criterios, como los de Schumacher, que todavía permanecen vigentes. Con el advenimiento de nuevos estudios paraclínicos, como los potenciales evocados (PE), RMN (revela las áreas brillantes multifocales en las secuencias ponderadas en T2 en > 95 % de los pacientes, a menudo en una ubicación periventricular) y el estudio de LCR (Hallazgos pleocitosis linfocitaria leve 5-75 células (25 %) , bandas oligoclonales (75-90 %) , elevación de IgG (80 %) ) TRATAMIENTO: Aunque aún no existe un tratamiento definitivo para la EM. En la actualidad, para los brotes o periodos de exacerbación de la enfermedad se utilizan la hormona adrenocorticotropa (ACTH) y otros corticoides. La ACTH puede administrarse siguiendo un protocolo de dosis elevadas de la medicación que incluye 80 UI diarias intramusculares o intravenosas por siete días, seguidas de 40 UI intramusculares o intravenosas por cuatro días, finalizado con 20 UI por tres días. En la actualidad, se utiliza en la mayoría de los centros la metilprednisolona, a dosis altas, de 500 mg a 1 g IV diarios por

CURSO ENARM CMN SIGLO XXI TEL: 36246001

Pharmed Solutions Institute

PÁGINA 161

MANUAL DE TRABAJO DEL CURSO ENARM CMN SIGLO XXI tres a cinco días. Se recomienda administrar esta medicación en infusión, disolviéndola en 100 ml de dextrosa al 5%, para ser administrada en 30 minutos a 1 hora. Si la sintomatología del paciente persiste después de un mes a pesar de este tratamiento, se recomienda repetir el ciclo de metilprednisolona, seguido por la administración de 1 mg/kg de peso de prednisona interdiaria, reduciéndola en el curso de 8 a 14 semanas. Se han realizado muchos estudios utilizando azatioprina; sin embargo, los resultados no son muy buenos. Se han utilizado muchos otros tratamientos, como la ciclosporina, la plasmaféresis, los an cuerpos monoclonales, el oxígeno hiperbárico y algunos más que están en inves gación sin resultados sa sfactorios. El interferón beta (reduce las tasas de exacerbación anual aprox. 30 % y también reduce el desarrollo de nuevas lesiones de MRI) ha demostrado ser muy útil en el tratamiento de la esclerosis múltiple especialmente en la forma de recaídas y remisión, ha sido recomendado por la Academia Americana de Neurología en sus guías de manejo. Se ha demostrado que la medicación disminuye el número de recaídas y la severidad de las mismas. Esta medicación debe utilizarse de por vida, si no hay disminución en el número de recaídas y no se observa beneficio debe retirarse la medicación. Terapia sintomática: la espasticidad puede responder a la terapia física , lioresal (20-120 mg/d) , diazepam (2-40 mg/d) , tizanidina (8-32 mg/d) , dantroleno (25-400 mg/d) y clorhidrato de ciclobenzaprina (10-60 mg/d) . Disestesia puede responder a la carbamazepina (100-1200 mg/día) , fenitoína (300 mg/d) , gabapentina (300-3.600 mg/d), pregabalina (50-300 mg/d) o amitriptilina (50-200 mg/d) .El tratamiento de los síntomas de la vejiga se basa en la fisiopatología subyacente investigados con pruebas urodinámicas: hiperreflexia vesical es tratado con restricción de líquidos por la noche y la micción frecuente, y si esto no funciona, anticolinérgicos como oxibutinina ( 5-15 mg/d) puede ser juzgado; hiporreflexia es tratado con el betanecol colinérgica drogas (10-50 mg c/8hrs) , y disinergia debido a la pérdida de la coordinación entre la pared de la vejiga y los músculos del esfínter se trata con anticolinérgicos y cateterismo intermitente . La depresión debe ser tratada agresivamente. PROGRESIVA: Esta enfermedad se observa en pacientes con trastornos linfoprolifera vos, como la enfermedad de Hodg in, leucemia linfocí ca y linfosarcomas. También puede presentarse en pacientes con síndrome de inmunodeficiencia adquirida (SIDA). El JC papovavirus ha sido encontrado en la mayoría de los casos de leucoencefalopa a mul focal progresiva. La enfermedad cursa en forma subaguda con cambios de personalidad y demencia. Otros síntomas muy comunes son hemianopsia, disartria y ataxia para la marcha. El curso de la enfermedad es progresivo e inexorablemente lleva al paciente a la muerte en un periodo de algunos meses. El LCR puede ser normal, la TAC de cerebro y la RMC ayudan mucho para el diagnóstico y muestran el compromiso de la sustancia blanca. Un diagnóstico definitivo de la enfermedad solo puede hacerse por patología. : Esta entidad la describieron primero Adams y Mancall (1959). Clínicamente se caracteriza por una paraparesia o cuadriparesia de evolución rápida, con síntomas seudobulbares, que se manifiestan por disartria y disfagia, en pacientes con cambios electrolíticos importantes. La causa real de la mielinolisis no es clara, pero se han postulado teorías vasculares y metabólicas. El cuadro se observa en alcohólicos y pacientes a quienes se les corrige rápidamente una hiponatremia. Esta entidad también se ha visto asociada a otras patologías, como la enfermedad de Wilson, la cirrosis, las nefropatías y la enfermedad de Wernicke. Histológicamente la lesión compromete toda o parte de la base del puente. Puede afectar el tegmento pón co y ascender hasta el mesencéfalo, produciendo desmielinización importante. Para diagnosticar la enfermedad son de gran importancia los potenciales evocados auditivos y la RMC. El tratamiento de estos pacientes deberá hacerse con solución salina, administrada con prudencia, y restricción de líquidos. ENCEFALOMIELITIS DISEMINADA AGUDA (ADEM): Un fulminante, a menudo devastadora, enfermedad desmielinizante que tiene un curso monofásico y puede estar asociada con la inmunización antecedente o infección. Los signos de enfermedad neurológica diseminada son siempre presentes (por ejemplo, hemiparesia o tetraparesia, respuesta plantar extensora, perdidos o reflejos tendinosos hiperactivos, pérdida de la sensibilidad, y la participación de tronco cerebral). Puede presentarse fiebre, dolor de cabeza, meningismo, letargia progresando al coma y convulsiones. Pleocitosis, generalmente 200 células/l, es común. RM puede revelar extensa realce de la materia blanca en el cerebro y la médula espinal. El tratamiento inicial es con glucocorticoides a dosis altas. Los pacientes que no responden pueden beneficiarse de un curso de la plasmaféresis o inmunoglobulinas. CASO CLINICO Paciente de 45 años con un cuadro clínico que comenzó hace 2 años con ligera disminución de la fuerza muscular, debilidad e inestabilidad para la marcha, tuvo una evolución desfavorable y en la actualidad muestra falta de coordinación, alteraciones visuales, rigidez muscular y trastornos del habla. Acude a nuestro centro para realizarse Resonancia Magnética por Imágenes de Cráneo y Columna cervical, encontrándose las siguientes alteraciones, en la Resonancia Magnética por Imágenes (RMI) de Cráneo (Secuencias T1, T2 y Flair axial), observamos imágenes ovoides, algunas miden 6 mm de diámetro, perpendiculares al eje mayor de los ventrículos que se observan isointensas y algunas hipointensas en T1 y que en T2 y Flair se muestran hiperintensas, localizadas nivel del cuerpo calloso, en la sustancia blanca profunda periventricular y a nivel de la protuberancia y bulbo; por las alteraciones antes descritas se solicita realizar Resonancia Magnética por Imágenes (RMI) de columna cervical (secuencias T1 y T2 sagital y axial T2), donde visualizamos a nivel de médula áreas focales hiperintensas con muy ligero efecto de masa y de características similares a las del encéfalo. PREGUNTA Considerando la clasificación de esta patologia, cual es el grado mas probable del caso?. RESPUESTA a.- 1. b.- 2. c.- 3. d.- 4. CASO CLINICO Masculino de 68 años de edad quien, previamente sano, sufre un accidente de tránsito al ser atropellado; produciéndose politrauma severo y fractura de base de cráneo, fractura expuesta del tercio medio de la tibia derecha y trauma de la rodilla izquierda. Al ingreso Glasgow 3 se describe pupila izquierda media no reactiva, sospechando un III par craneal compresivo se realiza TC que corrobora la

CURSO ENARM CMN SIGLO XXI TEL: 36246001

Pharmed Solutions Institute

PÁGINA 162

MANUAL DE TRABAJO DEL CURSO ENARM CMN SIGLO XXI fractura y demuestra una hemorragia subaracnoidea sin hematoma; se instaura tratamiento con fenitoína y diuréticos; se considera que presenta un III par craneal periférico traumático. En 12 horas Glasgow 7. A los 2 días movilizaba las 4 extremidades y abría los ojos al llamado, no acataba órdenes; posteriormente su sensorio mejoró manteniendo un Glasgow de 14. Durante la estancia el paciente recibió varios esquemas de antibióticos, debido a neumonía nosocomial asociada a ventilación mecánica la cual requirió los tres primeros días. Al mes de estancia presenta progresivo descenso de la natremia con valores desde 126 mmol/l, hasta 108 a las dos semanas cuando se inició el tratamiento de reposición; concomitantemente el paciente tenía progresivo deterioro del sensorio. Durante las primeras horas la corrección se logró llevar a un ritmo adecuado, pero no ocurrió así a las 12 horas de reposición que se pasó de un sodio de 116 a 125 mmol/l en 5 horas. A los 2 días el paciente nuevamente conciente pero se anota que presenta dificultad para la deglución; luego presenta un cuadro de diarrea asociada a antibióticos que lo hace llegar a una natremia de 138 mmol/l. PREGUNTA Cual es la velocidad de corrección de sodio para evitar esta complicación del caso? RESPUESTA a.- 05 mmol/l/dia. b.- 10 mmol/l/dia. c.- 15 mmol/l/día. d.- 20 mmol/l/dia. CASO CLINICO Paciente de sexo masculino, 44 años, diestro, infección por VIH, en tratamiento antirretroviral. Presentaba una población linfocitaria con CD4 inferior a 160 elementos/mm3. Comienza con un síndrome confusional, alteraciones conductuales y cefalea moderada. A los pocos días de su ingreso se percibe pérdida de fuerza de sus miembros izquierdos y alteraciones del habla que se agravaron en el curso de los días. No se detecta fiebre. El examen mostraba un paciente irritable, con desorientación temporal y espacial, tendencia al mutismo, síndrome tónico frontal bilateral predominando a derecha. No se detectó síndrome meníngeo ni fiebre. La TC craneal en fase de estado mostró una lesión frontal derecha y de cuerpo calloso, con compromiso menor del lóbulo frontal izquierdo que incidía especialmente sobre la sustancia blanca, sin efecto de masa y con realce con el contraste. La RM encefálica delimitó con mayor exactitud las lesiones observándose lesiones de baja señal en T1 y de aumento de señal en T2. Se resuelve practicar biopsia estereotáxica, Hay acúmulos de histiocitos de citoplasmas microvacuolados por fagocitosis de restos mielínicos lipídicos, Existe moderada astrogliosis con astrocitos de aspecto bizarro símil neoplásicos, con núcleos grandes, algunos multilobulados que no se acompañan de aumento de densidad celular. La oligodendroglía muestra una cariomegalia hipercuomática y algunos de ellos tienen gruesas inclusiones intranucleares acidófilas 'en vidrio esmerilado' que contienen el antígeno viral. Hay escaso o nulo exudado inflamatorio perivascular linfo-plasmocitario". PREGUNTA Considerando la clínica y los hallazgos histológicos, cual es el diagnostico mas apropiado? RESPUESTA a.- Leucoencefalopa a mul focal progresiva. b.- Encefalomielitis diseminada aguda. c.- Mielinólisis pón ca. d.- Escleroris lateral amniotrofica. CASO CLINICO Paciente de 21 años, con antecedentes de pan-colitis ulcerosa, en tratamiento esteroidal con prednisona 60 mg/día y Mesalazina 1 gr c/6 hrs durante 5 meses previos al ingreso. Días previos al diagnóstico completó tratamiento con ciprofloxacino por síndrome diarreico presuntamente bacteriano, sin confirmación etiológica y con respuesta favorable. Ingresó al Servicio de Urgencia por cuadro de 48 horas de evolución de compromiso de conciencia caracterizado por desorientación temporo-espacial y somnolencia, además de deposiciones sanguinolientas. Al examen físico general presentaba parámetros hemodinámicos dentro de rangos normales, afebril y como único hallazgo se describe candidiasis orofaríngea. Al examen neurológico el paciente se encuentra vigil, con tendencia a la somnolencia, desorientado en tiempo y espacio, nistagmo vertical, prueba de pequeña paresia positiva a derecha, y signo de Brudzinski. Resto del examen sin hallazgos. Se realiza TAC de cerebro que no muestra hallazgos patológicos y se realiza punción lumbar que da salida a líquido cefalorraquídeo claro, con proteínas 74,8/mm3, glucorraquia normal, glóbulos rojos 2/mm3, leucocitos 760/ mm3 de predominio linfocítico. Por antecedente de inmunosupresión crónica y candidiasis orofaríngea al ingreso, se inicia tratamiento antibiótico con ampicilina+ceftriaxona+aciclovir y corticoides en dosis de estrés (Hidrocortisona 100mg c/8 hrs). Evoluciona en buenas condiciones generales, estable desde el punto de vista hemodinámico y recuperando conciencia a las 48 horas. Se realiza Resonancia magnética que muestra imágenes con compromiso talámico bilateral asimético asociado a compromiso capsular posterior izquierdo con discreta captación de contraste. PREGUNTA Considerando la clínica y los estudios de imagen, cual es el diagnostico mas apropiado? RESPUESTA a.- Leucoencefalopa a mul focal progresiva. b.- Encefalomielitis diseminada aguda. c.- Mielinólisis pón ca.

CURSO ENARM CMN SIGLO XXI TEL: 36246001

Pharmed Solutions Institute

PÁGINA 163

MANUAL DE TRABAJO DEL CURSO ENARM CMN SIGLO XXI d.- Esclerosis lateral amniotrofica. DEMENCIAS: CIENCIAS BASICAS: La demencia es un síndrome clínico caracterizado por perdida adquirida de habilidades cognitivas y emocionales, lo suficientemente importante para interferir con el funcionamiento diario y la calidad de vida. También puede acompañarse de otras manifestaciones neuropsiquiátricas tales como alteraciones motoras, de la conducta, depresión, ansiedad, alucinaciones y/o delirium. El término demencia no implica una causa o proceso patológico específico, es de etiopatogenia heterogénea y puede ser mixta. Existen más de 55 enfermedades que pueden causar demencia. La definición de demencia según Clasificación de la demencia de acuerdo al DSM IV se divide de la siguiente manera: Demencia en enfermedad de Alzheimer; de inicio temprano, de inicio tardío, mixta o atípica, inespecífica. Demencia vascular; de inicio agudo, multi-infarto, vascular subcortical, mixta: subcortical y cortical. Demencias propias de otras enfermedades; Enfermedad de Pick, Enfermedad de Creutzfeldt-Jakob, Enfermedad de Huntington, Enfermedad de Parkinson, asociada a infección por VIH, asociada a otras enfermedades. Demencia no especificada. Así mismo una vez hecho el diagnóstico de demencia se puede clasificar: Leve: cuando afecta actividades de la vida diaria sin causar problemas para la independencia del paciente. Moderado: cuando condiciona incapacidad para vivir de manera independiente. El individuo no recuerda información básica acerca de su vivienda, actividades recientes o el nombre de ciertas personas familiares a él. Severo: Se caracteriza por la absoluta incapacidad de retener nueva información y solo se recuerdan fragmentos de experiencias o conocimientos pasados. El paciente no reconoce a sus familiares. SALUD PUBLICA: La E. Alzheimer en 2050 podría llegar al 30% de la población mundial. A medida que se prolonga la expectativa de vida también aumenta la prevalencia de enfermedades como las demencias. La frecuencia de E. Alzheimer se duplica cada 5 años apartir de los 60a. A los 60 años 1%, entre 80-84 años de 16%, >de 85años de 30-40%. Existen factores de riesgo no modificables como la edad siendo este el más importante para el desarrollo de demencia. En los pacientes con trastorno del aprendizaje como Síndrome de Down las demencias se presentan más temprano. Estudios de prevalencia muestran un mayor índice de demencia en mujeres que en hombres, especialmente en enfermedad de Alzheimer (2:1). El número de casos de demencia vascular es mayor en hombres que en mujeres aunque éstas últimas suelen igualarlos en edades más avanzadas. La historia familiar de demencia en un pariente de primer grado aumenta cuatro veces el riesgo de desarrollar demencia. Demencia vascular es la segunda causa de demencia después del Alzheimer representando hasta el 18% de las demencias, se han descrito prevalencias que varían desde el 3 al 21%. DIAGNOSTICO: Para un diagnóstico clínico confiable la pérdida de la memoria debe estar presente al menos durante seis meses; si la duración de las manifestaciones es menor, el diagnóstico es presuntivo. La severidad de la demencia se establece por el grado de decremento cognitivo o de la memoria, cualquiera que sea el más predominante. Cuando existen episodios concomitantes de delirium el diagnostico de demencia debe ser diferido. El diagnóstico de las demencias se debe realizar por un especialista experto en el tema y podría abordarse de la siguiente establece en dos pasos: se deberá hacer solo tras una extensa evaluación que debe incluir: historia clínica, evaluación del estado cognitivo, examen físico y otros estudios apropiados, una revisión de los medicamentos tomados por el paciente en orden de descartar efectos adversos que afecten el funcionamiento y minimizar su uso. ENFERMEDAD DE ALZHEIMER: Síndrome clínico adquirido y progresivo caracterizado por afectar las funciones intelectuales superiores. Deterioro de la memoria en el corto y largo plazo. Presencia de por lo menos uno de los siguientes déficits cognitivos: Afasia, Agnosias, Apraxia y/o alteraciones de funciones ejecutivas. Alteración de conducta: Laboral y social. No debe existir alteración de nivel de conciencia. En su patogenia exiten Hipótesis colinérgica: hay pérdida de neuronas colinérgicas en corteza e hipocampo con aumento de acción de butirilcolinesterasa. Hipótesis glutamatergica: L-glutamato neurotransmisor exitatorio SNC, interviene en los procesos de memoria, aprendizaje y plasticidad neuronal, el aumento parece jugar un rol principal en la patogénesis y en el daño producido por isquemia. Se genera por la producción anormal y acumulación de ß amiloide, aumenta la producción de ßA1-42 que es el que se deposita. Se generan las placas de amiloide que desencadenan, formación de ovillos neurofibrilares (por hiperfosforilación de proteína Tau). Oxidación y peroxidación de lípidos. Excitotoxicidad por glutamato, inflamación, activación de la cascada que lleva a la apoptosis. La muerte neuronal lleva al déficit de neurotrasmisores. En las guías de diagnóstico, se describe de manera didáctica 10 signos de alerta para pensar en Alzheimer: 1. Pérdida de la memoria 2. Dificultad para realizar tareas familiares 3. Problemas con el lenguaje 4. Desorientación en tiempo y espacio 5. Juicio empobrecido 6. Problemas con el pensamiento abstracto 7. Pérdida de cosas 8. Cambios en el ánimo o el comportamiento 9. Cambios en la personalidad 10. Pérdida de la iniciativa Alteraciones neuropsiquiátricas y motoras. Conforme la enfermedad avanza se presentan apraxias y con ello la incapacidad funcional para las actividades cotidianas, como asearse y vestirse, puede haber desinhibición y agresividad, las alucinaciones son poco frecuentes y si se presentan el diagnóstico debe orientarse en otra dirección, sin embargo pueden desarrollar delusiones, entre ellas se ha descrito el síndrome de Capgras hasta en el 10% de los pacientes, donde generan la idea de que su cuidador ha sido sustituido por un impostor, paralelamente hay alteraciones en el patrón de sueño vigilia, síntomas motores extrapiramidales como rigidez y sacudidas mioclónicas, hasta una etapa final de mutismo y postración. Tratamiento: Inhibidores de la Acetilcolinesterasa: Donepecilo, rivasigmina, galantamina. Inhibidores de los receptores NMDA: Memantina. Estabilizadores de membrana: Lamotrigina, carbamacepina, topiramato. Delirios y alucinaciones: Risperidona, olanzapina, haloperidol. Depresión y ansiedad: citalopram, paroxetina, sertralina. DEMENCIA VASCULAR: Se ha acuñado el término de daño cognitivo vascular para describir a la pérdida de funciones cognitivas asociada a otras manifestaciones neurológicas y que tienen como base el daño vascular. Factores de riesgo, la edad, el bajo nivel de escolaridad, la enfermedad hipertensiva, el daño cerebral vascular previo, especialmente infartos cerebrales, el padecer diabetes, obesidad, síndrome metabólico, niveles elevados de homocisteína y la ateroesclerosis. Clasificación: Demencia por daño cortical o enfermedad multi infarto relacionada generalmente con el daño de grandes vasos (23%) y la causada por daño subcortical o enfermedad difusa de la sustancia blanca relacionada a daño de pequeños vasos (50%). La región más afectada son los ganglios basales, dentro del daño subcortical se hace referencia a tres tipos de entidades, 1) infartos lacunares (16%), la distribución puede variar, sin embargo el daño se ha descrito con mayor frecuencia en la región frontal, 2) demencia talámica (8%) donde el daño puede venir desde la arteria basilar y 3) la encefalopatía subcortical ateroesclerótica o síndrome de Binswwanger donde hay gran daño a la sustancia blanca. El daño vascular, como ya se comentó puede ser secundario a infartos en grandes vasos corticales o a nivel subcortical predominantemente ya sea por hemorragias o principalmente por eventos vasculares isquémicos donde la hipoperfusión lleva a la gliosis y al daño en la sustancia blanca que posteriormente se manifestará como leucoaraiosis, otro tipo de daño es la isquemia crónica subcortical que da afección principalmente a nivel periventricular. Edad de inicio es en entre los 50 y 59 años, con otro pico entre los 60 y 69 años de edad. Clinica: pueden desarrollan alteraciones en la marcha,

CURSO ENARM CMN SIGLO XXI TEL: 36246001

Pharmed Solutions Institute

PÁGINA 164

MANUAL DE TRABAJO DEL CURSO ENARM CMN SIGLO XXI afectación de primera neurona motora, bradicinesia, rigidez, alteración en los esfínteres, síndrome pseudobulbar, alteraciones de la personalidad y del estado de ánimo, afasia, agnosias y alteraciones en la memoria. Establecer criterios diagnósticos definitivos ha sido complicado, las más utilizadas son las de Hachinski, Rosen. Tratamiento: donepezilo con dosis inicial de 5mg al día y de mantenimiento de 10-20mg después de 4 semanas, rivastigmina con dosis inicial de 1.5mg, incrementando de 1.5mg diarios hasta lograr la dosis de mantenimiento de 6mg o parches de 4.6mg por día con dosis paulatinas con un máximo de 9.6 mg/día. Dado que se trata de una demencia que se puede prevenir, es importante dar tratamiento a los factores de riesgo. DEMENCIA FRONTOTEMPORAL: Tercera demencia más común después de la Demencia tipo Alzheimer. La prevalencia general es de 17,6 por 100.000 habitantes. La edad de presentación es aproximadamente a los 58 años. Afecta por igual a hombre y mujeres. En el 30-50 % de los casos existe una historia familiar de Demencia frontotemporal (DFT) principalmente en la variante conductual, mientras que en la demencia semántica y la afasia progresiva no fluentes menos frecuente. El término de Enfermedad de pick se reserva para casos de DFT con inclusiones positivas teñidas de plata denominados cuerpos de Pick. Debido a su asociación de atrofia en lóbulos temporales y frontales. La mutación característica se encuentra en el gen que codifica a la proteína tau en el cromosoma 17, cuyo patrón es de herencia autosómico dominante. Esta proteína pertenece a la familia de las MAPT (proteínas tau asociadas a microtúbulos) y tiene participación en la regulación de la polimerización y ensamble de las subunidades de tubulina, por lo que es indispensable para determinar la estabilidad y función axonal. La DFT es una enfermedad progresiva con inicio insidioso, en etapas iniciales presentan más síntomas conductuales, sin afectar la memoria ni las capacidades visuoespaciales. Puede ser esporádica o familiar. Pueden presentar diversas alteraciones de acuerdo al síndrome de DFT, sin embargo se caracteriza por la presencia de afasia, alteraciones en la conducta y del lenguaje. Las alteraciones en el lenguaje pueden existir con o sin demencia asociada, aunque se estima que la mayora de los pacientes desarrollara demencia en etapas tardías de la enfermedad. Existe un deterioro de la función ejecutiva que incluye la planificación, organización, flexibilidad, juicio, resolución de problemas con conservación de la percepción visual y habilidades espaciales. Existe también una incapacidad para reconocer emociones, en particular las negativas como la ira, el miedo, tristeza. La orientación en tiempo y lugar, el recuerdo libre demorado son alteraciones frecuentes de la DMF, sin embargo están presentes también durante las primeras etapas de la enfermedad de Alzheimer, su distinción sigue siendo difícil mediante pruebas neuropsicológicas. CASO CLINICO Se trata de masculino de 73 años de edad el cual es originario de Mérida, actualmente vive solo, padece diabetes mellitus desde hace 15 años, hipertensión arterial, triglicéridos, colesterol y acido úrico elevados, al interrogatorio se desconoce la posología para sus padecimientos, refiere que le indicaron glibenclamida, bezafibrato, enalapril, alopurinol y pravastatina, el familiar refiere que desde hace mas de dos años presenta dificultad para recordar las cosas que está haciendo, se ha vuelto desconfiado, esconde las cosas para evitar que se las roben, pobreza de pensamiento, pensamiento con bloqueo e ideas de daño y robo. PREGUNTA Debido a las múltiples entidades nosológicas que presenta el paciente cual es complicación neurológica más probable. RESPUESTA a.- Demencia vascular es la más probable. b.- Demencia de Alzheimer es la más probable. c.- La demencia por priones es la más probable. c.- La demencia de Pick es la más probable. CASO CLINICO Masculino con neumonía por aspiración, con antecedente de pérdida progresiva de funciones mentales tipo demencia, previamente presento cambio de conducta, pérdida de peso y cefalea continua de tres meses de duración. A la EF alerta, mutista, el examen neurológico no revela alteraciones sensoriales, pares craneales sin afecciones sin embargo presenta mioclonos a la estimulación y expontaneos incluyendo durante el sueño, los estudio de laboratorio no muestran datos de importancia, el liquido cefalorraquídeo normal al igual que la TAC de cabeza. PREGUNTA Cuál es el agente causal más probable en este caso. RESPUESTA a.- Viral ADN. b.- Viral ARN. c.- Fungico. d.- Prion.

CURSO ENARM CMN SIGLO XXI TEL: 36246001

Pharmed Solutions Institute

PÁGINA 165

MANUAL DE TRABAJO DEL CURSO ENARM CMN SIGLO XXI EPILEPSIA: CIENCIAS BASICAS: Se define como la tendencia a tener convulsiones recurrentes. La epilepsia es una manifestación de la enfermedad cerebral subyacente. Convulsiones simples o aquellos que ocurren durante la enfermedad aguda no deben ser clasificados como epilepsia. Causas: Desconocido en dos tercios de los casos .En Reino Unido, las encuestas de la comunidad muestran: Enfermedad cerebrovascular, 15 %; tumor cerebral, 6 %; relacionados con el alcohol, 6 %; post-traumático, 2 %; trastornos genéticos, 1 %. Otras causas incluyen la esclerosis del hipocampo y malformaciones corticales y CLASIFICACION DE EPILEPSIA vascular. En los trópicos, la neurocisticercosis es una causa común. SALUD EPILEPSIAS GENERALIZADAS Y SÍNDROMES: Idiopáticas de aparición relacionada con la edad PUBLICA: 50/100 000/año. 1 de cada 200 tienen epilepsia activa (en el Reino a)Epilepsia de ausencia infantil Unido, 350.000) .Mayor incidencia en los países en desarrollo. En México las b)Epilepsia mioclonica juvenil (JME) crisis convulsivas de origen tardío son en su mayoría debidas a cisticercosis. c)Epilepsia con crisis tónico-clónica CLASIFICACION: Básica en generalizada (50 %) y focal (50 %), subdividido en Sintomáticas y criptogénicas a)Síndrome de West categorías etiológicos: Idiopática (predisposición genética con el desarrollo b)Síndrome de Lennox- Gastaut normal, el examen y EEG). Sintomático (anomalía estructural). Criptogénicas c)Epilepsias con ausencias mioclónicas (anomalía estructural supone, pero no probado). Subdivisiones: 1. Parcial Sintomáticas simple; alteración motora, sensitiva, autonómica o psíquica sin alteración del a)Encefalopatía mioclónica EPILEPSIAS FOCALES Y SÍNDROMES estado de conciencia. 2. Parcial compleja; alteración del estado de Idiopática de aparición relacionada con la edad conciencia, puede comenzar con pródromos o con síntomas motores, a)Epilepsia infantil benigna con puntas centrotemporales sensitivos, autonómicos o psíquicos, presenta automatismos y es seguida b)Epilepsia de lectura por un periodo de confusión (estado posictal). 3. Parcial secundariamente Sintomática: generalizada: Inicia con alteraciones motoras, sensitivas, autonómicas o a)Epilepsia con convulsiones parciales simples, complejas o secundariamente generalizadas, que surgen de cualquier parte de psíquicas, seguidas de pérdida del estado de conciencia, con aumento de la corteza tono muscular, contracciones rítmicas (clónicas), al final paciente comatoso b)Epilepsia parcial continua y recuperación lenta. Puede haber incontinencia y/o mordida de lengua. 4. c)Síndrome caracterizado por la activación especifica Generalizada de ausencia: Inicio súbito con periodo breve de desconexión Epilepsias EPILEPSIAS INDETERMINADAS (FOCAL O GENERALIZADA) con el medio y recuperación rápida, tono muscular puede aumentar o Epilepsias con puntas continuas y actividad de las ondas en el disminuir, se acompaña de automatismos o movimientos clónicos leves. 5. sueño Generalizada tónico-clónica: Perdida súbita del estado de alerta, acompañada de contracciones tónico clónica generalizadas y hay estado posictal. STATUS EPILEPTICO: Presencia de una crisis convulsiva continua o crisis repetitivas, discretas y con alteración del estado de conciencia en el periodo interictal, el límite de tiempo es de 5 min. La causa más frecuente es abstinencia a fármacos anticonvulsivantes o falta de apego al tratamiento. PATOGENIA: Es producida por el disparo sincrónico y sostenido de una población de neuronas cerebrales. Las estructuras más susceptibles al desarrollo de crisis convulsivas recurrentes son la corteza motora y la formación del hipocampo y complejo amigdalino del sistema límbico. DIAGNOSTICO: Clínico; Los ataques son estereotipados, eventos paroxísticos. El diagnóstico es clínico, testimonios son cruciales. Por lo general, seguido de un período de somnolencia. Ver ' La pérdida de la conciencia ". Desencadenantes incluyen: alcohol, la fatiga, privación del sueño, infecciones, hipoglucemia, el estrés, luces estroboscópicas (epilepsia fotosensible), lectura, el agua caliente (raro). Ausencias infantiles; raras después de 10 años de edad. F> M. Breve pérdida de la conciencia varias veces al día, accionado por la hiperventilación, remiten en la edad adulta. EEG característico; 3 Hz puntas y olas, sin fotosensibilidad. Epilepsia mioclónica juvenil (JME): inicio antes de los 30 años, sacudidas mioclónicas por la mañana, ausencias típicas, convulsiones tónico-clónicas generalizadas. EEG típico generalizado con puntas y ondas puede haber o no fotosensibilidad, remisión raro. Las convulsiones parciales: complejas asociado con anormalidad estructural que subyace, por ejemplo, esclerosis del hipocampo, automatismos (relamerse los labios, la masticación, la deglución, movimientos manuales estereotipados). TIPO DE EPILEPSIA 1RA. LINEA 2DA. LINEA Déjà vu y jamais vu, auras olfativas (desagradable). Comportamiento Generalizadas Valproato (200mg aumentar Levotiracetam o inusual o emocionalidad. En aquellos pacientes con epilepsia c/2 sem. maximo 2.5 grs) o Topiramato previamente diagnosticada y que presenta una crisis epiléptica, se Lamotrigina (25 mg/d, (25mg/d máximo deberán revisar los niveles séricos del fármaco, obtener electrlitos, máximo 400mgs) 400mg/d) Focal Carbamacepina (200mg/d) o Levotiracetam o BH, pruebas de función hepática y renal, así como panel toxicológico, Lamotrigina o Topiramato si todo sale normal, pensar en la modificación terapéutica, aumentar Valproato dosis máxima, disminuir dosis de 1er fármaco e iniciar un segundo. Mioclonicas, Valproato o Levotiracetam Pacientes que no tengan historia previa de epilepsia, solicitar BH, atónicas Lamotrigina (250mgs/d) electrolitos, calcio, magnesio, glucosa, función hepática renal, EGO y Ausencia Etosuximida Valproato o panel de toxicología, si salen normal o negativo hacer RMN o electroencefalograma, para buscar causas focales de las crisis (tumor, EVC, neuroinfección, trauma, enfermedad degenerativa) y tratar la causa específica así como considerar tratamiento anticonvulsivo. Si todo sale normal se puede pensar en crisis idiopáticas. TRATAMIENTO: Recomendaciones generales; Evite las actividades peligrosas, por ejemplo, nadar solo, el montañismo. Tome duchas en lugar de baños. Convulsiones simples: No hay un tratamiento a menos que exista un alto riesgo de recurrencia, por ejemplo, EEG anormal como en JME o un MRI anormal. Si los factores precipitantes (por ejemplo, alcohol), la evitación pueden prevenir la recurrencia. Después de un solo ataque no provocado, el riesgo de recurrencia es del 24%, sin causa y EEG normal y 65 % si se asocia con una anormalidad neurológica + EEG anormal. Profilaxis: No hay indicación para iniciar el tratamiento en pacientes con lesiones en la cabeza, craneotomía, tumores cerebrales, a menos que se presentan convulsiones. La resistencia al tratamiento farmacológico se presenta hasta en 20% de los casos. En focos epileptógenos bien definidos se debe considerar el tratamiento quirúrgico con resección del mismo. Estatus epiléptico: Minutos 0‐5 Documentación clínica. Diagnóstico correcto. Identificación del tipo de SE. Breve anamnesis y exploración física. Medidas básicas: Mantener función cardiorrespiratoria. Monitorización de constantes vitales. Vía IV permeable, realización de exámenes analíticos y administración de antiepilépticos. Si es posible, documentación EEG. Minuto 6‐10 Según sospecha clínica (etilismo, déficit nutricional, hipoglicemia). Tiamina IM (100mg). Glucosa IV (50 cc al 50% adulto, 2‐4 cc/kg al 25% niño). Finalización del SE clínico y eléctrico (tratamiento farmacológico).

CURSO ENARM CMN SIGLO XXI TEL: 36246001

Pharmed Solutions Institute

PÁGINA 166

MANUAL DE TRABAJO DEL CURSO ENARM CMN SIGLO XXI Diazepam IV (2mg/min hasta que cedan, máximo 0.3 mg/kg).Loracepam IV (0.1 mg/kg, ritmo 2mg/min hasta que cedan, máximo 10m. Pueden constituir el único tratamiento si las crisis ceden y la causa del SE es corregida. Si no es así, asociar; fenitoína IV (15a20 mg/kg, en suero salino, ritmo 50 mg/min en adultos y 1 mg/kg/min en niños). Minuto 30-40 Status resistente, ingreso en UCI, opciones: Diazepam, en perfusión continúa (100mg en 500cc glucosado 5%, ritmo de 40 cc/hr). Fenobarbital IV (20 mg/kg, ritmo 50‐100 mg/min). Fenitoína IV (hasta 30 mg/kg, dosis añadida de 510 mg/kg). Ácido Valproico 20mg/Kg. Minuto 60: SE refractario. Ingreso en UCI (intubación orotraqueal, ventilación mecánica, acceso vascular central, monitorización EEG. Anestesia general, elegir una opción: Midazolam IV/Propofol IV. Barbitúricos (pentobarbital, tiopental).Otros (clonazepam, clormetiazol, lidocaína, valproato sódico, paraldehido, agentes inhalantes, bloqueantes neuromusculares). Minuto 0‐60/90: Diagnóstico y tratamiento etiológico. Completar amnamnesis y exploración física detalladas. Exámenes complementarios (TAC, IRM, LCR), pero nunca posponer el tratamien Decidir tratamiento etiológico o de factores precipitantes. Corrección y prevención de complicaciones sistémicas. Hipoglicemia, alteraciones hidroelectrolíticas y del equilibrio ácidobase, hipotensión, hipoxia, edema cerebral, HIC, fiebre. Prevenir la recidiva del SE Tratamiento farmacológico antiepiléptico de mantenimiento. COMPLICACIONES DEL STATUS EPILÉPTICO: Neurológicas Encefalopatía hipóxica, Edema cerebral, Hipertensión endocraneal, Trombosis senos venosos, Vasculares (infartos, hemorragias) Cardiovasculares y respiratorias Hipertensión / Hipotensión, arritmias cardiacas, parada cardiaca, insuficiencia respiratoria / cardiaca, Edema pulmonar Endocrino‐metabólicas Acidosis metabólica / respiratoria, Hiperglicemia / Hipoglicemia, Alteraciones de la función endocrina, Deshidratación, Hiponatremia, Insuficiencia renal aguda, Pancreatitis aguda, Falla hepática Otras Coagulación intravascular diseminada. CASO CLINICO Hombre de 62 años, debuta con crisis de disnea, visión borrosa, caída al suelo, movimientos repetitivos de los brazos y estado confusional de duración total de 10 minutos. Fue tratado con fenitoína y ácido valproico con lo que permaneció estable, presentando sólo 3 años después nueva crisis con similares características. Comienza a notar cambios conductuales, desinterés, irritabilidad y comportamiento obsesivo. Se hospitaliza por cuadro de compromiso de conciencia progresivo de varios días de evolución, agitación psicomotora, desorientación, desviación de la mirada hacia arriba y derecha, temblor del mentón y parpadeo. Al ingreso se encontró además distonías de las extremidades y mantención espontánea de posturas de apariencia catatónica, fenómenos que cedieron con el uso de diazepam endovenoso. Durante esta hospitalización presentó varios otros episodios catatoniformes que duraban horas o días y que en algunas oportunidades se asociaron a mioclonías faciales y de la mano derecha. El EEG se mantuvo persistentemente alterado con actividad lenta generalizada. Se interpretó clínicamente como un cuadro epiléptico, con buena respuesta a los anticonvulsivantes. El estudio imagenológico mostró una atrofia frontal bilateral. El estudio de laboratorio obtuvo una amonemia alta (44 µmol/L) que fue interpretada como secundaria al uso de valproato, por lo que debió modificarse el tratamiento anticonvulsivante. PREGUNTA La asociación de demencia y epilepsia obliga a pensar en diagnósticos diferenciales, cual de los siguientes es más frecuente? RESPUESTA a.- Porfiria aguda intermitente. b.- Neuroacantocitosis. c.- Déficit de vitamina B12. d.- Enfermedad de Lyme. CASO CLINICO Se trata de femenino de 84 años de edad la cual ingresa a consulta por presencia de dificultad para deglutir los alimentos, presentando accesos de tos hemetizante durante las ingestas, y desde hace 48 hs., fiebre, dificultad respiratoria progresiva, somnolencia permanente y expectoración muco purulenta fétida, se observa pérdida total de la independencia funcional, postrada en cama desde hace aproximadamente 3 años, al cuidado permanente de 2 de sus hijas, portadora además de miocardiopatía dilatada, cumpliendo tratamiento en la actualidad con Digoxina, Furosemida, Enalapril, AAS y vasoactivos cerebrales. PREGUNTA Considerando los antecedentes y al cuadro clínico, cual es el diagnostico mas probable? RESPUESTA a.- Demencia vascular. b.- Demencia señil. c.- Demencia alzheimer. d.- Demencia de pick. CASO CLINICO Masculino de 69 años de edad con antecedentes de HTA de carácter leve -moderada desde hace 8 años aproximadamente además de Hipercolesterolemia desde hace 10 años según refiere, para lo cual ingiere como medicación, Enalapril 10 mg e hidroclorotiazida 12,5 mg una vez por día, complementando dicho tratamiento con dieta hiposódica e hipocolesterolemica. Manifiesta que no cumple estrictamente con la dieta descripta pero que siempre ingirió la medicación antihipertensiva. Consulta el día de hoy por cefalea a predominio frontal, la cual se manifiesta desde hace varios días acompañado de sensación de inestabilidad que no sabe referir exactamente pero si manifiesta sentirse muy extraño, situación que no es habitual en el pero, que lo inquieta y preocupa por lo cual decidió concurrir a la consulta medica, por propios medios. Manifiesta que últimamente no controló su T A como se le había indicado y que tampoco respetó la dieta habitual. Como antecedente importante refiere hipercolesterolemia leve. Además expresa que siempre

CURSO ENARM CMN SIGLO XXI TEL: 36246001

Pharmed Solutions Institute

PÁGINA 167

MANUAL DE TRABAJO DEL CURSO ENARM CMN SIGLO XXI llevó una vida activa, que solía concurrir desde el pueblo hasta su domicilio rural en bicicleta hasta hace unos 10 años aproximadamente. PREGUNTA Considerando los antecedentes y al cuadro clínico, cual es el diagnostico mas probable? RESPUESTA a.- Demencia vascular. b.- Demencia señil. c.- Demencia alzheimer. d.- Demencia de pick. CASO CLINICO Paciente de 73 años de edad que presenta temblor de reposo de predominio en miembro superior derecho, rigidez, bradicinesia y alteración de reflejos posturales. En la última visita, la familia que lo acompaña relata que a lo largo de los últimos 6-8 meses, lo vienen notando triste, poco comunicativo, "perezoso" y con escasa ilusión a la hora de iniciar actividades nuevas. Han notado que pasa horas ensimismado, sin llegar a concluir la tarea que estuviera realizando. Se muestra incapaz de programar las actividades que ha de realizar al día siguiente o las realiza a destiempo, de forma desorganizada. Parece mostrarse poco reactivo ante los problemas, disgustos o alegrías cotidianas, como si nada le importara. Se muestra olvidadizo, tiende a usar dietarios y notas cada vez con mayor asiduidad, sin que esto haya impedido el haber olvidado citas importantes o tareas inexcusables en su negocio. PREGUNTA Considerando los antecedentes y al cuadro clínico, cual es el diagnostico mas probable? RESPUESTA a.- Demencia por parkinson. b.- Demencia por prionopatia. c.- Demencia alzheimer. d.- Demencia de frontotemporal. CASO CLINICO Mujer de 56 años, con 12 años de escolaridad, diestra, dueña de casa, separada, con una convivencia estable. Inicia un delirio paranoide pero asociado a una gran pasividad (era necesario escogerle la ropa, abandonó las labores domésticas). Se presento descontrol urinario ocasional. En el examen mostraba una enferma vigil, prescindente, sin contacto visual, que no expresaba afectos y colaboraba poco al examen (lo que impidió evaluar el campo visual y la sensibilidad). Existía gran hipokinesia pero sin rigidez ni movimientos anormales; la metría era normal. Existía enganche digital pero no prehensión forzada ni succión. En el Minimental test de Folstein (MM) obtuvo sólo 10/30 puntos, fracasando en Orientación, Atención y cálculo, Lectoescritura y Dibujo, con respeto relativo de la memoria. Al preguntarle la fecha dijo "a ver... a ver... a ver"; lo mismo respondió al pedirle restas. Fue capaz de leer la orden escrita, pero no la cumplió; en vez de una frase escribió su nombre, y el pedirle copiar el dibujo recibió el lápiz pero no rindió, limitándose a leer una y otra vez fragmentos del test. En la prueba de Matrices Progresivas Coloreadas (MPC) repetía las instrucciones pero no rendía y musitaba en forma ininteligible; obtuvo 0/12 puntos. En Aprendizaje verbal (AV) decía "ya..." y se quedaba mirando al frente, con rostro inexpresivo, sin repetir las palabras propuestas. En Memoria semántica (MS) evocó 2 nombres de animales en 60 segundos. PREGUNTA Considerando los antecedentes y al cuadro clínico, cual es el diagnostico mas probable? RESPUESTA a.- Demencia por parkinson. b.- Demencia por prionopatia. c.- Demencia alzheimer. d.- Demencia de frontotemporal. CASO CLINICO Femenino de 47 años acude por fallas de la memoria, errores en la denominación y en actividades domésticas. Aparecieron cambios de conducta, se puso:"confianzuda", caprichosa, golosa, floja. Se negaba a colaborar en actividades domésticas, e insistía en forma majadera en diversos temas. Meses después fue evaluada con mayor detalle. En su casa era independiente para vestirse, alimentarse y asearse, pero a veces se orinaba. No era capaz de retener informaciones nuevas, a veces no reconocía a sus familiares, y mostraba una proximidad excesiva frente a los extraños. Tenía pérdida de libido y se mostraba inquieta e irritable. Se mostraba ágil y el examen neurológico básico era normal, no existían reflejos primitivos. Su actitud era inadecuada por animo subido, proximidad excesiva y gran tendencia al payaseo. Por ejemplo, en una oportunidad interrumpió el examen diciendo "¡apúrate abuelito por favor! porque tengo que ir a hacerle la comida a mi marido". No mostraba ninguna preocupación por sus rendimientos y con frecuencia no colaboraba; al pedirle interpretar proverbios sólo dijo "¡no sé!". Al denominar cometía errores caprichosos, pero que sugerían una afasia amnésica. Por ejemplo, ante el dibujo de un paraguas dijo riendo "¡es un sostén!" y otra vez "¡es una bicicleta!", pero ante el dibujo de un sofá dijo "es para sentarse". No existían defectos fonológicos ni morfosintácticos. Al pedirle interpretar el proverbio "a quien madruga Dios le ayuda" dijo "¿a mi marido? No, mi marido tiene negocio y yo trabajo con él, yo de cajera y él de vendedor...". Al dictarle "bajó a la tierra" lo repitió bien pero escribió -con buena letra- "monono nonino ninino". No existía apraxia constructiva (aunque a veces el dibujo

CURSO ENARM CMN SIGLO XXI TEL: 36246001

Pharmed Solutions Institute

PÁGINA 168

MANUAL DE TRABAJO DEL CURSO ENARM CMN SIGLO XXI era muy descuidado), oral ni ideo-motora; según su familia se vestía bien, pero seleccionando mal su vestimenta. Fracasó en cálculo oral y escrito; existía una agnosia digital visual y desorientación derecha-izquierda -ni siquiera fue capaz de decir cual era su mano derecha. PREGUNTA Considerando los antecedentes y al cuadro clínico, cual es el diagnostico mas probable? RESPUESTA a.- Demencia por parkinson. b.- Demencia por prionopatia. c.- Demencia alzheimer. d.- Demencia de frontotemporal. CEFALEAS Y DOLOR CRANEO-FACIAL SALUD PUBLICA: Mas común prevalencia en las mujeres, 18%, hombres, 6%. Edad de inicio promedio 19 años. 46 % tienen antecedentes familiares. El riesgo de que un niño desarrolle migraña 70% si ambos padres están afectados, el 45%, cuando uno de los padres afectados. Una condición hereditaria dominante rara, migraña hemipléjica familiar, debido a una mutación en el cromosoma 19 que codifica para una subunidad del canal de calcio dependiente de voltaje. La arteriopatía cerebral autosómica dominante con infartos subcorticales y leucoencefalopatía (CADASIL) puede presentarse con migraña hemipléjica y progreso a una encefalopatía isquémica. PATOGENIA: La migraña es un trastorno neurovascular en un individuo genéticamente predispúesto. Predisposición es una inestabilidad dentro de la red trigéminovascular originada en el tronco cerebral, en particular, el mesencéfalo dorsal y dorsolateral puente de Varolio. Proyecciones difusas del locus ceruleus y de la corteza cerebral resultan en deterioro del flujo sanguíneo cortical cerebral que causa la propagación de la depresión asociada con auras migrañosas. DIAGNOSTICO: Clínico; La migraña es una cefalea episódica generalmente asociada con náuseas (± vómitos) y fotofobia. Puede estar precedida por síntomas neurológicos focales. El aura no necesariamente puede ser seguida por dolor de cabeza (anteriormente conocido como equivalentes de migraña). 30 % puede tener otros dolores de cabeza que coexisten, por ejemplo tensión y dolores de cabeza por uso excesivo analgésicos. Características de dolor de cabeza: Unilateral en 2/3 de los pacientes y bilateral en 1/3. Dolor que se siente detrás o a lo largo del ángulo interno del ojo o regiones frontotemporal. Irradia a espalda, occipucio o el cuello. Sitio de dolor de cabeza puede ser tanto ipsilateral o contralateral a la perturbación neurológica focal. Algunos pacientes pueden quejarse de dolor del miembro ipsilateral del lado del dolor de cabeza. Carácter del dolor de cabeza es aburrido en el inicio y más tarde palpitante (que aumenta con cada pulsación). Otros pacientes sólo pueden describir un dolor de cabeza constante, o incluso un ligero dolor de cabeza embotada. Empeora con el movimiento. Características Aura: Auras visuales incluyen: alucinaciones visuales, escotomas, y espectros de fortificación (zig-zag líneas se asemejan a una muralla cuando se ve desde arriba). Por lo general, blanco y el brillo se mueven a través del campo visual, dejando un área de discapacidad escotomas centellantes. Otros fenómenos visuales incluyen destellos de luz (fotopsia). Nota: la epilepsia del lóbulo occipital provoca alucinaciones que son circulares o de formas geométricas y colores. Auras sensoriales son, por ejemplo parestesias generalmente positiva en lugar de adormecimiento, que se reparten en minutos u horas (5 %). Otras auras: hemiparesia (de minutos u horas), disfasia, alucinaciones olfativas y gustativas, y la distorsión de las partes del cuerpo, tales como inflamación de la lengua. Disparadores de migraña: El estrés y la relajación después de estrés. Falta o el exceso de sueño acostumbrados. Trauma (especialmente en niños).Estimulación sensorial: reflejos, luces parpadeantes, los olores. Los hábitos alimentarios y la alimentación: falta de una comida (hipoglucemia). Los alimentos como el vino tinto, queso, chocolate. Aditivos alimentarios: el glutamato monosódico. Ejercicio. El exceso de calor y la deshidratación. Drogas: vasodilatadores como la nitroglicerina. Variantes de migraña: Migraña vertebrobasilar; síntomas del tronco cerebral: diplopía, vértigo, incoordinación, ataxia y disartria se producen en posteriores ataques de migraña. También puede ser desmayos o pérdida de consciencia debido a la participación de la formación reticular del cerebro medio. En casos severos un estado de estupor o coma puede durar una semana (estupor migraña). La mayoría de los casos están asociados con otros síntomas vertebrobasilar. Migraña oftalmopléjica: Paresia extra-ocular el tercer nervio es el más afectado. Paresia puede durar días o semanas. Excluir una lesión compresiva tales como aneurisma de la arteria comunicante posterior. Migraña retiniana: Resultado de la constricción de las arteriolas retinianas, daña la visión en un ojo y se asocia con dolor de cabeza por detrás del mismo ojo. Vértigo benigno recurrente. Migraña que tiene anormalidades del sistema vestibular. Ataques de vértigo acompañadas por tinnitus, sordera y cefalea que puede responder con terapia antimigrañosa. Criterios abreviados de Sociedad Internacional de Cefaleas (IHS) para la migraña: Migraña sin aura; a) Dolor de cabeza una duración de 4 horas hasta 3 días. b) Náuseas/vómitos y/o sensibilidad a la luz y el ruido. c) Dos de los siguientes: dolor unilateral dolor moderado o grave intensidad, agravación por la actividad física sencilla, dolor palpitante. Migraña con aura; Al menos 3 de los siguientes a) tronco cerebral focal reversible o disfunción cortical. c) aura se desarrolla durante > 4 minutos o 2 auras sucesivas. d) cada aura < 60 min. e) cefalea < 60 min siguiente aura. Los criterios sugeridos para la migraña crónica o transformada: diario o casi a diario (> 15 días / mes) dolor de cabeza > 1 mes. Promedio de duración de la cefalea > 4 horas / día (sin tratar). Al menos uno de las siguientes: antecedentes de migraña de IHS. Historia de aumento de frecuencia de cefalea con la disminución de la gravedad de características migrañosas durante al menos 3 meses. Ataques superpuestas de dolor de cabeza que cumpla con todos los criterios de la IHS, excepto duración. MIGRAÑA EN EL EMBARAZO: Consejos para no toma de drogas. Para evitar las náuseas y los vómitos relacionados con el embarazo como resultado hipoglucemia y deshidratación: comer pequeños bocados carbohidratos frecuentes, ingesta adecuada de líquidos. Descanso adecuado. Minimizar la exposición al fármaco, especialmente durante el primer trimestre. Si es posible, deje los medicamentos profilácticos. Tratamiento agudo: paracetamol, en todos los trimestres y durante la lactancia, aspirina: probablemente seguro pero la precaución a corto plazo debido a ↑ riesgo de hemorragia postparto, hemorragia neonatal, y cierre prematuro del ductus arterioso. Evitar durante la lactancia de riesgo del síndrome de Reye y el sangrado en la mortalidad infantil. triptanos, no se recomienda,preparaciones de ergotamina contraindicado en el embarazo y la lactancia. MIGRAÑA MENSTRUAL: Gatillo hormonal es la exposición a los altos niveles de estrógenos seguido por una caída en los niveles. La liberación de prostaglandinas uterinas que ocurren alrededor de la menstruación un mecanismo adicional. 60 % de las mujeres reportan un aumento en la frecuencia

CURSO ENARM CMN SIGLO XXI TEL: 36246001

Pharmed Solutions Institute

PÁGINA 169

MANUAL DE TRABAJO DEL CURSO ENARM CMN SIGLO XXI de la migraña en torno a la menstruación. 14 % tiene exclusivamente migraña relacionada con la menstruación. Ácido mefenámico 500 mg 3-4 veces al día o naproxeno 500 mg dos veces 1-2 días antes de dolor de cabeza y de la duración del período (de -2 a 3 días de la menstruación). Ergotamina, 1 mg (o 1/2 supositorios) durante el periodo vulnerable. Naratriptán, 1 mg bd o frovatriptán 2,5 mg bd durante 3-5 días. TRATAMIENTO: Analgesia simple con antieméticos: si las náuseas y los vómitos no son un síntoma importante como la motilidad gástrica se ve afectada durante un ataque de migraña. La aspirina, 900-1200 mg (disuelto) + metoclopramida, 10 mg o domperidona, 10 - 20mg. Los medicamentos alternativos incluyen paracetamol, 1000 mg y los AINE por ejemplo, ibuprofeno, naproxeno, diclofenaco. Triptanos: Todas las drogas de esta clase (sumatriptán 50-100mgs, zolmitriptán 2.5mgs, naratriptán 2.5mg, rizatriptán 5-10mgs) tienen una alta eficacia con un máximo de 70 % que tiene una respuesta dentro de 2 horas y el 40 % es libre de dolor a las 2 horas. Zolmitriptan y rizatriptán disponibles como obleas no tienen una acción más rápida. Sumatriptan disponible como aerosol nasal e inyección. Las drogas funcionan mejor cuando se toma temprano, pero no durante el pródromo o de la fase de aura. Recurrencia de la cefalea a las 12-24 horas se produce en el 30%. Consejo habitual es tomar una dosis adicional de los triptanos, quizás combinarlo con un AINE. Si no hay respuesta intente otra triptanos. El uso excesivo puede dar lugar a dolores de cabeza de rebote de 10 %. Contraindicaciones: enfermedad arterial coronaria, enfermedad cerebrovascular, hipertensión no controlada, enfermedad vascular periférica, insuficiencia hepática significativa, y el embarazo. Efectos secundarios: molestias en el pecho o sensación de pesadez, la mandíbula, el hombro y rigidez del cuello, parestesia, fatiga y mareos. Interacciones con otros medicamentos: evitar IMAO. Propranolol ↑ concentraciones séricas de rizatriptán. Por lo tanto, utilizar dosis de 5 mg. El síndrome de la serotonina posible cuando se utiliza con los ISRS. Preparaciones de ergotamina: Aun así un papel para la consideración de tartrato de ergotamina en, aquellos pacientes que no toleran a los agonistas 5 - HT. Solo o en combinación con cafeína 1-2 mg puede administrarse por vía oral en el inicio o puede ser utilizado en aquellos pacientes que tienen síntomas premonitorios tales como ansiedad, bostezo, o la fatiga. También puede ser administrado por inhalador o supositorio. La sobredosificación causa náuseas, dolor de cabeza de rebote, y la vasoconstricción periférica. La dosis máxima recomendada por semana es de 10 mg. Dihidroergotamina (DHE) usaron por vía intravenosa en pacientes con migraña intratable en dosis de 0,3 a 1,0 mg cada 8 horas hasta una dosis total de 10mg. PREVENCION: La profilaxis de la migraña; Dolor de cabeza diario útil para controlar la frecuencia y patrones, por ejemplo, relación de los periodos, los fines de semana, el uso excesivo de analgésicos y de triptanos. La profilaxis ineficaz si el uso excesivo de medicamentos. Evite los desencadenantes dietéticos sólo en el 10 %. Considere la profilaxis si ≥2 ataques al mes o un ataque prolongado que afecta el estilo de vida. Medicamentos profilácticos: betabloqueantes (propanolol, timolol, nadolol, atenolol). Los 5-HT2 antagonistas; pizotifen, ciproheptadina, metisergida, amitriptilina especialmente si la migraña asociada con cefalea tensional. Valproato sódico. Topiramato. Gabapentina. CASO CLINICO Paciente 27 Años, femenino, refiere haber comenzado con cefaleas hace 12 hs, de carácter pulsátil, unilateral, que no cede con AINES, refiere nauseas que atribuye a los medicamentos, esta cefalea es distinta de otros episodios sufridos previamente donde el dolor era como una presión en zonas parietales y nuca. PREGUNTA Cual es el diagnostico mas adecuado en este paciente? RESPUESTA a.- Migraña. b.- Cefalea tipo-tensión. c.- Cefalea en racimos (cluster). d.- Cefalae autonómica trigeminal. CASO CLINICO Paciente masculino, 62 años, antecedentes de HTA, tabaquismo, EPOC, que es traído por la familia quienes refieren que el paciente comenzó hace días a consumir AINES por cefalea de reciente comienzo, posteriormente presentó conductas poco habituales y desde ayer se encuentra desorientado. PREGUNTA Considerando los antecedentes y cuadro clínico, cual de los siguientes son factores es el menos críticos para el pronostico? RESPUESTA a.- Cefalea subaguda y/o cefalea progresiva que empeora en el tiempo (meses). b.- Cefalea Nueva o distinta de la Habitual. c.- Cualquier cefalea de maxima severidad en su inicio. d.- Nueva cefalea en mayores de 40 años. CASO CLINICO Paciente femenino, 42 años que refiere cefalea súbita, intensa, explosiva, “siente que le estalla la cabeza”, refiere haber tenido hace unos días atrás un episodio similar aunque de menor intensidad, que la obligo a guardar cama por unas horas. PREGUNTA Cual es el claso diagnostico mas probable? RESPUESTA a.- A trauma de cráneo y/o cuello. b.- A desórdenes vasculares craneales o cervicales.

CURSO ENARM CMN SIGLO XXI TEL: 36246001

Pharmed Solutions Institute

PÁGINA 170

MANUAL DE TRABAJO DEL CURSO ENARM CMN SIGLO XXI c.- A desorden intracraneal no vascular. d.- A sustancias o a su supresión. CASO CLINICO Paciente de 43 años que acude a urgencias de atención primaria por cefalea. En el último año refiere cefalea basal diaria (Escala Visual del dolor (EVA) 5-6/10) de característica tensional, sin náuseas, fotofobia ni fonofobia, y aunque no limita por completo su actividad ya que se ha acostumbrado a ella, tiene episodios de reagudización más intensos, que la obliga a dejar sus labores, como en esta ocasión. Últimamente estos episodios son más frecuentes, y los relaciona con problemas de salud en un hijo. Refiere además que toma analgésicos prácticamente a diario y por rachas combinando paracetamol, ibuprofeno y lorazepam, desde hace casi año y medio. Además está en tratamiento desde hace 2 años con escitalopram. PREGUNTA Cual es el diagnostico mas probable? RESPUESTA a.- A trauma de cráneo y/o cuello. b.- A desórdenes vasculares craneales o cervicales. c.- A desorden intracraneal no vascular. d.- A sustancias o a su supresión.

CURSO ENARM CMN SIGLO XXI TEL: 36246001

Pharmed Solutions Institute

PÁGINA 171

MANUAL DE TRABAJO DEL CURSO ENARM CMN SIGLO XXI TRASTORNOS DEL MOVIMIENTO Y ATAXIAS: CIENCIAS BASICAS: Los trastornos del movimiento constituyen una patología relativamente frecuente en la práctica médica diaria, tanto neurológica como en Atención Primaria, y se pueden presentar como estados hipercinéticos o hipocinéticos. El prototipo de trastorno hipocinético es la enfermedad de Parkinson y también la representa la patología más frecuente dentro de los trastornos del movimiento. En general, los trastornos del movimiento tienen como sustrato anatómico común los núcleos de los ganglios basales. Además, entre la lista de efectos secundarios relacionados con una gran variedad de CLASIFICACIÓN ETIOLÓGICA DE SÍNDROMES COREICOS fármacos de uso común entre la población, está el desarrollo de trastornos del 1. COREAS HEREDITARIAS movimiento. SALUD PUBLICA: La enfermedad de Parkinson se presenta con la misma Enfermedad de Huntington incidencia en todo el mundo y afecta por igual a ambos sexos. En el 5 al 10% de las Coreoacantocitosis Corea hereditaria benigna personas que sufren dicha patología, ésta aparece antes de los 40 años de edad, en su Corea paroxística familiar forma precoz. La edad media de inicio es aproximadamente de 65 años. Síndrome de Enfermedades metabólicas hereditarias (cistinuria, piernas inquietas afecta al 5-10% de las personas entre 18 y 65 años, y aumenta hasta homocistinuria, fenilcetonuria, galactosemia) el 15-20% en los mayores de esa edad. Enfermedad de Huntington suele iniciarse de 2. COREAS SECUNDARIAS 1. Fármacos: Neurolepticos, levodopa, agonistas forma insidiosa hacia la cuarta década de la vida, tiene una prevalencia de 7-10 dopaminergicos, anticolinérgicos, amanrtadina, casos/100.000 habitantes y afecta por igual a ambos sexos ENFERMEDAD DE antiepilépticos, estimulantes noradrenergicos, PARKINSON: Idiopática, afección degenerativa que combina una bradicinesia corticoides, opiáceos, metoclopramida, asimétrica, típica facies inexpresiva, hipocinesia y rigidez (rueda dentada), algunas antidepresivos, reserpina, estrógenos. 2. Tóxicos: alcohol, monóxido de carbono, manganeso, veces acompañadas de temblor en reposo y alteraciones posturales. El temblor, mercurio. 3. Alteraciones metabólicas, inicialmente intermitente, suele comenzar en una mano para extenderse luego a las hipopglucemia, hiperglucemia. 4. Embarazo. 5. extremidades restantes y, a veces, a los músculos de la cara y del cuello. El temblor Infecciones y parasitosis de SNC: sarampión, clásico de la enfermedad de Parkinson es regular y rítmico. Está presente en reposo y varicela, herpes, toxoplasmosis, VIH, sífilis, tuberculosis. 6.Neoplasias disminuye o desaparece con el movimiento de la zona afectada. El temblor aumenta 3. COREAS DEL DESARROLLO con la ansiedad y cesa durante el sueño. Con frecuencia, el cuadro se acompaña de Corea fisiológica de la infancia alteraciones en el intelecto, aunque de forma leve, lo que incluye afectaciones Discinesia bucolinguofacial del anciano cognitivas, de percepción, de memoria y de expresión. Un 15% puede verse afectado de demencia, sobre todo en los casos en los que la enfermedad se ha iniciado tardíamente. La afección del estado emocional, con tendencia a la depresión, afecta hasta un 40% de los casos. Pueden presentar, en el 40% de los casos, síntomas de disfunción vegetativa, que incluyen estreñimiento, a veces grave, hiperhidrosis, sofocaciones y alteraciones en la micción (incontinencia, polaquiuria y enuresis nocturna). La sialorrea es frecuente y se debe al defecto de la deglución. Los pacientes pueden afectarse de forma diferente debido a una combinación de factores genéticos y ambientales, entre los que podemos citar: virus, toxinas, el consumo de aguas no potables, la vitamina E y el hábito tabáquico (el tabaquismo no influye de forma negativa al igual que el consumo de vitamina E), una vez más, presente, como responsable de la puesta en marcha de un pro- ceso patológico. Los familiares en primer grado de consanguinidad, duplican el riesgo de desarrollar la enfermedad (un 17% de probabilidad, a lo largo de su vida), comparándolos con la población general. Los criterios de diagnóstico clínico tienen una sensibilidad de 80% y una especificidad del 30%, comparado con el patrón de oro del diagnóstico de la autopsia. La patología primaria consiste en la pérdida progresiva de células de la sustancia negra del tallo cerebral donde se produce el neurotransmisor dopamina. El tratamiento está obligado a reemplazar la pérdida de dopamina. También se ven afectados varios sistemas de neurotransmisores catecolaminérgicos. Las pruebas, como la RMC, suelen ser normales. Por el contrario, en las pruebas de neuroimagen funcional, como el PET y el SPECT cerebral, se puede observar una reducción asimétrica de la captación estriatal de 18- fluorodopa o de DAT (transportador de dopamina), respectivamente, alteraciones que no se observan en pacientes con temblor esencial o con parkinsonismo inducido por fármacos. El tratamiento médico suele ser eficaz y debe de continuarse durante el resto de la vida del ciudadano. Se fundamenta en las siguientes medidas: Anticolinérgicos (con la intención de corregir la hiperfunción de las células colinérgicas del núcleo estriado y actúan bloqueando los receptores muscarínicos centrales)-Amantadina (posee propiedades anticolinérgicas y dopaminérgicas, ya que estimula la liberación de dopamina en las terminaciones nerviosas) -L-DOPA (fármaco más eficaz, traspasa con facilidad barrear hematoencefálica)-Otros fármacos dopaminérgicos-Medidas generales-Tratamiento quirúrgico PARKINSONISMOS: producidos por factores etiológicos conocidos, por ejemplo medicamentosos, reciben el nombre de secundarios o sintomáticos. Los secundarios incluyen aquellos producidos por infecciones (encefalitis letárgica), intoxicaciones (manganeso, monóxido de carbono, MPTP), administración de fármacos (neurolépticos, cinarizina), tumores, traumatismos, infartos cerebrales profundos y calcificaciones de los ganglios basales. Un tercer grupo en el que los síntomas del parkinsonismo aparecen en el contexto de otra enfermedad neurológica y se asocian a otros síntomas de disfunción neurológica se denominan parkinsonismos plus, con síntomas extrapiramidales, entre los que se encuentra la parálisis supranuclear progresiva, que se caracteriza por una oftalmoplejia supranuclear progresiva, asociada a rigidez distónica de los músculos del cuello y del tronco superior, con alteración de equilibrio y una demencia discreta de tipo frontal. Estos síntomas se inician en la quinta o sexta décadas de la vida y son de evolución progresiva. La falta de equilibrio y las molestias visuales constituyen los síntomas iniciales más frecuentes. SÍNDROME DE PIERNAS INQUIETAS: Trastorno neurológico del movimiento, caracterizado por la aparición de molestias inespecíficas en la parte inferior de las piernas durante el reposo, cuando el paciente está sentado o tumbado, es especialmente frecuente durante el inicio del reposo nocturno, cuando el paciente se acuesta. Consiste en una sensación desagradable y difícil de calificar, en forma de parestesias, disestesias, hormigueos, pinchazos, desasosiego o dolor que lleva al paciente a la necesidad irresistible de mover las piernas («inquietud motriz»), levantarse, pasear, darse masajes e incluso baños con agua fría, en un intento de alivio pasajero. Hay movimientos periódicos en las extremidades, en el que el paciente realiza movimientos bruscos de las piernas, repetitivos y estereotipados, cada 10-60 seg., a veces durante toda la noche, involuntariamente. Al impedir un descanso nocturno adecuado puede manifestarse con cansancio y somnolencia diurna e insomnio. Los fármacos con actividad dopaminérgica y en concreto los agonistas dopaminérgicos (pergolida, pramipexol y ropinirol), se consideran el tratamiento de elección. Ropinirol ha demostrado en varios estudios que reduce los movimientos periódicos de las piernas, y mejora la eficiencia del sueño en relación con el período basal. COREAS: Significa «baile» o «danza». Se utiliza para designar movimientos involuntarios, irregulares, de duración breve y de baja amplitud, que no son predecibles y no tienen una finalidad aparente; fluyen de una parte del cuerpo a otra sin una secuencia definida. Suelen localizarse en las partes distales de los miembros, generalmente en las manos o en la cara, y suelen asociarse a

CURSO ENARM CMN SIGLO XXI TEL: 36246001

Pharmed Solutions Institute

PÁGINA 172

MANUAL DE TRABAJO DEL CURSO ENARM CMN SIGLO XXI hipotonía o a impersistencia motora: dificultad para mantener una contracción muscular sostenida. En las primeras fases, los movimientos coreicos pueden aparecer al final de la ejecución de movimientos normales, pero en fases más avanzadas se hacen muy frecuentes, dispersas y amplias, por lo que terminan siendo muy incapacitantes. Los síndromes coreicos suelen relacionarse con lesiones o alteraciones funcionales del neoestriado (caudado y putamen). Su fisiopatología, desde el punto de vista neuroquímico, responde a una situación de hiperactividad dopaminérgica. Esto viene apoyado por la eficacia terapéutica de los fármacos antidopaminérgicos y por la capacidad de agravar o incluso inducir estos trastornos, que poseen los agonistas de la dopamina. ENFERMEDAD DE HUNTINGTON: Es la forma de corea hereditaria más frecuente. Enfermedad neurodegenerativa que se transmite de forma autosómica dominante con penetrancia completa, precisando un solo gen defectuoso en uno de los progenitores. Los hijos de una persona afectada, tienen el 50% de probabilidad de padecer la enfermedad. Hay atrofia cortical relacionada con el grado de evolución de la enfermedad, y también atrofia estriatal, fundamentalmente a nivel del núcleo caudado. Las neuronas afectadas por el proceso degenerativo son las de mediano tamaño. La disfunción estriatal, contraria a la que caracteriza a la enfermedad de Parkinson, es la responsable de que aparezcan las hipercinesias; y la degeneración neocortical y del núcleo caudado es la responsable de que aparezca demencia. El cuadro clínico incluye síntomas motores, trastornos psiquiátricos y alteraciones cognitivas: Síntomas motores: movimientos coreicos, que afectan inicialmente a la parte distal de las extremidades y posteriormente se van extendiendo a la musculatura craneal, faríngea y laríngea. Las anormalidades en la motilidad ocular, con dificultad para seguir con la mirada un objeto móvil, posturas distónicas axiales y de las extremidades, así como disfagia y disartria coreica por movimientos coreoatetósicos de los labios, la lengua, el paladar y el diafragma. Trastornos psiquiátricos: en ocasiones son la primera manifestación de la enfermedad, cambios en la personalidad, depresión, apatía, agitación, manía, alucinaciones, insomnio. En casos graves es frecuente el suicidio; más del 30% lo intentan durante el desarrollo de la enfermedad. Déficit de la memoria, la atención, la concentración y el aprendizaje; pueden llegar a una demencia con afectación de las funciones frontales: trastorno de la atención y de las funciones ejecutivas, sin afasia, apraxia o agnosia. El diagnóstico de sospecha se realiza mediante la clínica, la historia familiar positiva y la neuroimagen. La confirmación por genética molecular, que tiene una sensibilidad diagnóstica del 98% y puede utilizarse incluso en fase presintomática. No existe un tratamiento específico. Se están realizando ensayos con GABA, baclofen, ACTH y antagonistas selectivos de los receptores D2. COREA DE SYDENHAM: Es un cuadro de corea aguda, que aparece generalmente en la infancia en relación con una infección por el estreptococo betahemolítico. Está considerada como uno de los criterios mayores de la fiebre reumática. Su fisiopatología parece estar relacionada con un proceso autoinmune, en el que los anticuerpos antibacterianos provocan reacciones cruzadas con los antígenos tisulares en el cuerpo estriado. Los síntomas pueden aparecer hasta seis meses después de una infección por el estreptococo, que a veces no se consigue documentar y que hace el diagnóstico más difícil. CASO CLINICO Mujer de 58 años que presenta un cuadro de torpeza generalizada, lentitud y temblor postural en miembros superiores de dos años de evolución. Dos años presenta de forma fluctuante: conducta impulsiva, cambios de carácter con violencia y agresividad, ataques de llanto repentino y comportamiento infantil. Presenta asimismo dificultades en concentración, poca capacidad de comprensión y razonamiento y dificultad para hablar y expresarse. En ocasiones confunde ideas simples, también nombres de objetos y personas. Comienza a perder las rutinas de su vida diaria, mantiene poca relación con el entorno y muestra falta de cuidado personal. En la exploración neurológica se objetiva: clara alteración en prueba de secuencias alternantes y en patrones motores alternantes unilaterales (triple maniobra de Luria), lenguaje muy pobre con carencia de sustantivos y confusiones frecuentes, desorientación parcial en tiempo y espacio, movimientos extraoculares pobres (aunque sin limitación) junto rigidez y bradicinesia generalizadas. PREGUNTA Cual es el diagnostico mas probable? RESPUESTA a.- Enfermedad de Huntington b.- Enfermedad de Sydenham. c.- Enfermedad de Parkinson. d.- Enfermedad de Piernas inquietas. CASO CLINICO Paciente de 76 años que acude a la consulta diciendo que sufre de temblor en las manos, sobre todo la derecha, desde hace meses. Lo presenta de forma constante, No refiere otra sintomatología. El paciente presenta temblor en ambas manos, sobre todo la derecha, de reposo, que mejora con la realización de movimientos voluntarios, inexpresividad facial, dificultad para levantarse del asiento y lentitud al caminar. La movilización de las extremidades muestra aumento del tono en las piernas y rigidez en rueda dentada en los brazos. Se trata, por tanto, de un paciente que consulta por temblor en las manos. PREGUNTA Considerando el diagnostico cual es el tratamiento mas apropiado? RESPUESTA a.- Pergolida. b.- Penicilina. c.- Baclofen. d.- L-dopa.

CURSO ENARM CMN SIGLO XXI TEL: 36246001

Pharmed Solutions Institute

PÁGINA 173

MANUAL DE TRABAJO DEL CURSO ENARM CMN SIGLO XXI CASO CLINICO Se trata de un paciente masculino de 54 años de edad, el cual es referido por presentar intentossuicidas reiterativos (3 intentos) en un lapso de 2 meses. El último de ellos realizado en las últimas horas a su envío (03/05/00), al arrojarse a un pozo, sólo provocándose ligerascontusiones en diversas partes del cuerpo. El padecimiento inicia aproximadamente hace 4 años. De forma paulatina comienza a presentar miedo a la obscuridad, que cada vez aumentaba en intensidad, desde hace dos años se da la aparición de movimientos involuntarios en la frente (gesticulaciones), y sin motivos aparentes comienza con “decaimiento” del estado del ánimo, aislacionismo, mutismo por días, para luego regresar a su estado “normal,” presentó además irritabilidad fácil, así como ideas de desesperanza, refiriendo que su vida no tenía sentido, transcurre aproximadamente un año y seis meses, con poca afectación en su vida laboral, social y de familia, durante este tiempo comienza a presentar algunos movimientos involuntarios en manos y en cuello de tipo espasmódicos, haciéndose más evidentes los movimientos en cara, dichos movimientos se presentan de forma periódica y repetitiva. PREGUNTA Cual es el diagnostico mas probable? RESPUESTA a.- Enfermedad de Huntington b.- Enfermedad de Sydenham. c.- Enfermedad de Parkinson. d.- Enfermedad de Piernas inquietas. ENFERMEDADES NEUROMUSCULARES (ENM): CIENCIAS BASICAS: Las enfermedades neuromusculares son enfermedades de carácter genético y generalmente hereditario que afectan a la musculatura y al sistema nervioso. También se conocen con el nombre de miopatías. Su aparición puede producirse tanto en el nacimiento como en otras etapas de la vida. Las ENM son trastornos de la unidad motora, cuyos principales síntomas son la debilidad muscular, la fatiga, los calambres, el dolor, problemas articulares y la rigidez. Su distribución suele ser simétrica, a diferencia de lo que ocurre en las enfermedades que afectan al sistema nervioso central (SNC). La debilidad de los músculos respiratorios es la causa básica que conduce al fracaso de la bomba ventilatoria, y que se traduce en hipoxemia e hipercapnia. Sin embargo, el compromiso del sistema respiratorio no es igual en todas las entidades sino que está determinado por el grado de afección muscular y por la progresión de la enfermedad. Existen alrededor de 150 tipos de ENM: 1. Distrofias musculares: afectan predominantemente al músculo estriado y son debidas a un defecto alguno de las proteínas que forman parte de la fibra muscular, ya sean estructurales o enzimáticas (ejemplos son la distrofina calpaína, merosina y emerina, entre otras)2. Miopatías distales. 3. Miopatías congénitas. 4. Distrofia miotónica de Steinert. 5. Miotonías congénitas. 6. Parálisis periódicas familiares. 7. Enfermedades musculares inflamatorias. 8. Miositis osificante progresiva. 9. Miopatías metabólicas. 10. Enfermedades de la unión neuromuscular. 11. Amiotrofias espinales. 12. Neuropatías hereditarias sensitivo-motoras (enfermedades de Charcot-Marie-tooth). ESCLEROSIS LATERAL AMIOTRÓFICA (ELA): Tiene una incidencia anual de 1-2 casos/100.000 habitantes y su sustrato patológico es la degeneración de las neuronas motoras de la médula espinal, el tronco cerebral y el córtex motor. Clínicamente se caracteriza por espasticidad e hiperreflexia al inicio de la enfermedad, pero a medida que progresa se establecen otras síntomas, como amiotrofia asimétrica, debilidad muscular, fasciculaciones y síndrome bulbar. La debilidad de los músculos respiratorios, fundamentalmente de los intercostales y del diafragma, es la causa de la hipoventilación, y los síntomas respiratorios aparecen cuando la enfermedad está muy evolucionada, a pesar de que numerosos estudios han comprobado que puede haber alteración de la función ventilatoria incluso cuando la debilidad muscular periférica es ligera. A pesar de ello, lo más frecuente es que la insuficiencia respiratoria se presente en casos de ELA claramente establecida como consecuencia de la progresión natural de la enfermedad o de forma aguda, precipitada por una infección de tracto respiratorio. De forma ocasional, la insuficiencia respiratoria puede ser la primera manifestación de la ELA; en estos casos la lesión afecta fundamentalmente a las motoneuronas del nervio frénico localizadas en la médula cervical, y la afección bulbar implicará un grave compromiso de la vía aérea superior. ESCLEROSIS MÚLTIPLE (EM): Es una enfermedad desmielinizante del SNC, y se considera una de las principales causas de discapacidad neurológica en adultos jóvenes. Como posibles etiologías se han descubierto datos que hacen referencia a factores genéticos y ambientales. Los síntomas comunes de la enfermedad son debilidad muscular, espasticidad, incoordinación motora y pérdida de agudeza visual. Habitualmente esta enfermedad tiene dos formas de presentación clínica: en forma de brotes, en los que puede haber una remisión completa o parcial de los síntomas, o como una enfer medad lentamente progresiva44. En este sentido, el compromiso del sistema respiratorio en la EM45 está en relación con la forma de presentación clínica de la enfermedad, con el grado de debilidad muscular y con las estructuras del SNC que se ven afectadas por la desmielinización. En la mayor serie de pacientes con EM y disfunción respiratoria, las complicaciones respiratorias aparecieron a los 9,5 años de las manifestaciones neurológicas y se atribuyeron a debilidad muscular, disfunción bulbar, trastornos del control respiratorio, hiperventilación paroxística y apneas obstructivas. En las lesiones agudas de la médula espinal, las complicaciones respiratorias dependen de la extensión y la localización de la lesión y se deben a la interrupción de la inervación. Se consideran lesiones altas las que afectan a C1 y C2, y bajas las que se producen entre C3 y C8. Las motoneuronas que inervan al diafragma se originan entre C3 y C5, por lo que las lesiones medulares situadas por encima de C3 implican una parálisis total de la musculatura respiratoria, mientras que en las lesiones entre C3 y C5 la parálisis muscular es parcial. MIASTENIA GRAVIS (MG): Es una enfermedad autoinmune. Se presentan anticuerpos contra el receptor de acetilcolina (AchR) que interrumpen la función de la acetilcolina en la unión neuromuscular presentando en debilidad muscular. Los Músculos bulbar, facial y de las extremidades proximales son los más comúnmente afectados y justifican los síntomas de la debilidad general, ptosis y diplopía. La debilidad en los músculos de la respiración puede llevar a la falla respiratoria la cual es llamada crisis miasténica. Las complicaciones asociadas a falla respiratoria son la causa de muerte en paciente con Miastenia Gravis. La incidencia es de menos de 1 caso por 100 000. Es más común en mujeres durante la segunda y tercera década de la vida. Sin embargo en la Séptima y Octava década de la vida es más común en hombres. Los Afroamericanos también tienen una incidencia alta. La contracción muscular ocurre cuando la acetilcolina es liberada por una fibra nerviosa y se une al AchR de una fibra muscular. En la MG, la fibra nerviosa es normal, sin embargo, el número y función de los AchR nicotínicos del músculo esquelético están disminuidos. Los síntomas aparecen

CURSO ENARM CMN SIGLO XXI TEL: 36246001

Pharmed Solutions Institute

PÁGINA 174

MANUAL DE TRABAJO DEL CURSO ENARM CMN SIGLO XXI cuando el número de AchR esta disminuidos por debajo del 30% de lo normal. Los músculos cardiaco y liso no se afectan por que ellos tiene diferente antigenicidad que el músculo esquelético. La hiperplasia del timo o el timoma se ha visto en el 75% de los pacientes con MG. Por lo tanto, se sospecha que el timo es el sitio de producción de anticuerpos, pero el estímulo que inicia el proceso autoinmune es desconocido. La morbilidad por la debilidad muscular intermitente incluye la neumonía por aspiración y caídas. Los síntomas empeoran en el día progresivamente con el uso repetitivo de grupos musculares como leer o masticar. Los nervios craneales son los más comúnmente afectados, resultando en ptosis, diplopía, debilidad facial, disfagia y disartria. La respuesta pupilar a la luz permanece intacta. La debilidad muscular de las extremidades proximales empeora con el movimiento y mejora con el descanso. Los reflejos tendinosos profundos pueden estar disminuidos pero nunca ausentes. No hay déficit en la función cerebelar y sensorial. La crisis miasténica es el grado más severo de MG por que la debilidad extrema de los músculos respiratorios resulta en falla respiratoria requiriendo soporte ventilatorio. Otras enfermedades que se pueden presentar con síntomas similares son el botulismo, hipotiroidismo y lesiones de la masa intracraneal. La prueba de diagnóstico para MG incluyen la prueba de edrofonium, electromielograma con estimulación repetitiva del nervio y prueba serológica para anticuerpos anti AchR. El cloruro de edrofonium es un inhibidor de la colinesterasa de corta acción que incrementa la cantidad de acetilcolina en la unión neuromuscular. Este incremento vence el bloqueo de los receptores y la ptosis, oftalmoplejía y la debilidad muscular se corrige en 30 segundos pero regresa a lo basal en 1 a 2 minutos. La prueba de anticuerpos contra AchR es la prueba más específica para MG. El aspecto más importante para los médicos del departamento de urgencias es que en todos pacientes gravemente enfermos, la prioridad es establecer y mantener una ruta aérea y asegurar respiración adecuada. La Intubación endotraqueal por la inducción rápida de la sucesión y intubación (IRS) puede ser necesario es antes de ser capaz de diferenciar miastenia gravis contra una crisis de colinergica. El preferido es la Succinilcolina. Los pacientes con MG AchRs resistentes al succinilcolina y dosis más altas (2 mg/kg) debe ser usado para inducir la parálisis. Los agentes de no despolarizantes como (rocuronium o vecuronium) son los agentes paraliticos preferidos. Estos agentes no causan la parálisis prolongada en una dosis de 50% la dosis recomendada. La dosis en adultos de edrofonium es 2 mg IV lento. Sin presentar ninguna respuesta ni ningún efectos adversos de tipo colinergico, administrando 8 mg IV lento para una dosis total de 10 mg, las contraindicaciones del edrofonium es obstrucción gastrointestinal u obstrucción del trecho urinario. DISTROFIA MUSCULAR PROGRESIVA, O ENFERMEDAD DE DUCHENNE: Es una enfermedad de herencia recesiva ligada al cromosoma X causada por mutaciones en el gen de una proteína del citosqueleto: la distrofina. Es una miopatía proximal que comienza en la infancia y en la que, inicialmente, se afectan los músculos de las extremidades inferiores (cintura pelviana), de modo que el paciente refiere dificultad para correr o saltar y, a medida que progresa, afecta a los músculos de las extremidades superiores y del tronco; así, los enfermos a los 12 años de edad son dependientes por completo. También es característico de la enfermedad la miocardiopatía primaria, que cursa de forma asintomática o con trastornos del ritmo cardíaco. La mayoría de estos enfermos presentan un cierto déficit intelectual, y los pacientes suelen fallecer alrededor de los 20 años por complicaciones respiratorias o cardíacas. Muchas enfermedades neuromusculares son genéticas, lo que significa que hay una tendencia familiar o existe una mutación en los genes. Algunas veces, pueden ser provocadas por un trastorno del sistema inmunológico. La mayoría de ellas no tiene cura. El objetivo del tratamiento es mejorar los síntomas, aumentar la movilidad y el lapso de vida. CASO CLINICO Se trata de masculino de 65 años de edad el cual acude debido a que desde hace cuatro meses presenta debilidad distal asimétrica, además refiere contracturas musculares y fasciculaciones que se presentan con los movimientos voluntarios, a la exploración física se observa debilidad extensora de las manos así como dificultad para mover la lengua y la cara, los reflejos de estiramiento muscular se observan incrementados, la exploración de la sensibilidad no se observan alteraciones, al examen mental se encuentra pensamiento y lenguaje con curso y contenido adecuado, humor y afecto conservados, resto de funciones mentales superiores están intactas. PREGUNTA Cuál es el diagnostico clínico más probable. RESPUESTA a.- Esclerosis lateral amniotrofica. b.- Esclerosis multiple. c.- Mielitis transversa. d.- Síndrome de guillain-barre. CASO CLINICO Masculino de 23 años de edad el cual acude a consulta debido a que desde hace un mes presenta debilidad generalizada, dificultad para caminar y alzar objetos, estas manifestaciones predominan en la mañana, durante el interrogatorio refiere dormir bien, no consumo de drogas ó alcohol, no cuenta con enfermedades neurodegenerativas en la familia, actualmente se encuentra terminando la licenciatura pero le preocupa estos síntomas, a la exploración física usted observa ptosis bilateral, y disminución de fuerza generalizada. PREGUNTA Cuál es el diagnostico más probable. RESPUESTA a.- Esclerosis multiple. b.- Esclerosis lateral amniotrofica. c.- Sindrome de Guillain-Barre. d.- Miastenia gravis.

CURSO ENARM CMN SIGLO XXI TEL: 36246001

Pharmed Solutions Institute

PÁGINA 175

MANUAL DE TRABAJO DEL CURSO ENARM CMN SIGLO XXI TRASTORNOS TOXICO-METABOLICOS Y CARENCIALES DEL SISTEMA NERVISOSO: TRASTORNOS TOXICO-METABOLICOS: El sistema nervioso puede verse dañado cuando el fallo de un órgano permite el acúmulo de sustancias tóxicas que en circunstancias normales son eliminadas del organismo. También como consecuencia de drogas o toxinas exógenas o endógenas, disfunción de mecanismos homeostáticos o por el déficit de sustratos indispensables. Las principales herramientas de trabajo serán la historia clínica y la exploración física. El curso de los síntomas (monofásico, progresivo o con recaidas), la presencia de síntomas sistémicos, enfermedades previas o coexistentes así como el uso de fármacos, hábitos dietéticos o tóxicos y la exposición a tóxicos serán de gran importancia en la identificación del síndrome y su causa. Hallazgos de laboratorio, estudios más específicos (autoanticuerpos, estudio del LCR) o pruebas de imagen entre otros nos permitirán completar la aproximación etiológica. Las encefalopatias se desarrollan habitualmente de manera insidiosa y la norma en todas ellas es la alteración del estado mental, en ocasiones de forma tan sutil que puede no detectarse con exploraciones rutinarias. La lesión del sistema reticular y la corteza cerebral inducirán diferentes grados de alteración en la orientación, memoria, percepción, capacidad de concentración, juicio o planificación y ejecución de tareas. Los trastornos metabólicos sistémicos y la exposición a toxinas provocarán más frecuentemente degeneración axonal. La musculatura ocular será la más frecuentemente afectada así como los músculos flexores del cuello y de cinturas. Las enfermedades de los músculos se presentarán con debilidad, dolor y fatiga. Déficits nutricionales y tóxicos exógenos serán los responsables más frecuentes y el mecanismo patogénico en ocasiones está bien definido. ENCEFALOPATIA HIPÓXICO-ISQUÉMICA: El cerebro es un órgano con alto requerimiento metabólico y por ello muy susceptible a daño por deprivación del flujo sanguíneo. Las reservas de glucosa, glucógeno, ATP y posfocreatina se deplecionan a los 10-12 minutos de la isquemia. Después de 15 minutos de isquemia global con parada cardiaca más del 95% del tejido cerebral estará dañado. Más de la mitad de los supervivientes a una parada cardiaca presentan algún grado de daño cerebral permanente. La patogenia del daño cerebral dependerá de la etiología del mismo y el pronóstico del mecanismo implicado. Cualquier mecanismo conduce finalmente a una necrosis y apoptosis neuronal irreversible. Cambios bioquímicos diversos, daño mitocondrial precoz, alteración de citoesqueleto neuronal y la activación de receptores de Glutamato son aspectos funcionales subyacentes al daño celular tras un insulto hipoxico-isquémico cerebral. El daño neuronal puede ocurrir durante el periodo inicial de la isquemia (como consecuencia de los cambios bioquímicos y funcionales), en el periodo de reperfusión (como consecuencia de la formación de radicales libres y daño tóxico continuo), por deterioro del flujo sanguíneo (al reanudarse la circulación espontánea y como consecuencia de una microcirculación alterada) y como daño diferido (secundario a desmielinización). ENCEFALOPATÍA HEPÁTICA: La enfermedad hepática crónica o aguda puede acompañarse de síntomas neuropsiquiátricos (y neuromusculares) conformando distintos grados de encefalopatía. Es aceptado el papel del amonio como factor clave dentro de una cascada de sucesos entre los que aparece alteración de neurotransmisores, excesiva producción de glutamina o estrés oxidativo. El edema de astrocitos resultante se postula como la base fisiopatología subyacente en la forma de HE aguda y crónica. Factores como la hiponatremia o sedantes como las benzodiacepinas influyen en la aparición de encefalopatía hepática incluso con cifras de amonio en sangre normales. El aumento de osmolaridad intracelular causado por hiperamonemia produce reducción de los picos de colina y mioinositol y un aumento en el pico de glutamina y glutamato. Las anomalías metabólicas se correlacionan con la severidad clínica y son reversibles tras tratamiento. Las opciones terapéuticas dependen del estadio e instauración de la encefalopatía hepática. El objetivo será reducir la producción de amonio, aumentar su fijación y excreción, controlar los síntomas neurológicos presentes y modificar el shunt porto-sistémico. Bromocriptina o flumazenilo cayeron en desuso frente a lactulolosa o rifaximina. En ocasiones el trasplante de hígado será la opción terapéutica a considerar. ENCEFALOPATIA URÉMICA: Los signos de encefalopatía en pacientes con enfermedad renal no siempre obedecen al fracaso renal progresivo y pueden sumarse al daño neurológico secundario a tratamientos inmunosupresores, diálisis o trasplante renal. La fisiopatología permanece por determinar. Trastornos del equilibrio acidobase, alteración en las concentraciones de agua y electrolitos, metabolismo anómalo del calcio y glándulas paratiroides o cambios en las concentraciones plasmáticas de distintos neurotransmisores están presentes y determinan los síntomas neurológicos detectables.El tratamiento principal de la encefalopatía urémica es la diálisis. Así mismo existen dos síndromes neurológicos relacionados con ésta, el síndrome del desequilibrio de la diálisis y la demencia de la diálisis. ENCEFALOPATIA HIPOGLUCEMICA: Glucemia menor de 30mg/dl, o periodos prolongados de hipoglucemia, para que se produzca daño irreversible. La intensidad del metabolismo cerebral explica la vulnerabilidad particular del cerebro frente a desórdenes metabólicos. Ante la posibilidad de una disminución importante de glucosa en el SNC, la actividad de los centros cerebrales superiores disminuye para reducir las necesidades de energía. La hipoglucemia provoca respuestas nerviosas y hormonales para aumentar la producción de glucosa en el hígado, y reducir su uso en órganos no nerviosos. Se caracteriza por síntomas de estimulación del sistema nervioso simpático o de SNC, provocados por una concentración plasmática de glucosa anormalmente baja. Los síndromes hipoglucémicos pueden producirse por fármacos o sustancias como insulina, alcohol o sulfonilureas, y con menos frecuencia por salicilatos, propanolol, pentamidina, disopiramida, hipoglicina A. Clínica: confusión, convulsiones, estupor, coma. ENCEFALOPATIA HIPERCAPNICA: Debido a enfermedades respiratorias crónicas retenedoras de CO2, como fibrosis pulmonar, enfisema. Clínica: Presenta los síntomas de hipertensión craneal, cefalea holocraneal o frontal intensa, edema de papila, somnolencia hasta coma. ENCEFALOPATÍA SÉPTICA: Disfunción cerebral difusa o multifocal asociada con una infección sistémica, sin evidencias de infección intracraneal, y que no puede ser atribuida a otros factores tales como efectos farmacológicos o disturbios metabólicos. La encefalopatía de la sepsis puede ser clasificada como encefalopatía séptica o precoz, que se presenta antes de que se produzca la disfunción orgánica múltiple, o encefalopatía tardía, que es acompañada por fallo orgánico múltiple, hipotensión y otros fenómenos sistémicos. Probablemente se origine por la acción de mediadores inflamatorios en el cerebro o por una respuesta citotóxica de las células cerebrales a estos mediadores. Los efectos de la sepsis sobre el cerebro son detectables en cerebros previamente sanos, pero son amplificados en casos con lesión cerebral concomitante, como luego de la injuria traumática o la hemorragia subaracnoidea. La causa más común de encefalopatía en los pacientes con enfermedades médicas críticas, habiendo sido descrita en el 9 al 71% de todos los pacientes críticos que sufren sepsis. TRASTORNOS CARENCIALES. Ante cualquier trastorno del SN adquirido, siempre debe tenerse en consideración una carencia vitamínica ya que el tratamiento sustitutivo administrado al inicio del cuadro puede hacer remitir la sintomatología, mientras que la administración tardía, cuando las lesiones anatomopatologicas ya están establecidas, no evitara que el paciente quede con secuelas. Los síndromes más importantes y frecuentes son debidos a la carencia de las vitaminas de grupo B y en general son secunmdarios a estados carencialesde malnutrición, alcoholismo o patología del sistema digestivo que causen malabsorción. ENCEFALOPATIA DE WERNICKE-KORSAKOFF: Es un síndrome neuropsiquiátrico agudo que aparece como consecuencia del déficit de tiamina (B1) y se asocia con una morbi-mortalidad significativa. Las causas de depleción de tiamina y

CURSO ENARM CMN SIGLO XXI TEL: 36246001

Pharmed Solutions Institute

PÁGINA 176

MANUAL DE TRABAJO DEL CURSO ENARM CMN SIGLO XXI los mecanismos responsables son múltiples. Las lesiones se localizan de forma simétrica a nivel periacueductal, sustancia gris subependimaria del III ventrículo, vermis cerebeloso, tubérculos mamilares, núcleos hipotalámicos, porción medial y dorsal del tálamo o núcleos vestibulares. El cuadro típico clínico de confusión mental con disminución del nivel de conciencia. Puede o no haber parálisis oculomotora (VI) o nistagmo, oftalmoplejía y trastorno de la marcha aparece tan sólo en el 16% de los pacientes. Puede haber crisis comiciales y si no se trata al paciente aparece miosis arreactiva a la luz, síntomas vegetativos, piramidalismo, coma y muerte. Una correcta aproximación facilita el diagnóstico, la prevención en paciente con factores o situación clínica predisponentes y mejora el pronóstico con el adecuado soporte de tiamina parenteral 500mgs en 100ml de SF a pasar en 30 min 3 veces al dia. Los pacientes pueden superar la fase aguda con secuelas como el síndrome amnésico de Korsakov. DEFICIENCIA DE VITAMINA B12: Cianocobalamina, La metilcobalamina actua como factor indispensable para la enzima sintetasa (transformación de homocosteina a metionina), ruta indispensable para el metabolismo del ácido fólico y síntesis del ADN. La metinina es esencial para la síntesis de colina y fosfolípidos. La deficiencia se manifiesta con anemia megaloblastica, glositis, atrofia de mucosa intestinal, ocasionalmente vaginitis. Degeneración combinada subaguda de la medula (alteración de cordones posteriores y laterales, con lo que presenta trastornos sensitivos, parestesias que ascienden desde los pies hasta el tronco, a la que se añade una ataxia), alteraciones cognitivas/demencia (irritabilidad, psicosis), polineuropatia mixta sensitivo-motora. En casos de malabsorción o anemia perniciosa: 1000mg por vía IM diaria durante 2 semanas. Luego seguir con 1000mg IM al mes. DEFICIENCIA DE ACIDO NICOTINICO: Niacina esencial para síntesis de NAD y NADP. La deficiencia de niacina o su precursor el triptófano, provoca la pelagra. En el mundo occidental la pelagra es muy rara y se observa en alcohólicos, en el síndrome carcinoide y enfermedad de Harnup. Clínica: triada clásica es dermatitis, diarrea y demencia, puede haber cuadro neurológico sin manifestaciones cutáneas. De inicio es una encefalopatía inespecífica (síndrome confusional, alteración del nivel de conciencia, ataxia y mioclonias) que evoluciona a demencia. Tratamiento, administrar entre 100-250mg VO tres veces al día durante 5 días. La administración de 325 mg de aspirina media hora antes previene el “flushing” facial. DEFICIT DE VITAMINA E: Debida a defecto de malabsorción intestinal, atresia de vías biliares. Se manifiesta como una degeneración espinocerebelosa (ataxia, síndrome piramidal), más oftalmoplejía externa, retinitis pigmentaria, polineuropatía. Tratamiento administración de vitamina E hidrosoluble por VO o IV. CASO CLINICO Paciente masculino, 42 años de edad, nacionalidad italiana, con antecedentes de tabaquismo, alcohol esporádico, hernioplastía umbilical; biopsia hepática cirrosis hepática; familiar de primer grado con hepatitis de causa no filiada. Motivo de ingreso: cuadro agudo de confusión mas euforia con posterior deterioro del sensorio de 24 hs de evolución más catarsis negativa en la última semana. Al examen físico se constata somnolencia alternada por episodios de excitación, confusión témporo-espacial; flapping +; spiders en cara anterior de tronco, ascitis grado II; edemas en miembros inferiores. Laboratorio de ingreso: Bilirrubina Total: 3.27/B.Directa: 1.87/B.Indirecta: 1.4 / FAL: 741/ GGT: 504/GOT : 180/GPT : 148. TP: 44% Plaquetas: 65.000/mm/ Factor V: 34%. Se solicita estudios para determinar etiología de la misma; Serologías virales (VHC; VHB; HIV; VHA negativos)FAN; ASMA; AMA negativos;Ferremia:169 microg/dL(38-158); Ferritina184 nanogr/ml (8-110);Saturación de Transferrina 98,8%(20-50); Transferrina 171 microg/dL(180-350). Ceruplasmina 18 mg/dL (5-30); Cupruria 24h 424 microgramos (0-60); Lámpara de hendidura; anillos de Kayser- Fleischer. PREGUNTA Considerando el cuadro clínico, que grado de encefalopatía presenta? RESPUESTA a.- Grado I. b.- Grado II. c.- Grado III. d.- Grado IV. CASO CLINICO Mujer de 52 años con antecedentes de alcoholismo. Es traída al Servicio de Urgencia por cuadro de tres días de compromiso de conciencia cuali-cuantitativo, vómitos y diarrea. Al ingreso se constata paciente confusa, inatenta, poco cooperadora. Al examen cognitivo (Mini Mental, Test del Reloj, Test de Generación de Palabras, Test de trecho de digitos) destaca alteración de la atención y memoria episódica, con amnesia anterógrada y retrógrada. El resto de las funciones cognitivas estaban relativamente conservadas. Se observa confabulación durante el examen mental. En el examen físico neurológico se encontró dismetría y nistagmo bilateral. La tomografía computada de cerebro y los parámetros de laboratorio básicos eran normales. PREGUNTA Cual es la conducta terapéutica mas apropiada? RESPUESTA a.- Cianocobalamina. b.- Tiamina. c.- Niacina. d.- Riboflamina. CASO CLINICO Mujer de 60 años sindrome mielodisplásico tipo anemia refractaria simple en tratamiento, presenta cefalea hemicranea izquierda, pulsatil, severa, súbita (2 dias) agrega fotopsias, nauseas y mareo, no relevante, a la EF se observa confusión, desorientación, distractibilidad, alteración de la memoria a corto plazo. La resonancia nuclear magnética cerebral evidencia: Alteración en la intensidad

CURSO ENARM CMN SIGLO XXI TEL: 36246001

Pharmed Solutions Institute

PÁGINA 177

MANUAL DE TRABAJO DEL CURSO ENARM CMN SIGLO XXI de la señal de la sustancia blanca con extensión hacia las fibras U de predominio posterior, principalmente en la región parieto-occipital bilateral y simétrica, con compromiso parcial de la corteza. PREGUNTA Cual es el diagnostico mas probable en este caso? RESPUESTA a.- Enfecefalopatia hipoxico-isquemico. b.- Encefalopatia de wernicke-korsakoff. c.- Encefalopatía hepática. d.- Encefalopatia hipercapnica. CASO CLINICO Paciente del sexo femenino de 26 años de edad, refiere a los 10 minutos de histerorrafia, sentirse mal diciendo que le está dando sueño y que el dolor no ha cedido y la tensión arterial comienza a disminuir y la frecuencia cardiaca, la paciente se encontraba ligeramente cianótica y con tensión arterial de 80/40 torr y frecuencia cardiaca de 60 p.m. procedo a intubar sin aplicación de fármacos inductores ni relajantes y se continuó con la administración de oxígeno al 100% a 5 l/min, observándose movimientos de la bolsa que corresponden a inspiración y espiración del automatismo respiratorio, sin embargo se aplica ventilación asistida. Posteriormente la paciente presenta paro cardiorespiratorio detectado con ausencia del pulso carotídeo, por lo cual se comienza a realizar maniobras de resucitación cardiopulmonar y administramos 1 mg de epinefrina sin respuesta positiva por lo que se procede a administrar otro miligramo más de epinefrina, saliendo del paro la paciente con tensión arterial de 130/90 y frecuencia cardiaca de 150 p.m., comienza a disminuir nuevamente la frecuencia cardiaca y la presión arterial, por lo que administramos 0.6 mg de atropina y dopamina a dosis respuesta; posteriormente, a pesar de tener la paciente tensión arterial de 130/90 torr con apoyo farmacológico con una frecuencia cardiaca de 150 p.m. la paciente continúa con cianosis por lo que decidimos pasarla a terapia intensiva, practicándose una gasometría arterial a su ingreso la cual nos reporta los siguientes parámetros: pH 7.27, PCO2 31, PO2 252, CO2 total 14.90, HCO3 14, EB -11.40, Sat. O2 99.50%. Se practica una tomografía cerebral entre 36 y 48 horas después la cual únicamente observa edema cerebral. Se extuba dos días después de su ingreso con respuesta neurológica Glasgow 7; a las veinticuatro horas, presenta crisis convulsivas tónicoclonicas de una hora de duración por lo que sedan profundamente a la paciente y la intuban nuevamente, para que varios días después se declare coma y lesión cerebral con estado vegetativo. PREGUNTA Cual es el diagnostico mas probable en este caso? RESPUESTA a.- Enfecefalopatia hipoxico-isquemico. b.- Encefalopatia de wernicke-korsakoff. c.- Encefalopatía hepática. d.- Encefalopatia hipercapnica. DEPRESION, ANSIEDAD, ESQUIZOFRENIA Y TRASTORNOS DE ALIMENTACION: DEPRESIÓN: Trastorno del estado de ánimo con repercusión en distintos ámbitos de la persona. Existe una alteración a nivel emocional, con un sentimiento intenso de tristeza, desesperanza, abandono, inutilidad o culpa. En otras ocasiones, ánimo irritable. A nivel del contenido del pensamiento, predomina el pesimismo, con abundantes preocupaciones, ideas negativas repetitivas, angustia y falta de interés con desesperanza en relación al futuro. Desde el punto de vista somático se producen alteraciones del sueño y del apetito, con pérdida de peso, astenia y alteraciones digestivas que son expresión de la disfunción vegetativa. A nivel conductual se produce una disminución del rendimiento, con afectación de la memoria, la atención y la capacidad de concentración, apatía, disminución de la libido y tendencia al aislamiento social. Tanto en el trastorno depresivo como en el síndrome confusional agudo (SCA) puede existir una afectación de las funciones cognitivas con disminución de la atención y del ciclo vigilia-sueño. Cabe destacar la depresión con rasgos psicóticos, donde el paciente pierde el contacto con la realidad, combinando síntomas de depresión y psicosis, con aparición de alucinaciones o delirios. Habitualmente, el contenido de éstos es coherente con la depresión, con una temática relacionada con ideas de culpa, inducción al suicidio, etc. Existen varios tipos de episodios depresivos en función de su intensidad y curso: el trastorno depresivo mayor, que generalmente aparece de forma episódica, y el trastorno distímico, menos intenso y con curso crónico. El episodio depresivo mayor tiende a la recuperación completa, y puede darse de forma aislada en la vida de un sujeto. Sin embargo en general es una enfermedad con tendencia a la cronicidad y resulta más incapacitante que otras enfermedades médicas como la artritis o la diabete. Etiopatogenia, podría ser el modelo biopsicosocial, que resultaría de la integración de tres perspectivas diferentes, la biológica, la psicológica y la social. Los sistemas de neurotransmisión serotoninérgica, noradrenérgica, dopaminergica y peptidérgica están implicados en la depresión. La serotonina produce la inhibición o la activación de la neurotransmisión en el sistema nervioso central. Desde las neuronas de los núcleos dorsales y caudales del rafe se extienden numerosas proyecciones serotoninérgicas hacia áreas cerebrales asociadas a síntomas depresivos. Se ha observado disminución de los niveles de serotonina en el LCR de pacientes con depresión. El sistema noradrenérgico está también implicado en la depresión. El sistema dopaminérgico implica áreas cerebrales de las que depende el comportamiento y funciones fisiológicas alteradas en la depresión. La dopamina puede intervenir en la depresión sobretodo en la manía, ya que algunos agonistas dopaminérgicos se han asociado a la aparición de manía y los antagonistas son eficaces en su tratamiento. CRITERIOS DSM-IV-TR DEL EPISODIO DEPRESIVO MAYOR: Cinco o más de los siguientes síntomas (entre los que debe cumplirse los dos primeros) que representan un cambio respecto a la actividad previa, durante un periodo mínimo de dos semanas. Uno de los síntomas debe ser el estado de ánimo depresivo, o la pérdida de interés o de la capacidad para el placer. Estado de ánimo depresivo. Disminución del interés o la capacidad para el placer. Aumento o pérdida importante de peso. Insomnio o

CURSO ENARM CMN SIGLO XXI TEL: 36246001

Pharmed Solutions Institute

PÁGINA 178

MANUAL DE TRABAJO DEL CURSO ENARM CMN SIGLO XXI hipersomnia casi cada día. Agitación o enlentecimiento psicomotor casi cada día. Fatiga o pérdida de energía casi cada día. Sentimientos de inutilidad o de culpa excesivos o inapropiados. Disminución de la capacidad de pensar, concentrarse o indecisión, casi cada día. Pensamientos recurrentes de muerte, ideación suicida recurrente. El tratamiento en todo tipo de depresiones el uso de fármacos antidepresivos puede resultar beneficioso, no obstante hay que valorar el riesgo-beneficio especialmente en pacientes con otras patologías y en pacientes ancianos. Una excepción es la depresión bipolar, por lo que es importante hacer una buena exploración sobre los antecedentes personales de episodios maníacos e hipomaníacos, y de la historia familiar del paciente. En los pacientes bipolares, antes de prescribir un antidepresivo es necesario administrar un fármaco aprobado como estabilizador del estado de ánimo. Tratamiento de la depresión: Inhibidores selectivos de la recaptación de serotonina (ISRS), los inhibidores selectivos de la recaptación de noradrenalina (ISRSN), los moduladores selectivos duales de los receptores de la noradrenalina y serotonina (NaSSA), inhibidores de la recaptación noradrenérgica (IRN), inhibidores duales de la recaptación de la noradrenalina y dopamina (IRNaDa), Inhibidores de la monoaminooxidasa (IMAO) que comprenden varios fármacos. Los nuevos antidepresivos en su conjunto han dado lugar a mejoras en la farmacoterapia de la depresión por su facilidad de uso, menos efectos secundarios y mayor seguridad en sobredosis. ANSIEDAD: Estado afectivo fisiológico que consiste en tener una sensación difusa de aprensión. Es una sensación desagradable y vaga, de aparición aguda y transitoria, que se acompaña de una activación del sistema nervioso autónomo. Este estado actúa como mecanismo de alerta y adaptación ante una situación amenazante para la integridad física o psíquica del individuo, o percibida como tal. La ansiedad patológica constituye un estado de ansiedad que se desencadena sin un estímulo amenazante, a diferencia del miedo, que constituye una situación de ansiedad provocada por un estímulo realmente amenazante. La manifestación de la ansiedad se basa en tres componentes clínicos fundamentales: un componente somático (síntomas vegetativos), uno cognitivo (sensación de nerviosismo, aceleración, percepciones de malestar, etc) y uno motor (temblor,etc). Trastornos de ansiedad según la DSM-IV TR: 1. Ataques de pánico (crisis de ansiedad, crisis de angustia, panic attac ). 2. Agorafobia − Trastorno de angustia sin agorafobia (F41.0) − Trastorno de angustia con agorafobia (F40.01) − Agorafobia sin historia de trastorno de angustia (F40.00). 3. Fobia específica (F40.02)4. Fobia social (F40.1). 5. Trastorno obsesivo-compulsivo (F42.8). 6. Trastorno por estrés postraumático (F43.1) 7. Trastorno por estrés agudo (F43.0)8. Trastorno de ansiedad generalizada (F41.1) 9. Trastorno de ansiedad debido a enfermedad médica (F06.4) 10.Trastorno de ansiedad inducido por sustancias Además de estos trastornos, agrupados bajo el rótulo “trastornos de ansiedad”, en

Fobia especifica Fobia social

DURA CION No No

Trastorno de pánico

1 mes

Ansiedad generalizad a (TAG)

6 meses

Obsesivo compulsivo (TOC)

No

Estrés postraumti cos (EPT)

DIAGNOSTICO DIFERENCIAL DE TRASTORNOS DE ANSIEDAD TIEMPO DESENCADENANTE MANIFESTACIONES OBSERVACIONES PERIODICIDAD DE SINTOMAS Episodios esporádicos, Estimulo fóbico (presencia o Miedo a un objeto o situación determinada. Temor o Evitación de la situación que min/hrs anticipación) síntomas ansiosos provoca el temor Episodios Situaciones sociales o escrutinio social (presencia o Miedo a colocarse en una situación vergonzosa Evitación de la situación esporádicos, anticipación). Temor angustioso a en un medio social. Temor o síntomas ansiosos. que provoca el temor min/hrs determinadas circunstancias, reales o imaginarios Preocupación de que el desempeño pueda ser Preocupación por que el desempeño sea casa de causa de humillación o burla humillación. Episodios Aparición brusca de un medio intenso. Temblor, sudoración, miedo a morir, sensación de 1 de 3: ansiedad anticipatoria. esporádicos 5-20 Ninguno aparente, pero el sujeto lo frio/calor, nauseas, sensación de asfixia, marea, Preocupación constante, min puede atribuir a múltiples causas miedo a perder el control, dolor de pecho Ataques Repercusión de los AP. Cambio de pánico (AT)súbitos inesperados y recurrentes conductual significativo La mayor parte del Aparición progresiva y permanente Fatiga, inquietud, dificultad para dormir, El individuo está preocupado día casi todos los de síntomas de ansiedad, sin una irritabilidad, tensión muscular, permanentemente sin motivos aparentes y no días causa real que los provoque preocupaciones, nerviosisimos excesivos lo puede controlar. 3 de 6: intranquilidad, cuya intensidad han debilitado el control fatigabilidad, concentración, irritabilidad, sobre ellos tensión muscular, alteración en el dormir Min7hrs Obsesiones que pueden ser días o Cada obsesión lleva asociada una compulsión o Un ejemplo es el miedo al pensamientos que se repiten, y no conducta que compensa la angustia que provoca contagio, cuya composición desaparecen de le mente del individuo, correspondiente es lavarse las aunque lo intente por todos los medios manos rápidamente Aparece en aquellos individuos que se Perdida del sueño, irritabilidad, sobresalto, falta de Provoca alteraciones en la han visto expuesto a un evento concentración. Re exoperimentracion de la situación vida familiar, laboral y social traumático, que involucra un daño físico traumática: recuerdos intrusivos recurrentes, sueños del individuo o psicológico extremo estresantes recurrentes, flashbacks, temor ansiedad o reacciones neorovegetativas intensas al exponenrsea situaciones u objetosrelacionados con el evento.

la DSM-IV TR se incluye finalmente un trastorno de ansiedad infantil, el trastorno de ansiedad por separación. En el trastorno del pánico se podrán utilizaransiolíticos, y para prevenir la crisis se administrarán antidepresivos. Este tratamiento tendrá una duración de 6-12 meses. El trastorno obsesivo-compulsivo deberá ser tratado por el psiquiatra. El tratamiento consiste en la combinación de un fármaco junto a terapia psicológica de modificación de conducta. Las fobias. Hay que distinguir el tipo. Si es específica, se puede abordar en atención primaria con untratamiento semejante al de la ansiedad; pero si es grave, como la agorafobia, fobia social generalizada o algún tipo de fobia simple,será tratada por el especialista. TRASTORNOS DE LA ALIMENTACION: La ANOREXIA NERVIOSA: Se caracteriza por la negativa para mantener el peso corporal normal, lo que resulta en un peso corporal 90% de MS. PATOGENIA: Existe un descontrol total del corazón, debido a la presencia del fenómeno R sobre T (R/T), el cual se presenta cuando llega un estimulo en la parte final de la repolarización. Es de importancia el periodo refractario relativo (PRR), en el si llega un estimulo que rebase el umbral, el músculo se excita en su zona lábil, se desencadena una arritmia cardiaca y puede sobrevenir la muerte. (PRA=aunque el estimulo sea muy fuerte las células del miocardio no se van excitar). Los que tienen más riesgo de sufrir un fenómeno de R/T son: post IAM, porque se quedan cicatrices y estas son focos ectópicos que desencadenan estímulos, en los que tienen hace anómalos, en los hipertensos con miocardiopatía hipertrófica. Solo 2 arritmias descontrolan el corazón porque producen el fenómeno de R/T (en el ECG, vemos una extrasístole ventricular) y lo pueden llevar a la muerte; la taquicardia ventricular (ondas regulares >180 lpm, no hay ondas P, complejos anchos) y la fibrilación ventricular (desorden y ritmo caótico). Ninguna de las 2 produce gasto cardiaco, el primer signo es el sincope, el cerebro se desconecta (no llega oxigeno, ni glucosa), también se puede presentar una convulsión como manifestación de anoxia. La persistencia de la falta de riego lleva a la destrucción irreversible y al fallecimiento del individuo. Así tenemos isquemiaTVFVMS. Muerte súbita es el resultado de; Condición subyacente (cardiopatía coronaria o estructural) + Susceptibilidad individual (inestabilidad eléctrica, no todos los pacientes con SCA hacen FV solo el 15%) + Evento gatillante (isquemia, trastorno HE o acido-base, fármaco). DIAGNOSTICO: La muerte súbita cardíaca no tiene unos síntomas previos como tal, pero sí se pueden enumerar una serie de posibles factores que pueden influir en su aparición: 1. Que el sujeto haya experimentado un episodio de estas características anteriormente o que tenga familiares con antecedentes 2. Insuficiencia cardiaca. 3. Haber sufrido un infarto de miocardio. 4. Arritmias cardíacas. TRATAMIENTO: Sólo hay un tratamiento efectivo para frenar la muerte súbita cardíaca y es la desfibrilación precoz. Este procedimiento consiste en realizar una descarga eléctrica en el corazón, a través de unas palas o parches, con la que se pretende reiniciar la actividad eléctrica del corazón. En el caso de no disponer de un desfibrilador a mano, o de no saber utilizarlo, se puede realizar una reanimación cardiopulmonar. También existe la posibilidad de implantar un desfibrilador cuando se detecta, por ejemplo, un síndrome de Brugada. Que el porcentaje de sangre bombeada por el corazón en cada latido, denominada 'fracción de eyección', sea igual o inferior a un 40 por ciento. PREVENCION PRIMARIA: Identificar los grupos de riesgo; GRUPO DE MAYOR RIESGO DE SUFRIRI UN EPISODIO DE MS: A). Cardiopatía coronaria sintomática: con IAM en evolución o reciente. Angina inestable. B). Cardiopatía estructural asociada a cardiopatía coronaria, ICC  FEVI < 35 %. C). Con arritmias ventriculares, Definen riesgo de MS : TVNS inducible o espontánea, TVS inducible o espontánea. En este grupo siempre debe corregirse isquemia, ya sea médicamente o con procedimientos de revascularización. GRUPO DE RIESGO INTERMEDIO: A).Cardiopatía estructural adquirida (HTA, valvular, coronaria) o congénita. Estos grupos son heterogéneos, en general son pacientes cardiópatas sintomáticos o no sintomáticos, pero sin elementos coronarios activos ni arritmias ventriculares de alto riesgo, se benefician de terapia farmacológica que detiene o aminora progresión de cardiopatía demostrado por evidencia (IECA, espironolactona, bloqueadores AT2 ,BB). GRUPO BAJO RIESGO: Población general con factores de riesgo cardiovasculares que determinan finalmente aparición de cardiopatía coronaria, estructural o ambas , bajo riesgo de MS pero por número de afectados son los que proporcionalmente aportan más casos al año de MS. Fundamental manejar FR : HTA, DM , sedentarismo, tabaquismo, dislipidemias. Intervenciones terapéuticas tanto farmacológicas como no farmacológicas aplicadas a esta población son de alto impacto y relativo bajo costo, por lo que hay alta costoefectividad. A nivel poblacional también son aplicables medidas colectivas para evitar que aparezcan factores de riesgo, sobretodo en población infantil y juvenil (evitar tabaquismo, evitar sedentarismo, alimentación saludable), como también se vislumbran intervenciones como introducción de elementos dietéticos específicos. TRATAMIENTO: SI el paciente tiene una TV o FV, desfibrilar, para que el corazón retome su ritmo, se puede dar reanimación cardiovascular pero con CAB no con ABC (primero compresiones, para genera gasto cardiaco). Solo los grupos de mayor riesgo se benefician en términos de prevención primaria de DI (desfibrilador implantable), el resto de los pacientes debieran recibir terapia farmacológica que se ha demostrado que disminuye el riesgo de muerte atribuible a MS, como amiodarona (prolonga el PRA, reduce posibilidad de fenómeno R/T) o BB, estos últimos además han demostrado que reducen mortalidad global. Esta estrategia es independiente del manejo adecuado del paciente con cardiopatía demostrada con fármacos que mejoran sobrevida y progresión de enfermedad como IECA, antagonistas de Angiotensina 2, espironolactona, etc. El desfibrilador implantable es la terapia preferida en una gran proporción de pacientes

CURSO ENARM CMN SIGLO XXI TEL: 36246001

Pharmed Solutions Institute

PÁGINA 185

MANUAL DE TRABAJO DEL CURSO ENARM CMN SIGLO XXI sobrevivientes a MS. El valor de esta aproximación en pacientes con episodio previo de MS es encontrar condiciones de alto riesgo donde no cabe duda de la indicación de DI y condiciones donde no hay duda de que el paciente no se beneficiará. CASOS CLINICOS Un varón de 26 años de edad acudió por palpitaciones paroxísticas acompañadas de sudoración profusa, náusea improductiva y gran agitación psicomotriz sin relación con el esfuerzo. Presentó un deterioro inesperado de su estado general, con taquicardia ventricular por torsade de pointes y una parada cardiorrespiratoria recuperada mediante desfibrilación eléctrica cardiaca (tres choques de 200-200 y 360 J de energía respectivamente) y administración farmacológica (2 g de sulfato magnésico en bolo de 2 min y luego perfusión intravenosa pautada de 500 ml de SSF con 2 mg de sulfato magnésico, junto con 1,5 μg de isoproterenol/min. Una hora después presenta parada cardiorrespiratoria sin recuperación a la maniobras. PREGUNTA Cuál es la alteración del ritmo más probable que ocaciono la muerte. RESPUESTA a.- Bloqueo AV de tercer grado. b.- Taquicardia ventricular. c.- Fluter auricular. d.- Bradicardia ventricular. CASO CLINICO Varón de 23 años en parada cardiorrespiratoria no recuperada tras protocolo de reanimación cardiopulmonar avanzada, por lo que se activa código de donante en asistolia, mientras se mantiene el masaje cardíaco con ventilación mecánica y se conecta al paciente a circulación extracorpórea. En la radiografía de tórax que se realiza en el proceso de validación pulmonar se evidencia masa mediastínica radioopaca de localización inferior y media. PREGUNTA Cuál es la etiología más probable de este caso RESPUESTA a.- Aspiracion de contenido gástrico por maniobras. b.- Neumonia. c.- Edema agudo pulmonar. d.- Tromboembolismo. CASO CLINICO Un varón de 26 años de edad acudió por palpitaciones paroxísticas acompañadas de sudoración profusa, náusea improductiva y gran agitación psicomotriz sin relación con el esfuerzo. Presentó un deterioro inesperado de su estado general, con taquicardia ventricular por torsade de pointes y una parada cardiorrespiratoria recuperada mediante desfibrilación eléctrica cardiaca (tres choques de 200-200 y 360 J de energía respectivamente) y administración farmacológica (2 g de sulfato magnésico en bolo de 2 min y luego perfusión intravenosa pautada de 500 ml de SSF con 2 mg de sulfato magnésico, junto con 1,5 μg de isoproterenol/min. Una hora después presenta parada cardiorrespiratoria sin recuperación a la maniobras. PREGUNTA Cuál es la alteración del ritmo más probable que ocaciono la muerte. RESPUESTA a.- Bloqueo AV de tercer grado. b.- Taquicardia ventricular. c.- Fluter auricular. d.- Bradicardia ventricular. CASO CLINICO Se trata de paciente masculino de 61 años de edad el cual ingresa a urgencia por perdida del estado de despierto de forma súbita, produciendoce caída desde su propia altura. Se iniciaron maniobras de resucitación con recuperación luego de dos descargas con 360 j, presentando nuevamente perdida del estado de despierto. Tiene como antecedentes diabetes mellitus 2 desde hace 15 años, tratado con insulina glargina, actualmente en protocolo de dialisis peritoneal con KDOQI 5, y un infarto hace 6 meses. PREGUNTA Considerando la presentación del cuadro clínico y el ECG cual es la conducta a seguir? RESPUESTA a.- Descarga de 360 j. b.- Compresiones toraccicas. c.- Administracion de amiodarona. d.- Masaje carotideo.

CURSO ENARM CMN SIGLO XXI TEL: 36246001

Pharmed Solutions Institute

PÁGINA 186

MANUAL DE TRABAJO DEL CURSO ENARM CMN SIGLO XXI PREGUNTA Cual de las siguientes factores de riesgo es el mas grave para predisponer al paciente de presentar el estado actual? RESPUESTA a.- Insuficiencia renal crónica. b.- Infarto al miocardio antiguo. c.- Diabetes mellitus de larga evolución. d.- Hipomagnesemia crónica. CASO CLINICO Se trata de paciente masculino de 71 años de edad el cual ingresa a urgencias por perdida del estado de despierto, al ingreso se observa con palidez generalizada, hipotérmico, sin respuesta a estimulos y sin pulso palpable, se activa el código de alerta y se inicia con compresiones toraccicas por parte de medico de guardia, se realiza dos ciclos de compresiones con leve respuesta y se observa trazo fibrilar en ECG. PREGUNTA Cual es la conducta a seguir inmediata mas adecuada considerando el trazo. RESPUESTA a.- Desfibrilar a 360 j. b.- Reiniciar compresiones. c.- Administración de amiodarona. d.- Intubación orotraqueal inmediata. PREGUNTA Considerando la etiología de la FA, cual es la mas probable en este paciente?. RESPUESTA a.- Cardiopatia coronaria. b.- Infarto agudo al miocardio. c.- Hipertrofia ventricular. d.- Cardiomiopatia dilatada. PREGUNTA Luego de 4 ciclos de compresiones y desfibrilación el paciente presenta ritmo sinusal, cual es la razón de que solo con la descarga se corrija el ritmo? RESPUESTA a.- Periodo refractario. b.- Periodo refractario relativo. c.- Periodo refractario absoluto. d.- Periodo refractario con mecanismo de reentrada. CASO CLINICO Se trata de paciente masculino de 21 años de edad, es ingresado a urgencias debido a que perdió el estado de alerta mientras jugaba futbol, no referio síntoma alguno solo se cayo en la cancha, a la exploración se observa lijera palidez de tegumentos, llenado capilar de 2 segundos, pulso lento y débil, FC90, FR 22, TA 110/70 mmHg, responde a ordenes y refiere no recordar lo que paso. Como antecedente de importancia refiere que ocacionalmente se ha sentido mareado desde los 12 años, pero nunca se había desmayado, de niño le mencionaron que tenía un soplo que se quitaría posteriormente, cabe mencionar que su padre murió a los 31 años. Se realizo ECG en urgencias una hora después de su ingreso. PREGUNTA Considerando el cuadro clínico y el ECG cual es la etiología mas probable. RESPUESTA a.- Miocardiopatia hipertrófica. b.- Sindrome de QT largo. c.- Displasia arritmogenica. d.- Cardiopatia coronaria. PREGUNTA Considerando la condición subyacente cuales son las medidas mas adecuada para evitar otro evento del mismo orden potencialmente mortal. RESPUESTA a.- Evitar eventos gatillantes.

CURSO ENARM CMN SIGLO XXI TEL: 36246001

Pharmed Solutions Institute

PÁGINA 187

MANUAL DE TRABAJO DEL CURSO ENARM CMN SIGLO XXI b.- Considera susceptibilidad individual. c.- Estudiar antecedentes personales. d.- Modificación de estilo de vida. CASO CLINICO Ingresa a urgencias maculino de 21 años de edad el cual se observa ansioso, agitado con actitud alucinada, a la exploración se encuentra diaforético con palidez generalizada, súbitamente el paciente pierde el estado de alerta y presenta parada cardiaca, se activa el código de alerta e inicia compresiones toraccicas, los familiares solo refieren que lo golpearon en una fiesta, 10 minutos después presento los síntomas. Se observo el siguiente trazo del ECG. No hay respuesta, no se detecta pulso, ni constantes vitales. Se descarga con 360 j, se mantiene soporte por 20 minutos más. PREGUNTA No se presive pulso, cual de las siguientes causas es la mas probable en este caso? RESPUESTA a.- Neumotorax a tensión. b.- Sobredosis por drogas. c.- Tromboembolismo pulmonar. d.- Hipovolemia. CASO CLINICO Varón de 23 años en parada cardiorrespiratoria no recuperada tras protocolo de reanimación cardiopulmonar avanzada, por lo que se activa código de donante en asistolia, mientras se mantiene el masaje cardíaco con ventilación mecánica y se conecta al paciente a circulación extracorpórea. En la radiografía de tórax que se realiza en el proceso de validación pulmonar se evidencia masa mediastínica radioopaca de localización inferior y media. Se aprecian signos de broncoaspirado, por lo que se desestima la extracción pulmonar. PREGUNTA Cuál es la etiología más probable de este caso RESPUESTA a.- Aspiracion de contenido gástrico por maniobras. b.- Neumonia. c.- Edema agudo pulmonar. d.- Tromboembolismo.

CURSO ENARM CMN SIGLO XXI TEL: 36246001

Pharmed Solutions Institute

PÁGINA 188

MANUAL DE TRABAJO DEL CURSO ENARM CMN SIGLO XXI RCP BASICA Y AVANZADA: RCP BASICA: Ante una potencial victima siempre ALGORITMO RCP AVANZADA 1.-Paro cardiorrespiratorio, iniciar RCP básico verificar la ausencia de respuesta (inconsciencia) y activar inmediatamente el sistema de respuesta medica de urgencias para asegurar la llegada de un desfibrilador. Si hay un desfibrilador presente úselo NO SI 2.- Checar es un 9.- Asistolia/AESP 3.- FV/TVSP en este momento. La RCP básica se mantiene hasta la ritmo a desfibrilar? llegada de un desfibrilador o hasta que reinicie ventilaciones espontaneas. BUSCAR RESPUESTA: Ante 4.- Dar una descarga (12010.- Reiniciar RCP por 5 ciclos. una potencial victima de Muerte Súbita (MS) se debe 200J-bifasico o 360JAdmon.: adrenalina 1mg IV, proceder siempre de la misma manera independiente monofasico. Reiniciar RCP repetir cada 3-5 min. Considerar atropina 1mg IV en del lugar de los hechos o los recursos disponibles en el 5 ciclos asistolia o AESP lenta, repetir lugar. El primer paso es asegurarse de que la victima cada 3-5 min, hasta 3 dosis 5.- Checar es un realmente este inconsciente, hay que ponerla en ritmo a desfibrilar? 5 ciclos posición adecuada para la RCP (decúbito dorsal, 11.- Checar es un mirando hacia arriba) y tomándola de los hombros y ritmo a desfibrilar? 6.- Continuar RCP mientras hablándole en voz alta al oído buscar alguna respuesta carga desfibrilador. Dar (apertura ocular, emisión de sonidos o movimiento). una nueva descarga. 12.- Si asistolia, ir a cuadro 10. Debe evitarse realizar movimientos bruscos que Reiniciar RCP. Admon.: Si actividad eléctrica, checar puedan producir o agravar lesiones presentes, adrenalina 1mg IV repetir pulso: Si no hay ir a cuadro 10. cada 3-5 min durante RCP Si hay, iniciar cuidados postespecialmente de la columna cervical. Si la víctima no resucitación. responde las posibilidades son que este en coma, este 5 ciclos en paro respiratorio o paro cardiorrespiratorio (PCR). 7.- Checar es un Para determinarlo se debe continuar con la evaluación ritmo a desfibrilar? 13.- Cuidados postresucitación en forma ordenada. PEDIR AYUDA: Como la FV es el ritmo más frecuente y tratable en la MS en el adulto, 8.- Continuar RCP mientras carga desfibrilador. se pide ayuda para asegurar la llegada de un Dar una nueva descarga. Reiniciar RCP. desfibrilador y ayuda médica avanzada. Si el evento Considerar antiarrítmico, amiodarona: 300mg IV, considerar 150 adicional o lidocaína 1mg/kg. fue extrahospitalario y hay otras personas presentes, Considerar Magnesio 1-2g IV en torsade de el reanimador debe identificar a una de ellas y pointes. Luego 5 ciclos de RCP, ir a cuadro 5 solicitarle que active el sistema de respuesta medica de urgencias indicando que hay una víctima de muerte súbita y la necesidad de un desfibrilador dando la dirección exacta del evento. Si el reanimador esta solo debe pedir la ayuda el mismo aun si esto implica dejar a la victima solo por algunos segundos. Si el paro es intrahospitalario, el reanimador debe activar el sistema local de respuesta médica de urgencias (timbre de alarma, código azul, etc.). : Permeabilizar la vía aérea de la víctima. Durante el PCR la victima pierde complemente el tono muscular, asociado al efecto de la gravedad la lengua cae hacia atrás ocluyendo la vía aérea, siendo esta la principal causa de obstrucción. La maniobra más efectiva para permeabilizar la vía aérea es extender la cabeza y elevar del mentón. Si existe o se sospecha lesión de columna cervical solo se debe realizar la elevación del mentón manteniendo la cabeza en posición neutra. : Con la vía aérea abierta el reanimador acerca el oído a la nariz de la víctima, mirando hacia el tórax de la misma, Escuchando si hay respiración y Sintiendo el aire exhalado en un tiempo igual o menor a 10 segundos. Si la victima tiene una respiración adecuada el reanimador mantiene la vía aérea abierta y mientras espera la llegada de ayuda médica avanzada inicia la búsqueda de la causa y reevalúa periódicamente a la víctima. La presencia de respiración adecuada implica presencia de circulación. En ocasiones las victimas de muerte súbita presentan movimientos respiratorios muy superficiales o respiración agónica que no deben ser confundidos con respiración adecuada y el reanimador debe actuar de la misma forma que si no hubiera respiración. Con la vía aérea abierta el reanimador debe mirar, escuchar y sentir (MES), buscando la respiración. : Se debe checar ritmo cada 5 ciclos. El pulso carotideo ha mostrado ser impreciso. Una víc ma potencial de muerte s bita que efec vamente este en PCR y no recibe RCP básica morirá con seguridad. La FV progresa hacia asistolia rápidamente, disminuyendo la probabilidad de éxito con la desfibrilación en un 10% con cada minuto de retraso, la RCP básica disminuye el porcentaje a un 4% por cada minuto, aumentando el periodo en el cual el ritmo es desfibrilable y por lo tanto reversible. Durante los primeros minutos de una muerte súbita la sangre de la victima contiene suficiente oxigeno para cumplir con la demanda. Además, la demanda de oxigeno es menor ya que la circulación también esta disminuida hasta 1/3 parte de lo normal. El problema principal es entonces falta de circulación de sangre más que falta de oxigenación de la misma. Por lo tanto, si la persona esta inconsciente y sin respiración el reanimador debe iniciar la RCP con 30 compresiones torácicas externas seguidas de 2 ventilaciones artificiales. Una excepción para el uso precoz de la desfibrilación es la presencia de un PCR prolongado (>5 minutos) donde la RCP básica, aumentan el éxito de la desfibrilación. Existe un pequeño porcentaje de pacientes que tienen una muerte súbita de etiología hipoxica (obstrucción de vía aérea por cuerpo extraño) en quienes si se tiene la sospecha o certeza se podría privilegiar las ventilaciones artificiales y empezar la RCP con estas y continuar con compresiones torácicas externas. : Las compresiones torácicas se realizan colocando el talón de una mano en el centro del tórax entre ambas tetillas con la otra mano encima y los dedos entrelazados. Con los codos completamente extendidos se inician las compresiones empujando fuerte y rápido. El mayor determinante de éxito con la desfibrilación es la presión de perfusión coronaria. Las compresiones torácicas externas efectivas logran entregar una pequeña pero critica cantidad de oxigeno al cerebro y corazón, suficiente para mantener una presión de perfusión coronaria. Compresión torácica externa efectiva debe cumplir con las siguientes

CURSO ENARM CMN SIGLO XXI TEL: 36246001

Pharmed Solutions Institute

PÁGINA 189

MANUAL DE TRABAJO DEL CURSO ENARM CMN SIGLO XXI condiciones: 1.- Frecuencia >100 x min. 2. Profundidad entre 4-5 cm. 3.- Equilibrar el tiempo de compresión con el de descompresión del tórax (50 y 50%). 4.- Permitir el retorno del tórax a su expansión total durante la fase de descompresión. 5.- Minimizar las interrupciones en la compresión torácica externa. Las 30 compresiones torácicas son seguidas de 2 ventilaciones artificiales de 1 segundo cada una (la misma para uno o dos reanimadores). : Puede ser administrada con distintas técnicas: 1.- Boca a boca: Abriendo la vía aérea el reanimador toma una respiración normal y a continuación sella su boca alrededor de la de la victima e insufla lo suficiente como para elevar el tórax de la víctima. 2.- Ventilación con barreras de protección: Estas son dispositivos plásticos con una válvula unidireccional que en teoría evitan la transmisión de enfermedades infectocontagiosas (no demostrado). 3.- Boca a nariz: Se utiliza cuando no es imposible ventilar a través de la boca (lesiones bucales, imposibilidad de abrir la boca). 4.- Mascaraválvula-bolsa: Logra oxigenación y ventilación efectivas pero requiere de entrenamiento y práctica frecuente. En los primeros minutos de la RCP la máscara-válvula-bolsa es igual de efectiva y segura que un tubo endotraqueal. La intubación endotraqueal no tiene lugar en la RCP básica. Las compresiones torácicas externas agotan rápidamente al reanimador y cuando esto sucede disminuye notablemente su efec vidad. Es así que si hay dos o más reanimadores presentes deben rotarse las compresiones torácicas cada 2 minutos para garantizar que estas sean efectivas. Después de la desfibrilación (200J, 200-300J, 360J si es bifásico o cargas fijas de 360J si es monofásico) las compresiones torácicas y ventilaciones artificiales se mantienen por 2 minutos. Si la desfibrilación fue exitosa y se logro un ritmo de perfusión se suspende la RCP básica y se inicia la avanzada según necesidad. En cambio, si la desfibrilación no fue exitosa se siguen alternando ciclos de compresión torácica con ventilación artificial por 2 minutos y nuevos análisis del ritmo por el DEA y descargas si este las indica. RCP AVANZADO: A (Airway): asegurar la vía aérea: En el pasado se prefería la intubación oro-traqueal (IOT), pero la incidencia de complicaciones es muy alta cuando es realizada por inexpertos. Las alternativas serian el Combitube esófago-traqueal y la mascara laríngea, pero no están disponibles. La IOT permite mantener permeable la vía aérea, aspirar secreciones, administrar O2 en altas concentraciones, es una vía alternativa para la administración de medicación y cuando se insufla el manguito, protege la vía aérea de la aspiración. Se debe minimizar el tiempo de interrupción de las compresiones (10seg). Si no fue exitosa, asegurar un intervalo adecuado de compresiones y ventilaciones. Si el paciente tiene un ritmo con pulso, controlar con saturometria. Complicaciones de OIT: ruptura de dientes laceración traqueofaríngea, ruptura traqueal, perforación faringoesofagica, aspiración de contenido gástrico, intubación bronquial. B (Breathing): comprobación de la localización del tubo a través de métodos clínicos (visualización de expansión torácica y auscultación) y complementarios (detectores de CO2). C (Circulation): Obtener un acceso venoso de grueso calibre (antecubital), conectar a monitor y administrar drogas según ritmo. Cuatro ritmos producen el paro cardiaco: la FV, TV sin pulso, asistolia y AESP. Las drogas se administran en bolo seguidas de 20 ml de solución fisiológica y elevar el brazo. Algunas pueden administrarse por el tubo endotraqueal; atropina, lidocaína, adrenalina y naloxona, pero a 2 veces la dosis y seguida por 10ml de SF y 3-4 ventilaciones. Los fluidos deben ser administrados cuando se sospecha hipovolemia. Administrar bicarbonato solo si hiperkalemia persistente. Diagnósticos diferenciales: considerar las posibles causas descompensantes o que complican la resucitación. Regla de las 5Hs (hipovolemia, hipoxia, H+ (acidosis), Hiper/hipocalemia, hipotermia) y las 5Ts (tóxicos, taponamiento cardiaco, neumotórax a tensión, trombosis coronaria, TEP). CASO CLINICO Varón de 48 años, acude a consulta al hospital y cuando se dirige a recabar su ficha de consulta cae al suelo inconsciente, Circunstancialmente usted pasa por el lugar y se dispone a brindar auxilio e inicia los pasos del soporte vital básico. PREGUNTA Cual es la conducta a seguir inmediatamente? RESPUESTA a.- Iniciar compresiones. b.- Iniciar ventilación. c.- Activar sistema de emergencia. d.- Verificar pulso. CASO CLINICO Paciente de 47 años, con antecedente de un episodio de fibrilación auricular en que no tuvo mayor estudio y revirtió a ritmo sinusal (RS) espontáneamente. Sin terapia posterior y asintomático. Estando en reposo, presenta paro cardíaco que fue atendido inmediatamente por familiares con maniobras básicas de resucitación cardiopulmonar, siendo trasladado en un lapso de 5 a 10 min al Hospital local donde al ingreso se observa actividad eléctrica sin pulso. PREGUNTA Cual es su conducta a seguir inmediatamente? RESPUESTA RESPUESTA a.- Iniciar compresiones. b.- Iniciar ventilación. c.- Activar sistema de emergencia. d.- Verificar pulso.

CURSO ENARM CMN SIGLO XXI TEL: 36246001

Pharmed Solutions Institute

PÁGINA 190

MANUAL DE TRABAJO DEL CURSO ENARM CMN SIGLO XXI ESTADO DE CHOQUE: CIENCIAS BASICAS: Definición: Reducción de la perfusión tisular sistémica con decremento de la disponibilidad de O2, provocando hipoxia celular. Estado metabólico en el cual las demandas energéticas celulares de oxigeno, exceden el aporte. Disfunción celular, orgánica y sistémica. SALUD PUBLICA: Mortalidad mayor al 20%. Corresponde al 1% de las atenciones en el servicio de urgencias. En el shock anafiláctico cerca de 60% se muere por obstrucción de la vía aérea superior, el resto por hipotensión y arritmias. PATOGENIA: Varios factores influyen en el desarrollo como: desproporción entre el volumen circulante y el lecho vascular. Trastorno en la circulación capilar. Trastorno en el intercambio gaseoso. Hipoxia de los tejidos y acidosis metabólica. A 8ml/kg/min se inicia la isquemia celular DEFINICION

SIGNOS SINTOMAS

HIPOVOLEMICO Estado patológico asociado a procesos cuyo denominador es la hipo perfusión con hipoxia celular, secundaria a la perdida de volumen circulante, intra o extravascular Hipotensión arterial, hipotermia, taquicardia, palidez de tegumentos

TIPOS

Hemorrágico No hemorrágico: 3er espacio (pancreatitis, vasculitis, quemados) Perdidas gastrointestinales Perdidas urinarias (DM insípida)

CAUSAS

Se da como consecuencia de la perdida súbita de volumen intravascular

RVP GC PVC

Aumentado (>1500 dinas/min/cm-5) Disminuido (38° 0 90 por minuto. 3. Taquipnea: >20 por minuto o PaCO2 20 microgramos/kg/min), se recomienda norepinefrina o epinefrina. La noradrenalina es un medicamento vasoconstrictor potente, no general tantas arritmias, es de primera elección en choque septico. La vasopresina puede ser útil en pacientes refractarios a

CURSO ENARM CMN SIGLO XXI TEL: 36246001

Pharmed Solutions Institute

PÁGINA 191

MANUAL DE TRABAJO DEL CURSO ENARM CMN SIGLO XXI norepinefrina, y se recomienda en el choque en fase de vasodilatación (choque séptico). La adrenalina es Manifestaciones de Manifestaciones de bajo Inestabilidad de primera elección en choque anafiláctico, en bolos ICC gasto hemodinámica subcutáneos 3. Terapia vasodilatadora: su uso debe Disnea, polipnea, Palidez, oliguria, llenado Hipotensión arterial, estertores capilar prolongado, reacción PAS 15mmHg cristaloides; las mas empleadas solución salina isotónica y Hartman, clínicamente puede esperarse DIAGNOSTICO CHOQUE CARDIOGENICO de 100-200ml de expansión, después de infundir 1 CLASIFICACION DE CHOQUE litro de cristaloides isotónicos. Las soluciones HIPOVOLEMICO HEMORRAGICO hipertónicas tienen un potencial benéfico como GRADO I II III IV PERDIDA/ML 750 750-1500 1500-2000 >2000 incremento en la contractilidad miocárdica y PERDIDA % 15% 15-30% 30-40% >40% vasodilatación precapilar; sin embargo, pueden FC 100 >120 >140 conducir a estados hiperosmolares con facilidad. FR 14-20 20-30 30-40 >40 Coloides; en México se cuenta con 4 como albumina T/A NL NL Disminuido Disminuido (frasco de 50cc con concentración al 20%), gelatina P. pulso Normal Disminuido Disminuido Disminuido DIURESIS >30ml/h 20-30 5-15 0 (haemacel), dextranes, almidón (hidroximetilalmidon Edo. mental Ansiedad Ansiedad Confusión Letargo al 10%). Un litro de albumina al 5%, produce una Rest. liquida Cristaloides Cristaloides Crista +sangre Crista +sangre expansión de plasma de 500-1000 ml. Un litro de almidón expande 700ml. Tanto gelatinas como almidones son inductores de diátesis hemorrágica, en forma dosis dependiente. El dextran compromete dramáticamente la hemostasis, la gelatina un impacto moderado, aunque la agregación plaquetaria se ve moderadamente afectada. Sangre y hemoderivados: no se ha demostrado beneficio en la perfusión tisular, al transfundir pacientes en estado de choque con niveles de hemoglobina de 8-10mg/dl. Las siguientes condiciones clínicas podrán beneficiarse de hemotransfusion, taquicardia importante, desaturación venosa mezclada importante, disfunción cardiaca, enfermedad arterial coronaria coexistente, acidosis láctica persistente, incapacidad otros marcadores de hipoperfusión celular. No existe un nivel de hemoglobina que indique hemotransfucion con el sustento de mejorar el aporte sanguíneo a los tejidos. Las complicaciones de hemotransfusion masiva, lleva a mayor riesgo de infecciones y reacciones anafilácticas, puede llevar a SIRA, aquí se llama TRALI, puede dar a partir de las 6 hrs de trasnfundido. Apoyo ventilatorio; en todo momento asegurase del adecuado funcionamiento de la vía aérea, siendo la permeabilidad e integridad de la misma una condición indispensable para cumplir con la meta de oxigenación sistémica. Los elementos que pueden orientar para el empleo de asistencia en la ventilación son: administrar puntas nasales, mascara facial, cánula faríngea o mascarilla con reservorio en caso de contarse con ella, en cualquier situación clínica que sugiera estado de choque. Criterios recomendados para decidir intubación endotraqueal, con manejo de ventilador mecánico en un paciente en estado de choque; hipercarbia >32mmHg, taquipnea >20 rpm, cianosis distal con alguno de los elementos anteriores, acidosis respiratoria con pH=7.2, paciente con relación PaO2/FiO2 65 mmHg. c.- Gasto urinario (GU) 0.3 ml/kg/hr. d.- Saturacion de sangre venosa (SATVO2) > 70 %. PREGUNTA Cual de los siguientes criterios de reanimación del paciente con sepsis grave se encuentra fuera de rango? RESPUESTA a.- Soluciones cristaloides > 1000 ml/30 min ó 300 a 500 ml coloides. b.- Trasfusión de concentrados eritrocitarios para lograr un hematocrito > de 30 % con ScvO2 baja. c.- Administracion de plasma fresco congelado con elevación de TP, INR ó TPT. d.- Perfusion de dobutamina 50 mcg/kg/min PREGUNTA Cual de los siguientes parámetros guarda mayor relación de la principales complicaciones sin embargo menos especifico para vigilar al caso clínico actual? RESPUESTA a.- Proteina C reactiva. b.- Hiperlacticidemia. c.- Procalcitonina. d.- Creatinina serica PREGUNTA Cual de los siguientes parámetros guarda menor sensibilidad en relación con el síndrome de respuesta inflamatoria sistémica? RESPUESTA a.- Proteina C reactiva +/-1DS mayor. b.- INR > 1.5 c.- TTPa > 60” d.- Trobocitopenia < 100,000 celulas/mm3. CASO CLINICO Se trata de una mujer de 58 años que ingresó por fiebre y cefalea de aparición brusca. Dentro de sus antecedentes personales, destacaba la presencia de prótesis mecánica mitral, aórtica secundaria a valvulopatía reumática, y nefrectomía derecha. Se encontraba recibiendo tratamiento anticoagulante. A la exploración física, únicamente llamaba la atención cierto grado de rigidez nucal. En la analítica inicial, leve deterioro de su insuficiencia renal de base (creatinina de 1,83 mg/dL). PREGUNTA Ante tales manifestaciones, se realizó una TAC craneal así como punción del LCR que no presentaron alteraciones. Considerando los antecedentes y estado actual cual es la conducta a seguir para establecer un agente etiológico. RESPUESTA a.- Ecocardiograma transesofagico. b.- RMN craneal por síntomas neurológicos. c.- Cultivo con antibiograma. d.- BH y tiempos de coagulación.

CURSO ENARM CMN SIGLO XXI TEL: 36246001

Pharmed Solutions Institute

PÁGINA 198

MANUAL DE TRABAJO DEL CURSO ENARM CMN SIGLO XXI CASO CLINICO Se trata de femenino de 27 años de edad originaria de la sierra de Oaxaca, se encuentra en hospitalización en medicina interna, debido a que presenta 24 horas con exantema, hipotensión y fiebre, a la exploración física se observa lesión en miembro pélvico, dicha lesión es ulcerosa, sin eritema, no hay irritación ni secreciones, pero si hay presencia de exantema en palmas, conjuntivas y mucosa bucal, además descamación en las puntas de los dedos, los resultados de laboratorios reportan creatinina de 2.7 aspartato aminotransferasa de 250 U/L, alanina aminotransferasa de 328 U/L, aun no se cuenta con resultados de los cultivos, se instaura tratamiento antibiótico y reanimación con liquidos y vasopresores. PREGUNTA Cual es el diagnostico mas probable en este caso. RESPUESTA a.- Choque por Fiebre Tifoidea b.- Choque toxico estafilocócico. c.- Choque toxico estreptocócico. d.- Choque toxico botulínico. CRISIS HIPERTENSIVA: CIENCIAS BASICAS: Por convención los valores de tensión arterial (TA) severamente elevados se definen por tensión arterial sistólica (TAS) mayor a 180 mmHg y tensión arterial diastólica (TAD) mayor de 120 mmHg, este corte es meramente arbitrario, con poca relevancia para el médico, porque la mayoría de los pacientes no requerirá tratamiento urgente para reducción de la TA. De forma contraria paciente con elevaciones menores, que no reúnen criterios de HTA severa pueden requerir terapia antihipertensiva inmediata para situaciones de riesgo como en los aneurismas disecantes de aorta. Las crisis hipertensivas incluyen a las emergencias y urgencias. Emergencia hipertensiva: Presencia de daño de órgano agudo, que requiere rápido descenso de TA, en un intervalo de horas. Usualmente requiere internación en unidad de cuidados críticos con medicaciones por vía parenteral. En estos casos la TA cumple un rol patogénico en la génesis y progresión del cuadro clínico. Debemos recalcar que la emergencia está definida por la gravedad del cuadro clínico y no por los valores de TA, usualmente la TAD es mayor 120 mmHg. Los órganos que son afectado más frecuentemente en emergencias hipertensivas son el cerebro (EVC 24.5%, encefalopatía 16.3%, hemorragias intracerebrales o subaracnoideas 4.5%), corazón (EAP 22.5%, ICC 14.3%, IAM 12.0%, disección aortica 2.0%), grandes vasos, riñón y útero gravídico (eclampsia 4.5%, preeclampsia grave), siendo más frecuente la afectación única en el 83% de los casos, pero en porcentaje variable se pueden presentar más de un daño de órgano blanco en contexto de emergencias hipertensiva. Urgencia hipertensiva: cuando se produce una elevación brusca de la presión arterial (PA), en relación a las cifras tensiónales habituales del paciente o cuando la PA ≥ 210/120 mmHg (algunos consensos actuales hablan de sistólicas ≥180 mmHg), pero sin síntomas específicos ni daño de órganos diana. El objetivo terapéutico sería reducir gradualmente la PA (en 24-48h) con medicación oral ya que si se baja muy rápidamente puede favorecerse la hipoperfusión de órganos diana. Debe distinguirse de una pseudocrisis hipertensiva, que es la elevación de la PA reactiva a una situación de stress o dolor. Hipertensión severa aislada: Elevación de la TAD >120 mmHg, que requiere descenso de valores de TA en lapso de días con medicación y usualmente sin internación. Por definición se debe establecer previamente la ausencia de daño de órgano blanco, hay poca evidencia que el control de la TA, en un lapso de tiempo corto sea beneficiosos. SALUD PÚBLICA: La Hipertensión Arterial (HTA) es un proceso crónico que presenta una prevalencia de más del 25% en la población occidental adulta1. Aproximadamente un 1-2% de los pacientes hipertensos desarrollaran una crisis hipertensiva (urgencia o emergencia) en algún momento de su vida. El tratamiento y el adecuado seguimiento de estos pacientes aumentan su esperanza de vida y disminuye la incidencia de complicaciones. PATOGENIA: T/A = (gasto cardiaco) (resistencias vasculares periféricas). Para que la presión arterial aumente se requiere desequilibrio en esta relación, como falla de mecanismos autorreguladores, alteraciones en propiedades contráctiles del musculo arteriolar, activación del sistema SRAA, vasoconstricción y aumento de RVP. El rápido desarrollo de daño de órgano agudo de las emergencias hipertensivas es atribuido a la falla en el mantenimiento de la normal autorregulación de los lechos vasculares de órganos de choque (cerebro, riñón, retina y corazón) y por otro lado un aumento abrupto de las resistencias vasculares sistémicas. Conduciendo a una injuria endotelial con necrosis fibrinoide de arteriolas. Estableciéndose el ciclo isquemia, depósito plaquetario, y falla de la autorregulación con deterioro clínico del paciente. Los gatillos específicos de este fenómeno no son conocidos. DIAGNOSTICO: Historia clínica: historia de la HTA (edad de inicio, tiempo de evolución, grado de control, cifras basales, tratamiento seguido, síntomas acompañantes), momento del inicio de la clínica, evolución hasta el momento del ingreso, síntomas asociados. Antecedentes personales: hábitos higiénico-dieté cos (alcohol, drogas), enfermedades asociadas, factores de riesgo cardiovascular, repercusión sobre órganos diana, tratamientos. Antecedentes familiares: HTA, enfermedades cardiovasculares, enfermedades renales, obesidad, dislipemia. Exploración física: Priorizar en primer lugar la búsqueda de daño a órgano blanco. Medición de la PA. Peso, talla, índice de masa corporal (IMC) y perímetro abdominal. Exploración sistemática (cardiovascular, pulmonar, abdominal, neurológica, ocular, cutánea, cuello y palpación tiroidea). Palpación de pulsos periféricos (en las cuatro extremidades) y centrales valorando simetrías y amplitud. Fondo de ojo (hemorragias, exudados blandos, papiledema). Pruebas complementarias: En urgencia hipertensiva: sistemático de orina, ECG (isquemia, arritmia, hipertrofia VI), Rx posteroanterior y lateral de tórax. En emergencia hipertensiva: hemograma, bioquímica con CPK, CPK-MB, troponina I (si existe sospecha de SCA), sistemático de orina, ECG, gasometría arterial (GSA) (si existe sospecha de EAP o IC), fondo de ojo (grado de retinopatía), Rx posteroanterior y lateral de tórax, TAC craneal (si existe sospecha de ictus), ecocardiograma (si hay cardiopa a o sospecha de disfunción cardiaca), TAC tóracoabdominal (si existe sospecha de aneurisma disecante de aorta). TRATAMIENTO: Si daño a órgano blanco: Emergencia hipertensiva, Objetivo: Disminuir la PAM en un 25% en un periodo comprendido entre minutos y 2 horas o hasta niveles seguros (excepto en la disección aórtica), si el paciente estable continuar reducción hacia valores normales dentro de 8-24 hrs de admisión. Tratamiento inmediato y hospitalización. Protocolo A, B, C: soporte vital avanzado (SVA) si es necesario. Mantener vía aérea permeable y oxigenoterapia. Pulsioximetría, monitorización de la PA, canalización de vía venosa periférica: 1.Labetalol (disección aortica, IAM, sindromes coronarios agudos): bloqueador mixto β,α, dosis de impregnación 20mg, seguida por una dosis incremental de 20 a 80mg

CURSO ENARM CMN SIGLO XXI TEL: 36246001

Pharmed Solutions Institute

PÁGINA 199

MANUAL DE TRABAJO DEL CURSO ENARM CMN SIGLO XXI con intervalos de 10 min hasta conseguir el efecto deseado. Reducen la mortalidad asociada a arritmias. 2. Nicardipina (encefalopatía hipertensiva, hemorragia intracraneal, EVC): es un calcio antagonista con alta selectividad vascular y fuerte actividad vasodilatadora cerebral y coronaria. La infusión inicial es de 5mg/h con aumentos de 2.5mg/h cada 5min, hasta un máximo de 15 mg/h. 3.Nitroprusiato (EVC, isquemia, encefalopatía, hemorragia subaracnoidea): es un vasodilatador arterial y venoso que disminuye la postcarga y precarga. IV de 0.5.10 mg/kr/min. Es extremadamente potente, útil monitoreo intraarterial, puede generar toxicidad por cianuros a altas dosis o por el uso prolongado, útil en casi todas las crisis hipertensivas, puede provocar aumento de la presión intracraneana. 4. Nitroglicerina (ICC aguda o EAP, IAM, síndromes coronarios agudos): es un venodilatador potente. La dosis es 5 mcg/min con un incremento de hasta 20mg. 5. Hidralacina (embarazo): vasodilatador directo de acción predominante arteriolar, droga de elección en crisis hipertensivas en embarazo. IM de 10-50mgs. Eclampsia, metildopa, MgSO4. NO daño a órgano blanco: Urgencia hipertensiva: Si PAS ≥ 210 mmHg o PAD ≥ 120 mmHg. Objetivo: Disminuir la presión arterial media (PAM) en un 20% en un periodo de 24-48 horas o disminuir la PAD a valores < 120 mmHg. La disminución debe ser gradual a fin de prevenir isquemia orgánica (cardiaca, cerebral). Reposo 5-10 minutos en lugar tranquilo y en decúbito supino. Valorar ansiolítico suave (benzodiacepina) si ansiedad/nerviosismo. Comprobar el nivel de PA. Es importante saber si el paciente estaba recibiendo tratamiento antihipertensivo previo o no: Si estaba a tratamiento y lo ha abandonado, hay que reiniciarlo de nuevo, pero si continúa con la medicación hay que aumentar la dosis del que toma o asociar nuevos fármacos. Si no está a tratamiento y es necesario medicación antihipertensiva, debe emplearse un fármaco de acción lenta (ej. captopril) evitando los de acción rápida porque pueden inducir isquemia en los órganos diana (nifedipino sublingual). Nunca descender la PA por debajo de las cifras habituales del paciente. Tratamiento en primer nivel con medicamentos vía oral: 1. Inhibidores de la enzima convertidora de angiotensina: a. Captopril, a dosis de 25 mg, puede utilizarse por vía oral o sublingual comenzando su acción a los 15-30 minutos, hasta un máximo a los 50-90 minutos, durando su acción 4-6 horas. Si a pesar del tratamiento la PA ≥ 210/120 mmHg se puede repetir la dosis 2-3 veces a intervalos de 30 minutos hasta un máximo de 100 mg. b. Enalapril: 10mg cada 12 horas, dosis máxima 40mg al día 2.Betabloqueadores: a. Metroprolol: 50-100mg cada 12 horas b. Atenolol: 50mg cada 12-24 horas 3. Antagonistas de los receptores de angiotensina II: a. Losartan: 50mg cada 12 horas, b. Candesartan: 8-16mg cada 24 horas, c. Telmisartan: 40-80mg cada 24 horas. 4. Calcioantagonistas: a. Amlodipino: 510mg cada 24 horas, b. Nifedipino: 30-60mg cada 24 horas. 5. Diuré cos azídicos: a. Hidroclorotiazida b. Clortalidona. COMPLICACIONES: Encefalopatía hipertensiva: Ocurre como resultado de un abrupto y sostenido incremento de la TA, que excede los límites de autorregulación de las arterias de resistencia de pequeño calibre en el cerebro (FSC), es de inicio agudo y reversible, se presenta con cefalea intensa, vomito, somnolencia, confusión, puede ocurrir convulciones, déficit neurológicos focales, el edema de papila es frecuente. Se presenta cuando PAM sobrepasa los 160 mmHg aprox., dando lugar a la aparición de isquemia por vasoespasmo marcado, incremento de permeabilidad vascular, hemorragias puntiformes y edema cerebral. La reducción inmediata en un 30-40% revierte el vasoespasmo, pero la disminución excesiva debe ser evitada para prevenir mayor isquemia cerebral, que ocurre cuando la TA cae por debajo del límite inferior de la autorregulación. Debemos tener en cuenta que en sujetos normales este límite se encuentra alrededor de los 60mmHg. Verdadera emergencia médica, sin tratamiento, la evolución es el coma progresivo y muerte. El régimen estándar es nitroprusiato IV, hasta reducción de 25% de PAM, en una hora. Isquemia cerebral aguda: Caracterizado por déficit neurológico de más de 24 hrs de aparición en ausencia de alguna otra causa vascular, se manifiesta por alteraciones visuales, afasia, paresia, hemiparesia, hemiplejia, ataxia-hemiparesia, disartria. Hemorragia intracerebral, 80% de los pacientes son hipertensos crónicos, aumenta el riesgo con cifras elevadas de presión diastólica. En EVC isquémico TAD 120-130mmHg, disminuir 10-15% en 24 hrs. En EVC hemorrágico TAS 200 o TAD >110mmHg, disminuir 25% en 6-12 hrs. Hemorragia subaracnoidea, irrupción súbita de sangre al espacio subaracnoideo craneal o espinal, más común en ganglios basales 40-50%, sustancia blanca subcortical 20%, talamo 15%. Disección aortica, IAM, EAP, Insuficiencia renal. CASO CLINICO Se trata de paciente femenino de 68 años de edad, acude a consulta por presentar cefalea global, acufenos y fosfenos, mareo, refiere además parestesias de miembro toracicco, además hormigueo facial, a la exploración física se observa ansioso y levemente desorientado, TA 180/120 mmHg, FC 132 lpm, FR 41. Se envía al servicio de urgencia donde se le administra nifedipino subligual antes de enviar a segundo nivel. PREGUNTA Cual de las siguientes complicaciones por la maniobra realizada es menos probable para el caso. RESPUESTA a.- Infarto al miocardio. b.- Acidente cerebrovascular. c.- Edema agudo pulmonar. d.- Bloqueo cardiaco. CASO CLINICO Se encuentra paciente de 42 años de edad el cual se conoce hipertenso con buen apego a tratamiento, tiene antecedentes de endarteritis de takayasu, refiere cefalea global, con acufenos y fosfenos, refiere amaurosis fugax y dolor retrosternal ardoroso y opresivo, se observo soplo carotidea, disminución de la fuerza muscular, disminución de la sensibilidad en de miembros, disminución de pulsos, con hipotermia periférica, se observa desviasion de choque de punta, galope, sin de edema pulmonar e imagen de ensanchamiento mediastinal. Sus constantes vitales fueron TA 210/119 mmHg, FC 123 lpm, FR 31 rpm, PREGUNTA Cual de las siguientes complicaciones es la mas probable que se encuentra en este caso? RESPUESTA

CURSO ENARM CMN SIGLO XXI TEL: 36246001

Pharmed Solutions Institute

PÁGINA 200

MANUAL DE TRABAJO DEL CURSO ENARM CMN SIGLO XXI a.- Insuficiencias cardiaca aguda. b.- Disección de la aorta. c.- Bloqueo AV de tercer grado. d.- Encefalopatia hipertensiva PREGUNTA Cual es el objetivo principal del manejo de la presión arterial considerando la complicación con la que cursa actualmente? RESPUESTA a.- Disminuir la capacitancia coronaria. b.- Disminuir el volumen de eyección. c.- Disminuir el volumen circulante. d.- Disminuir la poscarga. CASO CLINICO Paciente masculino de 61 años de edad hipertenso mal controlado y diabetes mellitus, acude a urgencias por cefalea intensa, vomitos intenso, el paciente presenta datos compatibles con delirium caracterizado por agitación psicomotriz, presento vomito importante, su alteraciones neurologías no fueron focalizados y se presento lentamente, durante la exploración clínica presento disartria y hemiparesia, los signos vitales TA 205/120 mmHg, cual es la complicación mas probable que presento el paciente? RESPUESTA a.- Hemorragia cerebral. b.- Encefalopatia hipertensiva. c.- Edema pulmonar. d.- Meningoencefalitis. PREGUNTA Cual es la meta sobre la hipertensión arterial? RESPUESTA a.- Disminuir 25 % de la diastólica. b.- Disminuir 50 % de la diastólica. c.- Disminuir 25 % de la sistólica. d.- Disminuir 50 % de la sistólica. PREGUNTA Cual es tratamiento de primera elección en este caso RESPUESTA a.- Betabloqueadores. b.- Nitroglicerina c.- Nitroprusiato. c.- Calcioantagonistas. PREGUNTA Se agrego al tratamiento diuretico al manejo, el paciente presenta rahs cutáneo en alas de mariposa, cual es la causa de la presencia de este sindrome de lupus Like? RESPUESTA a.- Hidroclorotiazida. b.- Clorotiacida. c.- Furosemide. d.- Hidralacina. PREGUNTA El paciente diminuyo los síntomas durante la estancia en sala de choque, 8 horas después presento incremento de la tensión arterial nuevamente sin datos de focalización, con datos sugestivos de isquemia miocárdica. RESPUESTA a.- Nitroprusiato. b.- Clonidina. c.- Nitroglicerina. d.- Labetolol.

CURSO ENARM CMN SIGLO XXI TEL: 36246001

Pharmed Solutions Institute

PÁGINA 201

MANUAL DE TRABAJO DEL CURSO ENARM CMN SIGLO XXI HIPERTENSIÓN MALIGNA: CIENCIAS BASICAS: Es una hipertensión arterial rápidamente progresiva caracterizada, desde el punto de vista anatomopatológico, por lesiones arteriolares graves y clínicamente por la existencia de presión arterial muy elevada, hemorragias y exudados retinianos y, a menudo aunque no necesariamente, edema de papila. Las cifras de PA suelen ser superiores a 120 mmHg de diastólica, y el concepto de rápidamente progresiva nos indica que de no tratarse cuidadosamente aparecen sucesivas lesiones vasculares en órganos diana que condicionan el pronóstico de la enfermedad y puede llegar a ser fatal. Puede aparecer en cualquier forma de hipertensión, ya sea esencial o secundaria, e incluso puede ser la forma de presentación de una glomerulopatía como la nefropatía IgA o de una vasculitis necrotizante secundaria a un lupus eritematoso sistémico La encefalopatía hipertensiva es la consecuencia de un aumento súbito y generalmente intenso de la PA que cursa con intensa cefalea y alteraciones variables del nivel de conciencia y que es reversible al reducir, mediante el tratamiento, las cifras de PA. Puede acompañar una hipertensión maligna, pero también puede aparecer sin ésta. Se presenta con más frecuencia en individuos previamente normotensos (embarazadas con eclampsia, jóvenes) que en hipertensos crónicos cuyas cifras de PA aumentan progresivamente. SALUD PÚBLICA: Si la hipertensión arterial afecta alrededor de la quinta parte de la población general, la hipertensión maligna apenas alcanza el 1 % de los hipertensos. A medida que la detección y el manejo de la hipertensión ha ido mejorando, su incidencia va disminuyendo, pudiéndose considerar hoy en día como una patología rara. El 80 % de las hipertensiones malignas corresponden a una nefropatía o patología vascular renal. PATOGENIA: Los dos factores de mayor importancia para determinar el inicio de la fase maligna son, probablemente el nivel absoluto de PA y su rapidez de instauración. Aunque pueden intervenir factores hormonales, la fase maligna es, con seguridad una consecuencia no específica de cifras de PA muy elevadas. Las lesiones vasculares que se producen conllevan consecuencias isquémicas tanto a nivel de órgano (riñón) como a nivel de la propia pared vascular, estableciéndose un «círculo vicioso» entre isquemia y PA a ambos niveles. Factores hormonales intervienen en este círculo vicioso, aumentando no sólo la PA, sino también la permeabilidad de la pared vascular. Al 75 % de los pacientes con hipertensión maligna se les detecta una actividad renina plasmática elevada, con el consiguiente aumento de la secreción de aldosterona. La hipersecreción de renina secundaria a los cambios vasculares renales producidos por la elevación intensa de la PA es el mecanismo que sostiene el círculo vicioso al producir, a través de la angiotensina ll, una ulterior vasoconstricción renal con más isquemia y más secreción de renina. Además en este momento se produce una «natriuresis de presión», con la consiguiente reducción del volumen intravascular, que puede a su vez también estimular la secreción de renina y la actividad del sistema nervioso simpático. Las lesiones arteriolares de la hipertensión maligna afectan a muchos órganos: encéfalo, páncreas, corazón, intestino, bazo y riñón, siendo este último el más extensamente estudiad. En el riñón las lesiones más características son la necrosis fibrinoide y la endarteritis proliferativa. La necrosis fibrinoide es posiblemente el resultado de la lesión endotelial por la propia hipertensión, con el depósito de proteínas plasmáticas (fibrina) en el subendotelio. Esta lesión se reconoce como eosinofílica con detritus de células musculares necróticas y restos proteicos en la pared arteriolar. La endarteritis proliferativa (hiperplasia o proliferación miointimal) se caracteriza por un engrosamiento de la íntima, constituido por células musculares lisas, restos de membrana basal y mucopolisacáridos que conlleva un importante estrechamiento de la luz vascular con el consiguiente aumento de la resistencia al flujo de sangre e isquemia parenquimatosa. La lesión más frecuentemente observada en el riñón humano es una arteriosclerosis hiperplásica y una esclerosis glomerular isquémica, y la típica hiperplasia miointimal se observa con relativa frecuencia en el riñón de individuos de raza negra. DIAGNOSTICO: Clínico: Generalmente se caracteriza por un aumento relativamente brusco de la PA, con encefalopatía hipertensiva o sin ella. A diferencia de la hipertensión arterial «no maligna», que acostumbra a ser asintomática, la hipertensión maligna generalmente se acompaña de síntomas diversos como malestar general, cefaleas, visión borrosa, confusión, somnolencia, náuseas, vómitos, disnea, hematuria y oliguria. La cefalea es, sin duda, el síntoma más frecuente y precoz, y no es raro que preceda al resto de la sintomatología unos seis meses. En el 75 % de los casos el cuadro se acompaña de pérdida de peso, como resultado de la isquemia del músculo esquelético y la pérdida excesiva de sal y agua por parte del riñón que se observa en las fases iniciales de la hipertensión maligna. Sin embargo, tampoco es infrecuente observar pacientes que niegan sintomatología alguna, a pesar de presentar insuficiencia renal terminal, insuficiencia cardíaca grave o afectación de las capacidades cognitivas. Acostumbra a presentarse en individuos jóvenes o de mediana edad, y sólo excepcionalmente es una forma de presentación de una hipertensión en edades avanzadas. Gran parte de la sintomatología que presentan estos pacientes depende de las complicaciones acompañantes. De éstas las más propias de la hipertensión maligna son: 1. Encefalopatía hipertensiva: alteraciones neurológicas, normalmente transitorias y fluctuantes, reversibles al controlar la PA, que se manifiesta en forma de cefalea intensa, confusión mental, disfagia, alteraciones visuales y convulsiones. Puede también manifestar alguna focalidad (hemiparesia), en cuyo caso la posibilidad de una hemorragia intracraneal debe ser considerada, patológicamente existe edema cerebral más o menos generalizado. El descenso brusco de la PA durante el tratamiento en estos pacientes con hipertensión maligna puede producir isquemia cerebral por hipoperfusión, si se alcanzan niveles por debajo de una PA media de 100-120 mmHg pues por debajo de estas cifras el paciente es incapaz de autorregular su flujo cerebral. 2. Insuficiencia renal: que recibe el nombre de nefroesclerosis maligna, puede presentarse clínicamente como una insuficiencia renal progresiva o como una insuficiencia renal aguda, fases iniciales estos enfermos pueden presentar poliuria y polidipsia, con hiponatremia e hipopotasemia. Los factores que inducen estos trastornos son en parte hemodinámicos y en parte hormonales. Entre los primeros destacan la «natriuresis de presión» y entre los segundos el hiperreninismo e hiperaldosteronismo secundario a la isquemia renal, la hipertensión maligna puede clínicamente presentarse como un síndrome nefrítico agudo. 3. Anemia hemolítica microangiopática: es la traducción de un daño endotelial generalizado. Se manifiesta en forma de hemolisis intravascular con esquistocitosis, descenso de la haptoglobina, aumento de reticulocitos y evidencia de coagulación intravascular. La principal exploración que requiere un paciente con sospecha de hipertensión maligna es la exploración del fondo del ojo. Los hallazgos de una verdadera retinopatía hipertensiva (grado lllIV) son de capital importancia para el diagnóstico, el pronóstico y el tratamiento del paciente. En el fondo del ojo grado lll el calibre arterial es muy inferior al venoso (espasmo arterial generalizado), con espasmos focales, apariencia de «hilo de plata» en cuanto al reflejo luminoso, con desplazamiento venoso, cruces en ángulo recto y dilataciones venosas distales. Típicamente aparecen hemorragias y exudados. Las hemorragias pueden ser de dos tipos: «en llama», que son superficiales y están interpuestas entre las fibras nerviosas, y en forma de manchas y puntos, de situación más profunda que las fibras nerviosas. Las hemorragias son un signo de lesión vascular grave y reciente, que desaparecen por lo común a las pocas semanas del control efectivo de la presión arterial. Los exudados son también de dos tipos: exudados «duros» debidos a extravasación por lesión vascular entre las fibras nerviosas que pueden tener una distribución radial alrededor de la mácula («estrella macular»), el líquido se reabsorbe, los macrófagos aclaran el

CURSO ENARM CMN SIGLO XXI TEL: 36246001

Pharmed Solutions Institute

PÁGINA 202

MANUAL DE TRABAJO DEL CURSO ENARM CMN SIGLO XXI residuo proteico-lipídico y queda un depósito hialino que algunas veces puede persistir, y b) exudados «blandos» o algodonosos, que son mayores que los anteriores y de límites bien definidos, no son verdaderos exudados, sino fibras nerviosas infartadas por oclusión vascular que pueden tener una apariencia lacunar. Estas lesiones desaparecen a las pocas semanas del tratamiento adecuado. En el grado IV a las lesiones anteriores se añade papiledema. A la distensión venosa le sigue un aumento de vascularización del disco óptico, que aparece de color rosado, con límites borrosos. Alrededor de la papila hay hemorragias radiales y exudados algodonosos. PRONÓSTICO: La principal causa de muerte era la hemorragia cerebral o la insuficiencia cardíaca. Con un tratamiento inmediato y eficaz es posible lograr una supervivencia superior al 70 % a los cinco años, con una remisión, generalmente parcial, del cuadro tanto clínico como biológico. TRATAMIENTO: En la hipertensión maligna incurren varias circunstancias que la hacen catalogarse como una emergencia hipertensiva, a saber: encefalopatía hipertensiva, insuficiencia cardíaca, insuficiencia renal aguda y la anemia hemolítica microangiopática. Ello requiere una actuación inmediata y, por tanto, la administración de fármacos por vía parenteral. El objetivo del tratamiento es bajar la PA para evitar la aparición o agravación de la repercusión orgánica sin llegar, durante las primeras 24 horas, a niveles de normotensión. La PA ha de bajarse más o menos rápidamente según la intensidad de los signos y síntomas secundarios a la lesión orgánica producida por la hipertensión. CASO CLINICO Inicio con cefalea, visión borrosa, con dolor toraccico opresivo, el paciente es diabético e hipertenso con mal apego a tratamiento, la diatolica se observo en 115, refirió disminución de la agudeza visual, con necrosis fribinoide, con hemorragia de la retina, y exsudados algodonosos, sus antecedentes fueron tabaquismo, anemia hemolítica, con cardiomegalia sin datos de edema pulmonar. Uremia, Anteriormente presento hemorragia cerebral hace 2 años. PREGUNTA El paciente recibió previamente al traslado recibió dihidropiridina sublingual, cual es el tratamiento mas adecuado en este caso? TRATAMIENTO a.- Nitroprusiato de sodio. b.- Nitroglicerina. c.- Diuretico de asa. d.- Dihidropiridina.

CURSO ENARM CMN SIGLO XXI TEL: 36246001

Pharmed Solutions Institute

PÁGINA 203

MANUAL DE TRABAJO DEL CURSO ENARM CMN SIGLO XXI CAUSAS FRECUENTES DE DESCOMPENSACION DE IC

INSUFICIENCIA CARDIACA DESCOMPENSADA NO CARDIACAS: CIENCIAS BASICAS: Se trata de un estado fisiopatológico resultado del Falla de adhesión al tratamiento dietético o farmacológico Aumento de las demandas metabólicas: anemia, fiebre, embarazo, fracaso de los mecanismos de compensación ante la dificultad del corazón estrés) para llenarse (disfunción diastólica) o bien para bombear sangre (disfunción Prescripción reciente de fármacos (amiodarona, AINES) sistólica), con el objetivo de mantener un gasto cardíaco acorde a los Exceso de alcohol requerimientos metabólicos y que termina por afectar a otros sistemas Insuficiencia Renal corporales. La insuficiencia cardíaca (IC) es la vía final común de numerosas Embolismo de pulmón Hipertensión mal controlada (crisis) enfermedades cardiovasculares. Las principales causas de insuficiencia CARDIACAS: cardíaca son la hipertensión arterial y la cardiopatía isquémica, procesos a Taquiarritmias menudo concomitantes, seguidos de miocardiopatías y disfunciones Bradiarritmias valvulares, en general, cualquier proceso que altere aguda o crónicamente la Isquemia miocárdica Valvulopatía de nueva aparición o empeoramiento de la existente función cardíaca podría ser causa de IC. SALUD PÚBLICA: Su prevalencia Excesiva reducción de la precarga (diuréticos, IECAS) aumenta con la edad, siendo por tanto una enfermedad relacionada con el envejecimiento poblacional. Constituye en nuestro medio la principal causa de hospitalización en mayores de 65 años. Es el único de los grandes síndromes cardiovasculares cuya incidencia se encuentra en aumento. Cerca del 1% de la población mayor de 40 años padece IC, proporción que se incrementa en pacientes mayores de 65 años a un 6-10%. La forma más CRITERIOS DE GRAVEDAD común de IC es la crónica, con ocasionales descompensaciones agudas. PATOGENIA: La rapidez de Debut de IC la instauración del fracaso cardíaco condiciona su presentación. Ante una lesión cardíaca aguda Edad >70 años pueden aparecer síntomas severos de conges ón (edema agudo de pulmón) o de hipoperfusión Comorbilidades severas Enfermedad valvular severa conocida periférica (shock cardiogénico). Sin embargo, en la mayor parte de los casos, nos encontraremos Disnea de rápida instauración ante un paciente con clínica de descompensación de una IC crónica. DIAGNOSTICO: Según la Angina, sincope o déficit neurológico sociedad europea de cardiología, se define la IC como el síndrome clínico en el que el paciente Mala perfusión periférica presenta los siguientes hallazgos: 1. Síntomas típicos manifestados en la anamnesis. Derivados de Oliguria (30 rpm) venosa, edema en miembros inferiores y zonas declives, distención y dolor abdominal, nicturia. Estertores crepitantes gruesos (medios) Por bajo gasto, fatigabilidad, astenia, depresión, mareos, sincope. 2. Signos típicos evidenciados en Auscultación R3 la exploración física, por IC izquierda, estertores crepitantes húmedos, bilaterales y de comienzo HTA severa (crisis) o hipotensión Saturación sin O2 >90% en base, hipoventilación sibilancias. Por IC der., edemas en zonas declives, distención abdominal, Cambios en ECG ascitis, hepatomegalia, tinte ictérico, caquexia, ingurgitación venosa yugular, reflujo Taquicardia mayor de 100-120 lpm hepatoyugular. Signos cardiacos, soplos, arritmias, signo de galope y por bajo gasto (palidez, Bradicardia extrema cianosis, sudoración, alteración en la conciencia). La taquicardia, los crepitantes pulmonares y edema con fóvea, son frecuentes en la IC. 3. Evidencia objetiva de una anormalidad estructural o funcional del corazón en reposo mediante pruebas complementarias. 4. respuesta al tratamiento con diuréticos y vasodilatadores, en los casos en que existen dudas con los criterios anteriores. INSUFICIENCIA CARDIACA FORMAS CLINICAS: A) Insuficiencia cardiaca de novo o descompensación aguda de ICC= disnea o taquicardia con congestión pulmonar o edema intersticial en la RX (65.4%). B) Edema agudo de pulmón (EAP)=IC acompañada de edema alveolar en Rx de tórax o SAT O2 >90% (16.2%). C) Shock cardiogénico=ICA con hipotensión TAS 180/100 mmHg con síntomas de IC (disnea, taquicardia) y Rx con congestión pulmonar (11.4%). E) Insuficiencia cardiaca derecha= IC con aumento de presión venosa yugular y hepatomegalia, acompañada de edema pulmonar. TRATAMIENTO EN URGENCIAS DE ATENCION PRIMARIA: A). Sin compromiso vital inmediato: Grados I-III, sin criterios de inestabilidad y en ausencia de complicaciones agudas. Posición semisentada, examinar vía aérea, monitorización, ECG (12 derivaciones), TA y Sat O2 (conseguir saturación mayor a 90%). Control, de temperatura. Obtener vía venosa, restricción de líquidos, tratamiento diurético con furosemida a 40mg (2 ampollas) vía endovenosa. Efecto comienza a los 5-10 minutos y es máximo a los 30 min. Puede ser IM cuando no hay posibilidad IV. Si paciente mejora y no hay datos de gravedad, estabilidad hemodinámica (TAS >160mmHg Y >90 mmHg y FC >100 lpm), mejoría sintomática tras tratamiento, y buen soporte familiar y/o social, se remitirá el paciente a su domicilio (con ajuste de tratamiento diurético, para posterior control ambulatorio. Evitar sobrepeso, dieta hiposodica. Se puede usar IECAS, ARA II, digoxina, β-Bloqueaores. Si no mejora trasladar a hospital, tratamiento de ingreso Admon., oxigeno (>90%), digoxina 1 amp en bolo y luego 1 amp cada 6-8 hrs en 24 hrs, furosemida 1 amp. IV, posteriormente 1 amp c/8-12 hrs o en perfusión. IECA renitec 5-20mgs c/24 hrs o atacand 16 mg c/24 hrs VO. Betabloquenates contraindicados en descompensaciones graves. Dobutamina o vasodilatadores IV (nitroglicerina, nitroprusiato), si perfusión no adecuada o no diuresis. Medidas coadyuvantes; amiodarona único seguro en IC izq. B) Con compromiso vital inmediato: Grado IV, grado III con inestabilidad hemodinámica o complicación aguda, puede ser IAM, taquicardia ventricular o alguno de los criterios de gravedad, proporcionar medidas para evitar paro cardiorrespiratorio. Edema agudo de pulmón cardiogénico (EAP); es un cuadro de inicio brusco provocado por el aumento de la presión en la microcirculación pulmonar, secundario a insuficiencia aguda del ventrículo izquierdo, con la consiguiente acumulación de líquidos en el intersticio pulmonar y alveolos. En general responde adecuadamente al tratamiento precoz, por lo que es necesaria una actuación inmediata, sin demorarase en el examen físico, ni en espera de pruebas complementarias. El objetivo es; mantener una Sat02 normal (95-98%), para maximizar la liberación de O2 a los tejidos, ayudando a prevenir la disfunción de órganos distales y el fallo multiorgánico. Los datos de sospecha son disnea de aparición brusca, ortopnea, disnea paroxística nocturna, tos con expectoración rosada, oliguria, diaforesis profusa, hipotensión, cianosis periférica, tercer ruido, soplo cardiaco, taquicardia, sibilancias en fase intersticial, crepitantes bilaterales en fase alveolar, si hay insuficiencia de ventrículo derecho hay IY, hepatomegalia y edemas periféricos. El tratamiento es posición semisentada/incorporación con piernas colgando. Admon., de O2 (>90%). CPAP; (Funciones: la presión positiva conlleva a una menor presión intrapleural negativa durante la inspiración, lo que disminuye la presión de llenado del VD, y con ello mejorando el edema y la congestión pulmonar. La hiperinsuflacion pulmonar que se produce, aumenta el tono parasimpático, y con ello disminuye la frecuencia cardiaca. En pacientes con función sistólica deprimida va a producir un aumento de la FE del VI y una disminución de la poscarga. En pacientes con función sistólica normal, va a producir un descenso del volumen diastólico final); iniciar con 5 cmH2O y aumentos de 2-3 cm, el objetivo es mejoría del disconfort y la disnea, FR >25 rpm, desaparición actividad de músculos accesorios, disminuir la morbimortalidad, disminuir la necesidad de IOT, estancias prolongadas en pacientes con ventilador

CURSO ENARM CMN SIGLO XXI TEL: 36246001

Pharmed Solutions Institute

PÁGINA 204

MANUAL DE TRABAJO DEL CURSO ENARM CMN SIGLO XXI mecanico posteriormente progresar y retirar continuar con oxigenoterapia convencional o de lo contrario si empeora Intubación orotraqueal (IOT). Obtener vía venosa, llave de 3 pasos. Monitorización de ECG, TA y Sat O2. Control de diuresis. Furosemida (I-B) 4080mgs, repetir en función de respuesta, antes de producir diuresis tiene efecto venodilatador. Primera línea de tratamiento los nitrataos; Nitroglicerina (I-B) si TAS >90-95mmHg, mejora la congestión pulmonar sin aumentar el consumo de oxigeno miocárdico, inicialmente se utiliza la via sublingual, aunque se existe HTA severa o isquemia miocárdica refractaria se administrara vía IV. La dosis sublingual es de 0,4-0,8 mg/5-10min. Máximo de 3 comprimidos con intervalos de 5 minutos o 2 aplicaciones de nitroglicerina en spray, es mejor que el nitroprusiato por el efecto de “robo coronario”. Levosimedan (IIa-B); es un sensibilizador al calcio, que abre los canales de K, con efectos inotrópicos, metabólicos y vasodilatadores en el tratamiento de la ICA, con una mejoría clínica y hemodinámica rápida, esta indicado en la IC de bajo gasto descompensada, IC secundaria a cardiopatía isquémica, IC postoperatoria, Shock cardiogenico. Morfina es dilatador mixto, fundamentalmente venoso, reduciendo el retorno venoso y la presión de la aurícula izquierda. Dosis de 3-5mg IV que pueden repetirse en 5-10 min. Diluir una ámpula de cloruro mórfico (1ml) en 9 ml de SF, pasar 3-5 ml en 2-3 min cada 5 min hasta un máximo de 15 ml. Disponer de antagonistas de opiáceo (naloxona) y de atropina. Valorar detenidamente la administración de morfina en pacientes con insuficiencia respiratoria crónica y/o si existe disminución del nivel de conciencia. Dopamina y dobutamina indicado en ICA con hipoperfusión periférica (hipotensión, fracaso renal o signos cutáneos de mala perfusión renal), uso restringido a UCI. Dopamina diferente efecto según dosis: 5µ/kg/min sobre receptores alfa: aumento de resistencias vasculares y aumento de TAS. CASOS CLINICOS Masculino de 36 años de edad, desempleado. Tabaquista de 20cig/d durante 15 años. Obeso con un índice de masa corporal (IMC) Dislipidemia relacionada con el aumento del Índice de Masa Corporal) de 39. Sin patologías previas conocidas. Buena clase funcional habitual. Comienza 15 días previos a la consulta con cuadro de comienzo insidioso, caracterizado por disnea en clase funcional II (caminar 200 metros), asociado a tumefacción de miembros inferiores a predominio vespertino. También refiere de un mes de evolución, disnea paroxística nocturna, en número de 2 a 3 episodios semanales, que aumentan en frecuencia en los últimos días. EF: orientado. Peso: 120 kg Talla: 1,75 metros. (IMC): 39, TA: 130/80 mmHg. (FC): 90 latidos por minuto (lpm). Temperatura: 36,7º C. (FR): 15 respiraciones por minuto. Ingurgitación yugular 3/6 con colapso inspiratorio incompleto. Murmullo vesicular conservado con rales crepitantes bilaterales hasta tercio inferior. Latido apexiano en 5º espacio intercostal izquierdo, línea hemiclavicular. Ruidos cardíacos hipofonéticos, sin R3 ni R4. Soplo protomesositólico en foco mitral con irradiación a axila. Abdomen, globuloso, blando, indoloro, con hepatomegalia a 3cm del reborde costal, no dolorosa. Miembros inferiores con tono, trofismo y pulsos conservados; con edema a tensión desde raíz de miembros. Electrocardiograma: ritmo sinusal 90 latidos por minuto (lpm). QRS angosto. Sobrecarga de cavidades izquierdas, aurícula y ventrículo, con trastorno de repolarización en cara lateral secundario a hipertrofia de VI. Figura. Laboratorio: Hematocrito: 36%- Hemoglobina: 1,9 g/dl- Glóbulos Blancos: 12.840 /mm3 – Plaquetas: 330.000 /mm3 – Glicemia: 93 mg/dl – Sodio: 139 mEq/l – Potasio: 4,04 mEq/l – Urea: 27 mg/dl – Creatinina: 0,72 mg/dl – Bilirrubina total: 0,43 mg/dl – LDH: 348 UI/L – GGT: 39 UI/L – FAL: 187UI/L – TGO: 14 UI/L – TGP: 20 UI/L. Radiografía de Tórax: relación cardiotorácica conservada. Hipertensión venocapilar pulmonar grado III. Infiltrados algodonosos bilaterales a predominio de campos pules inferiores y medios. Sin derrame pleural. PREGUNTA Cual es la conducta a sintomática seguir mas adecuada? RESPUESTA a.- Oxigenoterapia. b.- Diureticos. c.- Antiarritmicos. d.- Digitalicos. CASO CLINICO Paciente femenino de 60 años, que presenta dolor de pecho, durante la preparación para colonoscopia, cuadro de deposiciones líquidas abundantes, con episodio de mareo, ortostatismo, sudoración fría, palpitaciones y dolor opresivo, retro-esternal, irradiado a región interescapular. Tras la realización del procedimiento e infusión de 2000 cc de cristaloides, la paciente mejora, pero persiste la molestia precordial. A su ingreso se encuentra hemodinámicamente estable TA 125 / 65 mmHg, (FC) 85 latidos por minuto (regular), eupneica con buen intercambio gaseoso. En el electrocardiograma (EKG) se evidencia ritmo sinusal, con imagen de hemibloqueo anterior de rama izquierda (HARI), sin signos de isquemia aguda en el momento actual. Se solicitan pruebas complementarias, con enzimas de daño miocárdico discretamente elevados (CPK 282, CPK-MB 42; Troponina T 0.67). Ingresa en UCI para monitorización y tratamiento. EF: Constantes: TA 110 / 65 mmHg; (FC) 80 latidos por minuto, FR: 12 (rpm). Saturación de oxígeno (SatO2) 98% con gases normales. Bien perfundida, normohidratada, normocoloreada. Buen relleno capilar. Buena perfusión distal. Eupneica, sin sensación de disnea. Afebril, no signos de sepsis actual. Ligera molestia precordial, que actualmente refiere que aumenta con movimientos respiratorios. Consciente, orientada, colaboradora. Auscultación cardiaca (AC): Rítmica a 80 latidos por minuto (lpm), soplo sistólico irradiado a punta cardiaca. (Hb) 14.9; (Htco) 41.2%; Leucocitos 9630 (Neutrófilos: 80%); Plaquetas 210000. Bioquímica: Cr 0.8; K 3.81; CPK 282; CPK 42.7; Troponina T 0.67. (EKG): ritmo sinusal (RS). PR normal. QRS/QT normal. Hemibloqueo anterior de rama izquierda (HARI). QS en V1-V2. No alteraciones ST - T sugestivas de isquemia y/o sobrecarga de cavidades. Rx tórax: índice cardiotorácico (ICT) normal. Estructuras pleuromediastínicas normales. No infiltrados consolidativos ni signos de sobrecarga hídrica. PREGUNTA Cual de las siguientes medidas terapéuticas es menos apropiadas al pronostico inmediato en el caso? RESPUESTA

CURSO ENARM CMN SIGLO XXI TEL: 36246001

Pharmed Solutions Institute

PÁGINA 205

MANUAL DE TRABAJO DEL CURSO ENARM CMN SIGLO XXI a.- Antiagregación con ácido acetilsalicílico (AAS). b.- Tratamiento antianginoso con B-bloqueantes. c.- Hipolipemiantes. d.- Anticoagulación con heparina de bajo peso molecular (HPBPM). CASO CLINICO Presentamos a una paciente femenina, blanca, 43 años de edad, con antecedentes personales de salud y madre hipertensa; desde agosto del año 2009 presenta episodios de tos seca, nocturna, acompañada de falta de aire progresiva a los esfuerzos, hasta presentarse en el decúbito, con alivio al sentarse, se constatan como elementos positivos al examen físico: edemas de miembros inferiores, frecuencia respiratoria de 24 inspiraciones por minuto, vibraciones vocales disminuidas hacia la base del hemitorax derecho, ligera matidez a la percusión a ese nivel, estertores crepitantes en base derecha y sibilantes en ambos hemotórax, latido de la punta visible y palpable en sexto espacio intercostal izquierdo por fuera de la línea media clavicular de ese lado; ruidos cardíacos taquicárdicos y de buena intensidad, reforzamiento del segundo ruido en focos pulmonar y mitral, soplo sistólico II/VI en foco pulmonar. Tensión arterial sistólica (TAS) 120 mmHg y tensión arterial diastólica (TAD) de 75 mmHg, frecuencia cardiaca (FC) de 140 latidos por minuto, ingurgitación yugular, reflujo hepatoyugular, hepatomegalia dolorosa de 4 cm, superficie lisa y borde romo. Complementarios de hematología y hemoquímica dentro de límites normales. Ultrasonido abdominal: Hepatomegalia de 3 cm, superficie lisa, con moderado aumento difuso de su ecogenicidad. Rx de tórax con cardiomegalia ligera, congestión hiliar bilateral con redistribución del flujo hacia los vértices (líneas B de Kerley, borramiento del seno cardiofrénico derecho; en el ecocardiograma contractilidad global y segmentaria conservada, dilatación de las cuatro cavidades, regurgitación mitral, aparatos valvulares estructuralmente normales, grosor de las paredes ventriculares normal, flujograma pulmonar grado I, con fracción de eyección del 40%. En el electrocardiograma (ECG), taquicardia sinusal, frecuencia cardiaca (FC) de 140 latidos por minuto. PREGUNTA Con estos elementos cual es la impresión diagnóstica mas adecuada? RESPUESTA a.- Miocardiopatia restrictiva. b.- Miocardiopatia constrictiva. c.- Miocardiopatía dilatada. d.- Miocardiopatia mixta. PREGUNTA Considerando la ICC cual es el abordaje mas apropiado? RESPUESTA a.- Diuréticos, IECA y betabloqueadores. b.- Diureticos, ARA II y Calcioantagonista. c.- Diuretico k ahorrador, IECA, Calcioantiagonista. d.- Diuretico K no ahorrador, ARA II, Calcioantagonista.

CURSO ENARM CMN SIGLO XXI TEL: 36246001

Pharmed Solutions Institute

PÁGINA 206

MANUAL DE TRABAJO DEL CURSO ENARM CMN SIGLO XXI TAPONAMIENTO CARDIACO: CIENCIAS BÁSICAS: Urgencia vital, es la compresión del corazón que resulta de la acumulación de líquido en el saco pericárdico y que produce un severo trastorno hemodinámico. La compresión cardíaca lleva a una disminución del gasto cardíaco y de la presión arterial en un espectro variable en el que el paciente severamente comprometido puede estar en shock cardiogénico. Puede ser de origen traumático o secundario a casi cualquier tipo de pericarditis aguda o crónica, aunque es poco CAUSAS MÉDICAS DE TAPONAMIENTO CARDÍACO común en la pericarditis viral y después de IAM. La causa más frecuente de taponamiento Taponamiento cardíaco agudo cardíaco agudo es el hemopericardio causado por heridas con arma de fuego, arma blanca o Hemopericardio trauma cerrado de tórax. Si la herida penetrante es relativamente pequeña suele haber Ruptura de la pared libre del ventrículo taponamiento cardíaco, y el aumento de presión en el saco pericárdico puede ayudar a reducir postinfarto de miocardio Aneurisma aórtico roto a cavidad la severidad de la hemorragia. En el caso del infarto, la ruptura de la pared libre está asociada a pericárdica la extensión de la necrosis miocárdica y tiene una mortalidad cercana al 90%. PATOGENIA: En Disección aórtica al pericardio condiciones normales la presión venosa periférica es superior a la presión venosa central y a su Neoplasias (CA metastasico de pulmón y vez ésta es mayor que la presión intrapericárdica en 5 Torr. Esta diferencia de presiones permite seno, linfoma, leucemias, mesotelioma) y favorece el flujo sanguíneo continuo desde el sistema venoso hacia el corazón derecho. Al Tratamiento anticoagulante Postoperatorio cirugía cardíaca (fugas, acumularse líquido en el espacio pericárdico se eleva la PVC y disminuye el gradiente entre la anicoagulacion, aneurismas, disecciones) presión de la aurícula derecha y el sistema venoso periférico; cuando la presión intrapericárdica Pericarditis aguda llega a 10 Torr se iguala a las presiones venosa central y venosa periférica, y de este punto en Bacteriana adelante, si se eleva la presión intrapericárdica, las otras dos también. La consecuencia de esta Tuberculosa Urémica igualdad en las presiones es que el ventrículo derecho pierde la presión efectiva de distensión y Enfermedades del tejido conectivo se colapsa, tanto por acción directa del líquido intrapercárdico como por la pérdida de flujo Taponamiento cardíaco crónico desde la aurícula derecha. En estas condiciones, el flujo hacia la circulación pulmonar, así como Causas infecciosas. Tuberculosis. Algunas el volumen diastólico que llega al ventrículo izquierdo, disminuyen y se reduce el gasto cardíaco. parasitosis. Causa no infecciosa. Urémico. En esta situación los mecanismos homeostáticos para tratar de mantener la presión de Neoplásico (posirradiacion) Mixedematoso. Postirradiación. Síndrome de Dressler. perfusión de los órganos vitales son tres: la utilización del volumen residual diastólico en el Síndrome postpericardiectomía. corazón, el aumento de la frecuencia cardíaca y la vasoconstricción arteriolar a nivel del músculo Quilopericardio estriado y el riñón. En condiciones normales el volumen del líquido pericárdico es aproximadamente 20 ml, como una capa de 5 mm que recubre el corazón. El espacio pericárdico, al igual que el pleural, tiene presión negativa que se aumenta con la inspiración y se hace ligeramente positiva durante la espiración. Cuando se añade líquido al espacio pericárdico la presión aumenta en forma lenta, mientras la membrana pericárdica permanece distensible, pero una vez que se encuentra a tensión la presión aumenta en forma rápida. Estos eventos son diferentes dependiendo de si el taponamiento cardíaco es agudo o crónico. Cuando la acumulación de líquido es aguda, la capacidad de distensibilidad pericárdica es muy baja y con apenas 100 ml de líquido se produce taponamiento cardíaco. Cuando la acumulación de líquido es lenta, como ocurre en los procesos crónicos, el pericardio se va adaptando y aumenta en forma progresiva su distensibilidad, por lo cual recibe cantidades tan grandes como 2.000 ml de líquido. Tal capacidad de distensibilidad del espacio pericárdico se debe a su estructura de tejido conjuntivo denso, el cual permite algún estiramiento al estar sometido a elevaciones de presión progresivas. Esto es especialmente importante en los pacientes jóvenes. DIAGNOSTICO: Con el aumento de la presión venosa se detecta IY severa, la cual disminuye en forma visible durante la sístole, por lo cual es posible encontrar la depresión X del pulso venoso magnificada sin que se modifique la depresión Y (signo de Friedrich). Con la acumulación de líquido en el espacio pericárdico la intensidad de los ruidos cardíacos disminuyen y llegan en ocasiones a no ser audibles. La presión arterial se encuentra generalmente disminuida y convergente (presión arterial diferencial menor de 30 mmHg) y el paciente está taquicárdico. El pulso paradójico es un signo casi constante en el taponamiento cardíaco, lo mismo que el signo de Kussmaul. En el taponamiento cardíaco, generalmente aquejan dolor torácico sordo o constrictivo que puede ser pleurítico. La disnea es frecuente, síntomas asociados con bajo gasto cardíaco, como frialdad, debilidad muscular y diafóresis, y puede en los casos graves, exhibir alteración neurológica del estado de conciencia y shock cardiogénico. En el enfermo con taponamiento cardíaco crónico los síntomas predominantes son los de la enfermedad de base y se pueden encontrar pérdida de peso, anorexia, debilidad marcada y compromiso del estado general con disnea y dolor torácico crónicos. En estos pacientes es frecuente encontrar signos de congestión venosa crónica como hepatomegalia, ascitis, y edema de miembros inferiores. Triada clínica del taponamiento agudo por hemorragia intrapericárdica aguda: 1. descenso de la presión arterial sistémica 2. Aumento de la presión venosa sistémica 3. Corazón chico y tranquilo. La Rx., del tórax aunque no brinda evidencia directa puede haber cardiomegalia, evidencia de derrame pericárdico y se puede obtener información de procesos patológicos primarios que son causa de tapónamiento, como neoplasia o tuberculosis. El ECG tampoco ofrece datos específicos, es usual encontrar taquicardia sinusal, bajo voltaje y anomalías inespecíficas de la repolarización ventricular como la elevación cóncava del segmento ST. Otros hallazgos pueden ser el desnivel inferior del segmento PQ y la alternancia eléctrica, la cual indica la presencia de derrame pericárdico, pero no es concluyente. El ecocardiograma es el método diagnóstico que más información aporta. En primer lugar, detecta la presencia de derrame pericárdico y permite hacer una aproximación de su cantidad, ofreciendo además múltiples signos de compromiso hemodinámico. La manera más simple de documentar el taponamiento cardíaco es por medio de un catéter colocado en la aurícula derecha, con el cual se pone en evidencia el aumento de la PVC y se puede obtener una curva que revele los cambios antes mencionados en el pulso venoso. Con el catéter de Swan-Ganz se pueden obtener, además, curvas de presión de la arteria pulmonar, mediciones de la presión pulmonar en cuña y una estimación del gasto cardíaco por termodilución, lo cual es de gran utilidad en el seguimiento y la evolución de los pacientes más graves. TRATAMIENTO: Extracción de líquido para eliminar la compresión del corazón, por medio de la pericardiocentesis o por métodos quirúrgicos. La elección del tipo depende de la disponibilidad de personal capacitado, de medios de ayuda pero, principalmente, de la enfermedad que llevó al taponamiento cardíaco. Se prefiere la cirugía en los casos de hemopericardio en que se desea evitar la repetición de hemorragias y está indicada en la pericarditis purulenta, con el fin de realizar drenaje completo y controlar la infección. La pericardiocentesis con guía ecocardiográfica u otro po de imagen, como la fluoroscopia o la TC, es buena alternativa en prácticamente todas las enfermedades pericárdicas. No debe olvidarse la probabilidad de recurrencia del taponamiento cardíaco cuando se trata con pericardiocentesis, por lo cual deberá repetirse el procedimiento o hacer cirugía. Son de utilidad medidas terapéuticas de sostén como los líquidos intravenosos, en especial en casos de hemopericardio traumático agudo, cuando el paciente está hipovolémico. Esta medida aumenta el volumen

CURSO ENARM CMN SIGLO XXI TEL: 36246001

Pharmed Solutions Institute

PÁGINA 207

MANUAL DE TRABAJO DEL CURSO ENARM CMN SIGLO XXI circulante, incrementa las presiones venosa central y pulmonar y la presión arterial. En ningún caso deben aplicarse vasodilatadores, como el nitroprusiato de sodio por vía intravenosa. Tratamiento quirúrgico por tres métodos. El primero pericardiectomía por esternotomía vertical o transversa, útil en pacientes con posible constricción (radiación o tuberculosis). El segundo la pericardiectomía parietal a través de toracotomía intercostal izquierda, que se lleva a cabo bajo anestesia general y posibilita la resección de gran parte del pericardio parietal anterolateral, lo cual permite una mejoría más definitiva. Es de utilidad en caso de pericarditis purulenta. El tercero la pericardiostomía subxifoidea, que se efectúa bajo anestesia local con resección del apéndice xifoides y una incisión pequeña en el pericardio, a través de la cual se drena el líquido y se conoce como procedimiento de ventana se puede liberar adherencias pericárdicas, obtener biopsias del pericardio y dejar una sonda de drenaje, en caso de taponamiento no constrictivo y reversible (pericarditis por diálisis o idiopática). La pericardiocentesis tiene ventajas como su rápida aplicabilidad, la facilidad de estudiar el líquido, la factibilidad de combinarla con estudios hemodinámicos y su eficacia en las 2/3 partes de los casos. Las desventajas consisten en la necesidad de personal bien capacitado en la técnica y de ecografía de buena calidad para obtener mejores resultados; además, no siempre permite llegar a un diagnóstico que requiera biopsia pericárdica, no garantiza su eficacia en todos los casos, puede retardar la intervención quirúrgica por el alivio temporal que proporciona y puede producirse hemopericardio por la punción del corazón. Sin embargo, la pericardiocentesis puede llegar a ser una medida salvadora en los casos de taponamiento cardíaco agudo. Se aconseja como premedicación a este procedimiento la administración de atropina (0,8 a 1,0 mg) por vía intravenosa o intramuscular, a fin de prevenir reacciones vasovagales. La mejor ruta de acceso para la pericardiocentesis es la subxifoidea, pues evita lesiones de las arterias coronarias. Previa asepsia con alcohol yodado y bajo anestesia local (en los casos urgentes se omite la anestesia) se efectúa la punción 5 cm por debajo de la punta del apéndice xifoides y 1 cm a la izquierda de la línea media, con la aguja en ángulo de 45 grados y dirigida hacia el hombro izquierdo. Se ejerce succión continua mientras se introduce la aguja, deteniéndose cuando se perciba una sensación de vencimiento de una resistencia lo cual indica la entrada al espacio pericárdico, hasta que se obtenga líquido o cuando se sientan las pulsaciones cardíacas transmitidas a la aguja, lo que significa que se ha tocado el miocardio y se debe retirar un poco la jeringa. El líquido pericárdico de aspirarse con lentitud durante 10 a 30 minutos y enviarse a estudio al laboratorio clínico de acuerdo con el caso. El condiciones electivas puede emplearse el electrocardiograma para evitar lesiones del miocardio. En caso de taponamiento cardíaco por herida del corazón, la extracción de algunos centímetros de sangre mejora el retorno venoso y puede salvar al paciente. La sangre obtenida no coagula porque los movimientos del corazón la desfibrinan rápidamente; si la sangre obtenida se coagula es porque se puncionó una cavidad del corazón. El tratamiento médico del taponamiento cardíaco agudo, incluido el soporte inotrópico, con o sin vasodilatadores, es relativamente controversial. La dobutamina, administrada para revertir la hipotensión, es teóricamente ideal. Durante el taponamiento, de cualquier manera, la es mulación endógena inotrópica del corazón es muchas veces máxima. CASOS CLINICOS Mujer de 81 años, sin antecedentes de interés, que ingresó con el diagnóstico de insuficiencia cardiaca desencadenado por infección respiratoria, con un cuadro inespecífico de 1 mes de evolución, de malestar y astenia, y los días previos al ingreso había aparecido disnea de esfuerzo, que llegaba a ser de reposo. Refería tos escasa, sin expectoración y sin fiebre. Laboratorios: hemoglobina, 11,9 g/ dl, con hematocrito, 36,8%, y leucocitosis, con 16,8. 705,109 plaquetas/l, y fibrinógeno, 464 mg/dl. La radiografía de tórax mostraba cardiomegalia con ligera redistribución en ambas bases. En el ECG, reducción en el voltaje del QRS y un aplanamiento difuso de las ondas T. A las 24 h presentó un deterioro clínico con datos clínicos y hemodinámicos, hipotensión, pulso paradójico y datos de presión venosa elevada. PREGUNTA Cuál es la conducta a seguir mas apropiada. RESPUESTA a.- Pericardiocentesis. b.- Pericardientomia urgente. c.- Manejo conservador. d.- Corrección del estado hemodinamico. CASO CLINICO Mujer de 69 años de edad que sufre un trauma torácico cerrado por compresión antero posterior al quedar atrapada por las puertas de un autobús y 30 días después comienza a presentar falta de aire a los esfuerzos, aumentando progresivamente hasta desencadenarse a los pequeños esfuerzos. Ingresa en el hospital con diagnóstico de cardiopatía isquémica. Se realiza ecocardiograma y se comprueba gran colección líquida pericárdica que se evacua parcialmente mediante pericardiocentesis, pero al profundizarse los síntomas de hipovolemia se decide realizar pericardiectomía anterior radical de urgencia a través de una toracotomía antero lateral izquierda y se constata el pericardio engrosado y tenso. Se comprueba el diagnóstico por punción pericárdica transoperatoria por la presencia de sangre y se procede a descomprimir lentamente la colección intrapericárdica. Se completa la pericardiectomía y se diagnostica una gran contusión miocárdica con movimiento cardiaco lento. La evolución temporalmente es satisfactoria con estabilización del electrocardiograma aunque tres meses después sufre infarto del miocardio y fallece. PREGUNTA Cual es el porcentaje de pacientes con aplastamiento torácica desarrollan tamponade cardiaco? RESPUESTA a.- 20%. b.- 30 %. c.- 40 %. d.- 50 %.

CURSO ENARM CMN SIGLO XXI TEL: 36246001

Pharmed Solutions Institute

PÁGINA 208

MANUAL DE TRABAJO DEL CURSO ENARM CMN SIGLO XXI DISECCION DE AORTA: CIENCIAS BASICAS: Consiste en la separación de la capa media de la pared aortica en la que penetra sangre precedente de la aorta con lo que se establece una falsa luz que puede comprimir la luz verdadera del vaso. Entre ambas encontramos el colgajo intimal. Esta separación es consecuencia de una rotura o perforación en la intima (puerta de entrada) que se propaga distalmente. La adventicia puede contener inicialmente el sangrado o evolucionar a la rotura, Alrededor de 95% de roturas ocurren en la aorta ascendente, distal a la válvula aortica. Dependiendo de la localización y la extensión pueden aparecer un taponamiento cardiaco, hemotorax, una insuficiencia aortica o un síndrome de mala perfusión. Este ultimo según los troncos arteriales afectados, se puede manifestar como un síndrome coronario agudo, una afección neurológica o visceral (por afección de troncos supraaórticos, arteriales medulares y ramas viscerales), o una isquemia de los miembros inferiores. Los factores predisponentes mas frecuentes son: hipertensión (72%) y ateroesclerosis (31%). En pacientes menores de 60 años encontramos generalmente un aneurisma de aorta, una cirugía cardiaca previa, un síndrome de Marfan o una válvula aortica bicúspide. DIAGNOSTICO: Clínico: el dolor es la característica más importante, es una sensación de desgarro agudo e intenso, migratorio. La intensidad del dolor es constante, por lo que se diferencia del dolor secundario a IAM. Un dolor retroesternal con irradiación a cuello es característico de la afección de la aorta ascendente, mientras que el dolor dorsolumbar orienta hacia una afección de la aorta torácica descendente. El sincope puede estar causado por dolor intenso, la obstrucción de los vasos cerebrales, la activación de barorreceptores aórticos o un taponamiento cardiaco. En función de los vasos afectados habrá manifestaciones de mala perfusión. Una disección origina una obstrucción dinámica de los troncos supraaórticos, que se puede manifestar con un cuadro de isquemia cerebral y asimetría o ausencia de pulsos en extremidades superiores. El desgarro de las arterias intercostales se puede mostrar como un déficit motor en los miembros inferiores. El compromiso de las arterias viscerales con dolor abdominal (isquemia medular) o con aligoanuria (arterias renales). Los pulsos femorales pueden estar disminuidos o ser asimétricos. Otras manifestaciones menos frecuentes son hemoptisis, síndrome de vena cava superior, obstrucción de la vía aérea y síndrome de Horner (compresión por el aneurisma o hematoma en el cayado), disfonía. La ausencia de pulsos, PACIENTES DE ALTO RIESGO DE presencia de sincope previo, signos neurológicos, estado de Shock y hemotorax se DISECCION AORTICA consideran factores de mal pronóstico. El diagnostico de disección aortica aguda debe Enfermedad vascular ateroesclerótica considerarse en todo paciente con un cuadro de dolor abdominal agudo, isquemia de las Anuloectasia aórtica extremidades, en caso de sincope o un cuadro de insuficiencia cardiaca aguda no explicada. Aneurisma aórtico Ulcera aortica Recientemente se ha publicado el score de Disección Aortica en Urgencias, valora la Calcificación de la intima presencia de dolor en espalada, una relación toraco-mediastinica mayor a 30%, una Alteraciones genéticas (conectivopatias) insuficiencia aortica y un diámetro aórtico mayor de 30mm por ecografía. La sensibilidad y Síndrome de Marfan especificidad de este score es de 93 y 77% respectivamente, si están presentes 3 o más de Síndrome de Ehlers-Danlos Síndrome de Turner estos signos. Si se sospecha una disección aortica y no hay disponibilidad diagnostica ni de Enfermedad de Noonan tratamiento, debe asegurarse la remisión rápida del paciente. Rx de tórax; suele mostrar Osteogénesis imperfecta ensanchamiento mediastinico o derrame pleural asociado. ECG: es normal en 30%, cambios Enfermedades congénitas isquémicos, signos de hipertrofia VI. Analítica sanguínea: anemia ligera, leucocitosis, Coartación aortica Valvula aórtica unicúspide o bicúspide trombopenia, alteración de pruebas de función hepática, elevación de Cr sérica. Elevación de Enfermedades degenerativas reactantes de fase aguda. Acidosis metabólica en casos de mala perfusión. Específicos: Hipertensión Ecocardiografía transtorácica (ETT), identifica disección aortica proximal, una insuficiencia Causas traumáticas aortica asociada y un taponamiento cardiaco, permite evaluar la función miocárdica y medir Lesión por desaceleración Lesión penetrante los diámetros de la raíz aortica. Ecocardiografía transesofágica (ETE), mas sensible y Enfermedades inflamatorias especifica, confirma disección en un 90% de los casos y permite localizar la rotura intimal en Sífilis la mayoría de los casos y valorar el flujo sanguíneo en la verdadera y falsa luz. TAC, tiene Arteritis de células gigantes sensibilidad superior al 95% y una especificidad del 87%, da información precisa de los Iatrogenia Cateterismo cardiaco diferentes diámetros de los segmentos aórticos. Permite evaluar, con precisión la extensión Canulación aórtica o femoral previa de la aorta afectada y mostrar la implicación de las arterias viscerales e iliacas. Tiene como Balón de contrapulsación intraaórtico inconveniente la nefrotoxicidad, el contraste y la necesidad de desplazar al paciente. TRATAMIENTO: Actuación inmediata: La afectación de la aorta ascendente requiere cirugía, mientras que la disección de la aorta descendente precisa abordaje médico en ausencia de complicaciones. Farmacológico: Antes de la realización de cualquier medida diagnostico-terapéutica el paciente debe estar monitorizado con ECG, medida de presión arterial (cada 5min) y canalización de una vía venosa de buen calibre. La analgesia contribuye a estabilizar al paciente y en, general se da opiáceos a dosis bajas (cloruro mórfico 3 mg IV cada 5-10 min), para evitar la progresión de la enfermedad y el riesgo de rotura aortica, se debe disminuir el estrés parietal aórtico y controlar la presión arterial (objetivo TAS entre 90-110 mmHg, si se mantiene la diuresis). Los fármacos más usados como tratamiento antiimpulso, son los bloqueadores β selectivos como: de vida corta esmolol IV carga 0.5mg/kg en 2-5 min, mantenimiento 0,10-0,20 mg/kg/min, vida intermedia atenolol IV, dosis carga 2.5 mg, mantenimiento 0,15 mg/kg/dia. No selectivos como: propanolol dosis de carga 0.5-1 mg en 5 min, mantenimiento 0.05-0,015 mg/kg cada 4-6 hrs y los bloqueadores α1β2 (labetalol IV de vida media larga en dosis crecientes 0.5-4 mg/min). En caso de hipertensión arterial se puede iniciar labetalol de primera elección. Si se requiere de un control adicional de la presión arterial se suelen asociar nitroprusiato, nitratos, agonistas de calcio (nimodipino IV) o incluso IECAS IV, Si los betas bloqueadores están contraindicados (EPOC) hay que pensar en verapamilo o diltiazem. El tratamiento farmacológico crónico se reserva para los pacientes ya intervenidos con disección tipo B no complicada y requiere un control estricto de la presión arterial, manteniéndola por debajo de 130-135/80mmHg. El pilar del tratamiento son los bloqueadores beta, asociando IECA o

CURSO ENARM CMN SIGLO XXI TEL: 36246001

Pharmed Solutions Institute

PÁGINA 209

MANUAL DE TRABAJO DEL CURSO ENARM CMN SIGLO XXI antagonista de calcio, si se precisa o el paciente no tolera lo bloqueadores beta. Se debe realizar revisiones periódicas y pruebas de imagen, se recomiendan a los 1, 3, 6, y 12 meses y posteriormente anuales. Tratamiento quirúrgico: Objetivo salvar la vida del paciente previniendo la rotura de la aorta torácica ascendente en el pericardio. El tratamiento de la disección aortica tipo A es quirúrgico (I B), excepto en situaciones de edad avanzada, comorbilidad importante o daño neurológico. La disección tipo B tiene un mejor pronostico. Su abordaje inicial es médico y se reserva el tratamiento quirúrgico en la fase aguda a las disecciones complicadas. TIPO A: Reemplazar la aorta ascendente disecada, con un injerto sintético de dacrón, la zona resecada debe incluir la zona con la rotura intimal. La mortalidad quirúrgica varía entre un 10-20% en función de la edad, la comorbilidad y la extensión de la disección. TIPO B: El tratamiento médico tienen buenos resultados, este se centra inicialmente en el control de la presión arterial mediante bloqueadores beta y vasodilatadores, y el alivio del dolor. Una vez superada la fase aguda, se continúan los bloqueadores beta orales y vasodilatadores. Un seguimiento radiológico cada 6 meses será necesario para detectar precozmente una evolución tórpida. La cirugía queda limitada a: dolor torácico persistente o recurrente, expansión aortica, hematoma periaórtico, hematoma mediastínico. En la actualidad, las intervenciones endovasculares en la disección aortica aguda tipo B suelen limitarse a aliviar las complicaciones con riesgo de muerte. CASO CLINICO Se trata de paciente masculino de 46 años de edad, gerente de una tienda departamental, contador, con antecedente de tabaquismo positivo (una cajetilla diaria) es hipertenso desde hace 5 años con mal apego a su tratamiento, acude a urgencias debido presencia de dolor torácico que se irradia a la espalda, refiere que nunca había presentado un dolor así de intenso, que inicio hace 6 horas por la mañana y se retiro de su trabajo porque comenzó a sudar y a presentar nauseas sin llegar al vomito, el dolor se ha vuelto lacerante, el trazo electrocardiográfico se observa elevación ST de 1 mm en DII y aVF el paciente se encuentra taquicardico con pulso carotideo salton. PREGUNTA Cual es el diagnostico mas probable hasta el momento. RESPUESTA a.- Infarto al Miocardio. b.- Diseccion aortica. c.- Prolapso valvular. d.- Ruptura papilar. ANEURISMA AORTICO (AA): CIENCIAS BASICAS: Los aneurismas de la aorta (AA) se definen como la “dilatación focal de la arteria que supone un aumento de más de 50% del diámetro esperado”, basado en medidas medias obtenidas en estudios con TAC en población general. En el caso de la aorta abdominal correspondería a un diámetro superior a 3 cm. También se habla de AA como una dilatación localizada de por lo menos 1.5 veces mayor al diámetro normal de la arteria; puede ser sacular o fusiforme, y ambas están adyacentes a un segmento de arteria normal. Esta patología se ha convertido en el motor del tratamiento quirúrgico preventivo de la cirugía vascular. Decimos que existe un aneurisma verdadero cuando afecta a las tres capas histológicas; pero cuando la íntima y la media están rotas y la dilatación es a expensas solo de la adventicia, entonces hablamos de seudoaneurisma. La aorta abdominal es el vaso donde se localizan con más frecuencia los aneurismas arteriales. Hay aneurismas aórticos abdominales (AAA) y aneurismas aórticos de torácica descendente (AAT). SALUD PÚBLICA: De los aneurismas aórticos 80% se ubican en el abdomen y tienen una presencia en la población de la tercera edad de 2 a 7%. Es mayor en hombres que en mujeres, en una proporción de 4:1. En pacientes con hipertensión arterial, enfermedad vascular cerebral u otros aneurismas, su presencia aumenta 40%. Los AAA afectan a más de 3% de la población de edades superiores a los 50 años. Los factores de riesgo, como hipertensión arterial, tendencia familiar (se relaciona con cromosoma , autosómico, predisposición genética), tabaquismo, diabetes, infecciones, dislipidemia y los relacionados con la ateroesclerosis, aumentan su prevalencia significativamente. El riesgo proporcional para las personas que tienen un familiar de primer grado con AAA, es de 6:1 en comparación con la población general. Como en un elevado porcentaje son asintomáticos, y la ruptura es la primera manifestación clínica, la mortalidad real es muy alta. La probabilidad de ruptura al año del AAA menor de 5 cm es de 0.4-5.4%, entre 5-6cmde 20-25%, y de >7cm entre 40-80%. PATOGENIA: La aorta es un conducto a través del cual la sangre impulsada, desde el ventrículo izquierdo, pasa y se distribuye en el lecho arterial sistémico. El diámetro máximo en adultos corresponde en su origen a 3 cm, disminuyendo caudalmente, siendo de 2.5 cm a nivel de la porción de aorta descendente torácica, hasta 1.8 a 2 cm en la porción abdominal de la misma. La pared vascular está formada por la ín ma delgada, compuesta de endotelio, capa subendotelial de tejido conjuntivo y una capa interna elástica; la túnica media, de células musculares lisas y matriz extracelular y una adventicia formada fundamentalmente por tejido conectivo, que engloba los vasa vasorum y la inervación del vaso. Debido a su exposición continúa a la presión pulsátil y fuerzas de sujeción, está par cularmente expuesta a sufrir lesiones secundarias a trauma mecánico, sobre todo en los casos de desaceleración, siendo mayor el riesgo de ruptura y de aparición de dilataciones aneurismáticas. Desde el punto de la física, hay dos factores que generan los AA: la excesiva aplicación de una fuerza interna y la inadecuada resistencia del material. Se piensa que el origen de los AAA y AAT inespecíficos, es una disminución en la cantidad de elastina de la pared arterial. La cantidad normal de elastina que se encuentra en las personas normales es de 12%, porcentaje que baja a 1% o menos en los pacientes con aneurismas detectado a través de un marcador genético; el aumento de la actividad de la elastasa y de la colagenasa afectan al desarrollo del aneurisma. Se ha encontrado un metabolismo anormal de la elastina y del colágeno, mayor turbulencia o disminución de vasa vasorum en esa zona,. Los AAA y AAT principalmente, se deben a atero y arteriosclerosis; en menor proporción trastornos del tejido conjuntivo, traumatismos, síndrome de Marfan, infección, necrosis quística de la media y arteritis. También pueden ser por fibrodisplasias o iatrogénicos. En la mayoría de los casos no es posible identificar la causa. El AAA inflamatorio, que representa 10% de todos los aneurismas aórticos, puede deberse a una infección o a alguna otra forma oscura de arteritis. La evolución natural de los aneurismas no tratados es hacia la expansión y la ruptura siguiendo la Ley de Laplace. CLASIFICACION: En los AAT (Crawford), tiene en cuenta la longitud de la aorta afectada, resultando en cuatro patrones que determinan la extensión de la intervención quirúrgica, el resultado del tratamiento y la naturaleza de las

CURSO ENARM CMN SIGLO XXI TEL: 36246001

Pharmed Solutions Institute

PÁGINA 210

MANUAL DE TRABAJO DEL CURSO ENARM CMN SIGLO XXI complicaciones. Tipo I; está afectada la mayor parte de la aorta descendente torácica y la parte proximal de aorta abdominal. Tipo II; el aneurisma afecta gran parte de la aorta descendente y la mayor parte o toda la aorta abdominal. Tipo III; afecta la aorta torácica distal y la totalidad de aorta abdominal. Tipo IV la mayor parte de la aorta abdominal, incluyendo el segmento de vasos viscerales. DIAGNOSTICO: Clínico; Una minoría presenta dolor a la palpación abdominal o más com n dolor lumbar confundible con cólico renal. Un bajo número de pacientes se presentan con dolor abrupto abdominal y en la región lumbar. Al examen físico se puede palpar, en algunos casos, una masa pulsátil en el abdomen. La ruptura en el aneurisma es una forma de presentación con alta mortalidad, hay extravasación de sangre, la cual puede ser masiva (hacia la cavidad peritoneal) o contenida (en el retroperitoneo). La tríada de hipotensión, dolor abdominal y masa pulsátil en el abdomen, es observada sólo en 15% de los pacientes con aneurisma abdominal roto, donde 78% de ruptura es hacia el retroperitoneo, y sólo 22% se rompe en su pared anterior hacia la cavidad peritoneal. tra manifestación es la embolización distal de material trombotico que ocupa el saco aneurismá co, y esto ocurre en 3 a 5% de los pacientes. Es obligada la exploración física del paciente mediante palpación profunda abdominal, buscando en algunos casos una masa pulsátil en el mesogastrio, especialmente si el paciente es delgado. La presencia de soplos a la auscultación debe hacernos sospechar enfermedad oclusiva visceral o de aorta terminal y más raramente la presencia de una stula aortocava con datos de falla cardiaca. El estudio físico debe completarse con palpación de pulsos en extremidades, y en algunos casos pueden estar disminuidos o ausentes. Las presentaciones clínicas pueden ser de un abdomen agudo por ruptura del AAA, embolias distales o sangrado de tubo digestivo cuando se comunica la ruptura al duodeno. En los AAT los datos de compresión a bronquios, nervios o tubo digestivo pueden hacer sospechar su presencia. Dx. De imagen: La mayoría de los AA se diagnostican en forma incidental (80% de AAA), desde la Rx simple en tórax, que deforma el mediastino, o bien, la sombra de la aorta torácica descendente se hace más evidente. En la Rx de abdomen, además de las calcificaciones se puede observar “borramiento” de las líneas del m sculo psoas. Asimismo, tanto en una evaluación ultrasonográfica, TAC, RNM. El ultrasonido representa una modalidad económica, de fácil acceso y es también precisa para la medición del aneurisma. Este estudio no es sensible para el diagnóstico de hemorragia retroperitoneal ni para la medición de la distancia que separa el aneurisma de las arterias renales o las arterias iliacas. TRATAMIENTO: La mayoría de los centros quirúrgicos están de acuerdo en que todo aneurisma de 5 cm o mayor debe ser operado en forma electiva o tratado en forma endovascular. Otra tendencia prefiere el seguimiento cada seis meses e, independientemente del tamaño del aneurisma, un crecimiento mayor de 0.5 cm en este periodo es indicación de tratamiento. Es conocido el riesgo quir rgico por enfermedad coronaria coexistente en estos pacientes. La modificación de los factores de riesgo es fundamental, en el manejo medico de los AAA, la hipercolesterolemia y la HTA deben controlarse adecuadamente. Los betabloqueadores (propanolol) han sido considerados como terapia para reducir el riesgo de expansión y rotura del aneurisma. Una vez sospechado el diagnostico debe colocarse un acceso venoso de grueso calibre e infusión de cristaloides, evitando o corrigiendo la situación de shock inicial; esta medida está directamente relacionada con la morbimortalidad. La mayoría de los autores coinciden en que deberá tener como finalidad mantener tensión arterial sitolica en torno a 80-100mmHg. En las técnicas quirúrgicas se puede reparar con injertos sintéticos de dacrón o PTFE (politetrafluoroetileno), en forma recta o bifurcada hacia las iliacas o hacia las femorales, con resultados variables, este procedimiento quirúrgico seguirá teniendo vigencia, ya que 40% de los pacientes no son candidatos para cirugía endovascular, por variantes anatómicas que dificulten el implante, pacientes jóvenes con enfermedades asociadas: renal, pulmonar, cardiaca, carotidea de bajo riesgo que toleren bien la cirugía convencional. Manejo endovascular: se desarrollo para hacerla vía transfemoral, siendo un procedimiento combinado, a través de la cual pasa la prótesis montada del sistema para ser colocada bajo fluoroscopia a nivel del aneurisma. La elección de los pacientes es la clave del éxito. Existe un protocolo estricto que debe incluir mediciones precisas del diámetro de las arterias iliacas. CASO CLINICO Se trata de paciente masculino de 59 años de edad el cual se encuentra bajo tratamiento por padecer hipertensión arterial desde hace 10 años, además hipercolesterolemia, actualmente recibe clortalidona, atenolol y pravastatina, se encuentra apegado a tratamiento sin embargo continua fumando casi una cajetilla diaria. Acude a su consulta de revisión durante la cual le comenta que ha sentido una bolita que pulsa a la altura del ombligo y quiere descartar un padecimiento cancerígeno ya que en la familia hay esos antecedentes, a la exploración se encuentra con sus constantes vitales controladas, ninguna alteración cardiopulmonar, al revisar abdomen palpa una masa depresible no dolorosa pero pulsatil a nivel de cicatriz umblical de al menos 3 dedos de longitud. PREGUNTA Cuál es su conducta a seguir. RESPUESTA a.- Solicita USG. b.- Solicita TAC. c.- Solicita IRM. d.- Envia para LAPE. CASO CLINICO El paciente regresa con los resultados de la tomografía solicitada donde se reporta aneurisma de la aorta abdominal de 4.5 cm infrarrenal, el paciente agrega que recuerda que desde hace algunos meses ha notado dolor ocasional de espalda que ha sido tratado por su medico familiar con diclofenaco y medidas locales. PREGUNTA Cual es la conducta mas apropiada a seguir en este caso. RESPUESTA a.- Realizar estudios para intervención quirúrgica. b.- Continuar con diclofenaco.

CURSO ENARM CMN SIGLO XXI TEL: 36246001

Pharmed Solutions Institute

PÁGINA 211

MANUAL DE TRABAJO DEL CURSO ENARM CMN SIGLO XXI c.- Vigilar al paciente. d.- Vigilancia y antiagregante plaquetario. CASO CLINICO El paciente ha presentado algunas ocasiones episodios de descontrol hipertensivo, continua su habito de tabaquismo, los niveles de colesterol no han cedido, el paciente refiere que ha cursado con mareos ocasionales acompañados de zumbido de oídos y luces brillantes, ha mejorado su alimentación pero refiere aumento del dolor de espalda y sensación de presión interna “como si le pulsara”, debido a los cambios observados se repite la TAC abdominal donde se reporta un incremento en las dimensiones del aneurisma de la aorta abdominal pasando de 4.5 cm hace 2 años a 6.7 cm. PREGUNTA Considerando las nuevas evidencias el paciente es sometido a cirugía, cual es el criterio más importante para el tratamiento elegido. RESPUESTA a.- El tamaño del aneurisma. b.- La presencia de dolor persistente. c.- La presencia de descontrol hipertensión. d.- Incremento de placas ateromatosas. ANGINA ESTABLE, INESTABLE Y PRINZTMETAL DEFINICION: La angina estable es un síndrome clínico caracterizado por malestar en el pecho, mandíbulas, hombros, espalda o brazos, que aparece con el ejercicio o estrés emocional, dura de 2 a 5 minutos, remite espontáneamente, con el descanso o con la administración de nitroglicerina, (las manifestaciones clínicas aparecen cuando al menos hay una oclusión del 70 % de la luz del vaso afectado. EPIDEMIOLOGIA: En ambos sexos, la prevalencia de la angina aumenta marcadamente con la edad, de un 0,1-1% en mujeres de edades comprendidas entre 45 y 54 años a un 10-15% en mujeres de 65-74 años y de un 2-5% en varones de 45-54 años a un 1020% en varones de 65-74 años. Con base en estos datos, se calcula que, en la mayoría de los países europeos, entre 20.000 y 40.000 individuos por millón de habitantes sufren angina. El principal factor de riesgo es la aterosclerosis. ETIOLOGIA: No hay factores espeficificos solo relativos como aterosclerosis, hipertensión, dislipidemia, enfermedad arterial coronaria, alteraciones vasculares, retinianas, soplos, infarto, presión yugular edema pulmonar, soplo mitral. FISIOPATOGENIA: Los datos anginosos se presentan cuando la demanda de oxigeno por el miocardio se incrementan, y no existe una respuesta coronaria adecuada para mantener el aporte, por lo regular se encuentran los síntomas anginosos cuando hay una obtruccion coronaria cercana al 90 % con una distensibilidad reducida de la coronaria afectada. Frecuentemente los pacientes con obsetrucciones menores a 70 % no muestran síntomas anginosos. CUADRO CLINICO: Sensacion de opresión, tensión o pesadez y sensación de estrangulamiento, constricción o quemazón. La severidad puede variar mucho. Falta de aire y malestar torácico, síntomas como fatiga, náusea, eructos, ansiedad ó sensación de muerte inminente. DIAGNOSTICO: Angina estable: El episodio de angina es breve, generalmente menos de 10 min. Relación con el ejercicio, una actividad o el estrés emocional y remitan rápidamente en unos minutos cuando desaparezcan los factores causales. Otra característica típica de la angina es la acentuación de los síntomas después de una comida pesada o a primera hora de la mañana. Los nitratos orales o sublinguales alivian rápidamente la angina. Angina inestable: a) angina en reposo (durante el reposo y períodos prolongados, 20 min); b) angina in crescendo (angina estable previa que evoluciona rápidamente en severidad e intensidad y con un umbral más bajo durante un período corto, de 4 semanas o menos), y c) angina de reciente aparición (episodio reciente de angina severa que limita marcadamente la actividad diaria del paciente y se presenta durante los 2 meses posteriores al episodio inicial). En la angina inestable hay evidencia de placas con rotura, hemorragia o laseracion lo cual las vuelve mas inestables y trombogenicas. Angina de Prinzmetal: Presentan dolor de localización típica, que ocurre en reposo o sólo ocasionalmente, durante el ejercicio. Puede coincidir con la angina típica de esfuerzo por lesiones coronarias fijas. El vasoespasmo puede ocurrir como respuesta al consumo de tabaco, alteraciones electrolíticas (potasio, magnesio), consumo de cocaína, estimulación por frío, enfermedades autoinmunitarias, hiperventilación o resistencia a la insulina. El ECG no es concluyente, prueba de esfuerzo en banda sin fin, ecocardiograma de esfuerzo, ecocardiograma con estimulación con dobutamina, coronariografia (estándar de oro), cuando hay datos anormales, biomarcadores proteína C de alta especificidad y péptido natriuretico cerebral. TRATAMIENTO: Antianginosos BB; reducen el consumo de O2, disminuyen TA, FC y contractibilidad, bloqueadores de canales de calcio, aumentan el flujo coronario y disminuyen el consumo de O2, verapamil o diltiazem pero con mayor de 30% de FE o disfunción sinusal o nodal; nitritos: vasodilatadores venosos con cierto efecto coronario, disminuyen precarga, isosorbide o nitroglicerina de 0.2 a 0.6 mg sblingual responde en 1 a 3 minutos y puede repetirse cada 5 minutos. Aspirina 150mg, clopidrogel si hubo intervencionismo, atorvastatina 80 mg/dia e IECA. PRONOSTICO: En varones y mujeres con una presentación clínica inicial de angina estable, la incidencia de infarto de miocardio no fatal y muerte por enfermedad coronaria a los 2 años era del 14,3 y el 5,5% en varones y del 6,2 y el 3,8% en mujeres, respectivamente. La tasa anual de mortalidad varía entre el 0,9 y el 1,4% por año 5-9, con una incidencia anual de infarto de miocardio no fatal del 0,5% y el 2,6%. COMPLICACIONES: En caso de no obtener buenos resultado o si continua la evolución tórpida de la angina se podrá realizar angioplastia o cirugía de derivación cardiaca, sin haber evidencia de sobrevida mayor con cualquier técnica. CASO CLINICO Se trata de paciente masculino de 52 años de edad, el cual acude a consulta externa debido a que desde hace 6 meses presenta opresión toracicca, refiere que considera que es por las presiones que tiene en el trabajo y en casa se encuentra en proceso de divorcio, refiere que el dolor dura menos de 5 minutos, desapareciendo respirando profundamente y reposando, agrega que últimamente ha presentado dolor epigástrico acompañado de nauseas independiente del malestar en el torax para lo cual emplea antiácidos casi diario, cuenta con antecedentes de importancia por tabaquismo y alcoholismo semanal hasta la embriaguez, asi como padre finado por IAM, madre viva con hipertensión arterial con tratamiento, tiene habitos alimenticios inadecuados, ricos en grasas y comida rápida, por el trabajo y no realiza ejercicio a la exploración física se observa con leve rubicundez facial, aliento al parecer alcoholico, presenta signo de

CURSO ENARM CMN SIGLO XXI TEL: 36246001

Pharmed Solutions Institute

PÁGINA 212

MANUAL DE TRABAJO DEL CURSO ENARM CMN SIGLO XXI Levine al interrogar sobre el sitio del dolor, queratosis seborreica importante y xantomas, al parecer tercer ruido en area precordial y estertores subcrepitantes bibasales. Signos vitales TA 150/95 mmHg, FR 28 rpm, FC 104 lpm. PREGUNTA Cuáles son las manifestaciones clínicas de la enfermedad mas importantes para considerar un dianostico presuntivo? RESPUESTA a.- Presencia del signo de Levine. b.- Dolor torácico. c.- Disminución del dolor con el reposo. d.- La duración del dolor. PREGUNTA Cuál es la manifestacion clínica que no es criterios diágnosticos para establecer la enfermedad del paciente? RESPUESTA a.- Molestia toracicca subesternal. b.- Inicio por estrés o ejercicio. c.- Disminuye con nitroglicerina. d.- Sexo, mas frecuente en mujer. PREGUNTA Considerando la clasificación de la enfermedad cual de los siguientes diagnosticos es el mas probable para el caso actual? RESPUESTA a.- Angina definitiva. b.- Angina probable. c.- Angina posible. d.- Angina no posible PREGUNTA Considerando la clasificación funcional de la Canadian cardiovascular society, cual es la clase actual del paciente? RESPUESTA a.- Clase I. b.- Clase II. c.- Clase III. d.- Clase IV. PREGUNTA Cuál es el diagnostico diferencial mas probable en este caso debido a los factores modificables de riego presentes? RESPUESTA a.- Angina de prinzmetal. b.- Neuritis intersticial. c.- Sindrome de la pared toracicca anterior. d.- Enfermedad gastrointestinal funcional. PREGUNTA Se realizaron los siguientes axiliares diagnosticos, cual de los siguientes resultados no excluye el diagnostico actual?, “revisar las imágenes” RESPUESTA a.- Prueba de esfuerzo. b.- Electrocardiograma en reposo. c.- Radiografia de torax. D.- Arteriografia coronaria. PREGUNTA Antes de realizar al paciente una coronariografia se encontraron los siguientes resultados, previos cual de ellos no es criterio? RESPUESTA a.- Depresión del segmento ST >1 mm. b.- Prueba de esfuerzo anormal. c.- Sintomatologia presente. d.- Resultados inconclusos.

CURSO ENARM CMN SIGLO XXI TEL: 36246001

Pharmed Solutions Institute

PÁGINA 213

MANUAL DE TRABAJO DEL CURSO ENARM CMN SIGLO XXI PREGUNTA Durante las pruebas se encontraron los siguientes resultados de laboratorio en el paciente: Creatinina 1,24, Glucosa 224, Colesterol 310, Trigliceridos 420, HDL 29, Urea 41, Na 138, K 3,42, Troponina T-U: 2,8, CK 105, CK-MB 1,67, ALT-GPT 19, Hemograma y coagulación normal. En relación a datos de laboratorio obtenidos cuales son los objetivos terapéuticos mas importantes? RESPUESTA a.- Incrementar el HDL. b.- Disminuir triglicéridos y colesterol. c.- Revascularización inmediata. d.- Buscar proteína C de alta sensibilidad. PREGUNTA Cuáles son las medidas terapéuticas no farmacológicas inicial en este padecimiento, para disminuir la demanda miocárdica de oxigeno? RESPUESTA a.- Propanolol. b.- Hidroclorotiazida. c.- Losartan. d.- Enalapril PREGUNTA Considerando los datos de lípidos y el informe de la coronariografia, que medida farmacológica es necesario agregar para estabilizar la placa? RESPUESTA a.- Clopidrogel 75 mg cada 12 hrs. b.- Atorvastatina 40 mg cada 12 hrs. c.- Enalapril 20 mg cada 12 hrs. d.- Aspirina 150 mg cada 24 hrs. PREGUNTA Cuáles son los siguientes esquemas terapéuticos secuenciales antianginoso tiene un efecto favorable sobre la presión arterial, la contractilidad y la frecuencia cardiaca más adecuado? RESPUESTA a.- Prazocin 1 a 2 mg cada 24 hrs. b.- Tamsulosina 0.4 mg cada 24 hrs. c.- Clonidina 0.075 a 0.150 mg cada 24 hrs. d.- Propranolol 40 a 80 mg en 24 hrs. PREGUNTA Cual de las siguientes comorbilidades es la menos frecuentes en la angina típica como es el caso? RESPUESTA a.- EPOC. b.- DM. c.- Espasmo vasocoronario. d.- Disfunción eréctil. PREGUNTA Considerando la fracción de eyección del paciente, cual fármaco calcio antagonista puede usarse con cuidado comparado con el resto? RESPUESTA a.- Amlodipino. b.- Verapamilo. c.- Diltiazem d.- Nifedipina PREGUNTA Considerando la fracción de eyección del paciente, cual fármaco calcio antagonista puede usarse con confianza comparado con el resto? RESPUESTA a.- Nitroglicerina. b.- Isosorbide.

CURSO ENARM CMN SIGLO XXI TEL: 36246001

Pharmed Solutions Institute

PÁGINA 214

MANUAL DE TRABAJO DEL CURSO ENARM CMN SIGLO XXI c.- Nitrito de amilo. d.- Nitrito de potasio. PREGUNTA Considerando la patogenia de la angina de pecho cual de los siguientes síntomas están menos relacionados a la isquemia del miocardio y si a la necrosis miocárdica? RESPUESTA a.- Disconfor o dolor en el pecho. b.- Maxilar inferior ó brazo. c.- Hombros y espalda. d.- Elevación de CPK-MB. PREGUNTA Se le solicito al paciente que usara un diario para registra el dolor, cual es la razón fisiopatológica que el paciente presente mayor frecuencia de dolor en la mañana? RESPUESTA a.- Debido a presencia de apnea. b.- Debido al ritmio endógeno de cortisol. c.- Debido al ritmo endógeno de secresión de catecolaminas. d.- Debido a la insensibilidad coronaria a vasoconstrictores. PREGUNTA Cual es la frecuencia de angina estable crónica al momento de solicitar consulta al especialista? RESPUESTA a.- 50 %. b.- 60 %. c.- 40 %. d.- 70 %. PREGUNTA Cuál es la edad más frecuente comparada con el caso del paciente para presentar angina típica vs angina atípica considerando además en sexo? RESPUESTA a.- Hombres de 50 a 59 años. b.- Hombres de 40 a 49 años. c.- Hombres de 60 a 69 años. d.- Hombres de 50 a 69 años PREGUNTA Considerando la escala de Duke Treadmill Scores para calificar el riesgo estratificado no invasivo cuál es la calificación que presenta actualmente? RESPUESTA a.- Riesgo alto (3 % de mortalidad anual). b.- Riesgo muy alto (5 % de mortalidad anual). c.- Riesgo intermedio (1-3 % de mortalidad anual). d.- Riesgo bajo (menor a 1% de mortalidad anual). CASO CLINICO Se trata de masculino de 68 años de edad al cual se encuentra diagnosticado y tratado por padecer angina típica desde hace 5 años, sin embargo de 6 meses a la fecha el disconfort del torax ha sido moderado y en ocasiones durante el reposo, acude al servicio de urgencias debido a que presenta opresión toracicca acompañado de dolor en hombro izquierdo que se iradia a el brazo, agrega que se encontraba discutiendo cuando comenzó las molestias, no cedió al reposo y duro más del tiempo habitual, cuenta con los antecedentes de hipertensión arterial desde hace 20 años con tratamiento actual con captopril 25 mg cada 12 hrs, hidralazina 25 mg /12 hrs, hace 5 años fue agregado propranolol, pravastatina y aspirina para el manejo de su angina inicialmente estable, a su ingreso se observa diaforética, con palidez generalizada, ansiosa, con cianosis distal, sus constantes vitales fueron TA 105/70 mmHg, FC 101 lpm, FR 29 rpm. PREGUNTA Considerando el cuadro clínico cuál es diagnostico más probable hasta este momento? RESPUESTA

CURSO ENARM CMN SIGLO XXI TEL: 36246001

Pharmed Solutions Institute

PÁGINA 215

MANUAL DE TRABAJO DEL CURSO ENARM CMN SIGLO XXI a.- Angina típica descompensada. b.- Angina atípica inicial. c.- Infarto agudo al miocardio. d.- Insuficiencia cardiaca por angina previa. PREGUNTA Cuál de los siguientes auxiliares diagnostico es mas útil en este momento para mantener una conducta clinica adecuada? RESPUESTA a.- Buscar biomarcadores de isquemia. b.- Realizar electrocardiograma. c.- Coronariografia. d.- Programar prueba de esfuerzo con dobutamina. PREGUNTA Cuál cual es su clase funcional con la clínica observada? RESPUESTA a.- Clase I. b.- Clase II. c.- Clase III. c.- Clase IV PREGUNTA Se practicaron los siguientes estudios al paciente cual es su opinión sobre la radiografia y ECG? RESPUESTA a.- Datos de edema pulmonar. b.- Datos de insuficiencia cardiaca. c.- Datos de IAM. d.- Datos de EPOC. RESPUESTA Cuál es la conducta inmediata más importante a seguir? RESPUESTA a.- Morfina, oxigeno, nitritos y antiagregantes. b.- Diuretico, morfina, verapamilo y oxigeno. c.- Diuretico, antiaginoso y trombolisis. d.- Morfina, nitrito, trombolisis, enoxoparina. SINDROME CORONARIO AGUDO I IAM SESS DEFINICION: Conjunto de cuadros clínicos por los que se pone de manifiesto de forma aguda la isquemia miocárdica secundaria en general, pero no exclusivamente, por ateroesclerosis coronaria, sin embargo tiene su base en la erosion de una placa aterosclerótica de las arterias coronarias, lo cual activa la coagulación culminando con la formación de un trombo oclusivo. EPIDEMIOLOGIA: 2.5 millones de ingreso a urgencias, 16.6 millones de muertes, primera causa de muerte, 1 muerte cada minuto. Alta morbilidad e incapacidad. Se estima que actualmente el 50% de todas las muertes en los países de alto ingreso y el 28% de las muertes en los PBMI son el resultado de ECV. En 2001, el 31% de todas las muertes en Latinoamérica (LA). Factores de riesgo cardiovascular clasicos: DM, HAS, Tabaquismo, LDL, estilo de vida, genes. Factores de riesgo cardiovascular actual; Infeccioso, inflamatorio, homocisteina, protrombosis. ETIOLOGIA: No existe un factor etiológico especifico, son una serie de sucesos concurrentes, la presencia de una placa ateromatosa que se erosiona, inicia una hemorragia que activa la coagulación generando un fenómeno oclusivo, es conveniente observar que no es necesario la presencia de angina previa, ni la presencia de grandes placas en las coronarias. FISIOPATOGENIA: Las placas ateromatosas consideradas como susceptibles a la erosion, en el caso del SICA 1, principalmente la centrolipidicas y con una delgada capa de fibrina, la desestabilización y consecuente rotura de la placa es consecuencia principal de la circulación de factores inflamatorios locales y sistémicos, asi como los cambios fisiológicos de las arterias coronarias por demanda u otros fenómenos, al presentarse lisis de la delgada capa de fibrina se activa la cascada de coagulación, adeshion y agregación plaquetaria por vol willebrand, glucoproteinas IIb/IIIa, tromboxano A, epinefrina, serotonina y factor activador de plaquetas, aumentando la afinidad de la protrombina y el fibrinógeno convirtiéndose en trombina y fibrina respectivamente consecuentemente acumulo de eritrocitos y plaquetas formando un trombo el cual no necesariamente es oclusivo, de ello dependerá en gran medida las modificaciones del ECG. CUADRO CLINICO: El paciente refiere dolor opresivo o transfictivo retrosternal, con irradiación a cuello, brazo o mandibula, síntomas agregados, nauseas, vomito, diaforesis, y sensación de muerte inminente. DIAGNOSTICO: Se observan manifestaciones clínicas referidas, alteraciones electrocardiográficas y biomarcadores, (troponinas y CPK-MB) que no se elevan inmediatamente. TRATAMIENTO: Inmediato: evaluación general, toma e interpretación de ECG, búsqueda de factores para fibrinoliticos, toma de biomarcadores y rutina. Secundario: morfina, oxigeno, nitritos y antiagregantes. Terapia de reperfusion: en caso de no existir contraindicaciones y por factibilidad se indica terapia fibrinolitica; debe realizarse dentro de las primeras 12 horas de iniciado el cuadro. PRONOSTICO: El manejo firbinolitico favorece del 60 al 80 % de los pacientes, es bueno cuando se realiza mas temprano, además de administrar heparina para

CURSO ENARM CMN SIGLO XXI TEL: 36246001

Pharmed Solutions Institute

PÁGINA 216

MANUAL DE TRABAJO DEL CURSO ENARM CMN SIGLO XXI evitar la reoclusiones. COMPLICACIONES: La arritmias cardiacas sin una de las complicaciones mas importantes, asi como el choque cardiogenico, otras complicaciones vienen del uso de fibrinoliticos. CASO CLINICO Mujer de 66 años, hipertensa de 10 años de evolución con atorvastatina y enalapril actualmente con adecuado control, acudió por opresión torácica en reposo de 2 h de evolución. Al ingreso se observo con leve palidez de tegumentos con sensación de ahogo y disnea aun con oxigeno suplementario, TA de 165/88mmHg, obesidad troncular y soplo sistólico II/VI en focos de la base. El ECG mostró elevación del ST en cara inferior y de V2 a V6. Se realizó fibrinolisis, se encontraba en las primeras 2 hrs de evolución. Se administró enoxaparina, ácido acetilsalicílico, nitroglicerina y morfina. Un ecocardiograma urgente demostró acinesia del ápex y la cara anterior, y se estimó una fracción de eyección del 40%. Se observó un ligero derrame pericárdico posterior. PREGUNTA Cual de las manifestaciones del cuadro clínico es el menos importante para la decisión para realizar fibrinólisis en este caso? RESPUESTA a.- Antecedente de angina inestable. b.- El tiempo de inicio del cuadro clínico. c.- La auscencia de factores de riesgo a la fibrinólisis. d.- La presencia de cambios electrocardiográficos. PREGUNTA Cuál de los antecedentes del caso es el de menor importancia para el actual IAM? a.- Hipertension arterial previa. b.- Antecedente de angina. c.- Sobrepeso. d.- Uso de atorvastatina subterapeutica. PREGUNTA Considerando los cambios electrocardiográficos, cuales de las arterias es la menos probable que se encuentre involucrada. RESPUESTA a.- Desendente anterior (ramos septales). b.- Desendente anterior (ramos diagonales). c.- Circunfleja. d.- Desendente posterior. PREGUNTA Cuál es la complicación mas probable considerando el sitio de compromiso coronario? RESPUESTA a.- Bloqueos de rama. b.- Disfunción ventricular izquierda. c.- Insuficiencia cardiaca. d.- Hipotensión. PREGUNTA Considerando las características del paciente, cual de los siguiente fármacos favorece la reducción de la extensión del daño miocárdico, ruptura miocárdica o reinfarto? RESPUESTA a.- Nitritos. b.- Betabloqueadores. c.- IECAS. d.- Calcioantagonistas. PREGUNTA Cuál de los cambios bioquímicos no son factores que desencadenan los cambios de actividad plaquetaria en este caso? RESPUESTA a.- Factor de von Willebrand. b.- Glucoproteina IIb/IIIa. c.- Produccion de tromboxano A2. d.- Leucotrienos. PREGUNTA Cuál de las siguientes manifestaciones bioquímicas no espera encontrar en el paciente en cuestión?

CURSO ENARM CMN SIGLO XXI TEL: 36246001

Pharmed Solutions Institute

PÁGINA 217

MANUAL DE TRABAJO DEL CURSO ENARM CMN SIGLO XXI RESPUESTA a.- Creatinincinasa se eleva entre las 4 y 8 hrs después del comienzo de la sintomatologia. b.- La fracción MB de la CPK a partir de las 3 del comienzo del infarto. c.- La troponina I se eleva a partir de que se inicia la sintomatologia. d.- La mioglobina es la ultima en enzima que se eleva. CASO CLINICO Varón de 66 años ingresado por IM lateral evolucionado, no tratado con fibrinolíticos por criterios de tiempo. Tras 72 h sin complicaciones, bajo tratamiento con nitratos. En el séptimo día de ingreso presentó TV polimorfa sincopal que precisó cardioversión eléctrica urgente. En las horas siguientes se objetivaron numerosos episodios de taquicardia sostenida y no sostenida, con un comportamiento incesante y degeneración ocasional en fibrilación ventricular (FV), precisando múltiples choques externos. Estos episodios no eran precedidos de bradicardia significativa, prolongación anormal del intervalo QT, cambios en el segmento ST o dolor torácico. Los valores de las enzimas cardíacas dentro de rango normal. PREGUNTA Cual es el manejo farmacológico mas adecuado. RESPUESTA a.- Lidocaina. b.- Amiodarona. c.- Verapamilo. d.- Propanolol. SICA II (IAM SESST) INTRODUCCION: Se trata de proceso coronario isquémico inestable, SICA II o Infarto miocárdico agudo sin elevación del segmento ST, divido en riesgo alto, medio y bajo, lo que permite considerar tratamiento de reperfusion inmediato o no, condición dinámica que puede evolucionar de angina inestable a infarto al miocardio, teniendo como punto cardinal la elevación de enzimas cardiacas. DIAGNOSTICO: Cuadro clínico caracterizado por dolor retrosternal de tipo opresivo, definido como opresivo, acompañado o no con sensación de pensantez torácica, irradiación a brazo, cuello, hombro, puede presentar dolor en espalda, síntomas generales, debilidad, mareo y nausea, acompañado por falta de aire. ECG especialmente se considera la falta de elevación del segmento ST, pero podemos observar alteraciones con depresión o aplanamiento de onda T, o del segmento completo. Finalmente para establecer el diagnostico deberán buscar enzimas cardiacas, si hay elevación el diagnostico será SICA SESST. Carateristicas del riesgo (Alto) multiples factores de riesgo para enfermedad coronaria, angina en reposo mas de 20 minutos, o posinfarto, elevación del ST, depresión de ST > 1 mm en 2 derivaciones continuguas, Inestabilidad hemodinámica, signos de insuficiencia cardiaca o disfunción ventricular izquierda, biomarcadores sericos elevados. Clasificación riesgo (Intermedio) DM antecedentes de IAM, angina prolongada resuelta, Inversion de T en 5 derivaciones, biomarcadores normales con marcadores inflamatorios elevados. Riesgo (Bajo) Angina inestable o cambiante, Biomarcadores o marcadores inflamatorios normales, ECG normal o sin cambios. PATOGENIA: El SICA II es casusado por ruptura o erosion de una placa ateromatosa con formación de trombo plaquetario que genera consecuentemente obstruccion predominantemente parcial del flujo sanguíneo coronario. Es un fenómeno dinamico que trata de mantener un balance entre la demanda del miocardio y la demanda que se observa durante la angina inestable y infarto miocárdico sin elevación del segmento ST, las cuales incluyen, 1.- reducción de la luz arterial coronaria debido a un trombo no oclusivo que se desarrolla seguido a ruptura o erosion de una placa aterotrombotica. 2.- severo estrechamiento de arteria coronario sin espasmo o trombosis por arteriosclerosis progresiva o con reestenosis con mas de 6 meses posterior a intervención coronaria percutánea. 3.- Espasmo focal intenso de segento epicardico coronario arterial (prinzmetal o angina variante) causando obstruccion dinámica del lumen arterial. 4.- Diseccion coronaria arterial (causado posparto) 5.- En presencia de factores precipitantes extrínsecos a las arterias coronarias que limitan la perfusión miocárdica, causando una alteración súbita de la demanda miocárdica de oxigeno como (sepsis, fiebre, taquicardia) o reducción de flujo coronario (hipotensión) o disminución de la entrega de oxigeno al miocardio (hipoxia, anemia severa, etc). TRATAMIENTO: Reposo y comodidad absoluta, ECG de 12 derivaciones identificando el patron ST posteriormente establecer estratificación del riesgo lo cual permite decidir el tratamiento, se inicia co inmediata: angioplastia primaria o trombolisis; AI/SICA SESST (riesgo alto e intermedio) con depresión del ST o cambios dinamicos de T (inversión) o elevación enzimática angioplastica coronaria percutánea; IA/SICA SESST (riesgo bajo) con ECG no diagnostico o normal, enzimas normales, se indicara tratamiento medico, monitorización, estudios de isquemia para estratificación. Analgesicos: morfina indicada si los nitritos no reducen el dolor, en paciente ansioso o agitado, en edema agudo de pulmon, reduce la precarga y la poscarga, el consumo de oxigeno miocárdico, sedante, venodilatador. O2 a 4 Ltx´ vigilando saturación (90%), Nitroglicerina via sublingual hasta 3 dosis intervalo de 5 minutos, monitorizar FC y TA, con persistencia de dolor inicio de via IV, su efecto es venodilatador, cuidado en infarto inferior o derecho, en hipotesion, bradicardia o taquicardia, evitar si hay uso previo de inhibidores de fosfodiesterasa. Aspirina de 160 a 325 mg de absorción rápida; Clopidrogrel inhibidor irreversible del receptor del adenosindifosfato 300 mg dosis única. Anticoagulantes, heparina no fraccionada no propaga el trombo sin utilidad lítica a 60 a 70 U por Kg mantenerTPT menor a 75 segundos. Heparina de bajo peso molecular, enoxoparina mayor seguridad de sangrado. Fibrinoliticos, deben evitarse por riesgo beneficio ya que en SICA II SESST la arteria se encuentra abierta. PRONOSTICO: SICA II SESST presenta morbilidad y mortalidad expectante por las condiciones de la enfermedad coronaria de base, la función sistólica del ventrículo izquierdo, la estabilidad de la lesión en si misma, la mortalidad por angina inestable es de 3.6 % a 6 meses y 6.2 % en caso de infarto al miocardio sin elevación del segmento ST.

CURSO ENARM CMN SIGLO XXI TEL: 36246001

Pharmed Solutions Institute

PÁGINA 218

MANUAL DE TRABAJO DEL CURSO ENARM CMN SIGLO XXI CASO CLINICO Hombre de 48 años, con antecedente familiar de cardiopatía coronaria, tabáquico y dislipidémico sin tratamiento farmacológico. Presentó intenso dolor precordial mientras jugaba fútbol, consultando en un servicio de urgencia donde presentó paro cardiorrespiratorio por fibrilación ventricular. Fue tratado con múltiples desfribilaciones y maniobras de reanimación durante 25 min. El ECG no mostro datos sugerentes de infarto. Al ingreso se encontraba intubado con asistencia ventilatoria mediante ambú, hemodinámicamente inestable requiriendo infusión de noradrenalina a 0,1 µg/kg/min. 2 horas después se observo cambios en el ECG mostró onda Q y supradesnivel del segmento ST hasta 3 mm en VI a V3. Troponina I: 292 ng/mL (normal 40. PREGUNTA Considerando la sintomatología observada en el paciente que clase funcional killip y Kimball. RESPUESTA a.- Clase funcional Killip I. b.- Clase funcional Killip II. c.- Clase funcional Killip III. d.- Clase funcional Killip IV. PREGUNTA La presencia de DM acelera el proceso de aterogénesis a través de diversos mecanismos, cual de los siguientes mecanismos es mayor en el paciente actual? RESPUESTA a.- Anomalías en concentraciones y la composición de las lipoproteínas. b.- Asociación con la hipertensión. c.- Oxidación lipídica.

CURSO ENARM CMN SIGLO XXI TEL: 36246001

Pharmed Solutions Institute

PÁGINA 219

MANUAL DE TRABAJO DEL CURSO ENARM CMN SIGLO XXI d.- Estado procoagulante y proinflamatorio. PREGUNTA Cuál es los paraclinicos en el primer nivel de atención es de mayor utilidad para confirmar el diagnóstico de la insuficiencia cardíaca en el paciente? RESPUESTA a.- ECG. b.- Banda sin fin. c.- Ecocardiograma. d.- Arteriografia. PREGUNTA Cual de los siguientes factores comorbidos presenta la mayor importancia para desencadenar un evento coronario en el paciente. RESPUESTA a.- Diabetes mellitus. b.- Hipertensión arterial. c.- Dislipidemia. d.- Tabaquismo. PREGUNTA Cual de los siguientes datos patológicos en el paciente presenta la mayor importancia para desencadenar un evento coronario en el paciente. RESPUESTA a.- Niveles de glucosa. b.- Hemoglobina glucosilada. c.- Trigliceridos d.- Colesterol. PREGUNTA Cual es la conducta a seguir con los siguientes criterios es el menos importante en este paciente para envio inmediato a tercer nivel? RESPUESTA a.- Dolor toracicco sugerente de angor pectoris. b.- Dislipidemia. c.- Hipertensión arterial. d.- Diabetes mellitus. PREGUNTA Cual de los siguientes diagnosticos diferenciales es el menos probable considerando las condiciones del caso? RESPUESTA a.- Hipertensión arterial pulmonar. b.- Pericarditis. c.- Sindrome de Tietzl. d.- Reflujo gastroesofagico. PREGUNTA Cual de las siguientes medidas debe iniciar antes de enviar al paciente a segundo nivel cuando existe alta sospecha de cardiopatía isquémica? RESPUESTA a.- Dieta y ejercicio progresivo. b.- Acetilsalicilico y pravastatina. c.- Diuretico y betabloqueador. d.- Estudios básicos y gabinete. CASO CLINICO Se presenta el caso de un paciente de 17 años de edad, sin antecedentes de interés, que acude a urgencias por un dolor centrotorácico, opresivo, no irradiado, de 48 hrs de evolución, sin fiebre ni otra sintomatología acompañante. El paciente refiere haber consumido cocaína y cannabis previamente al inicio de la sintomatología. A su llegada se encuentra estable, sus constantes son correctas y la exploración física es normal. El electrocardiograma muestra elevación del segmento (ST) en las derivaciones de la cara inferior; la analítica confirma una elevación de las enzimas cardíacas (creatincinasa [CK] de 1.194 U/l y troponina-I de 19,6 ng/ml).

CURSO ENARM CMN SIGLO XXI TEL: 36246001

Pharmed Solutions Institute

PÁGINA 220

MANUAL DE TRABAJO DEL CURSO ENARM CMN SIGLO XXI PREGUNTA Cual es terapéutica mas apropiada para este caso. RESPUESTA a.- Nitroglicerina, morfina, trombolitico y oxigeno. b.- Nitroglicerina, morfina, trombolitico y betabloqueador. c.- Angioplastia percutánea, nitoglicerina, morfina, trombolitico y betabloqueador. d.- Betabloqueador, morfina, betabloqueador. CASO CLINICO Se trata de femenino de 23 años de edad la cual llega a consulta refiriendo dolor torácico intermitente, durante la atención de urgencias de descarto cuadro isquémico, con marcadores negativos asi como ECG reportado como normal, la prueba de esfuerzo fue positiva reversible, se administra acido acetilsalicilico pero la paciente refiere malestar continuo por dolor torácico intermitente, sin embargo continua con sus actividades cotidianas, no cuenta con antecedentes familiares de cardiopatía isquémica, sin embargo es fumadora social. PREGUNTA Cuál es la conducta a seguir para establecer una aproximación diagnostica. RESPUESTA a.- Realizar cateterismo cardiaco. b.- Realizar tomografía computada cardiaca. c.- Realizar ecocardiograma de estrés. d.- Evaluación del dolor de origen no cardiaco.

CURSO ENARM CMN SIGLO XXI TEL: 36246001

Pharmed Solutions Institute

PÁGINA 221

MANUAL DE TRABAJO DEL CURSO ENARM CMN SIGLO XXI ARRITMIAS CARDIACAS: CIENCIAS BASICAS: La arritmia cardiaca se define como la alteración del ritmo cardiaco establecido como normal entre 60-100 lpm. Menor a 60 lpm, será bradicardia y si es mayor a 100 lpm, será taquicardia. La arritmia más frecuente es la fibrilación auricular. SALUD PUBLICA: La prevalencia de FA oscila 0,7-17,6%, La prevalencia de las otras 2 taquiarritmias supraventriculares más importantes, el flutter auricular y la taquicardia paroxística supraventricular (TPSV), es mucho más baja. La incidencia del flutter auricular es de 88 por 1,000,000 de personas/año. ACTUACION INICIAL EN SERVICIO DE URGENCIAS: Hay que sistematizar nuestra actuación para no pasar por alto aspectos esenciales. 1. Realizar ECG; verlo con calma y valorar los siguientes aspectos: a) si el ritmo es rápido o lento, b) si el ritmo es regular o irregular, c) si el QRS es estrecho o ancho, d) le presencia o no de ondas “p”, e) la relación de p/QRS, esto nos orientará la arritmia a la entidad etiológica más probable. 2. Monitorizar las constantes vitales: como la tensión arterial, saturación de oxigeno, frecuencia cardiaca y respiratoria, vigilar la perfusión tisular, para detectar precozmente los signos de bajo gasto cardiaco. Todas las alteraciones terapéuticas deben documentarse con tiras de registro de ECG. 3. Canalizar una vía venosa periférica y administrar oxigeno: es necesario ya que el shock puede no detectarse precozmente y la mala perfusión periférica dificulta obtener una via. El oxígeno lo administraremos en función de la oxigenación, para evitar hiperoxigenación. 4. Valorar estabilidad hemodinámica-los signos adversos y que indican mala evolución son; a) shock-palidez, sudoración, por aumento de actividad simpática, bajo nivel de conciencia, por disminución de FSC o hipotensión arterial. b) sincope-por disminución de FSC. c) insuficiencia cardiaca-edema pulmonar o fallo del VI o ingurgitación yugular, hepatomegalia por fallo de VD. d) isquemia miocárdica-por aumento de la demanda de oxigeno del miocardio. Si el paciente tolera bien la arritmia probablemente no haga falta hacerle nada o bien solo fármacos. Si la tolera mal, al final precisara una cardioversión eléctrica. Dos aspectos importantes en atención primaria: primero no debemos ser más agresivos con nuestra actuación que la propia arritmia lo es con el paciente y en segundo lugar, los fármacos antiarrítmicos combinados o a dosis altas pueden ser arritmogénicos: para evitar efectos secundarios es recomendable seguir la máxima de “un paciente, un solo antiarrítmico”. Adenosina y ATP son los nicos que escapan a esta máxima. DIAGNOSTICO DIFERENCIAL DE BRADICARDIAS:

Ritmo rápido o lento Ritmo regular o irregular QRS estrecho o ancho nda “p” Relación p/QRS Diagnostico probable Manejo

Ritmo rápido o lento Ritmo regular o irregular QRS estrecho o ancho nda “p” Relación p/QRS

Lento Regular Estrecho Presente Todas las ondas p conducen BRADICARDIA SINUSAL Causa más frecuente: síndrome del seno enfermo

Diagnostico probable Manejo

Ritmo rápido o lento Lento Ritmo regular o irregular Regular QRS estrecho o ancho Estrecho nda “p” Presente Relación p/QRS Alargamiento progresivo del PR hasta una onda “p” no conduce Diagnostico probable BLOQUEO AV 2° WENKEBACK (MOBITZ I) Manejo Habitualmente no precisa tratamiento

Ritmo rápido o lento Ritmo regular o irregular QRS estrecho o ancho nda “p” Relación p/QRS Diagnostico probable Manejo

Ritmo rápido o lento Ritmo regular o irregular QRS estrecho o ancho nda “p” Relación p/QRS Diagnostico probable Manejo

Lento Regular Estrecho Presente Todas las ondas p conducen, PR alargado (>0.21 mseg) BLOQUEO AV de 1° Habitualmente no precisa tratamiento

Lento Regular Estrecho Presente PR, constante, hasta que una onda “p” no conduce BLOQUEO AV 2° MOBITZ II Derivar a servicio de urgencias, precisara marcapasos

Lento Regular Estrecho Hay ondas “p” con ritmo regular Ninguna onda “p” conduce, no hay relación p/QRS BLOQUEO AV 3° ó COMPLETO Derivar a servicio de urgencias, precisara marcapasos

MANEJO DE LAS BRADICARDIA: Tratar las bradicardias, mal toleradas o que cumplan criterios de inestabilidad que se numeran a continuación: 1. Tensión arterial sistólica 5 sem), embolia de liquido amniótico 50%, abruptio placentae 50%, HELLP 15%, neoplasias, anormalidades vasculares, circulación extracorpórea, falla hepática severa, shunt arteriovenoso, rechazo de trasplante. PATOGENIA: La presencia de una condición asociada a CID. Como la sepsis o el trauma, inducen la activación del sistema de coagulación a través de la liberación de citoquinas, como IL-6, IL-1 y/o el factor de necrosis tu oral (TNF); integrados como parte de la respuesta inflamatoria e incrementando la cantidad de factor tisular asociado a factor VII. La liberación o exposición en alta concentración de sustancias procoagulantes, como el factor tisular en el SNC. Como ocurre en el traumatismo de cráneo, puede desencadenar un trastorno coagulopatico local y/o generalizarse en una CID. La activación del sistema de coagulación, con exceso de trombina y el subsiguiente deposito de fibrina en la microcirculación, favorecen la agregación plaquetaria y el consumo de factores de coagulación. Los glóbulos rojos quedan atrapados en las mallas de fibrina, otros sufren ruptura mecánica al pasar entre la malla de fibrina (responsable de la esquistocitosis). La microtrombosis intravascular, promovida en la micro-vasculatura parenquimatosa y sistémica, generan un aumento de plasmina, que degrada el fibrinógeno, la fibrina y otros factores de la coagulación; perpetuándose el circulo vicioso de coagulaciónfibrinólisis. El funcionamiento suboptimo de los sistemas anticoagulantes naturales, proteína C o antitrombina, desbordados por el incremento de trombina y plasmina, permiten que estas circulen libremente por el torrente circulatorio. El consumo de los factores de la coagulación y plaquetas, junto a la hiperfibrinolisis favorecen la aparición de hemorragias. FORMAS DE PRSENTACION: A) según el tiempo de instauración y progresión: aguda y crónica. B) Por compromiso y extensión: localozada, sistémica. C) Por sus manifestaciones: hemorrágica(reacción hemolítica trasnfucional, leucemia promielocitica), trombótica (síndrome de Trousseau). DIAGNOSTICO: Clínico; hemorragia, hemorragias cutáneo mocosas generalizadas (epistaxis, hematuria, petequias, equimosis, gingivorragia, hemorragia digestiva, hemorragia en SNC), hemorragia tras incisión quirúrgica, heridas, catéteres, o punciones vasculares, trombosis, púrpura fulminante, acrocianosis periférica, gangrena de extremidades, anemia microangiopatica, en SNC-delirio, coma, en pulmón-diestress, en riñóninsuficiencia renal aguda, necrosis cortical, en tubo digestivo-úlceras duodenales. La CID aguda se manifiesta como hemorragias, trombosis o ambas. Las hemorragias suelen ser de presentación aguda, en múltiples sitios, especialmente de punción venosa o arterial, y frecuentemente asociada a petequias y púrpura. La CID crónica se presenta de manera más solapada, con menores manifestaciones clínicas y de laboratorio, debe considerarse en paciente con patologías asociadas a esta entidad (neoplasias, feto muerto retenido, aneurisma de aorta, hemangioma, trasplante, HELLP, reacciones alérgicas), que presentan alteraciones leves de los parámetros de hemostasia o que sangran excesivamente frente a intervenciones quirúrgicas. El diagnostico de CID se fundamenta en el cuadro clínico: 1. Identificación de la entidad causal de CID. 2. Valoración de los sangrados y/o trombosis. 3. Alteraciones coaguló-métrica de laboratorio. Pruebas de hemostasia básicas para diagnostico de CID: 1. Recuento de plaquetas, un recuento bajo o en descenso progresivo, es una determinación sensible aunque no especifica. El 98% de casos de CID presenta plaquetopenia y el 50% se encuentra por debajo de 50,000 mm3. 2. Producto de degradación del fibrinógeno (PDF) y dímero D (DD), frecuentemente están elevados como manifestación de hiperfibrinolisis secundario al incremento de plasmina; son sensibles (95%) aunque poco específicos ya que otras condiciones como el trauma, cirugía reciente o tromboembolismo venoso pueden incrementarlos, al igual que el deterioro de la función renal o hepática, por una dos inducción de su eliminación. Aun no se han estandarizado los puntos de corte considerados incremento moderado y alto necesarios para el score diagnostico de CID. 3. Tiempo de pro trombina (TP) y tiempo de tromboplastina parcial (TTP), se encuentran prolongados en el 50-60% de los casos, aunque cerca de la mitad de los pacientes pueden presentar tiempos normales o incluso acortados por la presencia de factores de coagulación activados como la trombina o el factor X. 4. Fibrinógeno, su descenso es poco sensible (28%), se encuentra sobre todo en estadios avanzados de la enfermedad. 5. Frotis de sangre periférica, la presencia de esquistocitosis, suele encontrarse en el 50% de los casos. No existe un método diagnostico estándar pero se han desarrollado scores diagnostico. La sensibilidad del score de CID manifiesta la Sociedad Internacional de Hemostasia y Trombosis (ISTH) es de 91% y tiene una especificidad del 97%. TRATAMIENTO: La clave es la corrección de la enfermedad desencadenante y dar soporte. No se debe posponer la medida terapéutica requerida, por intentar corregir un parámetro de coagulación alterado. Hemoderivados; no están indicados para corregir parámetros de laboratorio, pero si en presencia

CURSO ENARM CMN SIGLO XXI TEL: 36246001

Pharmed Solutions Institute

PÁGINA 228

MANUAL DE TRABAJO DEL CURSO ENARM CMN SIGLO XXI de sangrado activo, previo a intervenciones invasivas o en aquellos con alto riesgo de sangrado. Plasma (15 o 30ml/kg), plaquetas (una unidad de plaquetas cada 10 kg de peso en pacientes sangrando con recuentos 80%), con este criterio al menos un 80% de las anafilaxias serian identificadas. Sin embargo existen presentaciones menos típicas, como las que presentan exclusivamente hipotensión. También se ha descrito que las manifestaciones digestivas se CRITERIOS CLINICOS PARA EL DIAGNOSTICO DE ANAFILAXIA asocian con una mayor gravedad. Por último es La anafilaxia es muy probable cuando se cumple uno de los tres criterios siguientes: importante tener en cuenta que la 1.Inicio agudo (minutos a horas) de un síndrome que afecta a la piel y/o mucosas (urticaria concurrencia de una exposición a un alérgeno generalizada, prurito, eritema, edema de labios, úvula o lengua) junto con al menos uno de los siguientes: potencial o conocido para el paciente apoya el a. Compromiso respiratorio (disnea, sibilancias, estridor, disminución del PEF, hipoxemia) diagnostico. Por ello se han establecido b. Disminución de la TA o síntomas asociados de disfunción orgánica (hipotonía, sincope, criterios, con ellos se espera identificar más del incontinencia) 95% de las anafilaxias. Cuando existe afección 2.Aparición rápida (de minutos a algunas horas) de dos o más de los siguientes síntomas tras la exposición a un alérgeno potencial para ese paciente cardiovascular con hipotensión se habla de a. Afectación de piel y/o mucosas shock anafiláctico. La dificultad en el b. Compromiso respiratorio diagnostico estriba en que no hay signos o c. Disminución de la TA o síntomas asociados de disfunción orgánica síntomas patognomónicos; lo que sí es típico es d. Síntomas gastrointestinales persistentes (dolor abdominal, cólico, vómitos) 3.Disminución de la TA en minutos o algunas horas tras la exposición a un alérgeno conocido para ese la rápida progresión en la gravedad e intensidad paciente de los síntomas. EVALUACION DE GRAVEDAD: a. Lactantes y niños: TA baja o descenso superior al 30% de la TA sistólica Se relaciona con la rapidez en la progresión de b. Adultos: TA sistólica inferior a 90 mmHg o descenso superior al 30% sobre la basal los síntomas, con el tipo de antígeno, vía de entrada y órganos afectados. Factores relacionados como edad avanzada, presencia de patología respiratoria o cardiovascular asociada, tratamiento con IECAS o betabloqueantes o una mastocitosis de base, se han asociado con reacciones graves y mayor mortalidad, por tanto es fundamental la evaluación ABCD. Las anafilaxias graves vienen definidas por la presencia de cianosis, saturación O2 40años, embarazo, parto y puerperio, obesidad, venas varicosas, viajes largos. Dentro de los estados hipercoagulables; están las trombofilias primarias hereditarias (deficiencia de antitrombina, proteína C y S, mutación del factor V de Leiden) y adquiridas (síndrome antifosfolipido secundario a LES). SALUD PUBLICA: Afecta aprox. a 2 millones de personas. Incidencia de 145 por 100,000. La TVP va de un caso por cada 10,000 de adultos jóvenes y 1 por cada 100 de adultos mayores. En personas de 65 a 69 años es de 1.8 habitantes por cada 1,000 y aumenta a 3.1 en el grupoentre 85-89 años. TEP Causa frecuente de mortalidad en pacientes hospitalizados (5-10%). Toda TEP, tiene su origen en una TVP o por lo menos el 80%. TEP masiva 0.3%. PATOGENIA: La TVP Se inicia con mayor frecuencia en las piernas (venas gemelares y soleas). SI hay daño a nivel endotelial se activa la cascada de coagulación, el trombo se asienta sobre el seno valvular, las corrientes de flujo cambian localmente, disminuye el calibre de la vena, se produce estenosis y mayor fuerza de rozamiento, que permite el crecimiento del trombo y posteriormente adherencia, al disminuir el retorno venoso de la extremidad, hay acumulación de líquido y aumento de la presión hidrostática, lo que lleva a Edema, en la zona donde ocurre la oclusión aumenta la extracción de O2, de los tejidos lo que produce cianosis. Lo peligroso no es que se asiente, si no que siga creciendo y se fragmente y produzca embolismo que llega a obstruir la circulación arterial pulmonar (TEP) en diferentes grados. En la TEP en estadios tempranos puede tener un GC normal o levemente elevado, hay hipoxemia (disminución de la relación ventilación/ perfusión), esto aumenta la precarga, generando un gradiente de presión que llevara a isquemia subendocardica y finalmente falla cardiaca derecha con la consiguiente disminución de la distensibilidad del VI. En la TEP masiva no hay mecanismos compensatorios, manifestándose clínicamente como taquicardia sinusal y finalmente como hipotensión arterial sistémica, que llevara a la muerte en poco tiempo. DIAGNOSTICO: TVP: No puede ser afirmado, ni excluido por clínica, la mayoría asintomático. En parte distal a sitio de oclusión; edema, dolor, calor local, cianosis (+ en región plantar), circulación colateral, cordón venoso palpable. En caso severo flegmasía cerulea dolens (edema severo, cianosis marcada, flictenas y puede llegar a gangrena por compromiso de retorno linfático). Signo d Homans: dolor en parte alta de pantorrilla al realizar dorsiflexion del tobillo. Maniobra de Olow: apretar pantorrilla posterior en busca de dolor. TEP: El dx. requerirá de un alto grado de sospecha clínica, basándose en; a) uno o mas factores de riesgo, b) comorbilidades (ej. cáncer), c) síntomas y signos clínicos, d) hallazgos paraclinicos, e) demostración objetiva de hipoxemia y trombosis. Sintomatología como: disnea súbita sostenida, taquicardia, hipotensión arterial sistólica (90°, T negativa profundas en V1-V4. Gasometría; hipoxemia y alcalosis respiratoria aguda. Dímero D (DD, es un producto de degradación de los puentes de fibrina, que se libera a la circulación), se eleva cuando existe un proceso trombotico, si el valor es 3 días +1. 4. Dolor localizado +1. Aumento de tamaño de miembros inferiores +1. Edema en pitting unilateral +1. Dilatación venosa superficial +1. Otros dx. posibles -2. Wells >2 Alta probabilidaf; 40a y duración > 30 min. o 40años con historia de TVP o TEP o en qx. abdominal o pelvica para tx. de neoplasia o qx. ortopédica mayor en miembros inferiores. Incidencia de TVP distal 40-80%; proximal 10-20%; TEP mortal 1-5%. Recomendaciones: Prótesis de cadera o rodilla, HBPM c/12 hrs SC. profiláctico cada 12 hrs. terapéutica cada 24 hrs. PRONOSTICO: Dependerá de la sospecha clínica, la instalación de la profilaxis, y recordar que una TVP hasta en 80%, precede a una TEP CASO CLINICO Se trata de paciente masculino de 62 años de edad el cual acude a consulta debido a la presencia de dolor en miembros inferiores, generando dificultad para caminar. Cuenta con antecedentes de tabaquismo positivo hasta la actualidad refiere que con menor intensidad que antes comenzando a los 20 años de edad, hace dos años ha presentado leve mareos y en una ocasión fue internado por dificultad para movilizar su cuerpo, pero se recupero sin secuelas aparentes.

CURSO ENARM CMN SIGLO XXI TEL: 36246001

Pharmed Solutions Institute

PÁGINA 233

MANUAL DE TRABAJO DEL CURSO ENARM CMN SIGLO XXI PREGUNTA Considerando la clasificación de Rutherford cual es la categoría que presenta el paciente? RESPUESTA a.- 2. b.- 3. c.- 1. d.- 4. PREGUNTA Considerando las manifestaciones clínica del paciente cual es la causa menos probable de este caso? RESPUESTA a.- Compresión de una raíz nerviosa. b.- Proceso artrítico inflamatorio. c.- Sindrome compartamental crónico. d.- Insuficiencia venosa. PREGUNTA Cual es la conducta a seguir mas adecuada para establecer una terapéutica adecuada en el paciente para identificar factores de riesgo modificables? RESPUESTA a.- Buscar niveles de glucosa, colesterol y triglicérido. b.- Suspensión de tabaquismo absoluta. c.- Holter para identificar origen cardiológico. d.- Ejercicio secuencial programado. CASO CLINICO Acude a consulta paciente femenino de 34 años de edad, originaria de yucatan la cual tiene como antecedentes de importancia 7 semanas de posparto y obesidad grado II, viajo mas de 12 horas por carretera para visitar a familiares, solicita atención debido a presencia de dolor e inflamación de miembro pélvico derecho, a la exploración se observa dicha extremidad con inflamación, hipersensibilidad y caliente al tacto, con signo de Hoffman positivo, sus signos vitales son estables TA 115/75 mmHg, FC 81, FR 17, temperatura 37 C, se realiza doppler apoyando el diagnostico de trombosis venosa, se ingresa para tratamiento. PREGUNTA Cuál es el manejo farmacológico más adecuado para esta paciente: RESPUESTA a.- Heparina IV b.- Enoxoparina. c.- Warfarina. d.- Acecumarol. PREGUNTA Cual es la complicación mas importante que debe evitarse en el paciente? RESPUESTA a.- Tromboembolia pulmonar aguda. b.- Evento embolico fatal subsecuente. c.- Sindrome compartamental. d.- ECV. PREGUNTA Considerando las reglas predictivas de Wells para trombosis venosa profunda, que calificación presenta el paciente en este momento? RESPUESTA a.- 2. b.- 3. c.- 4. d.- 1. PREGUNTA Considerando las reglas predictivas de Wells para embolia pulmonar? RESPUESTA

CURSO ENARM CMN SIGLO XXI TEL: 36246001

Pharmed Solutions Institute

PÁGINA 234

MANUAL DE TRABAJO DEL CURSO ENARM CMN SIGLO XXI a.- 6. b.- 4. c.- 9. d.- 3. CASO CLINICO Se trata de paciente femenino de 38 años de edad con antecedentes de importancia para el padecimiento actual, usuaria de anticonceptivo orales desde hace 8 años, tabaquismo positivo desde los 18 años, actualmente con IMC 32. Acude a urgencias refiriendo que hace 24 horas regreso de un viaje de 12 horas de camino, el cual realizo en auto ella era la conductora, refiere que comenzó con dolor en las pantorrillas de predominio izquierdo, con leve dificultad para caminar, posteriormente se presenta malestar generalizado, dificultad para respirar motivo por el cual se traslada al hospital, a la exploración física se observa diaforética con dolor torácico, disnea, sus constantes vitales son 95/60 mmHg, FC de 115 lpm, FC de 30 rpm, SaO2 de 90%, campos pulmonares si datos por agregar, se realiza radiografia de torax, y medición de gases arteriales siendo anormales. PREGUNTA Cual es el manejo mas apropiado para el caso. RESPUESTA a.- Enoxaparina. b.- Heparina. c.- Warfarina. d.- Tecneplasa SÍNDROME DE INSUFICIENCIA RESPIRATORIA AGUDA (SDRA): CIENCIAS BÁSICAS: Antiguamente denominado síndrome de dificultad respiratoria del adulto, es un trastorno que se caracteriza por una insuficiencia respiratoria aguda hipoxémica debida al edema pulmonar causado por el aumento en la permeabilidad de la barrera alveolocapilar. El SDRA constituye la manifestación más grave de una serie de respuestas al daño pulmonar agudo; estas respuestas traducen las complicaciones de una reacción sistémica más extensa a la inflamación o agresión agudas. Se dice que ocurre un daño pulmonar agudo cuando se observa una hipoxemia grave, de comienzo agudo, y opacidades bilaterales difusas en una radiografía anterior de tórax, después de excluir una hipertensión en la aurícula izquierda o en los capilares pulmonares. El SDRA se diferencia del daño pulmonar agudo por la gravedad de la hipoxemia y se define como una relación igual o menor a 200 mmHg entre la pO2 arterial y la fracción de oxígeno en el aire inspirado (PaO2/FIO2). El daño pulmonar agudo y el SDRA deben considerarse como las manifestaciones primeras y más fáciles de reconocer de un trastorno sistémico infeccioso o inflamatorio. El pulmón adquiere una enorme importancia en la agresión sistémica por que recibe todo el gasto cardíaco y porque las alteraciones de su función se reflejan rápidamente en clínica. Más de la mitad de los casos comienzan en las primeras 24 horas desde la primera agresión. El daño pulmonar agudo se asocia íntimamente a trastornos que producen un daño alveolar directo o un daño indirecto a través del lecho capilar pulmonar como e indicen a SDRA: Lesión directa del epitelio alveolar, aspiración, infección difusa, semiahogamiento, inhalación tóxica, contusión de la vía respiratoria, daño pulmonar indirecto, síndrome séptico, traumatismo no torácico grave, derivación cardiopulmonar. SALUD PÚBLICA: La probabilidad del SDRA varía según la causa desencadenante, desde un 13 % en los casos de sobredosis farmacológicas hasta un 43 % en la sepsis. PATOGENIA: El daño pulmonar agudo es la consecuencia de la expresión excesiva, sin ningún tipo de regulación, de las respuestas inflamatorias generales y habituales a la infección, la agresión o ambas. El daño afecta al epitelio alveolar y al endotelio capilar pulmonar, consecuencia del acontecimiento que inicia la cascada tan compleja de reacciones celulares y bioquímicas. Estos acontecimientos atraviesan tres etapas: 1. Iniciación; en la que el factor desencadenante activa la cascada celular. 2. Amplificación: en la que se reclutan y activan las células efectoras. 3. Daño; fase en la que los acontecimientos se expresan en los tejidos. El daño lo producen los acontecimientos celulares asociados a los neutrófilos, macrófagos, monocitos y linfocitos, que sintetizan diversas citocinas; estas, a su vez, determinan una activación, quimiotaxis y adherencia celular. Las células activadas producen una serie de mediadores inflamatorios, como los oxidantes, las proteasas, las caninas, los factores de crecimiento, los neuropép dos, los activadores de la cascada del complemento, la coagulación intravascular y la fibrinólisis. La característica fisiopatológica del SDRA es un aumento en la permeabilidad vascular a las proteínas, que determinan la falta de oposición al gradiente hidrostática; por eso incluso elevaciones discretas de la presión capilar (producidas por una sobrecarga de líquidos por vía IV o la disfunción cardiaca característica de la sepsis) aumentan de forma considerable el edema intersticiales y alveolar. Este efecto aditivo de la permeabilidad y los factores hidrostática, se ilustra como la mayor gravedad del proceso en las zonas declives. El aumento entre la relación del tejido pulmonar y el gas situado en estas zonas determina que las presiones de cierre alveolar excedan de las presiones transpulmonares locales y se produzca un cierre y un colapso alveolares. La tendencia al colapso se agudiza por la disminución cuantitativa de la síntesis de sustancia tensioac va, debido a la agresión de los neumocitos de po II y a otras anomalías en el tamaño, composición y metabolismo del resto de la sustancia tensioac va depositada. Estas zonas atelectásicas del pulmón contribuyen a reducir la distensibilidad del pulmón en su conjunto, pero las zonas de pulmón, no situadas en declive, poseen propiedades mecánicas y de intercambio gaseoso normales. Gran parte de la ventilación y del intercambio gaseoso se desplaza hacia estas regiones pulmonares intactas; para establecer un símil, es como si la función respiratoria del adulto fuera suplida por un par de pulmones de un bebé. Dada la distensibilidad menor de los pulmones, es necesario que los músculos respiratorios generen altas presiones inspiratorias, con lo que aumenta el trabajo de la respiración. Esta mayor carga mecánica explica la fatiga de los músculos respiratorios, la disminución consiguiente de los volúmenes corrientes y el empeoramiento del intercambio gaseoso. La hipoxemia y la estimulación de los receptores del parénquima pulmonar rígido determinan un aumento de la frecuencia respiratoria., una disminución del volumen corriente y un deterioro del intercambio gaseoso. La resistencia en las vías respiratorias aumenta porque el volumen minuto debe sostenerlo un menor n mero de vías respiratorias ven ladas y por el estrechamiento de las vías causado por el exceso de líquidos y el broncoespasmo. El intercambio gaseoso se caracteriza por relaciones ven lación-perfusión bajas y un cortocircuito amplio,

CURSO ENARM CMN SIGLO XXI TEL: 36246001

Pharmed Solutions Institute

PÁGINA 235

MANUAL DE TRABAJO DEL CURSO ENARM CMN SIGLO XXI acompañado de un gran espacio muerto. El cortocircuito obedece a la atelectasia, colapso vascular, mal funcionamiento de la sustancia tensioactiva y atenuación de la vasoconstricción hipóxica. El aumento del espacio muerto se debe a la obstrucción y obliteración del lecho capilar pulmonar. DIAGNOSTICO: Clínico; al inicio puede no mostrar síntomas, ni signos respiratorios. El signo más precoz es el aumento de la frecuencia respiratoria, seguido poco después de disnea. En el periodo inicial, la determinación de los gases en sangre arterial muestra disminución de la Po2 a pesar de la disminución de la Pco2, por lo cual aumenta la diferencia de oxígeno alveolo arterial. En este estado inicial, la administración de oxígeno produce un incremento significativo de la Po2 arterial. La exploración física puede ser poco llamativa, aunque se pueden auscultar algunos estertores inspiratorios finos. Radiológicamente, los campos pulmonares pueden ser claros o mostrar sólo mínimos infiltrados intersticiales focales. A medida que avanza la enfermedad, el paciente se torna cianótico, con disnea y taquipnea crecientes. Pueden hacerse intensos los estertores, que se oirán fácilmente en todos los campos pulmonares, junto a zonas de ruido tubáricos; la radiografía de tórax muestra extensos infiltrados intersticiales y alveolares de carácter bilateral y difuso. En este momento, la hipoxemia no se puede corregir simplemente aumentando la concentración de oxígeno en el aire inspirados y hay que comenzar con la ventilación asistida. En este estadio más avanzado, el mecanismo principal de la hipoxemia arterial es el cortocircuito de la sangre desde la derecha a la izquierda, a través de los alveolos colapsados u ocupados. El diagnostico de IRA se basa fundamentalmente en la determinación de gases arteriales al encontrar PaO2 50 torr. Las manifestaciones únicas de hipoxemia o hipercapnia, sirven para el reconocimiento de la presencia de anormalidades importantes en el recambio gaseoso, mas no para el diagnostico, ya que pueden ocurrir tardíamente o aun faltar en presencia de IRA. La falla respiratoria puede clasificarse en 2 tipos: Tipo I: llamada también oxigenatoria o hipoxemica; hipoxemia con PaCO 2 normal o bajo. Gradiente alveolo-arterial de O2 incrementado. Tipo II: denominada así mismo ventiladores o hipercapnica; hipoxemia con PaCO2 elevado y gradiente alveolo-arterial de O2 normal. TRATAMIENTO: Tratamiento de sostén de la hipoxemia. Para alcanzar una Pao2 de 60 mmHg (saturación de O2 de aprox. el 90 %) debe emplearse el método más sencillo y la fracción más baja de oxígeno en el aire inspirado. Los niveles más altos apenas añaden más oxígeno y, en cambio, conllevan el riesgo de toxicidad pulmonar por esta sustancia. Los tres métodos fundamentales para la oxigenación, en orden de eficacia creciente, son las gafas nasales blandas, las mascarillas faciales simples y las mascarillas faciales con una bolsa de reserva inspiratoria. Parece razonable empezar con flujos moderados (5-10 L/min de O2 al 100 %) y controlar la gasometría arterial; el flujo y la concentración de O2 se ajustan en función de los resultados. Soporte ventilatorio mecánico; en presencia de SDRA, estas medidas poco enérgicas no suelen mantener una oxigenación adecuada, sino que se requiere la intubación endotraqueal y la respiración asistida con un respirador volumétrico. El motivo de aplicar respiración asistida a un enfermo que hiperventila no es incrementar la ventilación sino el volumen pulmonar medio, abriendo las vías respiratorias previamente cerradas y mejorando la oxigenación. El objetivo de la ventilación en el SDRA se basa en proporcionar un soporte fisiológico para el intercambio gaseoso y evitar las consecuencias mecánicas desfavorables de la intervención. Como el pulmón sufre un daño heterogéneo en el SDRA, el ajuste de los volúmenes corrientes para lograr este objetivo en todo el pulmón acarrea el riesgo de sobredistender o romper los alveolos menos afectados y ocasionar la entrada de aire fuera del alveolo (barotraumatismo). Para evitar esta ultima complicación hay que restringir las presiones de distensión alveolares mediante el uso de volúmenes corrientes relativamente pequeños (aproximadamente 6 a 10 mL/kg de peso corporal magro) y ajustar la presencia respiratoria, de tal suerte que el volumen minuto se asocie a un pH superior a 7.25 – 7.30. Dado el volumen tan reducido del pulmón aireado, se necesitan frecuencias elevadas para alcanzar un volumen minuto adecuado. Tratamiento de la sepsis que causa SDRA: Identificar y tratar cualquier proceso infeccioso, y Tratar de controlar la respuesta inmunitaria, sin regulación, que provoca el daño pulmonar agudo. Todo foco localizado de sepsis se debe drenar de inmediato. La mortalidad quirúrgica es elevada, pero si no se drena un foco purulento, lo más probable es que el enfermo muera. Complicaciones: Las medidas empleadas para tratar el SDRA pueden asociarse a graves complicaciones. La toxicidad del oxígeno, debido al uso prolongado de una Fio2 superior al 50 %, y la hidratación excesiva empeoran a veces los infiltrados pulmonares. Un volumen y presión corrientes elevados, por parte del respirador, son causa de barotraumatismo. La ventilación mecánica prolongadas predispone a la neumonía hospitalaria. El SDRA y los trastornos subyacentes favorecen las fístulas boncopleurales y la coagulación intravascular diseminada. La fibrosis pulmonar, la hipertensión pulmonar irreversible y la insuficiencia multiorgánica sugieren un mal pronóstico en los enfermos con SDRA. CASO CLINICO Se trata de paciente masculino de 54 años de edad el cual es ingresado a urgencia traido por ambulancia desde una distancia de 4 horas de camino, refieren los paramédicos que es sobreviviente a accidente automovilístico, al parecer era el conductor, a la exploración conciente, orientado, con facies algicas, se observa contusion en esternón y parrilla costal de predominio izquierdo, discreta disnea, hematomas multiples en miembros pelvicos, se encuentra fractura de femur no expuesta, signos vitales TA 105/80 mmHg, FC 105, FR 32. Fue estabilizado y se procedió a realizar cirugía de femur, se traslado a piso al 3 dia el paciente inicia con fiebre, disnea progresiva, edema de miembros inferiores, se realizo radiografia de torax sin observarse hallazgo patológicos, el electrocardiograma mostro un eje de QRS mayor a 90 oC y ondas T negativas en V1 a V4. PREGUNTA Cual es su impresión diagnostica mas probable con el cuadro clínico del paciente hasta el momento? RESPUESTA a.- Tromboembolia pulmonar. b.- SIRA. c.- Embolia pulmonar grasa. d.- Neumotorax. PREGUNTA Se realiza tratamiento anticoagulante sin aparente mejoría, a las 12 horas posteriores se incrementa la disnea y ortopnea, intolerancia al decúbito y ansiedad generalizada, tos con expectoración, dificultad respiratoria con uso de musculos accesorio, se realiza Rx de torax portátil con sospecha de edema agudo de pulmon, cuales de los siguientes hallazgos no corresponden a ese diagnostico?

CURSO ENARM CMN SIGLO XXI TEL: 36246001

Pharmed Solutions Institute

PÁGINA 236

MANUAL DE TRABAJO DEL CURSO ENARM CMN SIGLO XXI RESPUESTA a.- Opacidades alveolares diseminadas y bilaterales. b.- Redistribución del flujo pulmonar. c.- Distribución en ala de mariposa. d.- Lineas B de Kerley. EDEMA AGUDO PULMONAR (EAP): CIENCIAS BÁSICAS: El EAP no es en sí una enfermedad, pero frecuentemente es una complicación de alguna otra anormalidad, la cual puede tener origen en los pulmones o más comúnmente, en órganos extrapulmonares, (corazón). El edema pulmonar constituye una verdadera urgencia médica, y por ello debe identificarse rápidamente. El cuadro puede ser de origen cardiogénico o no cardiogénico. En el EAP cardiogénico, un fallo de bombeo hace que la sangre retroceda a la circulación pulmonar. Cuando la presión hidrostática capilar pulmonar supera la presión osmótica coloidal, el líquido es impulsado fuera de los capilares pulmonares hacia el espacio intersticial y el alveolo. En el de origen no cardiogénico, las paredes de las células capilares pulmonares se vuelven más permeables. Constituye una de las urgencias más graves en pacientes con enfermedades del corazón izquierdo. curre por aumento de la presión capilar pulmonar secundaria a estenosis mitral o insuficiencia cardiaca izquierda, de ahí que también se le conozca como “edema pulmonar cardiogénico”. PATOGENIA: El EAP es el acumulo excesivo de líquido extravascular en el pulmón, ya sea en el intersticio (edema intersticial) o en el alveolo (edema alveolar). Para mantener seco el intersticio pulmonar funcionan varios mecanismos delicados: 1. Presión osmótica superior a la presión capilar pulmonar; Las fuerzas hemodinámicas básicas opuestas son la presión capilar pulmonar (PCP) y la presión osmótica del plasma. En los individuos normales la PCP oscila entre los 7 y los 12 mmHg, siendo la presión osmótica del plasma de 25 mmHg aproximadamente, por lo que .esta fuerza tiende a impulsar el líquido de regreso a los capilares. 2. Tejido conjuntivo y barreras celulares relativamente permeables a las proteínas plasmáticas; La presión hidrostática actúa a través del tejido conjuntivo y la barrera celular, que en circunstancias normales son relativamente impermeables a las proteínas plasmáticas. 3. Extenso sistema linfático; El pulmón posee una extensa red linfática que puede aumentar su flujo en 5 a 6 veces cuando se encuentra con un exceso de agua en el inters cio pulmonar. Cuando los mecanismos normales para mantener el pulmón seco funcionan mal o están superados por un exceso de líquidos el edema ende acumularse. Las causas de EAP son: Cardiogénicas: IAM, arritmias cardiacas, insuficiencia ventricular izquierda grave, shoc cardiogénico. No cardiogénicas: Inhalación de gases irritantes, neumonía por aspiración, Shock séptico, embolia grasa, síndrome de distrés respiratorio del adulto, administración rápida de líquidos intravenosos, sobredosis de barbitúricos u opiáceos. DIAGNOSTICO: Agitación, angustia, nerviosismo. La valoración del edema agudo del pulmón a nivel respiratorio es laborioso; disnea (el edema pulmonar dificulta la distensibilidad pulmonar y hace más difícil la expansión de los pulmones). Hipoxemia (debido al deterioro del intercambio gaseoso). Existencia de hipoxia con hipocapnia (hiperventilación) ya que elimina gran cantidad de CO2 al respirar tan dificultosamente, desarrollando una alcalosis respiratoria. Taquipnea 30-40 min (debido a la hipoxia el paciente respira más de prisa). Crepitantes y sibilancias (al pasar el aire a través de las vías aéreas llenas de liquido, durante la inspiración). Valoración cardiovascular: Taquicardia y aumento de la presión arterial (para compensar el déficit de aporte de oxigeno, el sistema simpático aumenta la frecuencia cardiaca). El sistema nervioso simpático provoca vasoconstricción que aumenta la presión arterial. Distensión de las venas yugulares. Esputo espumoso teñido de sangre (debido a la presión, algunos vasos pulmonares se rompen haciendo que los hematíes pasen a los pulmones). Piel pálida, fría y sudorosa (para conservar el oxigeno, el organismo deriva sangre arterial desde la piel hacia órganos vitales). Tercer ruido cardiaco (ritmo de galope), producido cuando durante el principio de la diástole la sangre entra de prisa en el ventrículo izquierdo distendiéndolo y da lugar a vibraciones anormales. En el tórax puede observarse el empleo de los músculos accesorios de la respiración (tiros intercostales). Puede haber estertores audibles a distancia y sin necesidad del estetoscopio. uscar intencionadamente la presencia de ritmo de galope que permi rá establecer el diagnós co de insuficiencia cardiaca, mientras que con la auscultación de un ritmo de Duroziez haremos el diagnóstico de estenosis mitral. La radiografía de tórax es el estudio más útil para el diagnóstico en conjunto con la historia clínica del EAP, ya que muestra la imagen correspondiente al edema alveolar como una opacidad diseminada en ambos campos pulmonares. Cuando los grados de hipertensión venocapilar pulmonar son menores, podrán observarse otros datos como son: la redistribución de flujo a los vértices, la cisura interlobar visible, las líneas de erley o un moteado fino difuso y en ocasiones la imagen en “alas de mariposa”; la presencia de cardiomegalia orientará hacia el diagnós co de insuficiencia cardiaca, mientras que la ausencia de ella hablará de disfunción diastólica. El electrocardiograma ayuda a identificar la posible cardiopatía subyacente o los factores desencadenantes, como un infarto del miocardio o una taquiarritmia como la fibrilación auricular. TRATAMIENTO: El manejo terapéutico de esta grave complicación tiene tres objetivos: 1. Disminuir la presión venocapilar: 2. Mejorar la ventilación pulmonar. 3. Tratamiento de la enfermedad causal. Medidas que disminuyen la hipertensión venocapilar: a). Sentar al paciente al borde de la cama con las piernas pendientes para disminuir el retorno venoso al corazón. b). Aplicación de torniquetes rotatorios en tres de las cuatro extremidades, con la fuerza necesaria para que no desaparezca el pulso arterial y se cambia cada 15 minutos a la extremidad que se encuentra libre (“sangría seca”). c). Isosorbide o nitroglicerina por vía sublingual. La administración de una tableta de cualquiera de estos dos fármacos tiene un poderoso efecto vasodilatador que reduce drásticamente el retorno venoso y la congestión pulmonar. d). La administración intravenosa de furosemida a razón de 20 a 60 mg promueve con gran rapidez la movilización de líquidos del intersticio pulmonar hacia el riñón, por lo que se reduce con gran eficiencia la congestión pulmonar. e). Nitroprusiato de sodio.- Este poderoso vasodilatador mixto se administra por vía intravenosa a razón de 0.3 a 0.8 microgramos/Kg/minuto, cuando el EAP es consecutivo a insuficiencia ventricular por crisis hipertensiva. En estos casos, esta medida es de elección para el tratamiento del edema pulmonar. El efecto arteriodilatador reduce significativamente la postcarga y con ello mejora la función ventricular y el gasto cardiaco, y por otro lado, el efecto venodilatador reduce el retorno venoso al corazón y la congestión pulmonar. Medidas que mejoran la ventilación pulmonar: a). Administración de oxígeno mediante puntas nasales a razón de 4 litros por minuto. b). Aminofilina 1 ampolleta de 250 mg muy lenta por vía intravenosa; su efecto broncodilatador mejora la ventilación pulmonar así como su efecto diuré co potencia la acción de la furosemida. Tratamiento de la enfermedad causal: a). La mayoría de los pacientes que presentan EAP por estenosis mitral mejoran con las medidas antes anotadas. Cuando a pesar de ellas continúa el cuadro clínico, se requiere la intubación del paciente para administrarle asistencia mecánica a la ventilación pulmonar, especialmente la presión positiva

CURSO ENARM CMN SIGLO XXI TEL: 36246001

Pharmed Solutions Institute

PÁGINA 237

MANUAL DE TRABAJO DEL CURSO ENARM CMN SIGLO XXI respiratoria final (PEEP) que evita el colapso de las vías respiratorias pequeñas y con ello asegura la ven lación alveolar. En estas condiciones, el paciente deberá ser llevado al quirófano para realizar comisurotomía mitral de urgencia, que en estos casos es la única medida que alivia el cuadro de edema pulmonar. b). Cuando la insuficiencia ventricular izquierda es la causa, se requiere la digitalización rápida con ouabaína o lanatósido C, además de las medidas para reducir la presión capilar y mejorar la ventilación pulmonar. c). El edema pulmonar que es causado por disfunción diastólica, usualmente se presenta en la evolución de un infarto del miocardio agudo; debe ser tratado con las medidas que reducen la presión capilar pulmonar asociada a aquellas otras que mejoran la ventilación pulmonar y ambas a su vez, a las que reducen el efecto de isquemia miocárdica como lo son la administración de betabloqueadores (propranolol 10 a 20 mg cada 8 horas) o calcio antagonistas (diltiazem 30 mg cada 8 horas), ya que estos fármacos reducen el efecto que la isquemia tiene sobre la relajación ventricular. En efecto, cuando la relajación se hace más rápida y completa, se reduce significativamente la presión diastólica del ventrículo izquierdo y con ello la hipertensión venocapilar. Estabilización hemodinámica: Se considera una buena respuesta si a los 15 a 30 minutos del inicio del tratamiento, mejora la disnea, la frecuencia cardiaca, disminuyen los estertores pulmonares, aparece una coloración más oxigenada de la piel y se produce una buena diuresis, después de la cual se le puede ofertar al paciente potasio, 1 ampolleta por vía oral con agua para compensar las pérdidas de de este ion o adicionárselo a la infusión de la venoclisis CASOS CLINICOS Mujer de 25 años de edad, primigesta, admitida por cuadro de preeclampsia severa, con presiones arteriales > 160/110 mmHg, manejada con sulfato de magnesio y labetalol intravenoso sin obtener adecuada respuesta, por lo que se realiza cesárea, que culmina con recién nacido sano, se traslada posteriormente a unidad de cuidados intensivos (UCI) para manejo con infusión de vasodilatadores y vigilancia neurológica por riesgo elevado de eclampsia. Al ingreso en UCI presentó presión arterial de 161/120 mmHg, taquicardia, inadecuado patrón respiratorio, saturación de oxígeno arterial (SaO2) del 89%, crepitantes bibasales y presión venosa central (PVC) de 16 mmHg. Se realiza radiografía de tórax que muestra cefalización del flujo, con hilios pulmonares congestivos; se considera entonces el diagnóstico de EAP. PREGUNTA Cual es la conducta mas adecuada para el caso? RESPUESTA a.- Ventilacion mecánica. b.- Antibioticoterapia profiláctica. c.- Furosemide y nitroglicerina. d.- Sindenafil. CASO CLINICO Paciente de 68 años de edad, fumador con criterios clínicos de EPOC, que acudió al servicio de urgencias por disnea progresiva hasta hacerse a mínimos esfuerzos de 2 semanas de evolución. No refería traumatismo torácico. Radiológicamente, se diagnosticó un neumotórax izquierdo. Se insertó un tubo de drenaje torácico, se reexpandió completamente el pulmón y mejoró así la clínica del paciente. Tres horas más tarde tuvo un deterioro clínico consistente en dificultad respiratoria grave. Se auscultaban estertores húmedos de gruesa burbuja en el hemitórax izquierdo. Una nueva RX de tórax mostró imágenes compatibles con edema pulmonar unilateral. Fue preciso conectarlo a ventilación mecánica y que recibiera tratamiento vasoactivo. PREGUNTA Cual es la etiología mas frecuente del caso. RESPUESTA a.- Arritmia cardiaca. b.- Isquemia miocárdica. c.- Insuficiencia cardiaca. d.- Complicaciones del parénquima pulmonar. NEUMOTÓRAX, NEUMOMEDIASTINO, HEMOTÓRAX, CONTUSION CARDIACA: NEUMOTÓRAX: Presencia de aire en el espacio pleural que causa un mayor o menor colapso del pulmón, con la correspondiente repercusión en la mecánica respiratoria e incluso en la situación hemodinámica del paciente. La presión intrapleural es subatmosférica durante todo el ciclo respiratorio, y es el resultado de las fuerzas elásticas opuestas del pulmón y de la pared torácica. La presión intraalveolar es relativamente positiva en relación con la intrapleural, ayudando a mantener el pulmón en expansión durante la respiración. La entrada incidental de aire en el espacio pleural provoca una pérdida de la presión negativa intrapleural y el pulmón tiende a colapsarse por su propia fuerza retráctil. Si la cantidad de aire es importante, la presión pleural se aproxima a la atmosférica, lo que puede causar un colapso de todo el pulmón. Es frecuente que exista hipoxemia arterial como consecuencia de la desproporción de la relación ventilación/perfusión que se produce en el pulmón colapsado (shunt intrapulmonar). La entrada de aire al espacio pleural desde el pulmón, como consecuencia del desgarro de la pleural visceral, es la causa más frecuente de neumotórax. NEUMOTÓRAX ESPONTÁNEO (NE): Ocurre en ausencia de antecedente traumático o yatrogénico que lo justifique. El NE se subclasifica en: NE primario; ocurre en individuos aparentemente sanos, sin enfermedades pulmonares conocidas. Afecta a jóvenes, con un pico de incidencia entre los 20 y 40 años de edad, y es más frecuente en varones (6:1), con predilección por individuos altos y delgados, fumadores. El sustrato patológico más frecuente es la presencia de pequeñas bullas subpleurales apicales (blebs) cuya rotura ocasiona la salida de aire desde el pulmón hacia la cavidad pleural. El mecanismo de formación de estos blebs y los factores que precipitan su rotura no son bien conocidos. NE secundario Ocurre en pacientes con patología pulmonar previa. Personas de mayor edad, excepto en

CURSO ENARM CMN SIGLO XXI TEL: 36246001

Pharmed Solutions Institute

PÁGINA 238

MANUAL DE TRABAJO DEL CURSO ENARM CMN SIGLO XXI los casos de fibrosis quística, EPOC es la causa más frecuente y la probabilidad de neumotórax es mayor cuanto más avanzada es la enfermedad. En estos pacientes el neumotórax se debe a la rotura de bullas intrapulmonares y, dado que su reserva funcional es ya muy limitada. Del 2 al 4% de pacientes con SIDA pueden desarrollar neumotórax. La neumonía por Pneumocystis carinii es la etiología más probable en estos pacientes. La tuberculosis pulmonar y las neumonías necrotizantes, fundamentalmente las causadas por estafilococo, también pueden ser causa de neumotórax. Aunque es rara, no debemos olvidar una entidad como el neumotórax catamenial, que ocurre en mujeres con antecedentes de endometriosis entre otras. NE ADQUIRIDO: Iatrogénico: consecuencia de procedimientos invasivos, como toracocentesis, biopsia pulmonar transbronquial, biopsia pleural, lavado broncoalveolar, punción pulmonar transtorácica, el neumotórax por barotrauma (complicación de la ventilación mecánica, por rotura de alveolos). NEUMOTORAX TRAUMATICO: neumotórax traumático abierto es consecuencia de una herida penetrante en el tórax, que pone en comunicación el espacio pleural y la atmósfera exterior (entrada de aire atmosférico), y a su vez suele lesionar también el pulmón (salida de aire alveolar). El neumotórax traumático cerrado está causado habitualmente por una fractura costal, rotura bronquial o lesión esofágica. Síntomas más frecuentes son: Dolor torácico pleurítico de inicio agudo, que puede ser intenso inicialmente, pero después de unas horas suele evolucionar a dolor sordo e incluso cede espontáneamente, disnea, que suele aparecer con mayor frecuencia e intensidad en pacientes con enfermedad pulmonar subyacente. En el NE primario suele ceder habitualmente en el plazo de 24 horas. Otros síntomas (menos frecuentes): tos improductiva, síncope, hemoptisis, debilidad de extremidades superiores, sensación de ruido extraño en el tórax. Exploración física: hiperinsuflación y disminución del movimiento del hemitórax afectado, disminución o abolición del murmullo vesicular en el hemitórax afectado, disminución de la transmisión de la voz, timpanismo, disminución de las vibraciones vocales. Palpación hepática por aplanamiento diafragmático y desplazamiento del hígado. El neumotórax a tensión debe sospecharse ante la presencia de taquicardia superior a 135 latidos por minuto, hipotensión, o cianosis. Tratamiento: se dirige a la eliminación del aire inrapleural, así como la prevención de ataques de recurrencia, se basa en administración de oxígeno, observación y aspiración de aire intrapleural. NEUMOMEDIASTINO: Aire en el mediastino no acompañado de enfermedad inflamatoria o infecciosa (también se conoce como enfisema mediastinal) y es primordialmente una complicación de la ventilación mecánica (barotrauma9, aunque también puede ocurrir espontaneo o secundario a trauma torácico o asma. Ocurre principalmente por ruptura alveolar microscópica, así como por escape de aire de la vía aérea superior o gastrointestinal. Se presenta con dolor subesternal, que empeora con los movimientos respiratorios y cambios de posición. También puede presentarse con disnea, disfagia, enfisema subcutáneo y disfonía. El tratamiento depende de la causa primaria. HEMOTÓRAX: Presencia de sangre en el espacio pleural. El término hemotórax está reservado para los casos en los cuales el hematócrito del líquido pleural es al menos un 50% del hematócrito de la sangre periférica. Cuando en una toracocentesis diagnóstica se obtiene líquido hemático, debemos considerar la medición de su hematócrito. Los podemos clasificar en traumáticos, no traumáticos o espontáneos y iatrogénicos. Traumático; la sangre puede llegar al espacio pleural por lesión de la pared torácica, diafragma, parénquima pulmonar, vasos sanguíneos o desde estructuras mediastínicas. Cuando la sangre se acumula en el espacio pleural, ésta tiende a coagularse rápidamente, como resultado de los movimientos producidos por el corazón y los pulmones. Si el impacto ha sido muy intenso y la situación del paciente lo permite, la realización de un TAC podrá aportar importante información de posibles lesiones asociadas. De acuerdo a la extensión radiológica el hemotórax puede clasificarse en tres grados: 1°. El nivel del hemotórax se encuentra por debajo del cuarto arco costal anterior. 2°. El nivel se encuentra entre el cuarto y segundo arco costal anterior. 3°. El nivel está por encima del segundo arco costal anterior. Esta clasificación es utilizada como guía por algunos autores para decidir la conducta terapéutica: drenaje con tubo pleural en hemotórax de primer grado, toracotomía en aquellos de tercer grado y en aquellos de segundo grado se puede comenzar con drenaje torácico y, eventualmente, toracotomía de acuerdo a la evolución clínica del paciente. Los pacientes con hemotórax traumático deben ser tratados lo antes posible con drenaje torácico. Esto permite una completa evacuación de la sangre del espacio pleural, cuantificar las pérdidas, que disminuya la incidencia posterior de empiema, realizar autotransfusión del material obtenido y la rápida evacuación de la sangre del espacio pleural evitando un fibrotórax. Aproximadamente 20% de los pacientes con hemotórax requiere toracotomía. La toracotomía inmediata está indicada en: heridas penetrantes con la sospecha o constatación de lesiones viscerales graves y en pacientes con hemorragia pleural persistente. Las complicaciones más frecuentes de un hemotórax son la retención de coágulos en la cavidad pleural, el empiema pleural, el derrame pleural persistente y el fibrotórax. Hemotorax iatrogénico; cuando un paciente ingresado presenta un derrame pleural hemático, se debe considerar el origen yatrogénico. La causa más frecuente de es la perforación de una vena central o arteria por la inserción percutánea de un catéter, tras una punción o biopsia pleural, punción pulmonar percutánea, manejo endoscópico para tratar las varices esofágicas. Hemotórax no traumático: son poco frecuentes. La causa más común son las metástasis pleurales, mientras que la segunda causa más frecuente es la complicación del tratamiento anticoagulante por embolia pulmonar o patología cardiaca. El hemotórax espontáneo puede ocurrir como resultado de la ruptura anormal de un vaso sanguíneo intratorácico, como un aneurisma de aorta, aneurisma de arteria pulmonar, un ductus arterioso o una coartación de aorta. En algunos pacientes la causa permanece desconocida a pesar de la toracotomía exploradora. CONTUCION CARDIACA: Es la más frecuente de las lesiones que resultan de traumatismos cardíacos contusos. Su intensidad es variable y raramente es fatal. El mecanismo más postulado para su producción es la desaceleración, donde el corazón golpea contra el esternón siendo comprimido por la columna. El diagnóstico de esta lesión es demorado o confundido por otras lesiones más graves y agudas que desvían la atención. Debe plantearse ante un trauma de pared torácica anterior con o sin fracturas costales o de esternón, shock sin taponamiento, insuficiencia cardíaca, arritmias y frotes pericárdicos a la auscultación. Diagnóstico: ECG; puede estar alterado, principalmente al ingreso (extrasístoles ventriculares, extrasístoles auriculares, fibrilación auricular, bloqueo de rama, alteraciones del ST). Tiene sensibilidad de 100% especificidad de 47%. Enzimas Cardíacas (CK-MB, troponina I): se elevan igual que en un IAM y su toma debe de ser seriada. Valores de troponina I < de 1.05 mcg/L entre el ingreso y las 6 primeras horas descartan CM en un paciente asintomático, mientras que valores > de 1.05 mcg/L son indicadores de continuar el estudio y la observación. Si un paciente con sospecha de CM tiene un ECG y una troponina normal a las 8 horas de observación se puede descartar prácticamente con certeza una CM. ECO: se observan alteraciones del movimiento de las paredes comprometidas, aquinesia, disquinesia e hipoquinesia. Se sugiere realizar ecocardiografía en los pacientes inestables, en los que presentan arritmias o en los que irán a cirugía. Manejo: Debido al riesgo de que aparezcan alteraciones del ritmo cardíaco, los pacientes deben estar monitorizados continuamente en una UCI o área controlada durante las primeras 24 horas, si en las primeras 24 horas no se pesquisa complicación la evolución suele ser favorable. Los pacientes con contusiones documentadas y compromiso hemodinámico, que requieren cirugía con anestesia general para tratar sus lesiones

CURSO ENARM CMN SIGLO XXI TEL: 36246001

Pharmed Solutions Institute

PÁGINA 239

MANUAL DE TRABAJO DEL CURSO ENARM CMN SIGLO XXI asociadas, deben monitorizarse perioperatoriamente con técnicas hemodinámicas invasivas (línea-A, catéter de Swan-Ganz). En resumen debe tratarse en forma similar al IAM, dejando en claro que este cuadro no aumentaría la mortalidad perioperatoria en el caso de ser necesaria la cirugía de urgencia. CASO CLINICO Paciente de 68 años de edad, fumador, que acudió por disnea progresiva hasta hacerse a mínimos esfuerzos de 2 semanas de evolución. No refería traumatismo torácico. Radiológicamente, se diagnosticó un neumotórax izquierdo. Se insertó un tubo de drenaje torácico, se reexpandió completamente el pulmón y mejoró así la clínica del paciente. Tres horas más tarde tuvo un deterioro clínico consistente en dificultad respiratoria grave. Se auscultaban estertores húmedos de gruesa burbuja en el hemitórax izquierdo. Una nueva RX de tórax mostró imágenes compatibles con edema pulmonar unilateral. PREGUNTA Cual es la conducta mas apropiada en el estado actual? RESPUESTA a.- Furosemide y nitroglicerina. b.- Sindenafil. c.- Ventilacion mecánica. d.- Aminas vasoactivas. CASO CLINICO Varón de 30 años de edad. Refería tos seca con una data de un mes, sudoración nocturna, dificultad respiratoria progresiva y baja de peso estimada en 10 kg. No refería fiebre y una semana antes de ser internado se le agregó diarrea líquida. Antecedente de promiscuidad homosexual y gonorrea 8 años antes. Ingresó vigil, orientado, temperatura de 37 ºC, presión arterial de 100/65, pulso de 108/minuto, taquipneico con frecuencia respiratoria de 50/minuto. EF: adenopatías pequeñas, móviles, no adheridas a planos profundos e indoloras, tanto cervicales laterales y posteriores así como axilares e inguinales; boca con placas blanquecinas que comprometían la lengua, cara interna de mejillas y faringe; pulmones con estertores finos en ambas bases. El examen cardiovascular y abdominal carecía de hallazgos patológicos. En la exploración de laboratorio tenía: LDH de 833 U/L; hematocrito 44%; leucocitos 6.900 x mm3 (segmentados 87%, linfoncitos 8%). Radiografía de tórax con infiltrados bilaterales y datos compatible con neumotórax. PREGUNTA Cual es el agente mas probable relacionado con el desarrollo de neumotórax considerando los antecedentes? RESPUESTA a.- M. avium. b.- P. jiroveci. c.- C. albicans. d.- S. pneumonie. CASO CLINICO Paciente de 53 años que es traído a Urgencias tras sufrir un accidente de tráfico, al parecer es el que conducía. Refiere que se ha salido de la carretera y recuerda todo lo que ha pasado (no ha perdido el conocimiento). Exploración física: Llega consciente y orientado, mueve todos los miembros y las pupilas son normales. Refiere intenso dolor torácico, especialmente en hemitórax izquierdo. Respira espontáneamente con gran trabajo respiratorio, taquipnea y sudoración. La auscultación pulmonar muestra hipofonesis en la mitad inferior del hemitórax izquierdo y la percusión es mate. El resto de campos pulmonares es normal. No tiene dolor a otros niveles y la exploración cardiaca, de abdomen y de miembros es normal. PREGUNTA Cual es el diagnostico mas probable? RESPUESTA a.- Neumotorax. b.- Hemotorax. c.- Hemomediastino. d.- Neumomediastino. CASO CLINICO Se trata de masculino de 27 años de edad el cual cuenta con antecedentes de importancia tales como padre con enfisema pulmonar y madre cardiópata, presenta tabaquismo positivo desde los 17 años, actualmente consume marihuana y ha probado el crack, refiere que hace 4 horas inicia súbitamente con dolor en el torax del lado derecho, asi como dificultad para respirar, niega tos, fiebre o malestar previo, refiere que se encontraba trabajando en su taller mecanico, a la exploración física se observa ansioso, se encuentra afebril, no se observa edema periférico, su frecuencia respiratoria es de 25 rpm, FC 84 lpm, TA 120/90 mmHg, saturación de oxigeno del 93 %, se realiza tele de torax donde se observa neumotórax del 50 % del pulmon derecho. PREGUNTA Cuál es la conducta a seguir.

CURSO ENARM CMN SIGLO XXI TEL: 36246001

Pharmed Solutions Institute

PÁGINA 240

MANUAL DE TRABAJO DEL CURSO ENARM CMN SIGLO XXI RESPUESTA a.- Sello pleural. b.- Aspiracion con aguja. c.- Observacion. d.- Toracoscopia. TRAUMA TORACICO CERRADO Y ABIERTO: CIENCIAS BASICAS: Es cualquier agresión o trauma sobre las paredes del tórax que producirá un daño en las estructuras sólidas y partes blandas comprendidas en la caja torácica. Los traumatismos torácicos pueden ser cerrados (contusos) o abiertos (penetrantes). Traumatismo abierto: se denomina a lesión que rompe la integridad del tejido (atraviesa pleura parietal). Traumatismo cerrado: resulta por aplicación de energía que provoca lesión sobre los tejidos sin dañar su integridad. SALUD PUBLICA: En México los traumas cerrados son mayoritariamente por accidente de tránsito, los penetrantes TRAUMA TORACICO CERRADO TRAUMA TORACICO ABIERTO Asociada a compresión y aceleraciónAsociada a heridas por arma blanca son por arma blanca, aunque han aumentado las heridas por armas desaceleración y arma de fuego de fuego. 8 de cada 100.000 son letales. Principales causas de Hay fracturas costales múltiples Puede o no haber fracturas costales traumatismo torácico asociadas: Accidentes de tránsito (43%), Puede haber hemo o neumotórax Hemo o neumotórax inmediato. Suicidios (29%), Homicidios (22%.). Nos da gravedad en un Trauma tardío (>24hrs del trauma) torácico: Impactos de alta energía: caída mayor a 6 metros. El trauma de vía aérea superior se El trauma de vía aérea cursa con Impactos de alta velocidad. Pasajeros despedidos del vehículo. manifiesta como estenosis gran escape aéreo Tratamiento quirúrgico: requerido en Tratamiento quirúrgico: requeridos Atropello. Lesión penetrante de cabeza, cuello, tórax, abdomen o menos del 10% de lesionados entre 15-30% de lesionados. región inguinal. Dos o más fracturas proximales de huesos largos. Quemaduras mayores al 15% de SC o que afecten cara o vías aéreas. Tórax inestable. TIPOS DE TRAUMA TORÁCICO: Lesiones torácicas letales: 1.-OBSTRUCCIÓN DE LA VÍA AÉREA; se puede producir por cuerpos extraños, secreciones, sangre, el manejo puede ser invasivo o no invasivo, otra clasificación es “vía aérea quir rgica” y “no quir rgica”. Consisten en el manejo manual de la vía aérea, intubación oro o nasotraqueal, cricotiroidotomia por punción o quirúrgica y la traqueotomía. 2.- NEUMOTÓRAX A TENSIÓN; es el escape de aire hacia la cavidad pleural, mas común neumotórax espontáneos, el mediastino y la tráquea se desplazan hacia al lado opuesto, comprometiendo la posibilidad de respuesta ventilatoria por parte del pulmón sano, y afectando el retorno venos. Clínicamente se manifiesta por dificultad respiratoria, taquicardia, hipotensión, desviación de la tráquea, ausencia unilateral de MV, timpanismo del pulmón afectado, ingurgitación yugular y cianosis tardía. El tratamiento inicial consiste en insertar una aguja, en 2° EIC en LMC del hemitórax afectado, siempre por el borde superior de la 3° costilla. El definitivo es la inserción de un tubo de tórax en el 5° EIC anterior a la LAM, siempre por el reborde costal superior de la 6° costilla, en el hemitórax afectado. 3.-NEUMOTÓRAX ABIERTO; Al producirse una herida en la pared torácica el aire penetrará preferentemente por la herida al igualarse las presiones intratorácica y ambiental y ofrecer menor resistencia al paso del aire por la herida. El tratamiento inicial será cubrir la herida con un apósito fijado en tres puntas. El tratamiento definitivo consiste en la instalación de un tubo de tórax, distante de la lesión, y el cierre de la herida será quirúrgico. Complicaciones: Inadecuada conexión drenaje – aspirador. Inadecuada colocación del drenaje. Oclusión bronquial. (Cuerpo extraño, coágulo, rotura). Roturas traqueobronquiales. Grandes laceraciones pulmonares. Severa disminución de la distensibilidad pulmonar. 4.HEMOTÓRAX MASIVO: Es el resultado de la acumulación de sangre en la cavidad pleural, igual o superior a 1500 ml. La principal causa: es la lesión de vasos hiliares y mediastinicos generalmente por heridas penetrantes. Clínicamente encontrará un paciente en shock, con colapso de los vasos del cuello por hipovolemia o con ingurgitación de estos por efecto mecánico de las cavidades. El tratamiento será simultáneamente con reposición de volumen de forma agresiva (cristaloides, coloides y sangre), y descompresión del hemitórax lesionado con un tubo de tórax único N° 28 – 32 f. Se hará toracotomía sí: Deterioro hemodinámico sin otra justificación. Persiste un drenaje superior a 1500 ml en las primeras 12 – 24 horas. Drenaje superior a 200 ml/hora en 4 horas. Persistencia de la ocupación torácica (hemotórax coagulado). 5.- TÓRAX INESTABLE: Clínicamente puede no ser detectado en primera instancia por la hipoventilación reactiva al dolor, y por los movimientos del tórax. El tratamiento se basa fundamentalmente en una buena ventilación, eventualmente mecánica, oxigenación, y tratamiento para el dolor. Si el paciente no está en shock la infusión de fluidos debe ser cuidadosa para evitar la sobrehidratación y el consiguiente edema pulmonar. Un adecuado y controlado balance hídrico. Cuando coexisten fracturas costales múltiples en varias costillas consecutivas se produce una inestabilidad de la pared con movimiento paradojico y alteración de la mecánica respiratoria, con la consiguiente hipoxia. La gravedad de la lesión es directamente proporcional al grado de alteración del parénquima pulmonar en combinación con el daño de la pared. 6.- CONTUCION PULONAR: Lesión del parénquima pulmonar que causa hemorragia y edema localizado, producto de traumas en los que hay rápida compresión y descompresión del tórax. Se observa falla respiratoria tardía, lenta, progresiva y sutil, con o sin tórax inestable. 7.- CONTUCION MIOCÁRDICA: Difícil de diagnosticar, se sospecha por alteraciones al ECG (arritmias, extrasístoles mono o bifocales, taquicardia sinusal inexplicable, FA, bloqueo de rama, o claramente un infarto), Eco cardiografía bidimensional e historia compatible. El tratamiento tratara la manifestación clínica o la arritmia específica. 8.- RUPTURA DE AORTA: Producto de traumatismos cerrados, por laceración o arrancamiento de los puntos de fijación de la Aorta. Signos clínicos alertan el diagnóstico: Mediastino ensanchado en Rx de Tx. Fx de 1ª y 2ª costillas. Desviación y elevación del bronquio principal, de la tráquea y el esófago hacia la derecha. Depresión del bronquio principal izquierdo. Opacidad pleural apical. Tratamiento: quirúrgico (reparación o implante). 9.- RUPTURA DIAFRAGMÁTICA: Presente con más frecuencia en el lado izquierdo ya que se carece de la protección del hígado. Se sospecha el diagnóstico por la presencia de intestino, estomago en el hemitórax izquierdo. Los traumatismos penetrantes por arma blanca o de fuego a veces pasan inadvertidos y solo se detectan años después cuando aparece la hernia diafragmática. 10.- LESIÓN TRAQUEOBRONQUIAL: Laringe Diagnóstico: ronquera, enfisema subcutáneo y crepitación palpable de fractura. Manejo, en caso de vía aérea obstruida: instalación de IOT o Traqueotomía. Tráquea: las lesiones penetrantes son más obvias que las provocadas por trauma. Se asocia a lesión de esófago y grandes vasos. Diagnostico a través de broncoscopía. Bronquios: La lesión de un bronquio mayor es rara y mortal, y ocurren a 2-3 cm. de la carina. Diagnóstico: hemoptisis y enfisema subcutáneo, y se sospecha en neumotórax a tensión con gran escape de aire y es confirmado por broncoscopía. “Una vez confirmado el diagnostico de lesión traqueobronquial el tratamiento es la reparación quir rgica”. 11.- RUPTURA ESOFÁGICA: Se sospechara cuando exista neumo o hemotórax a izquierda sin fracturas costales, trauma directo al esternón o epigastrio con dolor y shock no explicado, además puede haber salida de partículas de contenido digestivo por

CURSO ENARM CMN SIGLO XXI TEL: 36246001

Pharmed Solutions Institute

PÁGINA 241

MANUAL DE TRABAJO DEL CURSO ENARM CMN SIGLO XXI tubo de tórax. Tratamiento: La conducta será la reparación directa, esofagostomía cervical de escape y yeyunostomía de alimentación. 12.- TAPONAMIENTO CARDIACO: Producto de una herida penetrante, en su gran mayoría, pero también puede aparecer por lesiones de los vasos pericardicos o traumatismo cardíaco en un traumatismo cerrado. Desde el punto de vista clínico se manifiesta por la “tríada de ec ”, que consiste en el hallazgo de: aumento de la presión venosa central, disminución de la presión arterial, apagamiento de los ruidos cardiacos. Ingurgitación yugular con la inspiración en un paciente ventilando espontáneamente es signo inequívoco de taponamiento cardiaco (signo de Kussmaul). En el tratamiento, la pericardiocentesis por vía subxifoidea es de elección en el prehospitalario, para descompresionar el pericardio, basta extraer 15 – 20 ml, pero es una medida temporal, actualmente se postula que en él SU y Hospital no deben hacerse pericardiocentesis a menos que sea para dar tiempo a la preparación del pabellón. INSERCIÓN DE PLEUROTOMÍA; cuidados postoperatorios: El paciente deberá permanecer en el hospital hasta que el tubo torácico sea retirado. Debe quedar conectado a una trampa de agua donde el pivote del sello de agua debe quedar sumergido a 2cm, y oscilar cuando esté conectado al paciente. Mientras el tubo está colocado en el tórax del paciente, los enfermeros verifican con cuidado que no haya escapes de aire, dificultades o problemas al respirar o necesidad de administrar oxígeno adicional al paciente. El paciente tosa y respire profundo para facilitar que los pulmones se expandan de nuevo, ayudar con el drenaje y prevenir que los fluidos normales se alojen en los pulmones. ATENCIÓN Y MANEJO: Se realizara la evaluación y el tratamiento de forma paralela y simultánea al momento en que se avanza en el examen del paciente, esto es ir resolviendo los problemas vitales en la medida que se va encontrando, y siempre en este orden y no otro. A. Vía aérea y control de columna cervical. Permeabilidad, estabilidad y seguridad de la vía aérea (eventual IOT), aspiración de secreciones, fijación de columna cervical firme y segura, cricotiroidotomía por aguja o quirúrgica. B. Respiración: Oxigenación, movimientos respiratorios, ventilación asistida, oclusión de heridas torácicas abiertas, toracocentesis y drenajes torácicos. C. Circulación y control de hemorragias. Compresión directa de los sitios de hemorragia, evaluación de los pulsos, masaje cardiaco externo, instalación de dos vías venosas periféricas proximales de grueso calibre para alto flujo, reposición de volumen y uso de fármacos endovenosos, analgesia y sedación, monitorización cardiaca. D. Déficit neurológicos. E. Exposición corporal y abrigo. CASO CLINICO

TRAUMA ABDOMINAL ABIERTO Y CERRADO: CIENCIAS BASICAS: Lesión orgánica producida por la suma de la acción de un agente externo junto a las reacciones locales y generales que provoca el organismo ante dicha agresión. Todo paciente puede presentar lesiones en múltiples órganos abdominales y, por tanto, debe ser considerado como un paciente con traumatismo grave, desde el momento del ingreso en la unidad de urgencias. Un tercio de los pacientes que requieren una exploración abdominal urgente tienen un examen físico inicial anodino, tener en cuenta que puede tener un comportamiento impredecible y desestabilizarse en el momento más inesperado. Importante conocer el mecanismo lesional con el fin de anticipar las lesiones esperables. ANATOMÍA: Abdomen anterior: se define como el área localizada entre una línea superior que cruza por las mamilas, los ligamentos inguinales y la sínfisis del pubis como la línea inferior, y las líneas axilares anteriores lateralmente. Flanco: Área entre las líneas axilares anteriores y posteriores y desde el cuarto espacio intercostal hasta la cresta ilíaca. El espesor de la musculatura de la pared abdominal a este nivel, más que las capas aponeuróticas más delgadas de la pared anterior, actúa como una barrera parcial a las heridas penetrantes, particularmente por arma blanca. Espalda: localizada atrás de las líneas axilares posteriores, desde la punta de la escápula hasta las crestas ilíacas, el espesor de la espalda y los músculos paravertebrales actúan como una barrera parcial a las heridas penetrantes. Anatomía interna del abdomen: Cavidad peritoneal: cubierto por la parte baja de la parrilla costal, el abdomen superior incluye el diafragma, hígado, bazo, estómago y colon transverso. Debido a que el diafragma en una espiración total se eleva hasta el 4º espacio intercostal, las fracturas de costillas inferiores o heridas penetrantes en la misma área pueden involucrar estas vísceras abdominales. El abdomen inferior contiene el intestino delgado y el colon ascendente, descendente y sigmoides. Cavidad pélvica: rodeada por los huesos pélvicos, corresponde a la parte baja del espacio retroperitoneal y contiene el recto, la vejiga, los vasos ilíacos, y en la mujer los genitales internos. Espacio retroperitoneal: contiene la aorta abdominal, la vena cava inferior, la mayor parte del duodeno, el páncreas, los riñones, los uréteres, así como segmentos del colon ascendente y descendente. Las lesiones en las vísceras retroperitoneales son muy difíciles de reconocer porque el área es de difícil acceso al examen físico y sus lesiones no son detectadas por medio del lavado peritoneal diagnóstico, y difícilmente valoradas por la ecografía, además de ser de difícil exploración física. SALUD PUBLICA: Traumatismos de los más frecuentes, estimándose en 1 por cada 10 ingresos por traumatismo en los servicios de urgencias. Las principales causas de muerte en los pacientes con traumatismo abdominal son: 1. Por lesión de algún vaso principal, como vena cava, aorta, vena porta o alguna de sus ramas, o arterias mesentéricas. Las lesiones destructivas de órganos macizos, como hígado, bazo o riñón, o sus asociaciones, pueden originar una gran hemorragia interna. 2. Sepsis: la perforación o rotura de asas intestinales o estómago, supone la diseminación en la cavidad peritoneal de comida apenas digerida o heces, con el consiguiente peligro de sepsis. Los trastornos de vascularización de un asa intestinal por contusión de la pared intestinal o de su meso pueden manifestarse tardíamente como necrosis puntiforme parietal y contaminación peritoneal con sepsis grave. CLASIFICACIÓN: Abiertos (penetrantes y no penetrantes), presentan solución de continuidad en la piel. Cerrados; la piel no tiene solución de continuidad. PATOGENIA: Las principales causas de abiertos son las heridas por arma blanca (lesiones intrabdominales de 20-30%) y arma de fuego cuya frecuencia es creciente. La principal causa de cerrados son los accidentes de tráfico. Otras causas son los accidentes de trabajo, accidentes domésticos, accidentes deportivos, siendo estos mucho más frecuentes que los abiertos. TRAUMATISMO ABDOMINAL ABIERTO: Las heridas por arma blanca y de fuego de baja velocidad (< 600 m/seg) causan daño al tejido por laceración o corte. Ceden muy poca energía y el daño se localiza en la zona perilesional, afectando habitualmente órganos adyacentes entre sí, siguiendo la trayectoria de, objeto que penetra. Las heridas por proyectiles de alta velocidad (> 600 m/seg) transfieren gran energía cinética a las vísceras abdominales, teniendo un efecto adicional de cavitación temporal y además causan lesiones adicionales en su desviación y fragmentación, por lo que es impredecible las lesiones esperadas. TRAUMATISMO ABDOMINAL CERRADO: Impacto directo; la energía cinética a los órganos adyacentes a la pared abdominal, puede provocar lesión. Desaceleración: mientras el cuerpo es detenido bruscamente los órganos intra abdominales animados aún por la energía cinética tienden a continuar

CURSO ENARM CMN SIGLO XXI TEL: 36246001

Pharmed Solutions Institute

PÁGINA 242

MANUAL DE TRABAJO DEL CURSO ENARM CMN SIGLO XXI en movimiento produciéndose una sacudida, especialmente acusada a nivel de los puntos de anclaje, vasos y mesenterio que sufren desgarros parciales o totales. Compresión o aplastamiento: entre dos estructuras rígidas, estas fuerzas deforman los órganos sólidos o huecos y pueden causar su ruptura o estallido de estos. Este es el mecanismo típico de lesión del duodeno, en un accidente de automóvil con impacto frontal, donde aquel es comprimido entre el volante y la columna vertebral. VALORACIÓN INICIAL: El objetivo en evaluación primaria es evidenciar o descartar lesiones de riesgo vital e instaurar las medidas necesarias de soporte vital para preservar la vida del paciente. El paciente con traumatismo abdominal debe ser considerado como traumatismo grave o potencialmente grave y por lo tanto, el manejo de estos pacientes debe seguir las recomendaciones del ABC: A. Asegurar la permeabilidad de la vía aérea, con control cervical. B. Asegurar una correcta ventilación/oxigenación. Descartar neumotórax a tensión, abierto, hemotórax masivo. Valorar la necesidad de soporte ventilatorio. Si no es necesario administrar oxígeno a alto flujo con mascarilla (10-15 l/min). C. Control de la circulación. Detener la hemorragia externa. Identificación y tratamiento del shock. Identificación de hemorragia interna. Monitorización ECG estable. D. Breve valoración neurológica. E. Desnudar completamente al paciente, controlando el ambiente y previniendo la hipotermia. DIAGNOSTICO: Valoración clínica del estado de shock: aumento de la frecuencia del pulso, pulso débil y filiforme, piel pálida, fría y sudorosa, disminución de la presión del pulso, retardo en el relleno capilar, alteración de la conciencia, taquipnea, hipotensión y oligo anuria., dado que la hemorragia intraabdominal es la causa más frecuente de shock hipovolémico en estos pacientes. Inicialmente, se asume que el estado de shock es el resultado de la pérdida aguda de sangre y se la trata con una infusión rápida de volumen: un bolo inicial de 1-2 litros para un adulto de SF al 0’9 % o de solución de Ringer lactato. La restitución por medio de catéteres intravenosos periféricos de calibre grueso (14G – 16G), dada su mayor rapidez de canalización. El shock refractario a la infusión rápida de cristaloides sugiere sangrado activo y requiere de una laparotomía urgente. Primeras medidas: Colocar sonda gástrica: cuyo objetivo es aliviar la dilatación gástrica aguda, descomprimir y reducir el riesgo de broncoaspiración. Si existen graves fracturas faciales o la sospecha de una fractura de la base del cráneo, la sonda debe introducirse por la boca para evitar el riesgo del paso del tubo hacia el cerebro a través de la lámina cribiforme. Canalización de dos vías venosas periféricas con catéter de gran calibre. Sonda vesical para aliviar la retención de orina y descomprimir la vejiga, comprobar la presencia de diuresis, su flujo horario, descartando previamente la presencia de lesión uretral. Se deben sacar muestras de sangre y realizar una determinación de BH, tiempos de coagulación, gasometría, amilasa, niveles de alcohol, grupo sanguíneo y pruebas cruzadas. Evaluación Secundaria: Historia clínica, a partir del propio paciente, de sus familiares y de los profesionales que han llevado a cabo la atención prehospitalaria. Mecanismo de producción del traumatismo: a) En traumatismos cerrados es importante el tipo de impacto, daño del vehículo, uso de sistemas de seguridad, el estado de otras víctimas. b) Para las heridas penetrantes, puede ser útil una descripción del arma y de la cantidad de sangre perdida en el lugar del hecho. Tiempo de evolución desde el trauma hasta la recepción del paciente. APP: alergias, patologías previas, medicación habitual, cirugía previa, ingesta de drogas. Maniobras realizadas por los profesionales de la atención prehospitalaria: volumen infundido, vías canalizadas, necesidad de resucitación cardiopulmonar. Exploración: Inspección; observar el tórax, abdomen, espalda, pelvis, periné, espalada. Hay que observar las huellas en la piel y pared de los puntos de impacto del agente agresor. Auscultación; confirmar la presencia o ausencia de ruidos intestinales. La presencia de sangre libre intra peritoneal o contenido gastrointestinal pueden producir un íleo que produce una ausencia de ruidos intestinales. Las lesiones en estructuras adyacentes, por ejemplo, costillas, columna o pelvis, también pueden producir íleo. Percusión; detectar matidez (presencia de líquidos) en caso de hemoperitoneo; timpanismo (presencia de aire) si hay dilatación gástrica o desaparición de la matidez hepática por neumoperitoneo. Palpación; debe repetirse periódicamente, por lo que debe ser realizado por la misma persona para poder evaluar las diferencias que se originen. Primero, debe dirigirse al plano parietal buscando la presencia de hematomas, o contusiones musculares. Luego debe investigar la presencia de contractura abdominal refleja, que es un signo fiable de irritación peritoneal, al igual que el signo del rebote positivo. Finalmente, hay que realizar una palpación más profunda buscando la presencia de puntos o zonas dolorosa cuya topografía nos oriente a relacionarlas con los posibles órganos lesionados. Evaluación estabilidad pélvica; La exploración del anillo pelviano debe realizarse mediante una cuidadosa compresión lateral y antero posterior, siendo dolorosa cuando hay fractura pélvica. Una fractura de pelvis puede ser causa de shock hipovolémico, en ocasiones muy severo. Produce hematoma perineal y genital a las 24-48 horas del traumatismo y puede acompañarse de hematoma retroperitoneal y ausencia de hemoperitoneo. Examen del periné y genitales; lesiones externas y la presencia de signos de lesión uretral como sangre en el meato, hematoma escrotal o desplazamiento hacia arriba de la próstata. La laceración de la vagina puede ocurrir en heridas penetrantes o por fragmentos óseos de una fractura pélvica. PRUEBAS COMPLEMENTARIAS: Analítica: bioquímica, hemograma, tiempo de coagulación, pruebas cruzadas, niveles de alcoholemia y análisis de orina. ECG y monitorización de constantes vitales. Radiografía de abdomen. Radiografía de tórax: es importante para descartar la presencia de hemotórax, neumotórax o fracturas costales. Radiografía de pelvis. Exámenes complementarios: Lavado peritoneal diagnóstico (LPD); procedimiento invasivo que puede ser realizado de forma rápida presenta una sensibilidad del 68% y una especificidad del 83%. La indicación principal del LPD son Hallazgos abdominales equívocos. Exploración física no realizable por traumatismo raquídeo concomitante o alteración de la conciencia (traumatismo cráneo encefálico o tóxico). Imposibilidad de reevaluación continúa. Hipotensión inexplicable. Pérdida progresiva de sangre (descenso progresivo del hematocrito). Puede realizarse mediante un método abierto o cerrado. Un LPD negativo no excluye la presencia de lesiones retroperitoneales o desgarros diafragmáticos. El ultrasonido puede detectar presencia de hemoperitoneo, es un medio rápido, no invasivo y seguro en el diagnóstico de lesiones intra abdominales (cerrada o penetrante) y puede ser repetido frecuentemente. Es más sensible que el lavado peritoneal diagnóstico para la determinación de lesiones de vísceras macizas, aunque no lo es tanto como la TAC. Su indicación es absoluta en casos de embarazo, cicatrices abdominales por cirugías previas y alteración de la coagulación. La TAC requiere el transporte del paciente a la sala de rayos X, administración oral e intravenosa de contraste. Consume tiempo y es utilizado únicamente en pacientes hemodinámicamente estables en los que no existe la indicación inmediata de laparotomía. La TAC proporciona información relativa a la lesión específica de un órgano en particular y también puede diagnosticar lesiones en el retroperitoneo u órganos pélvicos que son difíciles de evaluar en la exploración física o en el LPD. Laparoscopia la utilidad de la laparoscopia efectuada bajo anestesia local para identificar lesiones diafragmáticas y cuantificar la cantidad de sangre intraperitoneal. MANEJO DEL PACIENTE TRAS LA VALORACIÓN INICIAL: 1. Traumatismo Abdominal Cerrado, Hemodinámicamente inestable: (imposibilidad de mantener una TA sistólica por encima de 90 mm Hg, frecuencia cardiaca inferior a 100 lpm o diuresis de 50 ml/h adulto) Si el paciente presenta signos abdominales patológicos (distensión abdominal, peritonismo, neumoperitoneo en la radiografía simple), entonces la indicación de laparotomía debe ser inmediata. Si los signos abdominales son dudosos y el paciente

CURSO ENARM CMN SIGLO XXI TEL: 36246001

Pharmed Solutions Institute

PÁGINA 243

MANUAL DE TRABAJO DEL CURSO ENARM CMN SIGLO XXI presenta un traumatismo craneoencefálico o espinal severo, alteraciones de la conciencia por toxicidad, traumatismos toracoabdominales, debemos llevar a cabo un estudio rápido del abdomen que nos ayude a descartar la presencia de patología abdominal, fundamentalmente líquido libre. Para ello disponemos de dos pruebas que no son excluyentes: Ecografía abdominal. Lavado peritoneal diagnóstico. Otras pruebas radiológicas que precisan mayor infraestructura y tiempo, como la TAC, no son posibles en el paciente inestable. Hemodinámicamente estable. Se deben tener en cuenta las siguientes premisas a la hora de manejar un paciente con un traumatismo abdominal cerrado: Las vísceras macizas se lesionan con más frecuencia que las huecas. Traumatismo Abdominal Abierto; Arma Blanca. - Si el paciente está inestable o presenta signos de irritación peritoneal, debe ser sometido a una laparotomía urgente. - Si está estable hemodinámicamente y no presenta signos de irritación peritoneal, la primera maniobra que se debe realizar es la exploración del orificio de entrada del arma, comprobando si la herida es penetrante o no. Arma de Fuego; Puesto que la trayectoria de una bala es difícil de predecir y dado que el 80-90% de los traumatismos por arma de fuego se asociaran a una o más lesión visceral, el tratamiento de estos pacientes será quirúrgico, realizándoseles una laparotomía urgente. Recomendaciones basadas en evidencia, Recomendaciones NIVEL I: Laparotomía exploratoria está indicada para pacientes con LPD (+) TAC está recomendado para pacientes hemodinámicamente estables con un EF equívoco (lesión neurológica, lesiones extrabdominales). TAC es la modalidad diagnóstica de elección para manejo no operatorio de lesiones de vísceras sólidas. En pacientes hemodinámicamente estables, LPD y TAC son modalidades diagnósticas complementarias. CASO CLINICO

INSUFICIENCIA RENAL AGUDA (IRA) Y GLOMERULOPATIAS AGUDAS: CIENCIAS BASICAS: Se denomina IRA a la reducción brusca, en horas o días, de la función renal; se produce una disminución del filtrado glomerular y un acúmulo de productos nitrogenados séricos (urea y creatinina en sangre) con incapacidad para regular la homeostasis (equilibrio ácido-base e hidroelectrolítico). Aunque se suele asociar a una disminución de la diuresis (IRA oligúrica), hasta un 40% de los casos no cursan con oliguria e incluso puede existir poliuria (IRA no oligúrica). La IRA suele presentarse como una complicación de enfermedades graves previas, apareciendo entre el 5 al 30% de enfermos hospitalizados. CLASIFICACION: IRA PRERRENAL O FUNCIONAL (60-70%); Inadecuada perfusión renal que compromete el filtrado glomerular; sería, por tanto, una respuesta fisiológica a la hipoperfusión renal pero el parénquima renal está íntegro. Es reversible si se actúa sobre la causa desencadenante de manera precoz. Causas de IRA prerrenal son: hipovolemia (hemorragias, perdidas GI, perdidas renales, secuestro de líquidos al espacio extravascular), disminución del gasto cardiaco (ICC, TEP, hipertensión pulmonar), vasodilatación periférica (sepsis, anafilaxia, antihipertensivos, anestesia), vasoconstricción renal (hipercalcemia, norepinefrina, ciclosporina, anfotericina B, síndrome hepatorrenal), alteraciones de las respuestas autorreguladoras renales (inhibidores de las PG, como los AINES, y/o IECAS). IRA RENAL, PARENQUIMATOSA O INTRÍNSECA (25%); Daño en las estructuras anatómicas; se clasifica según la estructura primariamente dañada: glomérulos, túbulos, intersticio o vasos renales. Se dividen las causas de IRA intrínseca en 4 apartados: 1.Necrosis Tubular aguda (NTA): Causa más frecuente de IRA intrínseca (70%), afecta a las células tubulares renales, desde lesiones mínimas hasta necrosis cortical, la lesión de los túbulos renales puede ser por mecanismos isquémicos (cursa con oliguria) principalmente o tóxicos (aminoglucosidos, cefalosporinas, contrastes radiológicos, AINES, anestésicos, toxinas endógenas), puede cursar con diuresis conservada, incluso aumentada. 2. Lesión glomerular: glomerulonefritis agudas y rápidamente progresivas, hipertensión maligna, vasculitis, síndrome hemolítico-urémico, purpura trombocitopénica trombótica, toxemia del embarazo, esclerodermia. 3. Lesión tubulointersticial: reacciones alérgicas a fármacos (antibióticos, AINES, diuréticos), infecciones (legionella, leptospira, CMV, candidas). 4. Lesión de grandes vasos: obstrucción de arterias renales (placa ateroesclerótica, trombosis, embolia), obstrucción de venas renales (trombosis, compresión). IRA POSRENAL U OBSTRUCTIVA (5%): Lesiones que produzcan un obstáculo en la vía urinaria que impida la salida de la orina formada, provocando un aumento de presión que se transmite retrógradamente, comprometiendo el filtrado glomerular. Pueden ser lesiones extrarrenales de uréteres, pelvis (litiasis, tumores, fibrosis), vejiga (litiasis, coágulos, tumores, prostatismo, vejiga neurógena), uretra (estenosis, fimosis) o también lesiones intrarrenales (depósito de cristales, coágulos, cilindros). Para que estas causas produzcan una IRA es necesario que la obstrucción sea grave, prolongada y que afecte a tracto urinario distal (meato uretral externo, cuello de la vejiga) o bien a los uréteres de manera bilateral o unilateral en paciente con un único riñón funcionante. DIAGNÓSTICO: Clínica de las diferentes formas de IRA dependerá de las causas desencadenantes. Así, en la forma prerrenal destacaran las manifestaciones de reducción verdadera de volumen (sed, hipotensión, taquicardia, disminución de la presión venosa yugular, disminución de peso, sequedad de piel y mucosas) o de reducción “efectiva” de volumen (en este caso la exploración revelará signos de hepatopatía crónica, insuficiencia cardíaca avanzada, sepsis). Es importante destacar que en estos casos la IRA desaparece rápidamente tras reestablecer la perfusión renal. En el caso de la forma renal o intrínseca hay que investigar la presencia de isquemia renal prolongada (shock hipovolémico, shock séptico, cirugía mayor). En estos casos existe oliguria o incluso anuria (diuresis diaria < 100 ml). La probabilidad de que estemos ante un cuadro de NTA aumenta aún más si la IRA persiste a pesar del reestablecimiento de la perfusión renal. La posibilidad de IRA nefrotóxica requiere el estudio de los medicamentos que ha recibido recientemente el paciente, exposición a contrastes radiológicos (sobre todo si se han realizado en pacientes de riesgo: ancianos, diabetes mellitus, deshidratación previa, mieloma múltiple). Las toxinas también pueden tener un origen endógeno como la mioglobina (tras una rabdomiólisis, por destrucción muscular aguda) o hemoglobina (tras hemolisis grave). Destacar que en la NTA por tóxicos la diuresis suele estar conservada. En la forma posrenal la causa más frecuente en el varón es la obstrucción del cuello de la vejiga por una enfermedad prostática (hiperplasia o carcinoma).La diuresis fluctuante es característica de la uropatía obstructiva. Laboratorios: BH: urea, creatinina, glucosa, iones. CK. La característica fundamental de la IRA es la aparición de uremia aguda de rápida aparición. A nivel práctico se considera que esto ocurre cuando la creatinina plasmática aumenta 0,5 mg/dl/día durante varios días. Si la IRA ocurre en el seno de una IRC, se considera que el aumento debe ser mayor de 1 mg/dl/día. La creatinina es más fiable que la urea para la el diagnóstico de IRA. También puede calcularse el grado de disfunción renal detectando el deterioro del aclaramiento de creatinina. Para ello, en urgencias, se puede usar esta fórmula, de aclaramiento de

CURSO ENARM CMN SIGLO XXI TEL: 36246001

Pharmed Solutions Institute

PÁGINA 244

MANUAL DE TRABAJO DEL CURSO ENARM CMN SIGLO XXI creatinina (Ccr), es una prueba aceptada como medida de filtrado glomerular. El valor normal de Ccr es de 100-120 ml/min. En el caso de IRA el Ccr calculado debe reducirse un 50%. Existirá hiperpotasemia en casos de IRA oligúrica o en estados hipercatabólicos, como sucede en la hemólisis, rabdomiolisis y en los casos de lisis tumoral. La hipopotasemía se da en las formas poliuricas. La hiponatremia es también un hallazgo frecuente. Un manejo incorrecto del paciente, con un aporte excesivo de agua en proporción a la de sodio, puede agravar aún más la hiponatremia. El aumento del ácido úrico es característico de la IRA aunque habitualmente es moderado y asintomático. Suele existir hipocalcemia, hiperfosforemia e hipermagnesemia. La severidad de estas alteraciones será paralela a la del daño renal que las ha ocasionado. Hemograma: Puede tener gran importancia en el diagnóstico diferencial entre IRA e IRC: así si aparece una anemia normocítica normocrómica, estará más en concordancia con una IRC. Gasometría arterial: El patrón ácido-base más frecuente del fracaso renal agudo es la acidosis metabólica ya que el riñón es incapaz de eliminar los ácidos fijos no volátiles. Volumen urinario: debido a las variaciones en la diuresis de las distintas formas de IRA en general no tiene gran valor diagnóstico, aunque si sirve para clasificar la IRA como oligúrica y no oligúrica Sedimento urinario: En la IRA prerrenal el sedimento no contiene células pero si cilindros hialinos. En NTA existen cilindros granulosos, pigmentados y de células epiteliales, generalmente en asociación con hematuria microscópica. Proteinuria: suele verse en la NTA, es de tipo tubular y menor de 1 gr/24 h. Sistemático de orina: el estudio de iones, urea, creatinina, osmolaridad y densidad junto al sedimento urinario son fundamentales para el diagnóstico difrencial de IRA prerrenal de NTA. ECG: Puede orientar hacia trastornos electrolíticos sobre todo la hiperpotasemia (prolongación del intervalo PR, ensanchamiento del QRS y aplanamiento de la onda T) o hipocalcemia. La Rx. simple de abdomen informa sobre la existencia de litiasis radiopaca y el tamaño y silueta renal y con la radiografía de tórax se puede valorar la existencia de sobrecarga de líquidos (edema agudo de pulmón). Ecografía abdominal: diferencial de la IRA. Se puede descartar patología obstructiva así como visualizar el tamaño renal, dato muy importante para distinguir entre IRA e IRC. TRATAMIENTO: Manejo de IRA prerrenal. Dieta rica en hidratos de carbono y con aporte de proteínas de alto valor biológico entre 0,6-0,8 gr/Kg/día. Monitorizar tensión arterial, frecuencia cardiaca y medición de la ingesta y pérdida de agua y sal. El mecanismo de control más simple es el peso diario. El sondaje vesical será necesario si se precisa la medición de diuresis horaria. Canalizar vía periférica y central y monitorizar presión venosa central (PVC), para ajustar el aporte de líquidos a una PVC entre 4-8 cmH2O. Si no existe contraindicación, se puede realizar una rehidratación rápida (en unos 30 min) con 500-1000 ml de SF, controlando la presión arterial, la PVC y vigilando la respuesta clínica y diurética. Si existe una pérdida hemática grave se usará concentrado de hematíes. Una vez corregida la volemia, el volumen urinario aumenta y se debe continuar con reposición de líquidos a ritmo de diuresis. Si hay una mala evolución en observación será necesaria la consulta con el nefrólogo que valorará la indicación de diálisis. Manejo de NTA: una serie de medidas conservadoras pueden, sino evitar la necesidad de diálisis, si al menos transformar una IRA oligúrica en otra no oligúrica lo que permitirá un mejor manejo del enfermo. Para ello, una vez adoptadas las medidas generales del apartado anterior se forzará la diuresis por medio de diuréticos y dopamina. Así se puede intentar el uso de furosemida a dosis de 20 a 40 mg cada 6 horas, según los valores de diuresis y creatinina. También pueden usarse diuréticos osmóticos como el manitol al 20%, a dosis de 80 ml cada 6-8 horas, en función de la respuesta. La dopamina a dosis de 3-5 µg/kg/min puede ser otra herramienta útil para mejorar la perfusión renal. Para ello se diluye una ampolla de 200 mg en 250 de glucosado y se perfunde a 510 gotas/min. El mecanismo de la dopamina es aumentar el filtrado glomerular y favorecer la acción de los diuréticos. Se deben usar antagonistas de los receptores de histamina H-2 para la prevención de hemorragias digestivas. No se recomienda el uso de antibióticos de manera profiláctica. Manejo de la IRA posrenal: Definitivo es desobstruir. De manera transitoria se procederá al sondaje vesical en el caso de patología prostática. En caso de obstrucción ureteral con hidronefrosis será necesario la realización de nefrostomia percutanea. Las indicaciones de diálisis en la IRA son: hiperpotasemia o hipontremia graves, acidosis metabólica con bicarbonato plasmático menor de 10 mEq/l, sobrecarga de líquidos con edema pulmonar o insuficiencia cardiaca, pericarditis urémica, encefalopatía urémica, diátesis hemorrágica urémica y azotemia severa (urea > 250 mg/dl o creatinina > 10 mg/dl). GLOMERULOPATIAS AGUDAS (GNA): Se caracteriza por inicio abrupto de hematuria macroscópica, oliguria, falla renal, disminución súbita de las tasa de filtración glomerular con retención de sodio y agua, manifestando edema e hipertensión y proliferación de células endocapilares del glomerulo. La proteinuria varía ampliamente en este síndrome y por lo general es menos de 3 g/dl. La principal es la post-infecciosa, donde el daño glomerular resulta de un compromiso inmune, desencadenado por una variedad de infecciones bacterianas, virales o de protozoarios. La más común es la postestreptocócica, afecta a niños entre los 2 y 10 años, predomina ligeramente en hombres. Sólo ciertas cepas nefritogénicas de estreptococo se asocian con la glomerulonefritis. La variedad más común de glomerulonefritis postestreptocócica es usualmente después de una infección faríngea con estreptococo beta-hemolítico del grupo A. En los países desarrollados, la nefropatía IgA es, en la actualidad, la más frecuente de las glomerulonefritis primarias. La glomerulonefritis postestreptocócica es una enfermedad aguda y reversible, caracterizada por recuperación espontánea en la mayoría de los pacientes. Típicamente la hematuria gruesa y el edema se presentan entre 7 días a 12 semanas después de la infección estreptocócica. La resolución espontánea de las manifestaciones clínicas es por lo general rápida. La diuresis reaparece en una a dos semanas y la concentración de creatinina sérica retorna a nivel basal dentro de las cuatro semanas. La hematuria microscópica desaparece dentro de los seis meses pero la proteinuria leve permanece estática en 15 % de los pacientes después de 3 años. El hallazgo de anticuerpos contra antígenos estreptocócicos proporciona evidencia de infección reciente, pero no es diagnóstica de glomerulonefritis postestreptocócica. Son los más solicitados los anticuerpos antiestreptolisina o, antiestreptoquinasa, antihialuronidasa y antinicotinamina dinucletidasa. El tratamiento de la glomerulonefritis postestreptocócica es de soporte, enfocado a la sobrecarga de fluidos con dieta hiposódica (2.4 g/sodio/día); la hipertensión responde a diuréticos de asa (furosemide) y antihipertensivos tipo IECA, pero se deben dar con precaución si hay insuficiencia renal con hipercalemia. La terapia antimicrobiana temprana del paciente con infección faríngea o de la piel, puede prevenir la propagación de la infección estreptocócica y atenúa la severidad de la glomerulonefritis postestreptocócica, pero no previene el desarrollo de la misma. CASO CLINICO Varón de 18 años, cuidador de animales, refiere un cuadro febril de larga evolución; en dicho ingreso desarrolla un fracaso renal agudo de evolución subaguda con creatinina plasmática (crp) de 6 mg/dl. Refería fiebre elevada de dos meses de evolución, que había aparecido una semana después de sufrir una dudosa picadura de garrapata. La fiebre se presentaba diariamente sin predomino horario y con sudoración nocturna. Había recibido distintos tratamientos antibióticos de forma empírica. En el momento del ingreso llevaba 20 días sin tratamiento antibiótico persistiendo el cuadro febril. Radiografía de tórax: al ingreso, normal; un mes después presenta

CURSO ENARM CMN SIGLO XXI TEL: 36246001

Pharmed Solutions Institute

PÁGINA 245

MANUAL DE TRABAJO DEL CURSO ENARM CMN SIGLO XXI pequeños infiltrados pulmonares bibasales con imágenes aerolares. La proteinuria que inicialmente fue de rango no nefrótico (1-2 g/24 h), posteriormente alcanza el rango nefrótico. PREGUNTA Cual es la conducta a seguir mas adecuada para establecer el diagnostico final. RESPUESTA a.- Gamagrama pulmonar. b.- Biopsia renal. c.- Urocultivo. d.- Hemocultivo. CASO CLINICO Hombre de 27 años; como antecedentes refería criptorquidia, adenoidectomía y amigdalectomía en la infancia, fumador activo, bebedor social y homosexual. Dos meses con una discreta faringodinia, aparecieron una induración en la ingle derecha, lesiones ulceradas en el glande de aspecto serpinginoso y un exudado uretral blanquecino que fue tratado inicialmente con azitromicina. A la espera de resultados serológicos, se observaron lesiones maculopapulosas en la zona proximal de muslos y tronco, que se extendían a los pies y a las manos, que evolucionaron en distintas fases sin acompañarse de fiebre, junto con edematización en los miembros inferiores y en los genitales, con ligero aumento del perímetro abdominal y disminución de la diuresis. Urea 61 mg/dl, creatinina 1,73 mg/dl, iones normales, proteínas totales 4,4 g/dl, albúmina 1,8 g/dl, colesterol total 295 mg/dl, HDL 61 mg/dl, LDL 206 mg/dl, triglicéridos 140 mg/dl y enzimas hepáticas normales. En la orina, destacaban proteinuria 13,4 g en 24 horas, 250 hematíes por microlitro y leucocituria negativa. Hemograma normal, coagulación normal. PREGUNTA Cual de las siguientes pruebas tiene mayor sensibilidad y especificidad para establecer un diagnostico. RESPUESTA a.- ANA. b.- ANCA. c.- Anticuerpo FTA. d.- VHC.

CURSO ENARM CMN SIGLO XXI TEL: 36246001

Pharmed Solutions Institute

PÁGINA 246

MANUAL DE TRABAJO DEL CURSO ENARM CMN SIGLO XXI SINDROME NEFRITICO Y SINDROME NEFROTICO: SÍNDROME NEFRÓTICO: Criterios diagnósticos Proteinuria >3-3,5g/24 horas o una relación proteína: creatinina >3-3,5 (ambas en mg/dL) en una muestra aislada. Albuminemia 385 mg/dL). Es consecuencia del aumento de la permeabilidad para las proteínas y es expresión de una enfermedad glomerular cuando existe una alteración funcional o morfológica de la barrera de filtración. Nunca aparece en las enfermedades extraglomerulares. El síndrome nefrótico constituye el motivo más frecuente de realización de biopsia renal en todos los grupos de edad. Manifestaciones clínicas: La pérdida de proteínas en orina origina una serie de alteraciones que constituyen las características del síndrome nefrótico: 1. Hipoalbuminemia: aparece cuando la capacidad de síntesis hepática se ve superada por las pérdidas urinarias de albúmina y el catabolismo renal. 2. Edema: suele ser la primera manifestación; aparece en partes blandas y en casos graves puede comportar ascitis, derrame pleural y anasarca, perorbitario, en miembros inferiores. Es debido a la reabsorción de sodio y agua secundaria a la disminución de la presión oncótica capilar o estimulada directamente por la proteinuria. 3. Hiperlipidemia con aumento de las cifras de colesterol total, lipoproteínas de densidad baja (LDL), muy baja (VLDL) e intermedia (IDL), lipoproteína A y, con menos frecuencia, hipertrigliceridemia y descenso de las cifras de lipoproteínas de alta densidad (HDL). La lipiduria se manifiesta con aparición de cilindros grasos en el sedimento urinario. 4. Pérdida de inmunoglobulinas y de factores del complemento, que conlleva una tendencia especial a la aparición de infecciones (peritonitis espontáneas, celulitis, e infecciones pulmonares, meníngeas y digestivas). 5. Trombosis, con una incidencia de entre el 5 y el 60%; suele aparecer en venas renales y de extremidades inferiores, aunque también pueden afectarse territorios arteriales. 6. Hipertensión arterial, que aparece en el 42,5% de los pacientes. 7. Insuficiencia renal aguda, que se observa con más frecuencia en pacientes de edad avanzada, con hipoalbuminemia grave, tratados con dosis altas de diuréticos. Causas de síndrome nefrótico: Enfermedad glomerular primaria; esclerosis glomerular segmentaria focal, enfermedad glomerular membranosa, enfermedad glomerular con cambios minimos, enfermedad glomerular membranoproliferativa (ej: IgA). Causas secundarias de síndrome nefrótico: DM, LES, amiloidosis, mieloma multiple, agentes antimicrobianos, AINES, penicilamina, VIH, hepatitis B y C, mycoplasma, sífilis. La biopsia renal está indicada en el estudio del síndrome nefrótico del adulto, excepto en pacientes diabéticos en los que no se sospeche otra enfermedad distinta de la nefropatía diabética. En niños con síndrome nefrótico, en principio no está indicada la realización de biopsia renal (el 90% de los casos se debe a glomerulonefritis de cambios mínimos), salvo en caso de síndrome nefrótico resistente a los corticosteroides o con recidivas frecuentes. El tratamiento general comprende: reducción del edema, control de la presión arterial y del perfil lipídico y profilaxis de la trombosis. SÍNDROME NEFRÍTICO: Se caracteriza por edema, oliguria, hematuria (con cilindros hemáticos en el sedimento urinario), disminución del filtrado glomerular e hipertensión arterial, normalmente de instauración aguda. Típicamente se ha relacionado con la forma de presentación de la glomerulonefritis aguda posinfecciosa, sobre todo asociada a infecciones estreptocócicas. También puede aparecer en otras afecciones (tabla 5). Manifestaciones clínicas: Aparece como macrohematuria en el 30-50% de los casos, con datos de hematuria glomerular. En la glomerulonefritis aguda posestreptocócica se presenta tras 2-3 semanas de una infección faringoamigdalar o 4-6 semanas de una infección cutánea. La hipertensión arterial suele ser moderada, con edemas en párpados y extremidades inferiores. En algunos casos puede producirse insuficiencia cardíaca congestiva. La proteinuria es inferior a 1-2 g/24 h. Es fundamental la realización de una historia clínica completa, interrogando sobre antecedentes familiares, manifestaciones clínicas sistémicas, infección reciente o consumo de fármacos. Patogenia: Las cepas nefritogénicas producen proteínas catiónicas identificadas en tejidos renales de pacientes con glomerulonefritis. Como consecuencia de su carga eléctrica, estas proteínas se depositan en el glomérulo y dan lugar a la formación in situ de complejos inmunes, además de los complejos inmunes circulantes formados por la unión de inmunoglobulinas con otros antígenos. Estos complejos circulantes es posible encontrarlos en la primera semana de la enfermedad y se sabe que están en relación con la gravedad de la enfermedad; de ahí su importancia a la hora de hacer el diagnóstico. Aproximadamente el 90% de los casos con glomerulonefritis presenta reducción de los niveles séricos de complemento, debido a que después del depósito en la membrana basal de estos inmunocomplejos se activa la cascada inflamatoria, en un inicio mediada por interleucina 1 con la subsecuente activación de linfocitos T y posteriormente mediada por interleucina 2, que al unirse con su receptor específico causa proliferación de los linfocitos activados y posterior depósito de complemento y formación de perforinas que incrementan la lesión en la membrana basal. Todas estas alteraciones reducen el calibre de los capilares glomerulares, disminuyendo la superficie de filtración, lo que lleva a la consecuente reducción en la filtración glomerular. Esta disminución tiende a ser compensada por el aumento de la presión de ultrafiltración que ocurre porque la presión oncótica peritubular tiende a disminuir, con disminución subsecuente de la fracción excretada de sodio. Tanto la reabsorción de sodio como de agua expanden el volumen del líquido extracelular, expresamente el volumen circulante efectivo, causando hipertensión arterial secundaria. En la exploración física valoraremos la presencia de edemas, hipertensión arterial o datos de insuficiencia cardíaca. La ausencia de manifestaciones clínicas sistémicas, el antecedente de infección, la hipocomplementemia y títulos elevados de anticuerpos antiestreptolisina establecen usualmente el diagnóstico de glomerulonefritis aguda posestreptocócica. La biopsia renal estaría indicada si existiera algún dato que hiciera dudar del diagnóstico o en caso de insuficiencia renal progresiva. Si se sospecha otro tipo de infecciones o se observan manifestaciones clínicas sistémicas, estaría justificado el estudio serológico, la realización de cultivos y el estudio de autoanticuerpos o inmunoglobulinas. Tratamiento: consiste en tratar la infección, o específicamente el trastorno subyacente si se debe a otra entidad. Las causas más comunes de síndrome nefrítico: Enfermedades sistémicas; LES, endocarditis, absceso viscerales, nefritis por cortocircuito, crioglobulinemia, poliarteritis nodosa, vasculitis por hipersensibilidad, granuloma de Wegener, purpura de Henoch-Schonlein. Enfermedades renales; glomerulonefritis aguda postinfecciosas, glomerulonefritis membranoproliferativa, nefropatía por IgA. El tratamiento general del síndrome nefrítico se basa en el manejo adecuado del volumen y el control tensional. Para el control del edema se emplean diuréticos de asa y restricción hidrosalina. En caso de insuficiencia cardíaca o edema agudo de pulmón, se emplea oxígeno, morfina y nitroglicerina. El control tensional se consigue con bloqueadores del sistema renina-angiotensina y diuréticos. En caso de uremia grave, hiperpotasemia y/o insuficiencia cardíaca refractaria puede estar indicada la diálisis.

CASO CLINICO

CURSO ENARM CMN SIGLO XXI TEL: 36246001

Pharmed Solutions Institute

PÁGINA 247

MANUAL DE TRABAJO DEL CURSO ENARM CMN SIGLO XXI TRASTORNOS ACIDO-BASE AGUDOS CIENCIAS BASICAS: Cuando se habla de equilibrio acido-base en realidad se hace referencia a la regulación de la concentración de hidrogeniones en los líquidos corporales. Pequeños cambios en la concentración de iones hidrogeno pueden producir grandes alteraciones en las reacciones químicas celulares, aumentando algunas e inhibiendo otras; por este motivo la regulación de la concentración de iones de hidrogeno es uno de los aspectos más importantes de la homeostasis. El pH sanguíneo arterial se mantiene entre 7.35-7.45, cualquier desviación de este rango implica desequilibrio acido-base. El pH plasmático se refiere habitualmente a la relación entre las concentraciones de TRASTORNOS PRIMARIOS Y RESPUESTAS COMPENSADORAS bicarbonato/ácido carbónico. El CO2, en TRASTORNO ALT. PRIMARIA RESPUESTA COMPENSADORA presencia de anhidrasa carbónica (AC), se Acidosis metabólica [HC𝑂3−] PaCO2 desciende 1,2mmHg por cada 1 hidrata de la siguiente forma: CO2 + mEq/l de aumento de la [HC𝑂3−] H2O H2CO3  H+ + HCO3-. Al valor de Alcalosis metabólica [HC𝑂3−] PaCO2 aumenta 0,7mmHg por cada 1 pH arterial por debajo de 7.35 se denomina mEq/l de descenso de la [HC𝑂3−] acidemia y cuando está por arriba de 7.45, Acidosis respiratoria PaCO2 Aguda: [HC𝑂3−] aumenta 1mEq/l por cada alcalemia; la alteración que cursa con niveles 10mmHg de aumento de PaCO2 de pH por debajo de 7.35 se le denomina Crónica: [HC𝑂3−] aumenta 3,5mEq/l por acidosis y la que cursa con niveles de pH por cada 10mmHg de aumento de PaCO2 Alcalosis respiratoria PaCO2 Aguda: [HC𝑂3−] desciende 2mEq/l por arriba de 7.45 se denomina alcalosis. Factores cada 10mmHg de desciende de PaCO2 de riesgo: Se recomienda tener alto índice de Crónica: [HC𝑂3−] desciende 5mEq/l por sospecha para detectar oportunamente cada 10mmHg de aumento de PaCO2 trastornos del equilibrio acido-base en las Valores normales: sangre arterial pH 7,36-7,44; PaCO2 36-44mmHg; [HC𝑂3−] 22-26mmHg personas con factores de riesgo para Sangre venosa pH 7,31-7,37; PaCO2 42-50mmHg; [HC𝑂3−] 23-27mmHg desarrollo. El equilibrio ácido-base requiere la integración de tres sistemas orgánicos: Hígado, Pulmones, Riñón. El hígado metaboliza las proteínas produciendo iones hidrógeno (H+), el pulmón elimina el dióxido de carbono (CO2), y el riñón generando nuevo bicarbonato (HCO3). Un ácido es una sustancia capaz de donar un H+ y una base una sustancia capaz de aceptarlo. Por tanto, la acidez de una solución depende de su concentración de hidrogeniones [H+]. SISTEMAS DE AMORTIGUAMIENTO: El funcionamiento celular requiere mantener la concentración de H+, del líquido extracelular (LEC) en límites muy estrechos (el pH compatible con la vida esta entorno a 6.80-7.80). Dado que los procesos metabólicos generan gran cantidad de ácidos, el organismo necesita neutralizar y eliminar los H+ para mantener constante el pH (-log + + [H ]) del LEC. Para ello, dispone de varios medios: 1. Amortiguadores plasmáticos. Bicarbonato El H se une al HCO3- en forma reversible, cuando el aporte o la producción de H+ aumenta, la reacción se desplaza hacia la derecha, con lo que incrementa la cantidad de H+ que es captado por el amortiguador, lo que minimiza los cambios de la H +. El HCO3- representa el 50% de la capacidad amortiguadora del plasma. Si la producción de ácidos no volátiles excede la excreción, el HCO3- disminuye y la H+ aumenta, resultando en acidosis metabólica, por otra parte si la excreción de HCO3- es mayor que la producción el HCO3- aumenta y la H+ disminuye resultando en alcalosis metabólica. H + + HCO3- H2CO3 H2O + CO2- Hemoglobina (Hb), Proteínas y fosfatos. Existen otros sistemas de amortiguamiento como la Hb, proteínas y fosfatos, los cuales proveen de sitios adicionales de unión de H + y por lo tanto amortiguamiento. La Hb proporciona el 30% de la capacidad amortiguadora del plasma, el restante 20% lo comparten las proteínas y los fosfatos (13 y 7% respectivamente). 2. Respuesta respiratoria El segundo sistema de amortiguamiento que hace frente a los trastornos del equilibrio AB es el pulmonar; la disminución en el pH actúa estimulando quimiorreceptores en el tallo cerebral con incremento en la ventilación minuto y eliminación del CO2. 3. Respuesta renal. El riñón es el tercer sistema de amortiguamiento, para mantener el equilibrio AB, éstos deben de excretar aniones de los ácidos no volátiles y reabsorber el HCO3-, esto lo logran por medio de tres mecanismos: reabsorción o excreción del bicarbonato filtrado, excreción de acidez titulable y excreción de amoniaco. En definitiva, según la ley de acción de masas, la acidosis (aumento de H +) puede producirse por una disminución del bicarbonato (acidosis metabólica) o por un aumento de la PaCO2 (acidosis respiratoria); y la alcalosis (disminución de los H+) por un aumento del bicarbonato (alcalosis metabólica) o por una disminución de la PaCO2 (alcalosis respiratoria). Así pues la alcalosis o la acidosis son estados en los que existe un acumulo de bases o ácidos. Por otra parte en todo trastorno acidobásico se producen necesariamente respuestas compensadoras (renales en trastornos respiratorios y respiratorios en trastornos metabólicos), que intentan mantener normal el pH.

CURSO ENARM CMN SIGLO XXI TEL: 36246001

Pharmed Solutions Institute

PÁGINA 248

MANUAL DE TRABAJO DEL CURSO ENARM CMN SIGLO XXI ACIDOSIS METABOLICA Ej: Ph: 7.32; PCO2: 45; HCO3: 16 (aguda).Ph:7.34; PCO2: 28; HCO3: 18 (subaguda). Ph: 7.36; PCO2: 34; HCO3: 19 (crónica) Clínica Causas Anorexia, Anión GAP elevado Fatiga, (normocloremicas) Deshidratación, Cetoacidosis diabética Confusión, Acidosis láctica Letargia, Insuficiencia renal Estupor, Coma Intoxicaciones (salicilatos, Taquicardia, etanol, metanol, Hipotensión formaldehido) arterial Rabdomiolisis sistémica, Anión GAP normal Taquipnea, (hipercloremicas) Disnea, Fatiga Perdidas digestivas de de músculos bicarbonato (diarrea, respiratorios fistulas pancreáticas, Respiración de biliares) Kussmaul Perdidas renales de bicarbonato (acidosis tubulares renales, hioperparatiroidismo, fármacos; acetazolamida, anfotericina, ciclosporina)

MANIFESTACIONES CLINICAS ACIDOSIS RESPIRATORIA ALCALOSIS METABOLICA Ej.: Ph: 7.24; PaCo2: 55; HCO3: Ej: Ph: 7.48; PCO2: 35; HCO3: 26 23 (aguda). pH 7.33; PaCO2: 53; (aguda). Ph: 7.48; PCO2: 50; HCO3: 28 HCO3: 28 (subaguda). pH: 7.35, (subaguda). Ph: 7.45; PCO2: 50; HCO3: PaCo2: 50, HCO3: 28 (crónica) 35 (crónica) Clínica Causas Clínica Causas Confusión, EPOC, Apatía, Perdidas digestivas Coma, neumonía, Vomito, (vomitos, aspiración Convulsiones, asma, SDRA Bulimia, nsaogastrica, Alteraciones Sedación Confusión, diarrea) del ritmo Hipoventilación Arritmias Diuréticos cardiaco, mecánica cardiacas, Hiperaldosteronismo Hipotensión Enfermedad Hiperreflexia, 1ro y 2ro. arterial, neuromuscular Clonus, Dieta pobre en sal Cefalea (Guillan-Barre, Convulsiones, Hipercalcemia esclerosis Hiporreflexia Hipopotasemia múltiple, crisis Administracion de miastenica) bicarbonato sódico Neumonia Transfusiones Derrame masivas de sangre pleural masivo

ALCALOSIS RESPIRATORIA Ej: Ph:7.50; PaCO2: 30; HCO3:23 (aguda). Ph: 7.46; PaCo2: 27; HCO3: 19 (subaguda) Clínica Vértigo, Mareo, Ansiedad, Euforia, Alucinaciones, Alteraciones del estado de conciencia, Mioclonus, Asterixis, Taquicardia

Causas Ansiedad Hipoxia Hiperventilación Dolor Fiebre Enfermedad del tronco cerebral

ANION GAP: La neutralidad química del plasma se mantiene por el equilibrio entre aniones (Cl 103; HCO3 24; otros aniones 10; proteínas 17 =154) y cationes (Na 143; K 4.5; Ca 5.0; Mg 1.5 = 154) excretados y retenidos. El número total de Cationes plasmáticos debe igualar a los aniones. Los aniones medidos en el laboratorio son inferiores a los cationes medidos. Esta diferencia se denomina ANION GAP. El anion gap normal es de 12+/- 2. Una AG superior indica la presencia anormal de ácidos. AG = cationes – suma de aniones. AG= Na – (Cl + HCO3). Según el valor de anión GAP calculado las acidosis metabólicas pueden dividirse en dos grupos: Con anión GAP elevado (por adición de ácidos fijos). Con anión GAP normal o hiperclorémicas (por perdida de Bicarbonato). El aumento en la concentración de cloro se debe a que si el bicarbonato desciende, el Cloro aumenta para mantener la electroneutralidad del medio. Corrección del anión GAP: Según valor de albúmina: por cada g/dl de albúmina por encima de 4 se suma al anión GAP calculado 2 puntos y, por cada g/l por debajo de 4 se restan 2 puntos. CASO CLINICO Mujer de 81 años con hipertensión arterial, dislipemia, diabetes tipo 2 y miocardiopatía dilatada (fracción de eyección [FE] del 30%). Tratamiento habitual: telmisartán, metformina a dosis de 850 mg/8 horas, atorvastatina, bezafibrato y omeprazol. Acude a urgencias por diarrea mucosanguinolenta y vómitos de una semana de evolución acompañados de oligoanuria en las últimas 24 horas. TA, 120/70 mmHg, FC de 95 lpm, temperatura 36 ºC. Glasgow 12, desorientación temporoespacial y bradipsiquia, sin signos de focalidad. PREGUNTA Cual es la prueba mas apropiada para establecer el diagnostico agudo del paciente. RESPUESTA a.- Realizar TAC. b.- Realizar IMR. c.- QS y BH. d.- Gases arteriales. CASO CLINICO Analítica: hemoglobina 11,7 g/dl, leucocitos 18.030 (78,9% neutrófilos), plaquetas 307.000, glucosa 68 mg/dl, urea 133 mg/dl, creatinina 6,89 mg/dl, sodio 134 mEq/l, potasio 4,4 mEq/l, pH 6,89, pCO2: 29 mmHg, bicarbonato 6,9 mmol/l, calcio iónico 3,85 mg/dl, anión gap 28. Coagulación normal. Orina: pH 6, creatinina 71 mg/dl, proteinuria 400 mg/dl, 100 hematíes/campo, 60 leucocitos/campo, cuerpos cetónicos positivos. PREGUNTA Cuál es la conducta terapéutica más adecuada. RESPUESTA a.- Bicarbonato. b.- Solucion salina. c.- Solucion glucosada. d.- Dobutamina.

CURSO ENARM CMN SIGLO XXI TEL: 36246001

Pharmed Solutions Institute

PÁGINA 249

MANUAL DE TRABAJO DEL CURSO ENARM CMN SIGLO XXI CASO CLINICO Varón de 75 años, hipertenso, con enfermedad pulmonar obstructiva crónica. Consultó por cuadro de infección respiratoria y deterioro funcional asociado consistente en apatía, somnolencia y períodos de agresividad. A la exploración afebril, desorientado, con tendencia al sueño, taquipnea superficial, roncus dispersos y crepitantes en la base izquierda, con imagen radiológica sugerente de bronquiectasias sobreinfectadas. Cloro (114 mEq/l), con función renal y resto de iones normales (sodio: 138 mEq/l; potasio: 4,2 mEq/l), y una gasometría arterial (pH: 7,24; pCO2: 33 mmHg, pO2: 67 mmHg; bicarbonato: 17 mmol/l, y exceso de bases (EB): –9,1 mmol/l). El valor del anión GAP (diferencia entre el valor del sodio sérico y la suma del cloro y el bicarbonato) fue 7 mEq/l. PREGUNTA Cuál es el estado clínico acido base del paciente. RESPUESTA a.- Acidosis metabolica. b.- Alcalosis metabolica. c.- Acidosis respiratoria. d.- Alcalosis respiratoria. CASO CLINICO Paciente de 61 años de edad ex-fumador, con hipertensión arterial (HTA), diabetes mellitus tipo II, dislipemia, portador de marcapasos por enfermedad del nódulo sinusal, claudicación intermitente y con valvulopatía aórtica, estenosis carotídea y cardiopatía isquémica intervenida. Creatinina de 1.56 mg/dl. En la exploración física destaca soplo carotídeo derecho, pulsos poplíteos débiles y ausencia de pulsos tibiales y pedios como hallazgos patológicos. BH con (Hb 10.4 mg/dl, VCM 91 fl, HCM 30.6 pg y CHCM 31 gr/dl), y en la bioquímica plasmática potasio 3.7 mEq/l. En la gasometría venosa: (ph 7,461) y (CO3H 29.5 mEq/l), con (pCO2 medida 42.3 mmHg, con PCO2 estimada de 43,85 +/- 2 mmHg). PREGUNTA Cual es el manejo más adecuado para corregir los gases. RESPUESTA a.- Disminuir Fi02 b.- Disminuir FR. c.- Disminuir volumen ventilarotio. d.- Disminuir volumen residual.

CURSO ENARM CMN SIGLO XXI TEL: 36246001

Pharmed Solutions Institute

PÁGINA 250

MANUAL DE TRABAJO DEL CURSO ENARM CMN SIGLO XXI TRASTORNOS ELECTROLITICOS AGUDOS: CIENCIAS BASICAS: El riñón desempeña un papel esencial en el equilibrio hidrosalino del organismo porque regula la excreción de sodio y del agua. La osmolalidad del sector extracelular es estrictamente una función de la natremia. La natremia depende de: el capital TIPOS DE SOLUCIONES INTRAVENOSAS sódico, el capital potásico y el agua total. En situaciones normales los riñones SOLUCION CONCEPTO/ACCIONES regulan la osmolalidad sérica entre 295 a 300 mosm/kg. de agua. La ISOTÓNICOS homeostasis del agua depende de la normalidad del mecanismo de la sed y de Suero Expansor de volumen extracelular para la la secreción de la hormona antidiurética (ADH) por la hipófisis posterior. Sobre fisiológico al hipovolemia estos últimos mecanismos interactúan la presión arterial, la osmolalidad 0.9% Un exceso puede provocar hipervolmia y celular y sérica, la angiotensina II y las prostaglandinas. La osmolalidad sérica Na: 154mEq/l acidosis por exceso de cloro Cl: 154mEq/l Solo aporta iones cloro y sodio se puede estimar a partir de las siguientes formulas: Osmolalidad sérica= 2Na 308 mOm/l Se puede usar en hiponatremia (mEq/L)+ K(mEq/L)+Urea (mg/dl)/2.8 +Glucosa (mg/dl)/18 o si la urea y la Único que se puede usar conjuntamente con glucosa son normales se puede utilizar la siguiente formula simplificada: sangre y derivados Osmolalidad= (Na+10) x 2. La osmolalidad sérica se expresa en mosm/kg. de Suero Igual que SF al 0.9%, pero la mezcla de glucosado al dextrosa en sangre provoca hipotonicidad agua. Las principales manifestaciones de un trastorno en el equilibrio del agua + 5% Aporta muchas calorías (170/l) son la hiponatremia y la hipernatremia. HIPONATREMIA: Sodio (Na ) 145mmol/l, y por tanto existirá un aumento de la osmolalidad plasmática superior a 300 mosm/L. Menos frecuente que hiponatremia, más común en niños y adultos mayores. Puede ser resultado de perdida de agua (frecuente) o de un aporte de sodio (raro). La pérdida de agua desencadena dos mecanismos para evitar la hipernatremia: la sed y la liberación de vasopresina (ADH). Siempre hay que tener en mente la sed es primordial, ya que incluso la máxima secreción de ADH puede no lograra retener agua suficiente para compensar las pérdidas si no se aumenta el aporte de agua. La hipernatremia con hipervolemia es poco frecuente y se produce por una ganancia neta de sodio. Es mucho más frecuente encontrar situaciones de normo o hipovolemia. Sus causas pueden ser: euvolémicas con Na+ normal y se observa en los estados febriles severos, hiperventilación pulmonar; hipovolémica con Na disminuido se observa en la deshidratación por sudoración profusa, uso de manitol y soluciones glucosadas, diarreas, quemaduras, diabetes insípida(central o nefrogénica su diferencial mediante test de deprivación de agua, midiendo la ADH plasmática); hipervolémica con Na aumentado la cual se observa cuando se utilizan en forma indiscriminada soluciones de bicarbonato de sodio, cuando se utilizan la carbenicilina y los esteroides y en el coma hiperosmolar no cetogénico. Clínica: sed, astenia, debilidad muscular, somnolencia, irritabilidad, letargia, confusión, temblor, convulsiones y coma. Laboratorio: sodio sérico y osmolalidad plasmática. El tratamiento consiste en administrar soluciones glucosadas al 5% en los pacientes deshidratados con sodio normal. En los pacientes con volumen intravascular aumentado se deben utilizar los diuréticos más soluciones glucosadas al 5%. En la diabetes insípida central se utilizará la vasopresina exógena y en nefrogénica tiazidas. En forma práctica, para calcular la cantidad de líquido a administrar se utiliza la siguiente formula: Agua corporal total (ACT) real = peso corporal (Kg) x 0.6. Na+ plasmático real/Na+ plasmático deseado X ACT real = ACT deseada. ACT deseada- ACT real = déficit de líquido. Es prudente

CURSO ENARM CMN SIGLO XXI TEL: 36246001

Pharmed Solutions Institute

PÁGINA 251

MANUAL DE TRABAJO DEL CURSO ENARM CMN SIGLO XXI administrar el 30% de la solución calculada en las primeras 24 horas. La corrección rápida de hipernatremia puede producir edema cerebral, convulsiones, daño neurológico permanente e incluso la muerte. Para disminuir riesgo se aconseja corregir Na + lentamente. En casos graves (>170mEq/l), no corregir a menos de 140mEq/l en primeras 48-72hrs y en formas crónicas no debe corregirse mas de 812 mEq/l. HIPOPOTASEMIA: Potasio (K+) 5.5 mEq/L y constituye una verdadera emergencia médica ya que el paciente corre el riesgo de morir por paro cardiaco. La hiperkalemia puede ser leve hasta 6.5 mEq/L, moderada hasta 7.5 mEq/L y severa superior a 7.5 mEq/L. Las causas más frecuentes dehiperkalemia son: insuficiencia renal aguda y crónica, uso de diuréticos ahorradores de potasio, insuficiencia suprarrenal aguda o crónica (enfermedad de Addison), hipercatabolismo, acidosis metabólica y la pseudohiperkalemia por muestra de sangre hemolizada, trombocitosis y leucocitosis por encima de 100.000 mm3. Se deben solicitar potasio sérico y electrocardiograma, ya que éste es fundamental para evaluar la hiperkalemia. Cuando es leve al ECG se observan ondas T altas picudas y simétricas, cuando es moderada se observa ensanchamiento del complejo QRS, disminución de la amplitud de la onda P y prolongación del intervalo PR y cuando es grave desaparición de la onda P, bradicardia, extrasistoles ventriculares, fibrilación ventricular y asistolia. Tratamiento, además del preventivo (evitar frutas cítricas, evitar uso de diuréticos ahorradores de potasio, no administrar sangre almacenada por largo tiempo). Moderada: se utilizan resinas de intercambio catiónico (poliestirensulfonato cálcico) que eliminan el potasio del tubo digestivo. Diuréticos. Grave: glucosa intravenosa junto con insulina de acción rápida (10UI de insulina en 500ml de glucosa al 10%). La insulina favorece la entrada de potasio a la célula y la glucosa previene la aparición de hipoglucemia. Administración de bicarbonato de sodio vía intravenosa para corregir la acidosis. Furosemida: 40 mg IV a repetir cada 4 horas o bumetanida 1 mg IV cada 4 hrs. Gluconato de calcio: 10 ml al 10% IV en 20 minutos. Bicarbonato de sodio: 60 ml IV rápidamente cada 8 horas. Solución glucosada al 10% 250 ml más 10 unidades de insulina cristalina IV. Sulfonato de poliestireno sódico (Kayexalate) intercambia sodio por potasio a nivel intestinal en enema a retener 50 a 100 gramos en 100 a 200 ml de agua o por vía oral 20 a 40 gramos más 20 ml de sorbitol al 50%. De ser necesario y en medio especializado hemodiálisis o diálisis peritoneal. HIPOCALCEMIA: Calcio (Ca 2+) 250mg/dl >600mg/dl Cetonuria +++ + angiotensina, aldosterona, este complejo proceso hace que los riñones Bicarbonato sérico 15 reabsorban sodio y agua. También la hipófisis posterior segrega hormona pH 7.30 antidiurética que ayuda también a conservar el agua y el sodio. La diuresis Brecha aniónica Na- (Cl+HCO3) >12 320 puede estar entre 5 y 7 litros. Además se produce también déficit de cloruro de 2Na+(glu/18)+(BUN/2.8) sodio entre 3-10mmol/kg, los niveles de sodio pueden estar falsamente alterados por la hiperglucemia presente. El potasio se encuentra también severamente disminuidos, sin embargo los niveles de potasio en suero pueden estar normales o incluso elevados durante el episodio, debido a la acidosis y a la hiperglucemia presente, disminuyendo severamente el potasio intracelular. Otros elementos que pueden estar alterados son el fosfato, el magnesio y el calcio. CAUSAS: Diagnostico de novo, infecciones, enfermedades intercurrentes, falla en la aplicación de insulina o en la toma de medicamentos orales (corticoides), excesos alimentarios, cirugía, traumatismo, desconocida. En los diabéticos diagnosticados, su causa desencadenante suele ser una situación estresante que incrementa las necesidades de insulina, aunque también puede obedecer a una descompensación de la enfermedad por no haber seguido correctamente el tratamiento prescrito CLINICA Y DIAGNOSTICO: En cuadro. Iniciar con Bh completa y gases arteriales, EGO, glucosa plasmática, BUN, electrolitos, cretinina, electrocardiograma. TRATAMIENTO: El objetivo principal e inicial es la corrección del déficit hídrico para expandir el volumen intra y extracelular y asegura una adecuada perfusión renal. Si no existen trastornos cardiacos se inicia con solución salina al 0.9% normal; si el sodio es mayor de 155meq/l se recomienda utilizar solución salina al 0,45 normal. COMPLICACIONES: La aparición de edema cerebral es raro afortunadamente, múltiples factores influyen en su origen, incluyendo la aparición de idiosmoles que causan un gradiente y una desviación del agua hacia las células, la terapia con insulina por si sola aumenta la entrada de sustancias osmóticamente activas en el espacio intracelular y un aumento rápido del déficit de sodio. Síndrome de dificultad respiratoria del adulto; esta complicación puede producirse por el aumento en el volumen de agua en los

CURSO ENARM CMN SIGLO XXI TEL: 36246001

Pharmed Solutions Institute

PÁGINA 255

MANUAL DE TRABAJO DEL CURSO ENARM CMN SIGLO XXI pulmones y una disminución en la adaptabilidad pulmonar. Acidosis metabólica hipercloremica; el mecanismo principal es la perdida de cetoácidos por la orina, los cuales son requeridos para la generación de bicarbonato

CASO CLINICO Una mujer de 76 años que no se conocía diabética ingresó por desihidratacion grave, refieren los familiares que vive sola y desconocen su tratamiento, refieren que la paciente presentaba desorientación, letargia, falta de respuesta a estimulos, se ingresa a la paciente con datos de desihidratacion glucemia de 596mg/dl pH 7.29, glucosuria y cetonuria, con osmolalidad en suero calculada de 318 mOsm/l. PREGUNTA Cual de las siguientes condiciones incrementan el riesgo de cetoacidosis. RESPUESTA a.- Infeccion de vías urinarias. b.- Desequilibrio hidroelectrolitico. c.- Osmolaridad alterada. d.- Dieta cetogenica. CASO CLINICO Una mujer de 76 años que no se conocía diabética ingresó por una descompensación hiperglucemica cetosica. Durante el ingreso sufrió 4 crisis parciales motoras con generalización secundaria, de inicio motor hemicorporal izquierdo y generalización tónico-clónica, con una duración de entre 1 y 3 minutos, con recuperación del nivel de conciencia entre las crisis y en un periodo de 5h, seguidas de un intenso déficit poscrítico hemisférico derecho, con hemiparesia, hemihipoestesia, hemianopsia y heminegligencia izquierdas, recuperándose en las siguientes 24h. La analítica mostró una glucemia de 596mg/dl pH 7.30, glucosuria y cetonuria, con osmolalidad en suero calculada de 318 mOsm/l. Se inició tratamiento con anticonvulsivante), sin recurrencia de las crisis. Se le realizó una RM craneal 18 horas después de la primera crisis, donde se apreció una hipointensidad en sustancia blanca subcortical parietal derecha en T2 con ligera hiperintensidad cortical en FLAIR, tenue captación giriforme de contraste y leve restricción de la difusión en dicha localización. El electroencefalograma (EEG) evidenció un foco de ondas delta frontotemporal derecho. PREGUNTA Considerando las complicaciones que se presentan en esta patología cual es su pronostico mas probable? RESPUESTA a.- Bueno ya que se resolvió adecuadamente. b.- Moderado posibles secuelas. c.- Puede continuar con crisis convulsivas. d.- Para evitar recurrencias es conveniente dejar anticonvulsivo. CASO CLINICO Mujer de 42 años con antecedentes de tiroiditis crónica de Hashimoto, 3 partos normales, sin macrosomía y una enfermedad mixta del tejido conjuntivo (EMTC), ante la aparición de una poliartritis de pequeñas articulaciones, esclerodactilia, fenómeno de Raynaud, miopatía, anticuerpos antinucleares (+) 1/5120 y anticuerpos antiENA (++++). Una tía era diabética tipo 2. Usaba levotiroxina 50 µg/día, prednisona 7,5 mg diarios y MTX 25 mg intramusculares semanales. Hace tres meses, se espació la dosis de MTX a 25 mg cada dos semanas, apareciendo debilidad muscular progresiva, mialgias y posteriormente sed y poliuria. La glicemia fue 286 mg/dl (glicemias

CURSO ENARM CMN SIGLO XXI TEL: 36246001

Pharmed Solutions Institute

PÁGINA 256

MANUAL DE TRABAJO DEL CURSO ENARM CMN SIGLO XXI previas siempre normales), prescribiéndose dieta y glibenclamida 5 mg/día. Tres semanas después consultó en un Servicio de Urgencia por sed intensa. La glicemia fue 550 mg/dl con cetonemia (-). Se aumentó la glibenclamida a 10 mg/día e inició metformina 850 mg y antiinflamatorios. Una semana después, consultó nuevamente por vómitos, sed, poliuria, mialgias, artralgias y compromiso importante del estado general. Fue hospitalizada, destacando deshidratación marcada, normotensión y polipnea. IMC: 27,3 kg/mt2. La piel de la cara estaba acartonada, enrojecida, violácea y tenía aspecto cushingoide. No existía acantosis nigricans. El tiroides se palpaba normal y el examen cardiopulmonar y abdominal era normal. Existía falta de fuerzas en las extremidades. La glicemia era 414 mg/dl, cetonemia (++), hemoglobina glicosilada A1C 12%, existía acidosis metabólica (pH 7,0, bicarbonato 2,9 mEq/L) con lactacidemia normal, creatinina 1,3 mg/dl y potasio 2,6 mEq/L. Se administró insulina cristalina subcutánea y luego en bomba de infusión en dosis crecientes, hasta 520 unidades en 24 h, sin lograr controlar la hiperglicemia ni la cetoacidosis. Recibió bicarbonato de sodio y potasio. Se descartó la existencia de una infección. A pesar de la administración de grandes dosis de insulina cristalina, persistieron la hiperglicemia y la cetosis. PREGUNTA Considerando las condiciones del caso cual de las siguientes medidas serian mas útiles para identificar la resistencia al tratamiento? RESPUESTA a.- Buscar anticuerpos antiinsulina. b.- Buscar anticuerpor antireceptores. c.- Buscar niveles de CD8. d.- Buscar anticuerpos antimicrosomales tiroideos TRAUMA CRANEOENCEFALICO (TCE): CIENCIAS BASICAS: En adultos, es definido como un intercambio brusco de energía mecánica que genera deterioro físico y/o funcional del contenido craneal. Se consigna como alteración del contenido encefálico el compromiso de conciencia, la amnesia postraumática y/o un síndrome vertiginoso o mareos persistentes. También debe considerarse como un signo de disfunción del contenido craneal la aparición de una cefalea holocránea persistente y progresiva que puede o no acompañarse de vómitos. Se distingue de la Contusión de cráneo, que corresponde a un impacto mecánico sobre la bóveda craneana que no produce alteración del contenido craneano, y que puede asociarse a dolor local. SIGNOS DE ALARMA, en la evaluación prehospitalaria o la llegada a urgencias: Deterioro progresivo de la conciencia (disminución de Glasgow), signos de focalidad neurológica, cefalea progresiva, vómitos explosivos recurrentes, agitación psicomotora, convulsiones, amnesia anterógrada de más de 30 min, cambio en el tamaño de las pupilas, sospecha de herida craneal penetrante, intoxicación, evidencia clínica o radiológica de fractura de cráneo, sat. O2 70mmHg, normocapnia, presión arterial media >90mmHg, euvolemia, Hb >10mg/dl, osmolaridad plasmática >290mOsm, glucemia 90mmHg y la sistólica es >140mmHg, o un incremento en la TA sistólica de por lo menos 30mmHg del valor basal o de diastólica de por lo menos 15mmHg sobre el valor basal. Proteinuria: Excreción urinaria de proteínas mayor de 30mg/dl o más en tiras reactivas (se requieren 2 determinaciones o más en un lapso de 6 horas en ausencia de infección). 300mgs o más en una colección de orina en 24h. SALUD PUBLICA: Los trastornos hipertensivos durante la gestación, son la primera complicación médica en muchos países del mundo, constituyendo una de las principales causas de morbilidad y mortalidad materna, fetal y neonatal. 7-10% de los embarazos se complican con HTA. La preeclampsia/eclampsia origina el 70% de los estados hipertensivos, y el 30% lo representan pacientes con hipertensión crónica preexistente durante el embarazo. En México su prevalencia varía entre 7-10% de la población gestante, existe mayor incidencia antes de los 20 años y después de los 35; aproximadamente 75% de los casos corresponden a primigestas. CLASIFICACION, de los trastornos hipertensivos durante el embarazo: A) Enfermedad hipertensiva inducida por el embarazo: Preeclampsia leve. Preeclampsia severa. Inminencia de eclampsia. Eclampsia. Síndrome de HELLP. Preeclampsia recurrente. Enfermedad hipertensiva no clasificable (imposibilidad de clasificar por carecer de elementos necesarios o por haberse instituido tratamiento previo a su estadificacion). Hipertensión transitoria (HTA después de la semana 20 o en las primeras horas posparto sin otros signos de preeclampsia). B) Enfermedad vascular crónica hipertensiva: Hipertensión sistémica esencial (HTA independiente de la gestación o anterior a las 20 semanas y que persiste más de 6 semanas posparto y que no sea a consecuencia de lesión de alteración anatómica o funcional renal). Hipertensión crónica con enfermedad hipertensiva agregada (HTA previa al embarazo agregándose preeclampsia, puede haber elevación de ácido úrico igual o mayor de 6mg/dl). De acuerdo a la Clasificación del Colegio Americano de Obstetricia y Ginecología, se identifican 4 categorías: HIPERTENSION CRONICA: Coexistencia de hipertensión de novo antes de las 20 SDG, al igual que si tiene el antecedente de hipertensión preexistente antes del embarazo. Puede ser: primaria (esencial) o secundaria (renal, suprarrenal, etc). Las mujeres con hipertensión crónica tienen un riesgo de 10-20% de desarrollar preeclampsia. Es conveniente, además, en las pacientes con hipertensión crónica, establecer si ya existe daño en algún órgano blanco antes del embarazo, incluyendo hipertrofia ventricular izquierda, retinopatía y/o enfermedad renal. En canto a los medicamentos, suspender el tratamiento con medicamentos del grupo de los IECA, así como ARA II. Existen comunicaciones sobre restricción del crecimiento fetal intrauterino, oligohidramnios, insuficiencia renal y muerte neonatal. Hay que reemplazarlos por alfametil-dopa, labetalol o los antagonistas de calcio. HIPERTENSION ARTERIAL CRONICA CON PREECLAMPSIA SOBREAGREGADA: Pacientes con hipertensión crónica, que presentan descompensación de las cifras tensionales y aparición o incremento de proteinuria después de las 20 SDG. El pronóstico es peor que con cada entidad por separado. Criterios diagnósticos: Aparición de proteinuria por primera vez, después de las 20 semanas de gestación. Incremento súbito de la proteinuria en pacientes que la presentaban antes del embarazo. Incremento súbito de la presión arterial en la gestante que la tenía bien controlada. Trombocitopenia (menos de 100 000 plaquetas/mL). Incremento de enzimas hepáticas por encima de valores normales. PREECLAMPSIA: la describimos más adelante y por ultimo HIPERTENSION GESTACIONAL: Presión arterial sistólica mayor o igual a 140mmHg y/o presión arterial diastólica mayor o igual a 90mmHg en una mujer previamente normotensa, después de la semana 20 de gestación. En dos tomas separadas por un intervalo de 4 a 6h entre una y otra con ausencia de proteinuria y se recupera después del parto. Las mujeres con hipertensión gestacional con inicio antes de la semana 34 de gestación, tienen probabilidad de 35% de desarrollar preeclampsia. CASO CLINICO Paciente femenino de 37 años de edad actualmente cursando su tercer embarazo con 14 semanas por FUM, acude a visita de control prenatal cuenta con 17 SDG por USG, antecedentes personales patológicos de pre-eclampsia, además obesidad previa al embarazo, cuenta con antecedentes familiares de diabetes y obesidad por parte de la madre e hipertensión por parte del padre, el cual fallece por infarto al miocardio. A la exploración física se observa paciente gestante con obesidad grado I, sin tratamiento previo, se envía EGO, QS y BH, donde se observa triglicéridos de 350, colesterol de 290, glucosa de 105 mg/dl, hemoglobina glucosilada de 7, tensión arterial en dos ocaciones de 135/95 mmHg. Tratada con dieta hiposodica, hipocalorica y restricción de liquidos. PREGUNTA Cuál es el factor más importante que presenta la paciente, para considerar Hipertension Arterial Pre-existente. RESPUESTA a.- Trigliceridemia. b.- Hipercolesterolemia. c.- Obesidad. d.- Cifras tencionales altas. CASO CLINICO Se trata de paciente femenino de 26 años de edad la cual acude a consulta de control prenatal actualmente contando con 21 semanas de gestación, refiere que ha presentado cefalea ocasional, mareo y cansancio a la exploración física se observa edema de miembros inferiores, la consulta anterior usted envió estudios de EGO, QS y BH, los datos de relevancia fue Hematocrito de 38, las constantes vitales son de FR 18, FC 89, TA 160/90. MmHg. Con edema leve sin hematuria ni proteinuria. PREGUNTA. Cuál es la conducta a seguir mas adecuada. RESPUESTA a.- Restrición de liquidos y sal. b.- Vigilancia estrecha de tencion arterial. c.- Monitorización de vitalidad fetal. d.- Iniciar con alfametildopa.

CURSO ENARM CMN SIGLO XXI TEL: 36246001

Pharmed Solutions Institute

PÁGINA 326

MANUAL DE TRABAJO DEL CURSO ENARM CMN SIGLO XXI PREECLAMPSIA. CIENCIAS BASICAS: Se define como la aparición de hipertensión y proteinuria después de la semana 20 del embarazo. Se suele acompañar de edemas pero no es necesaria la presencia de éstos para ser diagnosticada. Es una enfermedad característica y propia del embarazo de la que se pueden tratar los síntomas, pero sólo se cura con la finalización del mismo y si no se trata adecuadamente puede ser causa de graves complicaciones tanto para la mujer embarazada como para el feto. CLASIFICACION: Preeclampsia leve; presión arterial de 140/90mmHg o mas o elevación de 30mmHg en la sistólica y 15mmHg en la diastólica, cuando se conocen las cifras basales previas, se presenta después de la semana 20 hasta 30 días posparto, existe proteinuria de más de 300mg/24h, ausencia de síntomas de vasoespasmo. Preeclampsia severa; presión arterial de 160/110mmHg o más, después de la semana 20 hasta 30 días posparto, existe proteinuria mayor de 5g/24h, presencia de cefalea, acufenos, fosfenos, edema generalizado. Inminencia de eclampsia; se establece el diagnóstico cuando después de la semana 20 de gestación, parto o puerperio (no más de 30 días), aparece uno o más de los siguientes datos: Presión arterial sistólica mayor a 185mmHg con presión arterial diastólica >115mmHg, proteinuria mayor a 10g, estupor, pérdida parcial o total de la visión, dolor epigástrico en barra, hiperreflexia generalizada. Preeclampsia recurrente; presencia de cualquiera de los tipos de enfermedad hipertensiva inducida por el embarazo, que aparece por segunda ocasión o más en embarazos consecutivos o no. PATOGENIA: Factores de riesgo: Ausencia o deficiencia de control prenatal, desnutrición, obesidad, intervalo intergenésico menor a dos años, muer menor de 18 y mayor de 35 años, primigesta o nuligesta, preeclampsia/eclampsia en embarazos anteriores o antecdenetes familiares repetidos, hipertensión arterial crónica o cualquier otro trastorno hipertensivo durante lagestacion, sobredistension uterina de cualquier origen (embarazo gemelar o multiple, polihidramnios), infeccion en vías urinarias recurrentes, DM tipo 1 y 2, enfermedad renal previa o recurrente, enfermedad trofoblastica, enfermedades autoinmunes, factores psicosociales. Es un estado de vasoconstricción generalizado secundario a una disfunción en el epitelio vascular, en lugar de la vasodilatación propia del embarazo normal. Ello se asocia a isquemia placentaria desde mucho antes de la aparición del cuadro clínico, en lo que parece ser uno de los orígenes de los factores tóxicos para el endotelio vascular. Dicha isquemia parece ser debida a una deficiente placentación en la que no se produciría la habitual substitución de la capa muscular de las arterias espirales uterinas por células trofoblásticas, que es lo que produce una vasodilatación estable que permite aumentar varias veces el caudal de sangre asegurando así el correcto aporte sanguíneo a la unidad fetoplacentaria, su etiología última que sigue siendo desconocida, aunque vamos identificando factores. Así podemos citar factores hereditarios por vía materna pero también paterna, factores inmunológicos que explican que se produzca más en la primigesta y más si no ha estado expuesta antes al contacto directo con los antígenos paternos, a factores nutricionales como la falta de ingesta de calcio en algunas poblaciones. Todo ello se manifiesta por un desequilibrio entre prostaglandinas vasodilatadoras (prostaciclina) frente a las vasoconstricciones (tromboxano), quizá mediado por la aparición de exceso de producción de citoquinas proinflamatorias (IL-2, TNF), y de producción de lipoperoxidos capaces ambos de e inducir alteraciones endoteliales, esta disfunción endotelial general, que afecta a muchos órganos distintos como son el riñón, el cerebro, el hígado pero en especial al sistema cardiovascular por lo que la hipertensión es su expresión más genuina, que con frecuencia se acompaña de edema. El vasoespasmo y el edema facilitan la reducción del volumen plasmático que lleva a la hemoconcentración característica y a la hipercoagulabilidad. Todo ello hace que exista una hipoperfusión multiorgánica que empeora el cuadro a nivel general y fetoplacentario. DIAGNOSTICO: Se establece el diagnostico de preeclampsia leve cuando se presentan 2 o mas de los siguientes signos: 1) Presión sistólica >140mmHg o elevación >30mmHg sobre la presión habitual. 2) Presión diastólica, >90mmHg o elevación >15mmHg sobre la presión habitual. Proteinuria. Edema. La medición de la presión deberá realizarse con la paciente sentada y requiere de dos tomas consecutivas con un intervalo de 6h o mas en este lapso de horas debe permanecer en reposo. El incremento de proteinuria y de hipertensión en una paciente nefrópata o hipertensa previa se denomina preeclampsia sobreañadida y el manejo clínico es parecido. Se establece el diagnostico de preeclampsia severa cuando se presentan 2 o más de los siguientes signos: 1) presión sistólica >160mmHg y presión diastólica >110mmHg. 2) proteinuria >5g en orina de 24h o su equivalente en tiras reactivas (más de 3+). 3) Oliguria de menos de 500ml/24h. 4) Trastornos cerebrales o visuales. 5) Edema generalizado. Por otra parte también es grave cuando aparece Síndrome de HELLP (plaquetopenia, elevación de enzimas hepáticas y hemolisis), de insuficiencia cardiaca (edema agudo de pulmón), o de insuficiencia renal (creatinina>1,2 mg/dL), o dolor epigástrico. La inminencia de eclampsia se diagnostica con uno o más de los siguientes datos: 1) Presión arterial sistólica >185mmHg con presión arterial diastólica >115mmHg. 2) Proteinuria >10g. 3) Estupor. 4) pérdida parcial o total de la visión. 5) Dolor epigástrico en barra. 6) Hiperrreflexia generalizada. Por la gravedad de esta variedad clínica las pacientes deben ser manejadas como eclámpticas. Todas suelen acompañarse de signos de afectación fetal por insuficiencia placentaria crónica en forma de signos de restricción del crecimiento intrauterino o agudo con signos de Riesgo de Pérdida de Bienestar Fetal. Con el agravante de que la situación fetal suele empeorar al tratar la hipertensión materna grave ya que al descender sus valores se disminuye la perfusión placentaria, y de que los fármacos administrados a la madre dificultan la valoración del estado fetal a través del estudio de la frecuencia cardiaca fetal basal o test no estresante (NST en inglés), por lo que las unidades que traten los casos graves tendrían que disponer de la posibilidad de ecografía Doppler de forma continuada. TRATAMIENTO: Debe recordarse que existe una hemoconcentración a pesar de que puedan existir edemas, por lo que NO deben administrarse diuréticos para tratarlos o para disminuir la TA, excepto en los casos de edema agudo de pulmón o de oliguria. Por la misma razón tampoco se debe hacer una dieta hiposodica estricta. Los mejores resultados actuales se deben a la posibilidad de terminar la gestación en los casos graves a partir de la semana 32 o antes en caso necesario, con buenos resultados perinatales gracias a la posibilidad de madurar los fetos con corticoides (no contraindicados con buen control en la preeclampsia), y especialmente a las mejoras en el manejo neonatal de los prematuros extremos. Preeclamsia leve. Terminar la gestación al llegar a término sin sobrepasar la semana 40. No se ha demostrado que se mejore con reposo, ni con tratamiento hipotensor. Se debe controlar la aparición de signos de gravedad para poder iniciar el tratamiento en el momento oportuno. Antihipertensivos: alfametil-dopa 250-500mgs VO cada 8 h, si es preciso asociados a hidralazina 10-50mgs VO c/8hrs. Preeclampsia grave. En todos los casos se debe: 1) Controlar la TA con hipotensores manteniéndola de ser posible por debajo de los valores indicados como de gravedad, pero por encima de 140/90 para no disminuir la perfusión placentaria en exceso. Se usara, hidaralazina bolo inicial de 5 mg IV si diastólica >110mmHg, repetir en 20 minutos (5-10mg) de acuerdo a respuesta hasta un máximo de 30 mg (de no contar con hidralazina se recomienda nifedipino VO 10mg, repetir cada 10-30min). Labetalol, 20 mg por vía intravenosa, seguidos de 40 a 80 mg cada 10 minutos, hasta una dosis máxima de 220 mg. Alfametil-dopa 500mgs VO cada 6 h, si es preciso asociados a hidralazina 50mgs VO c/6hrs. No usar betabloqueantes (atenolol) ni IECAs, ni ARA-II. Indicado para evitar edema cerebral dexametasona (16mg IV dosis única). 2) Hacer prevención del riesgo de Eclampsia y la posible la hiperreflexia, con sulfato de magnesio (SO4Mg, la dosis de ataque es de 6g. en 100 de dextrosa al 5% pasar en 15 min.

CURSO ENARM CMN SIGLO XXI TEL: 36246001

Pharmed Solutions Institute

PÁGINA 327

MANUAL DE TRABAJO DEL CURSO ENARM CMN SIGLO XXI Continuar con dosis de mantenimiento a 1-2 g por hora administrados en infusión IV continua (preparar sol. Glucosada de 900cc al 5% mas 10 ampolletas de 1g. Pasar 100-200ml de solución por hora igual a 24-48 gotas por minuto). En caso de intoxicación utilizar 1 g de Gluconato de calcio IV diluido en 100cc de solución fisiológica y pasar en 15 min. 3) Vigilar la aparición de otros signos de gravedad Síndrome de HELLP, CID clínica, insuficiencia cardiaca, renal, etc. 4) controlar el bienestar fetal con NST periódico, perfil biofísico y Doppler para comprobar el grado de afectación fetal y si existen signos de redistribución vascular, para poder indicar la extracción fetal antes de la afectación de los vasos venosos, momento que por lo que sabemos hasta ahora, coincide con el inicio de la acidosis fetal. 5) Finalizar la gestación: 1) A termino: en cuanto la situación materna se estabiliza. 2) Pretérmino: >32 semanas, en cuanto se compruebe la madurez pulmonar fetal o antes si hay indicación materna o fetal. 3) Pretérmino 110mmHg, repetir en 20 minutos (5-10mg) de acuerdo a respuesta hasta un máximo de 30 mg (de no contar con hidralazina se recomienda nifedipino VO 10mg, repetir cada 10-30min). Labetalol, 20 mg por vía intravenosa, seguidos de 40 a 80 mg cada 10 minutos, hasta una dosis máxima de 220 mg. 3) Tratamiento anticonvulsivante con SO4Mg, la dosis de ataque es de 6g. en 100 de dextrosa al 5% pasar en 15 min. Continuar con dosis de mantenimiento a 1-2 g por hora administrados en infusión IV continua (preparar sol. Glucosada de 900cc al 5% mas 10 ampolletas de 1g. Pasar 100-200ml de solución por hora igual a 24-48 gotas por minuto), ajustando los niveles a 4.8-9.6 mg/dl de magnesemia, siendo necesario mantener los controles clínicos continuados (reflejo patelar, respiraciones /minuto, diuresis) para evitar que una sobredosificación pueda producir un paro cardiorrespiratorio. Esta medicación se ha comprobado en estudios prospectivos y randomizados más eficaz que cualquier otra, pero los límites terapéuticos y tóxicos están muy cercanos. También está indicado el uso de diazepam, como droga de segunda elección, que actuaría disminuyendo el consumo de oxigeno por el tejido cerebral, pero que posee efectos nocivos sobre el feto (depresión respiratoria, hipotonía). Las dosis recomendada son: bolo de 10 mg por vía IV y luego continuar con 50 mg en 500 cc de solución dextrosada al 5%. 4) Acabar la gestación en cuanto la situación clínica materna este estabilizada y lo permita. COMPLICACONES: De preeclampsia/eclampsia; Sindrome de HELLP. Desprendimiento prematuro de placenta. Insuficiencia renal aguda. Edema pulmonar agudo. Edema cerebral. Coagulacion intravascular diseminada. Ruptura hepática (Raro, alta letalidad, se manifiesta por dolor epigastrico en barra o en el cuadrante superior derecho, irradiado a hombro, región iterescapulovertebral derecha, en hemicinturon, pudiendogeneralizarse a todo el abdomen). Hemorragia cerebral. CASO CLINICO Se trata de paciente de 19 años de edad la cual acude por primera vez a consulta por amenorrea secundaria de 30 semanas por FUM, acude hasta ahora ya que desconocían su embarazo en casa, la paciente refiere mareo, dolor abdominal en barra, opresión de torax, cefalea e irritabilidad, a la exploración física se observa escleras hiperemicas, leve rubicundez facial, rot´s incrementados y edema de miembros inferiores, proteinuria, se mide fondo uterino con 28 cm, con disminución de actividad fetal, los signos vitales de la madre fueron FR 21, FC 96, TA 130/100 en 3 ocaciones. PREGUNTA Cuál es la conducta inmediata a seguir con la paciente. RESPUESTA a.- Identificar la viabilidad fetal. b.- Busqueda de síndrome de HELLP. c.- Sulfato de magnesio 4 g IV. d.- Prepara a la paciente para Cesarea.

CURSO ENARM CMN SIGLO XXI TEL: 36246001

Pharmed Solutions Institute

PÁGINA 328

MANUAL DE TRABAJO DEL CURSO ENARM CMN SIGLO XXI CASO CLINICO Se trata de femenino de 27 años con 32 semanas de gestación que llega al servicio de urgencias por presentar cefalea intensa, acufenos y fosfenos, dolor abdominal en barra, además dolor en región lumbosacra y sensación de pesantes pélvica, a la exploración física se observa fondo uterino de 28 cm, obesidad grado I, ROT´s incrementado, edema de miembros inferiores +++, al tacto observa dilatación de 2 cm, con presencia de moco con sangre, se observan proteínas en orina ++, nivel de conciencia disminida, leve desorientación temporo-espacial, el familiar refiere que la encontró en el suelo con somnolencia. PREGUNTA Cuál es la conducta a seguir de forma inmediata. RESPUESTA a.- Indica Sulfato de Magnesio. b.- Indica Hidralacina. c.- Preparar para Cesarea. d.- Preparar para Parto. CASO CLINICO Se trata de femenino de 18 años de edad con 32 semanas de gestación por fecha de última menstruación, sin atención prenatal, es llevada a urgencias por presentar en casa perdida del estado de alerta y con movimientos tipo crisis convulsivas, a la exploración se observa somnolienta con respuesta a estímulos dolorosos, ha sido estabilizada en sus constantes vitales, los resultados de laboratorios son los siguientes, plaquetas de 45,000, proteinuria +++, DHL de 590, AST 239, ALT 432. PREGUNTA Tomando en cuenta la gravedad del caso, cual es la mejor conducta a seguir. RESPUESTA a.- Realizar Cesárea. b.- Sulfato de magnesio. c.- Estabilizar plaquetas. d.- Maduración Pulmonar.

CURSO ENARM CMN SIGLO XXI TEL: 36246001

Pharmed Solutions Institute

PÁGINA 329

MANUAL DE TRABAJO DEL CURSO ENARM CMN SIGLO XXI DIABETES MELLITUS, INTOLERANCIA A LA GLUCOSA, DIABETES GESTACIONAL. GENERALIDADES: Cambios fisiológicos en el embarazo normal: El embarazo normal se considera un estado diabetogénico o de resistencia progresiva al efecto de la insulina, durante el primer trimestre y las etapas iniciales del segundo se eleva la sensibilidad a la insulina, lo que se ha atribuido a las mayores concentraciones de estrógenos circulantes. Este fenómeno incrementa el depósito de energía, sobre todo en el tejido adiposo, con expansión del mismo; pero a partir de las 24 a 28 semanas de gestación aumenta paulatinamente la resistencia a la insulina, que puede alcanzar los niveles que se observan en pacientes diabéticos tipo 2. Más evidente al final del segundo trimestre del mismo; el organismo de la madre se prepara almacenando nutrientes y en esta etapa la sensibilidad a la insulina es mayor; sin embargo, en la segunda mitad de la gestación hay una resistencia progresiva al efecto de la insulina, lo que causa una liberación de los nutrientes (glucosa, los aminoácidos, los ácidos grasos los triglicéridos y los oligoelemntos) para que el feto tenga un desarrollo adecuado. Las células β del páncreas elevan la secreción de insulina en un intento de compensar la resistencia a la insulina del embarazo, lo que origina pequeños cambios en la concentración de insulina en el curso de la gestación, comparados con los grandes cambios en la sensibilidad de la misma. El músculo esquelético es el sitio principal para utilizar la glucosa corporal, y junto con el tejido adiposo, empiezan a ser resistentes al efecto de la insulina, lo que es más evidente durante la segunda mitad del embarazo. Un embarazo normal se caracteriza por aproximadamente un 50% de disminución en la disponibilidad de glucosa mediada por insulina. Barbour señala un incremento en la secreción de insulina hasta de 200% para tratar de mantener euglucémica a la madre. Una gran cantidad de sustancias producidas por la placenta y por los adipocitos, (lactógeno placentario, hormona placentaria de crecimiento, prolactina, hormona liberadora de corticotropina-cortisol, insulinasa, FNT y adipocitocinas [leptina, resistina, visfatina, adiponectina]), son las que reprograman la fisiología materna y causan este estado de resistencia a la insulina para dirigir los nutrientes hacia el feto en desarrollo, sobre todo en la segunda mitad del embarazo. El lactogeno placentario se eleva unas 30 veces durante la gestación. En los obesos hay una correlación positiva entre el FNTa y el IMC e hiperinsulinemia. RESISTENCIA A LA INSULINA: En esta resistencia a la insulina participan numerosas sustancias producidas por la placenta y por los adipocitos, entre las cuales destacan la hormona de crecimiento placentaria y el FNTa, que por diversos mecanismos disminuyen el efecto de la insulina a nivel intracelular. La primera aumenta la cantidad de la subunidad p85a de la PI-3K (fosfatidil-inositol 3 cinasa), lo que al final disminuye un factor de trascripción AKT que altera el desplazamiento de los transportadores de glucosa (GLUT) hacia la membrana celular y disminuye la entrada de glucosa a la célula; el segundo altera la fosforilación de la subunidad beta del receptor de insulina y del IRS-1, ya que fosforila residuos de serina y treonina en lugar de los de tirosina. Este evento conduce a la mayor degradación de ambos y modifica la acción de la insulina. Otro mecanismo implicado es que en el musculo esquelético el GLUT-4 esta disminuido en el tejido adiposo en mujeres embarazadas, y es aún menor en la diabetes mellitus gestacional; además, está alterada la traslocación de estos trasportadores. Los cambios moleculares en el adipocito durante el embarazo muestran reducción en la trascripción del PPARg, receptor nuclear que regula la trascripción de varios genes centrales en el metabolismo del adipocito (adiponectina, la lipoprotein lipasa, la proteína P-2 fijadora de ácidos grasos intracelulares y la proteína no acoplada mitocondrial). Barbour halló hasta 40 a 50% de reducción en el ARNm del PPARg (un factor que suprime considerablemente a este receptor nuclear es el FNTa) en embarazadas obesas y DM gestacional; esto altera la concentración de ácidos grasos en la sangre, y las lipoproteínas ricas en triglicéridos (VLDL) con mayor resistencia para el efecto de la insulina ya que la adiponectina es un sensibilizador de esta hormona y estos ácidos grasos entran a la célula como fuente energética. SALUD PUBLICA: La prevalencia de diabetes gestacional a nivel mundial se ha estimado en 7%. En México se ha mencionado una frecuencia que varía entre 4 y 19% de la población obstétrica. Respecto a la edad de la madre, se ha señalado que la incidencia es de 0.4 a 0.5% en menores de 25 años y de 4.3 a 5.5% en mayores de esa edad. Lo cierto es que la frecuencia de este trastorno se ha duplicado en la última década, en forma paralela a la llamada pandemia metabólica que afecta a las sociedades modernas. Más de 90% de los casos de diabetes que complican a un embarazo son casos de diabetes gestacional. La creciente prevalencia de diabetes, que se diagnostica en mujeres a edades tempranas, favorece la presencia de diabetes y embarazo. Las mujeres que desarrollan diabetes gestacional tienen alto riesgo de desarrollar diabetes tipo II, en los 10 años que siguen al embarazo. PATOGENIA: Factores de riesgo: 1) Edad mayor de 25 o 30 años. 2) IMC > 25 kg/m2 SC. 3) Antecedentes de hijos macrosómicos. 4) Diabetes mellitus en familiares de primer grado 5) Antecedentes de intolerancia a la glucosa. 6) Glucosuria. 7) Ganancia de más de 20 kg de peso en la actual gestación. 8) Antecedentes de problemas obstétricos serios. 9) Peso bajo del feto al nacimiento ( del 90o percentil). 11) Origen étnico de riesgo alto de obesidad o diabetes mellitus. Los más importantes son: mayor edad en la madre, familiares de primer grado con diabetes y mayor índice de masa corporal pregestacional. En la segunda mitad del embarazo se requiere un estado fisiológico de resistencia a la insulina para dirigir los nutrientes almacenados en la madre hacia la unidad fetoplacentaria y dar un crecimiento adecuado al feto; sin embargo, cuando las mujeres desarrollan diabetes mellitus gestacional, la resistencia a la insulina es más acentuada, lo cual modifica el medio intrauterino y causa crecimiento acelerado del feto, con riesgo elevado de macrosomía. Los datos disponibles en la bibliografía muestran un aumento lineal en el índice de complicaciones en el feto conforme se eleva la cifra de la glucosa en la sangre materna: índices de mortalidad del feto de 4% cuando la glucosa es menor de 100 mg y hasta de 24% cuando la cifra excede 150 mg. DIABETES PREGESTACIONAL: o preexistente se refiere a pacientes conocidas con diabetes mellitus tipo I o II que se embarazan. Los hijos de mujeres con diabetes pregestacional tienen alto riesgo de sufrir malformaciones estructurales congénitas, aborto espontaneo y restricción del crecimiento intrauterino, lo que se debe a que la hiperglucemia y la hiperinsulinemia en el feto pueden ser teratogénicas, ya que existe una fuerte asociación entre un pobre control glucémico en el periodo periconcepcional y el riesgo de tales malformaciones. La incidencia de riesgo de estas malformaciones es de 6 a 8% y la mayoría ocurre en el sistema nervioso central, el corazón, el sistema urogenital y el tubo gastrointestinal. DIABETES GESTACIONAL: Es un padecimiento caracterizado por la intolerancia a los carbohidratos con diversos grados de severidad, que se reconoce por primera vez durante el embarazo y que puede o no resolverse después de este. Existe mayor riesgo de macrosomía y polihidramnios. Se incrementa el riesgo de prematurez, preeclamsia/eclampsia, trauma obstétrico, cesárea y mortalidad perinatal. DIAGNOSTICO: Escrutinio; prueba de escrutinio con 50 g de glucosa oral entre las semanas 24 a 28 de la gestación. Una cifra de 140 mg con diabetes mellitus gestacional o bien 130 mg a la hora, que identificaría al 90%. Sin embargo, a pesar de tener la mayor sensibilidad (79%) y especificidad (87%) de todas las pruebas de escrutinio disponibles, ésta se reserva para pacientes con alto riesgo para padecer la enfermedad, más que para la población general. Existen 4 formas de realizar el diagnostico de Diabetes Gestacional: 1) Glucemia en ayuno >126mg/dl en dos ocasiones. 2) Glucemia casual >200mg/dl. 3) Prueba de tamiz con 50g con resultado >180mg/dl (170mg/dl en mujeres mayores de 30 años). 4) Curva de tolerancia a la glucosa con 100g o 75 g. Se realiza el diagnostico al tener alterados 2 o más de los siguientes valores: ver cuadro anexo.

CURSO ENARM CMN SIGLO XXI TEL: 36246001

Pharmed Solutions Institute

PÁGINA 330

MANUAL DE TRABAJO DEL CURSO ENARM CMN SIGLO XXI En caso de tener alterado un solo valor se diagnostica intolerancia a los carbohidratos. COMPLICACIONES MATERNO-FETALES: En el feto; Macrosomía, muerte intrauterina, asfixia neonatal, distocia de hombro y lesión nerviosa, hipoglucemia, hipocalcemia, ictericia, SIRPA, Taquipnea transitoria, cardiomiopatía (hipertrofia septal), eritrocitosis, trombosis, sobre todo de la vena renal. Cuando son adultos: obesidad, diabetes mellitus, alteraciones neuropsicológicas. En la madre; Preeclampsia, eclampsia, polihidramnios, desgarros perineales, mayor número de cesáreas, riesgo de padecer diabetes mellitus II, parto prematuro. TRATAMIENTO: La meta terapéutica de glucosa sanguínea durante el embarazo es: Glucemia central en ayuno entre 60-90mg/dl y menor de 140mg/dl una hora pospandrial o menor a 120mg/dl a las 2 h pospandrial. El objetivo fundamental del tratamiento es mantener la normoglucemia con una dieta adecuada, reduciendo el consumo de carbohidratos a 40-45% de la ingestión calórica total. Las pacientes con IMC superior a 30 kg/m2 de SC deben disminuir su ingestión calórica en 30 a 33% o alrededor de 25 kcal/kg de peso corporal. El peligro de la restricción calórica muy estricta en la embarazada es la producción de cetosis, que puede alterar el desarrollo psicomotor del feto. La actividad física adaptada a la fisiología materna de acuerdo con la edad gestacional. Si con los dos recursos terapéuticos señalados no se obtiene un control adecuado, será necesaria la administración de insulina en forma juiciosa para mantener cifras de glucosa dentro de límites normales para la mujer embarazada. Se debe evitar un tratamiento intenso que cause hipoglucemia, lo que ocurre hasta en 60% de las pacientes, en muchos casos asintomática para la madre, pero con diversas repercusiones para el feto. En pacientes con insulina la dieta fraccionada con una colación nocturna, disminuye la probabilidad de hipoglucemias. Los análogos de insulina de acción rápida lispro y aspart son seguros en el embarazo y tienen algunos beneficios comparados con la insulina rápida, como, menos episodios de hipoglucemia, un mejor control de la glucemia postpandrial y mayor satisfacción del paciente. La insulina NPH es también segura y se utiliza en combinación con las anteriores. El esquema de tratamiento con insulina puede consistir en: Se indica el fraccionamiento de la dosis de insulina en dos aplicaciones cuando: 1) Continua glucemia en ayuno elevada a pesar de incremento de la dosis de insulina matutina. 2) Se requiere más de 20 U de insulina para lograr control metabólico. 3) Se adiciona insulina rápida cuando no se alcanza la meta terapéutica de la glucemia pospandrial. Esquema: Combinación de insulina rápida e intermedia administrada 30min antes del desayuno y de la cena. Los requerimientos varían de 0.3-1.5 UI por kg de peso real y de acuerdo a descontrol glucémico, así como a la presencia de sobrepeso, edad gestacional o enfermedades que modifiquen la glucemia como la hiperémesis gravídica o infecciones. La dosis total calculada se dividirá en 2/3 pre-desayuno y 1/3 pre-cena (30 min antes de los alimentos). La razón será NPH/rápida 2/1 pre-desayuno y 1/1 precena. Se recomienda iniciar con la mínima dosis de insulina e incrementarla gradualmente de acuerdo al automonitoreo con glucemias capilares pre y postpandriales, por lo menos 3 veces al día. Los análogos de insulina deben ser utilizados hasta que se disponga de mayores datos o exista respuesta a las interrogantes que aún existen acerca de su uso, como: teratogenicidad, incremento en el daño a la retina o incluso a la formación de anticuerpos y reactividad inmunológica de la insulina. Sólo dos de los fármacos hipoglucemiantes orales han sido utilizadas: una es un secretagogo de insulina (glibenclamida) y otro un sensibilizador de insulina (metformina). Aunque diversos grupos de expertos restringen su uso, los datos disponibles muestran que en algunas se logra un control adecuado, además que se encuentran dentro de la clasificación C o D, lo que significa que nos son seguros. La elevación posprandial de la glucosa causa alteraciones en el feto. Un fármaco que puede utilizarse es la acarbosa, que se absorbe en muy poca cantidad y, según algunos trabajos, ha logrado mejor control de la glucemia posterior a la ingestión de alimentos. El control adecuado de la glucosa sanguínea es el objetivo más importante del tratamiento de la diabetes mellitus gestacional y, en teoría, este control óptimo disminuiría la morbilidad y mortalidad maternofetal. PREVENCION: La Federación Internacional de Diabetes y la Asociación Americana de diabetes proponen los siguientes puntos para el consejo preconcepcional: 1) En toda mujer en edad fértil en etapa reproductiva con diabetes, identificar anualmente por interrogatorio directo el deseo de embarazo, en caso de no desearlo proporcionar consejo sobre método anticonceptivo. 2) En la eta preconcepcional y de embriogénesis, lograr un control glucémico estricto con una cifra ideal de Hb glicosilada menor a 6.1%, evitando la presencia de hipoglucemia, para disminuir la probabilidad de malformaciones. 3) Con base, en que a mayor daño vascular mayor riesgo de complicaciones materno-fetales, definir el riesgo establecido si existe daño por micro o macroangiopatia utilizando la clasificación de Priscilla White. 4) En caso de diabetes tipo I, determinar la función tiroidea asociada, ya que hasta un 17% pueden cursar con esta co-morbilidad. 5) En el momento en que se establece el diagnostico de embarazo, suspender hipoglucemiantes orales o insulina glargina si la paciente los estaba utilizando. 6) Suspender los IECA y ARA II, si la paciente los estaba utilizando y usar alfametildopa, nifedipina, labetalol o hidralazina. 7) Mantener las cifras de tensión arterial por debajo de 130/80mmHg. 8) Suspender estatinas y fibratos. 9) Suspender el consumo de alcohol y tabaco. 10) Iniciar ácido fólico tres meses antes del embarazo. No es recomendable el embarazo si la mujer diabética presenta cualquiera de las siguientes situaciones: HbA1c >10%. Cardiopatía isquémica. Nefropatía avanzada (depuración de creatinina 1.4mg/dl). Retinopatía proliferativa. CASO CLINICO Paciente femenino de 34 años de edad la cual acude a consulta debido a que presenta amenorrea de 8 semanas de gestacion, agrega que ha presentado leve fatiga, anorexia con nausea matinal, usted envía laboratorios de rutina presentando glucosa en sangre de 200 mg/dl y hemoglobina glucosilada de 7.5 refiere la paciente que fue diagnosticada con diabetes hace 5 años por lo que recibe Tolbutamida, además recibe captopril por hipertensión arterial diagnosticada hace 3 años. Al interrogatorio la paciente refiere antecedentes de madre diabética e hipertensa, sus antecedentes GO refiere menarca a 13 años, gesta 3, para 2, abortos 0, además refiere que su embarazo anterior hace 5 años presento un incremento de 14 kilogramos con producto que preso 3,750 grs. Por vía cesárea a las 36 semanas de gestación. PREGUNTA Cuál es la conducta menos adecuada a seguir. RESPUESTA a.- Suspender hipoglucemiente. b.- Suspender IECA. c.- Indicar interrupción de embarazo.

CURSO ENARM CMN SIGLO XXI TEL: 36246001

Pharmed Solutions Institute

PÁGINA 331

MANUAL DE TRABAJO DEL CURSO ENARM CMN SIGLO XXI d.- Indicar alfametildopa. CASO CLINICO Paciente femenino de 21 años de edad, la cual presenta 24 semanas de gestación, acude a control prenatal, refiere ardor al orinar, deseos continuos de orinar, con urgencia, usted identifico un incremento de peso hasta el momento a partir de su embarazo de 10 kilos. Sus antecedentes presentan, obesidad previa al embarazo, padre diabético actualmente con insuficiencia renal en tratamiento con hemodiálisis, madre hipertensa bajo tratamiento. PREGUNTA Considerando el cuadro clínico cuales son las complicaciones que el producto puede presentar. RESPUESTA a.- Producto Macrosomico. b.- Diestres respiratorio. c.- Hipoglucemia. d.- Parto pretermino. PARTO PRETERMINO. CIENCIAS BASICAS: El diagnóstico de parto pretérmino se obtiene en pacientes con ≤37 semanas (o un peso igual a mayor de 500g, que respira o manifiesta signos de vida) y que presentan contracciones dolorosas y regulares que ocurren al menos cada 10 minutos. Esto debe estar asociado con dilatación cervical y/o descenso. El diagnóstico se realiza al demostrar una dilatación progresiva del cérvix debido a la presencia de contracciones entre las semanas 20-37, lo que incluye hasta la semana 37. La definición de puede dividir a saber: Pretérmino: 33-37 semanas. Pretérmino moderado: 28-32 semanas. Pretérmino extremo: 20-27 semanas. SALUD PUBLICA: El parto pretérmino es la mayor causa de mortalidad perinatal en el mundo desarrollado y ocurre en aproximadamente un 7% de todos los nacimientos. Los niños pretérmino presentan causas severas de morbilidad como dificultad respiratoria del recién nacido, hemorragias intraventriculares, displasia broncopulmonar y enterocolitis necrotizante. También se pueden presentar complicaciones a largo plazo como parálisis cerebral, perdida visual y auditiva. Cerca de un 30% de los parto pretérmino son idiopáticos y espontáneos. PATOGENIA: La incidencia del parto pretérmino es generalmente más alta en los grupos socioeconómicos bajos en comparación con los grupos de mayor ingreso económico. De manera que se han establecido relaciones entre la incidencia de amenaza de parto, parto pretérmino y algunas condiciones asociadas con los problemas socioeconómicos, a saber: Bajo cumplimiento de la consulta prenatal, edades maternas extremas (adolescentes y madre añosa), fumado, condiciones laborales extremas, pobre educación materna, estrés en la vida diaria, ansiedad relacionada al embarazo, violencia doméstica. Ante la presencia de un parto pretermino se debe buscar de forma intencionada la presencia de infecciones a través de urocultivo y cultivo de secreción vaginal con el objetivo de especificar el uso de antibiótico. Es necesario corroborar por USG la edad gestacional. Para establecer el pronóstico de sobrevida del neonato y sus posibles complicaciones. El factor de riesgo más importante para predecir parto pretermino es el antecedente de parto pretermino. Las pacientes con antecedente de parto pretermino, deben recibir consejo preconcepcional o atención medica temprana en el embarazo. DIAGNOSTICO: No hay datos clínicos específicos y la exploración de cérvix es subjetiva e inexacta. Identifique los factores de riesgo de parto pretérmino en la historia clínica de las mujeres en control. Algunos síntomas tempranos sugestivos de amenaza de parto pretérmino son: Dolor abdominal bajo y/o calambres, dolor lumbar, presión pélvica, flujo vaginal aumentado, manchado o sangrado. Síntomas definitivos son: actividad uterina regular acompañada de borramiento y dilatación del cérvix. Criterios diagnósticos: A) actividad uterina regular entre las 20 y 37 semanas de gestación más contracciones uterinas clínicamente documentadas (1/10 minutos, 4/20 minutos o 6/60 minutos). B) Dilatación cervical igual o mayor a 2cm. C) Borramiento cervical igual o mayor a 80%. Laboratorio y gabinete: la evaluación ultrasonografica de la longitud cervical y prueba de fibronectina fetal tienen elevado valor predictivo negativo; por lo tanto, el uso único o de ambas, permite determinar qué tipo de pacientes no requieren tocolisis. En pacientes con riesgo de parto pretermino la medición de la longitud cervical se realizara entre la semana 20.1-34 de gestación y fibronectina fetal entre las semanas de 24-34 de gestación. La amniocentesis puede ser usada en mujeres con amenaza de parto pretermino para valorar la madurez pulmonar fetal e infección intraamniotica. FIBRONECTINA FETAL: La FNf es una glicoproteína producida por la membrana coriónica y está localizada en la decidua basal adyacente al espacio intervelloso. Su objetivo principal parece el de una molécula de adhesión (pegamiento), la cual favorece la unión entre las membranas corionicas con la decidua materna contigua. Puede ser encontrada, normalmente, en SCV en las primeras 22 SDG. Virtualmente, nunca es encontrada en la misma secreción entre las 24 y 34 SDG, a menos que el cérvix haya desarrollado borramiento y dilatación prematura, generalmente en asociación con contracciones uterinas sintomaticas. Existe una fuerte asociación entre la expresión de FNf en SCV y PP.(1,2,7) Numerosos ensayos han mostrado una asociación entre la presencia de FNf y nacimientos pretérmino, así como una reducción en el riesgo de nacimientos pretérmino cuando el resultado de esta prueba es negativo Los beneficios con el uso de la prueba de FNf(1) son: Disminución de ingresos hospitalarios y de la duración de los días de hospitalización. Identificación más apropiada de pacientes que necesitan esteroides y tratamiento con agentes tocolíticos. Disminución del uso de agentes tocolíticos en pacientes que no están en riesgo de nacimiento pretérmino. Reducción del estrés y ansiedad para la mujer embarazada y su familia debido a la tranquilidad de no realizar una transportación u hospitalización innecesaria. Las indicaciones para FNf son: embarazos de 24 a 34 SDG, síntomas de parto pretérmino (contracciones uterinas regulares >6 por hora y/o presión pélvica) membranas amnióticas intactas, cérvix 45 mmHg. c.- HCO3 > 40mEq/L. d.- SpO2 < 95. PREGUNTA Se realizaron electrolitos sericos por conituar con rechazo al alimento y secresion moderada oral, todos los resultados resultaron dentro de parámetro normales, cual de las siguientes diagnosticos diferencias del acidosis respiratoria es el mas probable presentarse? RESPUESTA a.- Ventilación inadecuada alveolar.

CURSO ENARM CMN SIGLO XXI TEL: 36246001

Pharmed Solutions Institute

PÁGINA 346

MANUAL DE TRABAJO DEL CURSO ENARM CMN SIGLO XXI b.- Desordenes musculares. c.- Defectos pulmonares. d.- Trastornos de las vías aéreas. PREGUNTA Considerando los valores gasométricos y de eletrolitos, cual es la causa de la modificación del anio gap, del caso? RESPUESTA a.- Error de laboratorio. b.- Toxinas no identificadas. c.- Mielomas. d.- Exceso de soluciones. PREGUNTA El paciente presenta persistencia de los síntomas por 3 horas más, se reporta que incrementa con la alimentación, con dificultad continua succionando, usted considera realizar laboratorios, cual de los siguientes resultados es el menos probable encontrar en este caso? RESPUESTA a.- Aumento de la trama vascular. b.- Presencia de infiltrados pulmonares. c.- Acidosis respiratoria. d.- Hipoxemia leve.

CURSO ENARM CMN SIGLO XXI TEL: 36246001

Pharmed Solutions Institute

PÁGINA 347

MANUAL DE TRABAJO DEL CURSO ENARM CMN SIGLO XXI SINDROME DE DIFICULTAD RESPIRATORIA (ENFERMEDAD DE MEMBRANA HIALINA). CIENCIAS BASICAS: El síndrome de dificultad respiratoria por deficiencia de surfactante es la principal patología respiratoria entre los recién nacidos; ocupa un papel preponderante por su alta morbimortalidad. Ha recibido diversos nombres a través del tiempo siendo los más com nmente reportados, “síndrome de insuficiencia respiratoria idiomática”, “síndrome de hipoperfusión pulmonar” y “síndrome de microatelectasias m ltiples”. El término de síndrome de dificultad respiratoria tipo I, se otorgó por las alteraciones radiológicas presentadas en esta patología, el Doctor Swischuk reportaba en las radiografías de esta patología pequeñas burbujas de tipo esférico de 1 a 1.5 mm. de diámetro a las cuales denomino burbujas tipo I, en otras patologías encontró a nivel radiográfico burbujas de mayor tamaño a las que denomino burbujas tipo II y otras de tamaño mucho mayor (quísticas) las denomino tipo III. El término más aceptado actualmente es el de síndrome de dificultad respiratoria por deficiencia de surfactante, ya que define sus principales características fisiopatológicas como la congestión difusa pulmonar, notoria disminución de la distensibilidad pulmonar y la presencia de tejido necrótico y membranas de aspecto hialino en bronquiolos y alvéolos. SALUD PUBLICA: Este síndrome continúa siendo una de las primeras causas de ingreso a las unidades de terapia intensiva neonatal a nivel mundial. La incidencia exacta de esta patología es difícil de precisar. Las tendencias actuales reportan incidencia de 60 a 80% en los recién nacidos menores de 30 semanas de gestación, un 15 a 30% de los de 32 a 36 semanas de gestación y solo 5% de los que tienen 37 semanas o más de gestación. La frecuencia también aumenta cuando son hijos de madre diabética (de evolución corta la diabetes materna) en asfixia perinatal, y algunos casos en que por error en la determinación de edad gestacional, se realiza operación cesárea antes de lo debido. Discreta tendencia a ser más frecuente en recién nacidos de sexo masculino que en sexo femenino. PATOGENIA: En el síndrome de dificultad respiratoria, la frecuencia respiratoria se encuentra elevada por lo que a pesar del volumen corriente (Vt) disminuido, la ventilación minuto inicialmente esta incrementada. Debido a la deficiencia o disminución en la cantidad o calidad del surfactante pulmonar la mayor parte del pulmón está colapsado o poco ventilado y la mayor parte de la ventilación alveolar se deriva a una región muy pequeña del pulmón lo que conlleva a una disminución de la capacidad residual funcional (CRF). Asimismo la distensibilidad está muy disminuida, no tanto por el tórax del recién nacido que es fácilmente distensible, sino que por los pulmones que con esta deficiencia de surfactante llegan a tener mediciones de la distensibilidad de 0.3-0.6 ml/cmH2O/kg en lugar de 2.0-2.5 ml/cmH2O /kg que es lo normal, lo cual traduce una distensibilidad menor al 30% de los normal. La resistencia de la vía aérea es normal pero con tendencia a incrementarse hasta en 40-50% como resultado de todo esto el trabajo respiratorio se verá incrementado en poco más del 50%. Edberg y colaboradores encontraron disminución de la distensibilidad, incremento de la resistencia, reducción del volumen pulmonar y disminución en la eficacia para la mezcla de gases en los recién nacidos prematuros con síndrome de dificultad respiratoria. A partir de estos datos y dado que la constante de tiempo (CT) depende de la distensibilidad y la resistencia (CxR=CT) , se verá afectada y como consecuencia se presentará un inadecuado intercambio del volumen alveolar. Esta disminución o acortamiento de la constante de tiempo no es uniforme en toda la vía respiratoria, se aprecia sobre todo en las áreas más distales, por lo que , en un mismo pulmón habrá constantes de tiempo diferentes lo que lleva a una ventilación no uniforme con riesgo de dañar a aquellas vías aéreas con constantes de tiempo normales, que son sometidas a la ventilación mecánica necesaria para forzar a abrirse a las vías aéreas con constante de tiempo acortada. DIAGNOSTICO: Los signos pueden manifestarse desde los primeros minutos de vida o después de algunas horas, y por lo general son de incremento gradual. Aunque en ocasiones estos signos de dificultad respiratoria son menos marcados debido a la debilidad de la musculatura respiratoria, lo que los llevará rápidamente a una falla respiratoria con hipoventilación y apna (conocido como respuesta paradójica a la hipoxemia). Los signos de dificultad respiratoria se manifiestan con taquipnea, tiraje intercostal, retracción xifoidea, disociación toracoabdominal, aleteo nasal, y quejido espiratorio, este último uno de los más frecuentes y es motivado por el cierre de la glotis en su afán de realizar un auto PEEP (presión positiva al final de la espiración) para conservar los alvéolos abiertos y aumentar el volumen residual pulmonar para un adecuado intercambio gaseoso. A la auscultación de campos pulmonares encontraremos disminución del murmullo vesicular habitualmente en forma bilateral. Además de los signos de dificultad respiratoria se puede presentar cianosis central que obligará al uso de oxigenoterapia en cualquiera de sus modalidades y que podrá variar desde los casos leves que solo requieran apoyo con oxígeno en fase de casco cefálico o bien casos moderados y severos que requerirán CPAP (presión positiva continua en vías aéreas) o ventilación mecánica en su diversas modalidades y según lo requiera cada paciente. Otro grupo de manifestaciones clínicas estará determinado por los trastornos a nivel hemodinámico que se derivan de los cortos circuitos de derecha a izquierda así como por la repercusiones por la acidosis y la hipoxemia, todo esto conllevará a trastornos de perfusión a todos los niveles con repercusiones y manifestaciones clínicas de cada uno de los órganos afectados (falla renal, falla cardiaca, trastornos por hipoperfusión a nivel cerebral, intestinal, etc.). La misma hipoxemia favorecerá incremento en las resistencias pulmonares y como consecuencia datos de hipertensión pulmonar con lo que se agravará la hipoxemia y acidosis, y ameritará manejos más enérgicos para poder restituir la oxigenación adecuada de todos los tejidos. La misma prematurez de estos pacientes favorece más complicaciones como la hipotermia que deberá ser corregida en forma oportuna ya que de lo contrario generará acidosis y está a su vez vasoconstricción e hipoxemia. El diagnóstico se podrá sustentar en base al cuadro clínico así como a los hallazgos a nivel laboratorial y radiográfico. Después del nacimiento, el surfactante pulmonar puede evaluarse en el líquido amniótico, ya que parte del líquido pulmonar fetal entra en la bolsa amniótica y por lo tanto medirse la lecitina de manera conjunta con la SP-C, pero con algunos cambios en la cantidad de líquido amniótico puede alterar la concentración de la SP-C. Otra prueba de laboratorio que puede ser utilizada es la relación entre la lecitina y la esfingomielina que permanece relativamente constante a lo largo de toda la gestación y se expresa como el índica L/S el cual se incrementa en forma lenta y gradual de la primera a la semana 32 de gestación, el índice es de 2 hacia la semana 28 y de l hasta cerca del término de la gestación; la incidencia de síndrome de dificultad respiratoria es sólo de 0.5% cuando el radio es de 2, pero es cercana al 100% si el radio es menor de 1; entre 1 y 2 , el riesgo disminuye de modo progresivo. También resulta útil la medición de los niveles de fosfatidilinositol el cual aumenta rápidamente en el líquido amniótico hasta la semana 36 y después disminuye, por medio de cromatografía se puede determinar su porcentaje del total de los lípidos, si es menor de 1% se correlaciona con síndrome de dificultad respiratoria. Estas dos últimas pruebas de laboratorio son los mejores predictores de esta patología con una correlación conjunta del 80%.También se han realizado mediciones de la SP-A y SP-C pero sus resultados no son tan confiables. Por otra parte, sin que sean indicadores del síndrome de dificultad respiratoria, se deberá determinar estudio gasométrico el cual nos mostrará diversos grados de acidosis, hipoxemia e hipercapnia, los cuales nos indicaran respuesta al manejo instalado. El estudio por medio de radiografías de tórax nos reportara gran utilidad, y nos mostrará un aumento difuso de la densidad en ambos campos pulmonares con una apariencia granular muy fina dando la apariencia de un “vidrio esmerilado” o de “vidrio despulido” , así como también se aprecian a las vías aéreas con mayor densidad que

CURSO ENARM CMN SIGLO XXI TEL: 36246001

Pharmed Solutions Institute

PÁGINA 348

MANUAL DE TRABAJO DEL CURSO ENARM CMN SIGLO XXI los campos pulmonares produciendo una imagen de broncograma aéreo y los diafragmas se muestran habitualmente elevados , todos estos cambios se deben a la pérdida de volumen pulmonar por colapso alveolar. TRATAMIENTO: Debe ser integral respetando siempre la secuencia recomendada y altamente probada como es: pasos iniciales de la reanimación, ventilación, compresión torácica, así como la intubación y uso de medicamentos. El soporte ventilatorio deberá ser oportuno y el necesario para cada paciente en particular y podrá ser: Oxigenoterapia en fase I (casco cefálico, puntas nasales) con oxígeno a flujo libre y en las concentraciones necesarias de la fracción inspirada de oxígeno (FiO2) para dar un soporte real al paciente y lograr una oxigenación tisular adecuada. Manejo con presión positiva continua en vías aéreas (CPAP), donde la presión suministrada así como la fracción inspirada de oxígeno deberán ser controladas para proporcionar un adecuado intercambio gaseoso y evitar en lo posible complicaciones. Ventilación mecánica en sus diversas modalidades: Ventilación convencional, sincronizada, activada por el paciente, de volumen controlado, de ayuda proporcional, de alta frecuencia. La finalidad del soporte ventilatorio es lograr un adecuado intercambio gaseoso reclutando los alvéolos colapsados por el déficit de surfactante, y de esta manera mejorar la acidosis, la hipoxemia y la hipercapnea. Este reclutamiento alveolar se logra manteniendo una presión positiva continua al final de la expiración. (PEEP) debido a que el llamado PEEP fisiológico es de 2, se recomienda rangos por arriba de 4 cmH2O. El tratamiento de restitución de surfactante ha logrado disminuir la mortalidad hasta en un 50% de los casos del síndrome de dificultad respiratoria; en la actualidad hay dos grupos de surfactante aprobados por la FDA: 1.surfactante natural (se obtiene de fuentes humanas o animales). 2.- surfactante sintético. Las modalidades del tratamiento con surfactante pulmonar exógeno incluyen la modalidad profiláctica (Admón. En UTQ (100 mg/kg), valorar de requrirse Administrar en primera hr vida 100 mg/kg) y la de rescate (100-200 mg/kg si clínica y FIo2 >40%, Rx Compatible). CASO CLINICO RN masculino de edad gestacional de 39 semanas, de 3,8 kg hijo de madre diabética el cual se obtuvo por cesaria previa administración de esterioides a la madre, el cual requirió de estimulación vigorosa cursando con hipotermia que mejoro en incubadora, se observo posteriormente, aleteo nasal, retracción esternal, quejido respiratorio, cianosis y polipnea, la Rx mostro un patro retículo granular difuso bilateral y disminución de la expansión pulmonar, PaO2 < 50 mmHg. PREGUNTA Cual es la conducta a seguir. RESPUESTA a.- Presión positivo nasal cotinua. b.- Oximetria de pulso. c.- Intubación traqueal. d.- Ventilación mecánica. CASO CLINICO RN de 31 semanas que presenta dificultad respiratoria. Tiene 2 horas de nacido por parto vaginal. La dificultad respiratoria tiende a incrementar. Madre diabética, salvo la edad gestacional no se identificaron otras complicaciones durante el embarazo. EF: Temp. 37.5 ºC, TA:86/58, FR:60/min, FC:148/min. Marcadamente taquipneico, con retracciones supraesternales y supraclaviculares, aleteo nasal, quejido. Faringe normal. CP normal. Usted decide intubación endotraqueal, toma laboratorios y RX de tórax. RX de tórax: Infiltrado bilateral, difuso, “vidrio despulido” en ambos pulmones, sin datos de atrapamiento de aire. PREGUNTA Cual de los siguientes resultados de la gasometría es menos frecuente observar? RESPUESTA a.- Hipoxemia. b.- Hipercapnia. c.- Acidosis metabólica. d.- Acidosis respitoria.

CURSO ENARM CMN SIGLO XXI TEL: 36246001

Pharmed Solutions Institute

PÁGINA 349

MANUAL DE TRABAJO DEL CURSO ENARM CMN SIGLO XXI TAQUIPNEA TRANSITORIA DEL RECIEN NACIDO (TTRN): CIENCIAS BASICAS: Es un proceso respiratorio no infeccioso que se presenta con más frecuencia en los recién nacidos de termino o cercanos a término, donde hay retención de líquido pulmonar. Se inicia en las primeras horas y se caracteriza por la presencia de taquipnea (FR >60x´), insuficiencia respiratoria y aumento de requerimiento de oxígeno, con niveles de CO2, normales o ligeramente aumentados. Es un proceso generalmente benigno y autolimitado que se resuelve aprox. De 24-72hrs. Factores que favorecen la aparición de TTRN: maternos; asma, DM, tabaquismo, manejo abundante de líquidos, sedación por tiempo prolongado, RPM >24hrs. Recién nacido; nacimiento por cesárea sin trabajo anterior, parto precipitado, nacimiento cercano a término o termino , asfixia perinatal, sexo masculino, macrosomia, demoras en el pinzamiento del cordón umbilical, sedación materna excesiva, embarazo múltiple. SALUD PUBLICA: Incidencia: 1 a 2% de los recién nacidos. Recién nacidos término, cercanos a término, o prematuros grandes. La TTRN abarca del 35-50% de los casos de insuficiencia respiratoria no infecciosa de los recién nacidos que ingresan a los servicios de neonatología. PATOGENIA: 3 factores: 1. Retraso en la resorción de líquido pulmonar fetal 2. Inmadurez pulmonar 3. Deficiencia ligera de surfactante. Al momento del nacimiento, el epitelio pulmonar del RN que durante el embarazo es un activo secretor de cloro (Cl-) y liquido hacia los alveolos, tiene que cambiar para convertirse en un activo absorbedor de sodio (Na++) y liquido con el objeto de remover este último, que está condicionado por la presencia de catecolaminas secretadas durante el trabajo de parto que estimulan los llamados canales epiteliales de Na++. La TTRN es el resultado de alveolos que permanecen “h medos” al no producirse esta reabsorción en forma adecuada. El niño nacido por cesárea o el que nace precipitadamente por vía vaginal tiene mayor riesgo de tener exceso de líquido pulmonar como resultado de no haber experimentado las fases de la labor y la falta de exposición a las catecolaminas mencionadas. El resultado final son alveolos que retienen líquido (comprometiendo el intercambio gaseoso que favorece la hipoxemia), el cual se acumula poco a poco en el intersticio, hasta que es removido por los vasos linfáticos o pasa al torrente sanguíneo. El acumulo de líquido produce edema intersticial y disminución de la distensibilidad pulmonar, siendo esto último la causa de taquipnea (compensatoria), y colapso parcial bronquiolar que condiciona a su vez atrapamiento aéreo. Durante el transcurso de las siguientes horas el líquido es removido progresivamente, mejorando la oxigenación y disminuyendo la frecuencia respiratoria. DIAGNOSTICO: Taquipnea (>60 x´), dificultad respiratoria de cualquier grado evaluado de acuerdo a la escala de Silverman, cianosis en casos graves, campos pulmonares sin estertores. Laboratorio y gabinete: Gasometría arterial; hipoxemia, CO2 en limite normal o ligeramente aumentado, acidosis respiratoria compensada. Radiografía de tórax; Imágenes de atrapamiento aéreo (rectificación de arcos costales, herniación de parénquima, hiperclaridad, aumento de espacios intercostales, aplanamiento de diafragmas), cisuritis, congestión parahiliar, cardiomegalia aparente. Si después de 72 hrs la taquipnea no remite o incrementa pensar en otra patología y/o en complicaciones. TRATAMIENTO: Mantener en todo momento ambiente térmico neutro. La forma e inicio de alimentación se determina de acuerdo al estado clínico de RN (alimentación por succión, sonda orogástrica). Se mantendrá en ayuno en los siguientes casos: FR >80x´y dificultad respiratoria con Silverman mayor de 3. El volumen de líquidos IV será de acuerdo a las necesidades para la edad gestacional y el peso. Si existiera alguna patología asociada a la TTRN el inicio y forma de alimento será a juicio del médico. Manejo y estimulación mínimos. Farmacológico: oxigenoterapia; habitualmente no requieren FiO2 mayores al 40%. El objetico es obtener saturaciones por púlsometria en el rango de 88-95%. No se recomienda el uso de medicamentos ya que no existe evidencia suficiente de su eficacia y seguridad en RN. La administración postnatal de epinefrina con el fin de estimular la reabsorción de líquido pulmonar, ha sido motivo de estudios experimentales sin que exista al momento evidencia. Los esteroides antenatales a madres entre las 34-37 SDG. Podría tener un efecto benéfico al disminuir la morbilidad respiratoria de los RN, aun se requieren estudios para establecer su recomendación. CASO CLINICO Se trata de paciente masculino de 15 horas de vida estrauterina, el paciente nació via cesarea, nació a la 35 semanas de gestacion, se administro maduradores a la madre debido a trabajo de parto distócico con ruptura de membranas 12 horas antes de la intervención quirúrgica, a la exploración física se observo taquipnea de 101 rpm, y signo moderados de dificultad respiratoria. PREGUNTA Considerando un propable diagnostico clínico realizado, cual de los diagnosticos diferenciales es menos probable encontrar? RESPUESTA a.- Sindrome de adaptación pulmonar. b.- Sindrome de aspiración de meconio. c.- Enfermedad de membrana hialina. d.- Neumonia neonatal hospitalaria. PREGUNTA Se decide realizar radiografia de torax, cuales son los datos no es probale espera encontrar para confirmar el diagnostico? RESPUESTA a.- Broncograma aéreo. b.- Hilio congenstivos. c.- Derrame cisurales. d.- Sobredistencion pulmonar. PREGUNTA Cuales son los factores de riesgo inducido para la taquipnea trasitoria del recién nacido, cual es la mas probable en el caso clínico?. RESPUESA a.- Sexo masculino b.- Grandote.

CURSO ENARM CMN SIGLO XXI TEL: 36246001

Pharmed Solutions Institute

PÁGINA 350

MANUAL DE TRABAJO DEL CURSO ENARM CMN SIGLO XXI c.- Cesarea. d.- Sedación materna. PREGUNTA Cual es el fenómeno fisiopatológico no es la mas adecuada para el caso? RESPUESTA a.- Falta de absorción de líquido amniotico. b.- Falta administración de esteroides. c.- Deficienca ligera de surfactante. d.- Edema pulmonar transitorio. PREGUNTA Considerando el diagnotico actual, el cual muestra singos de dificultad respiratoria, resultados de laboratorio normales al momento, FiO2 < 0.40 descartando los diagnosticos diferencias, cuál es la conducta a seguir en este caso es menos adecuada? RESPUESTA a.- Monitoreo de signos vitales. b.- Monitoreo de gases por puncion cada 6 horas. c.- Mantener glucosa y electrolitos. d.- O2 por casco al 70 % PREGUNTA Tomando en cuenta la evolución del caso, cual es su pronostico? RESPUESTA a.- Bueno. c.- Malo. b.- Fatal. d.- Excelente. PREGUNTA Considerando un propable diagnostico clínico realizado, cual de los diagnosticos diferenciales es menos probable encontrar? RESPUESTA a.- Sindrome de adaptación pulmonar. b.- Sindrome de aspiración de meconio. c.- Enfermedad de membrana hialina. d.- Neumonia neonatal hospitalaria. PREGUNTA Se decide realizar radiografia de torax, cuales de los datos no es probale espera encontrar para confirmar el diagnostico? RESPUESTA a.- Broncograma aéreo. b.- Hilio congenstivos. c.- Derrame cisurales. d.- Sobredistencion pulmonar. PREGUNTA Cuales son los factores de riesgo inducido para la taquipnea trasitoria del recién nacido cual es la mas probable en el caso clínic?. RESPUESA a.- Sexo masculino b.- Grandote. c.- Cesarea. d.- Sedación materna. PREGUNTA Cual es el fenómeno fisiopatológico no es la mas adecuada para el caso? RESPUESTA a.- Falta de absorción de líquido amniotico. b.- Falta administración de esteroides. c.- Deficienca ligera de surfactante. d.- Edema pulmonar transitorio.

CURSO ENARM CMN SIGLO XXI TEL: 36246001

Pharmed Solutions Institute

PÁGINA 351

MANUAL DE TRABAJO DEL CURSO ENARM CMN SIGLO XXI PREGUNTA Considerando el diagnotico actual, el cual muestra singos de dificultad respiratoria, resultados de laboratorio normales al momento, FiO2 < 0.40 descartando los diagnosticos diferencias, cuál es la conducta a seguir en este caso es menos adecuada? RESPUESTA a.- Monitoreo de signos vitales. b.- Monitoreo de gases por puncion cada 6 horas. c.- Mantener glucosa y electrolitos. d.- O2 por casco al 70 % PREGUNTA Tomando en cuenta la evolución del caso, cual es su pronostico? RESPUESTA a.- Bueno. c.- Malo. b.- Fatal. d.- Excelente. CASO CLINICO Un RN de 3,6 kg nació a las 37 semanas de gestación hijo de diabética con un agente hipoglicémico oral. Obetnido por cesárea electiva con Apgar de 6/9. Desarrollo taquipnea inmediatamente después de su nacimiento y requirio oxígeno suplementario. En la gasometria presento; fueron PO2 de 57 mm Hg, PCO2 de 52 mm Hg, y pH de 7,31. El niño se mantuvo en oxígeno por campana. A las 2 horas el paciente se encuentra hipotónico, cianótico, con saturación del 70 %. La Rx de observa rectificación, hiperclaridad e incremento de los espacios intercostales y congestión parahiliar. PREGUNTA Cual es la medida inmediata a seguir. RESPUESTA a.- Intubacion orotraqueal. b.- Alimentación por sonda orogastrica. c.- Mantener un ambiente neutro. d.- Realizar medidas de reanimación. PREGUNTA Cual de las siguientes datos radiológicos es menos probable para el diagnostico radiológico? RESPUESTA a.- Aumento de la trama vascular con imágenes algodonosas. b.- Liquido en cisusras interlobales. c.- Abatimiento de diafragma. d.- Cardiomegalia. PREGUNTA Cual de los siguientes diagnostico diferenciales es mas frecuente? RESPUESTA a.- Sepsis. b.- Neumonía c.- Asfixia perinatal. d.- Cardiopatía congenita. CASO CLINICO Un RN de 3,4 kilos nació a las 40 semanas de gestación con madre diabética tratada con insulina. El niño fue obtenido por cesárea. Puntuación de Apgar fue 5/6. Se presento taquipnea una hora después en observación, el niño fue colocado en oxígeno con casco cefálico. Las determinaciones de gases en sangre arterial en ese momento eran de PO2 52 mm Hg, pCO2 de 48 mm Hg, y pH de 7,31. PREGUNTA Cual es la conducta inicial a seguir. RESPUESTA a.- Iniciar con bicarbonato. b.- Verificar laboratorios. c.- Realizar estudios de gabinete.

CURSO ENARM CMN SIGLO XXI TEL: 36246001

Pharmed Solutions Institute

PÁGINA 352

MANUAL DE TRABAJO DEL CURSO ENARM CMN SIGLO XXI d.- Colocacion de factor surfactante. CASO CLINICO RN femenino de 31 SDG "después de un embarazo sin complicaciones”, con peso de 1480 g, y Apgar de 2/6. Fue obtenido por cesaria electiva por cesaria previa por periodo intergenesico de 6 meses. A los pocos minutos de nacer, su frecuencia respiratoria fue de 61, con significativo aleteo nasal, uso prominente músculos accesorios y retracciones subcostales. Su ritmo cardíaco era normal (121 latidos / min). Había signos de cianosis. PREGUNTA La paciente se encuentra en un hospital rural de concentración, cual es su conducta a seguir. RESPUESTA a.- Trasladarla a una unidad de cuidados intensivos. b.- Vigilancia estrecha ya que su evolución es buena. c.- Los datos radiográficos no son importantes. d.- Verificar el diagnostico inicial

CURSO ENARM CMN SIGLO XXI TEL: 36246001

Pharmed Solutions Institute

PÁGINA 353

MANUAL DE TRABAJO DEL CURSO ENARM CMN SIGLO XXI SINDROME DE ASPIRACION MECONIAL (SAM). CIENCIAS BASICAS: El síndrome de aspiración meconial (SAM) se manifiesta con distrés respiratorio y es producido por la aspiración de líquido amniótico (LA) teñido con meconio intra útero o intra parto. Constituye una causa de morbimortalidad en el recién nacido, principalmente en el niño a término y postérmino. Factores de riesgo: Embarazo postérmino, preeclampsia- eclampsia, hipertensión materna, DM materna, oligohidramnios, tabaquismo intenso, puntaje de Apgar (igual o menor a 6) bajo a los 5 minutos. Hipoxia aguda intraparto, hipoxia perinatal crónica, frecuencia cardiaca fetal anormal, RN pequeños para la edad gestacional. SALUD PUBLICA: Se reporta con frecuencia líquido meconial 14%. 10% SAM. Mortalidad 12 % y además deja secuelas. Se presenta 9-15% de nacidos vivos. La expulsión de meconio rara vez es antes de las 37 semanas. Puede ocurrir 30% de los embarazos que continúan después de las 42 SDG. PATOGENIA: 1. Evacuación del meconio in-útero: La asfixia y otras formas de estrés intrauterino pueden causar un aumento del peristaltismo intestinal, con relajación del esfínter anal externo y evacuación de meconio. El efecto de la hipoxia intrauterina sobre el peristaltismo y el tono esfinteriano parece aumentar con la edad gestacional. 2. Aspiración de meconio: Después de la evacuación de meconio en el líquido amniótico, las respiraciones jadeantes del feto asfixiado, ya sea in útero o durante el trabajo de parto, pueden determinar la aspiración del líquido amniótico teñido con meconio hacia las vías aéreas grandes del pulmón. El meconio espeso provoca obstrucción de la vía aérea, lo que ocasiona dificultad respiratoria. Con la aspiración distal de meconio puede ocurrir una obstrucción parcial o total de la vía aérea. En las áreas de obstrucción total se desarrollan atelectasias; en cambio, en las áreas de obstrucción parcial ocurre un fenómeno valvular que ocasiona atrapamiento aéreo e hiperinsuflación pulmonar. El atrapamiento aéreo aumenta el riesgo de escape de aire del 21 al 50%. Finalmente, se desarrolla neumonitis intersticial y química, con edema bronquiolar y estrechamiento de las vías aéreas de pequeño calibre. La ventilación desigual debida a áreas de obstrucción parcial y neumonitis sobreagregada produce retención severa de dióxido de carbono e hipoxemia. La resistencia vascular pulmonar aumenta como resultado de la hipoxia, la acidosis y la hiperinsuflación de los pulmones. El aumento de la resistencia vascular puede conducir a un cortocircuito de derecha a izquierda auricular o ductal y a una mayor desaturación. CLASIFICACION: Clínicamente se clasifica en: a) Leve: discreta polipnea, hiperinsuflación torácica. No existe alteración en la PO2 ni en la PCO2 FiO2 18 hrs, reanimación al nacer y procedimientos invasivos, fiebre materna periparto. Dependiendo de la gravedad de la infección y la intensidad de la respuesta inflamatoria, esta se presenta como: Sepsis (síndrome de respuesta inflamatoria sistémica más hemocultivo positivo) Sepsis grave (sepsis más disfunción orgánica, hipotensión o hipoperfusión) Choque séptico (sepsis grave con hipotensión que no responde a carga de líquidos) y Falla orgánica múltiple. SALUD PUBLICA: Incidencia: 1 a 5 por cada 1000 nacidos vivos. 4 millones mueren de sepsis en las primeras 4 semanas. Cerca de un 85% de los pacientes requieren ventilación mecánica. PATOGENIA: Etiologia: Sepsis neonatal temprana: E. Coli, Klebsiella sp, Enterococcus sp. Streptococcus agalactiae y Listeria monocytogenes. Sepsis neonatal tardía: Staphylococcus coagulasa negativa, staphylococcus aureus, Enterococcus sp, E Coli, Klepsiella, Candida spp, Streptococcus del grupo B. La presencia de los mediadores hormonales e inmunológicos que interviene directamente en la modulación de la respuesta inflamatoria a la infección, definen más claramente los mecanismos involucrados en la respuesta inflamatoria aguda a la infección, además de la respuesta de anticuerpos, la activación de las dos vías del complemento, la participación del sistema de coagulación, la respuesta fagocitica mononuclear y polimorfonuclear y de la activación linfocitaría, ciertos mediadores que son sintetizados principalmente por monocitos, linfocitos, fibroblastos y células endoteliales actúan de manera determinante en la regulación y modulación de la respuesta inflamatoria inmunológica a la infección. Las citocinas actúan como mediadores de la comunicación intercelular y efectores de la activación de funciones de las células que tiene receptores para estas proteínas. Las cuales son: interleucinas, interferones y factores de crecimiento. Según sea su función se pueden encontrar las de actividad proinflamatoria (IL-1, IL-6, TNFalfa, IL-2, IL-8, IL-12) otras antiinflamatorias o inhibitorias (Antagonistas, Receptores solubles, IL-4, IL-10, IL-13) y factores de crecimiento (GM-CSF, G-CSF, M-CSF, IL-3, IL-6, IL-5, IL-7). La magnitud de citosinas proinflamatorias que se producen en etapa temprana de la sepsis se relaciona con la intensidad del daño pulmonar. DIAGNOSTICO: El diagnóstico temprano y oportuno de sepsis neonatal no es fácil porque las manifestaciones clínicas son inespecíficas y pueden avanzar rapidamente a estadíos más avanzados. Los signos de alarma identificados por la Organización Mundial de la Salud (OMS) son los siguientes: convulsiones, rechazo al alimento, dificultad respiratoria, hipoactividad, polipnea. Las manifestaciones clínicas son inespecíficas y muy variadas dependiendo de la gravedad de presentación. Algunas de las principales son: distermias, dificultad respiratoria, ictericia, apneas (con más frecuencia en prematuros), distensión abdominal, hepatomegalia, letargia, sangrados, hipoactividad, palidez, oliguria, cianosis, piel marmórea, crisis convulsivas, irritabilidad, esplenomegalia, vómito, diarrea, hipotensión arterial, petequias o equimosis, trombocitopenia y acidosis. La sospecha clínica es lo principal para poder llegar al diagnóstico de sepsis neonatal e idealmente confirmarse con cultivos positivos en sangre, orina, líquido cefalorraquídeo (LCR) u otros sitios normalmente estériles. Ante cualquier cuadro sugestivo de sepsis, se debe realizar estudio diagnóstico completo. En sepsis temprana se deben incluir hemocultivos (central y periféricos) y cultivo de líquido cefalorraquídeo. En sepsis tardía se debe incluir además urocultivo. En casos de infecciones localizadas (por ejemplo osteoartritis), se debe cultivar el sitio de infección. Prácticamente cualquier problema neonatal puede presentarse como sepsis, debido a esto es importante que se realice una historia clínica y exploración física adecuadas, toma de biometría hemática completa. Si bien no existe un biomarcador de sepsis ideal, existen múltiples estudios que apoyan la utilidad de procalcitonina y proteína C reactiva para el diagnóstico de sepsis neonatal. Hemocultivo: sensibilidad 82%, especificidad 96%, BH: Leucocitos anormales: < 6 000 células/mm o > 30 000 células/mm en las primeras 24 hrs. 50% RN puede tener conteos normales. TRATAMIENTO: Primera elección: Ampilicina- amikacina. El manejo empírico inicial de antibióticos debe hacerse con base en la experiencia de cada hospital, siempre teniendo en cuenta el patrón de resistencia y sensibilidad. En sepsis neonatal temprana el tratamiento debe iniciarse con ampicilina y un aminoglucósido (gentamicina ó amikacina), en ocasiones especiales se puede sustituir el aminoglúcosido por cefotaxima, sobre todo si existe la sospecha de neuroinfección (está demostrado que esteriliza el LCR con mayor rápidez). En recién

CURSO ENARM CMN SIGLO XXI TEL: 36246001

Pharmed Solutions Institute

PÁGINA 357

MANUAL DE TRABAJO DEL CURSO ENARM CMN SIGLO XXI nacidos con sepsis tardía adquirida en la comunidad, es posible utilizar el mismo esquema, sin embargo en sepsis nosocomial, el tratamiento debe estar orientado a combatir los microorganismos presentes en cada institución. La evidencia actual de ensayos clínicos controlados aleatorizados no apoya el uso rutinario de inmunoglobulina intravenosa y factor estimulante de colonias de granulocitos. El uso de pentoxifilina como adyuvante en el manejo de sepsis reduce la mortalidad en neonatos pretérmino, sin embargo debido a debilidades metodológicas de los estudios al respecto, no es adecuado utilizarlo de manera rutinaria hasta la obtención de mejor evidencia. Dependiendo de las condiciones clínicas del paciente en el caso de sepsis grave o choque séptico se deberá proporcionar apoyo ventilatorio, suministro de líquidos, aminas e incluso corticoesteroides en el caso de hipotensión refractaria a las mismas o en caso de sospecha de insuficiencia suprarenal. Es indispensable realizar la corrección del equilibrio ácido base y proporcionar apoyo calórico y nutricional ya sea por vía enteral o parenteral según sea el caso. Pacientes con riesgo de sepsis: iniciar tratamiento revalorar 48 hrs. (el 95% de las infecciones se presenta en las primeras 24 hrs). Si no hay desarrollo de cultivos, biometría hemática normal y PCR normal suspender el tratamiento. CASO CLINICO Primigesta de 15 años de edad, con gestación de 40 semanas, presenta trabajo de parto en fase de dilatación rápida (5 cm de dilatación) y expulsión de líquido amniótico (LA) con tinte meconial. Tuvo una rotura precoz de membranas 1 h 20 min antes del ingreso. Se indicó anestesia de conducción con peridural continua y monitorización electrónica que reveló una variabilidad menor de 5 LCF, una frecuencia cardíaca fetal media de 150 latidos por min y una desaceleración variable prolongada (hasta 120 LCF por minuto durante 3 min). Se reevaluó luego de nueva desaceleración (60 LCF por minuto durante 7 min) y se encontró 8 a 9 cms de dilatación, presentación cefálica en II plano y LA con grumos de meconio con leve mal olor. Se trasladó inmediatamente a sala de partos con dilatación completa y presentación cefálica en tercer plano. Se obtuvo un recién nacido (RN) de 3,440 g de sexo masculino, con muy mal olor, con Apgar 1/3. El RN evolucionó grave, con compromiso del SNC, encefalopatía hipóxica isquémica grado III, asociado a shock séptico precoz, con compromiso multiorgánico y necesidad de apoyo ventilatorio y de drogas vasoactivas. PREGUNTA Cual es la complicación más probable en esta paciente. RESPUESTA a.- Coagulacion intravascular diseminada. b.- Falla multiple. c.- Neurosepsis. d.- Neumonia asociada. CASO CLINICO Se trata de paciente de 35 semanas de gestacion de 5 dias de nacido, con ruptura prematura de membrana el cual requirió reanimación intensiva debido a pagar 2/6, se observo fiebre materna durante el trabajo de parto, la madre vive en medio rural, apnea, crisis convulsivas, vomito, distermia, taquicardia, disnea y deterioro, hiporreactivo con piel marmórea, distención abdominal, se diagnostico sepsis neonatal. PREGUNTA Cuales de los diagnosticos diferenciales es el menos frecuente? RESPUESTA a.- Sindrome de adaptación del recién nacido. b.- Taquicardia transitoria del recién nacido c.- Hemorragia ventricular. d.- Neumonitis PREGUNTA Cual de los siguientes valores de laboratorio es mas probable esperaría?. RESPUESTA a.- PCR elvada y neutrofilia. b.- Leucocitosis con eosinofilia. c.- Linfocitosis mas plaquetopenia. d.- VSG elevada con leucopenia. PREGUNTA Cual es la conducta farmacológica empirico de primera eleccion en espera de los resultados de cultivos. RESPUESTA a.- Ampicilina mas gentamicina. b.- Ampicilina mas amikacina. c.- Inmoglobulina y cefotaxima. d.- Ampicilina y FEC.

CURSO ENARM CMN SIGLO XXI TEL: 36246001

Pharmed Solutions Institute

PÁGINA 358

MANUAL DE TRABAJO DEL CURSO ENARM CMN SIGLO XXI ENTEROCOLITIS NECROTIZANTE (ECN). CIENCIAS BASICAS: Se presenta como un síndrome gastrointestinal y sistémico que comprende síntomas variados y variables, como distensión e hipersensibilidad abdominal, sangre en heces, intolerancia alimentaria, apnea, letargia y en casos avanzados acidosis, sepsis, CID y shock. Constituye la urgencia gastrointestinal más frecuente en las UCI neonatales. SALUD PUBLICA: Es una enfermedad grave que afecta a recién nacido, en especial prematuros, con una incidencia y morbilidad elevados. Incidencia 1-3 por 1000 RN vivos y 1-7.7% de los RN ingresados a unidades neonatales. Aumenta la incidencia en los menores de 1500g, hasta un 2-10%. La edad gestacional media oscila en torno a las 31 semanas, con un peso medio al nacimiento de 1.460 g. PATOGENIA: Se acepta un mecanismo multifactorial en el huésped predispuesto. Entre los factores propuestos implicados en la patogénesis de la ECN se han descrito la prematuridad, alimentación láctea, inestabilidad hemodinámica, infección y alteración de la mucosa intestinal. Solo la prematuridad y la alimentación láctea tienen una base epidemiológica consistente. En prematuros, la predisposición de estos niños pudiera explicarse por la inmadurez de su tracto gastrointestinal con función luminal limitada que conlleva una absorción parcial de carbohidratos y grasas así como proliferación bacteriana, mayor permeabilidad de la mucosa e hipomotilidad. Junto a ello existe inmadurez de los sistemas defensivos sistémicos y de la mucosa intestinal, entre otros la IgA secretora y la barrera de mucina. Se ha especulado la posibilidad de un efecto protector de los glucocorticoides administrados prenatalmente. Una de las principales hipótesis es que la ECN proviene de efectos nocivos de la microflora intestinal sobre una mucosa entérica lesionada o isquémica. La lesión mucosa puede provenir de distintos procesos, incluyendo la asfixia o la isquemia secundaria a la derivación de la sangre lejos del intestino. Se desconoce la especificidad de este proceso. La flora microbiana involucrada en la ECN no es única, aunque representa organismos intestinales predominantes presentes en el recién nacido en el momento de la aparición de la enfermedad. Estos organismos pueden incluir las bacterias anaeróbicas, como las especies de Clostridium. Por el momento no se ha definido un papel especifico del Clostridium difficile o de una toxina especifica. Algunos virus pueden estar incluidos en este cuadro microbiológico. DIAGNOSTICO: El hallazgo más precoz suele ser un cambio en la tolerancia alimentaria en un niño prematuro, con buena evolución hasta ese momento y que comienza a presentar restos gástricos. Los síntomas sistémicos asociados son inespecíficos, desde aparición de apneas, alteración del patrón respiratorio, distermia, inestabilidad hemodinámica con bradicardias, hasta hipotensión, letargia o shock séptico y CID. Desde el punto de vista gastrointestinal, la ECN se presenta con distensión abdominal, restos gástricos, abdomen doloroso, vómitos, diarrea o hematoquecia. El curso de la enfermedad varía según recién nacidos. Con mayor frecuencia aparece como: 1) una presentación fulminante, rápidamente progresiva de signos congruentes con necrosis intestinal y sepsis, y 2) como una presentación lenta, paroxística, de distensión abdominal, íleo y posible infección. Laboratorio y gabinete: La radiografía abdominal (confirma diagnostico) suele revelar un patrón gaseoso anormal congruente con el íleo. Estas radiografías pueden revelar un edema de la pared intestinal, la posición fija de un asa en estudios seriados, la aparición de una masa, la neumatosis intestinal (95%) más frecuente en cuadrante inferior derecho, el aire venoso portal o hepática, la neumobilia o el neumoperitoneo (cuando hay perforación intestinal). Estudios sanguíneos. La trombocitopenia, la acidosis metabólica persistente y la hiponatremia refractaria grave son la triada más frecuente y ayudan a confirmar el diagnóstico. La trombocitopenia se asocia a necrosis intestinal y empeoramiento clínico. El análisis de las heces para detectar sangre e hidratos de carbono, si los hay, se ha utilizado para diagnosticar a los recién nacidos afectos de ECN, basado en los cambios de la integridad intestinal. La malabsorción de los hidratos de carbono, según refleja un Clinitest positivo (cuerpos reductores) en las heces, puede ser un indicador frecuente y precoz de ECN en el marco de los signos. Otros hallazgos bioquímicos inespecíficos son la elevación sérica de PCR y alfa-1glicoproteina o de la alfa-1-antitripsina en heces. Ante la posibilidad de sepsis deben recogerse cultivos (hemocultivo, cultivo de LCR, coprocultivo). Si se sospecha Clostridium son necesarios cultivos específicos y determinación de toxina. La visualización directa de la mucosa entérica mediante endoscopia se ha utilizado para detectar a los recién nacidos afectos de ECN, aunque la experiencia con esta técnica es demasiado limitada para ser de aplicaci6n directa. Los avances tecnológicos pueden hacerla posible en el futuro. Debe considerarse la ultrasonografía como método útil en el diagnostico cuando existe sospecha clínica, no confirmada por radiología. CLASIFICACION: ETAPA I: sospecha a.- signos sistémicos leves: apnea, bradicardia, inestabilidad térmica, letargia. b.- signos intestinales leves: distensión abdominal, restos gástricos (pueden ser biliosos), sangre oculta en heces. c.- radiografía de abdomen: normal o con signos no específicos. ETAPA II: enfermedad definida (signos radiológicos positivos) a.- signos sistémicos moderados. b.- signos intestinales adicionales: silencio abdominal, dolor a la palpación del abdomen. c.- signos radiológicos específicos: neumatosis intestinal o gas en el sistema portal. d.- alteraciones analíticas: acidosis metabólica, leucopenia, trombocitopenia. ETAPA III: enfermedad avanzada: shock séptico y neumoperitoneo. a.- afectación sistémica grave: hipotensión arterial, signos evidentes de shock. b.- signos clínicos de peritonitis. c.- signos radiológicos de gravedad: neumoperitoneo. d.- alteraciones analíticas: acidosis metabólica y respiratoria, leucopenia y neutropenia, trombocitopenia, coagulación intravascular diseminada, proteína C muy elevada. TRATAMIENTO: Medico: consiste en medidas de soporte, reposo intestinal, nutrición parenteral, antibioterapia y corrección de alteraciones hematológicas y/o electrolíticas que pudieran estar presentes. Medidas específicas son la dieta absoluta, descompresión intestinal con aspiración, reposición de líquidos considerando pérdidas a un tercer espacio, aporte calórico adecuado mediante nutrición parenteral y antibióticos endovenosos de amplio espectro. Deben suspenderse todos los fármacos relacionados como posibles factores de riesgo. Como medidas de soporte se incluye la asistencia respiratoria temprana ante la aparición de apneas o patrón respiratorio acidotico, la corrección de la acidosis, hiponatremia o trombopenia. La acidosis metabólica persistente es un indicador de progresión de la lesión intestinal e incluso necrosis. Se debe asegurar una perfusión y transporte de oxigeno adecuados, con un aporte suficiente de líquidos y manteniendo el nivel de hematocrito >35%. Puede ser necesario el uso de agentes inotrópicos. La dopamina a dosis bajas puede ser de ayuda para mejorar la perfusión sistémica y aumentar el flujo mesentérico. Habitualmente la ampicilina y gentamicina constituyen el tratamiento adecuado, debiendo asociar clindamicina o metronidazol ante sospecha de gérmenes anaerobios, si bien la clindamicina ha sido asociada con un aumento

CURSO ENARM CMN SIGLO XXI TEL: 36246001

Pharmed Solutions Institute

PÁGINA 359

MANUAL DE TRABAJO DEL CURSO ENARM CMN SIGLO XXI de la frecuencia de estenosis postenterocoliticas. La dieta y la antibioticoterapia han de mantenerse durante 10-14 días, con introducción progresiva de aporte enteral, con fórmulas hipoosmolares de hidrolizado de proteínas. Quirúrgico: El momento ideal sería aquel en que se ha producido una gangrena intestinal pero todavía no existe perforación, ni peritonitis secundaria. Indicada en pacientes que progresa con necrosis de pared intestinal y perforación debe ser urgente debido al rápido deterioro. La decisión es clara en aquellos en que destaca la presencia de neumoperitoneo. El empeoramiento progresivo nos alerta sobre necrosis intestinal como: signos de peritonitis como edema y eritema de pared, masa abdominal, trombopenia y acidosis persistentes. Paracentesis: un resultado positivo es altamente específico de necrosis intestinal. CASO CLINICO RN con parto en casa con control prenatal con 34 semanas de gestación por FUM, con un peso 2100 g. 7 dias despues desarrolló sepsis y fue hospitalizado en la unidad de cuidados intensivos. El examen clínico encontró una distensión abdominal y una radiografía abdominal mostró aire en las paredes del intestino y un neumoperitoneo grande (aire libre subdiafragmático, aire perihepático libre, doble signo de la pared). Requirio intubacion y se proporciono soporte inotrópico y antibióticos. PREGUNTA Cual es la causa más probable de la complicación. RESPUESTA a.- Nacimiento en casa. b.- Edad gestacional c.- Comorbilidad pulmonar. d.- Sepsis abdominal. CASO CLINICO Al segundo día de vida presenta resolución de su dificultad respiratoria con disminución de requerimientos de oxígeno y tolerancia al destete, por lo cual se inicia estímulo enteral con adecuada tolerancia. Los laboratorios iniciales hemograma normal y PCR negativa. Se continúo vigilancia clínica y aumento progresivo de la vía oral. A los 5 días de vida presenta cuadro de episodio emético de contenido alimentario sin otra sintomatología el cual se interpretó como reflujo gastroesofágico. Al día siguiente presenta deterioro clínico dado por taquicardia, distensión abdominal, persistencia de episodios eméticos postprandiales, residuo gástrico del 70%, deposiciones con sangre macroscópica e hipoglicemia. PREGUNTA Considerando los hallazgos clínicos cual es el estadio en el que se encuentra? RESPUESTA a.- Enterocolitis necrotizante I. b.- Enterocolitis necrotizante II. c.- Enterocolitis necrotizante III. d.- Enterocolitis necrotizante IV. PREGUNTA Cual es la conducta a segur mas adecuada en este momento? RESPUESTA a.- Se suspende la vía oral, BH, QS, cultivos, ampicilina y gentamicina. b.- Ampicilina + amikacina, Bh, QS, cultivos, ayuno. c.- Alimentacion parenteral, antibioticoterapia empirica, BH, QS, cultivos. d.- Ampicilina, cefotaxima, alimentación por sonda, BH y QS. PREGUNTA Tres días después el paciente presenta hipotensión e hipoperfusión asociado a falla respiratoria por lo que requiere soporte inotrópico y ventilatorio. Presenta acidosis metabólica, trombocitopenia y alteración en pruebas de coagulación, cultivos con E.coli multiresistente, considerando los cambios en de las imágenes de actuales cual es la conduca a seguir en el paciente? RESPUESTA a.- Transfusión de plasma y plaquetas. b.- Crioprecipitados, albumina y cristaloides. c.- Piperacilina tazobactam y nutrición parenteral. d.- Piperacilina tazobactam, amikacina y nutrición parenteral. CASO CLINICO Recién nacido (RN) varón, con antecedentes de parto de pretérmino a las 32 semanas de gestación, con peso de 1 300 g y Apgar 8 y 10 al nacer. Estando relativamente bien, a las 72 horas de vida comienza con rechazo alimentario y distensión abdominal. Un día más tarde se agregan vómitos biliosos, deposiciones con sangre e inestabilidad hemodinámica. PREGUNTA

CURSO ENARM CMN SIGLO XXI TEL: 36246001

Pharmed Solutions Institute

PÁGINA 360

MANUAL DE TRABAJO DEL CURSO ENARM CMN SIGLO XXI Se realizo una radiografia para confirmar diagnostico, cual de los siguientes datos no esta presente en esta? RESPUESTA a.- Dilatación de asas intestinales de colon e intestino delgado. b.- Engrosamiento de paredes y extensa neumatosis intestinal. c.- Múltiples imágenes aéreas pequeñas, en aspecto de "burbujas" . d.- Neumoperitoneo. PREGUNTA Considerando las imágenes observadas, cual es diagnostico final y complicaciones asociadas. RESPUESTA a.- Enterocolitis necrotizante complicada, con perforación intestinal y neumoperitoneo secundario. b.- Enterocolitis necrotizante complicada, sin perforación intestinal ni neumoperitoneo secundario. c.- Enterocolitis necrotizante simple, con preforacion intestinal. d.- Enterocolitis complicada con peritoneo secundario. PREGUNTA Cual de las siguientes complicaciones agudas es mas frecuente? RESPUESTA a.- Necrosis intestinal con perforación b.- Peritonitis secundaria. c.- Formación de abscesos intraabdominales. d.- Coagulación intravascular diseminada. CASO CLINICO Se trata de un paciente varón de 5 meses de vida que ingresó en nuestro hospital por presentar vómitos proyectivos en la mitad y al final de las tomas, acompañados de una pérdida de peso de 350g en el transcurso de una semana. Como antecedentes destacaba que era un exprematuro moderado de 30 semanas de gestación que al quinto día de vida desarrolló una EN con neumatosis gástrica e intestinal, por lo que precisó una resección de 10cm del yeyuno y 25cm del íleon. Tras la intervención quirúrgica presentó varios episodios de retención gástrica y vómitos, con resolución espontánea y adecuada tolerancia posterior, por lo que se le dio el alta a su domicilio. PREGUNTA Cual es la conducta a seguir en este caso, con la sintomatologia presente cual es estudio de primera elección para identificar la causa de la condución actual? RESPUESTA a.- Tránsito esofagogastroduodenal. b.- Radiografia de abdomen. c.- Ultrasonograma abdominal. d.- Endoscopia digestiva. PREGUNTA Cual de los siguiente diagnostico diferenciales del estado actual, es el menos frecuente? RESPUESTA a.- Ingestion de causticos. b.- Anomalias del ligamento falciforme. c.- Enfermedad granulomatosa caustica. d.- Gastroenteritis eosinofilica.

CURSO ENARM CMN SIGLO XXI TEL: 36246001

Pharmed Solutions Institute

PÁGINA 361

MANUAL DE TRABAJO DEL CURSO ENARM CMN SIGLO XXI MENINGITIS NEONATAL. CIENCIAS BASICAS: Es una enfermedad infecciosa con secuelas a corto y largo plazo, siendo su tratamiento costoso, con aumento de los días de hospitalización y número de días recibiendo antibióticos. Meningitis bacteriana: síndrome clínico compatible con meningitis, más aislamiento de un organismo por cultivo del líquido cefalorraquídeo o aislamiento de un organismo por cultivo de sangre y anormalidad del LCR consistente con infección bacteriana. SALUD PUBLICA: Frecuencia variable. 25-30% de los casos de sepsis neonatal se complica con meningitis. Meningitis bacteriana tiene incidencia hasta de 1 x 1000 nacidos vivos. CLASIFICACION: Meningitis temprana: Primera semana transmisión vertical (gérmenes localizados en el canal vaginal materno) gérmenes involucrados, Streptococcus del grupo B, E. coli, Listeria monocytogenes, Enterococcus. Meningitis tardía: Nosocomial adquirida en la comunidad gérmenes involucrados, transmisión horizontal Gram negativos, Estafilococos. PATOGENIA: Etiología: gram(): E. coli, Klebsiella, Serratia marcescens, Proteus, Enterobacter, Neisseria, Pseudomona, Salmonella, Citrobacter, etc. Gram(+): Stp, Enterococos, Staphylococcus aureus, estafilococo coagulasa negativo. Otros: Lysteria monocytogenes. Más frecuente: Enterobacterias gram negativas (30-50%). Estreptococo beta-hemolítico grupo B (30-40%). Lysteria monocytogenes (10%). S. pneumonie, H. influenzae. Es la diseminación hematógena en la mayoría de los casos, sin embargo ocasos se producen por propagación por contigüidad como consecuencia de la contaminación de defectos del tubo neural, fístulas congénitas o de heridas penetrantes causadas al obtener muestras de sangre, partos sépticos o domiciliarios. La cerebritis y los infartos sépticos son frecuentes en la meningitis bacteriana. La formación de abscesos, la ventriculitis, la hidrocefalia y los derrames subdurales aparecen más a menudo en recién nacidos que en niños mayores. Los hallazgos patológicos son similares entre los diferentes agentes bacterianos. El más común en autopsias es exudado purulento en meninges y superficie ependimal de los ventrículos. Se evidencia también inflamación perivascular y gran respuesta inflamatoria. Hidrocefalia y encefalopatía no infecciosa puede ser demostrada en aproximadamente 50% de los neonatos con meningitis. La efusión subdural raramente ocurre en neonatos. Varios grados de flebitis y arteritis de vasos intracraneales pueden presentarse. Ventriculitis puede ser demostrada en virtualmente todos los neonatos y en 75% al momento del diagnóstico. Factores de riesgo: Infección perinatal e intrauterina (corioamnioitis, endometritis, IVU, infección vaginal), prematurez, ruptura prematura de membranas, peso bajo al nacimiento, vía hematógena, aspiración, inhalación. Corioamnionitis clínica: presencia de fiebre materna >38° C, con dos o más de los siguientes hallazgos: taquicardia fetal (>160), sensibilidad uterina a la palpación, descarga vaginal fétida o leucocitosis materna. DIAGNOSTICO: Clínica: fiebre o hipotermia 60%, Irritabilidad 60% pobre alimentación/vomito 48% crisis convulsivas 42% dificultad respiratoria 33% apnea 31% diarrea 20% fontanela abombada 25% rigidez de nuca 13%. La sintomatología de un paciente con meningitis es muy inespecífica y puede ser muy similar a la de un paciente con sepsis y las manifestaciones como fontanela abombada, alteración del estado de conciencia, convulsiones y coma, cuando se presentan se asocian a pobre pronostico; por lo que es importante que dentro de la evaluación del neonato infectado se incluya la punción lumbar incluso en la ausencia de signos neurológicos evidentes. Laboratorios: 1. BH, recuento de leucos y porcentaje de PMN: < 20% ó >80% Índice inmaduros/ maduros totales: ≥ 0.2 - Plaquetas 10mg/dl 3. Glicemia: si presenta valor 120mg /dl 4. Hemocultivos: Siendo positivos hasta un 50 %. Citoquímico, Gram y cultivo de LCR: Se tendrán los siguientes criterios en el líquido cefalorraquídeo: 1) Glucosa menor a dos tercios la glucosa sérica medida por glucometria simultánea. 2) Relación glucosa LCR/ sérica mayor de 0.5. 3) Proteínas: Se considerará positivo para infección valor mayor de 90 mg/dl para recién nacido a término y de más de 150md/dl en prematuros. 4) Celularidad: Mayor de 10 células blancas. 5) Cultivo positivo. Contraindicaciones para la punción lumbar (LCR: 10-30 ml prematuro, RN termino 40 ml): compromiso cardiorespiratorio, CID, lesiones en el sitio de la punción, prematuros extremos (menores de 1000 gramos). Imagenología: Hacerla para detecta complicaciones, pacientes choqueados, falla respiratoria, déficit neurológico, cultivo + después de 48-72 Hs con apropiada terapia. TAC para descartar abscesos. TRATAMIENTO: Sepsis temprana: ampicilina+amikacina. Sepsis tardía: cefotaxima+amikacina. Específico para el agente identificado. Otros: Meropenem, Ticarcilina, Vancomicina. Duración terapia: 14-21 días. Esteroides. Inmunoglobulina. Aunque el manejo final deberá basarse en el resultado del antibiograma de las muestras de los hemocultivos, el tratamiento empírico iniciado con penicilinas más una cefalosporina de tercera generación y/o un aminoglucósido tipo gentamicina preferencialmente, después de haberse tomado las muestras, es el que ha mostrado la mejor de las respuestas clínicas. El tratamiento endovenoso deberá continuarse con el antibiótico apropiado para el germen aislado, hasta dos semanas después de la negativización del hemocultivo. Desafortunadamente, las secuelas severas a largo plazo son frecuentes (12% a 29% de los afectados), especialmente en las infecciones asociadas a Streptococcus del grupo B y a bacilos gram negativos, aún en los casos en los que el tratamiento ha sido oportuno y adecuado CASO CLINICO Paciente masculino 10 dias de nacimiento, que se obtuvo en casa sin control prenatal, ingresa por diarrea, llanto, e irritabilidad, vomito en proyectil, en casa, refiere la madre que solo fue en una ocacion, al ingreso se observo hipotónico, llanto agudo, mal estado generalizado, con tono cervical no se observo rigidez, pero fontanela abombada, su peso fue de 2050 grs, laboratorios con 150,000 de plaquetas, antecedentes de rinorrea hialina, se presento 35 grados, irritabilidad, durante la exploración se observo pedaleo y chupeteo, dificultad respiratoria. PREGUNTA Para establecer el diagnostico y conducir su terapéutica para el caso? RESPUESTA a.- Hemocultivo. b.- Punsion lumbar.

CURSO ENARM CMN SIGLO XXI TEL: 36246001

Pharmed Solutions Institute

PÁGINA 362

MANUAL DE TRABAJO DEL CURSO ENARM CMN SIGLO XXI c.- Tomografia. d.- Biometria hemática. PREGUNTA Se encuentra en espera de resultados, sin embargo el paciente se va deteriorando, cual es la agente etiolohico mas probable para dar una terapéutica dirigida? a.- H influenza b.- E Coli. c.- Listeria monocitogenes d.- Estreptoco beta hemolito. PREGUNTA Cual es la conducta farmologica más adecuada considerando la etiología? RESPUESTA a.- Ampicilina, amikacina. b.- Ampicilina mas metronizadol. c.- Ampicilina mas vancomicina. d.- Ampicilina mas cefotaxima. PREGUNTA Se administro esteroides en el paciente, cual es la razón mas correcta para realizar esta conducta en el caso? RESPUESTA a.- Disminuir los efectos de los factores inflamatorios. b.- Favorece la penetración del fármaco. c.- Disminuir la probabilidad de crisis convulsivas. d.- Prevenir el edema cerebral. PREGUNTA Cual es la secuela mas frecuente que los pacientes con esta patologia desarrollan? RESPUESTA a.- Empiema cerebral b.- Retazo en el neurodesarrollo c.- Hidrocefalia. d.- Paralisis cerebral. CASO CLINICO Recién nacido de 12 días de vida con fiebre de 38,5ºC de una hora de evolución y rechazo de la alimentación. Como antecedentes; ruptura espontánea de membranas intraparto, parto instrumental con fórceps a las 41 semanas de gestación y peso al nacimiento de 3880 g. Apgar 8/9. Comenzó su esquema de inmunización. Alimentación con lactancia materna exclusiva. La exploración física mostró decaimiento y regular perfusión periférica con constantes vitales normales. La analítica inicial mostró una determinación de proteína C reactiva cuantitativa (PC-R) 133,4 mg/L, con un hemograma coagulado y un sedimento de orina normal. La punción lumbar resultó hemorrágica. Ingresó con diagnóstico de síndrome febril sin foco y se inició antibioterapia empírica con ampicilina y cefotaxima intravenosas tras la toma de cultivos. En el hemograma repetido se observaron 5800 leucocitos/mm3, con fórmula normal, 13,4 g/dl de hemoglobina, 32 000 plaquetas/mm3. A las 18 h del ingreso empeoró su estado general y presentó crisis convulsivas generalizadas que requirieron tratamiento con fenobarbital y perfusión de midazolam. En la evolución desarrolló afectación sistémica con hipotensión arterial, acidosis metabólica, hiponatremia, oliguria y plaquetopenia grave; requirió ventilación mecánica convencional, transfusión de hemoderivados, soporte inotrópico con dopamina y diurético con furosemida. El tercer día del ingreso se informó el aislamiento en el hemocultivo de Streptococcus pyogenes sensible a penicilina, por lo que se suspendió la ampicilina y se mantuvo el tratamiento con cefotaxima intravenosa. PREGUNTA Cual es la causa más probable de las crisis convulsivas. RESPUESTA a.- Hemorragia cerebral. b.- Alteracion hidroelectrolitica. c.- Neurosepsis. d.- Encefalopatia hipoxico-isquemica.

CURSO ENARM CMN SIGLO XXI TEL: 36246001

Pharmed Solutions Institute

PÁGINA 363

MANUAL DE TRABAJO DEL CURSO ENARM CMN SIGLO XXI CASO CLINICO Neonato de 11 días de vida, previamente sana, ingresada por dificultad respiratoria, mala coloración, rechazo de tomas y decaimiento. Constantes vitales; TA de 95/65 mmHg, FR de 160 lpm, FR de 90 rpm y saturación de oxígeno del 94% con FiO2 del 30%. Se auscultan ruidos crepitantes e hipoventilación basal izquierda. La radiografía de tórax evidencia infiltrado basal izquierdo. PREGUNTA Cual es la conducta a seguir. RESPUESTA a.- Amoxicilina mas gentamicina. b.- Trimetoprim mas sulfametoxazol. c.- Cetriaxona. d.- Vancomicina. CASO CLINICO Se trata de paciente que se encuentra en la UCIN con diagnostico de sepsis neonatal, prematuro, bajo peso y masculino con 10 dias de nacido, inicia con llanto continuo, irritable, sin rigidez de nuca, se observa hipotónico, rechazo al alimento, vomito en proyectil. Antecedentes con aspiración de meconio con sufrimiento e hipoxia, por parto prolongado y distocia, se observo disminución de complemento y linfocito, PREGUNTA Cual es el estándar de oro para el dianostico mas adecuado? a.- Hemocultivo y urocultivo. b.- Liquido cefaloraquideo. c.- Biometria hemática y EGO. d.- Marcadores inflamatorios. PREGUNTA Cual es el agente etiológico mas probable en el caso clínico actual? a.- E. coli b.- Klebsiella c.- Pseudomona d.- Salmonela PREGUNTA Cuales son las siguiente manifestaciones es la que presenta mayor frecuencia y mayor valor diagnostico? a.- Fiebre e irritabiliad b.- Vomito y crisis convulsivas. c.- Alteraciones metabolicas d.- Fontanela abombada. PREGUNTA Cuales son los valores que no es probable observar en este paciente. RESPUESTA a.- Proteína de aumentada b.- Glucosa 2/3 del central. c.- Leucocitos elevados d.- Acido láctico disminuido. PREGUNTA Cual es la complicación anatomopatologia mas frecuente? RESPUESTA a.- Hemorragia ventricular. b.- Ventriculomegalia c.- Encefalomacia. d.- Ventriculitis. PREGUNTA Cual es el tratamiento empirico de primera elección? RESPUESTA

CURSO ENARM CMN SIGLO XXI TEL: 36246001

Pharmed Solutions Institute

PÁGINA 364

MANUAL DE TRABAJO DEL CURSO ENARM CMN SIGLO XXI a.- Ampicilina mas gentamicina b.- Ampicilina mas amikacina c.- Amikacina mas cefotaxima. d.- Vancomicina, ampicilina mas cefotaxima. PREGUNTA Al paciente se le administro esteroides, cual es el objetivo farmacológico para esta indicación? RESPUESTA a.- Como medida antiedema. b.- Para sinergismo con inmunoglobulina. c.- Sinergismo con los antibióticos. d.- Estabilización de membrana. PREGUNTA Cual de las siguientes complicaciones es menos frecuente esperar? RESPUESTA a.- Ventriculitis b.- Hidrocefalia. c.- Pérdida auditiva y ceguera. d.- Sindrome de Rett. PREGUNTA Condiderando los criterior para definir sepsis es un síndrome de respuesta inflamatoria sistémica, aunado a un agente infeccioso en el RN? RESPUESTA a.- Horas de nacido, >130 FC. b.- FR mas de 50 c.- Leucocitos mas de 16,500. d.- Temperatura 37.8 y 10% de bandas 10 % PREGUNTA Cual es el agente causal mas probable en este caso especifico? a.- Listeria monocitogenes. b.- Stafilococco c.- Enterococcus. d.- Candida A. HIPOGLUCEMIA. CIENCIAS BASICAS: Hipoglucemia neonatal: debajo de 45 mg/dl (2.5 mmol/L) para prematuros como de término y a cualquier edad extrauterina. Hipoglucemia neonatal transitoria: se auto limita durante los primeros 7 días de vida extrauterina, es consecuencia de reserva energética limitada, excesivo consumo periférico, agotamiento precoz de reservas energéticas y a inmadurez del sistema hipotálamo-hipofisiario, responsable de la secreción de hormonas contra regulación. Hipoglucemia neonatal persistente o recurrente: 10 ml/kg/min persisten cifras menores de 45 mg/dl. Factores de riesgo: Madre diabética durante el embarazo o ingestión de betabloqueadores o hipoglucemiantes orales. Prematurez. Peso bajo para la edad gestacional. Peso grande para la edad gestacional. Detección tardía y manejo inadecuado de la hipoglucemia neonatal tiene un impacto sobre el desarrollo. Causas de hipoglucemia transitoria: Estrés perinatal, septicemia, asfixia, hipotermia, policitemia, choque. SALUD PUBLICA: En RN a término la incidencia está en un rango de 5-7% y pue de variar entre valores de 3,2 % a 14,7 % en recién nacidos pretérminos. Casi el 40% de los neonatos hijos de madre diabética tienen hipoglucemia. PATOGENIA: La etiología más frecuente de hipoglucemia en el recién nacido está de manera general asociada a incremento de la utilización de glucosa, a un aporte inadecuado de glucosa endógeno o exógeno o a una combinación de ambos. Una de las causas más frecuentes de hipoglucemia en el RN ligada a hiperinsulinismo fetal es el hijo de madre diabética mal controlada. En estos niños la hipoglucemia frecuentemente se produce a las 4-6 hrs después del nacimiento, tienen una reducción incrementada de insulina secundaria a sensibilidad aumentada de células beta del páncreas a la glucosa; que persiste durante varios días después del parto. Síntomas clásicos se deben a activación del SNA con la libera catecolaminas (hormona contrarreguladora de la hipoglucemia). Deprivación de glucosa al cerebro con alteración función neurológica. La administración antenatal de clopropamida, benxotiazidas, beta-simpaticomiméticos, propanolol o la administración de glucosa a la madre a alta concentración y supresión brusca de su administración puede inducir insulinismo fetal transitorio y por lo tanto hipoglucemia neonatal. La eritroblastocisis fetal en RN con incompatibilidad Rh y el síndrome de Beckwith-Wiedemann, se caracteriza por RN con macrosomia, onfalocele, macroglosia, visceromegalia e hipoglucemia. Se ha explicado la hipoglucemia por hipertrofia de las células beta del páncreas y por lo tanto hiperinsulinismo. Otras causas: por deficiencias hormonales: déficit de hormonas del crecimiento, deficiencia tiroidea. Causas hereditarias: enfermedad de orina de jarabe de arce, intolerancia a la fructosa, galactosemia. DIAGNOSTICO: Apnea, hipotonía, reflejo de succión inadecuada, irritabilidad, somnolencia, respiraciones irregulares, cianosis, temblores, palidez, crisis convulsivas, letargia, cambios en el nivel de conciencia, inestabilidad de la temperatura, coma. Triada de Whipple: Características clínicas, glucosa sérica baja, resolución de síntomas con corrección de glucemia. Estándar de oro:

CURSO ENARM CMN SIGLO XXI TEL: 36246001

Pharmed Solutions Institute

PÁGINA 365

MANUAL DE TRABAJO DEL CURSO ENARM CMN SIGLO XXI Determinación enzimática de los niveles de glucosa en laboratorio por el método de la hexocinasa. TRATAMIENTO: Debe iniciarse una alimentación precoz en las dos primeras horas de vida (de preferencia en los primeros 30-60 minutos) y establecerse intervalos de alimentación cada 2-3 horas. Hipoglcemia asintomática: 1. Si mediante tira reactiva los niveles de glucosa periférica se encuentran entre 27mg y 45 mg hay que tomar muestra sanguínea por punción venosa y corroborar niveles de glucosa, ofrecer inmediatamente sin esperar el resultados alimentación al seno materno y posteriormente cada hora. Si el RN no puede recibir seno materno de manera adecuada, suplementar con sucedáneo de leche materna cuyo volumen se ajustara con base al peso y se ofrecerá cada 3 hrs. 2. Si mediante tira reactiva los niveles de glucosa periférica se encuentran por debajo de 27mg/dl, corroborar con una glucosa central. Indicar infusión de glucosa IV para propiciar una infusión de glucosa kilo minuto de 6mg/kg/min. No suspender alimentación enteral. Si el tratamiento inicial fue ofrecer únicamente alimentación enteral y con ello se normalizo la glucosa, se recomienda: Indicar infusión de glucosa IV con líquidos a 80ml/kg/día para proporcionar una infusión de glucosa kilo minuto de 6 mg/kg/min. Se sugiere que en RN con hipoglucemia asintomática que requieran tratamiento con soluciones IV, el volumen de leche materna o sucedáneo de leche materna sea de 10-15 ml/kg/día. Hipoglucemia sintomática: Tomar glucosa central para corrobora. Administrar solución glucosada al 10% IV en bolo: Si el paciente presenta crisis convulsivas administrar 4ml/kg. Si el paciente no presenta crisis convulsivas, administrar 2ml/kg. Después de administrar el bolo establecer infusión continúa de glucosa IV de mantenimiento de 6mg/kg/min, una vez controlado aumentar 2mg/kg/min hasta 12 mg/kg/min. Si las condiciones lo permiten continuar alimentación oral con leche materna o sucedáneo de la leche materna en volumen de 10-15 ml/kg/min. Cada vez que se detecte hipoglucemia asintomática se deberá administrar bolo de solución glucosada al 10%. Si a pesar de tratamiento apropiadamente instaurado, el paciente persiste con hipoglucemia durante 3 determinaciones consecutivas, se recomienda referencia a tercer nivel en cualquiera de las siguientes situaciones: persistencia de hipoglucemia a pesar de recibir aporte de glucosa kilo minuto de 10 mg/kg/min. Reaparición de hipoglucemia al disminuir de manera apropiada el aporte de glucosa kilo minuto. Se recomienda referencia a tercer nivel cuando exista hipoglucemia asociada a: Antecedente familiar de muerte neonatal súbita o síndrome de Reye, crisis convulsivas o alteración del estado de consciencia asociados a hipoglucemia, alteraciones de la termorregulación, defectos de la línea media, exoftalmos, micropene. En hipoglucemia refractaria o persistente: Glucagón (0.1mg/kg IM, máximo 1 mg) o hidrocortisona. Referir a tercer nivel: Persista hipoglucemia a pesar de recibir aporte de glucosa kilo minuto de 10 mg. Reaparición de hipoglucemia al disminuir el aporte de glucosa. Crisis epilépticas. Alteraciones de la termorregulación. CASO CLINICO RN de 36 SDG obtenido por cesarea por desprendimiento de placenta. Al nacimiento presenta Apgar 8/9, peso en -0,89 DE, longitud en +1,75 DE, sin datos del perímetro cefálico. Con 48 horas de vida ingresada por ictericia en su hospital de referencia presenta hipoglucemias no cetósicas (glucemias entre 25-45mg/dl), insulinemia máxima registrada: 7μU/ml coincidiendo con glucemia de 33mg/dl. PREGUNTA Cual es la conducta a seguir para identificar la causa de la hipoglucemia. RESPUESTA a.- Verificar Diabetes Mellitus en la madre. b.- Realizar escaneo abdominal. c.- Verificar funcionamiento hepático. d.- Verificar funcionamiento suprarrenal. CASO CLINICO Se trata de reciencia nacido de 24 horas de edad, presenta en cunero las siguientes alteraciones: reflejo de succion inadecuado, crisis convulsivas letargia, inestabilidad de la temperatura. PREGUNTA Cual es la conducta mediata mas adecuada para este caso? RESPUESTA a.- Administracion rectal de diacepam. b.- Administración oral de glucosa. c.- Intubacion orotraqueal. d.- Glucosa periférica. CASO CLINICO Se trata de paciente masculino de 37 semanas de gestacion obtenido por cesarea, la madre presento cuadro de pre-eclampsia, luego de 24 horas de nacido presenta disminución de la succion, letargia, falta de respuesta a estimulos con disminución de tono muscular, asi como alteraciones de la temperatura con tendencia a la hipotermia, los laboratorios reportaron 35 mg/dl de glucosa periférica, resto de datos de laboratorio y gabinete dentro de parámetro normales, sin embargo posterior a la administración de glucosa mediante catéter periférico presenta nuevos cuadro de hipoglucemia? PREGUNTA Cual de los siguiente antecedentes es menos frecuente para la patologia que presenta el paciente? RESPUESTA a.- Displasia de células beta.

CURSO ENARM CMN SIGLO XXI TEL: 36246001

Pharmed Solutions Institute

PÁGINA 366

MANUAL DE TRABAJO DEL CURSO ENARM CMN SIGLO XXI b.- Enfermedad de orina de arce. c.- Galactosemia. d.- Hipotiroidismo congénito. ICTERICIA. CIENCIAS BASICAS: La ictericia es una de las condiciones más comunes que requieren atención médica en los RN y se refiere a la coloración amarillenta de piel y mucosas causada por la fijación de bilirrubina en el tejido graso subcutáneo; generalmente, se observa cuando los niveles séricos de bilirrubina son mayores o iguales a 5-7 mg/dl.. Las causas de ictericia neonatal son múltiples y producen hiperbilirrubinemia directa, indirecta o combinada, de severidad variable. La hiperbilirrubinemia se refiere al aumento de los niveles de bilirrubina total en sangre (≥ 2-5 mg/dl), como producto final de metabolismo del heme, componente esencial de la hemoglobina. En la mayoría de los casos, suele ser benigna, y autolimitada, pero por el efecto neurotóxico de la bilirrubina, los neonatos de riesgo deben ser vigilados para evitar hiperbilirrubinemia severa que produzca alteraciones neurológicas como encefalopatía aguda y kernicterus, los cuales son causas prevenibles de parálisis cerebral. Los neonatos amamantados son más propensos a desarrollar, ictérica fisiológica (aparece después de las 24 hrs y desaparece antes de los 10 días) en la primera semana de vida y cerca del 10% son ictéricos al mes de vida. Ictericia patológica; cuando inicia en las primeras 24 hrs, se acompañe de otros síntomas, la bilirrubina aumente >5mg/dl diarios, sobrepase los 15mg% o 10mg% en enonatos a término y pretermino, la fracción directa sea superior a 2mg/dl o dure más de una semana en el RN a término (excepto si recibe lactancia materna). SALUD PUBLICA: La ictericia fisiológica es una situación frecuente, aparece después del segundo día de vida como expresión de una condición fisiológica hasta en un 80% de los prematuros y en un 60% de los a término. Ictericia patológica en 6% de RN. PATOGENIA: Factores de riesgo: Alimentación a pecho. Mayor pérdida de peso (más de 5%). Sexo masculino. Edad gestacional < 35 semanas. Diabetes materna. Hematomas. Raza Oriental. La principal causa de la apricion de la ictérica fisologica en el RN es la inmadurez del sistema enzimático del hígado, a esto se le suma una menor vida media del glóbulo rojo, la poliglobulia (el RN produce el doble de bilirrubina que el adulto), la extravasación sanguínea frecuente, el RN reabsorbe gran parte de la bilirrubina a través de la circulación enterohepatica y la ictérica por lactancia. Ictericia patológica, las causas más frecuentes son 1. Enfermedad hemolítica (incompatibilidad sanguínea materno-fetal, esferocitosis familiar, déficit de enzima G-6-PD), las cuales disminuyen la vida media de los eritrocitos. 2. Hematomas y hemorragias (cefalohematomas), cuya reabsorción aumenta la oferta de bilirrubina. 3. Incremento en la reabsorción intestinal como sucede en el retraso en la alimentación gástrica en RN enfermos o la presencia de obstrucción intestinal. 4. Policitemia, por mayor volumen y destrucción globular. 5. Defectos enzimáticos congénitos como Sx. De Crigler-Najjar. 6. Ictericia acolurica familiar transitoria (Sx. De Lucey-Driscoll), se presenta en RN cuyas madres son portadores de factor inhibitorio en el suero que impide la conjugación. ICTERICIA POR INCOMPATIBILIDAD DE FACTOR Rh: Es la causa más frecuente de ictérica patológica y el 97% de los casos se debe a isosensibilizacion para el antígeno Rh D. Un alto título materno de anticuerpos anti-Di ≥ 64 de se asocia a un riesgo elevado de hiperbilirrubinemia severa para los recién nacidos. La administración de profiláctica de inmunoglobulina Anti-D. DIAGNOSTICO: El tinte ictérico no solo está en la piel y conjuntivas, sino que también puede apreciarse en el LCR, lágrimas, saliva y especialmente en los casos patológicos. Es conveniente valorar la presencia de coluria y acolia, ya que son datos de gran valor diagnóstico. La presencia de hepatomegalia precoz es sugestiva de infección prenatal o de enfermedad hemolítica por incompatibilidad Rh; cuando esta es dura y de aparición más tardía, hará pensar en la posibilidad de afectación hepática primitiva (hepatitis, atresia), si bien en estos casos el resto de signos clínicos son distintos. La esplenomegalia hará sospechar que actúa como un foco hematopoyético extramedular o bien que el RN padece una infección prenatal con manifestaciones clínicas. Por la frecuencia con la que se presenta al tercer día de vida una hiperbilirribinemia secundaria a la reabsorción de hematomas, se deberán buscar colecciones de sangre extravasada. La ictericia inicia en cara u tienen una progresión cefalocaudal, útil para valorar el grado de ictericia. Otro síntoma frecuentemente asociado a hemolisis es la hipoglicemia, como resultado de la hiperpalsia pancreática. La presencia de petequias y purpura sugieren la posibilidad de infección connatal. Laboratorio: Dosaje de bilirruibinemia total y directa, si hay elevación de bilirrubina indirecta, sugiere hemólisis. Si hay elevación de la bilirrubina directa, sugiere enfermedad hepatobiliar. Reacción de Coombs directa e indirecta. Hematocrito y hemoglobina (para valorar la presencia de anemia asociada). Recuento de reticulociotos. TRATAMIENTO: Las alternativas son: 1. Fototerapia; la de elección y más difundida, su administración redujo en gran medida el uso de exanguineotransfusion, actúa por fotooxidacion (destrucción física de la bilirrubina en productos más pequeños y polares para ser excretados) y fotoisomerización, el RN debe estar desnudo, cubrir los ojos, y control térmico, actualmente existe fototerapia en fibra óptica 2. Exanguinotransfusion, se basa en la remoción mecánica de sangre del RN por sangre de un donador, ha sido reemplazada por la fototerapia, se reserva en especial para

CURSO ENARM CMN SIGLO XXI TEL: 36246001

Pharmed Solutions Institute

PÁGINA 367

MANUAL DE TRABAJO DEL CURSO ENARM CMN SIGLO XXI enfermedades hemolíticas severas, cuando la fototerapia no ha resultado eficaz. Terapia farmacológica; mesoporfirina, inhibe el catabolismo del hemo y por lo tanto la producción de bilirrubina. Fenobarbital, es un inductor enzimático que estimula las etapas de captación, conjugación y excreción de bilirrubina. Por estar asociado a un potencial desarrollo de adicción, sedación excesiva y efectos metabólicos adversos, han limitados su aplicación en el RN. Administración Oral de sustancias No absorbibles: estos al captar bilirrubina en la luz intestinal, reducen la absorción enteral de ésta y, así se puede disminuir los niveles de bilirrubina sérica. TOXICIDAD DE LA BILIRRUBINA: Hay dos fases en la neurotoxicidad de la bilirrubina una temprana y aguda que es reversible si el pigmento es removido y una lenta y tardía cuyos efectos son irreversibles. Los signos clínicos de toxicidad aguda son apatía, somnolencia o insomnio, junto con la alteración de los potenciales evocados auditivos, pero que luego revierten, una vez que los valores de bilirrubina descienden. La encefalopatía por bilirrubinas es un síndrome neurológico que resulta del deposito de bilirrubina no conjugada en el SNC, especialmente en los ganglios basales y nucleos del tallo cerebral. La causa de esta encefalopatía es de origen multifactorial relacionada con niveles elevados de bilirrubina no conjugada libre, cantidad de bilirrubina que se une a la albúmina, alteración de la barrera hematoencefálica (BHE) por otras enfermedades y susceptibilidad neuronal. CASO CLINICO RN presenta ictericia, que se hizo evidente en el segundo día de la vida. Nació de padres no relacionados a los 36 semanas de gestación con un peso al nacer 2800 g, después de embarazo y el parto expontaneo y normal aparentemente. Al examen físico, paciente estaba activo y cómodo. Sus signos vitales eran estables. En recuento de admisión de sangre completa fue en normal límites. PREGUNTA Cual es la conducta a seguir. RESPUESTA a.- Alta y medidas en casa. b.- Se ingresa para fototerapia. c.- Verifica niveles de bilisrrubinas. d.- Verifica grupo y factor en ambos padre. KERNICTERUS. CIENCIAS BASICAS: Es la coloración amarilla de los ganglios basales producida por impregnación de bilirrubina, descrita en autopsias de RN fallecidos con severa ictericia. Constituye la complicación más grave de la ictericia neonatal. El kernicterus es la secuela más importante de la encefalopatía bilirrubínica. Es una enfermedad devastadora, una entidad previsible cuando la hiperbilirrubinemia es tratada agresivamente. SALUD PUBLICA: Su incidencia aumento con las nuevas políticas de alta prematura, esto causa mayor riesgo de complicaciones debidas a ictericia temprana no detectada. PATOGENIA: Hay varias situaciones que alteran la barrear hematoencefalica y facilitan la entrada de bilirrubina al SNC, aumentando notablemente el kernicterus, como son; bajo peso al nacer, hipoglucemia, asfixia neonatal, acidosis metabólica, hemolisis, hipotermia-frio, hipoalbuminemia, drogas que compiten por la unión a albumina, diestres respiratorio. La bilirrubina no conjugada penetra en el cerebro y actúa como una neurotoxina, a menudo ésta se asocia con condiciones que dificultan la función de la barrera hemato-encefálica (ejemplo, sepsis), inhibiendo varios procesos bioquímicos muy importantes, como la fosforilación oxidativa de las mitocondrias y la síntesis proteica. Es necesario anotar que se desconocen los niveles toxicos de bilirrubina para SNC. Revisiones recientes han sugerido que 25mg% y aun unos puntos más serían los tóxicos. DIAGNOSTICO: Puede Ser asintomático en prematuros pequeños. Se caracteriza por atetosis, sordera neorosensorial parcial o completa, limitación de la mirada vertical, déficit intelectual, displasia dental. En la forma clásica se reconocen 3 estadios: Primera fase; caracterizada por inicio con vómitos, letargia, hipotonía, rechazo al alimento, succión débil y llanto agudo. Segunda fase; se caracteriza por irritabilidad, hipertonía y opistótonos. Tercera fase; observada en sobrevivientes de las 2 anteriores y caracterizada por la triada de hipertonía, atetosis y otros movimientos extrapiramidales y retardo psicomotor. Pueden quedar secuelas alejadas siendo las más frecuentes la sordera, los trastornos motores y los problemas de conducta. Las regiones del cerebro más comúnmente afectadas son los ganglios basales, particularmente los núcleos subtalámicos y el globo pálido, el hipocampo, el cuerpo geniculado, varios núcleos cerebrales, incluyendo el colículo inferior, vestibular, oculomotor, coclear y olivar inferior, y el cerebelo, especialmente el núcleo dentado y el vérmix. La necrosis posnatal es el hallazgo histopatológico dominante después de los 7-10 días de vida posnatal. El diagnóstico puede ser confirmado por resonancia nuclear magnética (RNM) cerebral, cuya imagen característica es de tipo bilateral, con alta señal de intensidad en el globo pálido, vista en los cortes de T1 y T2. Estas imágenes también se pueden observar en el hipocampo y el tálamo. CASO CLINICO RN de 35 SDG, vigoroso, peso 2,400 g, perímetro cefálico 34 cm, talla 47 cm, sin patología perinatal, alimentado a pecho direcho exclusivo, alta a las 48 horas, regreingresa a los 6 dias de vida por ictericia generalizada, con peso al ingreso de 2,130 grs. Succion vigorosa pero breve e ineficaz, escasas diuresis y deposiciones. Bilisrrubinas totales 32,52 mg/dl, bilisrrubina indirecta 32 mg/dl, Bh normal, glicemia normal. La EF se observa leve temblor distal PREGUNTA Cual es la conducta mas adecuada a seguir en este caso? RESPUESTA a.- Colchon de fibra óptica de luz continúa. b.- Exanguinotransfusion. c.- Fototerapia. d.- Luz halogenada.

CURSO ENARM CMN SIGLO XXI TEL: 36246001

Pharmed Solutions Institute

PÁGINA 368

MANUAL DE TRABAJO DEL CURSO ENARM CMN SIGLO XXI ATRESIA ESOFAGICA (AE) Y FISTULA TRAQUEOESOFAGICA (FTE). CINECIAS BASICAS: La atresia de esófago es una anomalía congénita en la cual la porción media del esófago está ausente (atresia); en un porcentaje importante se acompaña de una comunicación anormal entre la traquea y el segmento distal del esófago llamada fistula traqueoesofágica. La atresia de esófago es una malformación incompleta de la luz esofágica. La variante más frecuente es la tipo C de la clasificación de Gross o la III de Voght en 87% de los casos. Factores de riesgo: y agravantes del pronóstico: malformaciones congénitas, neumonías, bajo peso al nacer. Se encuentran anomalías cromosómicas entre 6–10% de los casos; las más frecuentes son las trisomías 18 y 21. SALUD PUBLICA: Es la malformación esofágica más frecuente. La atresia esofágica con o sin fístula traqueoesofágica es una alteración que aparece en 1:3,000 a 1:4,500 recién nacidos. El 95% de la AE tiene fístula traqueoesofágica asociada. El 50% de los casos se asocia con otras malformaciones congénitas y existe riesgo de recurrencia de 2 a 3%, y mayor riesgo relativo si se tienen hijos o familiares afectados. En virtud que en México nacen aproximadamente 2 millones de niños por año, se estima que cada año hay entre 500 y 600 casos nuevos de niños con atresia de esófago. PATOGENIA: Malformaciones congénitas asociadas: Cardiacas (35%): comunicación interventricular, conducto arterioso persistente, tetralogía de fallot, coartación de aorta. Gastrointestinales (20%): ano imperforado, atresia duodenal, malformaciones intestinales. Genitourinarias (20%): reflujo ureteral, agenesia renal. Músculo esquelético (13%): vertebrales, costales, defecto de las extremidades y del sistema nervioso central (10%). La asociación de algunas de estas anomalías se denomina asociación VACTERL (Vertebrales, Anoreactales, Cardiacas, Traqueales, Esofagicas, Radiales, renales y de extremidades (L; limbs en inlges) y se presenta con una frecuencia de 10%. El origen de la AE es poco claro aún pero se atribuye a una alteración en la migración de los pliegues laterales o a una detención del crecimiento en el momento de la evaginación. En la mayor parte de los casos el esófago posterior no se separa totalmente de la tráquea, lo que da lugar a distintas variedades de fístula traqueoesofágica o a hendiduras Esta alteración se produce entre la tercera y sexta semana de gestación. Fístula traqueoesofágica (TEF) Una fístula es una conexión entre el esófago y la tráquea. Esta conexión permite que la comida (desde el esófago) entre en los pulmones (aspiración). La comida en los pulmones puede causar neumonía (referida a neumonía por aspiración), lo cual puede ser muy serio. Si hay una fístula traqueoesofágica con atresia esofágica, toda la comida ingerida terminará en los pulmones, como el esófago no está conectado con el estómago, porque finaliza cerrado. El tipo más difícil para diagnosticar es la fístula traqueoesofágica tipo H, donde el esófago no termina cerrado, pero hay una fístula o conexión entre la tráquea y el esófago. En la fístula traqueoesofágica tipo H, el aire que entra al estómago (desde la tráquea) y la comida puede entrar a los pulmones. El aire en el estómago puede causar hinchazón en el abdomen del bebé y puede hacer que el bebé está molesto. DIAGNOSTICO: El diagnóstico prenatal se establece con ultrasonografía y puede sospecharse durante el embarazo por polihidramnios, incapacidad de identificar el esófago fetal o ausencia de burbuja gástrica. Diagnostico en sala de parto, la complicación al paso de la sonda para verificar la permeabilidad del esófago; sialorrea, distensión abdominal, dificultad respiratoria, tos o cianosis son algunas manifestaciones en el recién nacido. Al darle de comer: vómitos/atragantamiento (no pasa el contenido a estómago), crisis de sofocación con tos tras las tomas lo que sugiere fístula superior. Puede apreciarse abdomen escavado a la inspección o abdomen distendido si fístula inferior. De persistir la alimentación sin diagnosticarse: vómitos o neumonías de aspiración. Ante la sospecha debe realizarse sonda nasogástrica: no pasa hasta estómago; se enrolla en bolsón. No necesario contraste por que las sondas son radiopacas y hacer estudio estudio radiográfico para confirmar el diagnóstico; el estudio muestra el fondo de saco, ciego, del esófago atrésico o, bien, puede realizarse un examen endoscópico para confirmar la fístula. Se han evaluado la resonancia magnética, TAC, estudios de medicina nuclear, angiografía y ultrasonografía sin resultados concluyentes. CLASIFICACION: Clasificación de Vogt modificada por Ladd de la atresia de esófago: AE TIPO I (5-8%): Ambos cabos esofágicos ciegos sin fístula traqueoesofágica: Es de fácil diagnostico en el embarazo por que cursa con polihidramios y ausencia de imagen gástrica. Abdomen excavado por falta de pasaje de aire al intestino. Ambos cabos se encuentran muy separados entre sí. AE TIPO II (0.5-1%): Fístula traqueoesofágica superior y cabo inferior ciego. AE TIPO III (80-85%): Fístula traqueoesofágica inferior y cabo esofágico superior ciego: En éstos pacientes el reflujo gastroesofágico puede ser lesivo para los pulmones, son propensos a sufrir Neumonías Químicas. AE TIPO IV (0.5-1%): Fístula traqueoesofágica en ambos cabos del esófago. AE TIPO V (3-5%): Fístula en H o N. Es una fístula traqueoesofágica sin atresia de esófago: El diagnostico se realiza en la infancia ya que aparecen los síntomas en ese periodo. AE TIPO VI (0.-1%): Estenosis esofágica aislada. Tipos de atresia esofágica, clasificación de Gross: A. Atresia esofágica sin fístula: 3-5%. B. Atresia esofágica con fístula proximal: 2%. C. Atresia esofágica con fístula distal: 80-90%. D. Atresia esofágica con fístula proximal y distal: 3-5%. E. Fístula traqueoesofágica sin atresia: 6%. F. Estenosis esofágica. TRATAMIENTO: El adecuado ambiente térmico, el suministro de líquidos, glucosa y electrólitos, la posición adecuada del paciente (semisentada, con la cabeza elevada en 30° a 40°), sonda doble lumen. Drenaje del cabo proximal con sonda de doble lumen (sonda de Repogle) para aspiración constante), la aspiración cuidadosa y frecuente de la saliva y las secreciones acumuladas en el cabo ciego, si existe neumonía, iniciar doble esquema de antibiótico (ampicilina mas aminoglucocido). Intervención quirúrgica, interrumpir la comunicación entre la tráquea y el esófago. Reestablecer la continuidad del Esófago. Preservar el esófago existente. Gastrostomía y esofagectomía: Plastia esofágica, plastia esofágica y cierre fístula, sustitución esofágica, cierre fístula. Atresia tipo III: dentro de las primeras 24 hrs de vida se realizara cierre de FTE inferior y, si la distancia entre los cabos es menor de 3 cm, anastomosis termino-terminal (T-T) entre los cabos esofágicos a través de una toracotomía con abordaje extrapleural. COMPLICACIONES: Reflujo gastroesofágico. Refistulación traqueo esofágica. Estreches anastomótica. Traqueomalasia. Dismotilidad esofágica. CASO CLINICO Recién nacida, gemela I, de madre sana de 35 años de edad, producto del segundo embarazo, de 35 semanas de duración; el embarazo anterior terminó con aborto espontáneo en el segundo trimestre, seis años antes. El embarazo actual tuvo adecuado control prenatal, recibió ácido fólico, calcio, hierro y polivitaminas; se complicó por amenaza de aborto en el primer trimestre, pero se recomendó reposo. En la semana 31 del embarazo se le realizó ultrasonografía obstétrica que reportó. Embarazo gemelar con fetos vivos, placenta única, corporal, posterior, grado II y edad gestacional de 31 semanas, 3 dias antes del parto la madre tuvo infección de las vías urinarias,

CURSO ENARM CMN SIGLO XXI TEL: 36246001

Pharmed Solutions Institute

PÁGINA 369

MANUAL DE TRABAJO DEL CURSO ENARM CMN SIGLO XXI por lo que se hospitalizo para tratamiento y recibir esquema de tres dosis de inductores de la maduración pulmonar, tuvo rotura de membranas 4 horas antes del parto, se realizo bloqueo peridural y se realizo cesarea, al nacimiento el Apgar fue de 6/8 con peso de 2,000 g, dificultándose la aspiración de secresiones, continuando con tos, sialorrea, y cianosis leve. PREGUNTA Cual es la conducta a seguir inmediata mas adecuada? RESPUESTA a.- Colocar en silla porta-bebe a 45 grados. b.- Colocar sonda de doble lumen. c.- Prepara para cirugía. d.- Aplicación de oxigeno a 3 lt/x´. HERNIA HIATAL CONGENITA. CIENCIAS BASICAS: Sede be al cierre incompleto de la membrana pleuroperitoneal (abertura o dehiscencia congénita del diafragma) o al retorno prematuro del intestino a la cavidad abdominal con herniación de las vísceras abdominales a la cavidad torácica. SALUD PUBLICA: La hernia diafragmática congénita ocurre entre 1 en 2,000 a 1 en 5,000 recién nacidos vivos. La mortalidad varía entre un 40-70% dependiendo el grado de hipoplasia pulmonar y el desarrollo de hipertensión pulmonar. La relación hombre: mujer es de 1:1,8. El 20-53% presentan malformaciones asociadas, sobre todo defectos cardiacos (923%), defectos del tubo neural (28%), trisomías y ciertos síndromes bien definidos. PATOGENIA: Existen dos teorías acerca de la embriogénesis de la hernia diafragmática: 1. Crecimiento pulmonar anormal que produce desarrollo diafragmático anormal. 2. Defecto diafragmático con hipoplasia pulmonar secundaria. La etiología de la hernia diafragmática no es clara, han sido reportados casos familiares sugiriendo predisposición genética. En 20% de los casos se ha presentado polihidroamnios y es de mal pronóstico. Talidomida y quinidina se han reportado como causantes de hernia diafragmática en humanos. Las anomalías en la formación anatómica normal del diafragma explican la aparición de defectos congénitos. Si la detención del desarrollo diafragmático se produce precozmente en útero, el RN presenta una amplia comunicación entre tórax y abdomen. Si la detención de la formación se produce después de formado el tabique membranoso pero antes de formarse la caja muscular, el niño presenta un saco herniario que contiene los órganos desplazados hacia arriba. Esta patología se asocia prácticamente siempre con efecto de masas, que se manifiesta en el feto por desplazamiento del mediastino, compresión pulmonar y reducción del tejido pulmonar antes de la semana 16, momento en el que desarrollo bronquial es completo determina reducción del número de bronquios y alveolos del pulmón en desarrollo que conlleva a la hipoplasia pulmonar. El pulmón hipoplásico en hernia diafragmática presenta unareducción cuantitativa y cualitativa de surfactante y un compromiso de la distensibilidad pulmonar. CLASIFICACION: De acuerdo a su localización anatómica: 1. Hernia posteroexterna de Bochdalek es el tipo más común representa 85-90%, siendo la mayor ubicación izquierda 80%, derecha 15%, y mixta 5%. 3. Hernia de hiato esofágico. 4. Hernia retroesternal de Morgagni. DIAGNOSTICO: Algunos neonatos con este defecto no tienen manifestaciones clínicas perceptibles y se desarrollan sin problemas excepto cuando por alguna razón por lo general una infección respiratoria o problemas gastrointestinales se descubre en un estudio radiológico que tienen hernia diafragmática. El diagnóstico prenatal se realiza por ecografía, se basa en la visualización de órganos abdominales en el tórax y el signo ecográfico distintivo es una masa ocupada por líquido inmediatamente por detrás de la aurícula y el ventrículo izquierdos, en la parte inferior del tórax visualizando en una vista transversal. Otros signos ecográficos que hacen sospechar el diagnóstico son la ausencia del estómago en el abdomen, desplazamiento del mediastino, perímetro abdominal fetal pequeño y polihidramnios. TRATAMIENTO: El tratamiento incluye terapias como el uso de corticosteroides, la ventilación de alta frecuencia, oxigenación con membrana extracorpórea (ECMO), terapia con surfactante, óxido nítrico y cirugía fetal. HERNIA DE BOCHDALEK: Puede ser de presentación esporádica o familiar. El defecto ocurre cuando la membrana pleuroperitoneal no se fusiona con las otras porciones. La fusión de la membrana pleuroperitoneal ocurre primero del lado derecho y luego del izquierdo. Además la protección del hígado en el lado derecho, hace que sea más frecuente la hernia de Bochdalek del lado izquierdo. La hipoplasia pulmonar se debe a que los pulmones se encuentran en la fase glandular del desarrollo y el intestino torácico ocupando espacio, impide el normal desarrollo. Por lo tanto existe una disminución del lecho vascular y de la segmentación bronquial. Es frecuente la mal rotación de los intestinos debido a que los intestinos se desplazan al tórax antes de la fijación del ciego al cuadrante inferior derecho del abdomen. HERNIA DE MORGAGNI: La hernia de Morgagni es una anomalía congénita del diafragma, causada por un defecto anteromedial del diafragma, entre sus inserciones costal y esternal, que fue descrito por primera vez por Morgagni, Los pacientes con hernia de Morgagni se diagnostican a cualquier edad en forma accidental al efectuar un examen radiológico de tórax por otra patología, tal como infecciones respiratorias o molestias gastrointestinales, un considerable número de pacientes se diagnostica en la edad pediátrica por presentar dificultad respiratoria. El diagnóstico de hernia de Morgagni se lo efectúa en la edad adulta, la mayoría de las veces en forma incidental, y en los niños por presentar dificultad respiratoria. Su patogenia es desconocida, aunque actualmente se sugiere una etiología multifactorial en la que se implican factores hereditarios en relación con otros síndromes malformativos como los síndromes de Down, Turner, Prader-Willi y Nooan. El tratamiento es eminentemente quirúrgico en todos los casos y consiste en la plastia del defecto diafragmático; hoy en día varios centros han reportado exitosas reparaciones por vía laparoscópica. CASO CLINICO Lactante de 5 meses de edad, femenino, eutrófico, con historia de cuadros de aparente dolor abdominal a repetición. Consultó por presentar síntomas respiratorios, fiebre e irritabilidad, se solicitó radiografía de tórax en proyecciones anteroposterior y lateral que muestra reducción del parénquima pulmonar en hemitórax derecho y desplazamiento del mediastino y del corazón hacia la izquierda. PREGUNTA Cual es la conducta diagnostica siguiente mas adecuada? RESPUESTA a.- Serie gastroesofágica.

CURSO ENARM CMN SIGLO XXI TEL: 36246001

Pharmed Solutions Institute

PÁGINA 370

MANUAL DE TRABAJO DEL CURSO ENARM CMN SIGLO XXI b.- Serie gástrica completa. c.- Tomografia de abdomen. d.- Resonancia magnética. CASO CLINICO Recién nacido varón, que nace por Cesárea, SGB +, Transaminasas Maternas elevadas. Apgar 6/7, Peso 3670 gramos, talla 51 centímetros. Presenta, en el postparto inmediato, dificultad respiratoria con cianosis, aleteo nasal, quejido, tiraje intercostal, crepitantes bibasales, disminución de entrada de aire y del murmullo vesicular en hemitorax Izquierdo. PREGUNTA Cual es la conducta diagnostica mas adecuada? RESPUESTA a.- Serie gastroesofágica. b.- Radiografia de torax. c.- Tomografia de abdomen. d.- Endoscopia superior. PREGUNTA Cual de las siguientes manifestaciones es menos frecuente en esta patología? RESPUESTA a.- Insuficiencia Respiratoria Severa desde el nacimiento. b.- Disminución o ausencia de murmullo vesicular. c.- Ruidos hidroaéreos (RHA) en tórax. d.- Desplazamiento de ruidos cardiacos al lado contralateral. PREGUNTA Cual es la trisomía que no se ha relacionado en esta patología? RESPUESTA a.- X. b.- 18. c.- 21. d.- 13. ESTENOSIS HIPERTROFICA DEL PILORO. CIENCIAS BASICAS: La estenosis hipertrófica del píloro anteriormente se conocía como hipertrofiacongénita del píloro; también se le ha denominado estenosis pilórica hipertrófica infantil, para diferenciarla de la estenosis adquirida que se observa en el adulto. La estenosis hipertrófica del píloro es la causa más frecuente de cirugía en los lactantes menores de 6 meses, superada solamente por las hernioplastias. SALUD PUBLICA: Se estima que hay entre uno y cinco pacientes con esta patología por cada 1,000 recién nacidos vivos. 7% asociado a malformaciones: malrotación intestinal, uropatía obstructiva, atresia esofágica, hernia hiatal. Se presenta entre las 2 y 8 semanas de edad, con un pico entre las 3 y las 5 semanas. Es 4 a 5 veces más común entre varones que en mujeres, con una mayor incidencia en primogénitos. PATOGENIA: La causa exacta de la estenosis del píloro no se ha determinado todavía, pero se han desarrollado algunas teorías relacionadas con un desequilibrio neurohormonal, o de mediadores neuroendocrinos, en el control del tono del esfínter pilórico, en la producción de gastrina y la motilidad del estómago. Una propuesta es una descoordinación entre el peristaltismo gástrico y la relajación pilórica, lo que lleva a una contracción gástrica contra un píloro cerrado, que causaría hipertrofia en el músculo pilórico. Otras teorías proponen una elevación en las concentraciones de gastrina (hipergastrinemia), debido a un aumento hereditario en el número de células parietales de la mucosa gástrica que llevan a un ciclo de aumento en la producción de ácido gástrico, contracciones cíclicas periódicas en el píloro y vaciamiento gástrico lento, esto lleva a hipertrofia e hiperplasia de las fibras musculares del esfínter pilórico. Se ha propuesto herencia autosómica dominante cromosoma 16 q24 y herencia “multifactorial”. Administración de eritromicina/ azitromicina los primeros días de vida, se ha encontrado que la administración de macrólidos a las madres que dan alimentación al pecho también puede ser un factor para que los lactantes presenten estenosis pilórica. El hábito de fumar materno se ha reportado como posible factor de riesgo para estenosis pilórica. Otras investigaciones señalan que al estudiar muestras de las capas musculares depacientes afectados de estenosis pilórica, en comparación con controles, se hanencontrado cantidades disminuidas de las terminales nerviosas y de los neurofilamentos; disminución en los marcadores para células de soporte; disminución en las células intersticiales de Cajal; disminución en la actividad de la sintetasa de óxido nítrico, el cual actúa como relajante del músculo liso en diversos tejidos; disminución en la producción del ARN mensajero para la sintetasa de óxido nítrico. DIAGNOSTICO: El síntoma más característico son los vómitos posprandiales, no biliosos, progresivos hasta ser incluso en proyectil, que eventualmente impiden la alimentación adecuada del lactante. Esto lleva a pérdida de peso por disminución del aporte de calorías y por deshidratación. Por lo general el recién nacido ha tenido previamente un período libre de vómitos. De los antecedentes es importante indagar por el uso de eritromicina o macrólidos en las dos primeras semanas de vida, además de los antecedentes familiares. Conforme continúan los síntomas el paciente adelgaza (pierde peso) y se torna hambriento,toma con avidez la leche, pero rápidamente la vomita; se puede observar el vómito explosivo o en proyectil. Algunos pacientes presentan ictericia (10%) de grado variable. También se pueden encontrar signos clínicos de deshidratación como mucosas con saliva filante o secas, depresión de la fontanela, llenado capilar lento y signo del pliegue presente (se suma a la pérdida de peso). El abdomen no está distendido; cuando la emaciación ha progresado, es posible observar las ondas peristálticas en el epigastrio. El signo

CURSO ENARM CMN SIGLO XXI TEL: 36246001

Pharmed Solutions Institute

PÁGINA 371

MANUAL DE TRABAJO DEL CURSO ENARM CMN SIGLO XXI patognomónico es la masa palpable en el cuadrante superior derecho del abdomen, en forma de aceituna u oliva, que corresponde al píloro engrosado (oliva pilórica). Para encontrar la oliva es necesario que el paciente esté tranquilo y la musculatura abdominal relajada. Los porcentajes del hallazgo varían entre los autores y dependen de la experiencia del explorador; algunos indican cifras que van de 85 a 100 %. Signo del biberón: al tomar líquidos por biberón se observan ondas peristálticas. Signo de la pelota de golf: ondas peristálticas de izquierda a derecha hacia la zona pilórica. El hallazgo más característico es la alcalosis metabólica, por la pérdida de hidrogeniones a través del contenido gástrico; con el vómito se pierde también cloruro lo que lleva a hipocloremia, que completa el panorama de la bioquímica sanguínea. También puede haber hipokalemia. Se puede solicitar hemograma completo para descartar algún proceso séptico como causa de los vómitos. En la estenosis del píloro el hemograma está normal; se describe sin embargo que puede haber hemoconcentración por la deshidratación. Por ello puede encontrarse elevación de la creatinina; la densidad urinaria también puede estar elevada, sin datos de infección. Otro hallazgo es la elevación de la bilirrubina indirecta, que se puede explicar por la disminución del tránsito gastrointestinal que produce aumento de la circulación enterohepática de bilirrubina, aunque otros autores mencionan disminución de la actividad de la glucoroniltransferasa por el ayuno. La radiografía simple de abdomen muestra distensión de la cámara gástrica con poco o escaso aire distal. La serie esófago gastroduodenal muestra un estrechamiento del canal pilórico (cola de ratón) con una o varias imágenes de «cuerda»; es el medio de contraste que dibuja los pliegues hipertrofiados de la mucosa pilórica; se describe también un efecto de masa que se proyecta hacia el antro gástrico (signo del hombro); se observa falta de progresión de las ondas peristálticas del estómago en la fluoroscopia. En la actualidad, el método más utilizado para confirmar el diagnóstico clínico es el ultrasonido abdominal. Se logra evidenciar engrosamiento de la capa muscular del píloro mayor de 2.5 mm, aumento en la longitud del canal pilórico mayor de 15 mm y estrechamiento del canal; se logra visualizar también hipertrofia de la mucosa en grados variables, corte axial Imagen “ojo de bovino” “dona” Imagen tiro al blanco. La endoscopia tiene sensibilidad y especificidad 100%. TRATAMIENTO: El tratamiento actualmente es quirúrgico. Sin embargo se deben corregir los trastornos hidroelectrolíticos (alcalosis, hipocloremia, hipokalemia y deshidratación) antes de que el paciente sea llevado al quirófano. Se pueden suministrar bolos de solución de cloruro de sodio al 0,9 % (solución salina normal) a 20 mL/kg para restituir volumen y electrolitos. Luego se puede dejar una solución de mantenimiento, con dextrosa al 5 % y cloruro de sodio al 0,45 % o al 0,9 %, según el centro hospitalario, además se debe agregar cloruro de potasio a razón de 20 mEq/L (3.4mEq/kg/día) una vez asegurada la diuresis. La estabilización puede demorar unas 24 a 48 h. La cirugía no es urgente; si el paciente es llevado a sala de operaciones con alcalosis metabólica se aumenta el riesgo de apnea posoperatoria; además, el estrés quirúrgico puede empeorar los trastornos electrolíticos. Actualmente es raro observar desnutrición grave como en años atrás, cuando el diagnóstico se retrasaba. Se debe mantener una sonda nasogástrica a drenaje que ayuda a descomprimir el estómago, previene la aspiración posoperatoria y también la atonía gástrica. Niño con electrólitos normales menos del 5% de la deshidratación cirugía inmediata. Vitamina K: 0.4 mg/kg en niños con peso inferior a 2.5 kg. > 1 año 5-10 mg/día por vía I.V. o I.M. Ranitidina 1 mg/kg/dosis cada 8 hrs. Una vez compensado el paciente se lleva a cabo la Piloromiotomía de Fredet- Ramsted, se realiza el corte de la serosa y luego de la capa muscular circular del píloro, sin cortar la mucosa; luego se deja sin suturar la muscular y se cierra la serosa. La operación generalmente se hace por laparotomía, aunque también se puede a través de técnicas laparoscópicas. COMPLICACIONES: Las complicaciones están relacionadas con los trastornos electrolíticos y con la cirugía. Se citan principalmente: apnea posoperatoria, hipoglicemia, obstrucción posoperatoria (vómitos), perforación de la mucosa (duodenal), miotomía incompleta, eventración de la herida quirúrgica; el fallecimiento ocurre en menos del 1 % de los casos CASO CLINICO Se trata de un paciente masculino de cuatro semanas de edad, el cual acude por historia de 2 dias de evolución caracterizado por vomitos postpandriales abundantes, se establecen medidas antireflujo, sin embargo continuo la sintomatología aumentando el vomito en proyectil de contenido alimentario. PREGUNTA Cual es la conducta diagnostica mas adecuda en este momento? RESPUESTA a.- Radiografia de torax. b.- USG abdominal. c.- Radiografia de abdomen. d.- Endoscopia. PREGUNTA Cual de los siguientes diagnosticos diferenciales es el mas frecuente en esta patología? RESPUESTA a.- Intolerancia a la lactosa. b.- Reflujo gastroesofágico. c.- Meningitis. d.- Volvulus.

CURSO ENARM CMN SIGLO XXI TEL: 36246001

Pharmed Solutions Institute

PÁGINA 372

MANUAL DE TRABAJO DEL CURSO ENARM CMN SIGLO XXI CARDIOPATIAS CONGENITAS CIENCIAS BASICAS: Corazón primer órgano que alcanza desarrollo (de 3ra-8va semana de gestación) funcional completo, el día 17 de gestación escuchamos latido cardiaco. Definición: Conjunto de enfermedades caracterizadas por anomalías estructurales del corazón o de los grandes vasos intratorácicos. Los cambios en la transición de la circulación fetal a la neonatal; en el feto existen 3 estructuras únicas: el conducto venoso, el agujero oval y el conducto arterioso. La sangre oxigenada llega de la vena umbilical a la vena cava inferior a través del conducto venoso, evitando el hígado. Gracias a la anotomía de la aurícula derecha, la mayor parte de la sangre proveniente de la vena cava inferior (sangre oxigenada) se dirige al corazón izq. Por medio del agujero oval, mientras que la sangre proveniente de la vena cava superior (sangre desoxigenada), se dirige al ventrículo derecho a través de la válvula tricúspide. La sangre que irriga el pulmón tienen un contenido de oxigeno sumamente bajo y esto produce una constricción intensa de los vasos pulmonares que eleva la resistencia vascular. El ventrículo der. bombea la sangre hacia la arteria pulmonar; sin embargo la alta resistencia pulmonar hace que la sangre fluya principalmente a través del conducto arterioso hacia la aorta descendente. Los cambios fisiológicos y mecánicos que ocurren al nacer y que propician el cierre del conducto arterioso son: la disminución de la resistencia pulmonar (por la distención alveolar y el incremento del contenido de oxígeno en la sangre), la secreción de bradicinina por el endotelio pulmonar y disminución de la contracción sérica de las prostaglandinas placentarias. Junto con el flujo pulmonar, estos cambios incrementan el volumen y la presión en la aurícula izq., rebasando la de la aurícula derecha y cerrando el agujero oval. SALUD PUBLICA: Su incidencia es del 1%, lo que las coloca entre las malformaciones más frecuentes. En cerca de 90% de los casos no existe una causa identificable, pero existen ciertos factores como como la prematurez, cromosomopatías, infecciones congénitas, diabetes, alcohol, entre otros. CLASIFICACION: CARDIOPATIAS POR FISOPATOGENIA (por orden de incidencia) Obstructivas CORTOCIRCUITOS Coartación de la Derecha a izquierda (cianóticas) Izquierda a derecha (acianóticas) aorta (5-7) Sangre pobremente oxigenada pasa a la circulación sistémica, Propician aumento del volumen circulatorio Estenosis pulmonar ello condiciona a hipoxemia y cianosis en el sistema pulmonar, a expensas de una Estenosis aortica disminución del gasto cardiaco sistémico, no cianosis pero si hipertrofia del VD y vasos pulmonares Flujo pulmonar aumentado Flujo pulmonar disminuido Comunicación interventricular CIV (25-30) Comunicación interauricular CIA (6-8) Transposición de grandes vasos Tetralogía de Fallot (5-7) Conducto arterioso persistente CAP (6-8) (3-5) Atresia tricuspidea Defecto de cojinetes endocardicos Tronco arterioso (1-2) Atresia pulmonar Ventrículo único Hipoplasia de VI TETRALOGIA DE FALLOT: Cardiopatía congénita más común de las cianóticas (3-6 por 10,000). Implica obstrucción del tracto de salida del VD (estenosis pulmonar); comunicación interventricular (CIV); hipertrofia del VD y dextroposición de la aorta con cabalgamiento sobre la CIV, todo esto lleva a obstrucción del flujo del VD, hacia la arteria pulmonar y cortocircuito de derecha a izquierda. Leve no muestran cianosis hasta después del año, en grave cianosis poco después de nacimiento, de no corregirse el paciente presenta color azulado, acropaquias de pies y manos (dedos en palillo de tambor) y disnea, pueden sufrir ataques hipercianóticos paroxísticos con el esfuerzo que lo pueden llevar al sincope, si se ponen en cunclillas mejora su estado. La viabilidad del producto depende del conducto arterioso y del agujero oval. DIAGNOSTICO: Las Rx: corazón con aspecto de bota, trama vascular pulmonar disminuida, ECG: hipertrofia del VD y desviación del eje eléctrico, Ecocardiograma: “Estándar de oro”, para dx y gravedad. TRATAMIENT : Cirugía correctiva, inicialmente PG E1 para mejorar síntomas. TRANSPOSICION DE LOS GRANDES VASOS: Aquí la arteria pulmonar nace del VI y la aorta del VD. La cianosis y la taquipnea suelen ser evidentes en las primeras horas o días de vida y representan una urgencia médica. DIAGN STIC : La Rx: normal o datos de hiperflujo pulmonar. Ecocardigrafía es “estándar de oro” para dx. TRATAMIENT : Dar PG E1 para mantener abierto el conducto arterioso, la corrección quirúrgica final es la conmutación o switch arterial. TRONCO ARTERIOSO: Defecto en el desarrollo del tronco arterioso en aorta y arteria pulmonar, dejando un tronco arterial único que nace del corazón, para ambas circulaciones. COMUNICACIÓN INTERVENTRICULAR: Cardiopatía congénita más común en la infancia, durante los dos primeros años de vida casi la mitad de estos defectos se cierran espontáneamente y la mayoría lo hace para los 10 años. Se dividen en; membranoso (más común) y muscular. El cuadro clínico depende de la cantidad de sangre que fluye hacia la circulación pulmonar, en la mayoría es pequeño y permanecen asintomáticos. DIAGNOSTICO: Defectos grandes disnea, dificultad para la alimentación, infecciones pulmonares recurrentes e IC. Auscultación soplo holosistólico, a veces frémito. La Rx: cardiomegalia y aumento de trama vascular. La CIV muscular es menos común y su cierre espontaneo es más probable. TRATAMIENTO: Cirugía, solo si hay retraso en el crecimiento, grandes defectos o hipertensión pulmonar. COMUNICACIÓN INTERAURICULAR: Cardiopatia congénita más comun en el adulto (1 de cada 3 casos). En cualquier parte del septo, los principales tipos son: ostium secundum (90%), ostium primum y seno venoso, el grado de cortocircuito y mezcla de sangre depende del tamaño del defecto, generalmente no presentan síntomas hasta 4ta década de vida, la presentación más común es la falla cardiaca derecha. DIAGNOSTICO: Auscultación, S2 con desdoblamiento fijo, por hiperflujo pulmonar, un soplo de eyección de bajo grado. TRATAMIENTO: Corrección quirúrgica de defectos grandes, contraindicada cuando la relación de las resistencias arteriales pulmonar: sistémica es mayor a 0.7. Requiere profilaxis para endocarditis bacteriana. CONDUCTO ARTERIOSO PERSISTENTE: Su cierre normal ocurre entre 4-10 días de nacimiento, si no sucede la sangre aortica se desvía hacia la arteria pulmonar, el cierre después de la infancia es infrecuente, el principal factor de riesgo es la premadurez e infección materna por rubeola. DIAGNOSTICO: los conductos grandes pueden producir presión de pulso amplio, pulsos periféricos saltones, insuficiencia cardiaca y retraso en el crecimiento a largo plazo. Auscultación: soplo característico es sistólico; suele describirse como “ruido de maquinaria”. La Rx: incremento en la trama vascular y una arteria pulmonar ensanchada. Ecocardiograma estudio de elección. TRATAMIENTO: Las probabilidades de que cierre espontáneamente son escasas, para ayudar a que se cierre se da indometacina (inhibidor de la sintesisi de PG). En casos extremos es necesaria la cirugía, ya que disminuye el riesgo de endocarditis infecciosa.

CURSO ENARM CMN SIGLO XXI TEL: 36246001

Pharmed Solutions Institute

PÁGINA 373

MANUAL DE TRABAJO DEL CURSO ENARM CMN SIGLO XXI COARTACION DE LA AORTA: Causada por el estrechamiento de la aorta en algún punto desde el cayado hasta la bifurcación de las arterias iliacas. Este trastorno se relaciona con válvulas aorticas bicúspides (70%). Relación importante con el Sx. de Turner. Hay dos tipos: coartación juxtaductal antes conocida como del adulto y la infantil (coartación preductal) DIAGNOSTICO: Cuadro clínico depende de la localización; la presión arterial esta aumentada en los vasos proximales a la coartación y conlleva una disminución distal a la estenosis, signo clásico es la disparidad de la presión arterial entre los brazos y las piernas. La Rx: congestión pulmonar, cardiomegalia. Ecografía cardiaca “estándar de oro” para dx. El tratamiento de la forma infantil se administra PG E1, la cirugía correctiva es necesaria, en la juxtaductal se deben controlar la hipertensión y la falla cardiaca y después realizar la cirugía. PREVENCION: La profilaxis está indicada en todos los casos de cardiopatías cianóticas, durante los primeros 6 meses después de una reparación quirúrgica completa de un defecto y en todos los casos de reparaciones incompletas en las que exista material prostético cerca del defecto. Las opciones mas comunes son: 1. Amoxicilina 2 gr VO; 1 hr antes de procedimiento quirúrgico. 2. Ampicilina 2gr IV en caso de intolerancia a la VO. 3. Macrólidos o cefalosporinas de 3ra generación en alegría a las penicilinas. PRONOSTICO: Los avances en el tratamiento médico y quirúrgico de las cardiopatías congénitas han permitido que los pacientes que las padecen lleguen a la vida adulta CASO CLINICO Una recién nacida (RN) de 19 días de vida es admitida en la sala de emergencias en estado grave. En el examen físico, la paciente se presentaba hipotónica, con hipotermia, gimiente, cianótica con aleteo de la nariz, taquipneica (80 rpm), con tiraje y severa retracción subcostal y esternal, murmullo vesicular (MV) presente con crepitantes difusos, roncus, taquicárdica (190 lpm), pulso débil, ritmo cardíaco regular (RCR), ruidos cardíacos normofonéticos (RCNF), soplo sistólico +++/4+, abdomen distendido, hígado a 4 cm por debajo del reborde costal derecho. La paciente es remitida en la Unidad de Cuidados Intensivos Neonatales y se realizó intubación orotraqueal (IOT), con FiO2 al 100%, estableciéndose un acceso periférico e iniciándose corrección con solución de bicarbonato de sodio debido a la presencia de acidosis metabólica, disección venosa de la vena axilar derecha y la siguiente medicación: furosemida, dobutamina, milrinona, fentanilo y midazolam. Se realizaron exámenes (PCR negativa, cultivo de orina con presencia de Staphylococcus coagulasa resistente a ampicilina/penicilina), se realizó una radiografía de tórax que mostró la presencia de cardiomegalia. PREGUNTA Cual es su diagnostico mas probable? RESPUESTA a.- CIV. b.- CIA. c.- CAP d.- TGV

CURSO ENARM CMN SIGLO XXI TEL: 36246001

Pharmed Solutions Institute

PÁGINA 374

MANUAL DE TRABAJO DEL CURSO ENARM CMN SIGLO XXI KWASHIORKOR. CIENCIAS BASICAS: Forma de desnutrición que existe cuando no hay suficiente proteína en la dieta, es agudo. La palabra ' washior or', fue usada inicialmente en Ghana y traduce “enfermedad del niño destetado”, término utilizado por las madres para describir la enfermedad del primer hijo cuando nace el segundo. La desnutrición tipo kwashiorkor (edematosa) es más frecuente en lactantes con déficit de proteínas pero adecuada ingesta de carbohidratos y se caracteriza por depleción de proteínas séricas, principalmente la albumina, que tiende a ser más severa en esta forma de desnutrición. La gravedad o grado de desnutrición se determina siguiendo la clasificación de la OMS, basada en el grado de pérdida de peso: Desnutrición grado I, hay pérdida del 15-25% de peso. Grado II, hay pérdida de 25-40% de peso. Grado III, hay pérdida de más de 40% de peso. SALUD PUBLICA: Este tipo de DNT es más común en las zonas rurales de países en vías de desarrollo y afecta principalmente a los niños de 2 años de edad, asociado al periodo de destete e inicio de dieta rica en almidón y pobre en proteínas. Afecta aproximadamente una décima parte de los niños = 30 kg/m2 = obesidad adultos. Pero, al no tomar en cuenta la talla del sujeto, puede incluir como obesos a individuos normales de talla elevada o excluir a individuos de baja talla. El método más difundido ha sido la determinación del peso corporal y su relación porcentual con el peso ideal o esperada para la talla del sujeto. Así, valores entre 110 y 120 % se denominan sobrepesos y valores, superiores a 120 % obesos. Este criterio ha sido fuertemente criticado, ya que considera el peso para la talla independiente de la edad, lo que introduce un sesgo importante. Otro procedimiento de utilidad práctica es la medición de los espesores de los pliegues cutáneos en diversas zonas del cuerpo con un compás destinado al efecto; las más utilizadas son los pliegues tricipital, bicipital, subescapular y suprailíaca. Estos pliegues brindan información sobre la cantidad de grasa subcutánea, la que expresa indirectamente el estado de las reservas energéticas del organismo. Para medir la distribución de la grasa el indicador más práctico y útil es el índice cintura/cadera que es la resultante de dividir la circunferencia de la cintura por la cadera. SINDROME DE CARPENTER Obesidad, retraso mental, puente nasal plano, sindáctilia, craneosinostosis, hipogonadismo. SINDROME DE PRADER WILLI Hipotonía neonatal, baja estatura, hipogonadismo, hiperfagia compulsiva, retraso mental obesidad, cromosoma 15. TRATAMIENTO: El objetivo, es lograr un cambio en la composición corporal con reducción del peso en grasa y mantenerla después dentro del rango adecuado para su edad y sexo. El tratamiento no deberá interferir con el incremento de masa magra, es decir, con el crecimiento. La terapéutica está dirigida a modificar el desbalance energético para reducir el peso en grasa y a modificar el estilo de vida para garantizar que esta reducción sea duradera. Para ello, se sustenta en 3 pilares fundamentales: La dieta, el ejercicio, la inducción de cambios de conducta. Farmacológico: limitado

CURSO ENARM CMN SIGLO XXI TEL: 36246001

Pharmed Solutions Institute

PÁGINA 381

MANUAL DE TRABAJO DEL CURSO ENARM CMN SIGLO XXI en niños. Las dietas muy bajas en calorías solo deben hacer especial manejo en obesidad y nunca será la terapia indicada en niños. Reducir azúcares, agua. Frutas y cereales granos enteros Leche descremada, no comidas rápidas. Educar a las personas para lograr que ellas distribuyan el consumo de energía diario de forma tal que el 20 % se consuma en el desayuno, el 20 % entre comidas y el 30 % en almuerzo y comida, respectivamente. Solo al constituirse ese hábito con carácter familiar podrá ejercerse influencia sobre los miembros más jóvenes de la familia. Identificar los sujetos en riesgo y brindarles una atención especial, tomando en cuenta, entre otros, los factores siguientes: Obesidad en los padres, los hermanos y otros familiares cercanos. Obesidad en convivientes no familiares. Bajo nivel educacional. Sobreprotección: hijos únicos o valiosos, separación de los padres o muerte de uno de ellos. Tratamiento con antihistamínicos, anabólicos o esteroides. Alto consumo de azúcar y/o leche (más de un litro diario). Poca actividad física. Promover la lactancia materna extendiéndola por el mayor tiempo posible, y con carácter exclusivo hasta los 5 meses. No introducir ningún otro alimento en la dieta del lactante hasta esa fecha. Cuando se inicie la ablactación, no acostumbrar al bebé al consumo excesivo de alimentos dulces. Tratamiento farmacológico metformina: obesos adolescentes no diabéticos pero con comorbilidad, hiperinsulinemia o resistencia a la insulina. Efectos secundarios: náuseas, vómito Ovarios poliquísticos. Tratamiento quirúrgico: Cirugía gástrica y cirugía baipás: reduce la comorbilidad. Última opción la cirugía bariatrica. CASO CLINICO Niña de 10 años de edad, acompañada por su madre para una visita de niño sano. El peso del paciente, la altura y los signos vitales, fueron 42,1 kg, su altura es de 1,40 m, y su presión arterial es de 98/50 mmHg. IMC 21,6 kg/m2, que está en el percentil 91o para la edad y el género. PREGUNTA Cuál es el factores de riesgo más importante que buscaría para establecer un diagnostico adecuado. RESPUESTA a.- Familiares con sobrepeso y obesidad. b.- Interrogar sobre tiempo de ver televisión. c.- Actividad física. d.- Peso al nacimiento. PREGUNTA Cual es la comorbilidad más frecuente que el paciente desarrollará en caso de persistir el sobrepeso. RESPUESTA a.- Diabetes mellitus. b.- Esteatosis hepática. c.- Baja autoestima. d.- Desplazamiento epifisiario. PREGUNTA Cuál es el exámen no básico para pesquizaje intensional de complicaciones. RESPUESTA a.- Perfil lipidico. b.- Valores antropométricos. c.- Glucosa en ayunas e insulina. d.- USG por poliquistosis ovárica. PREGUNTA Considerando el cuadro clínico cual es la medida mas adecuada. RESPUESTA a.- Sibutramina. b.- Orlistat. c.- Metfomina d.- Dieta y estilo de vida. PREGUNTA Cual no es un criterio de referencia para segundo nivel inmediato para el tratamiento. RESPUESTA a.- Sobrepeso u obesidad con retrazo mental. b.- Lesion hipotalámica por cualquier causa. c.- Falla en el tratamiento instaurado. d.- Falla en el crecimiento y desarrollo.

CURSO ENARM CMN SIGLO XXI TEL: 36246001

Pharmed Solutions Institute

PÁGINA 382

MANUAL DE TRABAJO DEL CURSO ENARM CMN SIGLO XXI CASO CLINICO Un niño nacido de padres no relacionados, sanos después de un embarazo a término y el parto normal (peso al nacer 3200 g, longitud 50 cm, 34,5 cm OFC). Fue hipotónica en los primeros meses de vida, pero no se observaron problemas de alimentación en un principio. Excesivo aumento de peso con aumento del apetito y el comportamiento de búsqueda de alimento se observó a los 30 meses de edad. A los 5 años, su peso era de 26 kg (> 3 SD), longitud 112 cm (1 SD) y OFC 48 cm (-2DE) a los 9 años y su peso es de 72 kg (> + 6 SD), longitud 144 cm (+2 SD) y OFC 52,5 cm (-0,5 SD). Distintivos rasgos faciales incluyen braquicefalia, cara plana, la frente alta, hipertelorismo, nariz antevertidas, labio superior delgado, prognatismo, manos cortas, sindactilia de los dedos 2-3, y genitales anormales (criptorquidia e hipospadias). PREGUNTA Cual es la conducta a seguir. RESPUESTA a.- Tratamiento farmacológico. b.- Enviar a segundo nivel. c.- Enviar a tercer nivel. d.- Tratamiento quirúrgico.

CURSO ENARM CMN SIGLO XXI TEL: 36246001

Pharmed Solutions Institute

PÁGINA 383

MANUAL DE TRABAJO DEL CURSO ENARM CMN SIGLO XXI VACUNACION. CIENCIAS BASICAS: Las vacunas son sustancias que nos ayudan a prevenir muchas enfermedades infecciosas, las cuales son contagiosas y provocan discapacidad grave o incluso la muerte. Cartilla Nacional de Vacunación (0 a 19 Años): Es el documento Oficial para dar seguimiento al esquema de vacunación a los niños menores de 5 años, escolares y adolescentes hasta los 19 años de edad. Siempre que se acude a vacunación se debe de entregar para ver el esquema o anotar la vacuna que se está aplicando en ese momento. BCG: Vacuna contra la tuberculosis. Dosis única de 0.1 ml, RN, hasta los primeros 5 años, vía intradérmica, en la región deltoidea del brazo derecho, sin prueba tuberculínica previa, deja una cicatriz en el brazo posterior a su aplicación. Contraindicaciones: Fiebre > 38.5 °C. RN peso inferior 2 kg, dermatitis progresiva, pacientes con cuadro clínico de SIDA, Tx Transfusiones, o inmunoglobulina, esperarán cuando menos tres meses para ser vacunadas, no aplicar durante el embarazo, no aplicar BCG con profilácticas de medicamentos antituberculosos. HEPATITIS B: Se aplican 3 dosis, 10 mcg en 0.5 ml. IM profunda, en la cara anterolateral externa del muslo izquierdo en los menores de 18 meses de edad, a partir de los 18 meses de edad, en la región deltoidea del brazo derecho , la primera al nacer y a los 2 y 6 meses de edad. Previene este tipo de hepatitis, que afecta principalmente al hígado. Contraindicaciones: Fiebre > 38.5 °C. RN peso inferior 2 kg, enfermedad grave, Tx Transfusiones, o inmunoglobulina, esperarán cuando menos tres meses para ser vacunadas, hipersensibilidad a cualquier componente. PENTAVALENTE: Es la vacuna que previene la difteria, tos ferina y tétanos, además también a la poliomielitis y a las bacterias del Haemophilus Influenzae del tipo b, que provocan neumonías y meningitis. Se aplica en 4 dosis a los 2, 4, 6, y 18 meses de edad. Se aplica por vía IM profunda, las primeras tres dosis deben aplicarse en el tercio medio de la cara anterolateral externa del muslo derecho. A los 18 meses se debe aplicar en la región deltoidea del brazo izquierdo. ROTAVIRUS: Previene la gastroenteritis o la diarrea causada por el rotavirus en sus formas graves. Se aplican 3 dosis 0.1 ml sobre la parte interna de las mejillas, a los 2, 4 y 6 meses de edad y nunca después de los 8 meses de edad. Contraindicaciones: Hipersensibilidad después de la administración de la vacuna, antecedente de enfermedad gastrointestinal crónica, malformación congénita no corregida en el tracto gastrointestinal. SABIN: Ayuda a prevenir la poliomielitis. Se aplican 2 gotitas en los menores de 5 años a partir de los 6 meses como dosis adicional (de 0.1 ml) en cada Semana Nacional de Salud, siempre y cuando hayan recibido dos dosis previas de vacuna de poliovirus inactivada, uso de esta vacuna en actividades de control de casos y brotes. Contraindicaciones: Infección por VIH, tener inmunodeficiencia congénita, tener hemopatía o tumor sólido o terapia inmnunosupresora. NEUMOCÓCICA: Contra el neumococo. Se aplican 3 dosis, 0.5 ml, a los 2, 4 y 12 meses de edad. Vía IM en el tercio medio de la cara anterolateral externa del muslo derecho. Contraindicaciones: Procesos febriles de más de 38.5 °C, con enfermedad grave, reacción alérgica severa (anafiláctica) hacia un componente de la vacuna incluyendo toxoide diftérico, antecedentes de Síndrome de Guillain-Barré, antecedentes de hipersensibilidad al látex. No revacunar antes del tiempo establecido, ya que puede provocar reacciones locales severas. Aplicar un refuerzo antes del tiempo establecido puede provocar el desarrollo de neumonía neumocóccica posvacunal severa. ANTI-INFLUENZA: Previene el virus de la influenza y se aplica a niños de 6 a 35 meses de edad desde octubre a febrero, 0. 25 ml IM basto lateral izquierdo. En la primera ocasión se aplican dos dosis con intervalo de 1 mes y después cada año. Contraindicaciones: No aplicar a personas con alergia a las proteínas del huevo o a algún componente de la vacuna, fiebre, haberse aplicado el biológico en menos de un año, en lactantes menores de 6 meses, en personas que hayan tenido una reaccIón de hipersensibilidad, incluyendo una reacción anafiláctica al huevo o a una dosis previa de vacuna contra influenza, ante la presencia de fiebre mayor o igual a 38.5°C, pacientes con antecedente de síndrome de Guillain Barré. VACUNA TRIPLE VIRAL (SRP): Previene el sarampión, la rubeola y las paperas. Se aplica 2 dosis de 0.5 ml de vacuna reconstituida: 12 meses de edad y los 6 años de edad vía SC en el área superior externa del tríceps del brazo Izquierdo, inscritos o no en primer año de primaria. Contraindicaciones: Fiebre > 38.5, alérgicos al huevo, neomicina, antecedente de aplicación de inmunoglobulina o transfusiones 3 meses previos a la vacunación, neoplasias, inmunodeficiencias, tratamiento con corticoesteroides, citotóxicos, embarazo. Poco frecuentes, pueden presentarse: Parotiditis uni o bilateral, después de doce días y dura menos de cuatro días, meningitis aséptica, entre la segunda y la cuarta semanas posteriores a la vacunación, púrpura trombocitopénica. DPT: Sirve como un refuerzo que previne a la difteria, tos ferina y tétanos. Se aplica dosis de 0.5ml a los 4 años, se aplica por vía IM profunda en la región deltoidea del brazo izquierdo. Contraindicaciones: No mayores a 5 años, reacción anafiláctica a una aplicación previa, antecedentes de crisis convulsivas o alteraciones neurológicas, fiebre > 38. 5°C. Toxoide Tetánico Diftérico (TD): Se aplica a las personas desde los doce años hasta los adultos mayores, hombres y mujeres, especialmente a las embarazadas, dosis de 0.5ml. Previenen el tétanos en los recién nacidos y en los adultos. Las personas no vacunadas o con esquema incompleto de vacuna pentavalente o DPT recibirán al menos dos dosis con intervalo de 4 a 8 semanas entre cada una, y revacunación cada 10 años. Contraindicaciones: No suministrar a personas con hipersensibilidad a algún componente de la vacuna, a personas con

CURSO ENARM CMN SIGLO XXI TEL: 36246001

Pharmed Solutions Institute

PÁGINA 384

MANUAL DE TRABAJO DEL CURSO ENARM CMN SIGLO XXI inmunodeficiencias, a excepción de la infección por el VIH asintomática, fiebre superiores a 38.5°C y enfermedades graves, historia de reacción grave de hipersensibilidad o eventos neurológicos relacionados con la aplicación de una dosis previa, las personas transfundidas o que han recibido inmunoglobulina, deberán esperar tres meses para ser vacunadas, salvo en aquellos casos de traumatismos con heridas expuestas, ya que puede aplicarse, simultáneamente con antitoxina, independientemente de transfusión o aplicación de inmunoglobulina. EVENTOS TEMPORALMENTE ASOCIADOS A LA VACUNACIÓN ETAVS: Se define como eventos temporalmente asociados a la vacunación a todas aquellas manifestaciones clínicas que se presentan dentro de los 30 días posteriores a la administración de una o más vacunas y que no pueden ser atribuidos inicialmente a alguna entidad nosológica especifica (para la vacuna Sabin el periodo puede ser hasta de 75 días y para la vacuna BCG hasta seis meses ó un año). Falsa contraindicación para todas las vacunas: Reacción local de leve a moderada, enfermedad leve con o sin fiebre, tratamiento antimicrobiano actual, fase de convalecencia de alguna enfermedad, exposición reciente a enfermedades infecciosas, historia de alergia a penicilina u otras alergias inespecíficas o el hecho de que sus familiares tengan dichas alergias, alergia a proteínas del huevo, a neomicina manifestada sólo por rash. CASO CLINICO Un niño de 24 meses de edad, nació a término, cuenta con el antecedente que después de haber recibido la vacuna antipoliomielítica, fiebre (38,5 º C y 40,0 º C), al día siguiente presento vómitos y convulsiones tónico-clónicas. El líquido cefalorraquídeo mostró pleocitosis y aumento moderado de proteínas. El LCR fue positivo por PCR para enterovirus y una pleocitosis. Las muestras de heces tomadas los días 5 y 9 fueron positivos para enterovirus, que se caracterizó posteriormente como poliovirus tipo 3. PREGUNTA Cual es la conducta a seguir. RESPUESTA a.- Aplicar SRP b.- Vacuna pentavalente acelular. c.- Evitar DPT a los 4 años. d.- Revacunación anual. DERMATITIS HERPETIFORME (DH). CIENCIAS BÁSICAS: Es una enfermedad pápulovesicular, muy pruriginosa, crónica, de distribución característica. PATOGENIA: Se ha especulado sobre el papel de los anticuerpos del gluten en la etiopatogénesis de esta enfermedad. El gluten es una fracción proteica remanente de la harina después que ha sido lavada y eliminado el almidón. Las principales fuentes de gluten en occidente son: trigo, centeno, cebada; el maíz y la avena no parecen contener cantidades importantes. El 75% de los casos de DH tienen aplanamiento de las vellosidades intestinales y 20% presenta infiltrado linfocitario en el epitelio como evidencia de enteropatía sensible al gluten (ESG). Es bien establecido, que las manifestaciones cutáneas e intestinales mejoran con la dieta estricta, libre de gluten, sin embargo, el mecanismo fisiopatológico se desconoce. La asociación inmunogenética apoya la naturaleza autoinmune de la DH a otras enfermedades autoinmunes. Aunque no se ha descrito todavía el antígeno diana, hay asociación de la DH con otras enfermedades autoinmunes, como lupus eritematoso, enfermedades tiroideas y diábetes mellitus. DIAGNOSTICO: La DH se ha reportado raramente en niños, pero la edad de presentación es de 2 a 7 años. Las lesiones cutáneas características son pápulas y placas eritematosas, urticarianas, edematosas sobre las cuales hay vesículas, distribuidas simétricamente en superficie extensora de las extremidades. Los sitios característicos son: codos, rodillas, glúteos, nuca y cuero cabelludo. Una sensación de ardor o escozor suele preceder las lesiones. El prurito es intenso haciendo que el paciente esté inquieto. Hay lesiones secundarias al rascado como excoriaciones, costras y máculas hiper e hipopigmentadas residuales. Las membranas mucosas no están comprometidas. Sin embargo, se han descrito defectos en el esmalte dental similar a lo observado en la enfermedad celíaca, cambio de coloración y textura de la superficie dentaria. Los síntomas gastrointestinales pueden ser muy leves o estar ausentes, aunque por lo menos un tercio de los pacientes sufre de esteatorrea y anemia. La patología es característica. Se describen microabscesos en la punta de las papilas dérmicas y formación de ampolla subepidérmica. En la dermis superior y media se observa un infiltrado de linfocitos, neutrófilos e histiocitos alrededor de los vasos sanguíneos. Los eosinófilos son escasos. El diagnóstico se basa en criterios clínicos, histológicos, así como la respuesta favorable a las sulfonas y a la dieta libre de gluten. TRATAMIENTO: El tratamiento de elección, como en los adultos, es la dieta libre de gluten. Se describe una respuesta mucho más rápida en este grupo con control de las lesiones cutáneas a los 11 meses, aproximadamente. La dapsona a la dosis de 1.5-2 mg/kg/día, es efectiva para el control de las lesiones cutáneas y del prurito intenso. La suspensión de ésta, produce la reaparición rápida de las lesiones, por lo que el tratamiento debe ser combinado con la dieta libre de gluten y disminución progresiva y suspensión de la dapsona al lograr control, con lo que disminuye la aparición de los efectos secundarios de la droga: metahemoglobinemia, anemia hemolítica, agranulocitosis, y complicaciones neurológicas. Antes de su uso, los niveles de glucosa-6-fosfato deshidrogenasa deben ser determinados. En esos casos, en los que no se pueda utilizar la dapsona, se recomienda la sulfapiridina a 250 mg/día con ajuste de acuerdo a respuesta terapéutica. El curso de la DH en niños no está bien definido, la ESG persiste toda la vida, aunque con períodos de remisiones espontáneas en algunos casos. Sin embargo, hay que informar a la familia sobre la naturaleza de la enfermedad y su duración, así como, la importancia de la dieta estricta libre de gluten de por vida. CASO CLINICO Paciente varón de 13 años de edad que consulta por lesiones muy pruriginosas, que se localizan fundamentalmente en codos, rodillas y región lumbosacra. Se ha tratado con diversas pomadas de corticoides tópicos sin mejoría. En la exploración cutánea se observa en los codos, rodillas y región lumbosacra la presencia de pápulas eritematosas, excoriadas y alguna pequeña vesícula intacta. Se realizó biopsia cutánea de una de las lesiones y el estudio histopatológico demostró: microabscesos de neutrófilos a nivel de papilas dérmicas, El estudio de inmunofluorescencia directa demostró depósito de Ig A granular en las papilas dermicas.

CURSO ENARM CMN SIGLO XXI TEL: 36246001

Pharmed Solutions Institute

PÁGINA 385

MANUAL DE TRABAJO DEL CURSO ENARM CMN SIGLO XXI PREGUNTA Considerando la patología que presenta el caso cual de los siguientes diagnosticos diferenciales es el menos probable? RESPUESTA a.- Dermatitis alérgica de contacto b.- Dermatitis atópica c.- Sarna d.- Penfigoide ampolloso PENFIGO. CIENCIAS BÁSICAS: Es una enfermedad ampollar autoinmune, del epitelio estratificado, que normalmente afecta a individuos de edad media y mayores, pero puede ocurrir raras veces en niños, relacionado con factores genéticos y ambientales secundario a la producción de IgG. Las dos variantes del pénfigo son el vulgar (PV) y foliáceo (PF), que difieren en su presentación clínica, histológica y en los antígenos patogénicos. El pénfigo vegetante y eritematoso son variantes clínicas más raras. En Brazil existe la forma endémica del pénfigo foliáceo, denominada fogo selvagem. Más recientemente se describió el pénfigo paraneoplásico y el pénfigo IgA. SALUD PUBLICA: Es una enfermedad que afecta todas las razas sin distinción de sexo. Sin embargo, se reporta una mayor incidencia en judíos ashkenazi y menos en la raza negra. La edad de presentación en niños va desde 3 hasta 17 años y adultos pico de 50-60 años. En 1955 se publicó el primer caso de pénfigo vulgar en un niño. Mortalidad 17.7% (choque séptico, embolia pulmonar). CLASIFICACION: Pénfigo vulgar (80%), pénfigo violáceo, pénfigo inducido por fármacos, pénfigo paraneoplasico, pénfigo IgA. PATOGENIA: El pénfigo es una enfermedad autoinmune que afecta la piel y las mucosas, de etiología desconocida. Existe un aumento en la expresión de antígenos del complejo mayor de histocompatibilidad HLA-A, DR4, Dw10 y Dw6. Los pacientes tienen autoanticuerpos que reaccionan con los desmosomas, moléculas de adhesión localizadas en la superficie de los queratinocitos. Los autoanticuerpos detectados son de la clase IgG. En los últimos años se ha logrado un avance en la determinación de los antígenos responsables en todas las formas de pénfigo. De la unión de este autoanticuerpo con las desmogleína antigénica se produce la acantolisis, con pérdida de la cohesión de las células y la formación de las ampollas. DIAGNOSTICO: Clínicamente, suele comenzar en la mucosa bucal con erosiones del epitelio. Las lesiones cutáneas consisten en vesículas, ampollas fláccidas que se rompen con facilidad resultando en extensas erosiones. Se localiza predominantemente en cara, cuello, tronco, extremidades y áreas intertriginosas. Las erosiones causan dolor y ardor y secundariamente pueden infectarse con bacterias u hongos. El signo de Nikolsky es positivo (eritema y formación de ampollas como resultado de fricción o presión sobre la piel), signo de Asboe-Hansen (aumento periférico del tamaño de la ampolla al presionar verticalmente su superficie), no es especifica. Se ha reportado onicolisis y distrofia ungueal. En el PF infantil es endémico en Brasil, en la forma de fogo selvagem, pero su incidencia en otros países es menor a la del PV. Clínicamente se presentan erosiones superficiales con descamación y costras, aisladas o generalizadas (eritrodermia). En ocasiones se inicia en cuero cabelludo o áreas seborreicas, luego se extienden al tronco y extremidades. No se aprecian lesiones orales. Se deben escoger las vesículas intactas para el examen histológico. En el PV se evidencia la ampolla intraepidérmica suprabasal, con acantolisis (desaparición de puentes intracelulares) marcada. El piso de la cavidad está constituido por queratinocitos basales y leve infiltrado inflamatorio, compuesto por eosinófilos. En lesiones tempranas los eosinófilos invaden la epidermis en racimo. Este fenómeno se denomina espongiosis eosinofílica. La ampolla en el PF se localiza en la subcorneal de la epidermis. La acantolisis está presente inmediatamente por encima o por debajo de las células de la capa granulosa. Este patrón se ha denominado “panal de abejas” y constituye la herramienta diagnóstica en estas enfermedades, cuya positividad alcanza hasta el 100%. El diagnóstico se basa en la combinación de criterios clínicos, histológicos. Si hay duda observar la celularidad, células Tzank, tiene 100% sensibilidad y 43% especificidad. Se observan células acantoliticas (zona blanca). TRATAMIENTO: Las lesiones se alivian en 1-2 años y dejan hiperpigmentacion pósinflamatoria. En manejo de los pacientes pediátricos en PV es similar al adulto. Los casos con enfermedad localizada pueden ser tratados con corticoesteroides tópicos. Sin embargo, la mayoría de los casos desarrollan formas más severas que ameritan el uso de corticoesteroides sistémicos a altas dosis (2 a 3 mg/kg/día). Con la finalidad de disminuir los efectos adversos de éstas, se usan drogas adyuvantes como: azathioprina, ciclofosfamida, dapsona y metotrexato, sales de oro. CASO CLINICO Un niño de 12 años de edad se presentó con lesiones generalizadas en la piel eritematosas y costras de 4 meses de duración. Las lesiones aparecieron por primera vez en su cuero cabelludo y dentro de unos días, eritrodérmica se generalizó y, a continuación. Escalado y exudación también fueron vistos. Diferentes antibióticos y esteroides tópicos fueron prescritos sin mejora significativa. El niño también se quejo de pérdida de la audición desde un mes. En el examen físico, eritrodermia con escalamiento grave y maloliente con descarga y leve queratodermia palmoplantar todo el cuero cabelludo. Había dos pequeñas vesículas a lo largo del lado cubital de la palma derecha. Las superficies de las mucosas y las uñas eran normales. Tenía ganglios linfáticos submandibulares. El conducto auditivo estaba lleno de escamas y costras. pabellón de la oreja era sensible a la palpación. Después de quitar las costras, canal auditivo externo se encontró que era roja e hinchada PREGUNTA Cual es la conducta a seguir. RESPUESTA a.- Prednisona oral de 0.5 a 1 mg/kg al día. b.- Dapsona dosis de 100 mg/día. c.- Azatioprina es de 2.0 a 2.5 mg/kg, d.- Ciclofosfamida a dosis oral de 1 a 3 mg/kg al día

CURSO ENARM CMN SIGLO XXI TEL: 36246001

Pharmed Solutions Institute

PÁGINA 386

MANUAL DE TRABAJO DEL CURSO ENARM CMN SIGLO XXI HIPERQUERATOSIS EPIDERMOLITICA. CIENCIAS BASICAS: También llamada Eritrodermia ictiosiforme congénita ampollosa, es una genodermatosis infrecuente, con patrón de herencia autosómico dominante, aunque 50% de los pacientes experimentan mutaciones de novo. Se distingue por eritrodermia y ampollas secundarias a traumatismos mínimos, que se manifiestan desde el nacimiento y que evolucionan a ictiosis generalizada a lo largo de la vida, predominantemente en zonas de flexion. SALUD PUBLICA: No hay predilección por sexo o raza. Se manifiesta desde el nacimiento en un caso por cada 200,000 a 300,000 nacidos vivos. PATOGENIA: La causa son los defectos en las citoqueratinas que forman los filamentos intermedios. Las citoqueratinas afectadas son la K1 y la K10, las cuales predominan en los estratos suprabasales de la epidermis. Cuando las citoqueratinas experimentan mutaciones puntuales, se asocian con la hiperqueratosis epidermolítica y la eritrodermia ictiosiforme congénita ampollosa, y generan células suprabasales que se fragmentan fácilmente, con epidermis hiperproliferativa e hiperqueratósica. Forman parte del túbulo de las glándulas sudoríparas ecrinas y son marcadores de queratinización por excelencia. La hiperqueratosis y la ictiosis subsecuentes que se observan en estos pacientes se deben a un mecanismo de compensación para proteger contra la fragilidad cutánea, y así disminuir la probabilidad de formación de ampollas. CLASIFICACION: En 1994 DiGiovanna distinguió clínicamente los dos tipos de hiperqueratosis epidermolítica con base en la presencia o ausencia de queratodermia palmoplantar. DIAGNOSTICO: Se basa en manifestaciones clínicas y confirmación histológica. Aparece desde el nacimiento e inicia como eritrodermia generalizada, con fragilidad cutánea que causa descamación, así como ampollas, incluso con traumatismos menores, en la mayoría de los casos. Las ampollas son superficiales y grandes, y al romperse ocasionan grandes zonas de exulceración. En los pliegues de flexión aparecen ulceraciones superficiales y zonas extensas de piel denudada. Debido a la interrupción de la barrera epidérmica, en estos niños existe un mayor riesgo de infecciones severas, desequilibrio hidroelectrolítico y sepsis. Hay disminución gradual de la eritrodermia y de las ampollas, que a los pocos meses evolucionan a placas hiperqueratósicas, de aspecto verrugoso, en las zonas de flexión, la piel cabelluda y los glúteos. La hiperqueratosis inicia a los tres meses de edad, con engrosamiento poco evidente de la piel o con escamas finas y pequeñas. Con la edad algunas de las escamas se van haciendo más gruesas (adquieren aspecto de “cartón corrugado”) y la probabilidad de manifestar ampollas disminuye, aunque incluso 20% de los pacientes sigue teniendo ampollas hasta la vida adulta. La hiperqueratosis se agrava con el sol y el calor. La afección de las palmas y las plantas ocurre en 60% de los pacientes y se manifiesta con grados variables de hiperqueratosis, asociada, sobre todo, con mutaciones de CK1. La queratodermia palmoplantar puede originar fisuras y contracturas recurrentes y dolorosas, que pueden ocasionar incapacidad funcional. La biopsia de piel, confirma el diagnostico si observamos epidermis con hiperqueratosis ortoqueratósica, acantosis irregular moderada con áreas de degeneración de queratinocitos, espinosos, dermis con papilomatosis y focos mínimos de linfocitos perivasculares. La inmunohistoquímica muestra anormalidades de expresión de la citoqueratina 1 o 10. TRATAMIENTO: La piedra angular del tratamiento son los emolientes tópicos. Existen reportes de mejoría clínica con dosis altas de betacarotenos, retinoides sistémicos o tópicos, glicerina a 10%, ácido láctico, alfahidroxiácidos, calcipotriol y urea; sin embargo, no existe un consenso sobre la administración de estos fármacos. Los agentes queratolíticos son poco tolerados por estos pacientes debido a que las escamas se desprenden en bloques extensos. El sobrecrecimiento bacteriano es común, sobre todo por Staphylococcus aureus, secundario a la acumulación y maceración de la escama, por lo que pueden usarse jabones antibacterianos suaves o limpiadores con clorhexidina. PRONOSTICO: Es bueno para la vida, sin embargo los pacientes pueden padecer alteraciones funcionales, secundarias a queratodermia palmoplantar y contracturas en flexión de las manos. CASO CLINICO Lactante mujer, de un año de edad, sin antecedentes patológicos de interés, que consulta por aparición progresiva de lesiones cutáneas a nivel palmoplantar. El cuadro se inició a la semana del nacimiento con discreto eritema en dicha localización que evolucionó progresivamente hacia placas hiperqueratósicas amarillentas, simétricas y confluentes. En el momento de la consulta presentaba una hiperqueratosis difusa palmoplantar bien delimitada por un borde eritematoso en los márgenes laterales de las manos y los pies, sin extensión a la superficie dorsal de estos. PREGUNTA Cual es la conducta a seguir. RESPUESTA a.- Envio a segundo nivel. b.- Tratamiento farmacológico. c.- Tratamiento quirúrgico. d.- Terapia fotodinámica. PREGUNTA Cual de los antecedentes personales patológicos incrementa más esta patologia. RESPUESTA a.- Xeroderma pigmentoso. b.- Albinismo. c.- Inmunosupresion. d.- Sindrome de Bloom.

CURSO ENARM CMN SIGLO XXI TEL: 36246001

Pharmed Solutions Institute

PÁGINA 387

MANUAL DE TRABAJO DEL CURSO ENARM CMN SIGLO XXI FOLICULITIS. CIENCIAS BASICAS: Foliculitis Bacterianas: Se definen como el conjunto de infecciones originadas por bacterias que afectan al folículo piloso con una expresión clínica que varía de acuerdo a la profundidad y extensión. Foliculitis: Se definen infecciones superficiales localizadas a nivel del ostium folicular, acompañada en ocasiones de una reacción inflamatoria perifolicular. PATOGENIA: El agente causal más frecuente es Staph. aureus, pero también pueden infectar bacterias Gram negativas ó S. Aureus coagulasa negativo. Los folículos pilosos están en toda la superficie de la piel excepto en palmas y plantas, pero son abundantes sobre todo en piel cabelluda, zonas de varaba y bogote, axilas, pubis. Son estos los sitios más afectados por la foliculitis. DIAGNÓSTICO: Es clínico y de ser necesario se realizará gram y cultivo. Se presentan en cualquier zona pilosa, sobre todo en ambiente húmedo, maceración, falta de higiene, aplicación del emoliente oclusiva y drenaje de heridas adyacentes y abscesos, uso de agentes oclusivos o presencia de infecciones adyacentes, y más frecuentemente en pacientes con inmunosupresión local ó sistémica. Se presentan clínicamente como una pústula de base eritematosa, apenas de unos milímetros, a nivel de cada folículo, en ocasiones se dejan atravesar un pelo, son de color amarillento se abren rápidamente y dejan salir una gotita de pus que al secarse formara una costra melicerica. En general no es un proceso pruriginoso, ni doloroso.Las lesiones se extioenden rápidamente en superficie y mientras unas van curando, otras nuevas aparecen. Al desaparecer no dejan cicatriz. Se localizan con mayor frecuencia en zonas de pelo terminal; si la infección se extiende en profundidad formará Abscesos. A las formas más superficiales también se les denomina Impétigo de Bockhart. TRATAMIENTO: En primer lugar eliminar factores desencadenates, antisépticos n(fomentos de sulfato de cobre y aplicación de toques con solución yodada al 1%), antibióticos tópicos y sistémico (para evitar recidiva dicloxacilina, TMP/SFX, por 3-4 semanas) sabiendo que el agente causal es Staph. Aureus. CASO CLINICO Niña de 8 años de edad. Consultó por aparición en los 3 días previos de un brote de lesiones papuloeritematosas, algunas de aspecto seudovesiculoso o pustuloso, pruriginosas, en el tronco (de predominio en caderas y glúteos) y las extremidades inferiores. Con la sospecha inicial de exantema periflexural asimétrico de la infancia frente a picaduras, se inició tratamiento con antihistamínico por vía oral y metilprednisolona por vía tópica, con mejoría aparente de las lesiones. Seis días más tarde consultó por aparición progresiva de nuevas lesiones de similares características. En la exploración física presentó máculas hiperpigmentadas marrones en la zona abdominal inferior y en la región anterior de ambos muslos, junto con múltiples pápulas eritematoedematosas, algunas con centro pustuloso, en la zona lumbar y en ambos glúteos y otras con aspecto vesiculoso en costado izquierdo. La niña negaba ir a piscinas, o hacer cualquier tipo de actividad recreativa relacionada con agua. PREGUNTA Cual es el agente causal mas frecuente. RESPUESTA a.- Staphylococcus aureus. b.- Pseudomona aeruginosa. c.- Estreptococos d.- Proteus. IMPETIGO. CIENCIAS BASICAS: Es la infección bacteriana superficial y contagiosa más frecuente en los niños, producida por Estafilococo Aureus, Estreptococo B hemolítico del grupo A (pyogens) o ambos; puede ser primario en caso de invadir piel sana ó secundario cuando existe daño de la barrera cutánea previamente, como ocurre en la dermatitis atópica, escabiosis. El impétigo contagioso es la más común. Estas infecciones pueden dividirse en: 1. infección cutánea primaria 2. Infección secundaria de una enfermedad cutánea primaria y 3. lesiones cutáneas como manifestación de infección primaria en otro órgano, ejemplo de este último grupo tenemos: S.aureus causando síndrome de shock toxico y síndrome de piel escaldada estafilocócico, S. pyogenes escarlatina, N. meningitides, sepsis por meningococo (petequias), S. thiphy, fiebre tifoidea (roséola tifoidea), Rickettsia conorii, Tifus africano (rahs macular) IMPÉTIGO CONTAGIOSO: Costroso ó no ampollar, infección muy frecuente representa el 70% de los casos; predomina en niños de todas la edades , aunque también se puede ver en adultos; el contagio se origina sobre todo por rascado. La infección puede ser originada por Strep. Pyogenes (15%), Staph. Aureus (70 a 80 %) o ambos (15 a 100%). La cara es la zona de más frecuente de localización, sobre todo en las uniones mucocutáneas, por ejemplo: introito nasal, boca y en todas las zonas periorificiales, manos, etc, pero como es contagioso, se puede ver en cualquier zona corporal. La lesión inicial son pápulas eritematosas pequeñas que rápidamente evolucionan a una vesícula que se rompen rápidamente dando paso a erosiones cubiertas por costras melicéricas y lesión residual hipocrómica temporal (lesiones primitivas, ampollas, pústulas que al romperse dejan salir exudado que se seca, costra melicerica), sin dejar cicatriz; si hay dolor es leve y puede presentar linfadenopatía regional, pero no hay síntomas sistémicos. Las lesiones suelen propagarse por autoinoculación originando lesiones satélites. La resolución es espontánea entre la segunda y tercera semana de evolución, pero debe ser tratada con prontitud y así se acorta el período de enfermedad y el contagio, así como las eventuales complicaciones glomerulonefritis post estreptocócica, escarlatina ò psoriasis. Diagnóstico: se basa en la clínica pero se puede confirmar con un gram y cultivo. IMPÉTIGO AMPOLLAR: Representa el 30% restante, es observa más frecuentemente en neonatos y lactantes. Es ocasionado por la toxina exfoliativa tipo A del Staph. Aureus; esta toxina actúa como una serina proteasa de desmogleína 1, ocasionando de esta manera la acantólisis subcorneal que conduce a la formación de ampollas. Las lesiones se pueden localizar en cualquier área de la superficie corporal, pero lo más frecuente es alrededor de los orificios de la boca, nariz, área genital y en las extremidades Clínica:pueden presentar fiebre hasta de 40°C, ampollas flácida, que se rompen con facilidad en el curso de 1 a 2 días, dejando erosiones cubiertas por una costra periférica fina, de ahí que también se conozca como impétigo circinado; las lesiones aparecen usualmente agrupadas. Existe una tendencia a la curación espontánea sin cicatriz, pero pueden dejar hipopigmentación residual temporal. TRATAMIENTO: Es igual en ambos tipos clínicos: 1) Realizar Antisepsia, por ej con Clohexidine, sulfato de cobre l 1x1000 con eliminación de las costras., Mejorar la higiene persona y recortar las uñas. 2) Terapia Tópica, con antibacterianos preferiblemente de origen sintético, con espectro sobre los agentes causales más frecuentes: por ej mupirocina, ácido fusídico o más recientemente retapamulina. 3) Si hay más de 2 áreas corporales afectadas , presencia de muchas lesiones, enfermedad sitémica

CURSO ENARM CMN SIGLO XXI TEL: 36246001

Pharmed Solutions Institute

PÁGINA 388

MANUAL DE TRABAJO DEL CURSO ENARM CMN SIGLO XXI subyacente, como la Dermatitis Atópica, Inmunosupresión ó varios familiares afectados , se debe agregar terapia sistémica, por ej: amoxilinia con Ac Clavulanico , dicloxacilina, cefalexina, cefradina , macrólidos u otros si hay resistencia bacteriana. Si no hay una buena respuesta clínica en forma rápida se debería cultivar y realizar antibiograma CASO CLINICO Se trata de escolar masculino de seis años de edad quien es traído por su madre por presentar lesiones ampollosas en las manos, pecho, espalda y brazos de color amarillento con ardor e hipertermia de 39ºc;. PREGUNTA Cual es la conducta a seguir inmediata mas adecuada? RESPUESTA a.- Iniciar antibiótico. b.- Administrar antipirético. c.- Hidratación adecuada. d.- Laboratorio de rutina. PREGUNTA Considerando la exploración del caso cual es la conducta terapéutica para la etiológica mas adecuada? RESPUESTA a.- Mupirocina. b.- Acido fusidico. c.- Penicilina. d.- Amoxicilina. ERISIPELA. CIENCIAS BASICAS: Es una Infección bacteriana profunda dermoepidérmica pero más superficial que la Celulitis, localizada en dermis profunda y zona superior del tejido celular subcutáneo con afectación linfática significativa. Casi siempre muy bien delimitada y con tendencia a progresar hacia la periferia. Es una enfermedad potencialmente transmisible y reviste mayor gravedad en los niños. PATOGENIA: Los agentes causales más comunes son: Streptococccus pyogenes o Streptococcus beta-hemolítico del grupo A, con menor frecuencia por Streptococcus de los grupos B, C, D o G, sobre todo en pacientes postquirúrgicos ó con otras enfermedades. Los del grupo B suelen ser responsables de la erisipela del recién nacido. Otros gérmenes que pueden ocasionar erisipela: son Staphylococcus aureus, Pneumococcus, Klebsiella pneumoniae, Yersinia enterocolítica y Haemophilus influenzae tipo b. La lesión comienza a través de una pequeña solución de continuidad de la piel o de las mucosas, que constituye la puerta de entrada y que en muchas ocasiones pasa inadvertida, por ejemplo en pacientes con ulceras, prurigo, intertrigo, tiña pedis, etc ó paciente diabéticos o imunosuprimidos. De forma más excepcional se debe a una diseminación hematógena, procedente de focos a distancia, por ejemplo en dientes o faringe. El periodo de incubación es de 2 a 5 días, la localización más frecuente son los miembros inferiores, puede localizarse en cara y área periumbilical en caso de los lactantes. DIAGNOSTICO: Precede a la lesiones cutáneas síntomas como fiebre, escalofrío , náuseas , cefaleas y vómitos, malestar general y dolor local ; las lesiones tienen comienzo abrupto y se presentan como una placa ardorosa, eritematosa de color rojo intenso, indurada, edematosa, caliente, dolorosa a la palpación y bien delimitada , su crecimiento es centrífugo con un borde activo y bien definido, con la zona central de la placa aclarándose ; en algunas formas menos usuales se pueden presentar vesículas, ampollas, pústulas y lesiones necróticas. Su topografía más común es en las extremidades inferiores y en la cara a nivel de las mejillas. Se acompaña con frecuencia de adenopatías satélites regionales. Habitualmente las lesiones inician su involución en 10-12 días descamándose la región afectada sin dejar cicatriz. Habitualmente el diagnóstico es clínico pero si es posible obtener material de la puerta de entrada se debe realizar Gram y cultivo. TRATAMIENTO: Antisepsia. Antibiótico Tópico. Antibiótico Parenteral se puede iniciar con Penicilina (800,000 UI de penicilina procainica c/24 hrs por 8-10 días, continuar con benzatinica 1200,000 UI cada 8 días por 3-4 semanas), Cefalosporinas, Amoxicilina + Ac. Clavulánico, y en los países con cepas Meticilinoresistentes se pueden utilizar de acuerdo a estudios epidemiológicos previos: Trimetoprim-sulfametoxazole, Clindamicina, Rifampicina-, Linezolid y Vancomicina. Si hay alergia a penicilinas se puede usar eritromicina. Para evitar recidivas usar sultrim dos tabletas diarias por 8-10 dias. COMPLICACIONES: incluyen bacteremia, abscesos, gangrena, linfangitis, celulitis, septicemia, tromboflebitis y glomerulonefritis. Cuando se trata de brotes repetidos de erisipela la afección de los vasos linfáticos que al fibrosarse originan estados elefantiásicos, conocidos como elefantiasis nostra. CASO CLINICO Se trata de femenino de 2 años de edad la cual cuenta con el antecedente de haber jugado en la tierra y producirse escoriasiones en miembros pélvicos, a las 48 hrs posteriores inicia con fiebre no cuantificada, enrojecimiento y edema en la zona, la cual se encuentra caliente y eritematosa a la exploración. PREGUNTA Cual es la conducta a seguir. RESPUESTA a.- Limpieza y tratamiento ambulatorio con penicilina. b.- Debridación y tratamiento hospitalario. c.- Limpieza y alta con signos de alarma. d.- Limpieza local, vendaje y penicilina.

CURSO ENARM CMN SIGLO XXI TEL: 36246001

Pharmed Solutions Institute

PÁGINA 389

MANUAL DE TRABAJO DEL CURSO ENARM CMN SIGLO XXI PIODERMIA GANGRENOSA. CIENCIAS BASICAS: Fagadenismo geométrico de Brocq, a pesar del nombre este cuadro de causa desconocida, dista mucho de ser una enfermedad bacteriana, probablemente sea más una inmunopatia que una verdadera infección aun cuando no pueda descartarse del todo la intervención de bacterias. PATOGENIA: Su etiología es desconocida y se invocan alteraciones inmunológicas aun no bien determinadas, se piensa en una reactividad tisular local, necrótica a antígenos microbiana. DIAGNOSTICO: En cualquier área del cuerpo sobre todo en las extremidades, aparece repentinamente en personas en aparente estado de salud, un área eritematosa que pronto se cubre de pústulas o ampollas y se ulcera rápidamente. Esta ulceración se extiende con contornos geográficos, sus bordes son levantados y pueden aparecer abscesos en la periferia habitualmente estériles, el fondo es sucio, sangrante o necrótico. En poco tiempo estas lesiones que profundizan por lo general poco, han abarcado extensas zonas de la piel. La sintomatología no corresponde a la intensidad del proceso, pero puede haber dolor y ataque al estado general del paciente. Es frecuente que este proceso coincida con artritis, colitis ulcerosa, enteritis regional de Crhon o enfermedades hematológicas con las cuales no hay necesariamente una relación causa-efecto. TRATAMIENTO: Es empírico, se han usado los corticoesteroides, la colchicina, la diaminodifenilsulfona, la clofazimina, la talidomina. Algunos responden bien a determinados fármacos y otros casoso a otros o no responden. El proceso de repente y sin causa aparente se detiene y se inicia la cicatrización CASO CLINICO Femenino de 17 años que consultó por lesiones ampollosas dolorosas, de contenido claro, en rodilla derecha, acompañada de fiebre con escalofríos, de 1 semana de evolución. Las mismas lesiones aparecen gradualmente en toda la pierna derecha, antebrazo derecho y glúteo izquierdo, que revientan y dejan la piel exulcerada. Ingresó febril, estaba pálida. En piel se observan múltiples lesiones ampollosas exulceradas, con fondo fibrinoso, muy dolorosas, ubicadas en rodilla y pierna derecha, otras más pequeñas en muñeca derecha y glúteo izquierdo. PREGUNTA Cual es la conducta a seguir ahora? RESPUESTA a.- Limpieza y tratamiento ambulatorio con penicilina. b.- Debridación y tratamiento hospitalario con corticoides y antibióticos. c.- Limpieza y alta con signos de alarma. d.- Limpieza local, vendaje y penicilina. TUBERCULOSIS (TB) CUTANEA. CIENCIAS BASICAS: La TB cutánea se considera en la mayoría de los casos de reinfección, la primoinfección es excepcional, aunque posible presentándose el complejo primario en la piel: nódulo cutáneo, linfangitis y adenopatías. SALUD PUBLICA: Mycobacterium tuberculosis, quizá el germen de mayor ubicuidad en el mundo y en el organismo pues no hay órgano que se escape a su agresión. La tuberculosis sigue siendo un problema de salud pública, especialmente en la forma pulmonar. La TB cutánea ocupa el 4to o 5to lugar, en cuanto a localización del bacilo de Koch. Estadísticas mexicanos en varios años muestran cifras que van de 1-3% de Tb cutánea por cada 114 pacientes y cerca de 20% son niños menores de 15 años. PATOGENIA: Casi en todos los casos, los pacientes han recibido ya la primoinfección pulmonar inoculado con M. tuberculosis bovina y hominis, presentando una respuesta positiva al PDD. A partir de este foco original mediante reinfeccionendogena.se diseminan los bacilos a la piel, otras veces la reinfección es exógena por llegada de nuevos bacilos del exterior. De acuerdo con esta vía de reinfección, de la virulencia del bacilo y sobre todo de la respuesta inmunológica (hipersensibilidad) del organismo ante el bacilo, se van a producir lesiones en la piel y que parecen entidades diferentes. CLASIFICACION: de Latapí, Escalona y Estrada; se consideran 2 grupos básicos, el primer grupo está formado por entidades en las cuales hay respuesta normérgica al bacilo. El segundo grupo comprende las llamadas tuberculoides, que como su nombre indica son lesiones resultados más de la respuesta exagerada, hiperérgica al bacilo. En este caso no es posible encontrar bacilos en las lesiones y la respuesta a los antígenos bacilares es muy intensa. TB COLICUATIVA: También conocida como escrufulosis o escrofulodermia, en México es la variedad más frecuente. Se presenta sobre todo en niños y jóvenes y predomina en las áreas de población más desprotegidas. Es secundaria habitualmente a TB de ganglios, huesos y articulaciones. Su topografía habitual es donde hay ganglios, como regiones supraclaviculares, las axilas, las ingles, uni o bilateral. Puede estar en región esternal, codos, rodillas. Las lesiones son siempre nódulos y gomas, refieren “pequeñas bolitas en el cuello”, es decir ganglios infartados, no dolorosos, no móviles, un buen día se fijan a la piel y esta se pone eritematosa e infiltrados y se inicia la formación de un nódulo, que se reblandece y se abre al exterior dejando salir pus de color amarillo claro y espeso, pronto toda la región se convierte en un plastrón endurecido (nódulos, gomas, abscesos fríos), es posible que dejen cicatrices deformantes y retractiles. Puede haber febrícula vespertina o franca fiebre, anorexia y adelgazamiento, y a veces síntomas de TB activa. TB VERRUGOSA: A menudo inoculada en personas que manejan material contaminado como mozos de anfiteatro, matanceros, carniceros, laboratoristas. Se localiza en partes distales, de os miembros como los pies y las manos y algunas veces en nalgas. En sitio de inoculación aparece el tubérculo anatómico, que es nódulo verrugoso, que es punto de partida de las lesiones que son siempre placas verrugosas o vegetante de tamaños variables circulares u ovales, bien limitadas y cuya superficie siempre es áspera, con costras melicericosanguineas, a veces se observan pústulas. En algunas placas se ve cicatrización central con crecimiento periférico y puede haber compromiso de los linfáticos superficiales y profundos de la región y producirse una estasis linfática que ocasiona edema y más verrugosidades. TB LUPOSA: Era muy frecuente en el siglo pasado. Es más frecuente en niños y jóvenes y su clásica topografía es en la cara; mejillas y dorso de nariz, disposición en “mariposa”, también puede afectar a los pabellones auriculares. La lesión fundamental es el nódulo pequeño llamado lupoma que se cubre de escamas y verrugosidades que lo ocultan, de tal manera que se forman placas eritematosas, escamosas, verrugosas, circulares, bien limitadas de crecimiento periférico y cicatrización central y con tendencia a la ulceración. En el centro solían producir extensas y profundas destrucciones, el subtabique nasal se destruía y la nariz tomaba el aspecto de “pico de loro”. TB MILIAR AGUDA: Muy rara se presentaba en niños y jóvenes con TB avanzada y baja reactividad a los antígenos. Son pequeños nódulos rojizos acuminados, a veces ulcerado y cubiertos de costras en diferentes partes del cuerpo, en especial en el centro de la cara, que cicatrizan espontáneamente.

CURSO ENARM CMN SIGLO XXI TEL: 36246001

Pharmed Solutions Institute

PÁGINA 390

MANUAL DE TRABAJO DEL CURSO ENARM CMN SIGLO XXI TB NODULAR PROFUNDA: Descrita por Bazin, con el nombre de eritema indurado, casi exclusiva de mujeres de localización única en las piernas sobre todo en las caras posteriores y caracterizada por nódulos profundos, de evolución crónica y recidivante, muy dolorosos y que en su evolución dejan zonas atróficas depreimidas que deforman las piernas. TB NODULO NECROTICA: Es menos frecuente que la nodular profunda, aparece en personas jóvenes con topografía en partes salientes; codos, rodillas, nalgas y en ocasiones en la cara, sobre todo en los pabellones auriculares. La lesión es un pequeño nódulo que sufre una necrosis central y así se cubre de una costra negruzca que al caer deja una cicatriz varioliforme. Son asintomáticos y evolucionan por brotes. TB MICRONODULAR: Es muy rara y pasa inadvertida, son nódulos de 1-2mm que forman placas de aspecto folicular en las regiones lumbares u otras partes del tronco, son asintomáticas. DIAGNOSTICO: Es más bien de eliminación, dado que la comprobación del agente causal no es fácil en todos los casos. El laboratorio nos puede ayudar sobre todo para descartar procesos que suelen semejarse a las lesiones tuberculosas. Hallazgo del bacilo; puede hacerse por medio de baciloscopia y de biopsia difícil de encontrar en TB colicuativa y ulcerosa. El cultivo en medios apropiados como Lowestein es difícil de lograr. Histopatología; por definición la imagen de las lesiones tuberculosas es el clásico granuloma tuberculoide, constituido por linfocitos, células epiteloides y células gigantes tipo Langhans, resultado de la unión de las segundas., sin embargo no es patognomónica de la tuberculosis, por lo que solo será sugestiva de tuberculosis. Intradermoreacción con PPD; no es diagnostica ya que solo nos indica un contacto con el bacilo de Koch y un alto porcentaje de la población da positivo. Tiene importancia cuando es negativa, lo cual significa que no tienen un cuadro de origen tuberculoso. TRATAMIENTO: Se usan las mismas drogas que en la tuberculosis en general y a dosis semejantes. La estreptomicina es muy útil a dosis de 1 gr diario o cada tercer día, 50-60grs como dosis total. La isoniacida que se dice bacteriostática se usa a 5-8mg/kg. El etambutol a dosis de 15-20mg/kg, es más costoso y tienen efectos colaterales en nervio óptico, es reversible. Rifampicina a dosis de 10-20 mg/kg, para un adulto se aconsejan 600mgs, para niño la mitad. No usar monoterapia. Tratamiento ideal es; 1 gr de estreptomicina cada tercer día, 600mgs de isoniacida diarios o bien estreptomicina + etambutol. La TB colicuativa tarda de 3-6 meses en curar. La TB verrugosa y luposa requieren menos tiempo. En la actualidad se está aplicando con éxito un triple tratamiento, isoniacida, rifampicina, pirazinamida. CASO CLINICO Niña de 10 años que consulta por una lesión en mejilla izquierda seguida de la aparición de adenopatía cervical, de dos meses de evolución. Al examen físico, se observa una pequeña úlcera de 1 cm de diámetro, con leve descamación periférica y adenopatía satélite. PREGUNTA Cual es la conducta a seguir? RESPUESTA a.- Biopsia de la lesión. b.- PPD. c.- BAAR. d.- Cultivo de la lesión. PITIRIASIS ALBA (MAL DEL PINTO). CIENCIAS BASICAS: Es una condición benigna de lapiel común en la infancia, descrita por primera vez hace más de 80 años y su importancia radica en que es un motivo de consulta frecuente debido a su curso crónico, tendencia a reaparecer y su impacto estético. SALUD PUBLICA: La pitiriasis alba se encuentra en todas partes del mundo. Afecta a entre 1.9-5.25% de los niños en edades entre 6-16 años. En general, no hay predisposición en cuanto a sexo; sin embargo, algunas series describen una mayor afección de los varones, así como a individuos de color de piel oscura.3 Se cree que la mayor frecuencia de pitiriasis alba en personas de piel oscura es debido al hecho de que su piel se tiñe más fácilmente excepto en las áreas afectadas. Hay reportes que muestran mayor incidencia en niños escolares de baja situación socioeconómica. Desde el punto de vista clínico afecta la cara en el 50% de los casos. PATOGENIA: La etiología no está bien establecida y a la condición se le han adjudicado diversas denominaciones, como eritema streptogenes, pitiriasis streptogenes, e impetigo furfuracea, usualmente relacionadas con los agentes etiológicos propuestos, sin embargo, no se ha identificado una asociación causal definitiva. Microorganismos como Pityrosporum, Streptococcus, Aspergillus y Staphylococcus, se han implicado en su etiología pero ninguno de ellos se ha confirmado como agente causal. Varios factores además de los infecciosos, se han asociado con la etiología, la exposición solar, la humedad relativa del aire, la altitud y el viento. El uso de jabones abrasivos, duchas excesivamente largas y la temperatura caliente del agua durante el baño también se han relacionado con la xerosis que se aprecia en esta entidad. En cuanto a la exposición solar, se plantea que la radiación ultravioleta induce una irritación excesiva de la piel que conlleva a una reacción inflamatoria y que además la acción directa de la luz solar sobre los melanocitos modifica su número y su función conduciendo a una disminución de la intensidad del color de las lesiones en la pitiriasis alba. Esto además se apoya en el hecho de presencia de lesiones en áreas fotoexpuestas y en niños en edades en las cuales hay mayor número de actividades al aire libre. Se ha observado que el uso de protector solar frecuente FPS > 15 reduce el desarrollo de pitiriasis alba. La condición dermatológica que se asocia principalmente con la pitiriasis alba es la dermatitis atópica y la presencia de deficiencias nutricionales así como la xerosis es común a ambas, por lo que se ha implicado en la patogénesis de la misma. Las deficiencias nutricionales involucradas son de vitaminas y bajos niveles de cobre, el cobre es un cofactor para la tirosinasa, enzima necesaria para la producción de melanina, por lo que su deficiencia puede jugar papel patogénico en esta entidad. DIAGNOSTICO: Frecuentemente se inicia como placas rosadas con un borde elevado que luego de varias semanas se desvanece dejando una mancha pálida cubierta por una descamación blanquecina polvorienta. Posteriormente progresa a máculas hipopigmentadas de bordes difusos, de tamaño variable entre 0.5 a 5 cm de diámetro. Aunque puede haber ligero prurito en general las lesiones son asintomáticas, por lo que generalmente el paciente no consulta por ellos y se observan incidentalmente en el examen físico. Aunque en el caso de los niños, puede ser motivo de preocupación común en las madres, ya que la mayoría de las lesiones se localizan en la cara, principalmente en la frente y la zona malar, pero también pueden encontrarse en las extremidades superiores y ocasionalmente en las inferiores. Las lesiones pueden persistir por 6 meses a 7 años, pero este curso puede prolongarse en los pacientes atópicos; en general es una condición autolimitada. Por otro lado, las lesiones pueden ser más visibles en el verano, cuando la piel circundante está bronceada. Luego de la resolución las lesiones pueden reaparecer en la misma localización. Variantes clínicas: PITIRIASIS ALBA EXTENSA: esta entidad se caracteriza por

CURSO ENARM CMN SIGLO XXI TEL: 36246001

Pharmed Solutions Institute

PÁGINA 391

MANUAL DE TRABAJO DEL CURSO ENARM CMN SIGLO XXI lesiones de pitiriasis alba generalizadas y no precedidas de eritema. Se observa más frecuentemente en los adultos. Generalmente se afectan la porción inferior del tronco de manera simétrica. Es asintomática y los pacientes usualmente no tienen antecedentes o historia de atopia. La histología no es específica, se observa un número reducido de melanocitos funcionales con menor número de melanosomas, pero su distribución y transferencia a los queratinocitos no está perturbada. PITIRIASIS ALBA PIGMENTADA: es una variante que puede estar asociada con una infección por dermatofitos superficial y la forma clásica de PA. Se observa una hiperpigmentación azulada rodeada de un área hipopigmentada y descamativa muy parecida a la pitiriasis alba clásica. Casi siempre aparece en la cara y rara vez hay compromiso extrafacial. El área pigmentada se atribuye a depósitos de melanina en la dermis, lo que no se observa en la pitiriasis alba clásica. En general no es necesario ya que el diagnóstico clínico suele ser obvio, además de que la histología no es específica. Lo destacable es una menor cantidad de melanocitos y melanosomas en la capa basal de la epidermis, sin embargo, esto se ha asociado a inflamación determinada por cierto grado de espongiosis, lo que interfiere con la producción de melanina. Desde el punto de vista histológico puede llegarse a un diagnóstico si se observa: pigmentación irregular de la melanina en la capa basal, taponamiento folicular, espongiosis folicular y glándulas sebáceas atróficas. Además de infiltrado linfocitario y edema en la dermis. Estos cambios son variables dependiendo de la etapa en la que se encuentre la enfermedad. TRATAMIENTO: Inicialmente en convencer al paciente de que la enfermedad es benigna y autolimitada ya que hasta ahora, ninguna terapia es completamente exitosa. Se recomienda limitar la exposición solar, uso regular de protector solar FPS > 15 y reducir la frecuencia y la temperatura de los baños. Los emolientes y cremas hidratantes ayudan a disminuir la xerosis y la irritación. En los casos en etapa inflamatoria la desonida o la hidrocortisona tópica al 1% pueden ayudar a la resolución de las lesiones. En los niños solo deben prescribirse esteroides de baja potencia no halogenados. La forma extensa de PA no responde a los esteroides tópicos pero si lo hace a terapia con PUVA. Recientemente se publicó la efectividad de tacrolimus al 0.1% en el tratamiento de esta entidad. El tacrolimus es un macrólido que bloquea la función catalítica de la calcineurina, inhibiendo la síntesis y la liberación de IL-2. La eficacia y la seguridad de tacrolimus en ungüento en el tratamiento de la dermatitis atópica está bien documentada. Sin embargo, el mecanismo de acción en la pitiriasis alba no es bien entendido. Una intervención terapéutica definitiva será posible cuando se comprenda mejor la patogénesis de esta entidad, a la cual no se le da mucha importancia en la literatura médica, lo que ha conllevado a la escasez de conocimientos precisos en cuanto a su patogenia y de estudios controlados para dilucidar una terapéutica totalmente efectiva. CASO CLINICO Niño de 6 años de edad el cual acude a consulta de rutina, muestra en la cara manchas blancas, bordes irregulares no bien definidas, con leve descamamiento a la exploración, no son pruriginosas? PREGUNTA Cual es la conducta a seguir menos adecuada? RESPUESTA a.- Protectores solares. b.- Baños cortos con agua templada. c.- betametasona. d.- Tacrolimus. HERPES SIMPLE. CIENCIAS BASICAS: Los virus de herpes se encuentran ampliamente distribuidos en la naturaleza y la mayoría de las especies animales son hospederos naturales de más de uno. Se han aislado y caracterizado más de 100, varios de los cuales afectan al humano: herpes simple tipo 1 y tipo 2 (VHS-1, VHS-2), varicela zoster (VVZ), citomegalovirus (CMV), Epstein Barr, (VEB), virus herpes humano 6 (VHH6), virus herpes humano 7 (VHH7), virus herpes humano 8 (VHH8) y otros. SALUD PUBLICA: La infección por VHS-1 es frecuente en sitios hacinados y con condiciones precarias de higiene se tienen porcentajes de 90% de la población tienen anticuerpos antivirales El más frecuente en niños. El VHS-1 en ubicación oral se transmite por saliva, besos, por compartir vasos, cepillos de dientes y en otras partes del cuerpo se debe a contacto del virus con la piel, se autotransmite con frecuencia, principalmente a los ojos. La infección por VHS-2 depende de la actividad sexual. El VHS-2 se transmite por secreciones vaginales, contacto sexual (prepuberes) y al neonato durante el paso por el canal de parto infectado. La posibilidad de que VHS-1 y VHS-2 establezcan infecciones latentes con recidivas asintomáticas favorece su transmisión, ya que un individuo infectado puede ser transmisor durante toda su vida. Los virus infectantes se encuentran en el líquido de las vesículas. PATOGENIA: La patogenia de VHS-1 y VHS-2 es similar, con infección primaria generalmente asintomática, aunque pueden presentarse lesiones vesiculares. El virus inicia la infección en las membranas de las mucosas, se replica en las células mucoepitelilales originando infección lítica y se disemina a las células adyacentes y neuronas que inervan el sitio donde se inició la infección aguda. La infección latente en la neurona no produce daño aparente, pero diferentes estímulos la pueden reactivar. Una vez reactivado, el virus se multiplica, viaja a lo largo del nervio en forma centrifuga y ocasiona lesión en la terminal del nervio, por lo tanto todas las recidivas se producen en el mismo sitio. La expresión del genoma se requiere para la reactivación, pero no para el establecimiento de la latencia. El mecanismo para su establecimiento se desconoce, sin embargo, se piensa que para la expresión del genoma se requiere una proteína celular, tan es así que no en todas las estirpes celulares establece latencia. El tipo de infección que resulta depende del estado inmune del individuo; los sujetos susceptibles desarrollan infección primaria después de la primera exposición al virus. Sujetos seropositivos pueden ser reinfectados con virus de otro tipo. VHS-1 y VHS-2 se transmiten por diferentes vías e infectan diferentes sitios del cuerpo. A grandes rasgos se considera que el HSV-1 infecta de la cintura para arriba y el VHS-2 de la cintura para abajo, sin embargo esta diferenciación no es estricta. DIAGNOSTICO: La primoinfección por VHS-1 (ORAL): puede presentarse de distintas formas: La gingivoestomatitis herpética; forma más frecuente de infección primaria o inicial en niños. Se trata de un cuadro agudo con fiebre y mal estado general, aparición de lesiones vesiculosas y erosivas en mucosa oral que ocasionan odinofagia y halitosis características, dificultando la alimentación. Pueden existir lesiones vesiculosas, en labios e incluso en la cara, agrupadas en racimos (típicas). Podemos encontrar adenopatías submandibulares y el cuadro clínico es auto limitado (7-10dias). Otras formas de primoinfección, pueden aparecer lesiones en cualquier localización, solo es precisa una fuente de contagio (familiar), una puerta de entrada (una erosión, arañazo) y un huésped susceptible (niño). Podemos encontrar casos de primoinfecciónes

CURSO ENARM CMN SIGLO XXI TEL: 36246001

Pharmed Solutions Institute

PÁGINA 392

MANUAL DE TRABAJO DEL CURSO ENARM CMN SIGLO XXI con lesiones faciales extensas, en un miembro, oculares, y lo que es más frecuente y bien definido, el panadizo herpético, caracterizado por la aparición de lesiones cutáneas vesiculosas, que afectan al pulpejo y región periungueal de un dedo, muy dolorosas, que cursan con fiebre y adenopatías satélites. El cuadro también es autoinoculativo, pero dura algo más; unas 2 semanas. Es bien conocida por todos la capacidad de los virus simple de permanecer en el huésped, acantonados y recurrir o producir síntomas ante diversas circunstancias; enfermedades debilitantes, infecciones, fiebre, exposición solar. La clínica es variable pero suele localizarse en la cara; labios, región perioral, mejillas. Otras localizaciones son más raras en la infancia. El aspecto clínico es muy característico con la aparición de lesiones vesiculosas, agrupadas en racimo, sobre una piel eritematosa, que con el tiempo dan lugar a una costra. El cuadro puede ir precedido de sensación prodrómica de dolor, quemazón o disestesias de difícil descripción e interpretación por parte de nuestros pacientes y no suele acompañarse de fiebre, mal estado general, ni adenopatías. La infección por VHS-1 puede originar cuadros clínicos de variada severidad, que oscilan desde la gingovoestomatitis, herpes labial, panadizo herpético, meningitis, encefalitis con alta mortalidad y queratitis herpética que a su vez puede originar ceguera. VHS-2 (GENITAL): En general no ocurre en edad pediátrica, cuando aparece en un menor debe investigarse la posibilidad de abusos sexuales. La primoinfección es superponible a las formas orofacilaes, pero en la región genital. El cuadro también es agudo se acompaña, de dolor, fiebre, dificultad para la micción y adenopatías. Las lesiones vesiculosas, erosivas, típicamente agrupadas en racimos nos darán sospecha diagnostica. El diagnóstico de laboratorio económico y rápido se realiza mediante la prueba de Tzank que consiste en hacer una impronta de las células y teñirlas con el colorante de Wright o Giemsa, y observar células fusionadas con varios núcleos, sincitios, así como inclusiones nucleares de Cowdry. No es posible confirmar la presencia del virus por medio de esta técnica ya que otros virus producen el mismo efecto en las células, inclusive el VVZ. Actualmente se hace uso de técnicas inmunoenzimáticas, biológicas, bioquímicas y de biología molecular para detectar anticuerpos o antígenos virales. La detección de anticuerpos solo es de utilidad para detección de la primo infección y estudios epidemiológicos. TRATAMIENTO: Existen antivirales efectivos utilizados en el tratamiento de infecciones por VHS-1 y VHS-2, entre ellos famciclovirn (125mgs c/12hrs por 5 dias), Aciclovir (200mg 5 veces al dia por 5 dias), valaciclovir (500mgs c/8 hrs por 5 dias). Los antivirales no eliminan las partículas virales que se encuentran en ganglios neurales, solo impiden su replicación, por lo que pueden presentarse reactivaciones. En estos casos, el uso de dosis mínimas de los fármacos, durante un tiempo prolongado, tratamiento denominado supresivo, se indica en los pacientes con brotes constantes, prolongados o intensos, aunque debe contemplarse la posibilidad de resistencias. Los VHS se transmiten a través de las secreciones de lesiones, por lo es recomendable evitar contactos durante la lesión activa. El tratamiento tópico debe hacerse mediante fomentos con sulfato de cobre 1/1000 o sulfato de cinc 1/1000 y antibiótico tópico (mupirocina, ac. Fusidico) COMPLICACIONES: La más frecuente es la impetiginizacion o sobreinfección bacteriana, fundamentalmente estreptocócica y estafilocócica. Las lesiones oculares en particular, la queratitis puede llevar a ceguera. INFECCION HERPETICA NEONATAL: La infección herpética durante la gestación puede dar lugar a manifestaciones en el RN y en el feto. La vía de contagio puede ser transplacentaria o genital (en el canal del parto, o ascendente, con o sin rotura de membranas). Existen 2 posibilidades: infección primaria antes de la 20 SDG, aborto, muerte fetal, hidrocefalia y coriorretinitis. Herpes simple neonatal, en más de 90% de los casos es en relación con la primoinfección herpética materna durante la gestación, solo 5% se produce afección fetal en las recurrencias. Cuando la infección ha sido adquirida en la vida intrauterina (5-15%), la sintomatología aparece en las primeras 2448hrs. En caso de infecciones intrapartum, generalmente por VHS-2 (madre) o en el caso de VHS-1 (padres, familiares) la sintomatología se produce entre los 2-20 días. Clínica: afectación severa del estado general en un neonato aparentemente sano, lesiones cutáneas, coriorretinitis, y afección al SNC, afectación sistémica-visceral, lesiones cutáneas en más de 50% de los casos: vesículas, ampollas, pústulas, cicatrices, eritema generalizado, erosiones, anoniquia. Mortalidad elevada (40%) y elevada morbilidad. Tratamiento precoz cesárea. CASO CLINICO Femenino de 15 años de edad la cual acude a consulta por presentar desde hace dos días, vesículas agrupadas en el labio inferior, que se transforman en pustulas y costras hemorrágicas, cuenta antecedente de prurito y sensación de hormigueo en la zona. Comenta la paciente que en el último año se han repetido episodios de vesiculas similares en varias ocacion, antecentes de importancia actualmente embarazo de 22 semanas, ha tenido multiples parejas y es madre soltera. PREGUNTA Cual es la conducta a seguir mas adecuada? RESPUESTA a.- Tratamiento antiviral doble esquema. b.- Realizar USG para identificar anormalidades. c.- Tratar y esperar parto nornal. d.- Tratamiento especifico y programa cesarea aportuna. HERPES ZOSTER. CIENCIAS BASICAS: El virus de la varicela (VVZ) origina varicela y su reactivación causa herpes zoster. SALUD PUBLICA: Más frecuente en el adulto que en la infancia (0.74 casos por 1000 individuos al año). Suele ocurrir en niños que pasaron varicela precoz (primeros 2 años de vida). El virus se transmite por contacto directo, por gotitas de aerosol y por vía área en comunidades; también puede haber contagio por contacto con vesículas cutáneas. Es altamente transmisible. Se dispone de eficaz vacuna atenuada. PATOGENIA: Se caracteriza por una infección neurológica y dermatológica, posterior a un episodio de varicela (primoinfección) en los ganglios sensitivos espinales y craneales (donde esta latente el VVZ). Una vez reactivado, el virus se mueve a lo largo de las fibras sensitivas hasta él área de piel que inervan, que usualmente afectan de 1-3 dermatomas. Dicha reactivación ocurre, por lo general, una vez en la vida, sobre todo en pacientes de la tercera edad o en aquellos inmunodeprimidos (por HIV, cáncer, tratamiento con corticoesteroides, fármacos inmunosupresivos, por ejemplo). DIAGNOSTICO: Las lesiones cutáneas consisten en lesiones papulosas iniciales agrupadas en racimos sobre la piel eritematosa, las pápulas se convierten en vesículas en 12-24 hrs y luego estas se rompen y dan lugar a costras. La erupción cura en 7-14 días y suele acompañarse de dolor. El dolor puede ser prodrómico, es decir preceder a las lesiones cutáneas, acompañarlas e incluso persistir durante meses tras la resolución clínica dermatológica, en este último supuesto se

CURSO ENARM CMN SIGLO XXI TEL: 36246001

Pharmed Solutions Institute

PÁGINA 393

MANUAL DE TRABAJO DEL CURSO ENARM CMN SIGLO XXI habla de neuralgia postherpética, esta implica la pérdida de autonomía y una calidad de vida pobre. La infección por varicela puede ocasionar ocasionalmente que el virus atraviese la placenta, dando lugar al síndrome de varicela congénita que puede ser leve o severo. Cuando la madre ha estado en contacto con casos de varicela es recomendable determinar el nivel de anticuerpos anti-varicela. El diagnóstico se basa en la prueba de Tzank, con las dificultades mencionadas; el virus se puede aislar de las vesículas y recientemente se utilizan técnicas de biología molecular para su identificación. COMPLICACIONES. La neuralgia postherpética. En individuos inmunocomprometidos puede dar lugar a enfermedades progresivas y graves, principalmente encefalitis post-infección y neumonía, en algunos casos fatales. La respuesta inmune celular que generalmente es más potente en adultos que en niños causa durante la infección primaria lesiones y cuadros graves, principalmente en pulmones, y da lugar a 20-30% de los pacientes con neumonía intersticial. TRATAMIENTO: Resultados satisfactorios se han obtenido con inhibidores de la síntesis del ADN (aciclovir y derivados), sin embargo se requieren dosis más altas que las recomendadas para HSV-1 y 2. HERPES ZOZTER EN LA INFANCIA: Las principales características son: 1. Suele presentarse en niños que presentaron varicela en los 2 primeros años de vida. 2. Su curso es benigno y sin complicaciones. 3. Es poco doloroso (no pródromos), no suele complicarse con neuralgia postherpetica. 4. Se recomienda investigar existencia de inmunodepresión. 5. El tratamiento oral con antivirales solo está indicado en casos seleccionados, pero debe ser lo más precoz posible. CASO CLINICO Masculino de 6 meses de edad quien manifestó una dermatosis diseminada al tronco, región inguinal derecha y la cara lateral interna del muslo derecho en el tercio distal, asi como en el tercio medio y proximal de la pierna. La dermatosis era unilateral, constituida por vesículas agrupadas sobre una base eritematosa y escasas costras hemáticas que confluida entre si, formando varias placas de tamaño variable y de evolución aguda y dolorosa, se realizo diagnostico clínico de herpes zoster. PREGUNTA Cual es la conducta a seguir menos apropiada? RESPUESTA a.- Aciclovir 400 mg/kg/dosis. b.- Fomentos con sultafo de cobe y oxido de cinc. c.- Profilaxis antibiótica profiláctica. d.- Cerco sanitario a contactos. VERRUGAS VIRALES. CIENCIAS BASICAS: Son los virus del papiloma humano capaces de producir las verrugas virales y algunos cuadros relacionados con la carcenogénesis. Se han descrito ya 25 tipos de virus. La palabra verrugosidad indica solamente una lesión dura, anfractuosa, seca que histológicamente presenta hiperqueratosis y papilomatosis. La vegetación en cambio es blanda, anfractuosa, friable y solo presenta la papilomatosis, por ello estas lesiones solo se presentan en zonas húmedas de la piel y en mucosas. Las verrugas virales son neoformaciones epiteliales benignas que pueden afectar a cualquier persona, niños, adultos, hombres y mujeres, constituyendo una de las 5 entidades que con más frecuencia se ven en la consulta. Su transmisibilidad es baja y sabemos que son autoinoculables. Un estado inmunológico deteriorado permite sui extensión y persistencia. TIPOS: Vulgares, planas, plantares, acuminadas y filiformes. VERRUGAS VULGARES: Son los populares “mezquinos” y son las más frecuentes. Predominan en los niños. Son neoformaciones verrugosas solitarias o múltiples, de superficie anfractuosa, secas, duras, de color de la piel o más oscuras, semiesféricas y bien limitadas, presentan en su superficie un fino puntilleo oscuro. Son indoloras a menos que se les traumatice y su desaparición es espontánea y no deja secuelas, habituales en extremidades superiores. VERRUGAS PLANAS o juveniles: Son muy pequeñas y numerosas, no más de 1mm y muy aplanadas, levantan de la superficie, así es que a primera vista parecen machas. De color de la piel, llegan a sumar más de 100, localizadas de preferencia en las mejillas aunque también pueden verse en extremidades superiores; dorso de manos o antebrazos. También son asintomáticas. VERRUGAS PLANTARES: Se les conoce vulgarmente como “ojos de pescado” y como su nombre o indica se presentan en la planta del pie, la presión de peso del cuerpo las hunde en la gruesa capa cornea, por lo que solemos ver “la base de pirámide”, y ello explica el dolor que las acompaña, únicas o múltiples, con pequeñas hemorragias postraumáticas en su superficie. VERRUGAS ACUMINADAS: También llamadas condilomas acuminados o papilomas venéreos, la mayoría se adquieren por la relación sexual, en niños a veces por desaseo. Asientan en sitios húmedos y calientes, alrededor de orificios naturales; glande, labios mayores y menores, ano recto, boca, no se produce hipertrofia como en otras verrugas, la lesión toma más bien un aspecto vegetante, como coliflor, pueden ser pequeñas a penas salientes (crestas en el término) o alcanzar dimensiones monstruosas, deformando la región, se maceran e infectan y adquieren un olor muy desagradable. Debemos diferenciarlas siempre de condilomas planos o sifílides papuloerosivas, muy ricas en treponemas. Son pápulas aplanadas y erosionadas y no neoformaciones vegetantes, no tiende a persistir, evolucionan hacia la desaparición. Los condilomas acuminados no tienden a la involución, persisten y crecen. VERRUGAS FILIFORMES y digitiformes: Se presentan en los pliegues: cuello axilas, ingles en forma de neoformaciones alargadas, como hilos oscuros, algunas pediculadas o con prolongaciones transparentes. Se discute su etiología viral. PRONOSTICO: Aunque se menciona el papel oncogénico de algunas variedades de parvovirus, no debe tomarse a las lesiones verrugosas comunes como precancerosas, sin embargo en el caso de lesiones acuminadas, se piensa que pueden relacionarse con el cáncer cervicouterino y se ha observado que las mujeres que presentan esta neoplasia tienen con más frecuencia verrugas acuminadas y no solo eso, sino que se ha observado también que las parejas sexuales han padecido la infección genital por estos virus. TRATAMIENTO: Involución excepto en las acuminadas. Es una crueldad innecesaria traumatizar a un niño quemándole sus verrugas con ácidos mediante electrofulguración. Se han visto antiestéticas cicatrices, a veces queloides como resultado de estas iatrogénicas maniobras. La destrucción de pocas lesiones puede hacerse con electrofulguración o congelándolas con nitrógeno líquido. Las acuminadas requieren aplicación de soluciones de podofilina al 20-40% en solución alcohólica o en aceite, protegiendo tejidos vecinos con vaselina o colodión. Igualmente son molestas las elctrofulguraciones de las verrugas plantares, pues solo se destruye una mínima porción y es inútil poner al paciente en cama varios días, cuando la recidiva es la regla. Ya se sabe que las verrugas tienden a la involución espontáneamente, está probado científicamente y con estudios estadísticos, la utilización de los placebos (en dosis mínimas,

CURSO ENARM CMN SIGLO XXI TEL: 36246001

Pharmed Solutions Institute

PÁGINA 394

MANUAL DE TRABAJO DEL CURSO ENARM CMN SIGLO XXI magnesia calcinada, glucosa, cloruro de sodio, BCG, levamisol) en estas afecciones tan orgánicas Debe tenerse en mente que las verrugas virales tarde o temprano desaparecerán, no así las cicatrices que perduran toda la vida y recordaran al paciente los malos tratamiento recibidos. CASO CLINICO Se trata de masculino de 17 años con dermatosis localizada en cara, área de la barba con extensión al cuallo constituida por numerosas neoformaciones (90) que varia de 1 a 3 mm de diámetro con proyecciones digitiformes que dan aspecto verrugoso y papiliforme, del color de la piel, solo presentaba prurito ocasional, estudios de rutina dentro de parámetros normales. PREGUNTA Cual es la consuta a seguir mas adecuada? RESPUESTA a.- Electrofulguracion. b.- Nitrógeno líquido. c.- Podofilina. d.- 5-fluorouracilo. MOLUSCO CONTAGIOSO (MC). CIENCIAS BASICAS: El molluscum contagiosum (MC) es una patología cutánea benigna ocasionada por un virus DNA de la familia de los poxvirus. A pesar de tener una distribución por todo el mundo, su morbilidad es baja. No obstante, actualmente existen más individuos afectos al añadirse los pacientes inmunocomprometidos. SALUD PUBLICA: El MC tiene una distribución mundial y una incidencia de 4,5%. No tiene predilección racial o entre géneros. Los estudios epidemiológicos realizados sugieren que la transmisión podría estar relacionada con factores tales como la humedad y calor del clima, la pobreza, hacinamiento y la falta de higiene. Sin embargo, en otros trabajos se apoya más la idea del contacto entre personas, ya que se ha encontrado que se puede desarrollar en el 35% de los miembros expuestos de una familia y que en la mayoría de los casos las familias poseen excelentes estándares de higiene. En niños la transmisión se produce por contacto piel-piel, por fómites o por auto inoculación. La infección por MC ocurre en todos los grupos etáreos, y al parecer ha venido aumentando en la última década. La mayor incidencia se observa en niños menores de 5 años, por el mayor contacto físico casual entre ellos, y a la auto inoculación. La infección en lactantes es rara, esto se explica por la persistencia de anticuerpos maternos. Otro pico en la incidencia se observa en adultos jóvenes, debido a la propagación secundaria al contacto sexual. El MC está incrementándose cada vez más dentro de las enfermedades diagnosticadas en la población sexualmente activa. La transmisión en adultos normalmente es vía sexual. Esta vía se apoya en la localización mayoritaria encontrada en el área genital, la frecuencia de encontrar las parejas afectadas y la coexistencia de otras enfermedades de transmisión sexual entre los pacientes afectos de MC. Otras vías posibles han sido a través de saunas, masajes, cirugía. Desde 1983 se han recogido diversos casos de MC en pacientes con infección HIV. La prevalencia en este grupo de población es tan alta como del 5-18% . PATOGENIA: El molluscum contagiosum es producido por un miembro del grupo de los poxvirus, no clasificado, de ultraestructura análoga a la del virus de la viruela. El virus del MC es uno de los más grandes que causan enfermedad en los humanos, el vibrión maduro es una partícula que mide 150x350 mm2. El genoma, al igual que en los demás poxvirus, es una única molécula linear de DNA de doble hélice. Estudios recientes del DNA de los molluscum aislados confirman la presencia de dos diferentes subtipos, MCV I y MCV II, con genomas de 185 kb y 195 kb respectivamente. Las lesiones producidas por cada subtipo son indistinguibles. No existen diferencias en cuanto a la distribución de ambos subtipos, encontrándose ambos a nivel de lesiones genitales y fuera de esta zona, pero no se hallan MCV tipo II en ningún paciente de edad menor a 15 años. Todos los MC aislados de un mismo paciente pertenecen al mismo genotipo así como los aislados de los miembros infectados de su familia. Se ha observado la existencia de MC en pacientes inmunocomprometidos. La mayoría de los pacientes muestran una deficiencia en su función o número absoluto de linfocitos T. Un grupo heterogéneo de enfermedades se han descrito en asociación al MC: atopia, neoplasias, estados yatrógenos de inmunosupresión, los pacientes atópicos un incremento en la susceptibilidad a la infección cutánea viral podía estar relacionado con la piel eccematosa que abriría una puerta de entrada a la colonización vírica. DIAGNOSTICO: El periodo de incubación de la infección es de 14 a 50 días, aunque hay datos de recién nacidos con lesiones al cabo de 7 días postparto. Las lesiones se inician como neoformaciones que miden generalmente de 2 a 6 mm, aunque pueden llegar a medir 3 cm, son hemisféricas, cupuliformes, lisas, del color de la piel o perladas, algunas (20%) tienen una umbilicación central; la base es levemente eritematosa y son de consistencia firme, cuando se rompen. Se localizan en cualquier parte del cuerpo e incluso pueden afectar mucosas, generalmente se agrupan en un área específica, pero pueden estar diseminadas en personas infectadas con el virus de la inmunodeficiencia humana adquirida, siendo un marcador de enfermedad avanzada, el material es blanquecino y cremoso. En el caso de los niños las lesiones se localizan normalmente en cara, tronco, brazos y piernas a diferencia de los adultos jóvenes en quienes el molusco contagioso se adquiere por transmisión sexual, las lesiones tienen predilección por genitales, abdomen y cara interna de los muslos. Las lesiones del molusco suelen aparecer entre los 14 días a 6 meses después de la exposición, hay datos de recién nacidos con lesiones al cabo de 7 días postparto. Se pueden propagar por auto inoculación, pero es típico que se resuelvan espontáneamente en pocos meses. Las lesiones son asintomáticas en la mayoría de los pacientes, aunque en el 10% de los casos puede haber prurito y desarrollarse una reacción eccematosa. El diagnóstico se hace clínicamente y en algunos casos dudosos puede efectuarse biopsia con tinción H-E donde se encuentran los cuerpos de molusco (inclusiones intracitoplasmáticas grandes) o de Hendersen-Paterson; el 90% de los pacientes posee Ac tipo IgG. Puede realizarse microscopía electrónica, PCR, Elisa, e inmunohistoquímica. TRATAMIENTO: Enfermedad autolimitada que eventualmente se resolverá en la mayoría de los pacientes pero en niños atópicos e inmunosuprimidos la evolución es larga y tórpida. Primera elección es imiquimod 5% crema cada 2 dias hasta que desaparezcan las lesiones (aprox. 4 sem). Curetaje: Es un método sencillo y disponible de forma rápida. Se realiza con una cureta simple, doloroso por lo que se recomienda el uso de algún anestésico tópico previo a la realización del mismo. Tiene la ventaja de proveer de una muestra de tejido para la confirmación del diagnóstico en caso de dudas. Criocirugía: Es un método eficiente y rápido para el tratamiento de las lesiones. El nitrógeno líquido se aplica sobre cada lesión por pocos segundos. Se repiten las sesiones cada 2 a 3 semanas según sea necesario. Entre las desventajas se encuentran el dolor, hiper o

CURSO ENARM CMN SIGLO XXI TEL: 36246001

Pharmed Solutions Institute

PÁGINA 395

MANUAL DE TRABAJO DEL CURSO ENARM CMN SIGLO XXI hipo pigmentación y cicatriz residual. La podofilina se aplica en tintura al 25% en alcohol una vez a la semana en cada lesión. Los efectos secundarios descritos son locales como erosión de la piel sana circundante y sistémicos como daño renal, neuropatía, íleo paralítico, leucopenia y trombocitopenia. El podofilotoxina es uno de los componentes activos de la resina de podofilina, es más seguro que la podofilina y puede ser usado por el paciente en casa. La aplicación recomendada es 0,05 ml de podofilotoxina al 5% en etanol tamponado en lactato dos veces al día por 3 días. Está contraindicado durante el embarazo. Se coloca sobre las pápulas una solución de yodo al 10% y al secar se cubren con ácido salicílico al 50% y cubren con adhesivo. El proceso se repite cada 24 horas hasta que las pápulas se tornen eritematosas lo cual ocurre entre el 3er día y el 7mo, después de lo cual solo se coloca solución yodada. Se ha reportado resolución de las lesiones con una media de 26 días. Puede ocurrir maceración y erosión. CASO CLINICO Un varón de 10 años de edad, nacido del matrimonio consanguíneo fue llevado con múltiples lesiones elevadas blancas como perlas en todo el cuerpo desde hace 6 meses, asintomáticas que se fueron generalizando. No había antecedentes de múltiples y recurrentes lesiones llenas de pus en el pecho, las ingles, junto con infecciones pulmonares recurrentes. No había antecedentes personales o familiares de atopia. No había antecedentes de ningún dentales, problemas óseos, neurológicos o cualquier facies distintivas. De los tres hermanos, un hermano más joven tenía antecedentes de lesiones de la piel similar en todo el cuerpo. PREGUNTA Cual es la conducta a seguir. RESPUESTA a.- Crioterapia y curetaje. b.- Conducta expectante. c.- Nitrito acidificado. d.- Imiquimod tópico. ESCABIASIS. CIENCIAS BASICAS: También llamada sarcoptosis, sarna. Es una enfermedad altamente transmisible, cosmopolita, cuyo agente etiológico es el ácaro Sarcoptes scabiei var. hominis, del grupo Arachnida, del orden Astigmata. Los factores más importantes en la transmisión son la pobreza, hacinamiento y la promiscuidad sexual. Se le asocia a estratos socioeconómicos bajos en los que además es frecuente observar higiene y nutrición deficientes. SALUD PUBLICA: Se transmite principalmente por contacto directo, estrecho, sin descartar la evidencia de transmisión por fomites (por ejemplo, ropa de cama o personal en infestaciones severas) y contacto sexual. Debe tenerse en cuenta el contacto frecuente en algunas instituciones (asilos, escuelas, cuarteles, hospitales). Los perros y gatos y otros animales también padecen la parasitosis. Los animales de compañía con S. scabiei var canis pueden ser responsables de infestación y sensibilización, causando una dermatitis papular limitada, aunque generalmente existe especificidad de especie. PATOGENIA: El artrópodo adulto mide 1 - 3 mm de longitud, tiene un cuerpo aplanado, ovalado y 4 pares de patas. La hembra fertilizada se instala en la superficie de la piel y en un lapso de tiempo menor a una hora excava un túnel en el estrato córneo y granuloso. Puede desplazarse unos 2 - 5 cm/min sobre la piel cálida. El promedio de vida del artrópodo es de unos 25 - 30 días, durante los cuales el túnel excavado puede extenderse unos milímetros o hasta centímetros. El ácaro deposita durante su avance 2- 5 huevos/día y pequeñas pelotitas de excremento, oscuras y ovales, irritantes. La respuesta inflamatoria y los productos del ectoparásito son responsables, en gran medida, del prurito. Las larvas liberadas alcanzan la madurez unos 15 días después de la oviposición; los nuevos adultos copulan y el ciclo se repite; al cabo de varias semanas la infestación todavía es baja y los signos y síntomas leves. Se requieren unos 15 - 20 ácaros adultos y su diseminación para provocar el intenso prurito. DIAGNOSTICO: Los síntomas iniciales son leves y habitualmente se atribuyen a la picadura de algún insecto. El prurito es un síntoma cardinal, con agravamiento nocturno (reacción de hipersensibilidad a saliva, huevos, materia fecal de los ácaros). El rascado, que inicialmente alivia al paciente porque destruye los túneles y algunos parásitos se erige como una forma de diseminación de los mismos (y elimina las lesiones primarias, constituidas por túneles y vesículas escasas y puntiformes, con ácaros, y pápulas pequeñas). Los túneles son delgados, elevados, curvados o en forma de S y miden 2 - 20 mm de longitud. Su localización más frecuente se encuentra en pliegues interdigitales, muñecas, en los bordes de manos y pies, superficies extensoras de codos y rodillas, pene, escroto, pliegues mamarios e incluso pezones, glúteos, axilas, cintura y pliegues poplíteos (zonas incluidas por las "líneas de Hebra", divisiones imaginarias a nivel de hombros y rodillas). En niños pequeños también se ubican con frecuencia en palmas de las manos, plantas de los pies, cabeza y cuello, con vesículas y pústulas. Además pueden presentar nódulos en la zona del pañal y en la axilar. En adultos también es posible identificar nódulos escabióticos. Las lesiones secundarias dominan el cuadro clínico y son ocasionadas por el rascado, infección bacteriana secundaria y la automedicación: pústulas, excoriaciones, eccema, costras. Se desarrollan nódulos en codos, axilas, pene, escroto (masas firmes, oscuras, que persisten durante meses aún en ausencia de infestación activa). Datos de eczema inflamatorio se atribuyen a la automedicación. Diagnóstico epidemiológico: Cuadros similares en el ambiente familiar o comunal, en instituciones. Localización de lesiones y morfología de las lesiones. Biopsia de piel. Infiltrado inflamatorio compuesto por eosinófilos, linfocitos, histiocitos. Pueden llegar a observarse larvas, ninfas, hembras. Se ha reportado el empleo de PCR en pacientes con probable escabiasis y exámenes dermatológico y microscópico negativos. TRATAMIENTO: Debe ser aplicado a todos los miembros de la familia y contactos cercanos. Incluye aseo personal y de la ropa. Se sugiere lavar la ropa en agua caliente y guardar en bolsas de plástico aquéllo que no pueda lavarse durante 2 semanas. Antibióticos por vía sistémica en el tratamiento de impétigo bacteriano secundario y anthistamínicos para el control del prurito. La dosis única de ivermectina es efectiva. En casos severos se aconsejan hasta tres administraciones, c/1 - 2 semanas. Los agentes tópicos incluyen: Permetrina en crema o emulsión al 5% (Scabisan), benzoato de bencilo en emulsión al 25% para adultos (Hastilan), crotamitón en crema (Eurax), muy eficaz ante el prurito. El lindano ha sido prohibido en varios países. La FDA reporta que el 70% de las reacciones adversas por lindano son de origen neurológico, e incluyen ataxia, desorientación, tremores, convulsiones y muerte. La exposición crónica deriva en efectos sistémicos graves y/o mortales. En pacientes con escabiasis noruega se utiliza ivermectina y tratamiento tópico simultáneamente, anthistamínicos y emolientes. El tratamiento es difícil, con recaídas. Debe monitorearse a los pacientes. El prurito puede persistir en las 2 - 4 semanas posteriores, lo cual puede significar: tratamiento exitoso,

CURSO ENARM CMN SIGLO XXI TEL: 36246001

Pharmed Solutions Institute

PÁGINA 396

MANUAL DE TRABAJO DEL CURSO ENARM CMN SIGLO XXI recaída, nueva infección. SARCOPTOSIS COSTROSA O NORUEGA: Se presenta en sujetos con inmunodepresión (SIDA, virus linfotrópico o leucemia), ancianos institucionalizados, en pacientes con trastornos neurológicos, principalmente síndrome de Down, demencia, trastornos nutricionales y enfermedades infecciosas. Las lesiones, hiperqueratósicas, son extensas, semejantes a las que se presentan en la psoriasis, con escaso prurito en la mayor parte de los casos. Hay formación de costras gruesas que involucran manos y pies, escamas grisáceas y costras en tronco y extremidades, descamación en la zona facial, lesiones semejantes a verrugas en dedos y sitios de trauma, y caída profusa de cabello. La observación de escamas aclaradas con hidróxido de potasio permite apreciar abundantes ácaros en todas las fases de crecimiento. Un cuadro que se presenta con cierta frecuencia, es el de "las personas limpias", quienes presentan lesiones escasas, en pliegues. Un buen número de pacientes presentan títulos elevados de IgE, eosinofilia, activación de mastocitos y una reacción del tipo de hipersensibilidad inmediata. Ante una reinfestación los síntomas aparecen con mayor rapidez. COMPLICACIONES: impétigo secundario, hiperinfección. Las infecciones bacterianas secundarias se relacionan con frecuencia con Streptococcus del grupo A y Sytaphylococcus aureus, por lo que se estima que la ectoparasitosis es un factor de riesgo importante en zonas endémicas en el desarrollo de glomerulonefritis, fiebre reumática, celulitis y otras infecciones invasivas. CASO CLINICO Paciente de sexo masculino, de 15 meses de edad, previamente sano. Consulta a su pediatra de cabecera por presentar pápulas eritematosas en muñeca izquierda. Algunas de ellas se tornaron costrosas y, en pocos días, se sumaron pápulas, placas y pequeños nódulos eritematosos con escamas en axila derecha, región supraumbilical y axila izquierda. Se encontraba en buen estado general y afebril. PREGUNTA Cual de las siguientes medidas es conveniente prescribir como final de tratamiento mas adecuado? RESPUESTA a.- Cefalexina por vía oral b.- Permetrina al 5% (crema fluida) c.- Hidroxicina 2 mg/kg/día 3 veces/día. d.- Aplicación diaria de mometasona (crema) en los nódulos. PEDICULOSIS. CIENCIAS BASICAS: Es una ectoparasitosis ocasionada por los piojos, los cuales no reconocen barreras geográficas, socioeconómicas ni culturales. Es la parasitación del hombre y los animales por insectos del genero pediculus del cual existen 3 principales: el P. capitis (pediculosis de piel cabelluda), el P. vestimenti (pediculosis del vestido y el cuerpo) y el P. pubis o ladilla (en vello púbico). El piojo se alimenta succionando sangre; su saliva contienen sustancias vasodilatadoras y anticoagulantes. Los huevos del piojo, conocidos como liendres, están finamente adheridas a pelos individuales. SALUD PUBLICA: Es un problema cosmopolita. Su alta prevalencia ha sido relacionada, con la limitada disposición de agua y deficientes prácticas de aseo personal. Hay preferencia con el sexo femenino relacionado con el cabello largo, ya que estos facilitan las formas de transmisión. Aproximadamente 6-12 millones de casos/año se presentan entre niños de 3-12 años de edad en Estados Unidos. En México se reportó una prevalencia entre 18-33%, en algunas poblaciones. PEDICULOSIS DE PIEL CABELLUDA: El llamado piojo negro o gris es el causante y parasita sobre toda la piel cabelluda aunque puede observarse en la barba y el bigote. Es más frecuente en niños entre 5 y 15 años de edad. El piojo en realizad es gris, pero toma a menudo el color del cabello que parasita y pone sus huevecillos que originan larvas (liendres), las cuales se adhieren fuertemente al pelo y son difíciles de desprender. El piojo mide unos 3mm y su cuerpo es alargado, se sitúa sobre todo en la región occipital y sus movimientos originan intenso prurito, lo cual provoca no en pocas ocasiones un impétigo secundario en esta región que es a menudo el motivo de consulta, por ello ante un impétigo sobre todo en niño, en la región occipital debe pensarse en la posibilidad de pediculosis previa. Deben diferenciarse las liendres de simples escamas que en general se dejan atravesar por el pelo, mientras que la liendre está adherida a su costado y avanza con el pelo al crecer este. La hembra dura poco menos de 30 días, pero en ese tiempo es capaz de depositar entre 50-150 huevecillos. PEDICULOSIS DEL CUERPO Y DEL VESTIDO: En realidad los piojos llamados blancos se encuentran en la ropa de personas sucias, vagabundos, alcohólicos, que nunca se bañan, ni cambian de ropa y ahí mismo el piojo deposita sus huevos que pueden observarse junto con los adultos en las costuras de la ropa. Las lesiones que producen son paulas, costras hemáticas, manchas hipercromicas en abdomen, nalgas, muslos, con intenso prurito. El impétigo y la eczematización son complicaciones habituales. El piojo blanco transmite el tifo exantemico y otras ricketsias. PEDICULOSIS DEL PUBIS: El P. pubis vulgarmente llamado ladilla, es más corto y ancho que los anteriores y en sus extremidades presenta unos ganchos que le dan aspecto de cangrejo y que lo adhieren fuertemente al pelo del pubis, periné, pliegue intergluteo y en ocasiones puede subir hasta la ceja, pestañas y vello axilar. El parasito es sedentario, se mueve poco, se alimenta continuamente y deposita sus deyecciones en el sitio y en la ropa interior por lo que el paciente cuando llega habitualmente al consultorio ya tienen el diagnostico, porque ha sentido prurito en la región del pubis y hasta ha logrado visualizar y extraer el piojo. Al examinar la región se observan los parásitos y las liendres y en la trusa o pantaleta se puede observar un fino puntillo producido por el depósito de las deyecciones del piojo (signo de la trusa) dato muy seguro para el diagnóstico. En la piel se pueden ver costras hemáticas y manchas hemorrágicas de color azulado de medio a un cm, llamadas manchas cerúleas. La hembra dura unos 30 días y llega a depositar en ese tiempo unos 30huevecillos que se transformaran en adultos. La transmisión es en la mayoría de las veces por transmisión sexual. TRATAMIENTO: Sera necesario destruir los parásitos adultos sin olvidar las liendres que no son fácilmente desprendibles del pelo, de otro modo en pocos días, estas darán origen a nuevos adultos. La permetrina al 1% en loción o crema, es el tratamiento de elección para pediculosis, debido a su eficacia y ausencia de toxicidad, esta actúa como una neurotoxina, bloquea los canales de sodio ocasionando parálisis del sistema nervioso y musculoesqueletico e impidiendo la respiración del piojo. La piretrina mas el piperonyl es un insecticida de origen vegetal extraído del crisantemo, el cual presenta un efecto ovicida del 70% y pediculicida del 97%. Se debe repetir 7 días para asegurar su eficacia. Los adultos pueden destruirse con DDT, benzoato de bencilo al 25%, gamexano o lo más útil y menos irritante: vaselina con xilol que los ahoga rápidamente. Se aplica tal mezcla en la cabeza o en el pubis y se deja toda la noche con baño al día siguiente. Para destruir las

CURSO ENARM CMN SIGLO XXI TEL: 36246001

Pharmed Solutions Institute

PÁGINA 397

MANUAL DE TRABAJO DEL CURSO ENARM CMN SIGLO XXI liendres es necesario desprenderlas mediante la aplicación con fomentos de ácido acético o simplemente vinagre blanco en agua, que disuelve la sustancia que las adhiere al pelo. Al día siguiente baño y peinado con el peine de dientes apretados especial para piojos. CASO CLINICO Niña de 2 años que tras una extensa pediculosis en la cabeza tratada con 2 ciclos de permetrina al 1% en loción acude por parasitación en las pestañas. La exploración mediante lámpara de hendidura pone de manifiesto la presencia de algunas liendres y de un piojo maduro (Pediculus humanus capitis) adheridos a las pestañas. Se realiza retirada directa de gran parte de los parásitos y se inicia tratamiento con vaselina en pomada 3 veces al día durante una semana. No vuelve a presentar recurrencias tras el tratamiento. PREGUNTA Cual es la conducta a seguir. RESPUESTA a.- Permentina 1%. b.- Piretroides. c.- Permetrina 5 %. d.- Malathion 0.5 %. TIÑAS. CIENCIAS BASICAS: Bajo el término dermatofitosis se engloban una serie de dermatosis producidas por un grupo determinado de hongos que se conocen como dermatofitos y que se caracterizan por parasitar, y digerir, las estructuras córneas (piel, uñas y pelo). También se conocen como micosis superficiales o tiñas y comprenden seis grupos, en función del área corporal afecta. Generos: Trichopyton, Microsporum, Epidermophyton. SALUD PUBLICA: T. rubrum 36-52%, M. canis 14-24%, T. tonsurans 15-18%. El cpontagio es persona a persona, por animales, artículos contaminados a través del suelo y albercas. TIÑA DE LA CABEZA: La más frecuente en niños. Afecta a los cabellos de cuero cabelludo que se rompen a nivel del ostium folicular lo que determina zonas de alopecia (tiñas tonsurantes). La afectación puede realizarse implicando a todos los pelos de una zona determinada (tiñas tonsurantes microspóricas) o a unos pelos sí y otros no (tiñas tonsurantes tricofíticas). Estas formas clínicas afectan predominantemente a niños, curan habitualmente en la pubertad y no dejan lesiones residuales. La tiña fávica o favus se caracteriza por estar producida por T. schönleini, y producir una inflamación folicular profunda que desemboca en una alopecia cicatricial. En contra de lo que ocurre en las formas tonsurantes el favus no cura con la pubertad. Es poco frecuente en países desarrollados pero la emigración determina la aparición de casos en estas zonas. Existen procesos inflamatorios en forma de foliculitis supurativa, no exclusiva de cuero cabelludo, conocidas como Querion de Celso, que están producidas por hongos zoofílicos (T. rubrum en 75%) y que presentan exudación abundante a nivel folicular especialmente cuando se presiona la placa (signo de la espumadera). Dependiendo de la intensidad del componente inflamatorio podremos encontrar, o no, la existencia de alopecia cicatricial. El tratamiento de primera elección es la terbinafina. Niños 10mg/Kg/dia por 6 semanas. Otra opción es itraconazol VO 5mg/día por 4 semanas. TIÑA DEL CUERPO: Cualquier zona corporal no incluida en esta clasificación puede incluirse en este grupo. Su clínica es muy evocadora en forma de lesiones papulosas que tienden a crecer de forma excéntrica determinando “círculos” en los que existe una zona central sana o discretamente escamosa, y un borde inflamatorio, activo, papuloso o pustuloso. Al igual que en cuero cabelludo pueden existir formas inflamatorias cuando el agente etiológico corresponde a un hongo de origen animal. Se puede usar terbinafina tópica, cada 24 hrs por 4 semanas. Alkternativa, miconazol. TIÑA CRURAL: Se caracteriza por la aparición de placas anulares, semejantes a las descritas en la tiña corporal, pero que tienden a localizarse a nivel inguinal. Es excepcional en niños pero sí suele afectar a jóvenes que practican deporte de forma habitual, y no es rara su asociación a tiña de los pies. Su posible transmisión por contacto sexual es siempre necesario tenerla en cuenta. TIÑA DE LAS MANOS: Es un proceso poco frecuente, y excepcional en la edad pediátrica, que casi siempre aparece en mujeres con trastornos circulatorios, diabetes o en profesiones que maceran sus manos bien por el uso continuado de guantes o por precisar mantenerlas en agua. La forma clínica más frecuente es la aparición de descamación y maceración interdigital, no obstante pueden aparecer cuadros de descamación palmar. Sin duda alguna las levaduras son más frecuentes en las micosis de manos que los dermatofitos. Tratamiento con terbinafina tópica. 2 semanas. TIÑA DE LOS PIES: Rara en la etapa infantil pero frecuente en la pubertad, y en ocasiones asociada a tiña crural. Es debida a la parasitación por dermatofitos en el pie pudiendo adoptar morfologías variables. Forma interdigital (pliegues de 4to y 5to dedo): incluida dentro del cuadro genérico de “pie de atleta” se caracteriza por la aparición de descamación y fisuración a nivel de los espacios interdigitales. El proceso puede ser poco llamativo cuando la infección es exclusiva de dermatofitos (Tinea pedis simplex) o ser muy inflamatoria o exudativa cuando existe infección combinada de dermatofitos y bacterias (Tinea pedís complex). Examen microscópico directo con hidróxido de potasio 20%; observamos filamento slargos delgados o gruesos, artroconidias, blastoconidias, onicomicosis candida. Tratamiento con ternbinafina tópica por 2 semanas. TIÑA DE LAS UÑAS: Es un proceso habitual en las consultas de dermatología pediátrica. El patrón diagnóstico es la presencia de una hiperqueratosis subungueal que determina un levantamiento de la uña del lecho (onicolisis). Generalmente la lesión comienza en el pliegue subungueal distal (onicomicosis subungueal distal), aunque puede hacerlo en el proximal (onicomicosis subungueal proximal) o incluso acabar afectando a toda la uña (onicomicosis distrófica total). En contadas ocasiones se produce una parasitación exclusivamente de la lámina ungueal que presenta una coloración blanquecina (Onicomicosis blanca superficial). CASO CLINICO Niña de 7 años, que consultó de urgencias por aparición progresiva en los últimos 10 días de una placa anular eritemato-descamativa muy pruriginosa, de 3 x 3cm de diámetro y crecimiento centrífugo, con borde levemente sobreelevado. Clínicamente planteaba el diagnóstico diferencial entre una tinea corporis y un eczema numular. Para confirmar el diagnóstico se realizó un examen directo de escamas cutáneas, obtenidas después de raspar el borde de la lesión con un bisturí e incubadas con 0,5ml de KOH al 30% sobre una lámina portaobjetos. PREGUNTA

CURSO ENARM CMN SIGLO XXI TEL: 36246001

Pharmed Solutions Institute

PÁGINA 398

MANUAL DE TRABAJO DEL CURSO ENARM CMN SIGLO XXI Cuales son los factores de riesgo más importante para sospechar de este padecimiento. RESPUESTA a.- Deportes y recreación en equipo. b.- Exposición a lugares húmedos sin proteción c.- Compartir objetos personales. d.- Uso de ropa sintetica. PREGUNTA Cual de los siguientes factores de riesgo es el más frecuente que están relacionados con las tiñas. RESPUESTA a.- Uso de corticoides. b.- Inmunocompromiso. c.- Queratodermia. d.- Ictiositocis hereditaria. PREGUNTA Cual es la conducta terapéutica más adecuada. RESPUESTA a.- Terbinafina 250 mg por 12 semanas. b.- Itraconazol 10 mg/Kg/dia c.- Fluconazol 150 mg VO por 24 semanas. d.- Miconazol crema al 2 % PREGUNTA Al revalorar la paciente posteriormente la paciente continua con el cuadro clínico con complicaciones, cual de las siguientes no es indicativo de envio a segundo nivel. RESPUESTA a.- Diseminacion dentro de las 4 primeras semanas. b.- Presencia de onicomicosis comorbida. c.- Alteración de enzimas hepáticas. d.- Inmunosupresion de origen idiopático. PTIRIASIS VERSICOLOR. CIENCIAS BASICAS: Es una infección micótica del estrato córneo de la piel, es la micosis más superficial que se conoce, causada por Malassezia furfur (ptiriosporum furfur), caracterizada por lesiones discrómicas, que pueden manifestarse como manchas hipercrómicas o hipocrómicas irregulares y en ciertos casos, de manera vitiligoide, variedad conocida como acromiante, todas con descamación fina. Además del clima, se han señalado otros factores predisponentes como aplicación local de corticoesteriodes, desnutrición, recambio lento de la epidermis y predisposición genética. SALUD PÚBLICA: La infección se presenta a partir de la adolescencia y es raro encontrarla en personas de edad avanzada. Esta micosis es más frecuente en personas que habitan lugares de clima cálido y húmedo, tales como las regiones costeras tropicales, en donde la frecuencia puede llegar a ser hasta del 50%. No existe predominio de género. PATOGENIA: El agente etiológico es M. globosa que puede encontrarse como especie única o asociada con a otras especies, entre ellas M. sympodialis, M. slooffiae. DIAGNOSTICO: Las lesiones generalmente son asintomáticas, se inician como discretas manchas eritematosas cubiertas con una escama muy delgada (como salvado), que pronto toman dos aspectos hipocromiantes o hipercromiantes. En México predomina la primera, son manchas un poco más claras que la piel raras veces acromicas, lenticulraes confluentes, bien delimitadas de bordes no activos como deshilachados, con esa fina escama y prácticamente sin prurito. Se localiza con mayor frecuencia en el tronco, cuello, cara, piel cabelluda y los brazos, aunque se pueden observar en otras regiones corporales de acuerdo a los factores predisponentes de cada paciente. Técnicas para la toma de muestra: raspado de la lesión, de preferencia de la periferia. Se pueden utilizar dos portaobjetos estériles, uno para raspar y el otro para recibir las escamas o bien se puede emplear una caja de Petri estéril. Producto biológico: escamas. Observación microscópica: hidróxido de potasio del 15-30%, azul de algodón, tinta o tinta azul de marca Parker. En escamas de lesiones de pitiriasis versicolor la morfología de las estructuras parasitarias es diagnóstica de esta enfermedad. Las levaduras son esféricas de 2-8 µm de diámetro, agrupadas, asociadas con hifas de 10-25 µm de largo y 2-5 µm de ancho, las hifas pueden estar alineadas o ramificadas (conocida imagen de spagueti con albóndigas). El cultivo no es un procedimiento que se practique rutinariamente en el laboratorio clínico, sin embargo se pueden obtener los cultivos a partir del producto biológico. Cuando se toma una porción de la colonia, es difícil preparar una suspensión o estriar sobre el agar, ya que las células permanecen juntas formando pequeños grupos (o racimos). TRATAMIENTO: Ya que la mayoría de patologías causadas o asociadas a Malassezia son superficiales, el tratamiento tópico es lo más recomendado. Habitualmente se recomiendan lociones, cremas o jabones con ácido acetil salicílico al 5% o azufre al 1-3%; ungüento de Whitfield, hiposulfito de sodio al 20% en solución acuosa, tolnaftato, tolciclato, crema de piroxolamina al 1%; champú de disulfuro de selenio al 2.5%, con efecto antimicótico, bactericida y anfi-inflamatorio. Las lociones y cremas deben aplicarse diariamente durante 3 a 4 semanas. El champú se aplica diariamente, dejando actuar algunos minutos antes de enjuagar el pelo, durante 2 a 3 semanas. Para casos graves, está indicado el ketoconazol por vía oral; uno de los esquemas es 400 mg en una sola dosis; o bien 200 mg diarios por 10 a 30 días dependiendo de la gravedad del caso. El itraconazol también ha dado resultados satisfactorios: 100 a 200 mg/día durante 3 a 15 días, dependiendo de la

CURSO ENARM CMN SIGLO XXI TEL: 36246001

Pharmed Solutions Institute

PÁGINA 399

MANUAL DE TRABAJO DEL CURSO ENARM CMN SIGLO XXI extensión de las lesiones. En el caso de la pitiriasis versicolor, las manchas pueden persistir varios meses después del tratamiento, por lo que el paciente debe ser informado. CASO CLINICO Se trata de paciente masculino de 18 meses de edad el cual ha presentado diversos cuadros de dermatitis aatopica con importante rascada, acude a consulta de control donde observa placas blanquecinas peribucales sin comprometer mucosa. PREGUNTA Considerando su diagnostico y estadio del padecimiento cual es conducta terapéutica menos adecuada? RESPUESTA a.- Evitar sol e indica protectores solares. b.- Baños cortos y con agua templada. c.- Indica corticoide en crema. d.- Indica tacrolimus. CANDIDIASIS. CIENCIAS BASICAS: La candidosis o candidiasis es una micosis causada por diversas especies de levaduras del género Candida. Cualquier tejido puede ser afectado por lo que se presentan diversos cuadros clínicos, cada uno de ellos asociado directamente al estado inmunológico del paciente. Las candidosis de mucosas y piel son las más frecuentes, mientras que las sistémicas son de evolución aguda o crónica y generalmente severas. SALUD PUBLICA: La distribución geográfica de esta micosis es universal y más de 70 % de ellas son producidas por C. albicans observándose un porcentaje mayor por el serotipo B. Los casos de candidiasis sistémica están relacionados a pacientes con severas deficiencias en su sistema inmune. C. krusei y C. glabrata son habitualmente resistentes a los compuestos azólicos y su hallazgo como agentes infecciosos involucrados en enfermedades sistémicas intrahospitalarias ha aumentado en los últimos años. PATOGENIA: La C. albicans se ha considerado el ejemplo más clásico de lo que es un parasito oportunista, vive habitualmente en forma saprofita en las mucosas orales, nasal, vaginal, tracto gastroinetstinal, en cambio no es habitual encontrarla en piel. El niño al nacer puede recibir este parasito al pasar por el tracto vaginal de la madre. Cuando se convierte de saprofita a patógena, puede producir desde una simple algondoncillo hasta graves endocarditis o septicemia. Factores predisponentes: 1. Fisiológicos que suelen transformar el pH de las mucosas como la infancia y el embarazo. 2. Maceración, humedad y traumatismos, (pliegues interdigitales, submamarios, uñas y rebordes ungueales). 3. Dermatosis inflamatorias previas (como dermatitis del pañal que se complica con candida). 4. Mal estado de la dentadura y prótesis. 5. Enfermedades metabólicas (DM y obesidad). 6. Enfermedades inmunosupresoras (leucemias, Enf. De Hodking, SIDA). 7. Medicamentos que alteran la flora bacteriana (corticoides, citotóxica). DIAGNOSTICO: La manifestación más conocida es el algodoncillo que aparece en la boca de RN por su bajo pH. Las lesiones son placas cremosas, blanquecinas, como residuo de leche, que pueden afectar la mucosa de los carrillos, la lengua, el paladar, encías e incluso invadir toda la boca hasta la tráquea o salirse de la boca y producir fisuras cubiertas de material blanquecino en las comisuras labiales. Estas lesiones son dolorosas e impiden la alimentación al niño. La Candida albicans no es habitual en la piel, pero puede producir enfermedad cuando aumenta la susceptibilidad del in dividuo. Las lesiones a parecen a nivel de los pliegues: interdigitales en manos y pies, inguinales y submamarios, axilas, intergluteos y periné. Se trata de fisuras y erosiones eritematosas, maceración, vesícula y pústulas y algunas costras y escamas. En los pies semejan lesiones de tiña, frecuentemente producen mal olor y son pruriginosas. En los niños pequeños contribuyen a formar la llamada dermatitis por pañal que se observa sobre todo por la orina, la aplicación de pomadas, la maceración, todo lo cual favorece el desarrollo de Candida. El niño presenta entonces en regiones inguinales, glúteas y genitales, extensas zonas eritematosas con vesículas y pústulas, costras y escamas con intenso ardor y prurito. La perionixis y la afección de la uña se ha observado sobre todo en personas que mantienen mucho tiempo las manos dentro del agua, como empacadores de frutas, pescados y mariscos. El reborde ungueal se observa inflamado, eritematoso, desprendido de la uña y esta se afecta principiando de la matriz al borde libre hacia adentro. Poco aparecen estrías en las uñas, la cual se vuelve amarillenta y opaca y se empieza a despulir. La lesiones son habitualmente superficiales, es poco frecuente que al igual que los dermatofitos se introduzcan más allá de la capa cornea, dependiendo del estado inmunológico del huésped originándose granulomas tricofíticos o candidósicos. El simple hallazgo de Candida albicans no significa de ninguna manera su intervención patógena, es necesario encontrarla en grandes cantidades y correlacionar su hallazgo con lesiones clínicas. . Es fácil buscarla de manera directa, colocando material recolectado en un portaobjetos con solución de potasa al 20% y se podrá encontrara las clásicas formas levaduriformes, que a veces producen pseudofulamentos. El cultivo en medio Saboraud produce colonias típicas de aspecto cremoso. TRATAMIENTO: Corregir enfermedad de base. En boca usamos buches con agua de bicarbonato de sodio. Localmente puede usarse violeta de genciana al 1% que mancha mucho o la nistatina, antibiótico que solo es activo para levaduras de Candida. Los imidazoles tanto tópicos como sistémicos son altamente efectivos en candidiosis: miconazol, ketoconazol, clotrimazol. CASO CLINICO Paciente femenino de 10 años de edad en consulta por presentar engrosamiento y cambio de coloración en la uña del pulgar derecho, al examen físico, en cuero cabelludo, se observan tras placas de alopécicas, con cicatrización en la periferia y escasas costras serosanguinolentas en el centro, dos de estas placas se localizan alrededor del apice del cráneo y la tercera en región parietal derecha. En cavidad oral y labios se observan placas blanquecinas que cubren el centro y los bordes de la lengua, asi como ambas comisuras labiales; al remover estas placas se observa eritema y fisuras de mucosas. PREGUNTA Cual es la conducta a seguir mas adecuada? RESPUESTA a.- Itraconazol. b.- Miconazol.

CURSO ENARM CMN SIGLO XXI TEL: 36246001

Pharmed Solutions Institute

PÁGINA 400

MANUAL DE TRABAJO DEL CURSO ENARM CMN SIGLO XXI c.- Ketoconazol. d.- Clotrimazol. MICETOMA. CIENCIAS BASICAS: Es una infección crónica de la piel y de los tejidos subyacentes con tendencia a afectar los huesos. Se caracteriza por un aumento de volumen relativamente indoloro y fístulas a través de las cuales se elimina pus y granos constituidos por filamentos. Los agentes causales son de origen exógeno y pueden ser hongos (eumicetoma) o actinomicetales (actinomicetoma). Agentes etiológicos: a) Bacterias (ACTINOMICETALES); Nocardia brasiliensis 86.0 %. Actinomadura madurae 10.2 %. Streptomyces somaliensis 1.3%. N. asteroides, A. pelletieri, N. otitidiscaviarus. b) Hongos (EUMICETOMA). Los hongos dematiaceos forman granos negros visibles a simple vista, mientras que los de filamento hialino forman granos blancos o blanco-amarillentos. Hongos hialinos: Acremonium falciforme, A. recifei, A. kiliense, Aspergillus flavus. Hongos dematiaceos: Curvularia geniculata, C. lunata, Exophiala jeanselmei, Leptosphaeria senegalensis. PATOGENIA: El micetoma se adquiere por inoculación traumática de agentes etiológicos a través de la piel. En México existe gran número de cactáceas, las espinas son un mecanismo de infección, que probablemente tengan importancia en la transmisión de la enfermedad. Una vez que la bacteria o el hongo se localizan en los tejidos y sobrevive, se presenta una inflamación aguda a nivel local con llegada de PMN y activación de macrófagos, aunque la mayoría de bacterias son fagocitadas y destruidas, algunas logran sobrevivir hasta 16 días y pueden reproducirse. Los agentes no destruidos proliferan y forman colonias en el interior del tejido conocidas como "granos"; el desprendimiento de los pseudofilamentos de la periferia del grano ocasiona una inflamación continua con formación de nuevas estructuras parasitarias y colecciones de pus, que al fusionarse forman los trayectos fistulosos generalmente limitados por tejido fibroso; cuando las fístulas alcanzan la superficie, forman un nódulo que posteriormente se reblandece y abre liberando pus conteniendo colonias parasitarias del agente. El tejido conjuntivo formado alrededor de los microabscesos y de las fístulas produce una retracción que ocasiona el aspecto deprimido de las fístulas antiguas. La presencia de micro abscesos, fístulas, inflamación, edema y fibrosis provoca externamente la deformidad y dureza del área afectada e internamente obstrucción de la circulación por compresión mecánica y por arteritis. Estos cambios histológicos influyen en la pobre respuesta terapéutica en los pacientes con evolución prolongada. DIAGNOSTICO: Por la naturaleza saprofita de los agentes causales de micetoma que posibilita la infección principalmente por traumatismo en pie o pierna, todas las casuística de frecuencia muestran un franco predominio de afección a extremidades inferiores, pero de acuerdo con las costumbres de trabajo o de vestido y a las condiciones socioeconómicas, los porcentajes de frecuencia presentados en cada área topográfica por diferentes autores, pueden tener alguna variación. Examen directo; La observación del material purulento que drena a través de las fístulas permite identificar los granos macroscópicos en los eumicetomas y, al examen microscópico, en la mayoría de los actinomicetomas. Cultivo; Cuando es posible, los granos de actinomicetales se mezclan y se lavan con solución salina isotónica (SSI), se centrifugan a 3000 rpm y posteriormente el sedimento se siembra, el desarrollo de la mayoría de los agentes se presenta después de dos o tres semanas de incubación a 25°C. Otro medio adecuado para su desarrollo es el de Lowenstein-Jensen que se incuba en las mismas condiciones. Las colonias tienen bordes bien definidos e irregulares, aspecto céreo o membranoso, generalmente de superficie plegada. Los cultivos de Nocardia spp. Son de color blanco amarillento o anaranjado las colonias de A. madurae son blanco amarillentas, A. pelletieri forma colonias rojas y S. somaliensis tienen color oscuro, algunas casi negras. La identificación precisa de los agentes se realiza por medio de pruebas bioquímicas. Los granos de eumicetoma se siembran y se incuban a 25°C. La velocidad de crecimiento es variable y depende de cada agente. Así. Fusarium spp, Aspergillus spp. o Scedosporium apiospermum se desarrollan en menos de ocho días; mientras que Madurella spp, Leptosphaeria sp y Pyrenochaeta romeroi son de crecimiento lento, es decir más de tres semanas. Cuando las biopsias se toman de sitios con gran actividad parasitaria, los granos de todos los agentes se visualizan con relativa facilidad con la tinción de hematoxilina eosina (H-E). Los estudios radiológicos no permiten establecer el diagnóstico preciso de esta patología, sin embargo son de suma importancia para determinar el grado de afección ósea, permiten evaluar la respuesta terapéutica y emitir un pronóstico. Los cambios observables en estudios radiológicos simples pueden ser en los casos iniciales únicamente periostitis, o bien, destrucciones osteolíticas extensas con formación de geodas, destrucción de superficies articulares y lisis en huesos del pie que ocasionan el aspecto de 'caries', datos conocidos desde las primeras descripciones del padecimiento y que se presentan en pacientes con varios años de evolución. Otro estudio radiológico necesario, principalmente en los pacientes con más de cinco años de evolución del micetoma, es la arteriografía del área afectada. El aumento de volumen y la fibrosis, condicionan que algunas zonas del área afectada tengan un aporte sanguíneo deficiente, de tal manera que este procedimiento es un valioso auxiliar para el pronóstico. TRATAMIENTO: Debido a la posibilidad de que los micetomas sean causados por hongos o por bacterias, antes de prescribir la terapéutica es indispensable conocer el tipo de agente causal (micótico o bacteriano); esto puede lograrse mediante a la observación de las características de los granos en el examen directo y en el estudio histológico. Actinomicetoma; La primera elección es la combinación de TMP/SMX 800 mg/160 mg c/12 hrs asociada a la administración de 100 mg de diaminodifenilsulfona (DDS), cada 24 horas. La mayoría de micetomas causados por actinomicetales responden a este tratamiento y la mejoría es evidente después de dos meses de tratamiento. Los pacientes deben ser valorados periódicamente debido a que estos medicamentos pueden ocasionar diversos efectos adversos, entre los que se encuentran reacciones medicamentosas graves a nivel cutáneo como es el síndrome de Stevens-Johnson, necrólisis epidérmica tóxica, aplasia medular, anemia, intolerancia gástrica, alteraciones hepáticas, etc. Como segunda elección se emplea la asociación de TMP/SMX a la dosis mencionada previamente en forma continua, y agregando amikacina, aminoglucósido que se administra por vía intramuscular en ciclos de 21 días de tratamiento a dosis de 7.5 mg/kg de peso cada 12 horas. Entonces, se suspende el aminoglucósido durante ocho días. Considerar ahora el posible daño renal o auditivo causado por la amikacina; por lo tanto, antes de iniciar el tratamiento y después de cada ciclo de amikacina, se debe valorar la función de esos órganos. La curación generalmente se alcanza con tres ciclos del aminoglucósido, pero el TMP/SMX debe continuarse durante 12 a 24 meses de acuerdo con la evolución clínica. Aunque el estudio de sensibilidad antibacteriana debería ser una práctica habitual en todas las cepas aisladas de casos de actinomicetoma, estos estudios generalmente no se realizan, dando como resultado que se utilicen tratamientos basados únicamente en la experiencia clínica. Estos esquemas de tratamiento (cuasiempíricos) generalmente mejoran el estado del paciente, pero en caso de no haber respuesta, retrasan la curación. Entre los antibióticos utilizados en estas condiciones tenemos: tetraciclina, amoxicilina, estreptomicina o los antifímicos como la isonizida. Desde los primeros estudios inmunológicos relacionados con actinomicetoma fue evidente que algunos pacientes presentan inmunosupresión inespecífica severa, y no responden al tratamiento médico aun cuando se les administren los antibióticos adecuados. En estos casos es necesario valorar la respuesta inmunológica y dar los antibióticos efectivos basados en un estudio de

CURSO ENARM CMN SIGLO XXI TEL: 36246001

Pharmed Solutions Institute

PÁGINA 401

MANUAL DE TRABAJO DEL CURSO ENARM CMN SIGLO XXI sensibilidad acompañados de un esquema terapéutico inmunomodulador, como puede ser la administración de antígeno bacteriano y levamisol con lo que se logró la curación en sólo dos meses de un paciente con actinomicetoma causado por N. brasiliensis localizado en el abdomen, después de siete años de evolución sin respuesta al tratamiento tradicional. Se ha reportado la utilidad de la oxigenoterapia hiperbárica en el tratamiento de un caso de micetoma sin afección ósea, causado por N. brasiliensis; el paciente presentó mejoría a los cinco días de tratamiento y las fístulas cicatrizaron un mes después del inicio de la terapia. Eumicetoma: Este tipo de micetoma representa un grave problema de tratamiento, ya que no existen drogas altamente específicas. Durante muchos años la medida terapéutica empleada con mayor frecuencia fue la amputación de la extremidad afectada. Posteriormente se hicieron ensayos con griseofulvina a dosis de 500 a 1000 mg por día sin obtener curación; con base en estudios de sensibilidad in vitro la anfotericina B fue utilizada en el tratamiento de micetomas causados por M. grisea y M. mycetomatis. En opinión de algunos investigadores, la fibrosis característica que rodea los granos de los agentes etiológicos, impide que se alcancen las concentraciones adecuadas para la curación y, en consecuencia, el tratamiento no es efectivo. El advenimiento de los compuestos azólicos proporcionó nuevas alternativas para el manejo de esta patología, se hicieron pruebas con ketoconazol solo o asociado a cirugía; esta última alternativa mejoró el pronóstico de manera substancial. En la última década, se ha utilizado el itraconazol a dosis de 300 a 400 mg por día, reportándose algunos casos de curación y otros con mejoría. Algunos autores como Smith y Kutbi, mencionan que en casos de eumicetoma de corta evolución la cirugía acompañada de tratamiento antimicótico es la única posibilidad de curación definitiva. CASO CLINICO Masculino de 8 años, originario de zona rural de guerrero, fue llevado a consulta por una dermatosis localizada en el tronco que se extendia al toras y a la cara anterior del hombro derecho. La dermatosis era de aspecto monomorfo y estaba constituida por algunas fistulas, numerosos nódulos eritematoso de 0.3 a 2 cm aproximadamente, telangiectasias y costras sanguíneas y meliceras que formaban una placa mal circunscrita, de 6 x 4 aproximadamente, crónica y asintomática. Se realizaron cultivo los cuales mostraron colonias de aspecto yesoso, rogosas y de color blanco. La prueba de la hidrolisis de caseína resulto positiva y la colonia se identifico como Norcardia brasiliensis. No se observo compromiso oseo a los rayos x. PREGUNTA Cual es el manejo mas adecuado para el caso? RESPUESTA a.- Anfotericina. b.- Trimetroprim-sulfametoxazol. c.- Itraconazol. d.- Clotrimazol. URTICARIA. CIENCIAS BASICAS: Es una reacción vascular de la piel, caracterizada por la presencia de habones y prurito intenso donde ocurren procesos inflamatorios importantes, hay una alteración del equilibrio de sustancias vasoactivas, causando vasodiltaacion y edema a nivel de la piel (urticaria), o del tejido subcutáneo (angioedema). En la mayoría de los pacientes, una liberación masiva de histamina es la causa del desequilibrio. El nombre de urticaria se deriva de la palabra ortiga, una planta cuyas hojas causan lesiones urticariales pasajeras. SALUD PUBLICA: La prevalencia de la urticaria-angioedema es de 10-20% de la población lo sufrirá en su vida. En la edad pediátrica, la urticaria aguda es la forma que más prevalece. CLASIFICACION: Urticaria Aguda < de 6 semanas de evolución. Urticaria Crónica > de 6 semanas de evolución. Urticaria Aguda Recurrente: Episodios repetidos de brotes agudos con períodos asintomáticos prolongados entre ellos. PATOGENIA: Alimentos: Huevo, leche, nueces maní, mariscos, pescados, fresas, kiwi, otros. Drogas: Penicilinas, ASA, AINES, vacunas, medios de contraste, productos sanguíneos. Insectos: Abejas, avispas, hormigas. Infecciones: Parásitos, bacterias, virus, hongos. Estímulos Físicos: Frio, calor, presión, ejercicios, agua, sol. Aeroalergenos, alérgenos de contacto: Polvo casero, pólen, esporas, epitelios de mamíferos. Enfermedades sistémicas LES, ARJ, enfermedad del suero, vasculitis, cáncer, endocrinopatías, Enf celíaca. Otros A. Hereditario, amiloidosis, deficiencia de C3b, estrés, idiopático. El evento central en la urticaria-angioedema es la liberación de histamina desde unos gránulos preformados en las células cebadas. Existen múltiples factores que pueden provocar la degranulacion de las células cebadas. En la urticaria aguda, el mecanismo principal es el de hipersensibilidad tipo I, IgE mediada, también conocida como la reacción alérgica clásica. En las primeras exposiciones que el paciente tuvo alérgeno se inició una respuesta Th2 mediada que resulto en la formación de abundantes cantidades de IgE específica por parte de las células plasmáticas específicas. Esta IgE es liberada a la circulación sistémica y se fija de preferencia a receptores de alta afinidad para IgE (FcR1), en la superficie de la célula cebada (CC). Un subsecuente contacto con el mismo alérgeno provoca directamente el puenteo de dos IgE especificas vecinas en la membrana de la CC, y esto es la señal para la degranulación de la CC. Así se liberan grandes cantidades de sustancias vasoactivas, especialmente histamina, pero también leucotrienos, factor derivado de plaquetas. Las respuestas principales de estos mediadores son: prurito, vasodilatación (eritema), aumento en la permeabilidad vascular, reflejo axonal. La exposición al alérgeno de la urticaria puede ser por cualquier vía, incluyendo la oral, transdermica e

CURSO ENARM CMN SIGLO XXI TEL: 36246001

Pharmed Solutions Institute

PÁGINA 402

MANUAL DE TRABAJO DEL CURSO ENARM CMN SIGLO XXI inhalatoria. URTICARIA AGUDA: En niños la urticaria aguda es más frecuente que la crónica. Es frecuentemente por una reacción alérgica mediada por IgE a infecciones agudas generalmente del TRS y drogas. Infecciones. Virus: Enterovirus, Parainfluenza 1,2,3, Virus respirat. Sincitial, Adenovirus, rinovirus, Influenza A y B, VEB, CMV, Herpes simple, Parvovirus B19, Hepatitis A, B, C, Mycoplasma Neumoniae. Parásitos: Oxiuro, Toxocara, Giardia lamblia. Bacterias: Estreptococo beta hemolítico, H. pylori. URTICARIA CRONICA: En la Urticaria crónica los factores físicos como presión y frío son la causa principal. Luego siguen infecciones: tracto urinario, tracto respiratorio y las idiopáticas. Existe un porcentaje de Ac antitiroideos antiperoxidasa positivos en niños con urticaria crónica y también hay mayor incidencia de enfermedad celíaca en estos niños. DIAGNOSTICO: La urticaria se identifica por lesiones típicas que tienen 3 caracteristicas básicas: consisten en una inflamación central eritematosa, levemente elevada sobre el nivel de la piel, casi siempre rodeada por una piel eritematosa (reflejo axonal). Las lesiones son en esencia evanescentes, con una duración de menos de 24 hrts, que al desaparecer dejan una piel intactas. Con frecuencia son intensamente pruriginosas. La urticaria puede tomar diferentes formas variando de lesiones puntiformes de 2 milimetros (point lesions), que cubren grandes áreas de la piel (típico aspecto de urticaria colinérgica), lesiones aisladas de 1-2 cm o hasta lesiones lesiones confluyentes de varios centímetros. Historia y examen físico. Pruebas para urticarias físicas, prueba de suero autólogo, examen simple de orina y seriado de heces, hematología completa y VSG, pruebas tiroideas y Ac antitiroideos. Otros: según cuadro clínico. TRATAMIENTO: 1. Medidas generales como evitar contacto con alérgeno, eliminar el alérgeno si es posible: aditivos, alimentos, medicamentos, frío, calor, picaduras. Implementar dieta de eliminación. 2. Antihistamínicos: Primera línea en el tratamiento de la urticaria. Actúan por inhibición competitiva de la histamina bloquean receptores H1. Control del prurito. Primera generación sedantes (Dexclorfeniramina 0.15-0.2 mg/día QID, Hidroxicina 2 mg/día QID, Ciproheptadina 0.25 mg/día BID, Difenidramina 5 mg/dia TID) y Segunda generación No sedantes (Loratadina: 1-2 años 1cc/dia; 2-5 años 5mg/dia, Cetirizina : 6m-1año 0,5 mg/dia; 2-5años 2.5 mg/dia, Fexofenadina: 6-12 años 120 mg/día, Ebastina : 6-12-años 5-10 mg/dia, Desloratadina: 2-5 años 1,25 mg/dia; 6-11 años 2,5mg, Levocetirizina: >2 años 5/mg dia). Los de segunda generación provocan liberación de aminas vasoactivas. Desloratadina: Tratamiento de rápida acción y bien tolerado y por largo tiempo. Usado en casos de Urticaria crónica idiopática. Levocetirizina: Enantiómetro o metabolito activo de certirizina. Receptor Antagonista H1. Rápido inicio de acción, pocos efectos colaterales. Indicado en rinitis alérgica y en UCI. Efecto antiinflamatorio importante que aumenta su efecto terapéutico en enfermedades alérgicas. Uso por tiempo prolongado desde el año de edad. Dosis: 5mg/dia. 3. Antileucotrienos: Segunda línea en urticaria que no responde a antihistamínicos. Previene o mejora los síntomas asociados a procesos mediados por leucotrienos C4, D4, y E4. Util en urticaria por medicamentos ASA y AINES. Montelukast 2-5 años 4 mg/dia; 6-14 años 5 mg/dia. Zafinlukast 5-11 años 10 mg/BID. 4. Corticoides: Esteroides Sistémicos ayudan a disminuir la inflamación. Solo deben usarse en casos extremos y de angioedema, por corto tiempo para evitar efectos colaterales. Urticaria por presión y urticaria vasculitis. Prednisona 1mg/Kg/dia por 57 dias. 5. Ciclosporina: En la tercera línea de tratamiento. Es un inmunosupresor. Controles de funcionalismo renal. Dosis: 3 mg/Kg de peso. 6. Omalizumab. Urticaria Aguda con síntomas sistémicos: Dificultad respiratoria-Evidencia angioedema, Adrenalina 1/1000 subcutánea a 0.01 mg/Kg, repetir cada 15 a 20 minutos, hidrocortisona 7-10 mg/Kg/dósis y luego instaurar ciclo corto, menor a 10 días, clorfeniramina: IV. Dieta de exclusión de alimentos disparadores (maní, piña, chocolate). Combinación de anti-H1 (1ra y 2da) o Combinación de anti-H1 y H2 (o anti-LT). CASO CLINICO Paciente de 4 meses de edad, sin antecedentes patológicos conocidos, traido a consulta por presentar papulas generalizadas pririginosas de 2 meses de evolución, al EF mostraba papulas eritematosas generalizadas en torax y miembros superiores e inferiores que respetaban palmas y plantas. Dichas papulas, al ser frotadas, tomaban aspecto de roncha rodeada por un halo eritematoso. Según refirió su madre sobre una de ellas se había formado previamente una ampolla, el paciente se encontraba afebril y en buen estado general. PREGUNTA Cual es la conducta terapéutica mas adecuada? RESPUESTA a.- Clorhidrato de hidroxicina. b.- Maleato de clorfeniramina. c.- Loratadina. d.- Cetirizina. PRURIGOS. CIENCIAS BASICAS: El prurigo infantil o también llamado urticaria papular es una reacción alérgica producida por la picadura de insectos. Esta afección netamente pediátrica no debe ser confundida con una alergia a los alimentos; se caracteriza por el prurito intenso de difícil control, con brotes frecuentes y sucesivos que por lo general se autolimitan en el tiempo. SALUD PUBLICA: Afecta en su mayoría a niños entre los dos y diez años, frecuentemente en los meses de primavera y verano. TIPOS: La palabra prurigo define un proceso patológico cuya lesión elemental es la pápula y el síntoma principal es el prurito. El espectro clínico abarca un rango que va desde las pápulas (prurigo papular), nódulos (prurigo nodular) entre otros tipos de prurigo podemos mencionar al actínico, el atópico o de Besnier, el de Sutton y el pigmentoso. DIAGNOSTICO: Se observa una erupción estacional y recurrente, con presencia de grupo de pápulas y vesículas pruriginosas de localización frecuente en antebrazos, abdomen, área lumbar, glúteos, muslos y piernas. La respuesta va a depender del tipo de paciente y la calidad del inóculo, pudiendo también observarse formas ampollares. PRURIGO SIMPLE POR INSECTOS O URTICARIA PAPULOSA: Es causada como su nombre lo indica por la picadura de algún insecto. No está relacionada por factores dietéticos. La naturaleza alérgica de la condición fue demostrada cuando los cambios histológicos producidos por picaduras de insecto a nivel experimental demostraron que eran idénticos a los de la urticaria papular. Afecta principalmente a niños de 1 a 7 años, siendo una de las causas más frecuentes en la dermatología pediátrica, afecta igual a ambos sexos y a cualquier etnia. Numerosos insectos causan prurigo, siendo los más frecuentes el Cemex lectularius (chinche) produciendo la Cimiasis; la pulga (Pulex irritans) que ocasiona la puliciasis y las picaduras por mosquitos (Culicidae). Los alérgenos presentes en la saliva del insecto inducen una sensibilización del paciente con formación de anticuerpos específicos. El tipo e intensidad de la reacción originada por la

CURSO ENARM CMN SIGLO XXI TEL: 36246001

Pharmed Solutions Institute

PÁGINA 403

MANUAL DE TRABAJO DEL CURSO ENARM CMN SIGLO XXI picadura dependerá de si el paciente ha estado ya expuesto al insecto y de la capacidad del huésped de responder al estímulo antigénico. Las lesiones tempranas de prurigo por insecto se deben a una respuesta de hipersensibilidad tipo I causada por la liberación de IgE. Posteriormente interviene un mecanismo de hipersensibilidad tipo IV dependiente de linfocitos T que produce las lesiones tardías. El prurigo por insecto afecta cualquier área corporal, se presentan vesículas en la fase inicial posteriormente aparecen pequeñas pápulas eritematosas, las lesiones son muy pruriginosas por los que se observan costras hemáticas por rascado. Se observan lesiones en diferentes estadios y evolucionan por brotes. PRURIGO NODULAR: Es una dermatosis crónica caracterizada por nódulos muy pruriginosos que aparecen principalmente en las superficies extensoras de extremidades inferiores, puede desarrollarse luego de una picadura por insecto ó de otras formas de inflamación localizada. La lesión por sí misma es muy pruriginosa, lo cual es el resultado de un círculo vicioso establecido por el rascado, trauma mecánico e infección, etc. Diversas investigaciones señalan que existe un número elevado nervios dérmicos y células de Merkel en estas lesiones, sugiriendo que este cuantitativo aumento puede estar directamente involucrado en la etiología. Se ha reportado que la terapia antituberculosa produce una aclaración parcial o completa de las lesiones. El prurigo nodular puede ocasionalmente ser el resultado del prurito que acompaña a algunas enfermedades sistémicas como los procesos obstructivos biliares, anemia, enfermedad renal crónica, policitemia vera, DM, parasitosis, erupciones por droga y en los niños el más común el Linfoma de Hodgkins. Las lesiones del prurigo nodular presentan un diámetro aproximado de 0,5cm a 3cm, la superficie de las lesiones puede ser queratósica ó deprimida en el centro. La lesión inicial es eritematosa y puede semejar a la urticaria, sin embargo todas las lesiones tienden a pigmentarse con el tiempo. El número de lesiones varía desde 2 a 200 de las cuales algunas mejoran espontáneamente. PRURIGO DE SUTTON: Es una erupción liquenoide de los codos en niños, pitiriasis de codos y rodillas. Esta entidad clínica es una dermatosis eccematosa papular pruriginosa, la cual está limitada a codos, pudiendo extenderse a nudillos, manos y tórax. Se asocia a una historia personal de atópia o familiar, así como a niveles elevados de IgE sérica. Se ha descrito en niños de 3 a 13 años, siendo la lesión inicial una pápula eritematosa de 1-2mm de diámetro. PRURIGO PIGMENTOSO: Es una dermatosis rara, ocurre en Japón, se describe el trauma físico o fricción de la ropa como gatillo. Se ha detectado que puede ocurrir en adolescentes, se caracteriza por pápulas eritematosas pigmentadas que confluyen formando un patrón reticulado, las lesiones se presentan en verano y más frecuentes en mujeres jóvenes. El rash es de distribución simétrica en tronco, área cervical, región lumbosacra, abdomen y cara. PRURIGO DEL ATOPICO O DE BESNIER: También llamado neurodermatitis, o eccema del lactante. Es la dermatosis más frecuente en población pediátrica. La prevalencia ha mostrado incremento en las últimas décadas, siendo del 18-20%. Es más frecuente en áreas urbanas de países industrializados, especialmente en inmigrantes provenientes de países con menor prevalencia. Es una variante poco común en la infancia, se puede observar en adolescentes. De acuerdo con la morfología, pueden definirse tres tipos de lesiones, sin ser el tiempo de evolución determinante para su definición: 1. Aguda. Caracterizada por pápulas y vesículas muy pruriginosas, sobre un área de piel eritematosa, asociada a escoriaciones, erosiones, exudado seroso y costras melicéricas (eccema). 2. Subaguda. Caracterizada por eritema, pápulas, descamación y escoriaciones. 3. Crónica. Placas de piel engrosada, con liquenificación y pápulas fibróticas. Existen además tres fases cronológicas, con características específicas: 1. Lactante (2 semanas a 2 años). Afecta la cara, predominando en mejillas y respetando el triángulo central. Puede extenderse a piel cabelluda, pliegues retroauriculares y de flexión, tronco y nalgas; a veces puede generalizarse. Predominan las lesiones de dermatitis aguda por lo que se le ha denominado “eccema del lactante”. Se puede asociar con dermatitis seborreica en un 17%. Aparece por brotes, siendo frecuente la desaparición de a los 2 años de edad. 2. Escolar o infantil (3 a 14 años). Afecta pliegues de flexión (antecubitales y huecos poplíteos), cuello, muñecas, párpados, región peribucal y genitales. Puede presentarse con lesiones agudas o crónicas, evolucionando en brotes, con prurito intenso. Puede desaparecer (75-90%) o progresar a la última fase. Existen formas localizadas como el mal plantar juvenil, que afecta plantas y dorso de los pies, forma periorbitaria, queilitis exfoliativa, intertrigo auricular y eccema del pezón. Por otro lado, se describen formas atípicas como lo son la papular y folicular. 3. Adulto. (15 a 23 años de edad). PRURIGO ACTINICO: Es una erupción papular o nodular frecuente, persistente, pruriginosa y escoriada de la piel expuesta al sol y en menor medida de la no expuesta. Es frecuente en verano y en ocasiones no desaparece con el invierno, por lo general aparece en la niñez y a veces remite en la pubertad. Parece ser una variante persistente y algunas veces coexístente de la erupción de lumínica polimorfica (ELPM) aunque sus características clínicas son diferentes. La exposición de radiación UV sería el inductor del prurigo actínico (PA) ya que el trastorno es más intenso en primavera y en verano y las respuestas cutáneas anormales a la radiación están presentes en dos tercios de los pacientes, más a menudo con radiación UVA que con UVB. El PA podría ser una forma de ELPM exagerada, de evolución lenta y por lo tanto ser una reacción de hipersensibilidad de tipo retardado. Esto se sustenta que más pacientes con PA de los esperados presenta familiares cercanos con ELPM, el antígeno linfocitario humano (HLA) DR4, presentan el 30% de los sujetos normales, se encuentra en el 80-90% de los pacientes con PA y el subtipo DRB1 0407 de DR4 presentan en el 6% de los sujetos normales y frecuentan en indios estadounidenses se encuentra alrededor del 60% de aquellos con PA. El PA es más común en el sexo femenino y suele comenzar a los 10años tiende a mejorar y desaparecer en la adolescencia, aunque en ocasiones puede persistir en la vida adulta. La erupción por PA está presente durante todo el año aunque por lo general empeora en el verano, y en raros casos se produce en invierno. Así mismo las exacerbaciones de las lesiones tienden a producirse durante el clima soleado, en lugar de hacerlo después de episodios de exponerse al sol, aunque los brotes tipo ELPM son posibles. En los casos típicos las lesiones son pápulas, nódulos pruriginosos, excoriaciones asociadas con eccematización liquenificación o formación de costras, también se describe queilitis sobre todo en el labio inferior. El PA puede asociarse a una tendencia a la eccematización. TRATAMIENTO: Las medidas generales son de gran importancia, la explicación a los padres acerca del padecimiento, su predisposición atópica, la cronicidad y evolución por brotes. Se recomienda además el uso de pijamas largos, mosquiteros, insecticidas cuando no están los niños. Tópicamente se utilizan lociones antipruriginosas y esteroides tópicos. Por vía oral se utilizan antihistamínicos no sedantes. En algunas ocasiones se utiliza Tiamina por vía oral, la cual se ha recomendado empíricamente por mucho tiempo "tal vez sea un repelente natural que se excreta por la piel", dosis 200mg a 600mg por día, lo más importante es evitar la infección secundaría. Se utilizan emolientes, capsaicina tópica, radiaciones UV y sistémicamente Talidomida, la cual está contraindicada cuando hay riesgo de embarazo y de producir neuropatía periférica. CASO CLINICO Niña de 11 años de edad que consulta por haber presentado lesiones eccematosas en el brazo izquierdo, donde hace un mes, en un puesto ambulante, le realizaron un tatuaje de henna de color negro. A los 10 días, comenzó con eritema y vesículas muy pruriginosas, que cubrían toda la zona del tatuaje. Las lesiones se resolvieron tras un mes de tratamiento con corticoide tópico, con persistencia de

CURSO ENARM CMN SIGLO XXI TEL: 36246001

Pharmed Solutions Institute

PÁGINA 404

MANUAL DE TRABAJO DEL CURSO ENARM CMN SIGLO XXI hipopigmentación residual limitada a la zona del tatuaje. La paciente se hacía, desde hace tres veranos, tatuajes transitorios de henna color marrón en la playa. PREGUNTA Cual de los siguientes factores esta mas relacionado con el padecimiento. RESPUESTA a.- Infecciones de la piel. b.- Exposición a alérgenos. c.- Irritantes como jabon, shampoo y cremas. d.- Inhalacion ingestión de alérgenos. PREGUNTA Considerando el caso clínico cual es la conducta terapéutica mas apropiada. RESPUESTA a.- Indica emolientes. b.- Corticoides tópicos de leve potencia. c.- Corticoides tópicos de moderada potencia. d.- Corticoides tópicos de alta potencia. CASO CLINICO Masculino de 8 meses de edad que es llevado por su madre por presencia de tos y secresion nasal desde hace dos días y a la exploración se observa en cara, brazos enrojecimiento, la madre agrega que le pica y que se presentan y luego desapareces solos, al tacto la piel se siente aspera. PREGUNTA Cual es la conducta a seguir menos adecuada? RESPUESTA a.- Evitar ambiente caluroso, ropa de lana, platicos y otras fibras. b.- Bañarlo con agua templada y de preferencia de periodos largos para mitigar lesiones. c.- Aplicar crema hidratante y aceites varias veces al dia. d.- Indicar crema con corticoide. ERITEMA POLIMORFO. CIENCIAS BASICAS: El eritema multiforme (EM) o eritema polimorfo es una condición reactiva aguda, autolimitada, en ocasiones recidivante, mediada inmunológicamente, que afecta la piel y las membranas mucosas. Desencadenada fundamentalmente por el virus del herpes simple (VHS), presenta lesiones en diana y un patrón histopatológico inflamatorio. Se cataloga como una reacción inflamatoria aguda, autolimitada, con diversos niveles de gravedad y variado conjunto sintomático. CLASIFICACION: Eritema multiforme maculopapuloso o “eritema polimorfo menor (menor intensidad en los cambios anatomopatologicos, escasa o nula participación mucosa). Representa el 80 % de los casos. Eritema multiforme vesiculoampolloso o “eritema polimorfo mayor”. Las formas graves, copn afeccion del estado general y posible participación de órganos internos corresponderían al Sindrome de Steven-Jhonson. La Necrolisis toxica epidérmica puede coinsiderase como un proceso aparte aunque algunos autores lo consideran dentro del mismo espectro. SALUD PUBLICA: Predomina en adultos jóvenes, de 20-40 años, con afección de niños y adolescentes en menos del 20% de los casos. Es algo más frecuente en varones y no presenta predilección racial. Se observa con mayor frecuencia durante la primavera y verano, probablemente secundario a la reactivación del VHS desencadenada por la exposición solar. Entre un 25% y un 50% de casos no puede establecerse el agente desencadenante. PATOGENIA: Se han reportado numerosos factores desencadenantes, entre los cuales se encuentran infecciones, enfermedades malignas, enfermedades autoinmunes, radiación, inmunizaciones, medicamentos (AINE, sulfonamidas, anticonvulsivantes (30%), penicilinas, doxiciclina, tetraciclinas) y menstruación. El VHS es la causa más común y está involucrado en un 80-90% de los casos, con mayor asociación al VHS tipo 1. Se obtiene una historia de herpes labial en un 50% de los pacientes en las 2 semanas previas al desarrollo del EM; sin embargo, la lesión herpética puede ocurrir simultáneamente o desarrollarse luego de las lesiones en diana. Los mecanismos patogénicos han sido estudiados en los casos de infección por el VHS. En la recurrencia, material genético es fagocitado y transportado a la circulación periférica por monocitos/macrófagos que portan el antígeno linfocitario cutáneo, un antígeno que les permite transportarse a la piel, en donde se adhieren a las células endoteliales de la microvasculatura dérmica, y el ADN viral fragmentado es finalmente transferido a los queratinocitos del estrato basal y de las capas inferiores del estrato espinoso. La inflamación en la lesión cutánea se cree es debida a una respuesta inmune celular TH1 específica contra los queratinocitos que contienen el gen de la polimerasa de ADN (Pol), con la liberación de citoquinas efectoras como el interferón γ (IFN-γ), que lleva a la amplificación de una respuesta inflamatoria no específica a través de células T autorreactivas y que resulta en la apoptosis de queratinocitos aislados (necrosis celular satélite) y el resto de los hallazgos patológicos que observamos en el EM. Finalmente, aditivos de alimentos, como los benzoatos y terpenos, y agentes tópicos como los tatuajes y la dermatitis por contacto con hiedra venenosa, han sido reportados como causas de EM. DIAGNOSTICO: Tal como indica su nombre de la enfermedad, ls lesiones clínicas son polimorfas e incluyen maculas, papulas y lesiones vesiculo ampollosas. Las lesiones más características muestran una apariencia anular concéntrica “en iris” o “en diana” Generalmente no se observan pródromos; algunos pacientes pueden presentar fiebre, cefalea, malestar y mialgias leves una semana antes de la erupción. Las lesiones cutáneas se desarrollan en forma súbita, completándose el brote en 3-5 días. Las lesiones pueden diferir entre un paciente y otro, y variar durante la evolución de la enfermedad. Hay sensación de prurito o ardor. La erupción inicia como pápulas

CURSO ENARM CMN SIGLO XXI TEL: 36246001

Pharmed Solutions Institute

PÁGINA 405

MANUAL DE TRABAJO DEL CURSO ENARM CMN SIGLO XXI eritematoedematosas que simulan picadas de insectos, que permanecen fijas por más de una semana. Algunas lesiones aumentan de tamaño, formando placas menores de 3cm, redondeadas, de bordes bien definidos, que desarrollan anillos concéntricos (eritema iris). La lesión en diana típica consiste en una porción central de color rojo sucio, rojo vinoso o purpúrico, inmediatamente rodeada por una zona externa de color rojo intenso; le sigue un anillo edematoso pálido, rodeado finalmente de un halo eritematoso. La porción central en ocasiones puede presentarse vésico-ampollar, formando el herpes iris de Bateman, o adquirir un aspecto costroso. La distribución es acral y simétrica, con predilección por el dorso de las manos y superficies extensoras de las extremidades superiores; con menos frecuencia se afectan las palmas, cara, cuello, tronco y extremidades inferiores. Se han descrito lesiones siguiendo las líneas de Blaschko. Las lesiones mucosas se presentan en un 25-60%, y ocurren conjuntamente con las lesiones cutáneas. La mucosa oral es la más afectada, principalmente la mucosa no queratinizada del tercio anterior. Inicia de forma rápida como edema y enantema que afecta los carrillos, gingival y lengua, que progresa a erosiones superficiales con formación de seudomembranas; en los labios se observan fisuras, sangrado y costras serohemáticas. El prurito, la sensación de quemazón, así como la pobre alimentación y baja ingesta de líquidos debida al dolor de las erosiones mucosas, que puede ser severo, son causas importantes de morbilidad en el EM Laboratorio: No hay pruebas específicas para el diagnóstico. En casos severos puede encontrarse aumento de la velocidad de eritrosedimentación, leucocitosis o elevación de las pruebas de función hepática. El diagnóstico del eritema multiforme es clínico. Los cambios histológicos dependen de la morfología clínica, la duración y el área de la lesión de donde se toma la biopsia. El hallazgo más precoz es la apoptosis de queratinocitos. Se observa necrosis celular en satélite, con linfocitos unidos a queratinocitos necróticos aislados. ERITEMA MULTIFORME MENOR: Originado principalmente por infecciones virales herpéticas (herpes simple) o Mycoplasma pneumoniae y solo en 1 % por productos farmacéuticos. La erupción se produce en el término de 12 a 24 horas y en la mitad de los casos hay un pródromo similar a una infección de las vías respiratorias altas. Las lesiones cutáneas típicas son máculas rojo- azuladas, con 3 anillos concéntricos denominados en diana o arco de tiro y una ampolla central, si bien pueden estar afectadas las mucosas. De hecho, 20 % de los eritemas multiformes afecta a niños y adultos jóvenes; su inicio es “abrupto”, con fiebre alta y síntomas prodrómicos intensos. Se observan lesiones cutáneas en el tronco, similares al eritema multiforme menor, pero más extensas y necrosantes. Daña gravemente a más de 2 mucosas y ocasiona erosiones costrosas profundas en los labios; su evolución es más prolongada, especialmente con fármacos de vida media más larga y sus manifestaciones clínicas duran entre 3 - 6 semanas. Las formas menores de eritema multiforme desaparecen espontáneamente en alrededor de 3 semanas sin secuelas. SÍNDROME DE STEVENJOHNSON: Es una forma grave de eritema multiforme habitualmente de ori gen tóxico. El grado de afectación cutánea varía desde formas con gran lesionalidad hasta casos con escasas o nulas lesiones cutáneas. Sin embargo, se caracteriza por la severa participación de mucosas (oral, nasal, conjuntival, genital) y afectación del estado general. NECRÓLISIS TÓXICA EPIDÉRMICA: La característica clínica más importante es la formación de grandes ampollas flácidas que condicionan el desprendimiento de láminas de epidermis, dejan do extensas áreas denudadas. Se trata de una situación grave, en ocasiones mortal. TRATAMIENTO: La aplicación del tratamiento puede mejorar el cuadro sintomático y acortar la evolución del proceso; pero las formas mayores suelen ser mortales o causar graves secuelas. La tasa de mortalidad para el síndrome de Stevens-Johnson se sitúa entre 1 y 5%, según distintos autores. La existencia de una gran extensión de zonas denudadas, la edad avanzada, la insuficiencia renal concomitante y la afectación pulmonar empeoran el pronóstico. Dado el curso autolimitado del proceso, en eritema multiforme menor, solo se requiere tratamiento sintomático. En los casos postherpeticos recurrentes puede plantearse tratamiento con Aciclovir u otros agentes antiherpeticos. El Síndrome de Stevens Jhonson y La Necrolisis toxica epidérmica son procesos potencialmente graves que requieren de ingreso del paciente y manejo en cuidados intensivos, en condiciones de asepsia. Existe controversia respecto al empleo de corticoides sistémicos pues si bien suelen condicionar un alivio de sintomatología subjetiva no hay evidencia de que reduzcan la morbimortalidad. En ocasiones se precisa antibioticoterapia para combatir complicaciones sépticas. CASO CLINICO Preescolar de 3 años de edad, de sexo femenino, previamente sana, que consulta por cuadro de dos días de evolución, caracterizado por fiebre, odinofagia y discreto compromiso del estado general. Evaluada en el Servicio de Urgencia se diagnosticó amigdalitis aguda purulenta. Se indicó penicilina benzatina 600 000 UI intramuscular por 1 vez. Posterior a la administración y en el transcurso del mismo día, comenzó con prurito generalizado, más intenso en la región dorsolumbar. Horas más tarde, la madre notó aparición de lesiones maculares eritematosas en extremidades, cara y tronco, por lo cual consultó nuevamente, 48 horas después. En ese momento, al examen clínico se constató una paciente en buenas condiciones generales con temperatura de 38,5° C axilar y lesiones papulares, eritematosas, redondeadas, distribuidas en forma simétrica y comprometiendo todo el cuerpo, más marcado en cara y extremidades inferiores. Las lesiones presentaban aspecto de anillos concéntricos, con un halo periférico eritematoso y un área más central pálida y en algunas lesiones una pequeña vesícula. La mucosa oral presentaba petequias escasas en forma aislada. No había otras mucosas comprometidas. El hemograma mostró GB 11 700 (75% segmentados, 5% baciliformes), hematocrito 38%, plaquetas 384 000 x mm3. Además VHS 80 mm x hr, PCR 10,9, creatininemia 0,61 mg/dl, nitrógeno ureico 0,22 mg/dl. Radiografía de tórax y sedimento de orina fueron normales. PREGUNTA Dentro de los siguientes diagnosticos cual es el menos probable? RESPUESTA a.- Eritema Multiforme. b.- Stevens-Jonhson. c.- Necrolisis Epidérmica Tóxica d.- Vasculitis.

CURSO ENARM CMN SIGLO XXI TEL: 36246001

Pharmed Solutions Institute

PÁGINA 406

MANUAL DE TRABAJO DEL CURSO ENARM CMN SIGLO XXI ERITEMA NODOSO. CIENCIAS BASICAS: Es un síndrome caracterizado por una erupción cutánea nodular, eritematosa, caliente y dolorosa, que se localiza preferentemente en la región pretibial, aunque también en ocasiones en muslos, brazos, antebrazos y otras áreas corporales. PATOGENIA: La etiología de este síndrome es múltiple, pero podemos globalmente dividirla en dos causas fundamentales: 1. De causa infecciosa: Bacterianas; tuberculosis (bacilo de Koch), estreptococicas, por yersinia, salmonela, shigela, campilobacter, brucelosis, tularemia, enfermedad por arañazo de gato, rickettsias, clamidias, mycoplasma, leptospirosis. Virica; VEB, hepatitis B, parvovirus B19. Protozoos; Giardia lamblia, amebas, toxoplasma. 2. De causa no infecciosa: Medicamentos; antoconceptivos orales, sulfamidas, penicilinas, salicilatos, TMP, bromuros, yoduros, sales de oro. Las lesiones del eritema nodoso representan una reacción frente a varios estímulos, se consideran como un tipo de vasculitis cutánea alérgica, pudiendo, por lo tanto, ser consecutivas de cierto número de estímulos provocadores. DIAGNOSTICO: En ocasiones se acompaña de fiebre, con aceptable estado general, puede haber artralgias de localización en grandes articulaciones de extremidades inferiores, que puede ser la manifestación de un proceso subyacente causa de esta patología. Las lesiones cutáneas, son bastante características; se trata de nódulos de 1-3 cm de diámetro, dolorosos, indurados, brillantes, enrojecidos, calientes y sobreelevados. Aparecen en las áreas pretibiales fundamentalmente, pero también pueden tener otras localizaciones, como las nalgas, pantorrillas y también en las extremidades superiores. La forma evolutiva de estas lesiones es bastante característica; durante un periodo de varios días, se hacen prominentes y de color violáceo; después de 1-2 semanas van disminuyendo su prominencia e induración, modificando el color, pasando por purpura oscuro y al final desaparecen dejando un rastro parduzco, sin dejar ulceración, ni cicatriz. Estas lesiones aparecen por brotes, en un periodo entre 3-6semanas, para determinar desapareciendo definitivamente. El diagnóstico es clínico, pero debemos hacer el diagnostico etiológico se refiere a la necesidad de descartar una enfermedad asociada; esto puede ser en ocasiones difícil y se puede no llegar al mismo en 20% de los casos. Hay que practicar pruebas complementarias como PPD, analítica básica general, coprocultivo, serologías, cultivos, radiografía de tórax, tránsito intestinal si es necesario, etc. TRATAMIENTO: Generalmente tienen una resolución espontanea, y la adopción de medidas generales como el reposo y las medidas físicas es suficiente. El tratamiento debe ir dirigido a tratar la enfermedad de base que sea la causante de este trastorno. Las medidas generales van encaminadas a disminuir el dolor y el acortamiento de la enfermedad. Dentro de ellas es importante insistir en el reposo y en la elevación de las extremidades inferiores. Diversos fármacos antiinflamatorios han demostrado ser útiles en el tratamiento del eritema nodoso, pero especialmente los salicilatos, la indometacina y el ibuprofeno. El empleo de corticoides sitemicos se debe reservar para aquellos casos con manifestaciones sistémicas graves (artritis, fiebre muy elevada), enfermos con sarcoidosis y pacientes con eritema nodosp persistente y ausencia de respuesta a los demás tratamientos mencionados. ERITEMA NODOSO MIGRATORIO: Es una variante clínica de eritema nodoso, también denominada Paniculitis migratriz de Bäfverstedt o Vilanova. Presenta nódulos unilaterales; en general menos numerosos, dolorosos y persistentes que los de la forma clásica. Al progresar tienden a dividirse y extenderse en la periferia, esto les confiere un patrón arciforme con bordes eritematosos, brillantes y un centro más violáceo o pardo. CASO CLINICO Masculino de 12 años de edad acudió a consulta debido a que 15 dias previas inicio con lesiones eritematosa violácea nodulares y dolorosas en los miembros inferiores de predomio en la parte posterior de las piernas. De 1 a 2 cm de diámetro, no referia ingesta de fármacos, fiebre, síntomas respiratorios, digestivos ni articulares, en la anamnesis no se encontraron antecedentes personales ni familiares de interés. PREGUNTA Considerando la etiología mas frecuente de esta patología, cual es la conducta diagnostica mas apropiada. RESPUESTA a.- Biopsia. b.- PPD. c.- Hemocultivo. d.- Panel viral DERMATITIS DE CONTACTO. CIENCIAS BASICAS: La dermatitis de contacto o eczema de contacto constituye un síndrome motivado por la reacción cutánea de una sustancia aplicada en la piel. Pue de ser una reacción eczematosa aguda o crónica y está causada por un irritante primario o por un mecanismo de sensiblización a un alérgeno tópico Es una dermatosis frecuente, afecta a ambos sexos y a todas las razas y puede presentarse a cualquier edad. Es la causa dermatológica más frecuente de enfermedad profesional. SALUD PUBLICA: Es una de las enfermedades de la piel más frecuentes, alrededor de 10% de todas las dermopatías. Principales sensibilizantes: Medicamentos (antibióticos, sulfonamidas, mercuriale, antihistamínicos, anestésicos, psoralenos), cosméticos (parafenilenodiamina, peróxidos, formaldehido, colorantes azoados, perfumes), metales (niquel, cromo, cobalto), ropas y zapatos (telas sintéticas, hules, cueros), plásticos (epoxi, resinas, acrílicos, nylon), remedios caseros (ajo, limón, yerbas). PATOGENIA: Cualquier sustancia u objeto que este en contacto con la piel puede ocasinar una dermatitisde contacto. El mecanismo puede ser: A) Alérgico, por el modelo clásico de inmunidad celular (hipersensibilidad tipo IV). Es necesaria fase de sensibilización, fase de latencia y de reexposición o desencadenante. El antígeno suele ser un hapteno que una vez en la epidermis, previa unión a una proteína, conforma un antígeno completo que, mediante células de Langerhans, es presentado a los linfocitos T que proliferan como células T efectoras y de memoria penetrando en la circulación sanguínea. En la fase de desencadenamiento, 48 hrs después de la exposición, los linfocitos sensibilizados reconocen a los alérgenos dando lugar a una transformación blástica y proliferación clonal con liberación de citocinas mediadoras de la inflamación. También participan otras células sanguíneas. Es posible la sensibilización por vía oral o parenteral. El fenómeno de autoeczematizacion por proteínas epidérmicas es frecuente en lesiones en piernas. B) Irritativo que origina un inflamación de la piel secundaria a la exposición de un agente irritante, en concentración y tiempo suficiente o contactos repetidos del mismo que da lugar a lesión celular sin que exista mecanismos inmunológicos. DIAGNOSTICO: Localización variada y siempre en relación al agente causal. La clínica puede ser: 1. Aguda, con eritema, edema, vesículas, ampollas y en casos extremos necrosis. Existe prurito quemante. 2. Subaguda, con placas eritematosas con descamación. 3. Crónica con lesiones en placas liquenificadas y zonas de descamación y excoriación. A veces grietas y

CURSO ENARM CMN SIGLO XXI TEL: 36246001

Pharmed Solutions Institute

PÁGINA 407

MANUAL DE TRABAJO DEL CURSO ENARM CMN SIGLO XXI fisuras. En casos muy intensos y de exposición prolongada al antígeno, pueden aparecer lesiones a distancia o “ides”. El diagnóstico se basa en la clínica, profesión y hábitos del paciente. Las pruebas epicutáneas sirven para diferenciar el eczema de contacto de origen irritativo del de origen alérgico y, en esta caso identificar el antígeno. Los diferentes tipos de eczemas o dermatitis de contacto tienen los mismos rasgos histopatológicos y van a depender de la fase evolutiva en la que se biopsien. Como rasgos generales evidenciaremos espongiosis, exocitosis y en dermis superficial infiltrado perivascular, fundamentalmente de tipo linfocitario. TRATAMIENTO: Antes que nada debe indicvarse al paciente lo que no debe de hacer: No aplicarse nada en la piel, eliminar toda clase de pomadas, remedioscaseros, objetos, ropas cuando se sospecha de ellas. Evitar jabones y detergentes. No hay dietas. Puede comer de todo. Evitar el sol si hay fotosensibilidad. El tratamiento es sobre todo tópico y dependerá del estado de la piel. Una piel eczematosa requiere primero medicamentos para secarla, así están indicados los fomentos con agua de manzanilla o agua de vegeto al 50% (subacetato de plomo). Se aplican a la temperatura ambiente o fríos, cada 3 o 4 hrs, durante las primeras 24-48 hrs, hasta que la piel se seque y las costras melicericas se desprendan, cuando ello se logre se podrá aplicar las conocidas pastas inertes a base de óxido de znc y/o calamina. El uso de sedantes y antipruriginosos puede ser necesario en algunos casos y a veces hasta psicotrópicos del tipo de la clorpromazina. Los corticoides por vía tópica del tipo de la hidrocortisona están indicados solamente en las dermatitis por contacto por irritantes primarios y en casos muy limitados de sensibilización. Si hay mucha sensibilización y muchas ides, el esteroide mejorara de inmediato el cuadro, pero vendrán los rebotes cada vez más intensos y más ides y el resultado final será un estado eritrodermico difícil de controlar. CASO CLINICO Un niño de 12 años, en buen estado general y afebril, consultó, por presentar una placa eritematosa única, amplia, de bordes definidos, sobre la que asentaban pequeñas pápulas y pústulas, no fluctuante, localizada en la región lateral derecha del cuello, de 3 días de evolución; se acompañaba de leve prurito, pero era indolora. No presentaba adenopatías regionales. Había acudido a una guardia, donde se le indicó cefalexina en dosis de 100 mg/kg/día; en el momento de la consulta cursaba el primer día de tratamiento. En el interrogatorio se destacaba que, 48 horas antes a la aparición de las lesiones, el niño había participado en actividades recreativas al aire libre en la colonia de verano del club, y no se había aplicado repelente. PREGUNTA Cual es la conducta a seguir mas adecuada en este momento? RESPUESTA a.- Continuar con antibiótico por 5 dias. b.- Corticoides topicos. c.- Prednisona oral. d.- Cetirizina únicamente. DERMATITIS DEL PAÑAL. CIENCIAS BASICAS: Dermatitis del pañal, dermatitis amoniacal, “rozadura”. Es una erupción inflamatoria aguda del área cubierta por el pañal. SALUD PUBLICA: Afecta con mayor frecuencia a recién nacidos y lactantes, con un pico de incidencia a los 9 a 12 meses, aparentemente en relación con el cambio en la dieta. Se presenta además en niños mayores y adultos con incontinencia urinaria y/o fecal secundaria a alteraciones urológicas o neurológicas. La prevalencia es del 7 al 35% en población pediátrica, ocupando el 3ª a 4ª causa de consulta dermatológica en niños. No se ha descrito diferencia racial ni en cuanto al sexo. PATOGENIA: Koblenzer clasificó la dermatitis de esta zona en tres grupos: Grupo 1. Enfermedades que se producen en forma independiente del uso de pañales (epidermolisis bulosa, histiocitosis de células de Langerhans, etc.). Grupo 2. Enfermedades agravadas por el uso de pañales (dermatitis atópica, dermatitis seborreica, psoriasis, etc.). Grupo 3. Enfermedades provocadas por el uso de pañales, en niños no predispuesto y como consecuencia directa de su uso, es el más frecuente y ocupará nuestra revisión. Lo más característico es la dermatitis de contacto por irritante primario en la cual la humedad persistente provoca maceración de la piel; lo anterior aunado a la fricción generada por los movimientos del bebé altera la función de barrera y permite la acción de irritantes como son: heces (proteasas, lipasas y sales biliares), orina (urea y amoniaco), jabones, lociones, cremas y productos empleados para el lavado de los pañales; por lo tanto, los irritantes actúan en forma secundaria en piel ya dañada previamente. Por otro lado, la piel así dañada, es susceptible de infección por Candida albicans con mayor frecuencia (50%) y más raramente, agentes bacterianos (S aureus, estreptococos y enterobacterias). La dermatitis por contacto alérgica no es común el área del pañal, sin embargo algunas sustancias como gomas, plásticos, neomicina y mercurio pueden actuar como sensibilizantes. DIAGNOSTICO: Existen diferentes patrones de afección clínica: Dermatitis eritematosa simple. Es el más frecuente. Afecta áreas convexas (en W), respetando los pliegues. Las formas más leves se caracterizan por eritema y descamación; el eritema puede desarrollar una apariencia vidriosa y aparecer pápulas. Variedad sifiloide pápulo-erosiva o poserosiva de Sevestre-Jaquet. Es una forma moderada de la dermatitis eritematosa simple. Se presenta sobre las superficies convexas en que se observan pápulas del color de la piel o eritematosas, generalmente menores de 5 mm de diámetro, que por acción de la fricción o maceración se erosionan en la cúpula. Pueden volverse violáceas y liquenificadas. Ulcerosa. Se caracteriza por pérdida cutánea epidérmica o más profunda, siendo la presentación más grave de las anteriores; surge como consecuencia de la potencia y el tiempo de exposición a la agresión y sensibilidad individual de la piel. Dermatitis perianal. Limitada a esta área, se manifiesta como eritema, erosiones o pústulas; es más frecuente en el periodo neonatal. Puede deberse a cambios químicos (alteración del pH en procesos diarreicos o enteropatías) o infecciosos (dermatitis estreptocóccica perianal, parasitosis). El diagnóstico es clínico; puede solicitarse examen directo con KOH y cultivo para corroborar infección micótica. La biopsia se reserva a casos resistente al tratamiento, en que se sospecha otra enfermedad de base. TRATAMIENTO: Medidas preventivas, como son el cambio frecuente de pañal, recomendándose pañales desechables con geles absorbentes (alginatos), de mayor tamaño, para evitar el roce y facilitar la absorción; realizar el aseo del área con agua, dejando el área sin pañal el mayor tiempo posible. Las pastas con talco y óxido de zinc son útiles como aislantes y para reducir la fricción y se aplican posterior a cada cambio de pañal. En las lesiones con más de 72 horas de evolución o datos de infección por candida debe aplicarse algún agente antimicótico, 2 veces al día, por 3 semanas. En caso de impétigo segundario se recomienda el uso de antibióticos tópicos o sistémicos. La utilización de corticoesteroides deberá ser

CURSO ENARM CMN SIGLO XXI TEL: 36246001

Pharmed Solutions Institute

PÁGINA 408

MANUAL DE TRABAJO DEL CURSO ENARM CMN SIGLO XXI racional, restringiéndose a casos resistentes a las terapias convencionales, utilizando esteroides de baja potencia y nunca más de 2 semanas. COMPLICACIONES: Candidosis. Es la complicación más frecuente. Compromete los pliegues inguinales e interglúteos y se caracteriza por pápulas y pústulas sobre una base eritematosa, intensa, brillante, con bordes netos, elevados, geográficos y escama blanquecina; asimismo lesiones satélites constituidas por pápulas y pústulas que se presentan en la periferia. Se ha demostrado colonización por C albicans en la dermatitis de la zona del pañal con más de 72 horas de evolución (45-75%), aún en ausencia del cuadro clínico característico. Impétigo secundario. Agregándose pústulas y costras melicéricas. Granuloma glúteo infantil. Pápulas y nódulos rojizos o purpúricos de 0.5 a 4 cm, en región perianal y glútea; se ha asociado a irritación y fricción crónica, C. albicans y uso de esteroides fluorinados. CASO CLINICO Niña de 4 meses de edad que la trae su padre porque al ir a cambiarle el pañal le ha visto que tenía la zona muy eritematosa y además al limpiarle la niña se queja. PREGUNTA Considerando el agente etiológico mas frecuente, cual es la conducta especifica mas adecuada? RESPUESTA a.- Desecantes. b.- Emolientes. c.- Antifungico. d.- Antihistaminico. PSORIASIS. CIENCIAS BASICAS: Es una enfermedad autoinmunitaria eritematodescamativa muy frecuente de evolución crónica que cursa a brotes. Se caracteriza por lesiones bien definidas con escamas típicas blanco-nacaradas. SALUD PUBLICA: Afecta a 1-3% de la población general. Aproximadamente el 25% de los casos de psoriasis se inicia durante la infancia, y aumenta su incidencia con la edad hasta la adolescencia. PATOGENIA: Es una enfermedad hereditaria aunque no se conoce todavía el modo de herencia. Existe una predisposición genética compleja y probablemente multifactorial. Hay una frecuencia aumentada de algunos haplotipos HLA. Los factores genéticos influyen en el patrón de la psoriasis, severidad y edad de inicio. El riesgo para los familiares de primer grado de un caso aislado es menor de 10%, pero si los dos padres tienen psoriasis los hijos tienen un riesgo de un 50% de padecerla. Existen unos factores desencadenantes conocidos capaces de precipitar o exacerbar un brote. Las infecciones (particularmente por Streptcoccus), el frio, el estrés emocional, los traumatismos cutáneos repetidos y ciertos fármacos (cloroquina y corticoides sistémicos). DIAGNOSTICO: Las lesiones de psoriasis infantil son placas eritemato-escamosas similares a las de la edad adulta, de tamaño y forma variables, con los mismos signos característicos al raspado y con tendencia también a desarrollar lesiones de psoriasis en las zonas de roce o de rascados y traumatismos (fenómeno isomórfico de Koebner). La biopsia cutánea muestra una epidermis engrosada con acantosis y papilomatosis, elongación y edema de la dermis papilar y un adelgazamiento de la epidermis por encima de las papilas. Se han realizado numerosos estudios sobre la calidad de vida en pacientes con psoriasis3, y se ha demostrado que repercute en actividades cotidianas y en el bienestar físico y psicológico de los enfermos. TRATAMIENTO: En niños se basa en la experiencia y en las pautas terapéuticas empleadas en los adultos, con las limitaciones propias de sus efectos secundarios. Tópico: Emolientes y queratolíticos: No tienen efectos secundarios, alivian el prurito y disminuyen la descamación, aunque es el tratamiento menos eficaz. Son útiles para conseguir eliminar escamas importantes y así facilitar la penetración de otros productos más efectivos. Uno de los más utilizados es la vaselina salicílica al 3-5%. El alquitrán de hulla es la brea más utilizada; en niños se emplean los baños de breas sólo en casos muy extensos, por su efecto calmante y reductor. La antralina (dithranol) al 0,1% parece que es menos irritante y es útil en placas muy queratósicas; carece de toxicidad general. Se ha demostrado además que los tratamientos en aplicaciones cortas son efectivos y menos irritantes, mediante aplicaciones diarias de 10-30 min y aumentando progresivamente la concentración del 0,1 hasta el 2%. Corticoides: tienen en la psoriasis una triple acción: antiinflamatoria, inmunosupresora y antiproliferativa. Su uso se ve limitado por los efectos secundarios: efectos locales como atrofia, estrías, telangiectasias e hipertricosis a menudo irreversibles; las erupciones acneiformes son también frecuentes cuando se aplican en la cara corticoides de alta potencia, y aparecen pápulas y pústulas redondeadas y rojizas y posteriormente comedones; a diferencia del acné juvenil, las lesiones se encuentran todas en el mismo estadio evolutivo. En pliegues, genitales y cara causan más efectos secundarios locales que en el resto del tegumento. Efectos sistémicos: la absorción de corticoides potentes a través de la piel puede provocar un Cushing iatrogénico e incluso detención del crecimiento. La vitamina D como tratamiento para la psoriasis, dado que induce la diferenciación de queratinocitos y disminuye su proliferación. Calcipotriol. Proporciona buenos resultados en niños sin alterar la concentración sérica de calcio, siempre que no se sobrepasen los 50 g/semana. Inmunomoduladores tópicos: El tacrolimus y el pimecrolimus actúan inhibiendo la calcineurina y mejoran las placas de psoriasis poco hiperqueratósicas. Si son muy hiperqueratósicas, se requiere la aplicación previa de un queratolítico tópico para facilitar su absorción. Psoralenos + radiación ultravioleta A (PUVA): La PUVAterapia está basada en la interacción entre la radiación ultravioleta A (UVA) y un agente fotosensibilizante químico: los psoralenos, pero no se recomienda su empleo en niños menores de 10 años, ya que se acumulan en el cristalino y si éste está aún en desarrollo puede causar daños irreversibles; los rayos UVA penetran hasta el cristalino y pueden inducir la aparición de cataratas, por lo que hay que extremar la protección ocular durante la radiación. La alta permeabilidad del cristalino en los jóvenes contraindica este tratamiento antes de los 12 años. Por otra parte, son ya conocidos los efectos carcinogénicos a largo plazo de la radiación UVA15-17. Tratamiento sistémico. Retinoides, metotrexato, ciclosporina. Tratamientos biológicos: Son anticuerpos monoclonales o proteínas humanas recombinantes contra algunas citocinas: etanercept, infliximab, adalimumab (antifactor de necrosis tumoral), efalizumab. Hay pocos casos en niños, aunque con buenos resultados, pero de uso experimental y todavía no está aprobado

CURSO ENARM CMN SIGLO XXI TEL: 36246001

Pharmed Solutions Institute

PÁGINA 409

MANUAL DE TRABAJO DEL CURSO ENARM CMN SIGLO XXI CASO CLINICO Varón de 17 años que, tras pasar una amigdalitis, presenta este exantema cutáneo. A la EF se observan papulas y pequeñas placas eritematosas con minima descamaciones distribuidas de forma generalizada, pero principalmente en el tronco. PREGUNTA Considernado el tipo de psoriasis que presenta el caso, cual de las siguientes medidas es la mas adecuada para intervenir en el agente casual? RESPUESTA a.- Corticoide tópico de mediana potencia, en excipiente no demasiado graso. b.- Realizar baños con derivados de alquitrán. c.- Antibiotico especifico preferentemente por cultivo. d.- Exposición solar moderada y frecuencia. DERMATITIS SEBORREICA. CIENCIAS BASICAS: Es una dermatosis inflamatoria eritemato-descamativa de evolución crónica y recurrente que se localiza en cara, cuero cabelludo, pliegues y regiones medio-torácicas e interescapulares (zonas con mayor cantidad de glándulas sebáceas activas). También se le conoce como eczema seborreico o pitiriasis seca y esteatoide. SALUD PUBLICA: Enfermedad frecuente en países desarrollados, no existiendo diferencias de género. La dermatitis seborreica infantil aparece en la lactancia, habitualmente entre la 2ª y la 8ª semanas de vida Puede afectar tanto a niños como a adultos o ancianos. PATOGENIA: La causa es desconocida. Se consideran factores predisponentes, genéticos, emocionales, Atópicos, neurológicos, bacterianos, endocrinos, alimentarios, medicamentosos, estrés y alcoholismo. También se ha relacionado con la disminución de ácidos grasos esenciales y biotina. Aunque se conoce que existe un cambio cuantitativo y cualitativo en la composición del sebo con transformación de triglicéridos en ácidos grasos más irritantes con mayor cantidad de colesterol y menor de escualeno, no se sabe cuál es la causa de la citada alteración aunque se implica en ello la acción del Pityrosporum ovale, levadura lipófila que forma parte de la flora cutánea normal y se convierte en oportunista con potencial patógeno. En pacientes con SIDA hay disminución de linfocitos CD4. En niños se desconoce la causa aunque se sabe que del 15-35% de os pacientes durante la infancia padecieron dermatitis seborreica, padecen en la edad adulta, psoriasis o dermatitis atópica. Un grupo de autores suecos afirma que la dermatitios seborreica infantil (DSI) podría deberse a un déficit de la enzima d-6-desaturasa, que convierte el ácido linoléico en ácido gammalinolénico (AGL). Este déficit pare ce normalizarse hacia los 6 ó 7 meses de edad, coincidiendo con la desaparición de la DSI. La asociación entre las levaduras de Malassezia y la patogenia de esta enfermedad ha sido causa de controversia desde los años setenta. Evidencia reciente relaciona a la dermatitis seborreica con metabolitos del hongo y con la inducción de citocinas. El hallazgo de las levaduras en escamas de los pacientes es lo habitual. Las especies más frecuentes en cuero cabelludo son: M. globosa y M. restricta, y en rostro M. globosa y M. furfur. DIAGNOSTICO: En todas las áreas, las lesiones de DSI consisten en placas eritematosas de tinte rosado, con morfología redondeada o incluso circinada, con bordes bien definidos, aisladas o confluentes, que se cubren de una descamación amarillenta de aspecto grasiento, que merece la denominación de ‘seborreica’. La DSI no se acompaña de síntomas generales ni prurito. Para muchos autores, la forma más precoz de aparición de la DSI es la costra láctea. Otros, consideran que la costra láctea es una entidad diferenciada, sin ninguna relación con la DSI. Sea como fuere, la costra láctea se desarrolla en la primera o segunda semanas de vida, y se manifiesta como una gran placa descamativa, con escamas grasientas y adherentes, de color amarillento, bajo las cuales se puede apreciar un eritema más o menos vivo. Asienta sobre todo en el vértex, y puede extenderse hacia áreas frontales con menor frecuencia, puede ocupar una amplia superficie del cuero cabelludo. Las lesiones clásicas de DSI suelen afectar, además del cuero cabelludo, a la frente, pabellones auriculares, pliegues retroauriculares, nasogenianos, cejas, parpados y cuello. También puede extenderse a el tronco, con predominio en la zona periumbilical, axilas, inglés, tórax anterior y espalda En los miembros, suele afectar a los pliegues antecubitales y poplíteos. En ocasiones, puede ser un trastorno generalizado, de tipo eritrodérmico. La afectación de la zona del pañal es muy frecuente, especialmente en las áreas inguinales, pliegue interglúteo y genitales externos. El trastorno conocido como eritrodermia descamativa de Leiner cursa con eritrodermia descamativa ‘seborreiforme’, retraso del crecimiento y diarrea. No hay estudios a gran escala para establecer la histopatología de la DSI, aunque parece similar a la DS del adulto. Se observa paraqueratosis focal, acantosis y espongiosis. En la desembocadura de los folículos pilosos dilatados se observan polimorfonucleares neutrófilos. La dermis superficial muestra un infiltrado linfohistiocitario perivascular. TRATAMIENTO: No tienen tratamiento definitivo. Dependiendo del grado de afectación se puede utilizar desde jabones no alcalinos o sustitutivos del jabon, a lociones o crema a base de ácido salicílico, azufre, selenio, ketoconazol u otros derivados azolicos, ciclopiroxolamina, terbinafina, hidrocortisona, tacrolimus. A nivel general se puede complementar con biotina a altas dosis. CASO CLINICO En la revisión de un bebe al mes de nacido, la madre refiere que el niño presenta costras que le sale sobre todo en la cabeza y en la frente y cejas. PREGUNTA Cual de las siguientes medidas terapéuticas es la menos adecuada? RESPUESTA a.- Aplicar aceites y sustancias emoliente. b.- Champues con antifungicos. c.- Dieta rica en biotina. d.- Corticoide de baja potencia.

CURSO ENARM CMN SIGLO XXI TEL: 36246001

Pharmed Solutions Institute

PÁGINA 410

MANUAL DE TRABAJO DEL CURSO ENARM CMN SIGLO XXI ACNE. CIENCIAS BASICAS: Sinónimos acné vulgar, acné polimorfo o acné juvenil. Es la inflamación crónica folicular producida por retención de sebo, debida a obstrucción y distensión de la unidad polisebacea. SALUD PUBLICA: El “acné vulgaris” es la enfermedad más común de la piel, afectando al 80% de las personas entre los 11 y los 30 años de edad en algún momento. Además, a nivel de los adolescentes afecta más del 85%. Puede persistir por años y generar cicatrices permanentes y conllevar un daño importante a nivel del desarrollo psicosocial y un problema de autoestima. Aunque se observa en ambos sexos, predomina ligeramente en el hombre. PATOGENIA: La susceptibilidad a la seborrea está determinada genéticamente. Los factores patógenos fundamentales son: queratinización folicular anormal, secreción sebácea aumentada, colonización bacteriana e inflamación local. No hay influencia práctica de la dieta. Al llegar la pubertad, por aumento en la sensibilidad de las glándulas sebáceas, las hormonas andrógenas generan aumento de tamaño y de la secreción de las mismas, lo que aunado a la hiperqueratosis de la pared y el poro foliculares, originan la lesión fundamental e inicial del acné, el comedón (“espinilla”). Se ha demostrado la presencia de Propionibacterium acnes, quien transforma los triglicéridos del sebo en ácidos grasos libres, que tienen efecto irritante local, favorece la inflamación por este mecanismo. Si la inflamación se realiza en la parte terminal del conducto folicular, a nivel de la epidermis, se origina una pústula, pero si se hace en la parte proximal del folículo y en la misma glándula sebácea, se originan los abscesos. Los factores psicógenos pueden agravar esta dermatosis de modo secundario, sobre todo en aquellos pacientes que presentan excoriaciones y ulceraciones autoprovocadas. DIAGNOSTICO: Afecta cara (99%), cuello y tórax (espalda 60%, pecho 15%), sobre todo en su parte superior. En cuanto a la morfología, el acné se caracteriza por polimorfismo lesional encontrándose lesiones no inflamatorias que son los comedones abiertos y cerrados; el comedón abierto se reconoce como una concreción gris oscuro, café o negro dentro de un orificio folicular dilatado y el comedón cerrado es una pequeña pápula del color de la piel. Las lesiones inflamatorias son pápulas eritematosas, pústulas y nódulos. Las secuelas del acné son cicatrices y senos epiteliales. TIPOS: De lesiones acneicas en cuadro anexo. CLASIFICACION: De acuerdo al tipo de lesión predominante, el acné se clasifican en: comedónico, papuloso, pustuloso, quístico, conglobata (abscesos y fístulas), queloideo y fulminans (pápulas, nódulos y quístes acompañados de datos de infección sistémica). De acuerdo a la gravedad del acné, se clasifican en: a) ACNE LEVE: Predominan los comedones. b) ACNE MODERADO: Con predominio de pápulas y pústulas inflamatorias. c) ACNE GRAVE: Que consta de nódulos (abscesos}) y de pápulas, pústulas y comedones. TRATAMIENTO: medidas generales; explicar al paciente claramente la naturaleza de la enfermedad, los posibles efectos adversos de la terapia y su duración habitualemnte prolongada, para evitar las expectativas a corto plazo y el abandono terapéutico. Detectar posibles factopres gravantes o predisponenetes (medicaments, sustancias químicas). Desaconsejar la manipulación de las lesiones e insistir en una adecuada limpieza e higienen cutánea. Evitar el uso de cosméticos comedogenicos (cremas, lociones, geles, maquillajes) y recomendar como alternativa interesante los cosméticos libres de grasa (“oil free”). El tratamiento tópico es el pilar fundamental en el que se debe basar el manejo del acné en atención primaria. Considerando la clasificación clínica anterior, el manejo se efectuará de la siguiente manera. Acné leve. Jabones neutros o azufrados para el lavado dos veces al día y aplicación de lociones desengrasantes como el licor de hoffman, a las que pueden agregarse ácido salicílico, resorcina o azufre a 1 y 3%. Pueden usarse comedolíticos como retinoides tópicos o peróxido de benzoilo. Acné moderado: puede usarse clindamicina o eritromicina tópicas, en solución o gel, dos veces al día o valorar el uso de antibióticos sistémicos aunado al tratamiento tópico. Acné grave: Los antibióticos sistémicos son de primera elección para el acné moderado. Acné grave: los antibióticos sistémicos son de primera elección para el acné moderado a grave, los más recomendados son las tetraciclinas, eritromicina, minociclina y el TMP/SFX, por uno a tres meses. La diaminodifenilsulfona (DDS) da muy buenos resultados en acné conglobata. La isotretinoina oral se utiliza en acné conglobata y nódulo quístico, pero su toxicidad es muy alta. Tratamiento psiquiátrico: La dismorfofobia es una entidad que se asocia con frecuencia en el acné y consiste en la percepción exagerada de los padecimientos cutáneos. Los pacientes se preocupan por su “fealdad imaginaria”. Esto lleva a depresión clínica y a un trastorno obsesivo compulsivo o fobia social. CASO CLINICO Paciente de 15 años de edad, de sexo masculino estudiante, que consulta por presentar compromiso sistemico, con fiebre, astenia, mialgias, anorexia y pérdida de peso. El cuadro fue de inicio súbito; presenta lesiones dermatológicas que afectan pecho y espalda. Estas lesiones son de aspecto granulomatosas que sangran fácilmente, muy secretantes y necrosis dolorosas. Se solicitan análisis de laboratorio. Los resultados de estos estudios evidenciaron leucocitosis, anemia, eritrosedimentación elevada (VSG), cultivos bacterianos de secreción y sangre negativos. PREGUNTA Cual de las siguientes medidas tiene la menor evidencia dentro del tratamiento de esta entidad? RESPUESTA a.- Eritromicina. b.- Isotretinoína c.- Dapsona. d.- Prednisona.

CURSO ENARM CMN SIGLO XXI TEL: 36246001

Pharmed Solutions Institute

PÁGINA 411

MANUAL DE TRABAJO DEL CURSO ENARM CMN SIGLO XXI PÚRPURA DE SCHÖNLEIN-HENOCH (PSH). CIENCIAS BASICAS: Es una vasculitis sistémica de vaso pequeño y curso habitualmente agudo. Es una vasculitis leucocitoclástica, de mecanismo inmunologico la más común en la infancia. La causa es desconocida. Precedida, en ocasiones, por una infección de vías respiratorias que sugiere un posible agente infeccioso desencadenante, siendo larga la lista de patógenos implicados. Otros desencadenantes pueden ser fármacos (penicilina, ampicilina, eritromicina, quinina), alimentos, exposición al frío o picaduras de insectos. SALUD PUBLICA: Afecta en el 90% de los casos a niños, preferentemente varones (2:1), con una edad media de 6 años. PATOGENIA: Es una vasculitis mediada por IgA de los pequeños vasos. Hay aumento en la producción de IgA, aumento de inmunocomplejos circulantes de IgA y depósitos de IgA en las biopsias de piel y de riñón. La lesión renal de la PSH es indistinguible histopatológicamente de la nefropatía por IgA de la enfermedad de Berger. Ambas pueden producir insuficiencia renal. DIAGNOSTICO: Usualmente se presenta con la tétrada: púrpura palpable, artritis, dolor abdominal y enfermedad renal. La púrpura aparece en el 100% de los casos pero en una cuarta parte no es la manifestación inicial. Se caracteriza por lesiones palpables de 210mm, de color rojo-violáceo (90%), que se concentran característicamente en los glúteos y las extremidades inferiores aunque, en ocasiones, pueden afectar a los brazos y la cara, y rara vez al tronco. La afectación articular (50-85%) se manifiesta habitualmente como oligoartritis de miembros inferiores, afecta principalmente la articulación del tobillo y la rodilla. Las alteraciones gastrointestinales (5070%) se caracterizan por dolor abdominal cólico o anginoso (empeora con la ingesta) que puede acompañarse de náuseas, vómitos, diarreas, rectorragias y melenas Los síntomas gastrointestinales se deben al edema de la pared intestinal y a la hemoragia propia de la vasculitis, cuando estas lesiones son muy intensas pueden dar lugar a un infarto intestinal y perforacion. En la gastroscopia y/o colonoscopia se observan petequias, hemorragias o erosiones. Son complicaciones posibles una hemorragia intestinal grave y la invaginación o perforación intestinal. La enfermedad renal (20-50%), normalmente se presenta como una glomerulonefritis moderada que cursa con hematuria, habitualmente microscópica, y con menos frecuencia proteinuria leve y raramente como síndrome nefrítico, nefrótico y/o insuficiencia renal. En los adultos hay un mayor riesgo de afectación renal crónica que puede llegar a insuficiencia renal terminal. Aunque se trata de una vasculitis autolimitada hay recurrencias en la tercera parte de los casos. El diagnóstico es clínico y se utilizan los criterios del American College of Rheumatology: 1. Inicio de los primeros síntomas a los 20 años o menos. 2. Lesiones purpúricas sobreelevadas sin trombocitopenia. 3. Dolor abdominal anginoso (empeora con comidas), vómitos, diarreas, rectorragias y melenas. 4. Biopsia cutánea: neutrófilos peri o extravasculares, en arteriola y/o vénula. 5 Biopsia cutánea: neutrófilos en pared vascular de arteriola y/o vénula. La presencia de ≥ 2 criterios clasifica de PSH con sensibilidad: 87,1% y especificidad: 87,7%. Las crioglobulinas, FR, ANA y ANCA suelen ser negativos o positivos a títulos bajos. En los adultos la biopsia de piel y el control de la posible afectación renal son obligatorios. La biopsia cutánea muestra una vasculitis leucocitoclástica que afecta a los pequeños vasos (arteriolas, vénulas y capilares). El hallazgo más característico es la infiltración de neutrófilos de los pequeños vasos de la dermis, con frecuente fragmentación del núcleo de los mismos. Estudios mediante inmunofluorescencia muestran depósitos granulares de IgA y, en menores cantidades, C3 y fibrina en la pared de los vasos. La microscopia renal convencional suele mostrar una proliferación mesangial aislada y sólo en casos graves una glomerulonefritis con semilunas. TRATAMIENTO: Se resuelve espontáneamente en el 94% de los niños y en el 89% de los adultos, por lo que el primer objetivo es tranquilizar al paciente y utilizar tratamiento sintomático para la afectación articular y el dolor abdominal. El tratamiento con corticoides (1-2mg/kg) es controvertido. Una revisión sistemática indica que los corticoides reducen la duración del dolor abdominal y disminuyen el riesgo de invaginación intestinal, afectación renal y recurrencias. Su uso en la PSH no complicada no mejoró el curso de la enfermedad, según un ensayo clínico8. La afectación renal grave precisaría de corticoides, inmunosupresores y/o plasmaféresis. Lo único que tiende la cronicidad es la nefritis. CASO CLINICO Varón de 7 años de edad ingresado por presentar un cuadro de dolor abdominal difuso y una deposición melénica. La semana previa había consultado por varicela y había tomado ibuprofeno, 20 mg/kg/día. No presentaba antecedentes de interés, salvo dolor abdominal recurrente. En la exploración al ingreso no se objetivaron hallazgos patológicos. Al ingreso la coagulación era normal así como el sistemático de sangre. Al noveno día apareció un exantema eritematoso violáceo palpable, simétrico, en extremidades inferiores, y se estableció el diagnóstico púrpura de Schönlein-Henoch. PREGUNTA Cual el predominio vascular primordial en esta patologia. RESPUESTA a.- Vasculitis de grandes vasos. b.- Vasculitis de vasos medianos. c.- Vasculitis de pequeños vasos. d.- Vasculitis asociadas a conectinopatias. CASO CLINICO Niño de 6 años de edad, sin antecedentes personales ni familiares de interés, que fue valorado en el servicio de urgencias por presentar, desde hacía 24 h, lesiones cutáneas maculopapulosas y purpúricas distribuidas simétricamente por las nalgas, miembros inferiores y superiores asociadas a signos inflamatorios en las rodillas y tobillos, muy indicativo de púrpura de Schönlein-Henoch. Los análisis de orina, de función renal y la presión arterial eran normales. Se inició tratamiento con antiinflamatorios no esteroides y se dio de alta. PREGUNTA Considerando el cuadro clínico cual de las manifestaciones es la menos frecuente para orientar el diagnostico de PSH. RESPUESTA a.- Fiebre prolongada de origen desconocido. b.- Lesiones cutáneas sugestivas.

CURSO ENARM CMN SIGLO XXI TEL: 36246001

Pharmed Solutions Institute

PÁGINA 412

MANUAL DE TRABAJO DEL CURSO ENARM CMN SIGLO XXI c.- Artralgias, artritis, miositis y serositis. d.- Parametros de laboratorio de inflamación. PREGUNTA Cual de los siguientes factores etiológicos puede estar más relacionado con esta patologia. a. Estreptococo del grupo A beta hemolítico o M. pneumoniae, b.- Virus de hepatitis A, citomegalovirus ó virus de Ebstein Barr. c.- Vacuna del sarampión, influenza, rubéola, neumococo. d.- Antibióticos betalactámicos, macrólidos ó AINES. PREGUNTA Cual de las siguientes manifestaciones es la menos frecuente para hacer el diagnostico. RESPUESTA a.- Manifestaciones cutáneas. b.- Manifestaciones digestivas. c.- Manifestaciones articulares. d.- Manifestaciones renales. ANEMIAS. CIENCIAS BASICAS: Es la disminución de la hemoglobina o el hematocrito, por debajo de los valores normales de la localidad y de acuerdo a la edad y sexo, con la consecuente hipoperfusión tisular de oxígeno. A través de la BH (Hb, Hto, reticulocitos, VCM, CHCM), se obtienen los valores que indicaran si corresponde a un síndrome anémico. SALUD PÚBLICA: En México 67% de los menores de 2 años presentan ferropenia y en edad escolar 34%. En escolares y adolescentes la falta de una ingesta adecuada de carne de res y en el caso de las mujeres con la menstruación, favorecen esta entidad. De 2-11% de los niños menores de 12 años de edad presentan deficiencia de folatos y aquellos que ingieren vegetales verdes solo 0.22%. PATOGENIA: La anemia puede deberse a falta de producción (aplasia, nutricional), perdida (hemorragia) o destrucción de los eritrocitos (hemolisis). DIAGNOSTICO: En el RN a término que presenta anemia al nacimiento o durante los primeros días de vida, clínicamente puede “lucir bien” y ser un hallazgo por estudio rutinario de H o presentar manifestaciones de leves a severas: “lucir mal”, palido con ingesta inadecuada de leche, irritable. En los lactantes y preescolares puede observarse palidez, irritabilidad, somnolencia, astenia, menor incremento ponderal, infecciones de repetición y en algunos casos pica o geofagia. En escolares o adolescentes puede manifestarse además cefalea: acufenos, fosfenos, lipotimia, palpitaciones, disnea, parestesias, calambres. La palidez como dato objetivo de anemia, es valido en los lechos ungueales, ya que la mucosa oral o la conjuntiva pueden manifestar otras alteraciones que la eclipsan y la palidez global puede ser expresión de otras alteraciones como estrés coartación aórtica, estado de choque. Se puede encontrar de acuerdo a la intensidad y cronicidad de la anemia, manifestaciones de insuficiencia cardiaca como: taquicardia, diferencial amplia en la tensión arterial, edema de miembros inferiores, hepatomegalia. La ruta diagnostica inicia al determinar el porcentaje de reticulocitos corregidos: arregenerativa o falta en la producción cunado están por debajo de 3% y regenerativa cuando están por arriba de 3%. Habitualmente las segundas tienen cifras pór arriba de este porcentaje y es debida a hemorragia aguda o hemolisis. ANEMIA FERROPENICA: La mayor causa por falta de producción de eritrocitos, en la mayor parte de los países es la ferropenia. Se inicia con la depleción de eritrocitos de hierro (ferropenia latente), pasa por eritropoyesis ferropénica (inicialmente microcitosis y posteriormente hipocromía) y termina con la anemia. En el neonato este evento es poco frecuente porque la reserva de hierro es directamente proporcional a su peso corporal. El prematuro no recibe la cantidad de hierro suficiente de Fe y debe reponerse el mismo en las primeras semanas. Las complicaciones del embarazo que originan perdidas sanguíneas crónicas, disminuyen la dotación de hierro al neonato, como la hemorragia vaginal, amniocentesis traumática, varices aneurisma del cordón umbilical, inserción velamentosa, placenta previa, desprendimiento prematuro de placenta. ANEMIA MEGALOBLASTICA: Hay una eritropoyesis ineficaz, puede deberse a deficiencia nutricional por deficiencia de folatos y de vitamina B12. En el prematuro la deficiencia se presenta en el primer mes de nacido y requiere reposición de este elemento. Las reservas de folatos se mantienen por 2 semanas en cualquier etapa del desarrollo, la ingesta de vegetales verdes mantiene estos niveles, por lo que los lactantes alimentados solo con leche, desarrollan esta deficiencia. Algunos medicamentos interfieren con el metabolismo de los folatos como los barbitúricos, difenilhidantoina, TMP/SFX, excepcionalmente pueden llevar a anemia megaloblastica. En el caso de B12 el hígado almacena hasta por 6 meses, por lo que es raro observar anemia por este elemento. ANEMIAS CONGENITAS: Como la aciduria orótica hereditaria y la anemia diseritropoyética hereditaria se presentan excepcionalmente. En los RN si hay anemia normocitica, pudiese corresponder a infecciones de tipo congénito como la rubeola, parvovirus B19, CMV, toxoplasmosis, ingesta demediacmentos durante el embarazo o bien aplasia pura de la serie roja congénita. En el resto de las edades pediátricas debe descartarse aplasia pura de la serie roja adquirida, enfermedades crónicas como colagenopatias, endocrinopatías, infección o insuficiencia renal. Anemia arregenerativa: DFe acuerdo a las características morfológicas de los eritrocitos, se estudia por la presencia de hipocromía, macrocitosis o normocitosis. DISQUERATOSIS CONGÉNITA: (síndrome de Zinsser -Cole-Engman) es una enfermedad hereditaria caracterizada por hiperpigmentación de la piel, leucoplasia de las membranas mucosas y distrofia ungueal, que en aproximadamente la mitad de los casos se acompaña de pancitopenia con hipoplasia medular. Se ha demostrado que esta enfermedad se transmite en forma recesiva ligada al cromosoma X, habiéndose observado en algunas madres portadoras, lesiones de leucoplasia en la mucosa bucal. También se han descrito otros probables patrones de transmisión que incluyen la herencia dominante ligada al cromosoma X y autosómica dominante. Lo anterior marca una diferencia con la anemia de Fanconi que es una enfermedad heredada con carácter autosómico recesivo y que además se presenta asociada a múltiples alteraciones esqueléticas y cromosómicas. ANEMIAS HEMOLÍTICAS HEREDITARIAS: la alteración en el eritrocito se puede presentar a diferentes niveles; a. Defectos en la membrana del eritrocito. La forma más frecuente es la esferocitosis hereditaria, la cual ocupa el primer lugar entre las anemias hemolíticas en México. Otros variantes incluyen la eliptocitosis, ovalocitosis y la estomatocitosis hereditaria. b. Capacidad limitada para la síntesis de las cadenas normales de la hemoglobina. En este grupo se incluyen las talasemias y las alteraciones en la secuencia de los aminoácidos de las

CURSO ENARM CMN SIGLO XXI TEL: 36246001

Pharmed Solutions Institute

PÁGINA 413

MANUAL DE TRABAJO DEL CURSO ENARM CMN SIGLO XXI cadenas polipeptídicas lo cual determina un gran número de variantes moleculares; las más frecuentes de estas variantes son la hemoglobinopatía S o anemia africana (ocupa el segundo lugar en frecuencia dentro de las anemias hemolíticas hereditarias en México), la hemoglobina C, D y E. c. Deficiencias enzimáticas de los eritrocitos. Incluyen principalmente las deficiencias de piruvatoquinasa y de glucosa-6-fosfato deshidrogenasa. ANEMIA HEMOLÍTICA AUTOINMUNE: Destrucción exagerada de eritrocitos, producida por una actividad inmune aberrante que se dirige contra los glóbulos rojos del propio huésped. Clínicamente puede ser clasificada en primaria o secundaria, de acuerdo a si se halla asociada o no a otros estados patológicos. Estos últimos pueden incluir infecciones virales o bacterianas, enfermedades asociadas con producción de autoanticuerpos, síndromes de inmunodeficiencia y neoplasias. Se desconoce el mecanismo por el cual el organismo forma anticuerpos contra sus propios glóbulos rojos, pero sí se conocen los mecanismos inmunológicos capaces de destruir los glóbulos rojos in vivo. La anemia hemolítica autoinmune primaria (AHAIP) se caracteriza por la presencia de anticuerpos capaces de actuar contra los eritrocitos del propio paciente. Los anticuerpos han sido caracterizados como inmunoglobulinas G(IgG) o M(IgM). Éstas actúan directamente contra los antígenos de la membrana del eritrocito o bien, mediante la formación de complejos inmunes dirigidos contra ella. En otros casos, la lisis de la membrana eritrocitaria se produce por activación del sistema del complemento. Habitualmente el inicio es agudo con descenso rápido en los niveles de hemoglobina. Asimismo, la mayor parte de los pacientes presentan periodos breves de evolución, menores de tres a seis meses. Clínica; decaimiento, anorexia, palidez, dolor abdominal e ictericia de intensificación progresiva; puede ocurrir además emisión de orina de color pardo obscuro a consecuencia de la hemoglobinuria. A la exploración física puede observarse hepatomegalia y esplenomegalia de intensidad variable. Con menos frecuencia se manifiestan fiebre, tendencia hemorrágica, disnea, taquicardia y linfadenopatías. Los hallazgos de laboratorio característicos de los pacientes con AHAIP: anemia, macrocitosis, reticulocitosis, hiperbilirrubinemia indirecta y concentración de hemoglobina libre en plasma en valores superiores al normal. TRATAMIENTO: Anemia ferropenica sulfato ferroso: 20mg/kg/día, fumarato ferroso 15mg/kg/día, el tiempo de administración va en relación a corregir la anemia, y completar las reservas, se puede uno guiar con la BH y suspender cuando ha desaparecido la microcitosis o bien completarse por 6 meses. Excepcionalmente se requiere la administración por vía parenteral y solo cuando existe intolerancia gástrica. Anemia por deficiencia de folatos; ácido fólico 1 mg por VO, por un mes. Anemia por deficiencia de B12, es conveniente su administración por vía parenteral, ya que la absorción intestinal es insuficiente, cuando hay deficiencia. Se indica 100mg IM al día por 10 días. En la anemia hemolítica congénita el consumo exagerado de folatos por la reprodccion celular incrementada, requiere mantener niveles adecuados de este elemento. Se administra ácido fólico a 1mg/día VO por día. Anemia hemolitca autoinmunitaria; se indican folatos por la misma razón que en la congénita y esteroides por vía endovenosa si hay crisis hemolítica o por VO prednisona, iniciando 2 mg/kg/dia, con disminución paulatina cada 2 semanas, hasta completar 2 meses de tratamiento. Tratamiento quirúrgico; en el caso de talasemia o esferocitosis hereditaria, que cursen con esplenomegalioa o hemolisis intensa, está indicada la esplenectomía, a partir de los 4 años de edad. Tratamiento nutricional; en las anemias carenciales es necesario realizar cambios en los hábitos alimenticios, como el inicio temprano de la ablactación, con frutas, verduras y carne de res por lo menos 4 días de la semana y en especial las mujeres adolescentes por las pérdidas sanguíneas debidas a la menstruación. El trasplante alogenico de medula ósea está indicado en anemia hemolítica congénita, ya que permite la curación en la mayor parte de los casos, principalmente en pacientes con alfa talasemia, beta talasemia mayor y anemia falciforme. CASO CLINICO Lactante de 2 años von 6 meses con letargia, hiporexia, retraso ponderoestatural e involución psicomotriz (hipotonía, irritabilidad y pérdida de sonrisa social) de un mes de evolución. Como antecedentes personales destacaban un crecimiento intrauterino retardado disarmónico, una serología materna positiva para lúes durante la gestación y anemia perniciosa mal controlada. No había consanguinidad. La alimentación era con lactancia materna exclusiva. En la exploración física presentaba un estado general regular, hipoperfusión periférica, palidez e ictericia mucocutáneas, un soplo sistólico eyectivo, taquicardia, papilitis lingual, aftas palatinas. PREGUNTA Considerando el cuadro clínico cual de las manifestaciones es la menos importante para el diagnostico. RESPUESTA a.- Antecedentes personales. b.- Antecedentes maternos. c.- Soplo sistólico eyectivo. d.- Aftas palatinas. PREGUNTA Cuales son las manifestación para hacer el diagnostico diferencial para anemia ferropenica. RESPUESTA a.- Volumen corpuscular medio disminuido. b.- Hemoglobina corpuscular media normal. c.- Amplitud de la distribución eritrocitaria disminuido. d.- Recuento de plaquetas mormales o aumentadas. PREGUNTA Cual no es una indicación para referir al paciente antes referido. RESPUESTA a.- Ferritina serica no concluyente. b.- Perfiel de hierro no concluyente.

CURSO ENARM CMN SIGLO XXI TEL: 36246001

Pharmed Solutions Institute

PÁGINA 414

MANUAL DE TRABAJO DEL CURSO ENARM CMN SIGLO XXI c.- Paciente con buen apego que no responde. d.- Pérdida sanguínea aguda. LEUCEMIAS. CIENCIAS BASICAS: La leucemia aguda es el padecimiento más frecuente en pediatría, no solo México, sino en todo el mundo. Es un trastorno genético de las células hematopoyéticas. La característica principal de este padecimiento, es la proliferación de la célula más primitiva de la medula ósea, que puede ser desde la más indiferenciada hasta la mayor grado de diferenciación, proviene tanto de linfocitos B como T. Las leucemias crónicas son mucho más comunes en los adultos que en los niños. Suelen crecer más lentamente que las leucemias agudas, aunque también son más difíciles de curar. Las leucemias crónicas también se pueden dividir en dos tipos. SALUD PUBLICA: En el niño existen varios tipos de leucemias siendo la más frecuente leucemia aguda linfoblastica (LAL), comprende más de 65% de los casos, le sigue la leucemia mieloblastica aguda (LAM) en un 25% frecuencia (es la responsable del 30% de las muertes por leucemia en la edad pediátrica) y la leucemia granulocitica crónica en menos de 5% de los casos. Se estima alrededor de 1 caso por 2880 niños sanos, en pacientes con síndrome de Down la frecuencia es de 1 en 95 casos. La OMS señala que en México es más frecuente en el sexo femenino. El pico máximo de frecuencia es el alrededor de los 3-5 años de edad. PATOGENIA: La etiología de esta enfermedad es desconocida, existen factores predisponentes como: radiación (in útero, con las placas simples de radiografía), anomalías cromosómicas (Sx. Bloom, anemia de Fanconi, Sx de Down), otras entidades (Sx. de Poland, de Rubistein-Taybi, neurofibromatosis), fármacos (cloranfenicol, agentes citotoxicos), insecticidas, la frecuencia de leucemia aguda es mayor en los familiares de pacientes con LA. Las células leucémicas se pueden reproducir rápidamente, y puede que no mueran cuando deberían hacerlo, sino que sobreviven y se acumulan en la médula ósea, desplazando a las células normales. Un tipo común de anomalía del ADN que pueden dar lugar a la leucemia se conoce como translocación cromosómica. El ADN humano está empacado en 23 pares de cromosomas. En una translocación, el ADN de un cromosoma se desprende y se une a un cromosoma diferente. El punto en el cromosoma donde ocurre el desprendimiento puede afectar los oncogenes o los genes supresores de tumores. Por ejemplo, una translocación vista en casi todos los casos de leucemia mieloide crónica (CML) infantil y en algunos casos de leucemia linfocítica aguda (ALL) infantil es el intercambio de ADN entre los cromosomas 9 y 22, lo que conduce a lo que se conoce como cromosoma Philadelphia. Esto crea un oncogén conocido como BCR-ABL. También se han descubierto en leucemias infantiles muchos otros cambios en cromosomas o en genes específicos. Algunos niños heredan mutaciones del ADN de uno de sus padres que pueden aumentar su riesgo de desarrollar cáncer. CLASIFICACION: Citomorfológica, la clasificación Francoamericana-británica (FAB) reconoce 3 tipos morfológicos: L1; tamaño célula pequeño, forma del nucléolo regular, nucléolos ausentes y relación núcleo-citoplasma es elevada. L2; población celular más heterogénea e irregular, la relación núcleo-citoplasma es mayor y a menudo se perciben nucléolos. L3; células morfológicamente idénticas a las de linfoma de Burkit, grandes, homogéneas con nucléolo redondo y ovalado, así como presencia de vacuolas. En los niños L1 80-85%, L2 15% y L3 menos de 3%. Clñasificacion tipo celular afectado: Leucemia linfocítica crónica (LLC). Afecta a las células linfoides y es por lo general de crecimiento lento. Hay más de 15 000 casos nuevos de leucemia cada año. A menudo, las personas que son diagnosticadas con esta enfermedad son mayores de 55 años. Casi nunca afecta a niños. Leucemia mieloide crónica (LMC). Afecta a células mieloides y por lo general es de crecimiento lento al principio. Hay aproximadamente 5 000 casos nuevos de leucemia cada año. Afecta principalmente a adultos. Leucemia linfocítica (linfoblástica) aguda (LLA). Afecta a células linfoides y es de crecimiento rápido. Hay más de 5 000 casos nuevos de leucemia cada año. La LLA es el tipo de leucemia más común entre niños pequeños. También afecta a adultos. Leucemia mieloide aguda (LMA). Afecta a células mieloides y es de crecimiento rápido. Hay más de 13 000 casos nuevos de leucemia cada año. Afecta tanto a adultos como a niños. En forma general las LAL son positivas a la reacción de PAS, mientras que las LAM son negativas. La clasificación inmunológica en que los linfoblastos expresan antígenos correspondientes a las células linfocitarias B o T en distintas fases de maduración, aproximadamente 80% de las LAL. DIAGNOSTICO: Clínica; Inicio insidioso desde asintomático o presentar síntomas y signos atribuibles a cuadros virales de vías respiratorias altas, sospecha por descubrimiento incidental en una BH. Más de 50% presentan síntomas generales como fiebre elevada, anorexia, adinamia y malestar general con palidez de tegumentos. En 30% manifestaciones de hemorragia como petequias, equimosis y epistaxis. Las manifestaciones de dolor óseo son frecuentes y no siempre corresponden con los hallazgos radiológicos. Las manifestaciones neurológicas o de tumores es poco frecuente y se ha asociado con algunos tipos de específicos de leucemias como las LAL de células T. Laboratorio: Presencia de anemia, trombocitopenia y las cifras de leucocitos pueden ser normales. El estudio de medula ósea por aspiración es la prueba diagnóstica (90%), preferentemente se deberá efectuar en la espina iliaca posterosuperior, sin embargo se puede realizar en la espina anterosuperior, en general se muestra celularidad aumentada, con sustitución casi por linfoblastos. Después de realizar los frotis se deberá teñir con tinción de Wright. Si en la cuenta diferencial existen más de 25% de linfocitos, se establece el diagnostico de leucemia. Además se deben realizar pruebas de funcionamiento hepático. Otro estudio obligado para el diagnóstico y seguimiento es el de LCR en el que se buscan linfoblastos

CURSO ENARM CMN SIGLO XXI TEL: 36246001

Pharmed Solutions Institute

PÁGINA 415

MANUAL DE TRABAJO DEL CURSO ENARM CMN SIGLO XXI preferentemente, proteínas y glucosa. Es obligado realizar estudio de inmunofenotipo para confirmar el diagnóstico y linaje celular B o T. TRATAMIENTO: Para LAL comprende 3 fases de tratamiento, va de la inducción a la remisión el objetivo es erradicar la carga de linfoblastos de la MO, al término de esta fase que dura 28 días, la MO debe encontrase con menos de 5% de blastos, para considerar que el paciente está en remisión; la segunda fase es de consolidación; la tercera fase es la de mantenimiento en la que se indican reducciones con la finalidad de que al término de esta no exista enfermedad medible en la MO. La duración dependerá del grupo de riesgo si es de bajo o alto grado, actualmente para una LAL de alto riesgo es de 30 meses, mientras que opera leucemias de bajo riesgo es alrededor de 24 meses. La base del tratamiento es la quimioterapia: Algunos de los medicamentos comúnmente usados para tratar la leucemia en niños incluyen: Vincristina (Oncovin, Daunorubicina, también conocida como daunomicina (Cerubidina), Doxorrubicina (Adriamycin), Citarabina, también conocida como arabinósido de citosina o ara-C (Citosar), L-asparaginasa (Elspar), PEG-L-asparaginasa (pegaspargasa, Oncaspar), Etopósido (VePesid, otros). Hay que dar profilaxis al SNC, que se establece con la aplicación de quimioterapia intratecal. Actualmente se acepta que la indicación para trasplante de MO para el paciente de LAC es en aquellos que presentan una segunda remisión, esto quiere decir, pacientes que tienen una recaída a la MO; pacientes con factores de riesgo elevado como las alteraciones citogenéticas del cromosoma Philadelphia t (9,22), t (4,11) entre las más importantes, la indicación de trasplante debe ser una vez alcanzada la remisión. La cirugía tiene una función muy limitada en el tratamiento de la leucemia en niños. Como las células leucémicas se propagan por toda la médula ósea y a muchos otros órganos a través de la sangre, no es posible curar este tipo de cáncer con cirugía. CASO CLINICO Niño de 10 años que acude a urgencias por dolor en hombro y codo izquierdos desde hace 4 días, con signos de inflamación en codo izquierdo. El dolor en hombro izquierdo cede pero a las 24 h aparece dolor en hombro derecho, con limitación para la abducción y rotación interna. No refiere traumatismos. El día anterior hizo ejercicio (waterpolo). Hace un mes, presentó un episodio de ojo rojo diagnosticado de conjuntivitis aguda, que cedió con colirio antibiótico. No refiere procesos infecciosos previos. Antecedentes personales sin interés. En los antecedentes familiares destacan madre y tío materno con espondilitis anquilosante HLA B27 positiva. Exploración física: Temp. 38°C. Afectación discreta del estado general. Inflamación del codo izquierdo con derrame y limitación funcional de la flexo-extensión. Impotencia funcional de abducción de hombro derecho. El resto de la exploración es normal. PREGUNTA Cual es la conducta a seguir. RESPUESTA a.- Realizar BH, QS, EGO. b.- Realizar pruebas CPK, GPT, GOT, GGT y DHL. c.- Realizar serología para micoplasma, Borrelia, Yersinia enterocolitica (serotipo O:3), parvovirus, VHA, VHC, RPR con rosa de Bengala. d.- Realizar artrocentesis e iniciar antibiótico e anti-infamatorio. PREGUNTA Se obtiene los siguientes resultados: Hemograma: leucocitos 9.930/mm3 (neutrófilos 53,3%, linfocitos 35,8%, monocitos 6%, eosinófilos 2,1%), hemoglobina 14,2g/dl, VCM 80,6, plaquetas 308.000/μl, proteína C reactiva 5mg/dl; VSG 26mm/h. ioquímica: CP , GPT, GOT y GGT normales, factor reumatoide 14 U/ml; anticuerpos antiestreptolisina (ASLO), inmunoglobulinas y C3 y C4 normales. Serologías para Micoplasma, Borrelia, Yersinia enterocolitica (serotipo O:3), parvovirus, VHA, VHC, RPR con rosa de Bengala, negativas. HLA B27 positivo. Radiografías de tórax, codo izquierdo, hombro derecho y sacro-ilíacas, normales. Hemocultivos y cultivo de líquido articular, negativos. Fue diagnosticado con artritis reumatoide juvenil, se ajusta tratamiento, cual es la conducta a seguir. RESPUESTA a.- Indica ibuprofeno. b.- Indica prednisona. c.- Inicia con sulfaxalazina. d.- Inicia con cloroquina. PREGUNTA Tras 6 meses de seguimiento se inicia tratamiento con metotrexato por persistencia de los síntomas. Pasados 10 meses comienza con astenia y dolor abdominal. En control analítico se objetiva neutropenia (320/μl) y aumento de LDH hasta 588 U/l. Se realiza un nuevo hemograma por fiebre que objetiva un 36% de blastos. Cual es la conducta diagnostica a seguir. RESPUESTA a.- Aspirado de medula osea. b.- Escaneo con resonancia magnetica. c.- Frotis y gota gruesa. d.- Realizar Gamagrama. RETINOBLASTOMA (RB). CIENCIAS BASICAS: Es un tumor intraocular embrionario de origen retiniano que se presenta generalmente en niños menores de 5 años. Dejado a su evolución, el retinoblastoma es casi siempre fatal y un retraso en su tratamiento suele suponer un pronóstico visual precario, de ahí la transcendencia del diagnóstico precoz. SALUD PUBLICA: Representa aprox. El 3% de todos los canceres presentes en niños menores de 15 años. Su incidencia es de 1:15-20.000 recién nacidos vivos y su etiología esporádica en el

CURSO ENARM CMN SIGLO XXI TEL: 36246001

Pharmed Solutions Institute

PÁGINA 416

MANUAL DE TRABAJO DEL CURSO ENARM CMN SIGLO XXI 70-75% de los casos o hereditaria, en el 25-30%. De acuerdo con la Dirección General de Epidemiología de la Secretaría de Salud, podría representar la neoplasia sólida más frecuente después de los tumores del sistema nervioso central. Generalmente aparece antes de los 2 años de edad y 95% de los casos se diagnostican antes de los 5 años. PATOGENIA: El gen responsable, el RB1, está localizado en el cromosoma 13 y actúa de forma dominante; es decir, en células donde las dos copias del gen estén dañadas. El 35-40% de los pacientes corresponden a casos hereditarios que son portadores de una mutación germinal. De ellos, más de dos tercios representan nuevas mutaciones (“mutación de novo”) sin historia familiar previa. Fenómenos inflamatorios y glaucoma, ambos secundarios a un tumor que empuja hacia adelante el diafragma cristaliniano o a células tumorales que azolvan la malla trabecular. Los sitios afectados más frecuentes por metástasis son el SNC y el hueso (60%), dentro de la afecciono sea el sitio más frecuente fue la órbita y huesos craneales (90%), huesoso largos (18%). DIAGNOSTICO: Se presenta clínicamente con leucocoria 50-60% (reflejo blanco en la pupila), estrabismo y signos inflamatorios, más raramente con glaucoma, celulitis orbital, uveitis y hemorragia vítrea. El 60% de los RB son unilaterales con una edad media al diagnóstico de 24 meses, mientras que Los RB bilaterales son menos frecuentes, suelen ser multifocales y de más temprana edad de comienzo (15 meses). En la mayoría de los niños con tumores bilaterales, ambos ojos están afectados al diagnóstico. Sólo en algunos casos de RB unilateral se desarrolla un tumor contralateral más tarde. Las características de inicio precoz, bilateralidad y multifocalidad apuntan a un probable origen hereditario del proceso. Los retinocitomas o retinomas son tumores benignos resultantes de la regresión espontánea de retinoblastoma y que raramente pueden ser descubiertos en la edad adulta como lesión residual de un RB que cursó de forma subclínica en la infancia. Otros tumores asociados: Los portadores de la mutación germinal en RB1 tienen un exceso de riesgo de desarrollar otros tumores. La presencia de tumores extraoculares (Pinealomas, osteosarcomas, sarcomas de tejido blando, melanomas) llamados segundos tumores primarios, se manifiesta en la adolescencia o al inicio de la edad adulta, siendo el tiempo medio de aparición del segundo tumor 10-13 años. El diagnóstico de RB se establece mediante oftalmoscopia indirecta. La CT, MRI y ecografía sirven para confirmación del diagnóstico y estadio tumoral. En base a la focalidad tamaño y localización de los tumores existe un estadiaje de Reese-Ellsworth (en tabla adjunta) que evalúa el pronóstico, I Muy favorable para la conservación de vista. II Favorable para la conservación de vista. III Posible conservación de vista. IV Desfavorable para la conservación de vista. V Muy desfavorable para la conservación de la vista. TRATAMIENTO: La elección de la terapia adecuada depende no solo del estadio del tumor sino de factores tales como la focalidad (unifocal, unilateral multifocal, o bilateral, localización del tumor dentro del ojo y edad del paciente. Las opciones incluyen: Enucleación está indicado en todos aquellos casos unilaterales que cumplen alguno de los criterios siguientes: (1) Ocupar más de la mitad del ojo. (2)Extensión al humor vítreo. (3)Desprendimiento total de retina. (4)Crecimiento de nuevos vasos en el iris y (5) Implicación del tumor en otras estructuras del ojo. La Radioterapia externa incrementa en un 50% la aparición de otros tumores. Sin embargo, está indicado en RB bilaterales no susceptibles de tratamiento local. Este método se aplica en tumores mayores de 16 mm que se extienden a la órbita, cercanos a la fóvea, múltiples o cuando se extienden al humor vítreo. La radioterapia local puede ser efectiva en tumores grandes (>16 mm) no susceptibles a la crioterapia o fotocoagulación. Aunque reduce la probabilidad de aparición de tumores inducidos, está asociado a retinopatías y papilopatías. Fotocoagulación, este tratamiento se aplica en tumores pequeños (menos de 3 mm de diámetro y 2 de grosor) que no implican el disco óptico o la mácula. Llega a ser muy efectivo en tumores muy pequeños, con localización anterior al ecuador y en tumores donde la altura es igual o menor que la mitad del diámetro de la base. Crioterapia, es efectiva para el tratamiento de tumores de 5mm de diámetro y 3mm de grosor. Habitualmente son necesarias varias sesiones. Es de creciente interés el empleo de la Quimioterapia sistémica como terapia coadyuvante en niños previamente candidatos a enucleación o external beal radiation bilateral. El empleo de agentes alquilantes puede incrementar la incidencia de segundos tumores y está asociado al desarrollo de leucemia no linfoblástica e infertilidad. RETINOBLASTOMA TRILATERAL: Es un síndrome bien reconocido que se presenta en 5 a 15% de los pacientes con la forma hereditaria del retinoblastoma y que se define mediante la formación de un tumor neuroblástico de la línea media intracraneal, típicamente hasta más de 20 meses después del diagnóstico del retinoblastoma. Los pacientes que son asintomáticos al momento del diagnóstico de un tumor intracraneal tienen un pronóstico más favorable que aquellos pacientes sintomáticos. Considerando que el retinoblastoma trilateral tiene un pronóstico precario, y el corto intervalo entre el diagnóstico del retinoblastoma y la aparición de la enfermedad trilateral, el uso de exámenes mediante neuroimagenología de rutina podrían, potencialmente, detectar la mayoría de los casos dentro de los primeros 2 años del diagnóstico inicial. Ya que no es claro si el diagnóstico temprano puede incidir en la supervivencia, la frecuencia de los exámenes de detección con imágenes de resonancia magnética se recomienda para aquellos presuntos de tener una enfermedad hereditaria o para aquellos con enfermedad unilateral y antecedentes familiares positivos, con una frecuencia de cada 6 meses por 5 años. No es claro si esto tendrá un efecto en el desenlace o en la supervivencia. Se deben evitar las exploraciones por tomografía computarizada como métodos de detección de rutina en estos niños debido a la percepción de riesgo sobre la exposición con radiación ionizante. CASO CLINICO Niña de 7 meses, previamente sana, presenta leucocoria de 2 meses de evolución, no hay presencia de dolor ocular ni síntomas concomitantes. Al examen oftalmológico se constata ausencia de fijación y esotropia del ojo derecho con ausencia de reflejo rojo. PREGUNTA Cual de los siguientes diagnosticos diferenciales es el mas frecuente? RESPUESTA a.- Enfermedad de Coats. b.- Catarata.

CURSO ENARM CMN SIGLO XXI TEL: 36246001

Pharmed Solutions Institute

PÁGINA 417

MANUAL DE TRABAJO DEL CURSO ENARM CMN SIGLO XXI c.- Persistencia de vítreo primario hiperplásico. d.- Estadio V de la retinopatía de la prematuridad TUMORES DEL SNC. CIENCIAS: Representan la segunda causa de neoplasias en la infancia. Los tumores cerebrales primarios son un grupo diverso de enfermedades que, constituyen el tumor solido más común de la niñez. Los tumores más frecuentes son tres: astrocitoma cerebeloso (33.9%), el meduloblastoma (26.3%) y el glioma del tronco cerebral. Otros tumores importantes de considerar son el craneofaringeoma, tumeor de la región pineal y ependimoma. SALUD PUBLICA: En México no existen estadísticas, pero se estima 12%. En el INP, estos tumores ocupan el tercer lugar. Los tumores del sistema nervioso central (SNC) constituyen la segunda causa de muerte en los menores de 15 años, tan sólo superada por la leucemia. PATOGENIA: Se desconoce la causa de los tumores cerebrales infantiles pero existe una serie de factores predisponentes como: 1.- Genéticos.- Neurofibromatosis, Esclerosis Tuberosa , Enfermedad de von Hippel-Lindau, Síndrome de Li-Fraumeni, Síndrome de Turcot y el Síndrome de Nevo con Carcinoma de Células Basales, este último y aquellos que cursan con Tumor de Wilms son más propensos a desarrollar meduloblastoma. 2.- Inmunológicos.Inmunosupresión congénita o adquirida, individuos que serán sometidos a transplante renal ya sea antes o durante el mismo tienen 350 veces más riesgo de desarrollar Sarcoma de células reticulares y Linfomas, los pacientes con ataxia-Telangiectasia cursan con alteraciones inmunológicas y mayor riesgo de desarrollar neoplasias. 3.- Ambientales.- Exposición a Hidrocarburos Aromáticos, compuestos N-nitrosos, triazinas e hidrazinas. Una asociación menos documentada es el consumo, durante el embarazo de barbitúricos, exposición prenatal a rayos X, trauma, infección y anestésicos. El cáncer se produce por mutaciones de genes que regulan la proliferación y muerte celular. Las mutaciones genéticas pueden ocurrir dentro de la línea germinal o como mutaciones somáticas exclusivamente dentro de células tumorales. Solo una pequeña fracción de niños con Tumores cerebrales tienen mutaciones germinales adquiridas de sus padres o mutaciones nuevas. La causa de las mutaciones somáticas en la mayoría de todos los tumores cerebrales son desconocidas. Existe evidencia de que el crecimiento y diseminación de una neoplasia es dependiente de la angiogénesis (formación de nuevos capilares sanguíneos a partir del propio tumor lo que incrementa las posibilidades de incorporación de células neoplásicas a la circulación). Cada incremento sucesivo en el volumen del tumor es precedido por una fase de angiogénesis. CLASIFICACION: De acuerdo a la localización: existe un predominio de los tumores infratentoriales (43.2%), seguido de los tumores supratentoriales (40.9%), los de médula espinal (4.9%) y de sitios múltiples (11%). Infratentoriales y, de estos, 75% están situados en el cerebelo o cuarto ventrículo. Entre los tumores infratentoriales comunes (fosa posterior) se encuentran los siguientes: 1.ASTROCITOMA CEREBELAR: tumor de mejor pronóstico, generalmente de bajo grado. Existe un tumor específico denominado astrocitoma Pilocitico Juvenil, de excelente pronostico con terapia quirúrgica. No requiere radioterapia, estos tumores se ubican en los hemisferios cerebelosos y generalmente son quísticos. La porción tumoral en realidad es un nódulo mural. La cavidad quística visible en TAC es reactiva. 2.- MEDULOBLASTOMA: forma parte de los tumores neuroectodérmicos primitivos, por originarse de esta célula primordial, son muy agresivos y por ende de mal pronóstico, puede dar metástasis incluso vía LCR, se origina en la primera década de la vida en el techo del IV ventrículo, por lo cual puede causar hidrocefalia precoz, clínicamente se presenta con cefalea y vómitos, los signos serán relacionados con un síndrome cerebeloso, un niño con frecuentes caídas puede cursar una ataxia. Es de los tumores que con más frecuencia presenta diseminacionm extraneural (menos del 4%), siendo el hueso, médula ósea, linfáticos, hígado y pulmones los sitios más comunes. Tratamiento quirúrgico, radio y quimioterapia. 3.- EPENDIMOMA: ocurren mas frecuente en la región del torax. En raras ocasiones presentan malignizacion y en muchas ocasiones presentan asociación con quistes intramedulares. Es un tumor relativamente avascular y son encapsulados, es decir tienen un plano de clivaje que permite su extirpación completa y curación de los pacientes. Un grupo de ellos crece en el interior de la medula y otro en el cono medular. 4. GLIOMA DEL TALLO ENCEFÁLICO: puede ser de distintos grados, en general de mal pronóstico dado su natuiraleza infiltrativa. Puede estar a nivel alto manifestándose por síndrome cerebeloso e hidrocefalia. Pueden ubicarse a nivel bajo (bulbo medular), manifestándose por compromiso multiple de pares craneales bajos y por compromiso motor y sensitivo de extremidades. La radioterapia asociada a corticoides, mejoran la calidad de vida. 4. Tumor teratoide atípico. Los tumores supratentoriales incluyen aquellos que ocurren en la región selar o supraselar del cerebro y en otras áreas del mismo. Los tumores selares y supraselares representan aprox., el 20% de los tumores cerebrales infantiles, incluso los siguientes: 1. Craneofaringoma. 2. gliomas diencefalicos. 3 Tumores de células germinales. Otros tumores que ocurren de forma supratentorial incluyen los siguientes: 1. Astrocitoma o glioma de bajo grado. 2. Astrocitoma de alto grado o maligno. 3. Glioma mixto. 4. Oligodendroglioma. 5. Tumor neuroectodermico primario. DIAGNOSTICO: No existen cuadros patognomónicos para el diagnóstico de tumores cerebrales. Generalmente el inicio es insidioso y progresivo. Las manifestaciones clínicas de los tumores cerebrales infantiles depende de: 1.- Localización del tumor (zonas elocuentes, obstrucción o desplazamiento) 2.- Tipo histológico del tumor. 3.Edad y desarrollo del paciente. Los tumores intracraneales pueden manifestarse como déficit neurológico focal, crisis convulsivas o no convulsivas, aumento de la presión intracraneal, que puede deberse a efectos directos del tumor u obstrucción del líquido cefalorraquídeo. Un análisis de los síntomas tempranos de los tumores cerebrales infantiles mostró que los tumores supratentoriales se presentan con vómito en el 46 % de los casos, con cefalea en el 43% de los casos; mientras que los Tumores Infratentoriales se presentan con dificultades para la coordinación en el 59 %, vómito 76% y cefalea 56 %. Debe sospecharse de un tumor cerebral en todo niño con datos clínicos neurológicos insidiosos y progresivos. DESCRIPCION SINDROMÁTICA: I) Hipertensión Intracraneana: este cuadro puede manifestarse de manera aguda o crónica: Agudo: < 1 año incremento inesperado del perímetro cefálico, separación de suturas, alteraciones del estado de alerta (somnolencia, irritabilidad) disminución en la ingesta. En preescolares, escolares y adolescentes puede manifestarse con cefalea y vómito generalmente matutino que se exacerba con maniobras de Valsava y disminuye en el transcurso del día. El vómito puede ser en proyectil, irritabilidad, letargia, edema de papila, discromatopsia (pérdida de la visión de colores), escotomas centrales y la paresia del sexto par, este último inespecífico de topografía y explicado por la longitud del trayecto (falso focalizador). Crónico o Intermitente: irritabilidad, letargia, vómito, atrofia de papila o pérdida progresiva de la visión, cambios conductuales, de personalidad, del rendimiento académico, anorexia y pérdida o ganancia ponderal. II) Síndrome cefalálgico: La cefalea en tumores clásicamente se describe con un curso subagudo de 4 a 6 meses de duración, de tipo pulsátil, continua o intermitente. Despierta al niño por la noche, frontal, occipital o universal, de moderada intensidad medida por escala análoga visual del dolor, o porque el niño deja de hacer sus actividades. III) Crisis: Las crisis en los tumores generalmente son focales y pueden asociarse a patrones bioelétricos anormales focales. IV) Síndrome cerebeloso: Este puede ser vermiano con ataxia troncal o de la marcha, hemisférico con lateropulsión derecha, izquierda o indistinta, dismetría, disdiadococinesia, lenguaje escándido (disartria), reflejos

CURSO ENARM CMN SIGLO XXI TEL: 36246001

Pharmed Solutions Institute

PÁGINA 418

MANUAL DE TRABAJO DEL CURSO ENARM CMN SIGLO XXI osteotendinosos pendulares con o sin nistagmus horizontal. V) Neuropatía craneal: El involucro de pares craneales dependiendo de cuál sea, es sugestivo de la topografía de la lesión sin embargo esto puede ser muy sutil en virtud de que los tumores son más compresivos que destructivos. Entendiendo por neuropatía del III al XII par craneal. Los estudios necesarios para realizar el diagnostico son la TAC, siendo posible realizar el diagnostico hasta en 90%, la imagen por IRM, mielografía, mielotomografía, en caso de tumor como el meduloblastoma y ependimoma: siendo el diagnóstico definitivo el estudio histopatológico, obteniéndose a través de resección quirúrgica, con una toma de biopsia y exéresis del tumor, así como por biopsias por esterotaxia. Existe una ruta crítica para la evaluación de un niño con sospecha de neoplasia en SNC: Sospecha de tumor primario  estudios de laboratorio y gabinete interconsulta con neurocirugía y oncología procedimiento diagnostico-quirúrgico  clasificación histopatológica estadificacion  valoración radioterapia  valorar inicio de esquema de quimioterapia  seguimiento. TRATAMIENTO: Los cánceres dejados a su evolución natural son incontrolables y mortales en la mayoría de los casos. Se basa en la desaparición de los síntomas, así como el tratamiento específico, por lo que es importante individualizar cada caso, algunos pacientes requerirán el empleo de anticomisiales (Fenitoína 20 mg/kg/dosis de impregnación y mantenimiento 7 a 10mg/kg/día en dos dosis), así como de esteroides para el manejo de hipertensión craneal, ya que algunos autores mencionan que los esteroides tienen efecto vaso génico, aumentando la permeabilidad vascular, corrigiendo e edema cerebral y por tanto mejorando el flujo circulatorio de forma transitoria. En cuanto al tratamiento específico, la cirugía debe realizarse por el especialista experto en patología tumoral. En ocasiones los pacientes requieren colocación de válvulas ventriculoperitoneales para el manejo urgente de la hidrocefalia y posteriormente la cirugía de resección que siempre es recomendable sea completa, pero suele ser imposible. La radioterapia es una modalidad de tratamiento necesaria para los tumores malignos de SNC, los pacientes menores de 3 años presentarán detención del crecimiento y la posibilidad de retardo psicomotor, disfunción endocrina y radionecrosis. La quimioterapia es usada para las neoplasias del SNC, pero no todas son sensibles debido a la barrera hemartoencefalica CASO CLINICO Paciente de 9 años de edad, sexo masculino, que consultó por cuadro de 3 días de evolución caracterizado por cefalea holocránea de tipo opresivo y vómitos explosivos aproximadamente 6 veces al día. Durante la anamnesis la madre refirió que el paciente presentaba crisis de cefalea intermitente hace aproximadamente 5 meses, las que con el tiempo fueron aumentando en frecuencia e intensidad hasta constituir el cuadro de consulta actual. Al examen físico general el niño impresionaba en buenas condiciones generales, y se apreciaron varias lesiones vesiculares y costrosas en todo el cuerpo, compatibles con cuadro de varicela en sus últimas etapas. Al examen neurológico destacó paciente con un puntaje de 15 en la Escala de Glasgow, dismetría de extremidad superior derecha y leve paresia facial izquierda. PREGUNTA Cual es la conducta inmediata a seguir mas adecuada? RESPUESTA a.- Realizar puncion lumbar. b.- Realizar tomografía computada. c.- Medidas anti-edema. d.- Colocacion de derivación ventrículo peritoneal. CASO CLINICO Se trata de RN femenino, nacido por cesarea con 38 SDG por Capurro, apgar 8/9, sin datos de hidrocefalea, perímetro cefálico de 39.5 cm. Fontanelas anterior y posterior ampliar y plenas, con diastasis de la sutura sagital, tono muscular cervical disminuido, cardiopulmonar sin compromiso. Movilidad expontanea en las cuatro extremidades, discreta hiperreflexia miotatica, babinski espontaneo bilateral y reflejos primitivos globales disminuido, La TC mostro hidrocefalia supratentorial sin edema transependimario, dilatación de tercer ventrículo y en la fosa posterior una imagen hiperdensa ovalada de 46 x 42 mm que obliteraba el cuarto ventrículo en su parte posterior, se reporto una lesión tumora infratentorial probablemente intraaxial glial o extraaxial de origen mesenquimatoso. La RM reporto imagen hiperdensa en relación con el parénquima cerebral en la región pancerebelosa que infiltraba el tallo cerebral en el puente, mesencéfalo y lamina cuadrigeminal, que protruia por la hendidura tentorial encefálica y borra las foliar cerrebelosas. Por su efecto compresivo sobre acueducto de Silvio, además de ocacionar la dilatación ventricular supratentorial con hidrocefalia obstructiva. PREGUNTA Cual es el diagnostico mas probable? RESPUESTA a.- Astrocitoma congénito. b.- Meduloblastoma congénito. c.- Ependimoma congénito. d.- Craneofaringeoma congénito. PAROTIDITIS. CIENCIAS BASICAS: La parotiditis (paperas) es la inflamación de las glándulas salivares sobre todo de las parótidas. Es una enfermedad generalizada, viral, aguda y altamente contagiosa y que generalmente ocurre en la infancia. Se transmite por contacto directo e ingresa al organismo por vía respiratoria y usualmente tarda en aparecer de dos a tres semanas. Los casos de paperas en nuestro medio han disminuido dramáticamente con la introducción de la vacuna contra las paperas. SALUD PUBLICA: Es una enfermedad endemo-epidémica, con períodos de aparición que oscilan entre 2 y 6 años. Se estima que en ausencia de inmunización, el 85% de los adultos ha sufrido la enfermedad; una tercera parte de ellos sin síntomas evidentes (la mayoría antes de los dos años de

CURSO ENARM CMN SIGLO XXI TEL: 36246001

Pharmed Solutions Institute

PÁGINA 419

MANUAL DE TRABAJO DEL CURSO ENARM CMN SIGLO XXI vida). Se presenta en invierno y primavera. La afectación ha disminuido en los países que incluyeron la vacuna en forma rutinaria en sus planes de inmunización. PATOGENIA: El agente es un paramyxovirus (emparentado con los de influenza y parainfluenza), el virios esta formado por RNA, el virus penetra por una via respiratoria. Tiene como reservorio al hombre y el modo de transmisión es persona a persona (diseminación de gotitas y saliva del infectado). La incubación dura de 12 a 25 días (media de 18) y el período de contagio va desde 6-7días antes y hasta 9 días después del edema glandular (la máxima contagiosidad, ocurre 48 horas antes de las manifestaciones clínicas de la enfermedad). Los expuestos no inmunes, deben ser considerados infecciosos desde el 12° al 25° día de la exposición. La inmunidad suele ser permanente. DIAGNOSTICO: Sialoadenitis, la glándula que se afecta casi en forma constante es la parótida, puede comenzar bruscamente con tumefacción y dolor en la región parotídea, la piel suprayacente aparece tensa, pero no roja y sensible a la presión, consecutivamente el lóbulo de la oreja es empujado hacia arriba y fuera. Ocurre enrojecimiento y tumefacción del conducto de Stenon. Se caracteriza por fiebre, edema y dolor de una o más glándulas salivales (habitualmente las parótidas), dificultad para masticar a veces para hablar, cefalea. Por lo general es leve y autolimitada. Puede ser uni o bilateral (70%), alcanza su máxima expresión al tercer día remite lentamente en un plazo de tres a siete días. Si bien el compromiso generalmente es parotídeo, con cierta frecuencia las glándulas submaxilares también están afectadas, con mucho menos frecuencia ocurre lo mismo con las glándulas sublinguales. Orquiepididimitis se presenta en 20-30% de los varones pospuverales, es la manifestación extrasalival más frecuente, es unilateral y se instaura al ceder la tumefacción parotídea, con un nuevo ascenso febril COMPLICACIONES: Meningoencefalitis (inflamación de las membranas que cubren el cerebro y la médula espinal). Es la más frecuente dentro de la infancia. 1 por cada 400 casos, y más a hombres que a mujeres, en proporción de 1 a 4. Evoluciona en la mayoría de los casos hacia la curación espontánea en un período de cuatro días, sin dejar, secuelas. Orquitis (inflamación del testículo). Muy poco frecuente en la infancia, en la edad mayor se observa en un 30% de los casos. Comienza generalmente siete días después de la parotiditis, con fiebre, calofríos, dolor de cabeza y dolor en los testículos y en la parte baja del abdomen. En un porcentaje bajo es bilateral y en tercio evoluciona hacia la atrofia testicular. Sordera. Se afecta más severamente la audición de tonos altos observando la recuperación en pocas semanas. Es de baja incidencia: Oforitis, pancreatitis - inflamación del páncreas. El virus puede aislarse en saliva desde una semana antes hasta 4-5 días después del inicio de la parotiditis. Se le aísla en cultivos de tejidos humanos o de primates y en huevos embrionados. Hoy en día se prefiere la técnica de ELISA. La VSG permanece normal excepto en caso de orquitis o pancreatitis. TRATAMIENTO: limitado generalmente a la administración de medicamentos para aliviar el dolor y la ingestión abundante de líquidos. Algunas veces el reposo en cama es necesario durante los primeros días. Los niños no deben asistir a la escuela hasta que los síntomas hayan desaparecido, es necesario recomendar líquidos orales, y descartar los ácidos, limón, naranja etc. La orquitis se tratara con medidas locales y reposo en cama. El tratamiento de la meningoencefalitis es puramente sintomático con analgésicos. PROFILAXIS: La globulina inmunitaria específica antiparotiditis no previene la parotiditis y ya no se fabrica. Se dispone de una vacuna elaborada con virus vivos atenuados de la parotiditis que se ha mostrado muy eficaz (95%) en la profilaxis de la enfermedad. Debe administrarse durante el segundo año de vida o bien antes de la pubertad. CASO CLINICO Paciente varón de 4 años y medio, remitido por su pediatra para estudio por haber presentado, en los últimos 9 meses, cuatro episodios de tumefacción parotídea derecha, de 3-4 días de evolución cada uno, que cedían con tratamiento antiinflamatorio y antibiótico. Se asociaba sensación dolorosa de la zona, febrícula y, en una ocasión, supuración mucopurulenta por el conducto de Stenon. No destacaban antecedentes personales de interés y el calendario vacunal (incluido parotiditis) estaba correcto. La exploración física en el momento de la consulta (intercrisis) era normal. PREGUNTA Cuales son los estudios mas adecuados para buscar un diagnostico especifico. RESPUESTA a.- BH, QS, EGO. b.- Radiografia de cráneo y cara. c.- Serología para VEB, citomegalovirus, adenovirus y VIH. d.- Factor reumatoide y anticuerpos antinucleares (ANA, anti-Ro y anti-La). PREGUNTA Cual de las siguientes complicaciones es la más frecuente. RESPUESTA a.- Meningoencefalitis. b.- Orquitis/orquiepididimitis. c.- Pancreatitis. d.- Miocarditis. HEPATITIS. CIENCIAS BASICAS: Indica proceso inflamatorio del hígado caracterizado por hepatomegalia, anorexia, molestáis abdominales fundamentalmente gástricas, función hepática anormal, heces despigmentadas y orina oscura. Estas alteraciones pueden ser ocasionadas por infecciones producidas por bacterias, virus o parásitos o bien por ingestión excesiva de alcohol o fármacos, trastornos de tipi autoinmunitario o toxico. La mayor parte de las hepatitis son de origen viral. La hepatitis crónica se define como una lesión inflamatoria del hígado que puede progresar a lesión mas grave y a cirrosis, las causas más frecuentes son: virales, autoinmunes, toxicas y criptogenicas. En general se habla de cronicidad con una evolución de más de 6 meses, con excepción del caso de las hepatitis autoinmunes, en las cuales un diagnóstico más temprano, permitirá detectar el padecimiento antes de que se desarrolle cirrosis. Se sospecha una hepatitis crónica ante una sospecha de recaída de hepatitis aguda, persistencia de hepatitis aguda por más de 3 meses, datos clínicos de enfermedad hepática crónica y en pacientes asintomáticos de alto riesgo, con historia de transfusiones múltiples,

CURSO ENARM CMN SIGLO XXI TEL: 36246001

Pharmed Solutions Institute

PÁGINA 420

MANUAL DE TRABAJO DEL CURSO ENARM CMN SIGLO XXI como son pacientes con leucemias en remisión, hemofilia, hemoglobinopatías, IRC en programa de diálisis, y neoplasias en los que existe el riesgo de hepatitis post- transfusional. SALUD PUBLICA: En México que 95% de las hepatitis en niños menores de 215 años de edad es producida por el virus de hepatitis A. Cada año se registran aproximadamente 1,4 millones de casos de hepatitis A en todo el mundo. La hepatitis A se asocia a falta de agua salubre y a un saneamiento deficiente. Las epidemias se pueden propagar de manera explosiva y causar pérdidas económicas considerables. Las mejoras del saneamiento y la vacuna contra la hepatitis A son las medidas más eficaces para combatir la enfermedad. HEPATITIS POR VIRUS A: La sección hepática más frecuente, pertenece al grupo de los picornavirus de los enterovirus, y su huésped natural es el ser humano, el genoma está constituido por RNA. La hepatitis A se presenta en niños entre los 3-12 años de edad, con promedio en la edad preescolar. La vía de transmisión más común es la del tubo digestivo a través de alimentos o bebidas que se contaminan con matera fecal que contienen el virus. La contaminación por vía aérea no se ha confirmado. La transmisión también se da en los drogadictos que comparten agujas y jeringas para uso intravenoso. El mayor riesgo de diseminación del virus es en heces, ocurre 15 días posteriores al inicio de la enfermedad. El estado de portador no existe. La inmunidad con la infección es permanente. No se ha informado hasta la fecha evolución a la cronicidad. Clínica; Los niños menores de 2 años de edad, cursan sin ictericia en 90% de los casos. Antes de la aparición de la ictérica se presentan los siguientes síntomas, febrícula, anorexia, vomito, debilidad, cefalea, dolor difuso en abdomen y esplenomegalia. En la fase ictérica cuya duración puede ser de 8 días a 4 semanas, se hace aparente coluria, hipocolia, decaimiento y anorexia más acentuadas, con presencia de tinte amarillento en conjuntivas y piel. Laboratorio: Aumento de la bilirrubinas a expensas de la bilirrubina directa, las transaminasas, se encuentran elevadas, la colinesterasa y la colestasa están disminuidas, las pruebas de floculación de proteínas como es la turbidez al timol se encuentra elevada, ocurre aumento de la gammaglobulina con disminución de albumina. Se encuentran alteradas la concentración de inmunoglobulinas los tiempos de protrombina y la concentración de glucosa. Las pruebas serológicas específicas como es la detección de anticuerpos contra el virus de la hepatitis A por radioinmunoanálisis, sugieren infección reciente o convaleciente. Tratamiento: Las medidas son de tipo sintomático, ya que no existe medicamentos específicos para el virus de hepatitis A. Dieta; implica restricción de proteínas, de grasas y aumento en los hidratos de carbono. Reposo; mínima actividad para mantener un flujo circulatorio esplacnico y hepático adecuado. Aislamiento y protección a contactos; precauciones con el manejo de excretas durante el periodo ictérico y 2 semanas después. Aplicación de gammaglobulina estándar al 16% a todo contacto a razón de 0.2 a 0.12 ml/kg por vía IM en dosis única como inmunización pasiva. Inmunización: Debe aplicarse la vacuna contra hepatitis A en presentación pediátrica, dos dosis con intervalos de 1 mes con refuerzo a los 12 meses, la vacuna disponible en México precede de la cepa HM75 inactivada. Son 360 unidades en 0.5ml y se aplica vía IM en niños mayores de 1 año de edad. VIRUS DE HEPATITIS D: Se trata de un virus RNA defectuoso, formado por RNA del agenteᵹ, cubierto por antígeno de superficie del virus de la hepatitis B, sin este virus no resulta infectante. Se transmite por vía parenteral o vía percutánea, en mucosas, transfusión sanguínea, contacto sexual y etapa perinatal. La diseminación es por portadores y afecta más al adolescente. Su periodo de incubación es de 15-200 días. El 75% del diagnóstico inicial se hace en la fase cirrótica y por lo tanto tiende a la fase de cronicidad. En laboratorio a parte de la BH, transaminasas, bilirrubinas y EGO, debe hacerse cuantificación de anticuerpos antivirus de la hepatitis ᵹ (anti-VHD) en donde se encuentran titulaciones mayores de 1:100, las titulaciones de IgM se encuentran altas en fase temprana, además se cuantifican anticuerpos del virus de la hepatitis B (Ags-VHB). Tratamiento: Se ha utilizado el interferón sin obtener resultados satisfactorios. No existe vacuna alguna, sin embargo algunos autores sugieren usar la vacuna contra el virus de la hepatitis B. CASO CLINICO RN pretérmino de 36,6 semanas de EG, peso 2.555 g, por depresión neonatal y riesgo infeccioso. APGAR 5/9. La madre presentó, 24 h antes del parto, un cuadro febril acompañado de dolor abdominal. Por la sospecha clínica de corioamnionitis recibió tratamiento antibiótico y se indujo el parto, que finalizó en cesárea con anestesia general por no progresión. PREGUNTA Cual es la conducta a seguir mas adecuada. RESPUESTA a.- Egresar con vigilancia estrecha. b.- Hemocultivo e inicio de antibiótico. c.- BH, QS, EGO. d.- Analisis de LCR. PREGUNTA En las siguientes 48 h se deteriora su estado general, con acidosis metabólica, anemia, plaquetopenia (recuento mínimo 10.000/mcl) y hemorragia digestiva baja, por lo cual se modifica el tratamiento antibiótico, para cubrir una posible enterocolitis necrozante, y se suspende la alimentación oral. En las 24 h posteriores desarrolla coagulopatía y fallo hepático; presenta petequias generalizadas y sangrado importante por puntos de canalización venosa umbilical y periférica. Cual es la conducta a seguir. RESPUESTA a.- Transfusión de hemoderivados. b.- Administración de vitamina K (1 mg/kg/día) durante 72 h. c.- Pruebas de función hepática. d.- Administracion de albumina. PREGUNTA Se obtienen los siguientes resultados donde alcanza concentraciones máximas de transaminasas GOT 957 U/L, GPT 237 U/L y GGT 144 U/L, FA 93 U/L, LDH 5031 U/L, bilirrubina directa: 1,66 mg/dl, con disminución de las proteínas totales (3,75 g/dl) y albúmina (2,19 g/dl).

CURSO ENARM CMN SIGLO XXI TEL: 36246001

Pharmed Solutions Institute

PÁGINA 421

MANUAL DE TRABAJO DEL CURSO ENARM CMN SIGLO XXI Por la sospecha clínica de viremia, y ante la posibilidad de infección herpética, se inicia tratamiento con aciclovir. Cual de los siguientes patologias es el mas frecuente evaluar para el diagnostico diferencial. RESPUESTA a.- Hepatitis neonatal. b.- Infeccion por VEB, CMV. c.- Enterovirus y adenovirus. d.- Toxoplasma, rubéola y sífilis. INFLUENZA. CIENCIAS BASICAS: Es una infección viral aguda de las vías respiratorias, altamente contagiosa, que puede afectar la mucosa nasal, la faríngea, bronquios y en ocasiones hasta los alvéolos pulmonares. Los síntomas son parecidos a los del catarro común o resfriado; sin embargo, son más graves y su inicio es generalmente abrupto. La gripe es causada por el virus de la influenza (virus de influenza A, B y C), el catarro común es causado por múltiples virus respiratorios (rinovirus, coronavirus, adenovirus, virus parainfluenza). El cuadro clínico de la influenza puede variar, observándose desde un cuadro febril agudo leve hasta una infección pulmonar y/ocomplicaciones graves. SALUD PUBLICA: Puede tener un comportamiento endémico, epidémico o de una pandemia. Las epidemias de influenza son responsables de 36 000 a 50 000 muertes por año en países como EUA en los últimos años, afecta a todos los grupos etarios, pero principalmente a niños menores de dos años y adultos mayores de 65 años. PATOGENIA: Los virus asociados a la influenza humana son miembros de la familia Orthomyxoviridae, tienen una morfología helicoidal y su genoma está constituido por una cadena segmentada de ARN, los virus influenza A y B causan infecciones respiratorias en humanos y animales, el virus A es causa de pandemias. El virus de influenza C afecta en forma moderada a los niños y en forma ocasional a los adultos. Estos virus se distinguen por variaciones antigénicas en dos proteínas estructurales (la nucleoproteína y la proteína de la matriz). Los virus de la influenza A se clasifican en subtipos de acuerdo a dos antígenos de superficie: hemaglutinina (H) y neuraminidasa (N). La hemaglutinina es considerada el antígeno mayor para la cual está dirigida la producción de anticuerpos neutralizantes y cuya función es la adhesión del virus mediante residuos de ácido siálico en la superficie del epitelio respiratorio humano. La expresión de la neuraminidasa es menos abundante en la superficie viral y su papel es facilitar la liberación de viriones de células infectadas del hospedero. Para la clasificación de este virus se utiliza el siguiente orden en la nomenclatura: el tipo, el lugar en donde fue aislado, el año de aislamiento, el número de identificación del laboratorio y, en los aislamientos del virus de la influenza A, el subtipo H-N (por ejemplo: A/Moscow/10/99 [H3N2]). Estudios de epidemiología molecular de los virus de la influenza han demostrado que los subtipos H1N1, H1N2, y H3N2 son los que han circulado en los últimos años. Las aves constituyen un reservorio potencial de intercambio genético para los virus de la influenza, lo que representa un riesgo latente de pandemias. Los virus de la influenza A también infectan a pollos, cerdos, caballos y ocasionalmente a mamíferos marinos. El periodo de incubación es de 1 a 4 días (promedio de dos días). Los adultos pueden ser infectantes un día antes de que los síntomas inicien hasta aproximadamente 3-5 días después. Los niños pueden ser infectantes por 10 días o más. Las personas con estados de inmunosupresión pueden excretar los virus por períodos más prolongados. El virus de la influenza A se caracteriza por causar enfermedad moderada a grave. El virus B de la influenza causa cuadros clínicos menos graves que el tipo A y tradicionalmente produce infección y enfermedad durante la infancia. La capacidad del virus de la influenza A y B de sufrir cambios antigénicos graduales en sus dos antígenos de superficie, la hemaglutinina y la neuraminidasa, complica la vacunación contra esta enfermedad. El tipo de variaciones antigénicas menores o deslizamientos antigénicos (antigenic drift) resulta de la acumulación de mutaciones puntuales de los genes que transcriben para estas proteínas. DIAGNOSTICO: La influenza se caracteriza por el inicio agudo de síntomas y signos respiratorios que incluyen: fiebre (elevada de inicio abrupto), tos seca, coriza, cefalea, odinofagia, ardor faríngeo, mialgias, artralgias, y ataque importante al estado general (postración H3N2). Estos síntomas ocurren en 50 a 70% de las infecciones por influenza, tanto estacional como por el virus de influenza humana H1N1 2009. Menos comunes son: fotofobia, dolor abdominal, náusea, vómito y diarrea. La duración de la enfermedad sin complicaciones habitualmente es de una semana, aunque la tos y debilidad pueden persistir por más de 14 días. En particular, en los pacientes con infección por el virus A H1N1, se presentaron durante la epidemia cuadros graves de neumonía intersticial y progresión a pulmón de choque. Para la infección por el virus influenza A humana H1N1 2009, se establecieron definiciones epidemiológicas para la confirmación de los casos: los confirmados son aquellos que tienen síntomas y signos arriba mencionados y una prueba confirmatoria positiva que consiste en reacción de polimerasa en cadena en transcripción reversa (RT-PCR) en tiempo real o cultivo viral. Un caso probable es una persona sintomática que es positiva para influenza A por prueba rápida, pero negativa para H1 y H3 por RT-PCR. Una persona sospechosa es aquella que no cumple con la definición de probable o confirmado, no tiene prueba negativa para el virus nuevo, y: Es un individuo previamente sano, menor de 65 años de edad, hospitalizado por infección respiratoria baja. Vive en un lugar o estado donde no hay casos confirmados pero viajó recientemente a donde existe uno o más casos probables o confirmados. Tuvo contacto en los siete días previos con un caso confirmado o probable. Laboratorio: Idealmente, la muestra de exudado faríngeo, nasofaríngeo, lavado nasal, aspirado bronquial o traqueal debe tomarse en los primeros tres días del inicio de los síntomas. El cultivo viral es el estándar de oro, además permite que el virus sea tipificado y caracterizado antigénicamente. Los medios de cultivo utilizados son huevos embrionados de gallina o el cultivo de riñón canino de Madin-Darby, el riñón de chimpancé y otros. También se puede realizar el diagnóstico por determinaciones serológicas al mostrar un incremento de cuatro veces en la titulación de anticuerpos contra influenza en una segunda muestra. El suero en la fase de convalecencia debe obtenerse entre los días 10 a 21 del inicio del cuadro. Las técnicas serológicas más frecuentemente empleadas son las de neutralización y la inhibición por hemaglutinación. COMPLICACIONES: Neumonía primaria por influenza o neumonía bacteriana secundaria. La etiología más frecuente de la neumonía bacteriana es: Streptococcus pneumoniae en 48% de los casos, Staphylococcus aureus en 19%, y Haemophilus influenzae no tipificable en 11%. Otras complicaciones del tracto respiratorio incluyen: sinusitis bacteriana, bronquitis, traqueobronquitis y otitis media. En niños se ha asociado a otras manifestaciones clínicas como son: convulsiones febriles, encefalopatía por el virus o encefalopatía asociada a la utilización de salicilatos (síndrome de Reye), miositis, rabdomiólisis, miocarditis y pericarditis. En pocos casos se informa de complicaciones en sistema nervioso central como mielitis transversa y polirradiculoneuritis. TRATAMIENTO: Permanecer en casa, en reposo para mejorar los síntomas y evitar contagio a otras personas. En casos leves a mderados; ofrecer líquidos abundantes, aumentar el consumo de frutas con mayor contenido de vitamina A y C. No se recomienda suspender la lactancia materna si la madre enferma. Deben evitarse los lugares concurridos, así como cambios

CURSO ENARM CMN SIGLO XXI TEL: 36246001

Pharmed Solutions Institute

PÁGINA 422

MANUAL DE TRABAJO DEL CURSO ENARM CMN SIGLO XXI bruscos de temperatura, tabaquismo y exposición a contaminantes dentro de la casa. El control de la fiebre por medios físicos, evitando en los niños el uso de salicilatos (por la asociación con el síndrome de Reye). Cuando amerite, se podrán administrar paracetamol o AINES con efecto antipirético (ibuprofeno). No se recomienda utilizar antibióticos profilácticos. Es importante hacer hincapié en los datos de alarma que sugieren el agravamiento y necesidad de manejo hospitalario. En la edad pediátrica los síntomas incluyen: fiebre persistente, tos productiva con expectoración, dificultad para respirar, rechazo al alimento, hipotonía o convulsiones. Medicamentos con utilidad clínica al reducir la duración de los síntomas cuando son empleados en las primeras 48 horas del inicio de la enfermedad. También pueden ser efectivos en la quimioprofilaxis. La amantadina y la rimantadina inhiben la replicación de los virus de influenza A, pero no los de influenza B, se administran por vía oral y se utilizan para el tratamiento o quimioprofilaxis, disminuyen la excreción viral y reducen en promedio un día la duración de la enfermedad. El tratamiento se recomienda por cinco días y cuando se utilizan para quimioprofilaxis, son efectivos en 70-90%. Se recomienda el uso profiláctico de la amantadina y rimantadina para contactos o convivientes. Los inhibidores de la neuraminidasa (zanamivir, oseltamivir) tienen actividad contra influenza A y B, bloqueando el sitio activo de la neuraminidasa, disminuyen la excreción del virus y reducen la duración de los síntomas de la influenza por 36 horas. En ensayos clínicos se ha encontrado que el oseltamivir disminuye las complicaciones de las vías respiratorias bajas (neumonía y bronquitis), disminuye uso de antibióticos y el riesgo de hospitalización. PREVENCIÓN: La vacunación anual de personas en grupos de alto riesgo de desarrollar complicaciones y sus contactos, representa la principal estrategia. Las vacunas de mayor uso son producidas de virus crecidos en huevos embrionados inactivados con formaldehido o propiolactona, pueden producirse de virus completos tratados con detergentes o de antígenos de superficie (hemaglutinina y neuraminidasa) purificados. Son más recomendadas las que contienen antígenos purificados, ya que dan menos reacciones alérgicas. Habitualmente contienen los tres tipos virales recomendados por la OMS. Diversos estudios han demostrado una eficacia de 80% en niños y de 77% en adultos. Las vacunas contra la influenza son recomendadas en Mexico para: a) Vacunación a personas que pueden transmitir influenza a personas en grupos que están propensos a desarrollar complicaciones: 1) Vacunación a personal de salud (personal médico, enfermería, paramédico). 2) Personal que labora en asilos de ancianos y casas de estancia. 3) Personas que viven en contacto intradomiciliario con personas que tienen alto riesgo de desarrollar complicaciones (niños que viven con un adulto con asma). b) Vacunación de mujeres con embarazo de alto riesgo: específicamente aquellas en el segundo y tercer trimestre del embarazo por tener el mayor riesgo de complicaciones. c) Vacunación de personas mayores de 65 años de edad. Este grupo presenta el mayor riesgo de hospitalización y muerte asociado a complicaciones por influenza. d) Vacunación de personas de entre 50 a 64 años. En este grupo se concentra la mayor pre-valencia de personas con condiciones de alto riesgo (enfermedades crónico-degenerativas). e) Vacunación de pacientes de cualquier edad con padecimientos crónicos. Estos padecimientos incluyen: enfermedades pulmonares (asma o enfermedad pulmonar obstructiva crónica) o enfermedades cardiovasculares (insuficiencia cardiaca). Así como aquellos con enfermedades metabólicas crónicas (diabetes mellitus, insuficiencia renal) y en pacientes inmunodeprimidos (VIH positivos o trasplantados). f) Vacunación de niños de 6 a 36 meses de edad. Se ha demostrado que en esta población existe un riesgo aumentado de hospitalización por complicaciones asociadas a la influenza. Aunque el riesgo en niños menores de seis meses es aún mayor, la vacuna trivalente no está aprobada para su uso en este grupo. g) Viajeros. Se recomienda en aquellos viajeros que no fueron vacunados en el año precedente si planean viajar a los trópicos, o al hemisferio sur durante los meses de abril a septiembre. Medidas generales: Las medidas no farmacológicas que se recomiendan para ayudar a prevenir la propagación de enfermedades respiratorias como la influenza son: Cubrirse la nariz y la boca con un pañuelo desechable al toser o estornudar. Tirar el pañuelo desechable a la basura después de usarlo. Lavarse las manos frecuentemente con agua y jabón, especialmente después de toser o estornudar. Puede utilizarse alcohol-gel. Evitar tocarse los ojos, la nariz o la boca. Evitar el contacto cercano con personas enfermas. Al enfermarse, debe permanecer en casa por siete días a partir del comienzo de los síntomas o hasta que hayan pasado 24 horas desde que desaparecieron. Esto disminuye la posibilidad de infectar a otras personas y propagar más el virus. Se deben seguir las recomendaciones de la Secretaría de Salud con relación al cierre de escuelas, evitar frecuentar sitios con multitudes y tomar medidas de distanciamiento social. CASO CLINICO Paciente de 17 años con fiebre, tos y dolor de garganta, cefalea, rinorrea y dolor abdominal, por lo que iniciaron tratamiento sintomático para rinofaringitis con respuesta no favorable por lo que regresa a los 3 dias con empeoramiento de los síntomas. PREGUNTA Cuales son los síntomas o signos cardinales para considerar caso sospechoso de influenza. RESPUESTA a.- Fiebre con tos o dolor de garganta. b.- Cefalea, rinorrea y coriza. c.- Artralgias, mialgias y postración. d.- Dolor torácico, dolor abdominal y congestión nasal. PREGUNTA Cuales son las carateristicas que presentan mayor complicaciones. RESPUESTA a.- Edad >60 años y 80 %, Exacerbaciones NO Uno o más/ año Una vez/sem variabilidad de FEV1 20-30%. 3. ASMA MODERADA PERSISTENTE: Síntomas Diarios. Exacerbaciones que afectan la actividad o sueño. Síntomas nocturnos >1 vez a la semana. Uso diario de agonistas B2 Inhalados de acción rapida. FEV1 o PEF de 60-80%, variabilidad del FEV1 >30%. 4. ASMA SEVERA PERSISTENTE: Síntomas Diarios. Exacerbaciones frecuentes. Síntomas nocturnos frecuentes. Limitación de actividad física. FEV1 30%. CLASIFICACION DE LAS CRISIS ASMATICAS: Leve: limita actividades fuertes (ejercicio), habla sin problemas, no lo despierta. Flujometria: PEF > 80%, FC 120 lpm, sibilancias: intensas. Paro inminente: conciencia: confundido, tiros: respiración paradojia, bradicardia, sibilancias: silencio. TRATAMIENTO: Medidas Generales: Control ambiental, evitar exposición a desencadenantes, vacuna anti influenza, evitar AINEs, reducir o eliminar la inflamación. Las metas en el tratamiento del asma son: a.Control adecuado de los síntomas. b.- Prevenir las exacerbaciones. c.- Mantener FR dentro de la normalidad. d.- Mantener actividad normal. e.- Evitar efectos adversos de medicamentos. f.- Prevenir el asma fatal. Los medicamentos utilizados son: ESTEROIDES INHALADOS: Mejoran la función pulmonar, disminuyen la hiperreactividad bronquial, los síntomas y las exacerbaciones, mejoran la calidad de vida. Las dosis recomendadas son: Beclometasona: niños 100-800 ug/día (leve-moderada:200, grave: > 500 dosis máxima: 1000). Budesonida: niños 100-400 ug/día (leve moderada:200, grave: > 400, dosis máximas: 800). Fluticasona: niños: 100-200 ug/día (leve-moderada: 100, grave: > 200, dosis máximas: 500). Ciclesonida : Dosis 100 a 1200 mcg; Vía de administración : Inhalado. Triamcinolona: Dosis 400 a 2000 mcg/día vía de administración: Inhalado Reacciones secundarias: Candidiasis oral, disfonía, tos

CURSO ENARM CMN SIGLO XXI TEL: 36246001

Pharmed Solutions Institute

PÁGINA 442

MANUAL DE TRABAJO DEL CURSO ENARM CMN SIGLO XXI (irritación de la vía aérea), prevención: uso de espaciadores. ESTEROIDES SISTEMICOS: Esta terapia se utiliza para el control de los pacientes con asma severa persistente. Han demostrado disminución de signos patológicos de inflamación y mejoría de la hiperreactividad bronquial. Prednisona: 0.5-1 mg/kg/día dosis de reducción. Deflazacort : 5-10 mg/kg/día dosis de reducción. Metilprednisolona: Dosis 4-6 mg/kg cada 6-8 hrs. Hidrocortisona: Dosis 4-6 mg/ cada 6-8 hrs. Efectos secundarios: Osteoporosis, DM, glaucoma, obesidad, estrías cutáneas, supresión del eje hipotalámico, metilxantinas: broncodilatador cuyo mecanismo de acción está relacionado con la inhibición de la fosfodiesterasa. Infusión: 5 mg/kg/dosis IV 0.7-0.9 mg/kg/hr. Reacciones secundarias: Nauseas y vómito, cefalea, taquicardia, arritmias. B2 AGONISTAS: Relajan la musculatura lisa bronquial, mejoran el aclaramiento mucociliar, disminuyen la permeabilidad vascular y la liberación de mediadores inflamatorios. Los más utilizados son: Salbutamol: 200mcg PRN. Fenoterol: 50mcg PRN. Salmeterol: 25mcg cada 12 hrs. Terbutalina: 500mcg PRN. B2 Agonistas y Esteroides: Disminución de los síntomas y de crisis de asma nocturna, mejora la función pulmonar, reduce el número de exacerbaciones. MODIFICADOR DE LEUCOTRIENOS: Cistenil leucotrieno, Montelukast, Pranlukast, Zafirlukast. ANTICOLINÉRGICOS: Bromuro de Ipratropio y Bromuro de oxitropio, mecanismo de acción: Broncodilatador. Medicamentos controladores: Esteroides inhalados, agonistas de larga acción, Anti leucotrienos, Xantinas. Medicamentos de rescate: agonistas de corta acción, anticolinérgicos. Crisis asmáticas: Salbutamol + bromuro de ipratropio, 2-4 disparos c/20min x hora. LEVE: 2-4 disparos cada 3-4 hrs, MODERADA: 6-10 disparos por 1 ó 2 hrs, SEVERA: 10 disparos o más. ESTADO ASMATICO: Forma letal del asma, caracterizado por ataques cada vez peor que no mejora con tratamiento apropiado, que culmina con la insuficiencia de la válvula pulmonar: empleo de músculos accesorios de la respiración, pulso paradójico, cianosis. Criterios hospitalizar. Terapia intensiva: Ataques frecuentes y repetidos, última vez asma grave que culmino en hospitalización. Consumo diario y excesivo de broncodilatadores y corticoesteroides. Empleo de músculos accesorios de la respiración. Pulso paradójico que rebasa los 18 mmHg en adolescentes y 10 mmHg en niños. Cambios en la conciencia, cianosis. Neumotórax, neumomediastino. FEV1 o PEFR (velocidad de flujo espiratorio máximo) menor de 20% de la cifra calculada. PaO2 < 60mmHg. PaCO2 > 40mmHg en presencia de disnea y sibilancias. Acidosis metabólica. Anormalidades electroencefalográficas. CASO CLINICO Niña de 7 años que acude a urgencias por cuadro de tos, dificultad respiratoria de dos semanas de evolución y pico febril 38°C. con antecedentes patológicos de obesidad, roncadora habitual y respiración bucal. adenoidectomizada hace 2 años por sospecha clínica de SAHS sin control posterior. Peso 56kg (p>97), FC 135 lpm (p>95), TA 109/70, SatO2 82% con FiO2 21%. Facies anímica, implantación baja del pelo, estrabismo y obesidad mórbida. Aceptable estado general, normohidratada, palidez cutánea y retracciones subcostales moderadas. Auscultación cardiaca normal. Auscultación respiratoria con hipoventilación generalizada moderada, sibilantes espiratorios finos y roncus dispersos. PREGUNTA Cuál es la conducta farmacológica mas adecuada. RESPUESTA a.- Salbutamol, bromuro de ipatropio inhalados y corticoides endovenosos. b.- Salbutamol, corticoide, loratadina. c.- Salbutamol, oxigeno, ambroxol y prednisona. d.- Ambroxol, prednisona, Bromuro de ipatropio y oxigeno. PREGUNTA El paciente empeoro su cuadro clínico, con mayor dificultad respiratoria. Se decide enviar a segundo nivel. Al ingreso se realizó BH con discreta leucocitosis, bioquímica normal y gasometría capilar: pH 7,31, PC02 52,3mmHg, PO254,7mmHg, HC0326mmol/l. Precisa FiO2 inicial de 1 para mantener SatO2>92%. Presenta evolución lenta y respuesta parcial al tratamiento. Tras los primeros días de ingreso persisten necesidades de oxigenoterapia elevadas (Fi 2≈0,5) durante el sueño. En controles de gasometría destaca hipercapnia nocturna (PCO2 56,3mmHg). La evolución tórpida obliga a nuevo planteamiento diagnóstico y realización de pruebas complementarias. PREGUNTA Cuales fueron los criterios más importantes para el envio a segundo nivel. RESPUESTA a.- Duda diagnostica. b.- Asma complicada. c.- Asma no controlada con tratamiento. d.- Antecedetes de exacerbaciones. CASO CLINICO Se trata de un adolescente de 13 años y 90kg de peso con antecedentes de asma sin tratamiento de fondo. Acude por dificultad respiratoria de varias horas de evolución sin que hubiera recibido broncodilatadores previamente. Se constata una crisis asmática grave con taquipnea y retracción costal marcadas, y se inicia tratamiento con nebulizaciones de salbutamol (15mg en la primera hora) y bromuro de ipratropio (500 mg/h) asociados a metilprednisolona por vía intravenosa (60mg). Transcurrida 1 h de tratamiento, se realiza gasometría venosa que muestra acidosis metabólica con hiperlactacidemia. PREGUNTA Cuál es la conducta a seguir mas adecuada? RESPUESTA

CURSO ENARM CMN SIGLO XXI TEL: 36246001

Pharmed Solutions Institute

PÁGINA 443

MANUAL DE TRABAJO DEL CURSO ENARM CMN SIGLO XXI a.- Mantener el tratamiento. b.- Enviar a segundo nivel. c.- Ingreso a la UCI. d.- Realizar rx de torax. PREGUNTA Al ingreso hospitalario se mantiene tratamiento broncodilatador (salbutamol y bromuro de ipratropio) horario. Tras 4h de ingreso, se mantiene el trabajo respiratorio, la taquipnea y la taquicardia, con saturación de oxígeno del 92% y con fracción inspiratoria de oxígeno (FiO 2 ) del 35%. La radiografía de tórax presenta hiperinsuflación sin condensación neumónica, atelectasia o escape aéreo. La gasometría muestra empeoramiento de la acidosis láctica. Se ingresa al paciente en cuidados intensivos para el inicio de la ventilación no invasiva tipo presión positiva con dos niveles de presión con los parámetros iniciales (presión positiva inspiratoria de la vía aérea de 12; presión positiva espiratoria de la vía aérea de 6; FiO 2 del 40%), se mantiene el tratamiento broncodilatador y se expande la volemia. Cuál es pronostico esperado. RESPUESTA a.- Mejoría ventilatoria. b.- Descenso significativo de la FR. c.- Normotenso, caliente y bien perfundido. d.- Incremento de acido láctico. FIBROSIS QUISTICA (FQ). CIENCIAS BASICAS: Es la enfermedad genética más letal, de carácter recesivo, multisistémico y progresivo, afecta de preferencia a poblaciones caucásicas. La enfermedad se caracteriza por el espesamiento del mucus producido por las glándulas exocrinas induciendo compromiso sino-pulmonar con daño pulmonar progresivo, insuficiencia pancreática y por lo tanto síndrome de mala-absorción, con consecuente desnutrición, esterilidad masculina por atrofia de los conductos deferentes y elevación de electrolitos en el sudor. El íleo meconial siempre debe obligar a descartar FQ. Dependiendo de las mutaciones involucradas existe una gran diversidad de formas clínicas. SALUD PUBLICA: Su incidencia varía de 1 por cada 3,000 a 1 por cada 8,000 nacidos vivos. PATOGENIA: El gen de la FQ se encuentra localizado en el brazo largo del cromosoma 7, codifica una proteína de 1480 aminoácidos que se ha llamada Proteína Transportadora de Transmembrana (CFTR), esta se localiza en el polo apical de las células epiteliales. Es una glicoproteína cuya función es actuar como canal de cloro, está constituida por dos regiones transmembránicas (hidrofóbicas) separadas por una región de unión al ATP. La primera mutación encontrada fue la ∆F508, localizada en el dominio N D1, presente en alrededor del 75% de la población caucásica. El resultado de todas las mutaciones detectadas que alteran la función de la CFTR es el mismo: la imposibilidad de transportar cloruro. Cualquiera que sea la mutación en el gen CFTR, cada paciente muestra las siguientes anormalidades en distintos grados: A) Concentración anormal de los iones en las secreciones de las glándulas serosas, manifestada por aumento en la concentración de cloro y sodio en el sudor. B) Incremento en la viscosidad de las secreciones de las glándulas secretoras de moco, asociado con obstrucción y perdida secundaria de la función glandular. C) Aumento en la susceptibilidad a la colonización endobronquial crónica por grupos específicos de bacterias (Staphylococcus aureus, H. influenzae, Pseudomona aeruginosa, Burkholderia cepacia). Se han descrito 6 clases de mutaciones: las I a III son las más comunes y generalemnrte se relacionan con insuficiencia pancreática. En Mexico se han identificado 46 diferentes mutaciones que afectan a 77% de los cromosomas de la fibrosis quística. DIAGNOSTICO: La mayoría de los casos de FQ se manifiesta por la triada clásica: a) enfermedad pulmonar obstructiva progresiva crónica con infección agregada, b) insuficiencia pancreática exocrina, c) elevación en las concentraciones de Cl y Na en el sudor. Los RN afectados rara vez muestran síntomas respiratorios, aunque los menores de 6 meses de edad pueden experimentar taquipnea, sibilancias, incremento del trabajo respiratorio, sobresdistension del tórax y atelectasias. En 10-20% de los pacientes, el íleo meconial puede ser la primera manifestación de la enfermedad. Se produce por la implantación de meconio deshidratado en el íleo terminal, con un cuadro de obstrucción intestinal. Puede sospecharse antes del parto por ecografía u ocurrir al nacimiento con distensión abdominal progresiva y vómitos biliosos y falta o retardo en la eliminación de meconio en las primeras 24-48hrs de vida. La radiografía de abdomen suele revelar asas intestinales dilatadas con áreas de aire mezclado con meconio deshidratado. En la mayoría de los casos se encuentran una o más expresiones clínicas de la enfermedad, que se confirma con la prueba de sudor. Casi todos los pacientes exhiben enfermedad sinusopulmonar crónica y el 85-90% tiene insuficiencia pancreática exocrina. Es necesario hace notar que debe considerarse como FQ clásica, aquella con niveles de test del sudor altos, compromiso pulmonar severo, progresivo, de instalación temprana, rinosinusitis, síndrome de malaabsorción e infertilidad masculina. Se ha denominado como FQ no clásica o enfermedad vinculada a FQ aquella con al menos una mutación del gen de FQ, lo que le confiere función parcial a la proteína CFTR, generalmente no tienen síndrome de maladigestión por estar preservada la función exocrina y por lo tanto mejor estado nutricional, con niveles de test del sudor discretamente altos o normales, enfermedad pulmonar de instalación tardía y de menor severidad, rinosinusitis y azoospermia, con clara mejor sobrevida. DIAGNOSTICO PREIMPLANTACIONAL: determinar características genéticas del embrión a partir de una sola célula obtenida mediante biopsia embrionaria, sin prejuicio para la viabilidad del mismo. Por consiguiente, se requiere la obtención de embriones mediante fecundación in vitro. DIAGNOSTICO PRENATAL: Se analiza el ADN de células de vellosidades corionicas o líquido amniótico. Se realiza si los padres son portadores o si existe un hermano con fibrosis quística. ESTUDIO NEONATAL: Se basa en el hecho de que las concentraciones séricas de tripsina de los enfermos con insuficiencia pancreática pueden ser incluso 8 veces mayores a lo normal, se analizan: tripsina, tripsinogeno o complejo tripsina 1 anti-tripsina. El primer estudio se realiza entre el 1-5 días de vida; si es positivo se repite entre la segunda y octava semanas si las concentraciones se mantienen elevadas, se hace la prueba de Gibson y Cooke y estudio genético. Test del sudor: la iontoforesis de pilocarpina por el método de Gibson y Cooke, continua siendo el gold standard que permite medir los valores de sodio y cloro en el sudor; en el túbulo de la glándula sudorípara está bloqueado el reingreso de cloro a la célula, por lo cual tampoco lo hace el sodio, teniéndose un sudor con mayor cantidad de estos electrolitos. Se considera los siguientes valores: Positivo: >60 meq/lt, Limítrofe: 40 a 59 meq/lt, Negativo:
Manual de Trabajo del Curso ENARM CMN Siglo XXI_booksmedicos.org

Related documents

561 Pages • 459,037 Words • PDF • 24.3 MB

36 Pages • 14,264 Words • PDF • 565.5 KB

5 Pages • 1,561 Words • PDF • 79.1 KB

4 Pages • 856 Words • PDF • 90.3 KB

440 Pages • 116,425 Words • PDF • 7.7 MB

0 Pages • PDF • 4.4 MB

16 Pages • 2,872 Words • PDF • 640.1 KB

2 Pages • 596 Words • PDF • 287.4 KB

297 Pages • 99,870 Words • PDF • 2 MB

62 Pages • 12,998 Words • PDF • 1.8 MB